AP® Biology Instructor Solution Manual Part 1

You might also like

Download as docx, pdf, or txt
Download as docx, pdf, or txt
You are on page 1of 893

Table of Contents – Part 1

Table of Contents – Part 1


1 | The Study of Life................................................................................................................1

Review Questions.....................................................................................................................................................1

Critical Thinking Questions.......................................................................................................................................4

Test Prep for AP® Courses........................................................................................................................................6

2 | The Chemical Foundation of Life........................................................................................9

Review Questions.....................................................................................................................................................9

Critical Thinking Questions.....................................................................................................................................14

Test Prep for AP® Courses......................................................................................................................................18

Science Practice Challenge Questions....................................................................................................................20

3 | Biological Macromolecules...............................................................................................25

Review Questions...................................................................................................................................................25

Critical Thinking Questions.....................................................................................................................................34

Test Prep for AP® Courses......................................................................................................................................41

Science Practice Challenge Questions....................................................................................................................48

4 | Cell Structure...................................................................................................................58

Review Questions...................................................................................................................................................58

Critical Thinking Questions.....................................................................................................................................65

Test Prep for AP® Courses......................................................................................................................................71

Science Practice Challenge Questions....................................................................................................................76

5 | Structure and Function of Plasma Membranes.................................................................85

Review Questions...................................................................................................................................................86

Critical Thinking Questions.....................................................................................................................................89

Test Prep for AP® Courses......................................................................................................................................95

Science Practice Challenge Questions..................................................................................................................101

Advanced Placement Biology Instructor’s Solution Manual


Copyright 2018, Rice University. All Rights Reserved. This ancillary resource is intended for instructor use only and
may not be uploaded, redistributed, or reproduced without written approval by OpenStax.
Table of Contents – Part 1

6 | Metabolism....................................................................................................................105

Review Questions.................................................................................................................................................105

Critical Thinking Questions...................................................................................................................................114

Test Prep for AP® Courses....................................................................................................................................121

Science Practice Challenge Questions..................................................................................................................125

7 | Cellular Respiration........................................................................................................136

Review Questions.................................................................................................................................................136

Critical Thinking Questions...................................................................................................................................143

Test Prep for AP® Courses....................................................................................................................................149

Science Practice Challenge Questions..................................................................................................................155

8 | Photosynthesis...............................................................................................................163

Review Questions.................................................................................................................................................163

Critical Thinking Questions...................................................................................................................................169

Test Prep for AP® Courses....................................................................................................................................173

Science Practice Challenge Questions..................................................................................................................179

9 | Cell Communication.......................................................................................................186

Review Questions.................................................................................................................................................186

Critical Thinking Questions...................................................................................................................................191

Test Prep for AP® Courses....................................................................................................................................197

Science Practice Challenge Questions..................................................................................................................201

10 | Cell Reproduction.........................................................................................................208

Review Questions.................................................................................................................................................208

Critical Thinking Questions...................................................................................................................................213

Advanced Placement Biology Instructor’s Solution Manual


Table of Contents – Part 1

Test Prep for AP® Courses....................................................................................................................................218

Science Practice Challenge Questions..................................................................................................................225

11 | Meiosis and Sexual Reproduction.................................................................................229

Review Questions.................................................................................................................................................229

Critical Thinking Questions...................................................................................................................................232

Test Prep for AP® Courses....................................................................................................................................237

Science Practice Challenge Questions........................................................................................................................

12 | Mendel's Experiments and Heredity.............................................................................240

Review Questions.................................................................................................................................................240

Critical Thinking Questions...................................................................................................................................246

Test Prep for AP® Courses....................................................................................................................................249

Science Practice Challenge Questions..................................................................................................................254

13| Modern Understandings of Inheritance.........................................................................265

Review Questions.................................................................................................................................................265

Critical Thinking Questions...................................................................................................................................268

Test Prep for AP® Courses....................................................................................................................................270

Science Practice Challenge Questions..................................................................................................................275

14 | DNA Structure and Function.........................................................................................280

Review Questions.................................................................................................................................................280

Critical Thinking Questions...................................................................................................................................291

Test Prep for AP® Courses....................................................................................................................................303

Science Practice Challenge Questions..................................................................................................................312

15 | Genes and Proteins......................................................................................................320

Review Questions.................................................................................................................................................320

Critical Thinking Questions...................................................................................................................................326

Advanced Placement Biology Instructor’s Solution Manual


Copyright 2018, Rice University. All Rights Reserved. This ancillary resource is intended for instructor use only and
may not be uploaded, redistributed, or reproduced without written approval by OpenStax.
Table of Contents – Part 1

Advanced Placement Biology Instructor’s Solution Manual


Table of Contents – Part 1

Test Prep for AP® Courses....................................................................................................................................332

Science Practice Challenge Questions..................................................................................................................336

16 | Gene Regulation...........................................................................................................346

Review Questions.................................................................................................................................................346

Critical Thinking Questions...................................................................................................................................352

Test Prep for AP® Courses....................................................................................................................................359

Science Practice Challenge Questions..................................................................................................................367

17 | Biotechnology and Genomics.......................................................................................369

Review Questions.................................................................................................................................................369

Critical Thinking Questions...................................................................................................................................376

Test Prep for AP® Courses....................................................................................................................................381

Science Practice Challenge Questions..................................................................................................................384

18 | Evolution and Origin of Species....................................................................................389

Review Questions.................................................................................................................................................389

Critical Thinking Questions...................................................................................................................................395

Test Prep for AP® Courses....................................................................................................................................404

Science Practice Challenge Questions..................................................................................................................423

19 | The Evolution of Populations........................................................................................434

Review Questions.................................................................................................................................................434

Critical Thinking Questions...................................................................................................................................438

Test Prep for AP® Courses....................................................................................................................................444

Science Practice Challenge Questions..................................................................................................................452

20 | Evolutionary Relationships of Life on Earth...................................................................459

Review Questions.................................................................................................................................................459

Critical Thinking Questions...................................................................................................................................466

Advanced Placement Biology Instructor’s Solution Manual


Copyright 2018, Rice University. All Rights Reserved. This ancillary resource is intended for instructor use only and
may not be uploaded, redistributed, or reproduced without written approval by OpenStax.
Table of Contents – Part 1

Advanced Placement Biology Instructor’s Solution Manual


Table of Contents – Part 1

Test Prep for AP® Courses....................................................................................................................................469

Science Practice Challenge Questions..................................................................................................................479

21 | Viruses.........................................................................................................................486

Review Questions.................................................................................................................................................486

Critical Thinking Questions...................................................................................................................................493

Test Prep for AP® Courses....................................................................................................................................498

Science Practice Challenge Questions..................................................................................................................504

22 | Prokaryotes: Bacteria and Archaea...............................................................................510

Review Questions.................................................................................................................................................510

Critical Thinking Questions...................................................................................................................................520

Test Prep for AP® Courses....................................................................................................................................531

Science Practice Challenge Questions..................................................................................................................542

23 | Plant Form and Physiology...........................................................................................553

Review Questions.................................................................................................................................................553

Critical Thinking Questions...................................................................................................................................570

Test Prep for AP® Courses....................................................................................................................................582

Science Practice Challenge Questions..................................................................................................................590

24 | The Animal Body: Basic Form and Function..................................................................602

Review Questions.................................................................................................................................................602

Critical Thinking Questions...................................................................................................................................607

Test Prep for AP® Courses....................................................................................................................................612

25 | Animal Nutrition and the Digestive System..................................................................623

Review Questions.................................................................................................................................................623

Critical Thinking Questions...................................................................................................................................631

Test Prep for AP® Courses....................................................................................................................................639

Advanced Placement Biology Instructor’s Solution Manual


Copyright 2018, Rice University. All Rights Reserved. This ancillary resource is intended for instructor use only and
may not be uploaded, redistributed, or reproduced without written approval by OpenStax.
Table of Contents – Part 1

Science Practice Challenge Questions..................................................................................................................650

26 | The Nervous System.....................................................................................................652

Review Questions.................................................................................................................................................652

Critical Thinking Questions...................................................................................................................................654

Test Prep for AP® Courses....................................................................................................................................657

Science Practice Challenge Questions..................................................................................................................668

27 | Sensory Systems...........................................................................................................680

Review Questions.................................................................................................................................................680

Critical Thinking Questions...................................................................................................................................688

Science Practice Challenge Questions..................................................................................................................696

28 | The Endocrine System..................................................................................................701

Review Questions.................................................................................................................................................701

Critical Thinking Questions...................................................................................................................................710

Test Prep for AP® Courses....................................................................................................................................717

Science Practice Challenge Questions..................................................................................................................725

29 | The Musculoskeletal System.........................................................................................731

Review Questions.................................................................................................................................................731

Critical Thinking Questions...................................................................................................................................738

Science Practice Challenge Questions..................................................................................................................743

30 | The Respiratory System................................................................................................748

Review Questions.................................................................................................................................................748

Critical Thinking Questions...................................................................................................................................753

Test Prep for AP® Courses....................................................................................................................................758

Science Practice Challenge Questions..................................................................................................................764

31 | The Circulatory System.................................................................................................766

Review Questions.................................................................................................................................................766

Advanced Placement Biology Instructor’s Solution Manual


Table of Contents – Part 1

Critical Thinking Questions...................................................................................................................................776

Test Prep for AP® Courses....................................................................................................................................780

Science Practice Challenge Questions..................................................................................................................785

32 | Osmotic Regulation and Excretion................................................................................791

Review Questions.................................................................................................................................................791

Critical Thinking Questions...................................................................................................................................798

Test Prep for AP® Courses....................................................................................................................................807

33 | The Immune System.....................................................................................................823

Review Questions.................................................................................................................................................823

Critical Thinking Questions...................................................................................................................................831

Test Prep for AP® Courses....................................................................................................................................838

Science Practice Challenge Questions..................................................................................................................848

34 | Animal Reproduction and Development.......................................................................852

Review Questions.................................................................................................................................................852

Critical Thinking Questions...................................................................................................................................861

Test Prep for AP ® Courses...................................................................................................................................869

Science Practice Challenge Questions..................................................................................................................878

Advanced Placement Biology Instructor’s Solution Manual


Copyright 2018, Rice University. All Rights Reserved. This ancillary resource is intended for instructor use only and
may not be uploaded, redistributed, or reproduced without written approval by OpenStax.
1 | The Study of Life 1

1 | THE STUDY OF LIFE


REVIEW QUESTIONS
1 What is a suggested and testable explanation for an event called?
A Discovery
B Hypothesis
C Scientific method
D Theory
Solution The solution is (B). A hypothesis is a testable explanation for an event.
2 Which branch of science is NOT considered a natural science?
A Astronomy
B Biology
C Computer science
D Physics
Solution The solution is (C). Computer science is a practical and scientific approach to
computation and its application. It is not considered a natural science.
3 What is the name for the formal process through which scientific research is checked for
originality, significance, and quality before being accepted into the scientific literature?
A Publication
B Public speaking
C Peer review
D Scientific method
Solution The solution is (C). Peer review is the process in which scientific papers are reviewed
by experts in the scientist’s field to ensure that a scientist’s research is original,
significant, logical, and thorough.
4 What are (i) two topics that are likely to be studied by biologists and (ii) two areas of
scientific study that would fall outside the realm of biology?
A (i) diseases affecting humans, pollution affecting species’ habitat; (ii) calculating
surface area of rectangular ground, functioning of planetary orbits
B (i) calculating surface area of rectangular ground, functioning of planetary orbits;
(ii) formation of metamorphic rocks, galaxy formation and evolution

Advanced Placement Biology Instructor’s Solution Manual


2 1 | The Study of Life

C (i) plant responses to external stimuli, functioning of planetary orbits; (ii) formation of
metamorphic rocks, galaxy formation and evolution
D (i) plant responses to external stimuli, study of the shape and motion of physical
objects; (ii) formation of metamorphic rocks, galaxy formation and evolution
Solution The solution is (A). Diseases affecting humans and pollution affecting species are two
topics that can be studied by biologists, while calculating the surface area of a
rectangular plot of land and the function of planetary orbits are two topics that fall
outside of the realm of biology.
5 Which statement is an example of deductive reasoning?
A Most swimming animals use fins; therefore, fins are an adaptation to swimming.
B Mitochondria are inherited from the mother; therefore, maternally inherited traits are
encoded by mitochondrial DNA.
C Small animals lose more heat than larger animals. One would not expect to find wild
mice in the North and South Poles.
D Water conservation is a major requirement to survive in the desert. Large leaves
increase loss of water by evaporation. Therefore, desert plants should have smaller
leaves.
Solution The solution is (D). Water conservation is a major requirement to survive in the
desert. On plants, broad, thin leaves lose more water to evaporation than smaller,
thicker leaves. Therefore, desert plants should have smaller leaves. The answer best
demonstrates how a general principle or law is used to forecast specific results.
6 Why are viruses NOT considered living?
A Viruses are not made of cells.
B Viruses do not have genetic material.
C Viruses have DNA and RNA.
D Viruses are obligate parasites and require a host.
Solution The solution is (A). Viruses are not made up of cells. They are made up of DNA or
RNA surrounded by a protein coat. Biologists characterize living organisms as being
composed of cells.
7 The presence of a membrane-enclosed nucleus is a characteristic of what?
A Bacteria
B Eukaryotic cells
C All living organisms
D Prokaryotic cells
Solution The solution is (B). Unlike prokaryotic cells, eukaryotic cells contain a membrane-
bound nucleus.

Advanced Placement Biology Instructor’s Solution Manual


1 | The Study of Life 3

8 What is a group of individuals of the same species living in the same area called?
A A community
B An ecosystem
C A family
D A population
Solution The solution is (D). A population is a group of individuals that belong to the same
species, live in a particular geographical area, and have the capability to interbreed.
9 Which sequence represents the hierarchy of biological organization from the most
inclusive to the least complex level?
A Biosphere, ecosystem, community, population, organism
B Organelle, tissue, biosphere, ecosystem, population
C Organism, organ, tissue, organelle, molecule
D Organism, community, biosphere, molecule, tissue, organ
Solution The solution is (A). The biosphere is the highest level, which is the most inclusive,
and the organism is the lowest level and least inclusive in the given hierarchy.
10 Where in a phylogenetic tree would you expect to find the organism that had evolved
most recently?
A At the base
B At the nodes
C At the branch tips
D Within the branches
Solution The solution is (C). The "tips" of the tree branches represent the recently evolved
taxa.
11 What is a characteristic that is NOT present in all living things?
A Homeostasis and regulation
B Metabolism
C Nucleus containing DNA
D Reproduction
Solution The solution is (C). A nucleus containing DNA is not present in all living organisms.

Advanced Placement Biology Instructor’s Solution Manual


4 1 | The Study of Life

CRITICAL THINKING QUESTIONS


12 Is mathematics a natural science? Explain your reasoning.
A No, it is not a natural science because it is not used in the study of the natural world.
B No, it is not a natural science. Mathematics focuses on understanding mathematical
relations and calculations, which is useful in natural sciences but which is distinct.
C Yes, it is a natural science. Mathematics deals with verifying the experimental data.
D Yes, it is a natural science. It uses chemical and physical measurements.
Solution The solution is (B). No, it is not a natural science. Mathematics does not involve
measuring aspects of the natural world.
13 Although the scientific method is used by most of the sciences, it can also be applied to
everyday situations. A situation is given below. Using the scientific method try to arrange
the given steps in the correct order. Situation:
1. If the car doesn’t start the problem might be in the battery.
2. Car doesn’t start.
3. After changing the battery. Car starts working.
4. The car should start after charging the battery or changing the battery.
5. The car doesn’t start because the battery is dead.
6. The car doesn’t start even after charging the battery, the battery must have stopped
working.
A 1, 2, 3, 4, 5, 6
B 2, 1, 3, 4, 5, 6
C 2, 1, 5, 4, 6, 3
D 2, 1, 5, 6, 3, 4
Solution The solution is (B). 2, 1, 5, 4, 6, 3 is the correct sequence of the situation given.

14 Read questions 1 through 4.


Question 1. Is macaroni and cheese tastier than broccoli soup?
Question 2. Are hummingbirds attracted to the color red?
Question 3. Is the moon made out of green cheese?
Question 4. Is plagiarism dishonest?
Which questions lend themselves to investigation using scientific methods? In other
words, is the hypothesis falsifiable (can be proven false)?

Advanced Placement Biology Instructor’s Solution Manual


1 | The Study of Life 5

A Questions 1 and 2 are subjective and cannot be disproven using scientific methods.
Questions 3 and 4 can be tested using scientific methods.
B Questions 3 and 4 are subjective and cannot be disproven using scientific methods.
Questions 1 and 2 can be tested using scientific methods.
C Questions 1 and 3 are subjective and cannot be disproven using scientific methods.
Questions 2 and 4 can be tested using scientific methods.
D Questions 1 and 4 are subjective and cannot be disproven using scientific methods.
Questions 2 and 3 can be tested using scientific methods.
Solution The solution is (D). Questions 1 and 4 are subjective because taste, dishonesty, and
plagiarism are subjective, whereas questions 2 and 3 can be tested using scientific
methods. The hypotheses for questions 2 and 3 are falsifiable (can be disproven).
15 Consider the levels of organization of the biological world and place each of these items in
order from narrowest level of organization to most encompassing: skin cell, elephant,
water molecule, planet Earth, tropical rain forest, hydrogen atom, wolf pack, liver.
A Hydrogen atom, water molecule, skin cell, liver, elephant, wolf pack, tropical rain
forest, planet Earth
B Hydrogen atom, skin cell, water molecule, liver, elephant, wolf pack, tropical rain
forest, planet Earth
C Hydrogen atom, skin cell, water molecule, liver, wolf pack, elephant, tropical rain
forest, planet Earth
D Water molecule, hydrogen atom, skin cell, liver, elephant, wolf pack, tropical rain
forest, planet Earth
Solution The solution is (A). The narrowest level of organization to the most-encompassing is:
hydrogen atom, water molecule, skin cell, liver, elephant, wolf pack, tropical
rainforest, planet Earth.
16 What scientific evidence did Carl Woese use to determine that there should be a separate
domain for Archaea?
A A sequence of DNA
B A sequence of rRNA
C A sequence of mRNA
D A sequence of tRNA
Solution The solution is (B). Woese used sequences of rRNA as evidence to determine that
there should be a separate domain for Archaea.
17 Both astronomy and astrology study the stars. Which one is considered a natural science?
Explain your reasoning.

Advanced Placement Biology Instructor’s Solution Manual


6 1 | The Study of Life

A Astrology is a natural science because it indirectly influences human affairs and the
natural world.
B Astronomy is a natural science because it deals with observations and predictions of
events in the sky, which are based on the laws of physics.
C Astrology is a natural science because it deals with observations and predictions of
events in the sky, and influences human affairs and the natural world.
D Astrology is a natural science because it deals with the study of asteroids and comets,
which is based on the laws of natural sciences.
Solution The solution is (B). Astrology does involve observations of celestial bodies, but it is a
way to predict human affairs based on astronomical data that do not rely on testable
hypotheses. Astronomy is the science of observing, measuring, and predicting
events in space based on past observations and the laws of physics.

TEST PREP FOR AP® COURSES


18 Which structure is conserved in all living organisms and points to a common origin?
A All living organisms have mitochondria that produce energy.
B All living organisms store genetic material in DNA/RNA.
C All living organisms use the energy from sunlight.
D All living organisms have a nucleus.
Solution The solution is (B). This is true of all organisms and, therefore, logically suggests a
common origin.
19 Which statement is the strongest argument in favor of two organisms, A and B, being
closely related evolutionarily?
A A and B look alike.
B A and B live in the same ecosystem.
C A and B use the same metabolic pathways.
D The DNA sequences of A and B are highly homologous.
Solution The solution is (D). All living organisms store genetic material in DNA/RNA so this is
one of the strongest ways to probe for similarities.
20 In the phylogenetic tree shown, which organism is most distantly related to organism 2?

Advanced Placement Biology Instructor’s Solution Manual


1 | The Study of Life 7

A Organism 1
B Organism 3
C Organism 4
D Organism 5
Solution The solution is (D). Organism 5 is most distantly related to organism 2 as it is evident
in the phylogenetic tree that 5 diverged from the main branch much earlier than rest
of the organisms.
21 In the phylogenetic tree shown, which organism is the most recent common ancestor of
organisms 1 and 3?

A Organism A
B Organism B
C Organism C
D Organism D
Solution The solution is (B). Organism B is the recent common ancestor of 1 and 3 because B
is the base of 1 and 3.
22 The French scientist Jacques Monod famously said, “Anything found to be true of E. coli
must also be true of elephants.” How is this statement based on the notion that living
organisms share a common ancestor?
A E. coli is a eukaryote and shares similarities with most living organisms.
B E. coli is a prokaryote. The various metabolic processes and core functions in E. coli
share homology with higher organisms.
C E. coli contains a nucleus and membrane-bound cell organelles that are shared by all
living organisms.
D E. coli is a prokaryote and reproduces through binary fission, which is common to
most living organisms.
Solution The solution is (B). All organisms share core structures and functions; therefore,
basic research on E. coli metabolic pathways, molecular biology, and other topics
applies to higher organisms.
23 Birds have been reclassified as reptiles. What is one line of evidence that has led to this
reclassification?

Advanced Placement Biology Instructor’s Solution Manual


8 1 | The Study of Life

A Archeopteryx is the connecting link between birds and reptiles, which shows that birds
and reptiles are related.
B Birds have scales, so they have the same origin as reptiles.
C Birds and reptiles have the same circulatory and excretory systems and both are egg-
laying animals.
D Birds and reptiles have similar anatomical and morphological features.
Solution The solution is (B). Birds have feathers, which are understood to be modified scales,
as well as scales on their feet. They show some skeletal similarity, which supports
their reclassification as reptiles.

Advanced Placement Biology Instructor’s Solution Manual


2 | The Chemical Foundation of Life 9

2 | THE CHEMICAL FOUNDATION OF LIFE


REVIEW QUESTIONS
1 What are atoms that vary in the number of neutrons found in their nuclei called?
A Ions
B Isotopes
C Isobars
D Neutral atoms
Solution The solution is (B). Isotopes are atoms that have the same number of protons but
differ in the number of neutrons.
2 Potassium has an atomic number of 19. What is its electron configuration?
A Shells 1 and 2 are full, and shell 3 has nine electrons.
B Shells 1, 2, and 3 are full, and shell 4 has three electrons.
C Shells 1, 2, and 3 are full, and shell 4 has one electron.
D Shells 1, 2, and 3 are full, and no other electrons are present.
Solution The solution is (C). Shells 1, 2, and 3 are full, as they hold a total of 18 electrons, and
shell 4 has the remaining one electron and is unfilled.
3 Which type of bond exemplifies a weak chemical bond?
A Covalent bond
B Hydrogen bond
C Ionic bond
D Nonpolar covalent bond
Solution The solution is (B). A hydrogen bond forms between hydrogen and an
electronegative atom. It is weak because electrons are not shared as they are in a
covalent bond nor are they transferred to form opposite, attracting charges, as they
are in ionic bonds. Instead, hydrogen has a slight positive charge when covalently
bonded to a more electronegative atom, because that atom draws the hydrogen’s
electron away from it, causing it to become attracted to nearby electronegative
atoms with unpaired electrons.
4 Which statement is false?
A Electrons are unequally shared in polar covalent bonds.
B Electrons are equally shared in nonpolar covalent bonds.

Advanced Placement Biology Instructor’s Solution Manual


10 2 | The Chemical Foundation of Life

C Hydrogen bonds are weak bonds based on electrostatic forces.


D Ionic bonds are generally stronger than covalent bonds.
Solution The solution is (D). Ionic bonds are generally stronger than covalent bonds.
5 If xenon has an atomic number of 54 and a mass number of 108, how many neutrons does
it have?
A 27
B 54
C 100
D 108
Solution The solution is (B). The number of neutrons can be calculated by subtracting the
element’s atomic number from its mass number. When you subtract 54 from 108,
the number of neutrons is 54.
6 What forms ionic bonds?
A Atoms that share electrons equally
B Atoms that share electrons unequally
C Ions with similar charges
D Ions with opposite charges
Solution The solution is (D). Ions with similar charges will not likely come together to form
ionic bonds; they must be oppositely charged.
7 Based on the information provided, which of the following statements is correct?

A In the nitrogen atom acquires a partial positive charge and the hydrogen atoms
acquire a partial negative charge.

B In the hydrogen atoms acquire a partial negative charge, and the oxygen atom
acquires a partial positive charge.

Advanced Placement Biology Instructor’s Solution Manual


2 | The Chemical Foundation of Life 11

C In aqueous HCl, the hydrogen atom acquires a partial positive charge, and the chlorine
atom acquires a partial negative charge.
D In the lithium atom acquires a positive charge, and the fluorine atom acquires a
negative charge.

Solution The solution is (C). In the hydrogen atom acquires a partial positive charge,
and the chlorine atom acquires a partial negative charge.
8 Which statement is NOT true?
A Water is polar.
B Water can stabilize the temperature of nearby air.
C Water is essential for life.
D Water is the most abundant molecule in Earth’s atmosphere.
Solution The solution is (D). Water is the most abundant molecule in Earth’s atmosphere.
9 Why do hydrogen and oxygen form polar covalent bonds within water molecules?
A Hydrogen is more electronegative than oxygen, generating a partial negative charge
near the hydrogen atoms.
B Hydrogen is more electronegative than oxygen, generating a partial positive charge
near the hydrogen atoms.
C Oxygen is more electronegative than hydrogen, generating a partial negative charge
near the oxygen atom.
D Oxygen is more electronegative than hydrogen, generating a partial positive charge
near the oxygen atom.
Solution The solution is (C). The oxygen atom nucleus is more attractive to the electrons of a
hydrogen atom than the hydrogen nucleus is to the oxygen’s electrons. Therefore,
the hydrogen atom acquires a partial positive charge while the oxygen atom
acquires a partial negative charge.
10 What happens to the pH of a solution when acids are added?
A The pH of the solution decreases.
B The pH of the solution increases.
C The pH of the solution increases and then decreases.
D The pH of the solution stays the same.

Solution The solution is (A). An acid is a substance that increases the ion concentration of
the solution by dissociating its atoms. Thus, the pH of the solution decreases on
addition of a hydrogen atom.

Advanced Placement Biology Instructor’s Solution Manual


12 2 | The Chemical Foundation of Life

11 Which statement is true?


A Acids and bases cannot mix together.
B Acids and bases can neutralize each other.
C Acids, not bases, can change the pH of a solution.

D Acids donate hydroxide ions ( ); bases donate hydrogen ions ( ).

Solution The solution is (B). Acids add ions in a solution, while bases add ions to a
solution. These ions neutralize each other by forming water molecules.
12 What is water’s property of adhesion?
A The force that allows surface water molecules to escape and vaporize
B The attraction between water molecules and other molecules
C The intermolecular force between water molecules
D The force that keeps particles dispersed in water
Solution The solution is (B). Adhesion is the tendency of different particles to cling to one
another. It is sometimes a strong adhesive force, such as when water is exposed to
charged surfaces.
13 In a solution, what kind of molecule binds up excess hydrogen ions?
A Acid
B Base
C Donator
D Isotope

Solution The solution is (B). Bases bind excess ions in solution, resulting in a neutral
solution. For example, the addition of to produces water.
14 What is the maximum number of atoms or molecules a single carbon molecule can bond
with?
A 4
B 1
C 6
D 2
Solution The solution is (A). The carbon atom has four electrons in its valence shell.
Therefore, bonding four more electrons will complete its octet, allowing it to attain a
stable configuration. A carbon molecule can therefore bond with a maximum of four
other atoms.

Advanced Placement Biology Instructor’s Solution Manual


2 | The Chemical Foundation of Life 13

15 Which statement is true?

A Molecules with the formulas and could be structural isomers.


B Molecules must have a single bond to be cis-trans isomers.
C To be enantiomers, a molecule must have at least three different atoms or groups
connected to a central carbon.
D To be enantiomers, a molecule must have at least four different atoms or groups
connected to a central carbon.
Solution The solution is (D). To be enantiomers, a molecule must have at least four different
atoms or groups connected to a central carbon.
16 What is NOT a functional group that can bond with carbon?
A Carbonyl
B Hydroxyl
C Phosphate
D Sodium
Solution The solution is (D). Sodium is not a functional group; it is an element. Thus, it does
not bond with carbon as a functional group.
17 Which functional group is NOT polar?
A Carbonyl
B Hydroxyl
C Methyl
D Sulfhydryl
Solution The solution is (C). Methyl groups are nonpolar because carbon and hydrogen have
similar electronegativity. This means the covalent bond between carbon and
hydrogen has electrons that are equally shared by the carbon and hydrogen.
Therefore, it is a nonpolar molecule.
18 What are enantiomers?
A Hydrocarbons consisting of closed rings of carbon atoms
B Isomers with similar bonding patterns differing in the placement of atoms along a
double covalent bond
C Molecules that share the same chemical bonds but are mirror images of each other
D Molecules with the same chemical formula that differ in the placement of their
chemical bonds
Solution The solution is (C). Enantiomers are molecules that share the same chemical bonds
but are mirror images of each other.

Advanced Placement Biology Instructor’s Solution Manual


14 2 | The Chemical Foundation of Life

CRITICAL THINKING QUESTIONS


19 What are the mass number and atomic number of carbon-12 and carbon-13, respectively?
A The mass number and atomic numbers of carbon-12 are 12 and 6, while those of
carbon-13 are 13 and 6.
B The mass number and atomic numbers of carbon-12 are 12 and 6, while those of
carbon-13 are 13 and 12.
C The mass number and atomic numbers of carbon-12 are 12 and 6, while those of
carbon-12 are 13 and 13.
D The mass number and atomic numbers of carbon-12 are 12 and 12, while those of
carbon-13 are 13 and 12.
Solution The solution is (A). The mass number and atomic number of carbon-12 are 12 and 6,
while those of carbon-13 are 13 and 6. Use the figure to determine the mass and
atomic numbers.

20 Why are hydrogen bonds and van der Waals interactions necessary for cells?
A Hydrogen bonds and van der Waals interactions form weak associations between
molecules, providing the necessary shape and structure for DNA and proteins to
function in the body.
B Hydrogen bonds and van der Waals interactions form strong associations between
molecules, providing the necessary shape and structure for DNA and proteins to
function in the body.
C Hydrogen bonds and van der Waals interactions form weak associations between
different molecules, providing the necessary shape and structure for acids to function
in the body.
D Hydrogen bonds and van der Waals interactions form strong associations between the
same molecules, providing the necessary shape and structure for acids to function in
the body.

Advanced Placement Biology Instructor’s Solution Manual


2 | The Chemical Foundation of Life 15

Solution The solution is (A). Hydrogen bonds and van der Waals interactions form weak
associations between different molecules or within different regions of the same
molecule. They provide the structure and shape necessary for proteins and DNA
within cells so that they function properly.
21 Using the figure, which two groups will form a strong ionic bond?

A Group 1 and Group 17


B Group 1 and Group 14
C Group 14 and Group 18
D Group 1 and Group 18
Solution The solution is (A). Group 1 and Group 17 will form a strong ionic bond. Group 1 has
one valence electron, so to obtain a stable configuration, it will donate the electron
readily. Group 17 has 7 electrons in its valence shell. In order to obtain a stable
configuration, it will accept one electron readily.
22 Why can some insects walk on water?
A Insects can walk on water because of its high surface tension.
B Insects can walk on water because it is a polar solvent.
C Insects can walk on water because they are less dense than water.
D Insects can walk on water because they are denser than water.
Solution The solution is (A). Some insects can walk on water, although they are denser than
water, because of the surface tension of water. Surface tension is a result of
cohesion, or the attraction between water molecules at the surface of the body of
water (the liquid-air/gas interface).

Advanced Placement Biology Instructor’s Solution Manual


16 2 | The Chemical Foundation of Life

23 Which statement describes how buffers help prevent drastic swings in pH?
A Buffers absorb excess hydrogen and hydroxide ions to prevent increases or decreases
in pH. An example is the bicarbonate system in the human body.
B Buffers absorb excess hydrogen ions to prevent increases or decreases in pH. An
example is the bicarbonate system in the human body.
C Buffers absorb excess hydroxide ions to prevent increases or decreases in pH. An
example is the bicarbonate system in the human body.
D Buffers absorb excess hydrogen and hydroxide ions to prevent increases or decreases
in pH. An example is carbonate system in the human body.
Solution The solution is (A). Buffers absorb the free hydrogen ions and hydroxide ions
produced by chemical reactions. Because they can bond to these ions, they prevent
increases or decreases in pH. An example of a buffer system is the bicarbonate
system in the human body. This system is able to absorb hydrogen and hydroxide
ions to prevent changes in pH and keep cells functioning properly.
24 What are three examples of how the characteristics of water are important in
maintaining life?
A First, the lower density of water as a solid versus a liquid allows ice to float, forming an
insulating surface layer for aquatic life. Second, the high specific heat capacity of
water insulates aquatic life or bodily fluids from temperature changes. Third, the high
heat of vaporization of water allows animals to cool themselves by sweating.
B First, the higher density of water as a solid versus a liquid allows ice to float, forming
an insulating surface layer for aquatic life. Second, the high specific heat capacity of
water insulates aquatic life or bodily fluids from temperature changes. Third, the low
heat of vaporization of water allows animals to cool themselves by sweating.
C First, the lower density of water as a solid versus a liquid allows ice to float, forming an
insulating surface layer for aquatic life. Second, the low specific heat capacity of water
insulates aquatic life or bodily fluids from temperature changes. Third, the high heat
of vaporization of water allows animals to cool themselves by sweating.
D First, the lower density of water as a solid versus a liquid allows ice to float, forming an
insulating surface layer for aquatic life. Second, the low specific heat capacity of water
insulates aquatic life or bodily fluids from temperature changes. Third, the low heat of
vaporization of water allows animals to cool themselves by sweating.
Solution The solution is (A). The lower density of ice compared to liquid water allows it to
float on water. In lakes and ponds, ice will form on the surface of water creating an
insulating barrier that protects the animals and plant life in the pond from freezing.
Water’s lower density in its solid form is due to the orientation of hydrogen bonds as
it freezes: the water molecules are pushed farther apart compared to liquid water.

Advanced Placement Biology Instructor’s Solution Manual


2 | The Chemical Foundation of Life 17

Water is used by warm-blooded animals to more evenly disperse heat in their


bodies. Water has the highest specific heat capacity of any liquid, a property caused
by hydrogen bonding between water molecules.
In many living organisms, the evaporation of sweat allows organisms to cool to
maintain homeostasis of body temperature. This is because water has a high heat of
vaporization. As liquid water heats up, hydrogen bonding makes it difficult to
separate the liquid water molecules from each other.
Other examples include water’s solvent properties as well as water’s cohesive and
adhesive properties.
25 What is the pH scale, and how does it relate to living systems? Give an example of how
drastic pH changes are prevented in living systems.
A The pH scale ranges from 0 to 14, where anything below 7 is acidic and above 7 is
alkaline. The bicarbonate system in the human body buffers the blood.
B The pH scale ranges from 0 to 14, where anything below 7 is alkaline and above 7 is
acidic. The bicarbonate system in the human body buffers the blood.
C The pH scale ranges from 0 to 7, where anything below 7 is acidic and above 7 is
alkaline. Water in the human body buffers the blood.
D pH scale ranges from 0 to 7, where anything below 4 is acidic and above 4 is alkaline.
Water in the human body buffers the blood.
Solution The solution is (A). The pH scale ranges from zero to 14. Pure water has a pH of
seven, which is known as neutral pH. The pH of human cells and blood is maintained
at pH seven. Anything below pH seven is acidic and anything above seven is alkaline.
Extreme pH is usually considered inhospitable for life. Buffers maintain pH
homeostasis. For example, the bicarbonate system in the human body absorbs
hydrogen and hydroxide ions to prevent changes in pH.
26 What property of carbon makes it essential for organic life?
A Carbon can form up to four covalent bonds, allowing it to form long chains.
B Carbon can form more than four covalent bonds, allowing it to form long chains.
C Carbon can form more than four covalent bonds, but it can only form short chains.
D Carbon can form up to four covalent bonds, but it can only form short chains.
Solution The solution is (A). Carbon is found in all living things because it can form up to four
covalent bonds. These strong covalent bonds allow the formation of long chains that
combine to form proteins and DNA.

Advanced Placement Biology Instructor’s Solution Manual


18 2 | The Chemical Foundation of Life

27 What property of carboxyl makes carboxyl-containing molecules hydrophilic? Which


macromolecules contain carboxyl?
A Carboxyl groups release H+, making its parent molecule hydrophilic. It is found in
amino acids and fatty acids.
B Carboxyl groups absorb H+, making its parent molecule hydrophilic. It is found in
phospholipids and triglycerides.

C Carboxyl groups release making its parent molecule hydrophilic. It is found in


phospholipids, phosphates, and triglycerides.

D Carboxyl groups release making its parent molecule hydrophilic. It is found in


phospholipids and DNA.
Solution The solution is (A). The carboxyl group is found in amino acids, some amino acid side
chains, and the fatty acids that form triglycerides and phospholipids. This carboxyl
group ionizes to release hydrogen ions (H+) from the group, resulting in the
negatively charged group, which contributes to the hydrophilic nature of
whatever molecule it is found on.
28 Compare and contrast saturated and unsaturated triglycerides.
A Saturated triglycerides contain single bonds and are solid at room temperature, while
unsaturated triglycerides contain double bonds and are liquid at room temperature.
B Saturated triglycerides contain double bonds and are solid at room temperature,
while unsaturated triglycerides contain single bonds and are liquids at room
temperature.
C Saturated triglycerides contain single bonds and are liquid at room temperature, while
unsaturated triglycerides contain double bonds and are solid at room temperature.
D Saturated triglycerides contain double bonds and are liquid at room temperature,
while unsaturated triglycerides contain single bonds and are solid at room
temperature.
Solution The solution is (A). Saturated triglycerides contain no double bonds between carbon
atoms; they are usually solid at room temperature. Unsaturated triglycerides contain
at least one double bond between carbon atoms; they are usually liquid at room
temperature.

TEST PREP FOR AP® COURSES


29 What property of water makes it a good insulator within the bodies of endothermic
(warm-blooded) animals?
A Adhesion
B Surface tension

Advanced Placement Biology Instructor’s Solution Manual


2 | The Chemical Foundation of Life 19

Advanced Placement Biology Instructor’s Solution Manual


20 2 | The Chemical Foundation of Life

C Heat of vaporization
D Specific heat capacity
Solution The solution is (D). Specific heat is defined as the amount of heat one gram of a
substance must absorb or lose to change its temperature by 1 °C Therefore, warm-
blooded animals use water to more evenly disperse heat in their bodies. It takes a
large amount of energy to heat or cool water.
30 The unique properties of water are important in biological processes. For the following
three properties of water, define the property and give one example of how the property
affects living organisms:
 Cohesion
 Adhesion
 High heat of vaporization
A Cohesion is the attraction between water molecules, which helps create surface
tension. Insects can walk on water because of cohesion. Adhesion is the attraction
between water molecules and other molecules. Water moving up from the roots of
plants to the leaves as a result of capillary action is because of adhesion. Heat of
vaporization is the amount of energy required to convert liquid into gas. This property
helps humans maintain homeostasis of body temperature by evaporation.
B Cohesion is the attraction between water and other molecules, which helps create
surface tension. Insects can walk on water because of cohesion. Adhesion is the
attraction between water molecules. Water moving up from the roots of plants to the
leaves as a result of capillary action is because of adhesion. Heat of vaporization is the
amount of energy required to convert liquid into gas. This property helps humans
maintain homeostasis of body temperature by evaporation.
C Cohesion is the attraction between water molecules, which helps create surface
tension. Insects can walk on water because of cohesion. Adhesion is the attraction
between water molecules and other molecules. Water moving up from the roots of
plants to the leaves as a result of capillary action is because of adhesion. Heat of
vaporization is the amount of energy required to convert solid into gas. This property
helps humans maintain homeostasis of body temperature by evaporation.
Solution The solution is (A). Cohesion is the attraction between water molecules, which helps
create surface tension. Insects can walk on water because of cohesion. Adhesion is
the attraction between water molecules and other molecules. Water moving up
from the roots of plants to the leaves as a result of capillary action is because of
adhesion. Heat of vaporization is the amount of energy required to convert liquid
into gas. This property helps humans maintain homeostasis of body temperature by
evaporation.

Advanced Placement Biology Instructor’s Solution Manual


2 | The Chemical Foundation of Life 21

SCIENCE PRACTICE CHALLENGE QUESTIONS


2.1 Atoms, Isotopes, Ions, and Molecules: The Building Blocks
31 At a time when the theory of evolution was controversial (the year following the Scopes
Monkey Trial), Macallum (Physiological Reviews, 2, 1926) made an observation that is still
contested by some who do not see the pattern in the data in the table showing
percentages (g solute/100 g solution) of major biologically important inorganic elements
in a variety of sources.
Source
Ocean water 0.306 0.011 0.012 0.0037 0.55
Lobster 0.903 0.0337 0.0438 0.0156 1.547
Dog fish 0.5918 0.02739 0.0160 0.0146 0.9819
9
Sand shark 0.6173 0.0355 0.0184 0.0172 1.042
Cod 0.416 0.0395 0.0163 0.00589 0.6221
Pollock 0.4145 0.017497 0.0128 0.00608 0.5613
6
Frog 0.195 0.0233 0.0062 0.00155 0.2679
7
Dog lymph 0.3033 0.0201 0.0085 0.0023 0.4231
Human Blank blank blank blank blank

Blood 0.302 0.0204 0.0094 0.0021 0.389


Lung 0.2956 0.02095 0.0083 0.0021 0.3425
9
Testes 0.3023 0.01497 0.0084 0.001914 0.3737
2
Abdominal 0.2935 0.0164 0.0091 0.00184 0.3888
cavity
A. Using a spreadsheet, or by sharing calculations with your classmates, construct a
quantitative model of these data from these percentages as ratios of mass fractions
relative to that of sodium, %X/%Na. Of course, you will not be asked to use a spreadsheet
on the AP Biology Exam. However, you may be assessed on the ability to develop a
quantitative model through the transformation of numerical data. The question that led
Macallum to investigate the elemental composition of different species and compare
these with the composition of seawater follows from the central organizing principle of
biology: the theory of evolution.
B. The elements in the table all occur in aqueous solution as ions. Cells expend energy to
produce a charge gradient and an ion gradient that is necessary for basic cellular

Advanced Placement Biology Instructor’s Solution Manual


22 2 | The Chemical Foundation of Life

functions. Large differences in the concentrations of ions inside versus outside of the cell
results in stresses that the cell must expend energy to relieve. Based on this constraint on
the total number of ions, connect this refined model based on ratios of ion concentration
rather than absolute ionic concentrations to the modern concept of shared ancestry.
Frequently, a follow-up question regarding scientific data on the AP Biology Exam will ask
you to pose questions that are raised by the data. You will be awarded credit for scientific
questions. These questions usually look for a cause-and-effect relationship, and are
testable.
C. Examine relative concentrations of potassium and magnesium ions in terrestrial and
marine organisms. Pose a question that could be investigated to connect concentrations
of these ions to adaptations to a change in the environment.
Macallum noted the high potassium to sodium ratio relative to seawater, and made this
claim about what the ratio implied about the oceans of early Earth:
“At once it is suggested that as the cell is older than its media is [presently] the relative
proportions of the inorganic elements in it are of more ancient origin than the relative
proportions of the same amount of elements which prevail in the media, blood plasma
and lymph or in the ocean and river water of today.”
D. In your own words, summarize the argument that Macallum is using to justify this
claim.
Solution Sample answer:
A.
Source
Ocean water 0.036 0.039 0.012 1.8
Lobster 0.037 0.048 0.017 1.7
Dog fish 0.046 0.027 0.025 1.7
Sand shark 0.057 0.030 0.028 1.7
Cod 0.095 0.039 0.014 1.5
Pollock 0.042 0.031 0.015 1.35
Frog 0.12 0.032 0.0080 1.4
Dog lymph 0.07 0.028 0.0076 1.4
Human blank blank blank blank

Blood 0.067 0.031 0.0070 1.3


Lung 0.071 0.028 0.0071 1.2
5
Testes 0.049 0.028 0.0063 1.2

Advanced Placement Biology Instructor’s Solution Manual


2 | The Chemical Foundation of Life 23

Source
Abdominal 0.056 0.031 0.0062 1.3
cavity
Table of Ratios %ion/%Na
B. The hypothesis that all animal life originated in the oceans seems to be supported
by the observation that the ratios of ions in the tissues of terrestrial animals are
similar to those found in ocean water. The absolute amounts of ions may vary from
animal to animal, however, relatively similar ratios point to inherited cellular
mechanisms that require similar ratios of ions.

C. The lower concentrations of and in terrestrial animals compared to


those found in ocean water can be attributed to adaptation to life in air rather than
a reflection of concentrations present in ancient oceans. One question to pose
would be why the concentrations of and changed over time in the oceans.
Examination of ancient sediments, and studies of how the weathering of rock adds
salts into the ocean may provide insights into the composition of ancient oceans.
D. The proportion of ions in the blood and tissues of terrestrial animals are a
reflection of the composition of salts in the marine environment existing at the time
when animals moved to land. It is not tied to present day ocean composition.
2.2 Water
32 Approximately half the energy that flows through Earth’s biosphere is captured by
phytoplankton, photosynthetic microscopic organisms in the surface waters of the
oceans. Scientists think the growth of phytoplankton in the Atlantic Ocean is limited by
the availability of nitrogen, whereas growth in the Pacific Ocean is limited by the
availability of iron.
The concentration of oxygen (O2) in the atmosphere of early Earth was low and, therefore,
so was the concentration of dissolved oxygen in the early ocean. Because insoluble iron

oxides (rust) do not form in the absence of oxygen, soluble iron ions were more
available in the early ocean than at present. Nitrogen (N2), while always abundant in the
atmosphere, was not biologically available until the evolution of molybdenum-based
enzymes that allow nitrogen fixation.
The graphs (Anbar and Knoll, Science, 297, 2002) show models of concentrations of two
trace elements, iron (Fe) and molybdenum (Mo), in ocean waters (first), as well as the
concentration of various elements dissolved in seawater versus within cells (second). The
model describes the change over time of these elements from early Earth (>1.85 billion
years ago, Gya) to a modern era (<1.25 Gya) and a period of transition between these.
Surface waters of the oceans lie to the left of the vertical double line while deep ocean
water lies to the right. Modern concentrations of dissolved iron and molybdenum (relative
to dissolved carbon) are shown.

Advanced Placement Biology Instructor’s Solution Manual


24 2 | The Chemical Foundation of Life

A. The principal chemical processes of life today have been conserved through evolution
from early Earth conditions. Using this fact, justify the selection of these data shown in
the graphs in the construction of a model of ocean photosynthetic productivity.
Iron and molybdenum are two of 30 elements that are required by the chemical processes
supporting life on Earth. Concentrations of these two and 15 other elements are shown in
the second graph. Of these elements, the three most abundant in cells are also found in
seawater in approximately the same concentrations. By increasing the mass of
phytoplankton in the ocean, we may be able to compensate for the increasing
concentration of carbon produced by the combustion of gas, oil, and coal.

Advanced Placement Biology Instructor’s Solution Manual


2 | The Chemical Foundation of Life 25

B. Select, with justification, the element or elements that, if added in large amounts to
the ocean, could boost the growth of phytoplankton.
C. Before implementing a large-scale geo-engineering effort to avert the effects of climate
change due to carbon pollution, we must test the legitimacy of this solution. Describe a
plan for collecting data that could be used to evaluate the effect of enrichment on
phytoplankton productivity.
Solution Sample answer:
A. The first graph shows the concentration of rate-limiting elements, including iron
and molybdenum, available for photosynthesis. At the surface molybdenum is
abundant. Iron is less available, perhaps because it reacts with oxygen in the
atmosphere to form insoluble iron oxide that sinks.
B. The second graph shows that iron, manganese, and cobalt are also abundant in
the ocean than in the cells. This means cells will have a hard time obtaining enough
of these elements, and adding more would increase productivity. Manganese is an
essential component of the reaction center of photosystem II where it is involved in
the splitting of water molecules.
C. In controlled laboratory conditions, supplemental metal could be added to
phytoplankton grown and the productivity could be measured. It is important to
stress that the oceans are vast areas with different local conditions. The unintended
effects of overgrowth of phytoplankton must also be addressed. Eutrophication, or
overgrowth of algae leading to dead zones in lakes and coastal areas, occurs when
excess nitrogen is present in the water.

Advanced Placement Biology Instructor’s Solution Manual


26 3 | Biological Macromolecules

3 | BIOLOGICAL MACROMOLECULES
REVIEW QUESTIONS
1 Dehydration synthesis leads to the formation of what?
A Monomers
B Polymers
C Carbohydrates only
D Water only
Solution The solution is (B). Polymers are formed through dehydration synthesis, during
which monomers combine to release a water molecule.
2 What is removed during the formation of nucleic acid polymers?
A Carbon
B Hydroxyl groups
C Phosphates
D Amino acids
Solution The solution is (C). Unlike proteins, carbohydrates, and lipids, the molecule that is
released in the formation of nucleic acid polymers is pyrophosphate (two phosphate
groups bound together).
3 During the breakdown of polymers, which reaction takes place?
A Condensation
B Covalent bond
C Dehydration
D Hydrolysis
Solution The solution is (D). Hydrolysis is a reaction in which a water molecule is used during
the breakdown of another compound or a polymer.
4 Energy is released as a result of which chemical reaction?
A Condensation
B Dehydration synthesis
C Hydrolysis
D Dissolution
Solution The solution is (C). Hydrolysis reactions typically release energy that can be used to
power cellular processes.

Advanced Placement Biology Instructor’s Solution Manual


3 | Biological Macromolecules 27

5 In the metabolism of the cell, why is hydrolysis used?


A Hydrolysis is used to break down polymers.
B Hydrolysis is used to form linkages in DNA.
C Hydrolysis is used to produce proteins.
D Hydrolysis is used to synthesize new macromolecules.
Solution The solution is (A). Polymers can be broken down by hydrolysis, or the addition
of water.
6 Plant cell walls contain what in abundance?
A Cellulose
B Glycogen
C Lactose
D Starch
Solution The solution is (A). The cell walls of plants are mostly made of cellulose, which
provides structural support to the cell.
7 What makes up the outer layer of some insects?
A Carbohydrate
B Protein
C RNA
D Triglyceride
Solution The solution is (A). Arthropods like insects have an outer skeleton, called the
exoskeleton, which protects their internal body parts. The exoskeleton is mainly
made up of chitin, a nitrogen-containing polysaccharide.
8 What is an example of a monosaccharide?
A Cellulose
B Fructose
C Lactose
D Sucrose
Solution The solution is (B). Fructose is a simple ketose monosaccharide found in
many plants.
9 Cellulose and starch are examples of —
A disaccharides
B lipids

Advanced Placement Biology Instructor’s Solution Manual


28 3 | Biological Macromolecules

C monosaccharides
D polysaccharides
Solution The solution is (D). A long chain of monosaccharides linked by glycosidic bonds
is known as a polysaccharide, which can contain the same or different
monosaccharides. Starch is the stored form of sugars in plants and is made up
of a mixture of amylose and amylopectin, while cellulose is made up of glucose
monomers that are linked by -1-4 glycosidic bonds.
10 What type of bond joins the molecules in the disaccharide lactose, and what molecule is
joined with glucose to form lactose?
A A glycosidic bond between glucose and lactose
B A glycosidic bond between glucose and galactose
C A hydrogen bond between glucose and sucrose
D A hydrogen bond between glucose and fructose
Solution The solution is (B). Lactose is a disaccharide consisting of the monomers glucose and
galactose, which are joined together by a glycosidic bond.
11 What is structurally different about cellulose when compared to starch?
A An extra hydrogen atom is left on the monomer.

B -1,4 glycosidic linkages are used.


C -1,6 glycosidic linkages are used.
D An extra hydroxyl group is removed during synthesis.
Solution The solution is (B). Starch is made up of glucose monomers that are joined by -1-4
or -1-6 glycosidic bonds. Cellulose is made up of glucose monomers that are linked
by -1-4 glycosidic bonds only.
12 What are classified as lipids?
A Disaccharides and cellulose
B Essential amino acids
C mRNA and DNA
D Oils and waxes
Solution The solution is (D). Lipids are a group of naturally occurring molecules that include
oils, waxes, and other hydrophobic substances.
13 What is cholesterol specifically classified as?
A A lipid
B A phospholipid

Advanced Placement Biology Instructor’s Solution Manual


3 | Biological Macromolecules 29

Advanced Placement Biology Instructor’s Solution Manual


30 3 | Biological Macromolecules

C A steroid
D A wax
Solution The solution is (C). Cholesterol is a steroid having four linked carbon rings with a
short tail. It is mainly synthesized in the liver and is the precursor to many steroid
hormones.
14 Which fat serves as an animal’s major form of energy storage?
A Cholesterol
B Glycerol
C Phospholipid
D Triglycerides
Solution The solution is (D). Triglycerides, stored in adipose tissue, are a major form of energy
storage in animals. Fat cells are designed for continuous synthesis and breakdown of
triglycerides in animals.
15 Which hormones are made from cholesterol?
A Estradiol and testosterone
B Insulin and growth hormone
C Progesterone and glucagon
D Prolactin and thyroid hormone
Solution The solution is (A). Cholesterol is the precursor to many steroid hormones such
as testosterone and estradiol, which are secreted by the gonads and adrenal
endocrine glands.
16 Which characteristic is NOT true for saturated fats?
A They are solid at room temperature.
B They have single bonds within the carbon chain.
C They contain mostly hydrogen and carbon atoms.
D They tend to dissolve in water easily.
Solution The solution is (D). Saturated fats are solid at room temperature, and have single
bonds between carbon and hydrogen atoms. However, they are nonpolar molecules.
Water is a polar solvent, so it is not true that saturated fats do not dissolve in water.

Advanced Placement Biology Instructor’s Solution Manual


3 | Biological Macromolecules 31

17 Which fat has the least number of hydrogen atoms?


A Trans fat
B Saturated fat
C Unsaturated fat
D Wax
Solution The solution is (C). Unlike saturated fats, in which extensive hydrogenation leads to
close packing of molecules and results in a solid state, unsaturated fats contain the
least number of hydrogen atoms, which results in multiple bonds between carbon
atoms and loose packing of molecules.
18 Of what are phospholipids important components?
A The double bond in hydrocarbon chains
B The plasma membrane of animal cells
C The ring structure of steroids
D The waxy covering on leaves
Solution The solution is (B). Phospholipids are major constituents of the plasma membrane,
the outermost layer of animal cells. Phospholipids are responsible for the dynamic
nature of the plasma membrane.
19 What is diacylglycerol 3-phosphate?
A A phospholipid
B A phosphatidylcholine
C A phosphatidylserine
D A phosphatidate
Solution The solution is (D). A phosphate group alone attached to a diglycerol does not
qualify as a phospholipid; it is phosphatidate (diacylglycerol 3-phosphate), the
precursor of phospholipids.
20 What is the basic structure of a steroid?
A Four fused hydrocarbon rings
B Glycerol with three fatty acid chains
C Two fatty-acid chains and a phosphate group
D Two six-carbon rings
Solution The solution is (A). All steroids have four linked carbon rings and some of them
have a short tail. Cholesterol and many hormones such as estrogen and cortisol
are steroids.

Advanced Placement Biology Instructor’s Solution Manual


32 3 | Biological Macromolecules

21 Besides its use in hormone production, for what does the body use cholesterol?
A mRNA transport
B Production of bile salts
C Water reabsorption in the kidney
D Wax production
Solution The solution is (B). Cholesterol is also the precursor of bile salts, which help in the
emulsification of fats and their subsequent digestion.
22 Where is cholesterol found in cell membranes?
A Attached to the inner side of the membrane
B Attached to the outer side of the membrane
C Floating in the phospholipid tail layer
D Penetrating both lipid layers
Solution The solution is (D). Cholesterol is a component of the plasma membrane of animal
cells and is found within the phospholipid bilayer. The plasma membrane of each
cell is made up of a continuous double layer of phospholipids, interwoven with
cholesterol and proteins.
23 Which type of body cell would have a higher amount of cholesterol in its membrane?
A A cartilage cell
B A liver cell
C A red blood cell
D A spleen cell
Solution The solution is (C). Blood cells tend to travel all along the blood vessels of the body
and therefore require flexibility to travel. The cholesterol embedded in their
membrane provides them with the required flexibility.
24 What is a function of proteins in cells?
A Energy storage
B Gene storage and access
C Membrane fluidity
D Structure
Solution The solution is (D). Proteins provide structure and support for cells. They also fulfill
many other functions. For example, they allow the body to move (e.g., actin),
catalyze reactions as enzymes, act as cellular messengers and receptors, and provide
defense against pathogens as antibodies.

Advanced Placement Biology Instructor’s Solution Manual


3 | Biological Macromolecules 33

25 What type of protein facilitates or accelerates chemical reactions?


A An enzyme
B A hormone
C A membrane transport protein
D A tRNA molecule
Solution The solution is (A). Enzymes accelerate, or catalyze, chemical reactions. Almost all
metabolic processes in the cell need enzymes to occur at rates fast enough to
sustain life. Enzymes increase the rate of a reaction by lowering its activation energy.
26 What type of amino acids would you expect to find on the surface of proteins that must
interact closely with water?
Solution Amino acids that contain acidic, basic, or polar side groups are most likely to be
found on the surface of water-soluble proteins. Proteins with these charged ide
groups can interact more readily with water than if the protein had a neutral net
charge.
27 What are the monomers that make up proteins called?
A Amino acids
B Chaperones
C Disaccharides
D Nucleotides
Solution The solution is (A). Amino acids are organic compounds (monomers) that combine to
form proteins. There are 20 amino acids, 10 of which are referred to as essential
amino acids in humans because the body cannot synthesize them and they must be
supplied by the diet.
28 Where is the linkage made that combines two amino acids?
A Between the R group of one amino acid and the R group of the second
B Between the carboxyl group of one amino acid and the amino group of the other
C Between the six-carbon of both amino acids
D Between the nitrogen atoms of the amino groups in the amino acids
Solution The solution is (B). A peptide bond is formed by dehydration between the amino
group of one amino acid and the carboxyl group of a second amino group. Each
amino acid has the same fundamental structure, which consists of a central carbon
atom, also known as the alpha carbon, bonded to an amino group (NH2), a
carboxyl group (COOH), and a hydrogen atom. The fourth position bound to the
central carbon is occupied by a side chain that distinguishes the amino acids from
each other.

Advanced Placement Biology Instructor’s Solution Manual


34 3 | Biological Macromolecules

29 The -helix and the -pleated sheet are part of which protein structure?
A The primary structure
B The secondary structure
C The tertiary structure
D The quaternary structure
Solution The solution is (B). The local folding of the polypeptide in some regions gives rise to
the secondary structure of the protein. The -helix and -pleated sheets are the
secondary structure of proteins that forms because of hydrogen bonding between
carbonyl and amino groups in the peptide backbone.
30 Which structural level of proteins is most often associated with their biological function?
A The primary structure
B The secondary structure
C The tertiary structure
D The quaternary structure
Solution The solution is (C). Tertiary structure is generally stabilized by nonlocal interactions,
most commonly by the formation of a hydrophobic core, but also through salt
bridges, hydrogen bonds, disulfide bonds, and even posttranslational modifications.
Therefore, the tertiary structure is what controls the basic function of the protein.
Some proteins acquire their biological functions only in their quaternary structures,
for example, antibodies.
31 What may cause a protein to denature?
A Changes in pH
B High temperatures
C The addition of some chemicals
D All of the above
Solution The solution is (D). If the protein is subject to changes in temperature, pH, or
exposure to chemicals, the protein structure may change, losing its shape without
losing its primary sequence, in a process known as denaturation.
32 What is a protein’s chaperone?
A A chemical that assists the protein in its enzymatic functions
B A second protein that completes the quaternary structure

Advanced Placement Biology Instructor’s Solution Manual


3 | Biological Macromolecules 35

C A chemical that helps the protein fold properly


D A chemical that functions as a cofactor for the protein
Solution The solution is (C). Proteins receive assistance in the folding process from protein
helpers known as chaperones that associate with the target protein during the
folding process. They prevent aggregation of polypeptides, and then disassociate
themselves from the protein once the target protein is folded.
33 What are the building blocks of nucleic acids?
A Nitrogenous bases
B Nucleotides
C Peptides
D Sugars
Solution The solution is (B). DNA and RNA are nucleic acids made up of monomers known as
nucleotides. Each nucleotide is made up of three components: a nitrogenous base, a
pentose sugar, and a phosphate group.
34 What may a nucleotide of DNA contain?
A Ribose, uracil, and a phosphate group
B Deoxyribose, uracil, and a phosphate group
C Deoxyribose, thymine, and a phosphate group
D Ribose, thymine, and a phosphate group
Solution The solution is (C). Each nucleotide consists of a pentose sugar (deoxyribose in DNA
and ribose in RNA), a nitrogenous base (adenine, cytosine, guanine, or thymine), and
a phosphate group.
35 What is DNA’s structure described as?
A A step ladder
B A double helix
C A tertiary protein-like structure
D A barber pole
Solution The solution is (B). DNA has a double-helical structure with the two strands running
in opposite directions (antiparallel), connected by hydrogen bonds and
complementary to each other.

Advanced Placement Biology Instructor’s Solution Manual


36 3 | Biological Macromolecules

36 What is found in RNA that is NOT in DNA?


A Deoxyribose and adenine
B Fructose and thymine
C Glucose and quinine
D Ribose and uracil
Solution The solution is (D). The nucleotide molecule RNA consists of ribose sugar. Also, in
RNA, uracil replaces thymine and pairs with adenine (U-A).
37 What is the smallest type of RNA?
A mRNA
B miRNA
C rRNA
D tRNA
Solution The solution is (B). MicroRNAs (miRNAs) are the smallest RNA molecules and their
role involves the regulation of gene expression.
38 Where is the largest amount of DNA found in a eukaryotic cell?
A Attached to the inner layer of the cell membrane
B In the nucleus
C In the cytoplasm
D On ribosomes
Solution The solution is (B). DNA in the form of nucleosomes is found inside the nucleus.

CRITICAL THINKING QUESTIONS


39 The word hydrolysis is defined as the lysis of water. How does this apply to polymers?
A Polymers break apart by separating water into hydrogen and a hydroxyl group that are
added to the monomers.
B Polymers are synthesized by using the energy released by the breaking of water
molecules into hydrogen and a hydroxyl group.
C Polymers are separated into monomers, producing energy and water molecules.
D Polymers are hydrolyzed into monomers using water in a process called dehydration
synthesis.
Solution The solution is (A). In the process of hydrolysis, water is separated into a hydrogen
atom and a hydroxyl group. These are added to the separated monomers to
complete their structures.

Advanced Placement Biology Instructor’s Solution Manual


3 | Biological Macromolecules 37

40 What role do electrons play in dehydration synthesis and hydrolysis?


A Electrons are added to OH and H ions in dehydration synthesis. They are removed
from OH and H in hydrolysis.
B Electrons are transferred from OH and H ions to monomers in dehydration synthesis.
They are taken up by the H and OH ions from monomers in hydrolysis.
C Electrons are removed from OH and H in dehydration synthesis. They are added to OH
and H in hydrolysis.
D Electrons are transferred from monomers to H and OH ions in hydrolysis. They are
transferred from OH and H to monomers in dehydration synthesis.
Solution The solution is (A). Electrons are added to the hydroxyl group and the hydrogen ion
during dehydration synthesis to constitute water. During hydrolysis, they are
removed from the OH and H of water to create ions to reconstitute monomers.
41 Which bodily process would most likely be hindered by a lack of water in the body?
A Digestion
B Protein synthesis
C DNA copying
D Breathing
Solution The solution is (A). Digestion would be negatively affected by an inability to perform
hydrolysis. Digestion requires hydrolysis to break larger polymers within food
molecules into monomers.
42 Why is it impossible for humans to digest food that contains cellulose?
A There is no energy available in fiber.
B An inactive form of cellulase in the human digestive tract renders it undigested and
removes it as waste.
C The acidic environment in the human stomach makes it impossible to break the bonds
in cellulose.

D Human digestive enzymes cannot break down the -1,4 glycosidic linkage in
cellulose, which requires a special enzyme absent in humans.

Solution The solution is (D). Human digestive enzymes cannot break down the -1-4 linkage.
It requires a special enzyme, cellulase, which is secreted by bacteria and protists in
the digestive tract of herbivores.

Advanced Placement Biology Instructor’s Solution Manual


38 3 | Biological Macromolecules

43 Which statement describes some of the differences between cellulose and starch?
A Cellulose is unbranched and starch is branched. Both molecules are found in animals.
B Starch is unbranched and cellulose is branched. Both molecules are found in plants.
C Starch is branched and cellulose is unbranched. Both molecules are found in plants.
D Cellulose is branched and starch is unbranched. Both molecules are found in animals.
Solution The solution is (C). Starch contains branches and cellulose does not. Both molecules
are made by plants but neither is made by animals.
44 Which statement best describes the production of sucrose, maltose, and lactose?
A Glucose and fructose combine to form sucrose. Glucose and galactose combine to
form lactose. Two glucose monomers combine to form maltose.
B Glucose and fructose combine to form sucrose. Glucose and galactose combine to
form maltose. Two glucose combine to form lactose.
C Two glucose combine to form lactose. Glucose and galactose combine to form
sucrose. Glucose and fructose combine to form maltose.
D Two galactose combine to form sucrose. Fructose and glucose combine to form
lactose. Two glucose combine to form maltose.
Solution The solution is (A). Glucose and fructose are combined to form sucrose. Glucose and
galactose are combined to form lactose. Two glucose monomers are combined to
form maltose.
45 What are the four classes of lipids? Provide a common example for each one.
A Lipids like margarine, waxes like the coating on feathers, phospholipids like cell
membrane constituents, steroids like cholesterol
B Lipids like phosphatidylserine, waxes like phosphatidic acid, phospholipids like oleic
acid, steroids like epinephrine
C Lipids like phosphatidic acid, waxes like margarine, phospholipids like
phosphatidylcholine, steroids like testosterone
D Lipids like cholesterol, waxes like the coating on feathers, phospholipids like
phosphatidylserine, steroids like margarine
Solution The solution is (A). Margarine is a fat with higher trans fatty acid content, wax is
present as a feather-coating material, cell membranes are made of phospholipids,
and cholesterol is a steroid.
46 What are three functions that lipids serve in plants and/or animals?
A Lipids serve in the storage of energy, as a structural component of hormones, and as
signaling molecules.
B Lipids serve in the storage of energy, as carriers for the transport of proteins across
the membrane, and as signaling molecules.

Advanced Placement Biology Instructor’s Solution Manual


3 | Biological Macromolecules 39

C Lipids serve in the breakdown of stored energy molecules, as signaling molecules, and
as structural components of hormones.
D Lipids serve in the breakdown of stored energy molecules, as signaling molecules, and
as channels for protein transport.
Solution The solution is (A). Lipids serve as a source of long-term energy storage, and as a
structural component of some hormones in animals. Hormones are signaling
molecules. A class of lipids, the phospholipids, is a major constituent of cell
membranes in both plants and animals.
47 How are trans fats created? Why have they been banned from some restaurants?
A Trans fat is produced by the hydrogenation of oil that makes it more saturated and
isomerized. It increases LDL in the body.
B The dehydrogenation of oil forms trans fat, which contains single bonds in its
structure. It increases HDL in the body.
C Trans fat is produced by the dehydrogenation of oils, which makes it unsaturated. It
increases LDL in the body.
D The hydrogenation of oil makes the trans fat, which contains double bonds in its
structure. It decreases the HDL in the body.
Solution The solution is (A). Trans fats resemble saturated fats in their chemical and physical
characteristics, and they increase LDL. They are produced by bubbling hydrogen gas
through unsaturated lipids under pressure, adding hydrogen atoms to the fatty acids
in the lipids.
48 How do phospholipids contribute to cell membrane structure?
A Phospholipids orient their heads toward the polar molecules and tails in the interior of
the membrane, forming a bilayer.
B Phospholipids orient their tails toward the polar molecules of water solutions and
their heads toward in the interior of the membrane, forming a bilayer.
C Phospholipids orient their heads toward the nonpolar molecules and their tails toward
the interior of the membrane, forming a bilayer.
D Phospholipids orient their tails toward the polar molecules and their heads toward the
nonpolar side of the membrane, forming a bilayer.
Solution The solution is (A). The hydrophilic head orients toward polar molecules such as
water or the cytoplasm of the cell, whereas the hydrophobic tails of the molecules
orient toward other nonpolar molecules. This forms the middle of the membrane,
with heads on the outermost and innermost surfaces of the cell membrane.

Advanced Placement Biology Instructor’s Solution Manual


40 3 | Biological Macromolecules

49 What type of compound functions in hormone production, contributes to membrane


flexibility, and is the starting molecule for bile salts?
A All steroid molecules help in the mentioned functions.
B Cholesterol, which is a lipid and also a steroid, functions here.
C Glycogen, which is a multi-branched polysaccharide of glucose, is the compound.
D Phosphatidylcholine, which is a phospholipid with a choline head group, serves these
functions.
Solution The solution is (B). Cholesterol is a steroid and is involved in hormone production,
membrane flexibility, and production of bile salts.
50 What part of the cell membrane gives flexibility to the structure?
A Carbohydrates
B Cytoskeleton filaments
C Lipids
D Proteins
Solution The solution is (C). Lipids add flexibility to the membrane, allowing it to bend and
twist as necessary.
51 How do the differences in amino acid sequences lead to different protein functions?
A Different amino acids produce different proteins because of the secondary bonds
they form.
B Differences in amino acids lead to the recycling of proteins, which produces other
functional proteins.
C Different amino acids cause rearrangements of amino acids to produce a functional
protein.
D Differences in amino acids cause post-translational modification of the protein, which
reassembles to produce a functional protein.
Solution The solution is (A). Differences in amino acid sequences result in different
configurations of the finished protein. This allows different protein shapes to bind
with different chemicals, giving each protein its function.
52 What causes the changes in protein structure through its three or four levels of structure?

A The primary chain forms secondary helix and pleated sheets that fold onto each
other forming the tertiary structure.
B The primary structure undergoes alternative splicing to form secondary structures that
fold on other protein chains to form tertiary structures.

Advanced Placement Biology Instructor’s Solution Manual


3 | Biological Macromolecules 41

C The primary structure forms secondary helix and pleated sheets. These further
undergo phosphorylation and acetylation to form the tertiary structure.
D The primary structure undergoes alternative splicing to form a secondary structure,
and then disulfide bonds give way to tertiary structures.
Solution The solution is (A). The primary structure is based on the bonds between individual
amino acids while the secondary structure is based on the formation of alpha and
beta pleated sheets. The tertiary structure describes the folding of the secondary
structure.
53 What structural level of proteins is functional? Why?
A The secondary structure is functional as it attains its two-dimensional shape, which
has the necessary bonds.
B The tertiary structure is functional, as it possesses the geometric shape showing the
necessary loops and bends.
C The tertiary structure is functional as it has the non-covalent and covalent bonds along
with the subunits attached at the right places, which help it function properly.
D The quaternary structure is functional, as it has the essential set of subunits.
Solution The solution is (B). The folding pattern is what creates the shape of the protein,
which determines its use as well as, in the case of enzymes, what substrates it can
bind to.
54 How does a chaperone work with proteins?
A Chaperones assist proteins in folding.
B Chaperones cause the aggregation of polypeptides.
C Chaperones associate with proteins once the target protein is folded.
D Chaperones escort proteins during translation.
Solution The solution is (A). Chaperones (or chaperonins) associate with the target protein
during the folding process. They act by preventing aggregation of polypeptides that
make up the complete protein structure, and they disassociate from the protein
once the target protein is folded.
55 What is a difference between DNA and RNA?
A DNA is made from nucleotides, while RNA is not.
B DNA contains deoxyribose and thymine, while RNA contains ribose and uracil.
C DNA contains adenine, while RNA contains guanine.
D DNA is double stranded, while RNA may be double stranded in animals.

Advanced Placement Biology Instructor’s Solution Manual


42 3 | Biological Macromolecules

Solution The solution is (B). Both DNA and RNA are made of nucleotides containing a sugar
(deoxyribose in DNA, ribose in RNA), a nitrogenous base (DNA has thymine, RNA has
uracil). Both also have adenine, cytosine, guanine, and a phosphate group.
56 Which molecule carries information in a form that is inherited from one generation to
another?
A DNA
B mRNA
C Proteins
D tRNA
Solution The solution is (A). Hereditary information is stored in the sequence of nucleotides
found in DNA.
57 What are the four types and functions of RNA?
A mRNA is a single-stranded transcript of DNA. rRNA is found in ribosomes. tRNA
transfers specific amino acids to a growing protein strand. miRNA regulates the
expression of mRNA strands.
B mRNA is a single-stranded transcript of rRNA. rRNA is translated in ribosomes to make
proteins. tRNA transfers specific amino acids to a growing protein strand. miRNA
regulates the expression of the mRNA strand.
C mRNA regulates the expression of the miRNA strand. rRNA is found in ribosomes.
tRNA transfers specific amino acids to a growing protein strand. miRNA is a single-
stranded transcript of DNA.
D mRNA is a single-stranded transcript of DNA. rRNA transfers specific amino acids to a
growing protein strand. tRNA is found in ribosomes. miRNA regulates the expression
of the mRNA strand.
Solution The solution is (A). Messenger RNA (mRNA) is a single-stranded copy of the
sequencing in DNA. It leaves the nucleus and attaches to ribosomes for protein
synthesis. Ribosomal RNA (rRNA) plus protein make up the ribosomes, which are
used as points of protein synthesis. Transfer RNA (tRNA) is a single-stranded RNA
that carries or transfers specific amino acids to the growing protein chain. MicroRNA
(miRNA) is a single-stranded RNA that has been folded back on itself. This is trimmed
into a short strand of about 20 nucleotide pairs. One of the strands is degraded,
while the other binds onto protein. The miRNA-protein complex can attach to mRNA
with complementary sequences and functions to regulate the expression of that
mRNA strand.

Advanced Placement Biology Instructor’s Solution Manual


3 | Biological Macromolecules 43

TEST PREP FOR AP® COURSES


58 Urey and Miller constructed an experiment to illustrate the early atmosphere of Earth and
possible development of organic molecules in the absence of living cells. Which
assumption did Urey and Miller make regarding conditions on Earth?
A Electric sparks occurred to catalyze the reaction.
B The composition of the gases in the atmosphere
C There was sufficient oxygen for creating life.
D It produced water-soluble organic molecules.
Solution The solution is (B). They assumed the composition of gases in the atmosphere
included methane, ammonia, and hydrogen gases.
59 Urey and Miller proposed that a series of reactions occurred that ultimately resulted in
amino acid formation. What is true based on their theory?
A Hydrogen, methane, water, and ammonia combined to create amino acids.
B Hydrogen, methane, and oxygen combined to create macromolecules.
C Nitrogenous bases combined to form monomers, then RNA.
D Periodic elements combined to create molecules, then DNA.
Solution The solution is (A). Their experiment resulted in the spontaneous formation of
amino acids, which form from hydrogen, nitrogen, and other compounds.
60 How does Urey and Miller's model support the claim that simple precursors present on
early Earth could have assembled into the complex molecules necessary for life?
A The simple molecules assembled to form amino acids and nucleic acids.
B The organic molecules assembled to form large complexes, such as water and
methane.
C The inorganic molecules assembled to form amino acids and nucleic acids.
D The inorganic molecules assembled to form large complexes, such as water and
methane.
Solution The solution is (A). Miller and Urey’s experiment resulted in the production of
organic molecules, such as amino acids and nucleic acids, from the building blocks of
water, methane, ammonia, and hydrogen gas. This was done in the absence of living
organisms, but could be used for the development of them.
61 Which statement most accurately describes the importance of the condensation stage
during Urey and Miller’s experiment?
A Condensed water enabled the formation of monomers.
B Condensation and evaporation simulated lightning storms.

Advanced Placement Biology Instructor’s Solution Manual


44 3 | Biological Macromolecules

C Condensation and evaporation simulated the water cycle.


D Condensed water enabled the formation of polymers.
Solution The solution is (C). Condensation and evaporation are main components of the
water cycle.
62 According to the findings of the Urey and Miller experiment, the primitive atmosphere
consisted of water in the form of steam, methane, ammonia, and hydrogen gases. If there
was so much hydrogen gas in the early atmosphere, why is there so little now?
A Hydrogen gas is so light with a molecular weight of 1 that the excess diffused into
space over time and is now absent from the atmosphere.
B Hydrogen combined with ammonia to make ammonium.
C It was all used up in the production of organic molecules.
D The excess hydrogen gas was dissolved in the early oceans.
Solution The solution is (A). Hydrogen gas is so light (molecular weight of 1) that the excess
diffused into space over time and is now absent from the atmosphere.
63 Could the primitive atmosphere illustrated by the Urey and Miller experiment be
reproduced on today’s Earth? Why or why not?
A The primitive atmosphere cannot be created due to the oxidizing atmosphere and lack
of hydrogen.
B The primitive atmosphere can be created because the atmosphere is reducing and
Earth has sufficient hydrogen to reproduce the conditions.
C The primitive atmosphere cannot be created due to the presence of abundant water
and hydrogen in the atmosphere.
D The primitive atmosphere can be created because the atmosphere is oxidizing and has
less hydrogen.
Solution The solution is (A). The atmosphere reproduced in the Urey and Miller experiment
could not exist on present-day Earth. Most of the hydrogen gas has been used or
diffused into space. The presence of large amounts of oxygen has created an
oxidizing atmosphere that would break down any organic molecules that might be
produced.
64 What is structurally different between starch and cellulose that gives them different
physical properties?

A Cellulose is formed by 1-4 glycosidic linkages and crosslinks, making it rigid, while
starch has 1-4 and 1-6 glycosidic linkages without the tight crosslinks of
cellulose.

B Cellulose has rigid 1-4 glycosidic linkages, while starch has less rigid 1-4
glycosidic linkages.

Advanced Placement Biology Instructor’s Solution Manual


3 | Biological Macromolecules 45

C Cellulose has amylose and amylopectin, making it more rigid than starch.
D Starch has amylose and amylopectin that make it more rigid than cellulose.
Solution The solution is (B). Starch is made up of glucose monomers that are joined by 1-4
or 1-6 glycosidic bonds. Cellulose is made up of glucose monomers that are
linked by 1-4 glycosidic bonds.
65 Complex polymers are built from combinations of smaller monomers. What type of
reaction is illustrated in the figure, and what is the product of the reaction?

A A synthesis reaction producing glucose


B A hydrolysis reaction producing fructose
C A condensation reaction producing lactose
D A dehydration reaction producing water
Solution The solution is (D). Two six-carbon rings with hydroxyl groups are shown. The
hydroxyl group on one is highlighted red, and the hydrogen of a hydroxyl group of
the other is highlighted red. An arrow points to two five carbon rings connected by
an oxygen. The molecule is sucrose and is formed by the condensation of glucose
and fructose.

Advanced Placement Biology Instructor’s Solution Manual


46 3 | Biological Macromolecules

66 The fatty acids of triglycerides are classified as saturated, unsaturated, or trans fat. What
about the structure of these compounds gives them their physical characteristics?
A Saturated fats and trans fats contain the greatest possible number of hydrogen atoms,
while unsaturated fats do not.
B Saturated and unsaturated fats have stable configurations, while trans fats are
transient.
C Unsaturated fats and trans fats have some double-bonded carbon atoms, while
saturated fats do not.
D Unsaturated and trans fats are the same; fatty acids are only found on opposite sides
of a trans fat.
Solution The solution is (C). Unsaturated fats and trans fats have some double-bonded carbon
atoms, while saturated fats do not.
67 Carbohydrates serve various functions in different animals. Arthropods, like insects,
crustaceans, and others, have an outer layer, called the exoskeleton, which protects their
internal body parts. This exoskeleton is made mostly of chitin. Chitin is also a major
component of the cell walls of fungi, the kingdom that includes molds and mushrooms.
Chitin is a polysaccharide.
What is the major difference between chitin and other types of polysaccharides?

A Chitin is a nitrogen-containing polysaccharide, with repeating units of N-acetyl- -D-


glucosamine, a modified sugar.
B Chitin is similar to amylase, but with sulfur linkages between the monomers.
C Chitin is similar to inulin, a polysaccharide with fructose plus additional glucose
monomers.
D Chitin contains phosphate groups that give it a stiffness not found in other
polysaccharides.
Solution The solution is (A). Chitin is a nitrogen-containing polysaccharide, with repeating
units of N-acetyl- -D-glucosamine, a modified sugar.
68 Which categories of amino acids would you expect to find on the surface of a soluble
protein, and which would you expect to find in the interior?
A Nonpolar and charged amino acids will be present on the surface and polar in the
interior of the membrane, whereas nonpolar will be found in the membrane-
embedded proteins.
B Nonpolar and uncharged proteins will be found on the surface with nonpolar in the
interior, while only nonpolar will be found in the embedded proteins.

Advanced Placement Biology Instructor’s Solution Manual


3 | Biological Macromolecules 47

C Polar and charged amino acids will be found on the surface, whereas nonpolar will be
found in the interior.
D Polar and charged amino acids will be found on the surface of a membrane protein,
whereas nonpolar will be found in the interior. The membrane protein will be polar
and hydrophobic.
Solution The solution is (C). Polar amino acids—such as proline, asparagine, and glutamine—
would be found at the surface of a soluble protein, while nonpolar amino acids—
such as leucine, methionine, and glycine—would be oriented toward the interior.
69 You have been identifying the sequence of a segment of a protein. The sequence to date
is: leucine-methionine-tyrosine-alanine-glutamine-lysine-glutamate. You insert arginine
between the leucine and methionine.
What effect would this insertion have on the segment?
A Arginine is a negatively charged amino acid and could attach to the glutamate at the
end of the segment.
B Inserting arginine places a positively charged amino acid in a portion that is nonpolar,
creating the possibility of a hydrogen bond in this area.
C There would be no effect other than an additional amino acid.
D The arginine could attach to the lysine and bend the protein chain at this point.
Solution The solution is (B). Inserting arginine places a positively charged amino acid in a
portion that is nonpolar, creating the possibility of a hydrogen bond in this area.
70 What happens if even one amino acid is substituted for another in a polypeptide? Provide
a specific example.
A The change will definitely not be sufficient to have any effect on the function and
structure of the protein.
B The amino acid may not show any significant effect on the protein structure and
function, or it may have a significant effect, as in the case of hemoglobin in individuals
with sickle cell trait.
C These changes would increase the possibility of having extra bends and loops in the
proteins, as seen in Leber congenital disease.
D These changes would modify the structures of proteins, making them nonfunctional.
Solution The solution is (B). There are two possibilities when one amino acid is substituted
for another: there may be no effect on the protein if the amino acid’s position was
not critical to the tertiary structure of the protein, or it may cause an extra bond
to be made or not made that would significantly alter the functional structure. A
classic example of a single amino acid change that significantly modifies the function
of a protein is in the case of sickle cell hemoglobin in which a valine replaces a
glutamic acid.

Advanced Placement Biology Instructor’s Solution Manual


48 3 | Biological Macromolecules

71 HIV is an RNA virus that affects CD4 cells, also known as T helper cells, in the human body.
Which mechanism is most likely responsible for the fast rate at which HIV can spread?
A Recombination
B Mutation
C Reassortment
D Replication errors
Solution The solution is (B). The high rate of mutations allows HIV to develop resistance to
antiviral drugs. Furthermore, as the virus mutates, it is not recognized any longer by
existing antibodies and is not tagged for destruction.

72 For many years, scientist believed that proteins were the source of heritable information.
There are many thousands of different proteins in a cell, and they mediate the cell’s
metabolism, producing the traits and characteristics of a species. Researchers working
with DNA viruses proved that it is DNA that stores and passes on genes. They worked with
viruses that have an outer coat of protein and a DNA strand inside.
How did scientists prove that it was DNA, not protein, that is the primary source of
heritable information?
A The DNA and protein of the virus were tagged with different isotopes and exposed to
the host cell whereas only the DNA was transferred to the host.
B The DNA was tagged with an isotope, which was retained in the virus, proving it to be
the genetic material.
C The viral protein was tagged with an isotope, and the host cell was infected by it. This
protein was transferred to the host.
D The viral DNA, when sequenced, was found to be present in the host cell, proving it to
be the hereditary material instead of protein.
Solution The solution is (A). Researchers tagged the DNA and protein coat of the virus with
different isotopes. Then, they exposed host cells to the virus and determined the

Advanced Placement Biology Instructor’s Solution Manual


3 | Biological Macromolecules 49

tagged proteins did not enter the cell, but the tagged viral DNA did. The virus was
replicated in the host cells, showing that the DNA was responsible for the passing of
genetic information.
73 The genetic code is based on each amino acid being coded for by a distinctive series of
three nucleic acid bases called a codon. The following is a short segment of DNA, using the
slash symbol ( / ) to separate the codons for easy viewing:
ATC/GTT/GAA/CTG/TAG/GAT/AAA
A change has occurred in the segment resulting in the following:
ATC/GTT/GTA/CTG/TAG/GAT/AAA
What kind of change has occurred?
A A substitution of T for A, resulting in a coding change for the third codon
B An addition of C for G, lengthening the strand and changing every codon past the
addition
C A deletion of an A, resulting in a shortening and changing every codon past the
deletion
D No change has occurred; the same base was replaced with the same base.
Solution The solution is (A). In the third codon, A has been substituted by T, which may or
may not change the amino acid.
74 A change in DNA on a chromosome affects all proteins made from that gene for the life of
the cell. A change in the RNA involved in protein production is short lived.
What is the difference between the effects of the changes in the two types of nucleic
acids?
A DNA is the genetic material that is passed from parent cells to daughter cells and to
future generations.
B DNA would not affect the individuals as the proteins made are finally altered and
modified. RNA would cause harm to the person as the RNA encoded by the DNA and is
not altered.
C DNA is the genetic material and is transferred from one generation to another, making
use of repair mechanisms for every mutation. The RNA does not use a repair
mechanism.
D DNA, when mutated, makes use of the repair mechanisms and can be repaired,
whereas RNA is not repaired and is transferred in generations.
Solution The solution is (A). The DNA in the chromosome will last the life of the cell, and
possibly beyond, as shown in the DNA harvested from archeological research. Any
protein made from the information on the chromosome will reflect the change in
DNA. The effect is permanent and will possibly cause the expression of a genetic
disease. If the change happened in a gamete, the change may be transmitted to an

Advanced Placement Biology Instructor’s Solution Manual


50 3 | Biological Macromolecules

offspring. Any change in the type of RNA lasts only as long as that strand of RNA is
intact. All RNA is eventually degraded and replaced with new strands, depending on
the needs of the cell. Any change in the RNA will only have an effect until it
disappears with the degradation of that RNA. RNA changes alone are not
permanent.

SCIENCE PRACTICE CHALLENGE QUESTIONS


3.1 Synthesis of Biological Macromolecules
75 The capture of radiant energy through the conversion of carbon dioxide and water into
carbohydrates is the engine that drives life on Earth. Ribose, C5H10O5, and hexose, C6H12O6,
form stable 5- and 6-carbon rings.

The numbering of the carbons on these rings is important in organizing the description
of the role these molecules play in biological energy transfer and information storage
and retrieval. Glycolysis is a sequence of chemical reactions that converts glucose to two
3-carbon compounds called pyruvic acid.

A. Create visual representations to show how when bonds in the glucose molecules are
broken between carbon number 1 and the oxygen atom and between carbons 3 and 4,
two molecules of pyruvic acid are produced.
Several enzymes in the cell are involved in converting glucose to pyruvic acid. These
enzymes are proteins whose amino acid sequences provide these functions. This protein
structure is information that was inherited from the cell’s parent and is stored in
deoxyribonucleic acid (DNA). The “deoxyribo” component of that name is a shorthand for
2-deoxyribose.
B. Create a visual representation of 2-deoxyribose, 5-phosphate by replacing the OH at
carbon 2 with a hydrogen atom and replacing the OH at carbon 5 with a hydrogen

phosphate ion, whose structure is shown in problem AP3.2. Use your


representation to show that both phosphorylation (the addition of a phosphate ion) at

Advanced Placement Biology Instructor’s Solution Manual


3 | Biological Macromolecules 51

carbon 5 and removal of the hydroxide at carbon 2 produce water molecules in an


aqueous solution where hydrogen ions are abundant.
DNA is a polymer formed from a chain with repeated 2-deoxyribose, 5-phosphate
molecules.
C. Create a visual representation of three 2-deoxyribose, 5-phosphate molecules forming
a chain in which an oxygen atom in the phosphate that is attached to the 5-carbon
replaces the OH on the 3-carbon of the next ribose sugar.
Solution Sample answer:
A. Drawing should show that by breaking the molecule of glucose between carbon 1
and the oxygen atom and between carbons 3 and 4 two molecules of 3 carbon
atoms are produced: one molecule will contain carbons 1, 2, and 3. The second
molecule will contain carbons 4, 5, and 6. Hydrogen atoms are lost in oxidation
reactions.
B. A molecule of deoxyribose phosphate is shown.

When this molecule forms, an OH group is lost from position two and position five.
Each OH group can react with a hydrogen ion to form water.
C.

Advanced Placement Biology Instructor’s Solution Manual


52 3 | Biological Macromolecules

3.2 Carbohydrates
76 Cells are bounded by membranes composed of phospholipids. A phospholipid consists of
a pair of fatty acids that may or may not have carbon-carbon double bonds, fused at the
carboxylic acid with a three-carbon glycerol that is terminated by a phosphate, as shown
in the figure below. Most cell membranes comprise two phospholipid layers with the
hydrophilic phosphate ends of each molecule in the outer and inner surfaces. The
hydrophobic chains of carbon atoms extend into the space between these two surfaces.

The exchange of matter between the interior of the cell and the environment is mediated
by this membrane with selective permeability.
A. Pose questions that identify
 The important characteristics of this lipid bilayer structure
 The molecules that must be acquired from the environment and eliminated from

the cell

Advanced Placement Biology Instructor’s Solution Manual


3 | Biological Macromolecules 53

 Relationships between the structures of these molecules and the structure of the
bilayer
Because the plasma cell membrane has both hydrophilic and hydrophobic properties, few
types of molecules possess structures that allow them to pass between the interior of the
cell and the environment through passive diffusion. The fluidity of the membrane affects
passive transport, and the incorporation of other molecules in the membrane, in
particular cholesterols, has a strong effect on its fluidity. Fluidity is also affected by
temperature.
Measurements of the speed of movement of oxygen molecules, O2, through three types
of membranes were made (Widomska et al., Biochimica et Biophysica Acta, 1,768, 2007)
and compared to the speed of movement of O2 through water. These measurements were
carried out at four different temperatures. One type of membrane was obtained from the
cells in the eyeball of a calf (lens lipid). Synthetic membranes composed of palmitic acid
with cholesterol (POPC/CHOL) and without cholesterol (POPC) were also used. The results
from these experiments are shown in the table.
Material 15 °C 25 °C 35 °C 45 °C
Lens lipids 15 cm/s 30 cm/ 65 cm/s 110 cm/s
POPC/CHOL 15 cm/s 30 cm/s 65 cm/s 110 cm/s
POPC 55 cm/s 100 cm/s 155 cm/s 280 cm/s
Water 45 cm/s 55 cm/s 65 cm/s 75 cm/s
B. Represent these data graphically. The axes should be labeled, and different symbols
should be used to plot data for each material.
C. Analyze the data by comparing transport of oxygen through the biological membrane,
water, and the synthetic membranes. Consider both membrane composition and
temperature in your analysis.
The plasma membrane separates the interior and the exterior of the cell. A potential to do
work is established by defining regions inside and outside the cell with different
concentrations of key molecules and net charges. In addition to the membrane defining
the cell boundary, eukaryotic cells have internal membranes.
D. Explain how internal membranes significantly increase the functional capacity of the
cells of eukaryotes relative to those of prokaryotes.
Solution Sample answer:
A. Phospholipids orient themselves in solution forming a bilayer that allows stable
interaction with the polar inside and outside of the cell while separating the two
environments.
Nutrients must be acquired from the environment and waste must be eliminated.
Ions must move across the membrane to maintain homeostasis. Certain gases, such
as oxygen and carbon dioxide, must also enter or leave the cell. Gases and can cross

Advanced Placement Biology Instructor’s Solution Manual


54 3 | Biological Macromolecules

freely the membrane. The membrane is also permeable to small hydrophobic


molecules, but is impermeable to ions and polar molecules. However, small polar
molecules can enter or leave the cell through transporters. Ions can enter or leave
through channels. Large polar molecules can be packaged into lipid vesicles that fuse
with the membrane, allowing the molecules to move across.
B. By convention, the independent variable is plotted on the x-axis and the
dependent is plotted on the y-axis. In this graph, temperature is the independent
variable and rate of diffusion is the dependent variable. Each medium should be
represented by a separate line. The graph should have a title, the axes should be
labeled and show units. A legend showing the symbol for each medium should be
included.

C. The speeds and slopes for the plots representing the lens membrane and
POPC/CHOL are similar. The speed of diffusion of oxygen in water is the least
affected by change in temperature as should be expected. Both the speed of
diffusion in and the slope are greater in POPC.
D. The increase in membrane surface increases the potential to do work by allowing
different types of concentration gradients to be established inside the same cell.
3.3 Lipids
77 Proteins are polymers whose subcomponents are amino acids connected by peptide
bonds. The carboxylic acid carbon, , of one amino acid can form a bond with the

amine, , of another amino acid. In the formation of this peptide bond, the amine

replaces the OH to form . The other product of this reaction is water, .

Advanced Placement Biology Instructor’s Solution Manual


3 | Biological Macromolecules 55

Amino acids can be synthesized in the laboratory from simpler molecules of ammonia
(NH3), water (H2O), methane (CH4), and hydrogen (H2) if energy is provided by processes
that simulate lightning strikes or volcanic eruptions (Miller, Science, 117, 1953; Johnson et
al., Science, 322, 2008).
A. The synthesis of amino acids in solutions under laboratory conditions consistent with
early Earth was a step toward an explanation of how life began. Pose a question that
should have been asked but was not until 2014 (Parker et al., Angewandte Chemie, 53,
2014), when these solutions that had been stored in a refrigerator were analyzed.
The diversity and complexity of life begins in the variety of sequences of the 20 common
amino acids.
B. Apply mathematical reasoning to explain the source of biocomplexity by calculating the
possible variations in a polymer composed of just three amino acids.
Polarity in a bond between atoms occurs when electrons are distributed unequally.
Polarity in a molecule also is caused by charge asymmetry. Life on Earth has evolved
within a framework of water, H2O, one of the most polar molecules. The polarities of the
amino acids that compose a protein determine the properties of the polymer.
The electric polarity of an amino acid in an aqueous solution depends on the pH of the
solution. Here are three forms of the general structure of an amino acid.

C. Qualitatively predict the relationship between solution pH and the form of the amino
acid for three solutions of pH: pH < 7, pH = 7, and pH > 7.

Advanced Placement Biology Instructor’s Solution Manual


56 3 | Biological Macromolecules

The properties of proteins are determined by interactions among the amino acids in the
peptide-bonded chain. The protein subcomponents, especially amino R (variable) groups,
can interact with very strong charge-charge forces, with attractive forces between groups
of atoms with opposite polarities and with repulsive forces between groups of atoms
with the same or no polarity. Attractive polar forces often arise between molecules
through interactions between oxygen and hydrogen atoms or between nitrogen and
hydrogen atoms.
D. Consider particular orientations of pairs of three different amino acids. Predict the
relative strength of attractive interaction of all pairs; rank them and provide your
reasoning.
In an amino acid, the atoms attached to the -carbon are called the R group.

Interactions between R groups of a polypeptide give three-dimensional structure to the


one-dimensional, linear sequence of amino acids in a polypeptide.
E. Construct an explanation for the effect of R-group interactions on the properties of a
polymer with drawings showing molecular orientations with stronger and weaker polar
forces between R groups on asparagine and threonine and between asparagine and
alanine.

Advanced Placement Biology Instructor’s Solution Manual


3 | Biological Macromolecules 57

Solution Sample answer:


A. Can amino acids form in the absence of volcanic eruptions and lightning (rare
events) if common inorganic molecules can act as catalysts and facilitate synthesis of
amino acids from ammonia, methane, water, and hydrogen?
B. The number of possible variations if of the 20 possible amino acids form a peptide
is because each amino acid added independently from the pool
of 20 amino acids.
C. At acidic pH when pH < 7, excess H+ ions will bind to amino acids. The prevalent
form will be the structure on the right with an ionized carboxyl group and NH 2 bound
to the alpha carbon.
At neutral pH, pH=7, the center structure which is electrically neutral is prevalent.
At basic pH, pH > 7, the structure on the left, with a positively charged amine group

and a carboxyl group, is prevalent.


D. From the strongest to the weakest interactions:
Threonine has a carboxyl group facing an amine on asparagine. Positive and negative
charges form an ionic bond.
Serine and threonine will attract because of hydrogen bonding between the
hydrogen in the hydroxyl group of threonine and the oxygen in the carbonyl group
of serine.
Last the serine and asparagine will attract each other as they are both polar
molecules.
E. The amide R group of asparagine will interact more strongly with the hydroxyl of
threonine because both R groups are polar.
Alanine which has a hydrophobic R group–CH2 will not interact strongly with either
asparagine or alanine side chains.
The asparagine amide group will form hydrogen bonding with the carboxyl group on
the alpha carbon of either threonine or asparagine.
3.4 Proteins
78 The nucleobase part of deoxyribonucleic acid encodes information in each component in
the sequence making up the polymer. There are five nucleobases that are commonly
represented by only a single letter: A (adenine), C (cytosine), G (guanine), T (thymine), and

Advanced Placement Biology Instructor’s Solution Manual


58 3 | Biological Macromolecules

U (uracil). These molecules form a bond with the 1-carbon of deoxyribose. In this
problem, we need to look at the molecules in slightly more detail so that you can develop
the ability to explain why DNA, and sometimes RNA, is the primary source of heritable
information.
Edwin Chargaff and his team isolated nucleobases from salmon sperm and determined
the fraction of each (Chargaff et al., Journal of Biological Chemistry, 192, 1951).
Experiments in which the fraction of all four nucleobases was determined are shown. Also
shown are averages of two standard deviations and the sum of total fractions for each
experiment. Precision is calculated with each average.
Shown below are the chemical structures of these four nucleobases. In these structures,
the nitrogen that attaches to the 2-deoxyribose, 5-phosphate polymer is indicated as N*.
The partial charges of particular atoms are indicated with δ+ and δ−.

A. Analyze Chargaff’s data in terms of the partial charges on these molecules to show
how molecular interactions affect the function of these molecules in the storage and
retrieval of biological information.
Experiment Adenine Guanine Cytosine Thymine Total
5 0.28 0.20 0.21 0.27 0.96
6 0.30 0.22 0.20 0.29 1.01
7 0.27 0.18 0.19 0.25 0.89
8 0.28 0.21 0.20 0.27 0.94
11 0.29 0.18 0.20 0.27 0.94
12 0.28 0.21 0.19 0.26 0.94
13 0.30 0.21 0.20 0.30 1.01
Average 0.29 ± 0.02 0.20 ± 0.03 0.20 ± 0.01 0.27 ± 0.02 0.96 ± 0.08

Advanced Placement Biology Instructor’s Solution Manual


3 | Biological Macromolecules 59

The interactions between nucleobase molecules are strong enough to produce the
association of pairs observed in Chargaff’s data. However, these pairs are bonded by
much weaker hydrogen bonds, which are chemical bonds within the molecules.
Demonstrating understanding of the replication of DNA requires the ability to explain how
the two polymer strands of the double helix interact and grow. To retrieve information
from DNA, the strands must be separated. The proteins that perform that task interact
with the polymer without forming new chemical bonds. In their paper (Watson and Crick,
Nature, 3, 1953) announcing the structure of the polymer considered in this problem,
Watson and Crick stated, “It has not escaped our notice that the specific pairing we have
postulated immediately suggests a possible copying mechanism for the genetic material.”
Eschenmoser and Lowenthal (Chemical Society Reviews, 21, 1992) asked why the 5-carbon
sugar ribose is used in DNA when the 6-carbon sugar glucose is so common in biological
systems. To answer the question, they synthesized polymeric chains of artificial DNA using
glucose. They discovered that the strength of the interaction between pairs of
nucleobases increased in the DNA with glucose. Paired strands of hexose-based polymers
were more stable.
The AP Biology Curriculum Framework (College Board, 2012) states, “The double-stranded
structure of DNA provides a simple and elegant solution for the transmission of heritable
information to the next generation; by using each strand as a template, existing
information can be preserved and duplicated with high fidelity within the replication
process. However, the process of replication is imperfect.”
B. Based on the findings of Eschenmoser and Lowenthal, why didn’t DNA evolve to use
glucose rather than hexose? What does this have to do with the idea that “replication is
imperfect” in DNA? Thoroughly explain your answers.
Solution Sample answer:
A. Adenine and thymine are present in a similar percentage. Cytosine and guanine
are also present in a similar percentage. The results are reproducible within small
error margins. The molecular interactions between the single two pairs are stabilized
by opposite partial charges.
B. The interactions between the nitrogenous bases should be weak enough for the
two strands of DNA to separate during replication and transcription and specific
enough that adenine pairs only with thymine and cytosine with guanine. More
stable interactions would require too much energy to open the double helix. Weaker
interactions also mean that mismatches between nitrogenous bases are possible,
making replication an imperfect process that may lead to mutations if errors are not
repaired.

Advanced Placement Biology Instructor’s Solution Manual


60 4 | Cell Structure

4 | CELL STRUCTURE
REVIEW QUESTIONS
1 When viewing a specimen through a light microscope, what is a method that scientists use
to make it easier to see individual components of cells?
A A beam of electrons
B High temperatures
C Radioactive isotopes
D Special stains
Solution The solution is (D). Special stains are used to enhance visualization of the cell or
certain cellular components under a microscope. Cells may also be stained to
highlight metabolic processes or to differentiate between live and dead cells in
a sample.
2 What is the basic unit of life?
A Cell
B Organism
C Organ
D Tissue
Solution The solution is (A). The cell is the basic unit of life. All organisms consist of one or
more cells.
3 Which statement is part of the cell theory?
A All living organisms are made of cells.
B All cells contain DNA that they pass on to daughter cells.
C All cells depend on their surroundings to provide energy.
D All cells have a nucleus.
Solution The solution is (A). The cell theory given by Schleiden and Schwann states the cell is
the basic unit of life and all the living cells are composed of cells. The third tenet of
the cell theory is that every cell originates from another cell. There is no
spontaneous generation.
4 What could most effectively be visualized with a scanning electron microscope?
A Cells swimming in a drop of pond water
B Details of structures inside cells

Advanced Placement Biology Instructor’s Solution Manual


4 | Cell Structure 61

C A three-dimensional view of the surface of a membrane


D The movement of molecules inside the cell
Solution The solution is (C). The SEM uses the electron beam to view the surface of a dried
and fixed specimen. The molecules in the specimen get excited and release
secondary electrons, which are captured by a detector.
5 Who was the first to clearly identify and name individual cells?
A Anton van Leeuwenhoek
B Matthias Schleiden
C Robert Hooke
D Theodor Schwann
Solution The solution is (C). Robert Hooke coined the term cell when he was viewing cork
tissue through a lens and saw some box-like structures.
6 Which observation contributed to the cell theory?
A Animal and plant cells have nuclei and organelles.
B Nonliving material cannot give rise to living organisms.
C Prokaryotic and eukaryotic cells are surrounded by a plasma membrane.
D Viruses replicate.
Solution The solution is (B). The theory states cells reproduce and give rise to other cells,
which implies nonliving materials cannot give rise to living organisms; that means
that there is no spontaneous generation.
7 To obtain some materials and remove waste, what process is used by prokaryotes?
A Cell division
B Diffusion
C Flagellar motion
D Ribosomes
Solution The solution is (B). The changes in the concentration help the movement of the
particles and waste across the plasma membrane. This process is known as diffusion,
where molecules move from high to low concentration.
8 When bacteria lack fimbriae, what are they less likely to do?
A Adhere to cell surfaces
B Retain the ability to divide
C Swim through bodily fluids
D Synthesize proteins

Advanced Placement Biology Instructor’s Solution Manual


62 4 | Cell Structure

Solution The solution is (A). Fimbriae, the “attachment pili,” are used by bacteria to adhere to
one another or to surfaces. They can contribute to motility by causing twitching.
9 What is a difference between prokaryotic and eukaryotic cells?
A Both cells have a nucleus, but prokaryotic cells lack cytoplasm.
B Both cells have cytoplasm, but prokaryotic cells lack a nucleus.
C Both cells have DNA, but prokaryotic cells lack a cell membrane.
D Both cells have a cell membrane, but prokaryotic cells lack DNA.
Solution The solution is (B). Prokaryotic cells lack a well-defined nucleus where DNA is
surrounded by a nuclear membrane. Instead, DNA is folded into a compact structure
called the nucleoid.
10 Eukaryotic cells contain complex organelles that carry out their chemical reactions.
Prokaryotes lack many of these complex organelles, although they have a variety of
unique structures of their own. However, most prokaryotic cells can exchange nutrients
with the outside environment faster than most eukaryotic cells. Why is this so?
A Most prokaryotic cells are smaller and have a higher surface-to-volume ratio than
eukaryotic cells.
B Most prokaryotic cells are larger and have a higher surface-to-volume ratio than
eukaryotic cells.
C Most prokaryotic cells are smaller and have a lower surface-to-volume ratio than
eukaryotic cells.
D Prokaryotic cells are larger and have a lower surface-to-volume ratio than
eukaryotic cells.
Solution The solution is (A). Due to their small size (0.1 to 5 µm in diameter), ions and organic
molecules that enter prokaryotic cells can quickly diffuse to other parts of the cell.
Likewise, waste products produced within a prokaryotic cell can also quickly diffuse
out. By contrast, eukaryotic cells have developed different structural adaptations to
enhance intracellular transport.
11 What is surrounded by one phospholipid bilayer?
A Lysosomes
B Ribosomes
C Nucleolus
D Nucleus
Solution The solution is (D). The nucleus is surrounded by a single phospholipid bilayer, the
nuclear membrane. Protein-lined channels called nuclear pores allow traffic of
molecules such as RNA through the nuclear membrane.

Advanced Placement Biology Instructor’s Solution Manual


4 | Cell Structure 63

12 Peroxisomes got their name because hydrogen peroxide is —


A a cofactor for the organelles’ enzymes
B incorporated into their membranes
C produced during their oxidation reactions
D used in their detoxification reactions
Solution The solution is (C). Certain enzymes found in peroxisomes use molecular oxygen to
remove hydrogen from specific organic substrates, producing hydrogen peroxide.
Because of the production of hydrogen peroxide, the organelle has been named
peroxisome.
13 In plant cells, what carries out the function of the lysosomes?
A Nuclei
B Peroxisomes
C Ribosomes
D Vacuole
Solution The solution is (D). The vacuole plays a role similar to lysosomes and fulfills many
functions. It contains digestive enzymes, isolates harmful materials, stores waste
products and secondary compounds such as pigments in petals, latex, and alkaloids
that deter predators. It helps maintain the pressure within a cell by changing its
content of water, contributes to the balance of pH, exports products, and stores
proteins for seed germination.
14 What is found in both eukaryotic and prokaryotic cells?
A Mitochondrion
B Nucleus
C Ribosomes
D Centrosomes
Solution The solution is (C). Prokaryotes and eukaryotes both require proteins for their
survival; thus, both contain ribosomes, the protein-synthesizing machinery found in
both prokaryotic and eukaryotic cells.
15 Which structure is NOT found in prokaryotic cells?
A Plasma membrane
B Chloroplast
C Nucleoid
D Ribosome

Advanced Placement Biology Instructor’s Solution Manual


64 4 | Cell Structure

Solution The solution is (B). Chloroplasts and other organelles are found only in the
eukaryotic cells. Chloroplasts and mitochondria originated in engulfed bacteria that
were not digested by the host; they lost their autonomy and became symbionts.
16 Where would you find DNA, the genetic material, in an animal cell?
A In the centriole
B Only in the mitochondria
C In the mitochondria and the nucleus
Solution The solution is (C). DNA is present in the nucleus as well as mitochondrion.
Mitochondria contain their own DNA and ribosomes. Most DNA is located in the
nucleus.
17 What is most likely to have the greatest concentration of smooth endoplasmic
reticulum (SER)?
A A cell that secretes enzymes
B A cell that destroys pathogens
C A cell that makes steroid hormones
D A cell that engages in photosynthesis
Solution The solution is (C). Steroid-secreting cells are characterized by abundant SER. For
example, cells in the gonads and the adrenal gland have an extensive network of SER
because they synthesize cholesterol as a precursor for steroid hormones or take up
this substrate from plasma lipoproteins. Many of the enzymes for sterol and steroid
synthesis are localized in the SER.
18 Which sequence correctly lists in order the steps involved in the incorporation of a protein
within a cell membrane?
A Synthesis of the protein on the ribosome; modification in the Golgi apparatus;
packaging in the endoplasmic reticulum; modification in the vesicle
B Synthesis of the protein on the lysosome; modification in the Golgi; packaging in the
vesicle; distribution in the endoplasmic reticulum
C Synthesis of the protein on the ribosome; modification in the endoplasmic reticulum;
tagging in the Golgi; distribution via the vesicle
D Synthesis of the protein on the lysosome; packaging in the vesicle; distribution via the
Golgi; modification in the endoplasmic reticulum
Solution The solution is (C). The protein is synthesized by ribosomes and modified by the
endoplasmic reticulum. The sorting, tagging, packaging, and distribution of lipids and
proteins takes place in the Golgi apparatus, which is then distributed by vesicles.

Advanced Placement Biology Instructor’s Solution Manual


4 | Cell Structure 65

19 What is NOT a component of the endomembrane system?


A Endoplasmic reticulum
B Golgi apparatus
C Lysosome
D Mitochondrion
Solution The solution is (D). The endomembrane system does not include the membranes of
mitochondria or chloroplasts because these organelles have separate DNA. These
were actually engulfed bacteria that evolved into symbionts.
20 What has the ability to disassemble and reform quickly?
A Intermediate filaments and microtubules
B Microfilaments and intermediate filaments
C Microfilaments and microtubules
D Only intermediate filaments
Solution The solution is (C). Microfilaments, such as actin, and microtubules found in
flagellum have the capacity to polarize and depolarize quickly to perform their
functions.
21 What does NOT play a role in intracellular movement?
A Intermediate filaments and microtubules
B Microfilaments and intermediate filaments
C Microfilaments and microtubules
D Only intermediate filaments
Solution The solution is (D). Recall that intermediate filaments provide the cell its shape and
help in fastening organelles in place.
22 Which components of the cytoskeleton are responsible for the contraction of muscles?
A Intermediate filaments
B Microfilaments
C Microtubules
Solution The solution is (B). Microfilaments are made up of actin filaments, which serve as a
track for movement of a motor protein called myosin. When actin and myosin slide
past each other, the muscles contract.

Advanced Placement Biology Instructor’s Solution Manual


66 4 | Cell Structure

23 What type of junctions prevents the movement of chemicals between two adjacent
animal cells?
A Desmosomes
B Gap junctions
C Plasmodesmata
D Tight junctions
Solution The solution is (D). Tight junctions composed of occludins and claudins are essential
for forming an impermeable barrier to fluids. They are typically found in cells of the
epithelial tissues that line exposed internal and external surfaces in animal bodies.
For example, cells in the intestinal mucosa are joined by tight junctions to prevent
leakage of intestinal content directly into the underlying tissues.
24 Gap junctions are formed by —
A gaps in the cell wall of plants
B protein complexes that form channels between cells
C tight, rivet-like regions in the membranes of adjacent cells
D a tight knitting of membranes
Solution The solution is (B). Gap junctions are made of protein complexes that make way for
the movement of the substances across the cells. For example, cardiac muscle cells
are connected by gap junctions that allow rapid transfer of the electric signal for
synchronized muscle contraction.
25 Some animal cells produce an extensive extracellular matrix. You would expect their
ribosomes to synthesize large amounts of which proteins?
A Actin
B Collagen
C Myosin
D Tubulin
Solution The solution is (B). Collagen is interwoven with proteoglycans, and collectively these
molecules form the extracellular matrix. Actin, myosin, and tubulin are found inside
the cells and are part of the cytoskeleton.
26 Which molecule is typically found in the extracellular matrix?
A Nucleic acids such as DNA
B Peptidoglycans
C Cellulose
D Proteoglycans

Advanced Placement Biology Instructor’s Solution Manual


4 | Cell Structure 67

Solution The solution is (D). Proteoglycans are a major component of the extracellular matrix.
Nucleic acids are found inside the cell, and cellulose forms the cell wall of plant cells.

CRITICAL THINKING QUESTIONS


27 Which element of the cell theory has practical applications in health care because it
promotes the use of sterilization and disinfection?
A All cells come from preexisting cells.
B All living organisms are composed of one or more cells.
C A cell is the basic unit of life.
D A nucleus and organelles are found in prokaryotic cells.
Solution The solution is (A). All cells come from preexisting cells. If all microorganisms are
removed or killed, then the environment, medical devices, or other items are sterile
or, at least, have fewer potential pathogens.
28 What are the advantages and disadvantages of light microscopes? What are the
advantages and disadvantages of electron microscopes?
A Advantage: In light microscopes, the light beam does not kill the cell. Electron
microscopes are helpful in viewing intricate details of a specimen and have high
resolution. Disadvantage: Light microscopes have low resolving power. Electron
microscopes are costly and require killing the specimen.
B Advantage: Light microscopes have high resolution. Electron microscopes are helpful
in viewing surface details of a specimen. Disadvantage: Light microscopes kill the cell.
Electron microscopes are costly and low resolution.
C Advantage: Light microscopes have high resolution. Electron microscopes are helpful
in viewing surface details of a specimen. Disadvantage: Light microscopes can be used
only in the presence of light and are costly. Electron microscopes use a short
wavelength of electrons and hence have lower magnification.
D Advantage: Light microscopes have high magnification. Electron microscopes are
helpful in viewing surface details of a specimen. Disadvantage: Light microscopes can
be used only in the presence of light and have lower resolution. Electron microscopes
can be used only for viewing ultra-thin specimens.
Solution The solution is (A). The advantages of light microscopes are that they are easily
obtained, and the light beam does not kill the cells. However, typical light
microscopes are somewhat limited in the amount of detail they can reveal. Electron
microscopes are ideal because you can view intricate details; however, they are
bulky, costly, and preparation for the microscopic examination kills the specimen.

Advanced Placement Biology Instructor’s Solution Manual


68 4 | Cell Structure

29 Mitochondria are observed in plant cells that contain chloroplasts. Why do you find
mitochondria in photosynthetic tissue?
A Mitochondria are not needed but are an evolutionary relic.
B Mitochondria and chloroplasts work together to use light energy to make sugars.
C Mitochondria participate in the Calvin cycle/light-independent reactions of
photosynthesis.
D Mitochondria are required to break down sugars and other materials for energy.
Solution The solution is (D). The cell needs energy constantly. When light is available, the cell
can produce energy directly from photosynthesis; but, in the dark, it generates
energy as ATP mainly through respiration, which involves the activity of
mitochondria.
30 In what situation(s) would the use of a light microscope be ideal? Why?
A A light microscope is used to view the details of the surface of a cell, as it cannot be
viewed in detail by the transmission microscope.
B A light microscope allows visualization of small cells that have been stained.
C A standard light microscope is used to view living organisms with little contrast to
distinguish them from the background, which would be harder to see with the
electron microscope.
D A light microscope reveals the internal structures of a cell, which cannot be viewed by
transmission electron microscopy.
Solution The solution is (B). Light microscopes are advantageous for viewing living organisms,
but since individual cells are generally transparent, their components are not
distinguishable unless they are colored with special stains. Staining, however, usually
kills the cells.
31 The major role of the cell wall in bacteria is protecting the cell against changes in osmotic
pressure, pressure caused by different solute concentrations in the environment. Bacterial
cells swell but do not burst in low solute concentrations.
What happens to bacterial cells if a compound that interferes with the synthesis of the
cell wall is added to an environment with low solute concentrations?
A Bacterial cells will shrink due to the lack of cell wall material.
B Bacterial cells will shrink in size.
C Bacterial cells may burst due to the influx of water.
D Bacterial cells remain normal; they have alternative pathways to synthesize cell walls.
Solution The solution is (C). The cells will burst because of the movement of water inside the
cell because there is no cell wall protection against bursting. Important antibiotics
such as penicillin kill bacteria by interfering with the cell wall synthesis.

Advanced Placement Biology Instructor’s Solution Manual


4 | Cell Structure 69

32 There is a lower limit to cell size. What determines how small a cell can be?
A The cell should be large enough to escape detection.
B The cell should be able to accommodate all the structures and metabolic activities
necessary to survival.
C The size of the cell should be large enough to reproduce itself.
D The cell should be large enough to adapt to the changing environmental conditions.
Solution The solution is (B). A cell must be large enough to accommodate all the structures
and metabolic activities that are required to sustain life. A smaller size also facilitates
predation and reduces the ability to bind to surfaces.
33 Which statement is a possible explanation for the presence of a rigid cell wall in plants?
A Plants remain exposed to changes in temperature and thus require rigid cell walls to
protect themselves.
B Plants are subjected to variations in osmotic pressure, and a cell wall helps them
against bursting or shrinking.
C Plant cells have a rigid cell wall to protect themselves from grazing animals.
D Plant cells have a rigid cell wall to prevent the influx of waste material.
Solution The solution is (B). Plant cells are subject to extreme changes in osmotic pressures.
The rigid cell wall protects tissues against excessive shrinking or bursting of cells.
34 Bacteria do not have organelles, yet the same reactions that take place on the
mitochondria inner membrane, the phosphorylation of ADP to ATP, and chloroplasts,
photosynthesis, take place in bacteria.
Where do these reactions take place?
A These reactions take place in the nucleoid of the bacteria.
B These reactions occur in the cytoplasm present in the bacteria.
C These reactions occur on the plasma membrane of bacteria.
D These reactions take place in the mesosomes.
Solution The solution is (C). ATP formation and photosynthesis occur on the plasma
membrane. In some photosynthetic bacteria such as the cyanobacteria,
photosynthesis occurs on loose internal membranes.

Advanced Placement Biology Instructor’s Solution Manual


70 4 | Cell Structure

35 What are the structural and functional similarities and differences between mitochondria
and chloroplasts?
A Similarities: double membrane, intermembrane space, ATP production, contain DNA.
Differences: Mitochondria have inner folds called cristae; chloroplast contains
accessory pigments in thylakoids, which form grana and stroma.
B Similarities: DNA, intermembrane space, ATP production, and chlorophyll. Differences:
Mitochondria have a matrix and inner folds called cristae; chloroplast contains
accessory pigments in thylakoids, which form grana and a stroma.
C Similarities: double membrane and ATP production. Differences: Mitochondria have
intermembrane space and inner folds called cristae; chloroplast contains accessory
pigments in thylakoids, which form grana and a stroma.
D Similarities: double membrane and ATP production. Differences: Mitochondria have
intermembrane space, inner folds called cristae, ATP synthase for ATP synthesis, and
DNA; chloroplast contains accessory pigments in thylakoids, which form grana and
a stroma.
Solution The solution is (A). Both are enveloped in a double membrane, have an
intermembrane space, and make ATP. Mitochondria and chloroplasts have DNA, and
mitochondria have inner folds called cristae and a matrix, while chloroplasts have
chlorophyll and accessory pigments in the thylakoids that form stacks (grana) and
a stroma.
36 Is the nuclear membrane part of the endomembrane system? Why or why not?
A The nuclear membrane is not a part of the endomembrane system, as the
endoplasmic reticulum is a separate organelle of the cell.
B The nuclear membrane is considered a part of the endomembrane system, as it is
continuous with the Golgi body.
C The nuclear membrane is part of the endomembrane system, as it is continuous with
the rough endoplasmic reticulum.
D The nuclear membrane is not considered a part of the endomembrane system, as the
nucleus is a separate organelle.
Solution The solution is (C). Because the external surface of the nuclear membrane is
continuous with the rough endoplasmic reticulum, which is part of the
endomembrane system, it is part of the endomembrane system.
37 What happens to the proteins that are synthesized on free ribosomes in the cytoplasm?
Do they go through the Golgi apparatus?
A These proteins move through the Golgi apparatus and enter in the nucleus.
B These proteins go through the Golgi apparatus and remain in the cytosol.

Advanced Placement Biology Instructor’s Solution Manual


4 | Cell Structure 71

C The proteins do not go through the Golgi apparatus and move into the nucleus for
processing.
D The proteins do not go through the Golgi apparatus and remain free in the cytosol.
Solution The solution is (D). Most of the proteins synthesized on free ribosomes stay in the
cytosol. They do not go through the Golgi apparatus to be tagged for export or
integration in membranes.
38 What are the similarities and differences between the structures of centrioles and
flagella?
A Centrioles and flagella are made of microtubules but show different arrangements.
B Centrioles are made of microtubules, but flagella are made of microfilaments, and
both show the same arrangement.
C Centrioles and flagella are made of microfilaments. Centrioles have a 9 + 2
arrangement.
D Centrioles are made of microtubules, flagella are made of microfilaments, and both
have different structures.
Solution The solution is (A). Centrioles and flagella are made of microtubules. Centrioles have
two rings of nine microtubule “triplets” arranged at right angles to one another.
Flagella, in turn, show 9 + 2 arrangement.
39 Inhibitors of microtubule assembly, vinblastine for example, are used for cancer
chemotherapy. How does an inhibitor of microtubule assembly affect cancerous cells?
A The inhibitors restrict the separation of chromosomes by the mitotic spindle.
B The inhibition of microtubules interferes with the synthesis of proteins.
C The inhibitors bind the microtubule to the nuclear membrane.
D The inhibitors interfere with energy production.
Solution The solution is (A). Cancerous cells divide rapidly. By inhibiting microtubules’
movement, cell division stops because the chromosomes cannot be separated by
the centrioles.
40 How do cilia and flagella differ?
A Cilia are made of microfilaments and flagella of microtubules.
B Cilia are helpful in the process of engulfing food. Flagella are involved in the
movement of the organism.
C Cilia are short and found in large numbers on the cell surface, whereas flagella are
long and fewer in number.
D Cilia are found in prokaryotic cells and flagella in eukaryotic cells.
Solution The solution is (C). Cilia and flagella are alike in that they are made up of
microtubules. Cilia are short, hair-like structures that exist in large numbers and

Advanced Placement Biology Instructor’s Solution Manual


72 4 | Cell Structure

usually cover the entire surface of the plasma membrane. In contrast, flagella are
long, hair-like structures; when flagella are present, a cell has just one or two.
41 In which human tissues would you find desmosomes?
A Bone cells and cartilage cells
B Muscle cells and skin cells
C Nerve cells and muscle cells
D Secretory cells and muscle cells
Solution The solution is (B). Muscle and skin cells undergo most of the mechanical stress, thus
show the presence of desmosomes.
42 If there is a mutation in the gene for collagen, such as the one involved in Ehlers-Danlos
syndrome, and the individual produces defective collagen, how would it affect
coagulation?
A The syndrome affects the clotting factors and platelet aggregation.
B The syndrome leads to hypercoagulation of blood.
C Coagulation is not affected because collagen is not required for coagulation.
D The syndrome occurs due to the breakdown of platelets.
Solution The solution is (A). The Ehlers-Danlos syndrome affects the connective tissues and
indirectly the coagulation process as well. It manifests in type I to type X, which
show the malfunctioning of different clotting factors and platelet aggregation. This
leads to impaired coagulation, causing patients to bruise and bleed easily. The
research on this disease is ongoing.
43 How does the structure of a plasmodesma differ from that of a gap junction?
A Gap junctions are essential for transportation in animal cells, and plasmodesmata are
essential for the movement of substances in plant cells.
B Gap junctions are found to provide attachment in animal cells, and plasmodesmata
are essential for the attachment of plant cells.
C Plasmodesmata are essential for communication between animal cells, and gap
junctions are necessary for attachment of cells in plant cells.
D Plasmodesmata help in transportation, and gap junctions help in attachment in
plant cells.
Solution The solution is (A). A plasmodesma is a channel between the cell walls of two
adjacent plant cells. Plasmodesmata allow materials to pass from the cytoplasm of
one plant cell to the cytoplasm of an adjacent cell. A gap junction is a protein-lined
pore that allows water and small molecules to pass between adjacent animal cells.

Advanced Placement Biology Instructor’s Solution Manual


4 | Cell Structure 73

TEST PREP FOR AP® COURSES


44 Which organism appears first in the fossil record?
A Archaea
B Fish
C Protists
D Plants
Solution The solution is (A). The earliest fossils reported so far are layered structures called
stromatolites from Greenland dating from 3.7 billion years ago and hydrothermal
vent structures from Canada dating from 3.7 to 4.2 billion years ago. Both structures
are of microbial origin.
45 Why is it challenging to study bacterial fossils and determine if the fossils are members of
the domain Archaea rather than Bacteria?
A Bacteria lack rigid structures and thus do not form fossils.
B Bacteria have rigid structures, but their fossil impression is scarce.
C Fossils of bacteria are rarely found because bacteria were not abundant in the past.
D Fossils of bacteria changes over time due to the presence of new bacteria living
on them.
Solution The solution is (A). Bacteria do not easily form fossils because they lack rigid
structures such as shells or skeletons. It is hard to identify the biochemical features
that differentiate archaea from bacteria in fossils.
46 Pictured are two cells along with their radius.

What does cell (b) likely have when compared to cell (a)?
A Smaller surface area and larger volume
B Larger surface area and smaller volume
C Smaller surface area-to-volume ratio
D Larger surface area-to-volume ratio

Advanced Placement Biology Instructor’s Solution Manual


74 4 | Cell Structure

Solution The solution is (C). Cell (b) will have the lowest ratio and would not exchange
materials as rapidly as cell (a).
47 Consider the shapes. The diameter of the sphere is equal to 1 mm and the side of the
cube is also equal to 1 mm.

What is the ratio of the surface-to-volume ratios for the sphere and the cube?
A 3:1
B 4:1
C 1:1
D 2:1
Solution The solution is (C). The surface area-to-volume ratio of circle to square will be the
same. Remember that the diameter (D), not the radius (r), is equal to 1 mm.
Therefore, .
For the sphere:

For the cube:

48 Consider the shapes. The diameter of the sphere is equal to 1 mm and the side of the
cube is also equal to 1 mm.

What is true regarding the surface-to-volume ratios of the cube and the sphere?
A The sphere will have a higher surface area than the cube.

Advanced Placement Biology Instructor’s Solution Manual


4 | Cell Structure 75

B The sphere will have a higher volume than the cube.


C The sphere will have a higher surface area-to-volume ratio than the cube.
D Their surface area-to-volume ratios will be equal.
Solution The solution is (D). Remember that the diameter (D), not the radius (r), is equal to 1
mm. Therefore, .
For the sphere:

For the cube:

49 What is the major consideration in setting the lower limit of cell size?
A The cell must be large enough to fight the pathogens.
B The cell must be large enough to attach to a substrate.
C The lower limit should be small enough for the cell to move in the fluid efficiently.
D The cell size must be small as to fit all the processes and structures to support life.
Solution The solution is (D). The cells must be large enough to include all of the necessary
structures and materials, including DNA, ribosomes, enzymes, and other cellular
structures, determining the lower end of the scale. It should be able to support life.
50 Which structure has the same general structure in the domains Archaea, Bacteria, and
Eukarya, pointing to a common origin?
A Centriole
B Cytoplasmic membrane
C Golgi apparatus
D Nucleus
Solution The solution is (B). Cytoplasmic membrane is found in all the three domains. The
centriole, Golgi apparatus, and the nucleus are found only in the domain Eukarya.
Archaea and Bacteria are prokaryotes.

Advanced Placement Biology Instructor’s Solution Manual


76 4 | Cell Structure

51 Why does the structure of the cytoplasmic membrane point to a common ancestor?
A The presence of a cytoplasmic membrane in every organism does not point to a
common ancestry.
B The similar arrangement of phospholipids and proteins points to common ancestry.
C The lipid nature of the membrane makes it the most primitive trait.
D The similar effect of temperature on the membrane makes it the ancestral trait.
Solution The solution is (B). The similar arrangement of macromolecules, phospholipids, and
embedded proteins in the lipid bilayer makes it a primitive trait. Archaea, Bacteria,
and Eukarya display a closely related general architecture of the plasma membrane.
52 Which organelles would be present in high numbers in the leg muscles of a marathon
runner?
A Centrioles
B Chloroplasts
C Mitochondria
D Peroxisome
Solution The solution is (C). Mitochondria produce the required energy in the form of ATP to
run a marathon. The higher number of mitochondria is needed to sustain a
prolonged activity.
53 Macrophages ingest and digest many pathogens. Which organelle plays a major role in
the activity of macrophages?
A Chloroplast
B Lysosome
C Nucleus
D Peroxisome
Solution The solution is (B). Lysosomes contain the hydrolytic enzymes that digest the
ingested pathogen. The pathogens are engulfed in a vesicle called a phagosome that
fuses with a lysosome in the cytoplasm forming, a phagolysosome where digestion
takes place. This process is called phagocytosis.
54 You are looking at a sample under a light microscope and observe a new type of cell. You
come to the conclusion that it is a bacterium and not a eukaryotic cell.
What would you observe to come to this conclusion?
A The cell has a cell wall.
B The cell has a flagellum.

Advanced Placement Biology Instructor’s Solution Manual


4 | Cell Structure 77

C The cell does not have a nucleus.


D The cell is small.
Solution The solution is (C). Only a bacterial cell would lack a nucleus. Plant and fungal cells,
and some protists have cell walls. Flagella are found in both prokaryotic and
eukaryotic cells. Bacteria are generally smaller than eukaryotic cells, but there are
notable exceptions. The bacterium Thiomargarita namibiensis can be seen with the
naked eye.
55 Thiomargarita namibiensis is a large single-cell organism, which can reach lengths of
700 μm. The cell is classified as a bacterium.
What is the main argument to justify the classification?
A This organism shows simple diffusion for the uptake of nutrients and is thus classified
as a bacterium.
B This organism does not show presence of any cell organelles and is thus classified as a
bacterium.
C This organism appears pearl-like and exists in long chains and is thus classified as a
bacterium.
D This organism demonstrates characteristics of gram-negative bacteria and is thus
classified as a bacterium.
Solution The solution is (B). Like the other prokaryotes, Thiomargarita namibiensis does not
possess a nucleus or any other organelles thus is classified as a bacterium.
56 Radioactive amino acids are fed to a cell in culture for a short amount of time. This is
called a pulse. You follow the appearance of radioactive proteins in the cell
compartments.
In which organelles and in what order does radioactivity appear?
A Endoplasmic reticulum - lysosomes - Golgi body - vesicle - extracellular region
B Endoplasmic reticulum - vesicles - Golgi body - vesicles - extracellular region
C Golgi Body - vesicles - endoplasmic reticulum - vesicles - extracellular region
D Nucleus - endoplasmic reticulum - Golgi body - vesicle - extracellular region
Solution The solution is (B). Radioactive proteins will first appear associated with the rough
endoplasmic reticulum where proteins are synthesized on ribosomes; then in
vesicles in transit to the Golgi apparatus; in the Golgi apparatus itself where they are
modified for export; and finally, in vesicles associated with exocytosis.
57 The extracellular matrix interacts with which cellular structure?
A Cytoskeleton
B Lysosome

Advanced Placement Biology Instructor’s Solution Manual


78 4 | Cell Structure

C Nucleus
D Smooth endoplasmic reticulum
Solution The solution is (A). The cytoskeleton, which provides the cell with rigidity and shape,
is made up of microfilaments and intermediate filaments that interact with the
extracellular matrix.
58 Which structure or structures allow bacteria to move about?
A Fimbriae only
B Flagella only
C Flagella and fimbriae
D Plasmid and capsule
Solution The solution is (C). Flagella, made of flagellin, and fimbriae, made of pilin, are both
involved in the movement of the bacteria in the fluid or on surfaces such as rocks, or
the mucous membranes lining body cavities.
59 Cells lining the intestine absorb a lot of nutrients. How did those cells adapt to their
function?
A Cells use cilia to move nutrients to their surface.
B Cells grow much larger than adjacent cells to increase intake.
C Cells are flat and thin to absorb more nutrients.
D Membrane folds called microvilli increase the surface area.
Solution The solution is (D). The microvilli are the cellular membrane protrusions of the
intestinal membrane, which increase the surface area for maximum absorption.

SCIENCE PRACTICE CHALLENGE QUESTIONS


4.3 Eukaryotic Cells
60 Describe structural and functional similarities between mitochondria and chloroplasts
that provide evidence of common ancestry.
Solution Both organelles are enclosed by a pair of phospholipid membranes with folds that
define inner and outer regions. Both have their own DNA and ribosomes. These
similarities are unlikely to have arisen independently and therefore provide evidence
of a common ancestor. Core conserved properties include information storage using
DNA, electron transport chains in both respiration and photosynthesis, ATP-based
energy storage, and inner membrane separation, creating potential gradient due to
a proton gradient associated with membrane-bound proteins facilitating active
transport. These similarities are unlikely to have arisen independently and therefore
provide evidence of a common ancestor.

Advanced Placement Biology Instructor’s Solution Manual


4 | Cell Structure 79

61 Explain how the structural and functional differences between mitochondria and
chloroplasts provide evidence of adaptations among common ancestral organisms.
Solution In mitochondria, organic compounds and oxygen are converted to carbon dioxide
and water. In the chloroplast, carbon dioxide and water are converted to organic
compounds and oxygen. This provides evidence of a common ancestor from which
these two organelles diverged. Electrochemically, the function of the mitochondria
and chloroplasts are complementary. In the mitochondria, carbon in organic carbon
compounds is oxidized by oxidative phosphorylation to carbon dioxide with the
synthesis of ATP from ADP. This oxidation is coupled to the reduction of molecular
oxygen to the oxygen in water. In the chloroplast, the carbon of carbon dioxide is
reduced in the synthesis of organic carbon molecules while the oxygen in water is
reduced to molecular oxygen. Electrochemically, the structure of the mitochondria
and chloroplasts are complementary. In the chloroplast, in the space contained by
the inner membrane, referred to as the thylakoid lumen, the concentration of
protons is higher than in the space bounded by the outer and inner membranes. The
proton gradient is established by a light-activated sequence of reduction-oxidation
reactions. This proton gradient is coupled to the synthesis of ATP from ADP by the
thylakoid membrane-bound enzyme ATP synthase. In contrast, the mitochondrion in
the space bounded by the inner membrane, the matrix, has a lower concentration of
protons. This concentration gradient is produced by the active transport associated
with a sequence of reduction-oxidation reactions among membrane-bound proteins.
This gradient is coupled to the synthesis of ATP from ADP by the membrane-bound
enzyme ATP synthase. These differences arise through of the interaction of light with
chromophores. The explanation of these complementary structures and functions is
provided by the theory of endosymbiosis, in which both organelles diverge from a
photosynthetic prokaryotic ancestor.
62 Examine the differences and similarities in the structural features of animal and plant
cells. Justify the claim that both animals and plants have common ancestors based on
your observations.
Solution Students could enumerate these structural similarities:
 Nucleus with nuclear membrane
 Smooth and rough endoplasmic reticulum
 Plasma membrane
 Nucleolus
 Golgi apparatus
 Mitochondria
 Peroxisome

Advanced Placement Biology Instructor’s Solution Manual


80 4 | Cell Structure

 Cytoskeleton
 Vacuole
Students could enumerate these differences:
 Chloroplasts
 Lysosomes
 Plastids
 Cell walls
So many similarities in structure lead to the conclusion that it is unreasonable that
all of these structures arose independently, and so there must be a common
ancestor.
63 What conserved core processes are common to both animals and plants? Construct an
explanation of the differences based on the selective advantages provided in different
environments.
Solution Students could enumerate the following common core processes:
 Information storage and retrieval from a nucleus separated by a nuclear
membrane
 Free-energy transduction by mitochondria
 Coordinated synthetic processes distributed throughout the cell interior
 Post-synthesis processing of proteins at the Golgi apparatus
 Structural integrity and transport provided by a cytoskeleton
 Peroxisome for recycling waste
 Plasma membrane regulation of transport and signaling into and out of
the cell
Students could enumerate the following differences in processes:
 Photosynthesis within chloroplasts
 Cell rigidity provided by a cell wall and a central vacuole
Students’ explanations of the divergence revealed by process differences should
include
 Selective advantage in an environment with high-intensity radiation is
provided by specialized chromophores
 Selective advantage in an environment with stresses associated with
intense mechanical forces and/or variations in concentrations of water in the
environment is provided by the external mechanical support provided by a
cell wall

Advanced Placement Biology Instructor’s Solution Manual


4 | Cell Structure 81

 Selective advantage where mobility is limited is provided by free-energy


procurement through interaction with radiation
 Selective advantage where free-energy resources are distributed is
provided through mobility that arises with deformable cells
4.6 Connections Between Cells and Cellular Activities
64 Louis Sullivan described architectural design as “form follows function.” For example, a
window is designed to add light to a space without heat transport. A door is designed to
allow access to a space. Windows and doors have different functions and therefore take
different forms. Biological systems are not designed, but selected from random trials by
interaction with the environment. Apply Sullivan’s principle to explain the relationship of
function and form for each pair of cellular structures below.
A. Plasma membrane and endoplasmic reticulum
B. Mitochondrion and chloroplast
C. Rough endoplasmic reticulum and smooth endoplasmic reticulum
D. Flagella and cilia
E. Muscle cells and secretory cells
Solution A. The plasma membrane forms the outer boundary of the cell and regulates the
transport of materials to and from the environment. One function of the
endoplasmic reticulum is to provide a surface on which subcellular structures
manage protein and lipid production for processes inside or outside of the cell.
Therefore, the plasma membrane has a relatively small area-to-volume ratio and
few folds that increase surface area, while the ER is highly convoluted.
B. The function of mitochondria is the synthesis of ATP from ADP using oxygen and
organic carbon compounds. The function of chloroplasts is the synthesis of ATP from
ADP and organic carbon compounds using radiant energy, carbon dioxide, and
water. Therefore, mitochondria have no systems for the capture of light energy,
while chloroplasts do.
C. The function of the rough ER is the synthesis of proteins within ribosomes, while
the function of the smooth ER is the synthesis of materials other than protein.
Therefore, the surface of the rough ER is studded with ribosomes, while the surface
of the smooth ER is not.
D. The function of flagella is motility. The function of the cilia is primarily the
redistribution of materials on the cell surface. Therefore, the structure of the flagella
is elongated and directional, while cilia are shorter and distributed over the surface.
E. The function of muscle cells is movement that is achieved by metabolically active
changes in cell shape. The function of secretory cells such as those of the liver,
pancreas, or glands is the production of protein-rich materials. Therefore, muscle
cells are dense with rough ER, while secretory cells are dense with smooth ER.

Advanced Placement Biology Instructor’s Solution Manual


82 4 | Cell Structure

65 Complex multicellular organisms share nutrients and resources, and their cells
communicate with each other. A society may encourage cooperation among individuals
while discouraging selfish behavior to increase the overall success of the social system,
sometimes at the expense of the individual. Scientific questions are testable and often
attempt to reveal a mechanism responsible for a phenomenon. Pose three questions that
can be used to examine the ways in which a social system regulates itself. Be prepared to
share these in small group discussions with your classmates about the similarities
between these regulatory strategies and the analogous roles of plasmodesmata and gap
junctions in cell communication.
Solution Sample answers:
 How are material resources distributed among individuals in a society?
Students might be drawn to contrast the exchange of time for money and
the exchange of money for goods with withdrawal from a shared cache of
goods. The former is analogous to active transport among cells through
structures such as plasmodesmata in plants and gap junctions in animals,
while the latter is analogous to diffusion over a concentration or pressure
gradient.
 How does a society exert control on individuals? Students might be
drawn to contrast the regulation of individual behaviors through a delegated
judicial system with an authoritarian system. The former is analogous to the
coordinated exchange among cells of molecules that regulate the behaviors
of others and is consistent with many relatively small and possibly specialized
cells. The latter is analogous to regulatory control predominantly within the
cell in competition with other cells for resources.
 How is information shared within a society? Students might be drawn to
contrast simple cell adhesion and clustering with diffusion of materials
through the extracellular space with active transport along specialized
structures such as plasmodesmata or gap junctions. These are analogous to
the isolation (hermit-like), low-density populations with very low-speed
communication, and high-density populations that are tightly connected.
 How are resources allocated for shared infrastructure? Students might be
drawn to contrast societies by rates of taxation. Higher rates of taxation
produce a more structured support for resource sharing analogous to greater
investment of free energy in cytoskeleton.

Advanced Placement Biology Instructor’s Solution Manual


4 | Cell Structure 83

 How are individuals within a society differentiated? Students might be


drawn to contrast educational systems. In some societies, individual roles are
differentiated at an early age, and these roles are relatively unchanging. In
other societies, differentiation is deferred and/or roles change over time,
requiring retraining. These would be analogous to cell differentiation and the
ability of some cells under particular environmental conditions to
redifferentiate. In some societies, the roles of individuals are highly
specialized, while in others, roles are very similar. This would be analogous to
the transition to complex multicellular organisms.
66 Plasmodesmata in vascular plants and gap junctions in animals are examples of
specialized features of cells. Mechanisms by which transport occurs between cells evolved
independently within several eukaryotic clades. Explain, in terms of cellular cooperation,
the selective advantages provided by such structures.
Solution Student explanations of the selective advantages provided should include spatial
and temporal coordination: organisms with tissue and/or subcellular structures that
are specialized functions and event timing that are integrated are more efficient
than organisms without these characteristics. Coordination is achieved through cell-
cell signaling.
67 Mammalian red blood cells have no nuclei, must originate in other tissue systems, are
relatively long lived, are small with shapes that actively respond to their environment, and
are metabolic anaerobes. Other vertebrates have red blood cells that are usually
nucleated and are often relatively large, aerobic, self-replicating, and short lived.
To connect these facts to biology, you need to ask questions. The questions that you pose
will depend on the path your class is taking through the curriculum. Begin by summarizing
what you know:
 What are the functions of a eukaryotic cell nucleus?
 What is the approximate average size of a human red blood cell?
 What is the range of blood vessel diameters in adult humans?
 What is the range of red blood cell size in vertebrates?
 What is the average lifetime of a human red blood cell?
 How can you show how cell production is stimulated using examples
from particular systems?
 How is cell death controlled?
 What biochemical cycles are associated with anaerobic and aerobic
respiration, and what are the important differences between these?
 What process is involved in the transport of oxygen and carbon dioxide
into and out of red blood cells?

Advanced Placement Biology Instructor’s Solution Manual


84 4 | Cell Structure

 What behaviors and dynamic homeostatic processes might be associated


with the properties of red blood cells in mammalian and nonmammalian
organisms?
 What do you know about the evolutionary divergences among
vertebrates?
Your summary has revealed some similarities and differences among vertebrate
erythrocyte and circulatory system structures. Scientific questions are testable. They can
be addressed by making observations and measurements and analyzing the resulting
data.
A. Pose three scientific questions that arise from your summaries of what you know about
erythrocytes and capillary size.
B. For each question you pose, predict what you believe would be the answer and provide
reasoning for your prediction.
C. Describe an approach you think can be used to obtain data to test your prediction.
D. In the production of mammalian red blood cells, erythrocytes that have not yet
matured and are still synthesizing heme proteins are surrounded by a macrophage.
Predict the role of the macrophage in the maturation of a red blood cell.
Solution Sample answers:
 What are the functions of a eukaryotic cell nucleus?
 The function of the nucleus is storage of genetic information and
transcription of that information into RNA.
 What is the approximate, average size of a human red blood cell?
 between 5 and 10 micrometers
 What is the range of blood vessel diameters in adult humans?
 between 3 centimeters and 5 micrometers
 What is the range of red blood cell size in vertebrates?
 between 1 and 20 micrometers
If only Chapters 3 and 4 have been considered, then the first two bullets are
appropriate. Within these, and with a bit of help from Google, students should be
able to locate these answers.
A and B. The questions that students are able to pose will also depend on the
instructional sequence.
 What are selection pressures? Fitness will be determined by oxygen
availability and temperature variation, as well as other factors that are
integrated with the homeostatic strategy of the organism.

Advanced Placement Biology Instructor’s Solution Manual


4 | Cell Structure 85

 What are the trade-offs of a coordinated interaction among tissue


systems versus an autonomous cell production? Cell division does not
require the integration of tissue systems. So rather than investing in energy
costs of maintenance and communication, self-reproductive red blood cells
would be short lived and large in number, requiring a different type of energy
investment.
 Are cells that transport oxygen while not consuming oxygen more
efficient? Efficiency could be measured in terms of energy provided through
aerobic respiration at the target cell per energy required to deliver the
resource. The denominator would include the cost of production for the
anucleate strategy and the cost of aerobic respiration for the nucleated
strategy.
 Are the structures of capillaries different in mammalian and reptilian or
avian organisms? Thermal regulation and pulmonary structures vary, as well
as heme proteins, and the sizes and densities of capillaries are consistent
with these factors.
 How is thermoregulation affected by the cellular structure of the red
blood cell? Anucleate and nucleate strategies are coordinated with thermal
regulation, which is coordinated with temperature variation and behaviors.
How are production and apoptosis of red blood cells different in mammalian
and nonmammalian vertebrates? Mammals generally have red blood cells
without nuclei or mitochondria that are synthesized in bone marrow.
Reptiles and amphibians have nucleated cells with mitochondria, and
multiple organs are reported as the site of synthesis, for example liver and
kidney, depending on maturity. Birds have both nuclei and mitochondria.
 How are low oxygen states in tissues communicated to process that
control erythrocyte production? Hypoxia detection is at the level of the
tissues that secrete erythropoietin. It is a well conserved hormone in all
vertebrates. Erythropoietin induces cell division in tissues that produce red
blood cells, such as bone marrow. What do we know about the phylogeny of
this signaling system? Vertebrates differ in whether their red blood cells
contain mitochondria and a nucleus. Mammal red blood cells lack a nucleus
and mitochondria in their mature form, but the red blood cells of fish,
reptiles, and birds contain mitochondria and a nonfunctional nucleus that
cannot divide. We could expect that the ancestors of systems without
mitochondria emerged in a period with low atmospheric oxygen
concentrations and that those with mitochondria emerged where oxygen
levels were higher. What do we know about the phylogeny of mammalian
and nonmammalian oxygen transport? Mollusks, insects, and other
invertebrates have an open circulatory system, though there is a pump. The
structure of the pump varies with the number of chambers and their

Advanced Placement Biology Instructor’s Solution Manual


86 4 | Cell Structure

connection to pulmonary organs. So the conserved features of oxygen


transport are connected among vertebrates and invertebrates.
C. Data that connect the organism to its environment, including behaviors, and to its
ancestors have been pursued for each of the questions posed above.
D. The surrounding macrophage forms an erythropoietic niche that increases
availability of iron and, when the nucleus has been isolated, cleans up the debris
through phagocytosis.
68 Mitochondria have DNA that encode proteins related to the structures and functions of
the organelles. The replication appears to occur continuously; however, many questions
about control of replication rate and segregation during mitosis are yet unanswered.
Many diseases are caused by mitochondrial dysfunction. Mitophagy, as the name
suggests, leads to the destruction of mitochondria. Predict whether or not cellular control
mechanisms involving the regulation of mitochondrial DNA by the nucleus exist. Make use
of what you know about selection and homeostasis as they apply to both the organism
and to the organelle.
Solution Sample answer: Although mitochondria possess DNA and protein-synthesizing
machinery, they are not autonomous. Because mitochondria provide energy for
metabolism through oxidative phosphorylation, the control of the number of
mitochondria in the cell is essential to the maintenance of homeostasis. Therefore,
the cell must control both the biogenesis and the destruction of mitochondria. The
fact that mitochondria cannot replicate outside of the cell also implies that,
ultimately, DNA replication is under the control of the nucleus. It is possible to
predict that mtDNA replication is likely under the control of nuclear-encoded
proteins, for example, mitochondrial transcription factors. Mitophagy is a process by
which the cell regulates organelle number in response to developmental or
physiological signals. This process offers an evolutionary advantage by allowing the
cell to destroy defective or superfluous mitochondria and recycle their content.

Advanced Placement Biology Instructor’s Solution Manual


5 | Structure and Function of Plasma Membranes 87

5 | STRUCTURE AND FUNCTION OF


PLASMA MEMBRANES
REVIEW QUESTIONS

1 Which plasma membrane component can be either found on its surface or embedded in
the membrane structure
A Carbohydrates
B Cholesterol
C Glycolipid
D Protein
Solution The solution is (D). Proteins can be found either on the surface or embedded in the
membrane structure.
2 In addition to a plasma membrane, a eukaryotic cell has organelles, such as mitochondria,
that also have membranes. In which way would these membranes differ?
A The proportion of phosphate within the phospholipids will vary.
B Only certain membranes contain phospholipids.
C Only certain membranes are selectively permeable.
D The proportions of proteins, lipids, and carbohydrates will vary.
Solution The solution is (D). The membranes differ in proportions of proteins, lipids, and
carbohydrates, depending on the organelle or the cell type. For example, the
myelinated membrane of neurons is rich in lipids. The membranes of mitochondria
contain a high percentage of proteins.
3 Which characteristic of a phospholipid increases the fluidity of the membrane?
A Cholesterol
B Its head
C Saturated fatty acid tail
D Unsaturated fatty acid tail
Solution The solution is (D). Unsaturated fatty acids contain some double bonds between
adjacent carbon atoms; a double bond results in a bend in the string of carbons,
resulting in fluidity. Cells adapt to variations in temperature by changing the ratio of
saturated to unsaturated phospholipids in the membranes.

Advanced Placement Biology Instructor’s Solution Manual


88 5 | Structure and Function of Plasma Membranes

4 How would an organism maintain membrane fluidity in an environment where


temperatures fluctuate from very high to very low?
A Greater proportion of unsaturated phospholipids in the membranes
B Greater proportion of saturated phospholipids in the membranes
C Greater proportion of carbohydrates in the membranes
D Greater proportion of proteins in the membranes
Solution The solution is (A). The unsaturated fatty acids have “kinks” in their tails, which
increases fluidity in the membrane at low temperatures. The membranes with
saturated fatty acid tails in their phospholipids will “freeze,” or solidify.
5 According to the fluid mosaic model of the plasma cell membrane, what is the location of
carbohydrates in the cell membranes?
A Carbohydrates are in contact with the aqueous fluid both inside and outside the cell.
B Carbohydrates are present only on the interior surface of a membrane.
C Carbohydrates are present only on the exterior surface of a membrane.
D Carbohydrates span only the interior of a membrane.
Solution The solution is (C). Carbohydrates are generally attached to proteins on the outer
surface of the membrane and form glycoproteins, which play an important role in
cell-cell recognition. Carbohydrates can also bind to lipids, forming glycolipids.
6 What do double bonds in phospholipid fatty acid tails contribute to?
A The fluidity of membranes
B The hydrophobic nature of membranes
C The hydrophilic nature of membranes
D The prevention of high temperatures from increasing the fluidity of membranes
Solution The solution is (A). Double bonds in phospholipid fatty acid tails increase the fluidity
of a membrane because they create a bend in the molecules, preventing the tight
packing of the fatty acid tails and the ensuing rigidity.
7 What is the principal force driving movement in diffusion?
A Concentration gradient
B Membrane surface area
C Particle size
D Temperature
Solution The solution is (A). Diffusion is the net movement of a substance from a region of
high concentration to a region of low concentration. The principal driving force in
diffusion is this concentration gradient.

Advanced Placement Biology Instructor’s Solution Manual


5 | Structure and Function of Plasma Membranes 89

8 Which of the following is an example of passive transport across a membrane?


A The movement of H+ into a thylakoid disc during photosynthesis
B The uptake of glucose in the intestine
C The uptake of mineral ions into root hair cells of plants
D The movement of water from the descending loop of a nephron into the interstitium
Solution The solution is (D). Water crosses the collecting duct epithelium by osmosis, which is
passive transport. Protein channels called aquaporin facilitate the movement of
water. Hydrogen ions use the energy of photosynthesis to cross into the thylakoids
against their concentration gradient. They flow out into the stroma through the ATP
synthase along their gradient. The uptake of glucose by the intestine epithelium and
mineral ions into roots are examples of active transport.
9 Water moves via osmosis across plasma cell membranes in which direction?
A From an area with a high concentration of other solutes to a lower one
B From an area with a high concentration of water to one of lower concentration
C From an area with a low concentration of water to one of higher concentration
D Throughout the cytoplasm
Solution The solution is (B). Osmosis is the spontaneous net movement of water from a
region of higher concentration to lower concentration of water. For example, red
blood cells suspended in distilled water will swell and burst because water is flowing
inside the cells.
10 What problem is faced by organisms that live in fresh water?
A They will have higher concentrations of body solutes.
B Without compensating mechanisms, their bodies tend to take in too much water.
C They have no way of controlling their tonicity.
D Their bodies tend to lose too much water to their environment.
Solution The solution is (B). The fish are hypertonic to the fresh water environment they live
in; thus, too much water diffuses into their body. Compensating mechanisms include
excreting diluted urine and taking up salt through their gills.
11 Which of the following questions can be asked about organisms that live in fresh water?
A Will their bodies take in too much water?
B Can they control their tonicity?
C Can they survive in salt water?
D Will their bodies lose too much water to their environment?

Advanced Placement Biology Instructor’s Solution Manual


90 5 | Structure and Function of Plasma Membranes

Solution The solution is (A). Organisms contain salts and solutes that make their internal fluid
hypertonic to freshwater. Losing water to their environment is not the challenge
they face. They must limit the intake of water, and they must develop mechanisms
to get rid of excess water.
12 Why must active transport of molecules across plasma membranes function
continuously?
A Diffusion cannot occur in certain cells.
B Diffusion is constantly moving solutes in opposite directions.
C Facilitated diffusion works in the same direction as active transport.
D Not all membranes are amphiphilic.
Solution The solution is (B). Diffusion allows movement down the concentration gradient to
maintain equilibrium. Active transport must continuously pump solutes against the
concentration gradient to counteract diffusion.
13 How does the sodium-potassium pump make the interior of the cell negatively charged?
A By expelling anions
B By pulling in anions
C By expelling more cations than it takes in
D By taking in and expelling an equal number of cations
Solution The solution is (C). The sodium-potassium pump transports three sodium ions
outside the cell and takes two potassium ions into the cell, making the interior of the
cell less positively charged, therefore, more negatively charged.
14 What is the difference between primary and secondary active transport?
A Primary active transport is indirectly dependent on ATP, while secondary active
transport is directly dependent on ATP.
B Primary active transport is directly dependent on ATP, while secondary active
transport is indirectly dependent on ATP.
C Primary active transport does not require ATP, while secondary active transport is
indirectly dependent on ATP.
D Primary active transport is indirectly dependent on ATP, while secondary active
transport does not require ATP.
Solution The solution is (B). Primary active transport is directly dependent on ATP and
transports compounds against their concentration gradient, whereas secondary
active transport is indirectly dependent on ATP. For example, glucose and sodium
ions are part of a co-transport where a primary Na+/K+ active transport across the
membrane of epithelial cells powers the transport of glucose from the lumen of the
intestine into the cell against a concentration gradient.

Advanced Placement Biology Instructor’s Solution Manual


5 | Structure and Function of Plasma Membranes 91

15 What happens to the membrane of a vesicle after exocytosis?


A It leaves the cell.
B It is disassembled by the cell.
C It fuses with and becomes part of the plasma membrane.
D It is used again in another exocytosis event.
Solution The solution is (C). It fuses with and becomes part of the plasma membrane as the
contents of the vesicle are secreted out of the cell.
16 In what important way does receptor-mediated endocytosis differ from phagocytosis?
A It transports only small amounts of fluid.
B It does not involve the pinching off of the membrane.
C It brings in only a specifically targeted substance.
D It brings substances into the cell, while phagocytosis removes substances.
Solution The solution is (C). Receptor-mediated endocytosis brings in only targeted
substances whereas phagocytosis is not specific.

CRITICAL THINKING QUESTIONS


17 Why do phospholipids tend to spontaneously orient themselves into something
resembling a membrane such as the lipid-bilayer sphere, single-layer lipid sphere, and
lipid-bilayer sheet?
A Phospholipids are amphipathic molecules. The polar head faces toward water, and the
nonpolar fatty acid tails face toward other fatty acid tails.
B Phospholipids are lipophilic molecules. The polar head faces toward water, and the
nonpolar fatty acid tails face toward other fatty acid tails.
C Phospholipids are amphipathic molecules. The nonpolar head faces toward other fatty
acid tails, and the polar fatty acid tails face toward water.
D Phospholipids are hydrophilic molecules. The polar head faces toward water, and the
nonpolar fatty acid tails face toward other fatty acid tails.
Solution The solution is (A). The hydrophobic, nonpolar regions must align with each other for
the structure to have minimal potential energy and, consequently, higher stability.
The fatty acid tails of the phospholipids cannot mix with water, but the phosphate
head of the molecule can. Thus, the head orients to water, and the tail to other
lipids.
18 The fluid mosaic model described the plasma membrane as a mosaic of components. Why
is it advantageous for the plasma membrane to be fluid in nature?
A Fluidity allows greater flexibility to the cell and the motion of membrane components
required for transport.

Advanced Placement Biology Instructor’s Solution Manual


92 5 | Structure and Function of Plasma Membranes

B Fluidity helps only in transport of some materials, but does not contribute to the
flexibility.
C Fluidity helps in maintaining the pH of the intracellular fluid, and helps in maintaining
the physiological pH of the cell.
D Fluidity helps provide mechanical strength to the plasma membrane.
Solution The solution is (A). The fluid characteristic of the plasma membrane allows greater
flexibility to the cell than it would if the membrane were rigid. It also allows the
motion of membrane components required for some types of membrane transport.

Advanced Placement Biology Instructor’s Solution Manual


5 | Structure and Function of Plasma Membranes 93

19 List four components of a plasma membrane and explain their function.


A Phospholipids: form the bilayer; carbohydrates: help in adhesion; cholesterol:
provides flexibility; integral proteins: form transporters; peripheral proteins: part of
the cell’s recognition sites.
B Phospholipids: form the bilayer; carbohydrates: help in adhesion; cholesterol: forms
transporters; integral proteins: provide flexibility; peripheral proteins: part of the cell’s
recognition sites.
C Phospholipids: form the bilayer; carbohydrates: part of the cell’s recognition sites;
cholesterol: provides flexibility to the membrane; integral proteins: form transporters;
intermediate filaments: help in adhesion.
D Phospholipids: form the bilayer; carbohydrates: function as adhesion; cholesterol:
provides flexibility to the membrane; integral proteins: form transporters;
intermediate filaments: part of the cell’s recognition sites.
Solution The solution is (A). Phospholipids are the main fabric of the membrane. The
hydrophilic heads are in contact with aqueous fluid both inside and outside of the
cell. The hydrophobic tails face each other forming a lipid bilayer.
Carbohydrates are found on the exterior surface of cells and are bound either to
proteins (forming glycoproteins) or to lipids (forming glycolipids) and cell-cell
interactions.
Cholesterol is attached between phospholipids and between the two phospholipid
layers. It tends to dampen the effects of temperature on the membrane.
Integral proteins are embedded in the membrane structure. They include
transporters, channels, enzymes, and so on.
Peripheral proteins are found on the exterior and interior surfaces of membranes.
They include enzymes, transporters, signaling receptors, and so on.
20 Which explanation identifies how the following affect the rate of diffusion: molecular size,
temperature, solution density, and the distance that must be traveled?
A Larger molecules move faster than lighter molecules. Temperature affects the
molecular movement. Density is directly proportional to the molecular movement.
Greater distance slows the diffusion.
B Larger molecules move more slowly than lighter molecules. Increasing or decreasing
temperature increases or decreases the energy in the medium, affecting molecular
movement. Density is inversely proportional to molecular movement. Greater
distance slows the diffusion.

Advanced Placement Biology Instructor’s Solution Manual


94 5 | Structure and Function of Plasma Membranes

C Larger molecules move more slowly than lighter molecules. Temperature does not
affect the rate of diffusion. Density is inversely proportional to molecular movement.
Greater distance speeds up the diffusion.
D Larger molecules move more slowly than lighter molecules. Increasing or decreasing
temperature increases or decreases the energy in the medium, affecting molecular
movement. Density is inversely proportional to the molecular movement. Greater
distance speeds up the diffusion.
Solution The solution is (B). Larger molecules move more slowly than lighter ones. It takes
more energy in the medium to move them along. Increasing or decreasing
temperature increases or decreases the energy in the medium, affecting molecular
movement. The denser or more viscous a solution is, the harder it is for molecules to
move through it, causing diffusion to slow down due to friction. Living cells require a
steady supply of nutrients and a steady rate of waste removal. If the distance these
substances need to travel is too great, diffusion cannot move nutrients and waste
materials efficiently enough to sustain life.
21 Both of the regular intravenous solutions administered in medicine, normal saline and
lactated Ringer’s solution, are isotonic. Why is this important?
A Isotonic solutions maintain equilibrium and avoid the exchange of materials to or from
the blood.
B Isotonic solutions disrupt equilibrium and allow for better exchange of materials in the
blood cells.
C Isotonic solutions increase the pH of the blood and allow for better absorption of
saline in the blood cells.
D Isotonic solutions decrease the pH of the blood and avoid the exchange of materials to
or from the blood cells.
Solution The solution is (A). Injection of isotonic solutions ensures that there will be no
perturbation of the osmotic balance and no water taken from tissues or added to
them from the blood.
22 If a doctor injected a patient with what was labeled as an isotonic saline solution, but then
the patient died and an autopsy revealed that many red blood cells had burst, would it be
true that the injected solution was really isotonic. Why or why not?
A False, the solution was hypertonic.
B False, the solution was either hypotonic or hypertonic.
C False, the solution was hypotonic.
D True, the solution was isotonic.
Solution The solution is (C). The solution would have been hypotonic. Hypotonic solutions
have a lower salt concentration compared to the red blood cells; therefore, the cells
have low water potential compared to the saline solution. The water moves from

Advanced Placement Biology Instructor’s Solution Manual


5 | Structure and Function of Plasma Membranes 95

the region of high water potential to low water potential, causing the red blood cells
to swell and burst.
23 How does the sodium-potassium pump contribute to the net negative charge of the
interior of the cell?
A The sodium-potassium pump forces out three (positive) Na+ ions for every two
(positive) K+ ions it pumps in; thus, the cell loses a net positive charge of one at every
cycle of the pump.
B The sodium-potassium pump expels three K+ for every two Na+ inside the cells,
creating a net positive charge outside the cell and a net negative charge inside
the cell.
C The sodium-potassium pump helps the development of a negative charge inside the
cell by making the membrane more permeable to negatively charged proteins.
D The sodium-potassium pump helps in the development of a negative charge inside the
cell by making the membrane impermeable to positively charged ions.
Solution The solution is (A). The sodium/potassium pump maintains a negative potential
across the cell membrane by expelling three positive ions (Na+) for every two
positive ions (K+) pumped into the cell. The net result is that there is a deficit in
positive charges. Na+ ions are constantly expelled to balance osmotic pressure
between the cell and its environment; otherwise, the extracellular fluid would
become hypotonic and water would flow into the cells, causing them to swell
and burst.
24 Potassium is a necessary nutrient to maintain the function of our cells. What would
happen to a person who is deficient in potassium?
A The excess sodium disrupts the membrane components.
B The excess sodium increases action potential generation.
C The cells would not be able to get rid of extra sodium.
D The cells would not be able to bring in sodium.
Solution The solution is (C). Cells typically have a high concentration of potassium in the
cytoplasm and a high concentration of sodium outside of the cells. A deficiency of
potassium would prevent cells from getting rid of sodium using the sodium-
potassium pump.
25 Which statement describes processes of receptor-mediated endocytosis, exocytosis, and
the changes in the membrane organization involved with each?
A Receptor-mediated endocytosis involves the binding of a ligand to its receptor,
resulting in the formation of a clathrin-coated vesicle that enters the cell. In
exocytosis, waste material is enveloped in a vesicle that fuses with the interior of the
plasma membrane via attachment proteins.

Advanced Placement Biology Instructor’s Solution Manual


96 5 | Structure and Function of Plasma Membranes

B In receptor-mediated endocytosis, waste material is enveloped in a membrane that


fuses with the interior of the plasma membrane via attachment proteins. Exocytosis
involves the opsonization of the receptor and its ligand in clathrin-coated vesicles.
C In receptor-mediated endocytosis, waste material is enveloped in a membrane that
fuses with the interior of the plasma membrane via attachment proteins. Exocytosis
involves the opsonization of the receptor and its ligand in caveolae-coated vesicles.
D Receptor-mediated endocytosis involves the opsonization of the receptor and its
ligand in clathrin-coated vesicles that enter the cell. In exocytosis, waste material is
enveloped in a membrane that fuses with the exterior of the plasma membrane via
attachment proteins.
Solution The solution is (A). In receptor-mediated endocytosis, clathrin is attached to the
cytoplasmic side of the plasma membrane. The process involves the inward budding
of plasma membrane vesicles containing proteins with receptor sites specific to the
molecules being absorbed. After the binding of a ligand to the receptor, a signal is sent
through the membrane, leading to membrane coating and formation of a membrane
invagination. The receptor and its ligand are then opsonized in clathrin-coated
vesicles. Exocytosis is the process by which cells expel the contents of secretory
vesicles out of the cell membrane into the extracellular space. Waste material is
enveloped in a membrane, which fuses with the interior of the plasma membrane via
attachment proteins. This fusion opens the membranous envelope on the exterior of
the cell, and the waste material is expelled into the extracellular space.

26 Describe the process of potocytosis. How does it differ from pinocytosis?


A Potocytosis is a form of receptor-mediated endocytosis where molecules are
transported via caveolae-coated vesicles. Pinocytosis is a form of exocytosis used for
excreting excess water.
B Potocytosis is a form of exocytosis where molecules are transported via clathrin-
coated vesicles. Pinocytosis is a form of receptor-mediated endocytosis used for
excreting excess water.
C Potocytosis is a form of receptor-mediated endocytosis where molecules are
transported via caveolae-coated vesicles. Pinocytosis is a mode of endocytosis used
for the absorption of extracellular fluid.
D Potocytosis is a form of receptor-mediated endocytosis used for the absorption
of water.
Solution The solution is (C). Potocytosis is a type of receptor-mediated endocytosis in which
small molecules are transported across the plasma membrane of a cell. The
molecules are transported by caveolae rather than clathrin-coated vesicles.
Pinocytosis is a mode of endocytosis in which small particles are brought into the
cell. It is used primarily for the absorption of extracellular fluids.

Advanced Placement Biology Instructor’s Solution Manual


5 | Structure and Function of Plasma Membranes 97

TEST PREP FOR AP® COURSES


27 One type of mutation in the CFTR protein prevents the transport of chloride ions through
the channel. What is most likely to be observed in the lungs of patients with this
mutation?
A Dehydrated epithelial cells
B Dehydrated mucus
C Mucus with excess water
D Mucus with a high electrolyte concentration
Solution The solution is (B). When the normal movement of chloride ions is prevented, water
that follows by osmosis does not move to produce freely flowing mucus. The result
is cells that line the airways of the lungs secrete mucus that is thick and sticky.
28 Arsenic poisoning disrupts ATP production by inhibiting several of the enzymes in the
oxidative phosphorylation pathway. Some of the symptoms of arsenic poisoning are
similar to cystic fibrosis (CF; a hereditary disorder that results in difficulty breathing and
frequent lung infections).
What impact may arsenic poisoning have on components of the plasma membrane and
transport that would result in CF-like symptoms?
A Arsenic poisoning disrupts ATP production, leading to a decreased transport of ions
by epithelial cells. This leads to decreased electrolyte concentration in the mucus and
retention of water into the cells. The mucus becomes dehydrated, as in CF.
B Arsenic poisoning disrupts the Na+/ pump, leading to decreased transport of Cl–
ions out of the epithelial cells. This increases the electrolyte concentration in the
mucus and movement of water out of the cells. The mucus becomes hydrated, as
in CF.
C Arsenic poisoning affects the oxidative phosphorylation pathway, leading to a
decreased transport of Na+ ions out of the epithelial cells. This leads to increased
electrolyte concentration in the mucus and movement of water into the cells. The
mucus becomes dehydrated, as in CF.
D Arsenic poisoning disrupts the binding sites for ions, leading to a decreased
transport of Cl– ions outside the epithelial cells. This leads to decreased electrolyte
concentration in the mucus and movement of water out of the cells. The mucus
becomes hydrated, as in CF.
Solution The solution is (A). The CFTR protein requires ATP to transport ions out of
epithelial cells to the covering mucus. Arsenic poisoning disrupts ATP production;
therefore, ions will not be transported out of epithelial cells, total electrolyte
concentration in the mucus will not increase, and water will not move out of

Advanced Placement Biology Instructor’s Solution Manual


98 5 | Structure and Function of Plasma Membranes

epithelial cells to mucus via osmosis. The mucus will become dehydrated, as seen
in CF.
29 In individuals with a normally functioning CFTR protein, which substances are transported
via active transport?
A
B Mucus
C Na+
D Water
Solution The solution is (A). The CFTR protein transports ions out of epithelial cells and
into mucus in the intercellular space using ATP hydrolysis. Water follows by osmosis,
maintaining a fluid mucus in the passageways of the body, e.g., in the lungs and
digestive tract.
30 The sodium-potassium (Na+/K+) pump functions like an anti-porter transporting Na+ and K+
icons across membranes using ATP. This protein spans the membrane with intracellular
and extracellular domains. It has a binding site for Na+, K+, and ATP. An experiment was
conducted to determine the locations of these binding sites. Artificial cells were created
and incubated in buffers containing ATP, ouabain (or g-strophanthin, a cardiac glycoside),
Na+, and K+ in varying combinations inside and outside of the cells, as indicated in the
given table. The transport of Na+ and K+ was measured to determine the activity of the
Na+/K+ pump.
ATP ATP Ouabain Ouabain
present present present present
inside outside inside outside Was Na+ Was K+
Experiment cells? cells? cells? cells? transported? transported?
1 Yes Yes No No Yes Yes
2 Yes No No No Yes Yes
3 No Yes No No No No
4 No No No No No No
5 Yes No Yes Yes No No
6 Yes No Yes No No Yes
7 Yes No No Yes Yes No
Which of the following conclusions is supported by the data?
A Ouabain does not disrupt ATP binding to the Na+/K+ pump.
B ATP is required for the transport of Na+ and not for transport of K+.

Advanced Placement Biology Instructor’s Solution Manual


5 | Structure and Function of Plasma Membranes 99

C The ATP binding site of the Na+/K+ pump is located on the intracellular domain of
the pump.
D The ATP binding site of the Na+/K+ pump is located on the extracellular domain of
the pump.
Solution The solution is (C). The ATP binding site of the Na+/K+ pump is located on the
intracellular domain of the pump. If ATP is not present extracellularly but present
inside the cell, the exchange takes place. If there is no ATP inside the cell, there is no
exchange. If ATP is present outside the cell but not inside, the exchange does not
take place whether ouabain is present or absent. There is no evidence presented
that ATP cannot bind to the pump in the presence of ouabain.
31 Paramecia are unicellular protists that have contractile vacuoles to remove excess
intracellular water. In an experimental investigation, Paramecia were placed in salt
solutions of increasing osmolarity. The rate at which a Paramecium’s contractile vacuole
contracted to pump out excess water was determined and plotted against the osmolarity
of the solutions, as shown in the graph.

Which of the following is the correct explanation for the data?


A At higher osmolarity, lower rates of contraction are required because more salt
diffuses into the Paramecium.
B In an isosmotic salt solution, there is no diffusion of water into or out of the
Paramecium, so the contraction rate is zero.
C The contraction rate increases as the osmolarity decreases because the amount of
water entering the Paramecium by osmosis increases.
D The contractile vacuole is less efficient in solutions of high osmolarity because of the
reduced amount of ATP produced from cellular respiration.
Solution The solution is (C). The contraction rate increases as the osmolarity of the solution
decreases because water will diffuse into the paramecium’s cytoplasm. The
increased contraction rate pumps the incoming water out of the cell.

Advanced Placement Biology Instructor’s Solution Manual


100 5 | Structure and Function of Plasma Membranes

32 What is most likely to happen if Paramecia are moved from a hypertonic solution to
solutions of decreasing osmolarity?
A The rate of contraction would increase with decreasing osmolarity because more
water diffuses into the Paramecium.
B The rate of contraction would decrease with decreasing osmolarity because more
water diffuses into the Paramecium.
C The rate of contraction would increase with decreasing osmolarity because more salt
diffuses into the Paramecium.
D The rate of contraction would decrease with decreasing osmolarity because more salt
diffuses into the Paramecium.
Solution The solution is (A). The rate of contraction would increase with decreasing
osmolarity because more water diffuses into the Paramecium.
33 Describe the Na+/K+ pump, labeling the binding sites for Na+, K+, and ATP. Explain how the
data indicate the location of the binding sites for Na+ and K+ on the pump. Based on the
data in the table, which statement describes the location of the binding sites for Na + and
K+ on the pump?
ATP ATP Ouabain Ouabain
present present present present
inside outside inside outside Was Na+ Was K+
Experiment cells? cells? cells? cells? transported? transported?
1 Yes Yes No No Yes Yes
2 Yes No No No Yes Yes
3 No Yes No No No No
4 No No No No No No
5 Yes No Yes Yes No No
6 Yes No Yes No No Yes
7 Yes No No Yes Yes No
+
A The binding of Na occurs on the outer surface of the cell, as its transportation
remains unaffected by the presence of ouabain. The binding of K+ occurs on the
inner surface of the cell, as its transportation is blocked when ouabain is present
inside the cell.
B The binding of K+ occurs on the outer surface of the cell, and its transportation is
blocked when ouabain is present outside the cell. The binding of Na+ occurs on the
inner surface of the cell, and its transportation is blocked by the presence of ouabain.

Advanced Placement Biology Instructor’s Solution Manual


5 | Structure and Function of Plasma Membranes 101

C The binding of K+ occurs on the outer surface of the cell, and the binding of Na+ occurs
on the inner surface of the cell because they are not transported when ATP is absent.
D The binding of Na+ occurs on the outer surface of the cell, and the binding of K+ occurs
on the inner surface of the cell because they are not transported when ATP is absent.
Solution The solution is (B). K+ binds to the outer surface of the cell while Na+ binds to the
inner surface of the cell. The addition of ouabain to the outside of the cells inhibits
transport of K+ only, while ouabain inside the cell inhibits Na+ transport.
34 An experiment was set up to determine the movement of molecules through a dialysis-
tubing bag into water. A dialysis-tubing bag containing 5% lactose and 5% fructose was
placed in a beaker of distilled water, as illustrated in the given figure. After four hours,
fructose is detected in the distilled water outside of the dialysis-tubing bag, but lactose is
not.

What conclusions can be made about the movement of molecules in this experiment?
A Fructose, being a monosaccharide, diffused through the dialysis bag into the
distilled water. However, lactose, being a disaccharide, could not diffuse through
the dialysis bag.
B Fructose was homogenized by lactose, allowing the fructose to diffuse through the
dialysis bag and into the distilled water. Lactose is not homogenized, so it could not
pass through the dialysis bag.
C Fructose and lactose are oppositely charged and separated out due to the force of
repulsion.
D Fructose diffused because of the pore specificity of the semipermeable membrane,
not because of its concentration gradient.
Solution The solution is (A). Fructose is a monosaccharide, so it will be able to cross the
membrane of the bag. Lactose is a disaccharide and is unable to cross the
membrane. Therefore, fructose is observed in the distilled water outside of the
dialysis-tubing bag, but lactose is not.

Advanced Placement Biology Instructor’s Solution Manual


102 5 | Structure and Function of Plasma Membranes

35 This is the 3D structure of clathrin, with an individual subunit highlighted in blue. What is
a functional reason for this structure?
A Clathrin stabilizes the section of the membrane forming the vesicle.
B Clathrin is heat resistant.
C The interlocking clathrin subunits provide a rigid cell structure.
D Clathrin fuses with the trans Golgi apparatus.
Solution The solution is (A). Clathrin stabilizes the membrane-forming lattice of the vesicle.
36 Which of the following statements appropriately describe the role of clathrin in
neutrophils based on your understanding of phagocytosis?
A Clathrin tethers the antigen to the cytoskeleton.
B Clathrin opposes phagocytosis.
C Clathrin marks the antigen on the invading cell for phagocytosis by neutrophils.
D Clathrin stabilizes the inward facing surface of the plasma membrane, which engulfs
the antigen.
Solution The solution is (D). Clathrin is involved in the engulfing of the antigen once the
neutrophils bind to the target.
37 Mast cells produce signals that activate inflammation. Neutrophils are phagocytic white
blood cells. Monocytes are the largest type of white blood cell that engulf pathogens.
Based on the information provided in the chapter, which cell types produce endosomes?
A Monocytes and mast cells
B Neutrophils, monocytes, and mast cells
C Neutrophils and mast cells
D Neutrophils and monocytes
Solution The solution is (D). Neutrophils and monocytes produce endosomes for degrading
the pathogens. Mast cells are not phagocytic.

SCIENCE PRACTICE CHALLENGE QUESTIONS


5.2 Passive Transport
38 Membrane fluidity is influenced by the number of C-C double bonds (unsaturation) in the
hydrocarbon tails of the lipids composing cell membranes. Fluidity is also dependent on
temperature. The transit of materials through the cell membrane is controlled by fluidity.
To maintain homeostasis, all organisms, including the simple bacterium E. coli, must sense
the temperature of the environment and adapt to changes.

Advanced Placement Biology Instructor’s Solution Manual


5 | Structure and Function of Plasma Membranes 103

Samples of E. coli were grown at four different temperatures, and then researchers
determined the fatty acid composition of their plasma membranes. The data are shown in
the given table.
Fatty Acid 10 °C 20 °C 30 °C 40 °C
Myristic 17% 14% 14% 16%
Palmitic 18% 25% 29% 48%
Palmitoleic 26% 24% 23% 9%
Oleic 38% 34% 30% 12%
Ratio (U/S) Blank Blank Blank Blank

Fatty acid compositions of the plasma membrane of E. coli were incubated at the
temperatures shown. Myristic and palmitic acid are saturated, while palmitoleic and oleic
acids each have one C-C double bond.
A. Analyze the data to calculate the ratio of the fraction of unsaturated (U) to the fraction
of saturated (S) fatty acids in the plasma membrane, and complete the table.
B. Graph the ratio U/S versus growth temperature.
C. Explain the response of E. coli to the temperature of the environment.
D. We know that the temperature of the environment is sensed by E. coli through the
temperature-dependent confirmation of enzymes that convert a single bond in the lipid
tail to a double bond, and vice versa. Explain how the discovery of a mutant strain of
E. coli could lead to this insight.
Solution Teaching Tip—This question connects concepts drawn from Big Ideas 2, 4, and 1.

Advanced Placement Biology Instructor’s Solution Manual


104 5 | Structure and Function of Plasma Membranes

A. The ratio is calculated here as the sum of the percent unsaturated fatty acid
divided by the sum of the percent saturated fatty acid.
Fatty acid 10 °C 20 °C 30 °C 40 °C
Myristic 17% 14% 14% 16%
Palmitic 18% 25% 29% 48%
Palmitoleic 26% 24% 23% 9%
Oleic 38% 34% 30% 12%
Ratio (U/S) 1.8 1.5 1.2 0.32
B.

C. The organism has a composition of the plasma membrane in which unsaturation


increases with decreasing temperature. Homeostasis is maintained by response to
the environment.
D. A mutant was discovered in which the enzyme that converts a single bond to a
double is defective. This organism cannot synthesize unsaturated fatty acids;
therefore, it is not adapted for life at lower temperatures and dies.
39 Aquaporins, which allow for the movement of water across a cell membrane, are gated.
Both low and high pH within a plant cell can cause alterations of the membrane-spanning
protein. Describe the advantage of this feedback mechanism.
Predict how conditions of flooding or drought could activate this mechanism.
Solution To maintain the pressure within a plant cell, water concentration must be regulated.
When water is transported into the cell, the pH drops, and a negative feedback loop
changes the configuration of the membrane-spanning peptides; the gate closes.
When water concentrations with the cell are low, the pH is high, and the gate opens.
In flooding or drought conditions, the regulation cannot simply be reversed. Instead,
drought stress and flooding have been shown to induce (drought) and suppress
(flooding) the synthesis of proteins that provide the channel.

Advanced Placement Biology Instructor’s Solution Manual


5 | Structure and Function of Plasma Membranes 105

5.3 Active Transport


40 Rice plants grown in high-salt environments can actively transport sodium ions into the
vacuole by the antiporter movement of protons out of the vacuole. In a study aimed at
the development of salt-tolerant rye, researchers produced several varieties of transgenic
rye. Measurements of height and stem diameter for the transgenic varieties (TG1–TG4)
are compared with the wild-type varieties WT1 and WT2. Shown in the table are the
means and standard deviations from measurements of a very large sample size.
Variety Height (cm) Stem Thickness (cm)
WT1 9.667 ± 0.333 1.975 ± 0.095
WT2 11.867 ± 0.376 2.238 ± 0.204
TG1 15.420 ± 1.146 2.723 ± 0.261
TG2 15.600 ± 0.909 2.903 ± 0.323
TG3 14.925 ± 0.767 2.633 ± 0.073
TG4 16.100 ± 0.682 3.160 ± 0.169
A. Analyze the data. Are the heights and stem thicknesses in the transgenic plants
significantly different than in the wild-type plant? Justify your claim with evidence.
B. Are the heights and stem thicknesses among the transgenic plants significantly
different? Justify your claim with evidence.
C. Plants from which these data were taken were grown in 10 mM NaCl solutions. Pose
one question that researchers can investigate by growing the same varieties in a series of
lower salinity conditions.
D. The Na+/H+ antiporter is an active transport system. Briefly explain negative feedback
regulation of the movement of sodium into the vacuole of rye cells.
Solution A. In each transgenic variety, the height of the plant is significantly different from
the wild-type varieties. A comparison of the intervals of two measurements is not
significant if the intervals overlap. (More sensitive tests of significance are out of
scope on the AP Biology Exam.) Stem thickness is more complex. TG1 and WT2
nearly overlap, so stem thickness might not be significantly different for TG1. In each
of the other cases, one may be confident that the transgenic varieties have thicker
stems.
B. Overlap of the measurement intervals occurs for each, so they are not different.
C. Since the goal is the development of salt-tolerant rye plants, the question
underlying this project is probably, “Does introduction of the genes that express the
antiporter movement of sodium and hydrogen increase yield?” But a preliminary
question that can lead to a selection among the transgenic varieties might be, “Do
height or stem thickness depend on the salt concentration in the environment?”

Advanced Placement Biology Instructor’s Solution Manual


106 5 | Structure and Function of Plasma Membranes

D. An antiporter is bidirectional. As sodium diffuses into the vacuole, hydrogen ions


diffuse out along their concentration gradient. Hydrogen ions are pumped into the
vacuole by and ATPase-driven process. As the pH of cytoplasm falls, the antiport
process halts.

Advanced Placement Biology Instructor’s Solution Manual


6 | Metabolism 107

6 | METABOLISM
REVIEW QUESTIONS
1 Energy can be taken in as glucose, and then has to be converted to a form that can be
easily used to perform work in cells. What is the name of the latter molecule?
A Anabolic molecule
B Cholesterol
C Electrolyte
D Adenosine triphosphate
Solution The solution is (D). Sugars are stored in the body, usually as starch or glycogen.
Glucose forms the basic unit of these sugars, which serve as a source of energy. ATP
contains potential for a quick burst of energy that can be harnessed to perform
cellular work.
2 When cellular respiration occurs, what is the primary molecule used to store the energy
that is released?
A AMP
B ATP
C mRNA
D Phosphate
Solution The solution is (B). ATP is considered the energy currency of cells. It contains the
potential for a quick burst of energy that can be harnessed to perform cellular work.
3 DNA replication involves unwinding two strands of parent DNA, copying each strand to
synthesize complementary strands, and releasing the resulting two semiconserved
strands of DNA. Which description is accurate for this process?
A This process is anabolic.
B This process is catabolic.
C This process is both anabolic and catabolic.
D This process is metabolic, but it is neither anabolic nor catabolic.
Solution The solution is (A). This is an anabolic process. An anabolic process is one in which
simpler molecules combine to form complex molecules with input of energy.
Synthesis of new DNA strands from nucleic acid building blocks is an anabolic
process.

Advanced Placement Biology Instructor’s Solution Manual


108 6 | Metabolism

4 Which process is a catabolic process?


A Digestion of sucrose
B Dissolving sugar in water
C DNA replication
D RNA translation
Solution The solution is (A). Digestion of sucrose is a catabolic process because sucrose, being
a disaccharide, is a complex molecule. Its digestion produces the simpler compounds
glucose and fructose with the release of energy.
5 What food molecule used by animals for energy and obtained from plants is most directly
related to the use of sun energy?
A Glucose
B Protein
C Triglycerides
D tRNA
Solution The solution is (A). Glucose is synthesized by plants when light energy is converted
to chemical energy through the process of photosynthesis. Glucose is then passed
on to the food chain and is then used as a part of the animal cell’s cellular
respiration reactions, in which it is first broken down in the process of glycolysis to
make ATP, the chemical energy the cell uses to perform work. The image below
shows how plants absorb solar energy and use it to drive the process of
photosynthesis, thereby producing glucose and oxygen as by-products which are
then used for the animal cell’s cellular respiration needs.

Advanced Placement Biology Instructor’s Solution Manual


6 | Metabolism 109

6 What reaction will release the largest amount of energy to help power another reaction?
A AMP to ATP
B ATP to ADP
C DNA to proteins
D Glucose to starch
Solution The solution is (B). The hydrolysis of ATP is the reaction yielding the largest amount
of energy to help another power reaction. ATP hydrolysis occurs when a molecule of
ATP (the cell’s energy currency) reacts with a molecule of water, leading to the
transformation of ATP into ADP (adenosine triphosphate) and inorganic phosphate
(Pi). Several reactions in the cell are coupled with this hydrolysis reaction, thus
leading to a large number of reactions being powered by this reaction.
7 Consider a pendulum swinging. Which type(s) of energy is/are associated with the
pendulum in the following instances?
1. The moment at which it completes one cycle, just before it begins to fall back
toward the other end
2. The moment that it is in the middle between the two ends
3. Just before it reaches the end of one cycle, before step 1
A 1. Potential and kinetic
2. Potential and kinetic
3. Kinetic
B 1. Potential
2. Potential and kinetic
3. Potential and kinetic
C 1. Potential
2. Kinetic
3. Potential and kinetic
D 1. Potential and kinetic
2. Kinetic
3. Kinetic
Solution The solution is (C). The energy possessed by the pendulum after completion of one
cycle would be potential since it is at some elevation and energy is in stored form.
When the pendulum is in between the two ends, then it is in motion and thus
possesses kinetic energy.
8 Which of the following best describes energy?
A The transfer of genetic information
B The ability to assemble a large number of functional catalysts

Advanced Placement Biology Instructor’s Solution Manual


110 6 | Metabolism

C The ability to store solar output


D The ability to do work
Solution The solution is (D). The ability to do work or cause some change is called energy.
9 What is the ultimate source of energy on this planet?
A Glucose
B Plants
C Metabolic pathways
D The sun
Solution The solution is (D). The main source of energy for this planet is the sun. Solar energy
includes light, radio waves, and X-rays, all of which provide energy on Earth.
10 Which molecule is likely to have the most potential energy?
A ATP
B ADP
C Glucose
D Sucrose
Solution The solution is (D). Sucrose is a disaccharide molecule made of glucose and fructose.
The stored potential energy in it would be the greatest.
11 Which of the following is the best way to judge the relative activation energies between
two given chemical reactions?
A Compare the values between the two reactions.
B Compare their reaction rates.
C Compare their ideal environmental conditions.
D Compare the spontaneity between the two reactions.
Solution The solution is (B). The activation energy of a particular reaction determines the rate
at which it will proceed. The higher the activation energy, the slower the chemical
reaction will be. Thus, relative activation energies between two chemical reactions
can be judged by comparing their reaction rates.
12 Which term in the Gibbs free energy equation denotes enthalpy?
A
B
C
D

Advanced Placement Biology Instructor’s Solution Manual


6 | Metabolism 111

Solution The solution is (B). Total energy change in a system is called enthalpy and is denoted
by .
13 Which chemical reaction is more likely to occur?
A Dehydration synthesis
B Endergonic
C Endothermic
D Exergonic
Solution The solution is (D). Exergonic reactions can occur spontaneously, and free energy is
released from the reaction.
14 Which comparison or contrast between endergonic and exergonic reactions is false?
A Both endergonic and exergonic reactions require a small amount of energy to
overcome an activation barrier.
B Endergonic reactions have a positive and exergonic reactions have a negative
C Endergonic reactions consume energy, and exergonic reactions release energy.
D Endergonic reactions take place slowly, and exergonic reactions take place quickly.
Solution The solution is (A). Both endergonic and exergonic reactions require a small amount
of energy to overcome an activation barrier.
15 Is each system high or low entropy?
1. Perfume the instant after it is sprayed into the air
2. An unmaintained 1950s car compared with a brand new car
3. A living cell compared with a dead cell
A 1. Low
2. High
3. Low
B 1. Low
2. High
3. High
C 1. High
2. Low
3. High
D 1. High
2. Low
3. Low

Advanced Placement Biology Instructor’s Solution Manual


112 6 | Metabolism

Solution The solution is (A). The entropy in case (1) would be low, since the disorder of the
perfume is low immediately after it is sprayed; (2) would be high because, over time,
the disorder in a 1950s car would increase; and (3) would be low because a living cell
would have less disorder in comparison to dead cell.
16 What counteracts entropy?
A Energy release
B Endergonic reactions
C Input of energy
D Time
Solution The solution is (C). Input of energy lowers the entropy of the system and decreases
the disorder. Thus, input of energy counteracts high entropy and makes the system
stable.
17 What is the best example of the first law of thermodynamics?
A A body getting warmer after exercise
B A piece of fruit spoiling in the fridge
C A power plant burning coal and producing electricity
D An exothermic chemical reaction
Solution The solution is (C). A power plant burning coal and producing electricity is an
example of the first law of thermodynamics. Energy is transformed from one form to
another.
18 What is the difference between the first and second laws of thermodynamics?
A The first law involves creating energy, while the second law involves expending it.
B The first law involves expending energy, while the second involves creating it.
C The first law involves conserving energy, while the second law involves the inability to
recapture energy.
D The first law discusses creating energy, while the second law discusses the energy
requirement for reactions.
Solution The solution is (C). The first law of thermodynamics states that energy can neither
be created nor destroyed. It is just transformed from one state to other. The second
law of thermodynamics states that every energy transfer involves some loss of
energy in an unusable form, such as heat energy, resulting in a more disordered
system.
19 What best describes the effect of inputting energy into a living system?
A It decreases entropy within the system.
B It fuels catabolic reactions.

Advanced Placement Biology Instructor’s Solution Manual


6 | Metabolism 113

C It causes enthalpy.
D The energy is used to produce carbohydrates.
Solution The solution is (A). This is the best answer because inputting energy into a living
system will allow cells to perform a variety of functions that decrease entropy, such
as moving substances against their concentration gradient, repairing damage to cells
and molecules, and storing energy as ATP.
20 Why is ATP considered the energy currency of the cell?
A It accepts energy from chemical reactions.
B It holds energy at the site of release from substrates.
C t is a protein.
D It can transport energy to locations within the cell.
Solution The solution is (A). This is the best answer because inputting energy into a living
system will allow cells to perform a variety of functions that decrease entropy, such
as moving substances against their concentration gradient, repairing damage to cells
and molecules, and storing energy as ATP.

21 What is ATP made from?


A Adenosine + high-energy electrons
B ADP + pyrophosphate
C AMP + ADP
D The conversion of guanine to adenosine
Solution The solution is (B). An ATP molecule is made from the addition of pyrophosphate to
an ADP molecule, which contains two phosphate groups.
22 What is true about the energy released by the hydrolysis of ATP?
A It is equal to .
B The cell harnesses it as heat energy to perform work.
C It is primarily stored between the alpha and beta phosphates.
D It provides energy to coupled reactions.
Solution The solution is (D). When ATP is used in a reaction, it transfers its third phosphate
with energy to the chemical reaction. It phosphorylates another molecule and
makes the energy available. Also, ATP hydrolysis performs work in cells by energy
coupling.
23 What part of ATP is broken to release energy for use in chemical reactions?
A The adenosine molecule

Advanced Placement Biology Instructor’s Solution Manual


114 6 | Metabolism

B The bond between the first and second phosphates


C The bond between the first phosphate and the adenosine molecule
D The bond between the second and third phosphates
Solution The solution is (D). The terminal phosphate bond of ATP is broken upon hydrolysis
of ATP to ADP. This phosphate molecule, also referred to as the phosphate, is
linked to the other two phosphates of ATP via high-energy bonds called
phosphoanhydride bonds.

Advanced Placement Biology Instructor’s Solution Manual


6 | Metabolism 115

24 What does an allosteric inhibitor do?


A Binds to an enzyme away from the active site and changes the conformation of the
active site, increasing its affinity for substrate binding.
B Binds to an active site and blocks it from binding substrate.
C Binds to an enzyme away from the active site and changes the conformation of the
active site, decreasing its affinity for the substrate.
D Binds directly to the active site and mimics the substrate.
Solution The solution is (C). An allosteric inhibitor binds to an enzyme away from the active
site and changes the conformation of the active site, decreasing its affinity for the
substrate. An allosteric inhibitor is a molecule that binds to an enzyme, thus
inhibiting the enzyme’s activity. An allosteric inhibitor may be a competitive or
noncompetitive inhibitor.
25 What happens if an enzyme is NOT functioning in a chemical reaction within a living
organism that needs it?
A The reaction stops.
B The reaction proceeds, but much more slowly.
C The reaction proceeds faster without the interference.
D There is no change in the reaction rate.
Solution The solution is (B). The reaction proceeds, but much more slowly. If the enzyme
needed for the reaction is not be available, then the reaction will occur slowly,
potentially so slowly that an organism may die before the reaction is complete.
26 Which of the following is NOT true about enzymes?
A They increase the of reactions.
B They are usually made of amino acids.
C They lower the activation energy of chemical reactions.
D Each one is specific to the particular substrate, or substrates, to which it binds.
Solution The solution is (A). They increase of reactions. Enzymes have no effect on
of a reaction, that is, they do not change it even if a reaction is exergonic or
endergonic.
27 Which analogy best describes the induced-fit model of enzyme-substrate binding?
A A hug between two people
B A key fitting into a lock
C A square peg fitting through the square hole and a round peg fitting through the round
hole of a children’s toy

Advanced Placement Biology Instructor’s Solution Manual


116 6 | Metabolism

D The fitting together of two jigsaw puzzle pieces


Solution The solution is (B). The enzyme changes its configuration to bind to the transition
state of the substrate, which validates the lock and key analogy.
28 What is the function of enzymes?
A To increase the of reactions
B To increase the of reactions
C To lower the entropy of the chemicals in the reaction
D To lower the activation energy of a reaction
Solution The solution is (D). Enzymes are proteins that accelerate chemical reactions by
lowering their activation energies. As such, enzymes are catalysts of chemical
reactions, causing an increase in the rate at which reactions occur.

CRITICAL THINKING QUESTIONS


29 What is the connection between anabolic and catabolic chemical reactions in a metabolic
pathway?
A Catabolic reactions produce energy and simpler compounds, whereas anabolic
reactions involve the use of energy to make more complex compounds.
B Catabolic reactions produce energy and complex compounds are formed, whereas in
anabolic reactions, free energy is used by complex compounds to make simpler
molecules.
C Catabolic reactions use energy and gives simpler compounds, whereas in anabolic
reactions, energy is produced and simpler compounds are used to make complex
molecules.
D Catabolic reactions produce energy and water molecules, whereas in anabolic
reactions, this free energy is used by simpler compounds to make only proteins and
nucleic acids.
Solution The solution is (B). Catabolic reactions give out energy and simpler compounds,
whereas in anabolic reactions, free energy is used by simpler compounds to make
complex molecules. Anabolic reactions depend on energy to complete their
reactions. This energy is obtained as a result of catabolic reactions, so each one
leads to the next, with the energy coming from one to run the next one. A catabolic
reaction occurs when a complex compound breaks down, giving simpler molecules
with the release of energy. The energy released is used to power the anabolic
reactions, where simpler molecules use the energy to form complex molecules.
Therefore, one reaction leads to another.

Advanced Placement Biology Instructor’s Solution Manual


6 | Metabolism 117

30 Does physical exercise involve anabolic processes, catabolic processes, or both? Give
evidence for your answer.
A Physical exercise involves both catabolic and anabolic processes. Glucose is broken
down into simpler compounds during physical activity. The simpler compounds are
then used to provide energy to the muscles for contraction by the anabolic pathway.
B Physical exercise is just a catabolic process. Glucose is broken down into simpler
compounds during physical activity, and the simpler compounds are then used to
provide energy to the muscles for contraction.
C Physical activity involves only anabolic processes. Glucose is broken down into simpler
compounds during physical activity, and the simpler compounds are then used to
provide energy to the muscles for contraction by anabolic pathways.
D Physical exercise involves both anabolic and catabolic processes. Cellulose is broken
down into simpler compounds during physical activity. The simpler compounds are
then used to provide energy to the muscles for contraction by anabolic pathways.
Solution The solution is (A). Physical exercise involves both catabolic and anabolic reactions.
Glucose must be broken down in metabolic pathways, which are both catabolic and
anabolic, and used to supply energy to muscles for contraction. Glucose is broken
down to provide energy to the body. This is a catabolic process, and the simpler
molecules use that energy to form complex compounds. This is an anabolic pathway.
31 How do chemical reactions play a role in energy transfer?
A Energy from the breakdown of glucose and other molecules in animals is released as
ATP, which transfers energy to other reactions.
B Energy from the breakdown of glucose and other molecules in animals is released in
the form of NADP, which transfers energy to other reactions.
C Energy is released in the form of glucose from the breakdown of ATP molecules. These
ATP molecules transfer energy from one reaction to other.
D Energy is released in the form of water from the breakdown of glucose. These
molecules transfer energy from one reaction to other.
Solution The solution is (A). Energy must be stored in a cell in the form of a bond between
two molecules; it cannot be stored in the cell as free energy. The energy is attached
to a chemical such as ATP, which transfers the energy from one reaction to another
and from one area of a cell to another safely. Energy never exists in the living cells as
free energy. These ATP molecules are obtained from the metabolism of glucose.
32 What are two cellular functions that require energy?
A Phagocytosis helps amoebae take up nutrients, and pseudopodia help the
amoebae move.
B Phagocytosis allows amoebae to move, and pseudopodia help in the uptake of
nutrients.

Advanced Placement Biology Instructor’s Solution Manual


118 6 | Metabolism

C Phagocytosis helps amoebae take up nutrients, and cilia help amoebae move.
D Phagocytosis helps amoebae in cell division, and pseudopodia help amoebae move.
Solution The solution is (A). Phagocytosis helps amoebae in uptake of nutrients, and
pseudopodia help the amoebae move. Amoebae take up some nutrients through
phagocytosis and combine the resulting vacuoles with lysosomes to digest materials,
as humans do. Amoebae move by constricting actin and myosin within the cell, just
as human muscles move. The protrusion in cytoplasm that helps in movement is
called pseudopodia.
33 What is the conversion of energy that takes place when the sluice of a dam is opened?
A Potential energy stored in the water held by the dam will convert to kinetic energy
when the water falls through the opening of the sluice.
B Kinetic energy stored in the water held by the dam will convert to potential energy
when the water falls through the opening of the sluice.
C Potential energy stored in the water held by the dam will convert to electrical energy
when the water falls through the opening of the sluice.
D Hydrothermal energy stored in the water held by the dam will convert to kinetic
energy when the water falls through the opening of the sluice.
Solution The solution is (A). The potential energy of the water held in the reservoir behind
the dam converts to kinetic energy as it falls through the opening of the sluice.
Potential energy is referred to the energy stored in a system due to its position.
Therefore, water held in a dam possesses potential energy, whereas kinetic energy is
referred to as energy possessed by the motion of the object. When the sluice of the
dam is opened, the potential energy is converted to kinetic energy.
34 What is the difference between a spontaneous reaction and one that occurs
instantaneously?
A A spontaneous reaction is one that releases free energy and moves to a more stable
state. Instantaneous reactions occur rapidly with a sudden release of energy.
B A spontaneous reaction is one that uses free energy and moves to a more stable state.
Instantaneous reactions occur rapidly with a sudden release of energy.
C A spontaneous reaction is one that releases free energy and moves to a more stable
state. Instantaneous reactions occur rapidly within a system by the uptake of energy.
D A spontaneous reaction is one that occurs rapidly with the sudden release of energy.
Instantaneous reactions release free energy and move to a more stable state.
Solution The solution is (A). A spontaneous reaction occurs without the input of energy and
decreases the free energy of a system, making the system more stable and releasing
that energy to do work. An instantaneous reaction occurs very rapidly with a sudden
release of energy. A spontaneous reaction does not require energy to carry out the
reaction. It releases free energy that is used by the system to move to a stable state.

Advanced Placement Biology Instructor’s Solution Manual


6 | Metabolism 119

The difference lies in the duration of reactions: spontaneous reactions can take
seconds to years, while instantaneous reactions occur right away.
35 Which option describes the position of the transition state on a vertical energy scale, from
low to high, relative to the position of the reactants and products, for both endergonic
and exergonic reactions?
A The transition state of the reaction exists at a lower energy level than the reactants.
Activation energy is always positive, regardless of whether the reaction is exergonic or
endergonic.
B The transition state of the reaction exists at a higher energy level than the reactants.
Activation energy is always positive, regardless of whether the reaction is exergonic or
endergonic.
C The transition state of the reaction exists at a lower energy level than the reactants.
Activation energy is always negative, regardless of whether the reaction is exergonic
or endergonic.
D The transition state of the reaction exists at an intermediate energy level than that of
the reactants. Activation energy is always positive, regardless of whether the reaction
is exergonic or endergonic.
Solution The solution is (B). The transition state of the reaction exists at a higher energy state
than the reactants and, thus, activation energy is always positive regardless of
whether the reaction is an exergonic or endergonic reaction.
36 Imagine an elaborate ant farm with tunnels and passageways through the sand where
ants live in a large community. Now imagine that an earthquake shook the ground and
demolished the ant farm.
In which scenario, before or after the earthquake, was the ant farm system in a state of
higher or lower entropy? Why?
A The ant farm is in the state of higher entropy after the earthquake, and energy must
be spent to bring the system to low entropy.
B The ant farm is in the state of lower entropy after the earthquake, and energy must be
spent to bring the system to high entropy.
C The ant farm is in the state of higher entropy before the earthquake, and energy is
given out of the system after the earthquake.
D The ant farm is in the state of lower entropy before the earthquake, and energy is
given out of the system after the earthquake.
Solution The solution is (A). The ant farm is in a state of higher entropy (disorder) after the
earthquake. The tunnels have been destroyed, and energy must be spent to rebuild
them. Entropy is defined as the measure of randomness or disorder. Therefore, after
the earthquake, the randomness or disorder increases, leading to higher entropy.

Advanced Placement Biology Instructor’s Solution Manual


120 6 | Metabolism

Energy should be given to the system to rebuild the tunnels and passageways to
lower the entropy.
37 Energy transfers take place constantly in everyday activities. Think of two scenarios:
cooking on a stove and driving. How does the second law of thermodynamics apply to
these scenarios?
A Heat is lost into the room while cooking and into the metal of the engine during
gasoline combustion.
B Heat gained while cooking helps to make the food, and heat released due to gasoline
combustion helps the car accelerate.
C The energy given to the system remains constant during cooking, and more energy is
added to the car engine when the gasoline combusts.
D The energy given to the system for cooking helps to make food, and energy in the car
engine remains conserved when gasoline combustion takes place.
Solution The solution is (A). In both examples, there is an input of energy that results in work
being done—cooking and moving the car—and loss of heat as a result. The heat loss
travels into the room during cooking and into the metal of the engine during
gasoline combustion. Energy must be continuously put into the systems to maintain
the activities. Run out of natural gas or propane, and the cooking stops. Run out of
gasoline, and the car stops. The second law of thermodynamics states that no
energy transfer is efficient, and some form of energy in the form of heat is released.
While cooking, heat is lost and travels into the room. The engine metal heats up
during gasoline combustion. If propane or gas runs out, then both of the activities
would stop.
38 What does it mean for a system to be at a higher level of entropy? How can it be
reduced?
A Higher level of entropy refers to a higher state of disorder in the system, and it can be
reduced by input of energy to lower the entropy.
B Higher level of entropy refers to a higher state of symmetry in the system, and it can
be reduced by the release of energy to lower the entropy.
C Higher level of entropy refers to low disorder in the system, and it can be reduced by
input of energy to increase the entropy.
D Higher level of entropy refers to a higher state of disorder in the system, and it can be
reduced by providing a catalyst to lower the entropy.
Solution The solution is (A). The system has lost energy and is in a higher state of disorder.
Energy or work must be invested into the system to restore order and lower
entropy. When a system is in a state of disorder, then it possesses high entropy. The
entropy can be reduced by input of energy, which reduces the randomness and
lowers the entropy of the system.

Advanced Placement Biology Instructor’s Solution Manual


6 | Metabolism 121

39 When the air temperature drops and rain turns to snow, which law of thermodynamics is
exhibited?
A First law of thermodynamics
B Second law of thermodynamics
C Third law of thermodynamics
D Zeroth law of thermodynamics
Solution The solution is (A). This is an example of the first law of thermodynamics. Energy is
being transferred as the atmosphere gets colder and the water in rain solidifies into
snow. The first law of thermodynamics states that energy can neither be created nor
destroyed. It is transferred from one state to another. Energy is being transferred as
the atmosphere gets colder and the rainwater solidifies into snow.
40 How does ATP supply energy to chemical reactions?
A ATP dissociates, and the energy released by breaking a phosphate bond within ATP is
used for phosphorylation of another molecule. ATP hydrolysis also provides energy to
power coupling reactions.
B ATP uses energy to power exergonic reactions by hydrolysis of ATP molecules. The
free energy released as a result of ATP breakdown is used to carry out metabolism of
products.
C ATP uses energy to power endergonic reactions by dehydration of ATP molecules. The
free energy released as a result of ATP breakdown is used to carry out metabolism of
products.
D ATP uses the energy released from coupling reactions, and that energy is used to
power the endergonic and exergonic reactions.
Solution The solution is (A). ATP transfers its third phosphate with its energy to the chemical
in the reaction, phosphorylating that chemical and making the energy available.
When ATP is used in a reaction, it transfers its third phosphate with energy to the
chemical reaction. It phosphorylates another molecule and makes the energy
available. Also, ATP hydrolysis performs work in cells by energy coupling.
41 Is the EA for ATP hydrolysis relatively low or high? Explain your reasoning.
A EA for ATP hydrolysis is low because considerable energy is released.
B EA for ATP hydrolysis is high because considerable energy is released.
C EA for ATP hydrolysis is intermediate because considerable energy is released.
D EA for ATP hydrolysis is high because a low amount of energy is released.
Solution The solution is (B). The EA of ATP hydrolysis is high due to the high energy level of
the compound and the ultimate energy release.

Advanced Placement Biology Instructor’s Solution Manual


122 6 | Metabolism

42 What is phosphorylation as it occurs in chemical reactions?


A The attachment of a phosphate to another molecule to facilitate a chemical reaction
B The uptake of a phosphorous molecule by an ATP molecule to power chemical
reactions
C The release of a third phosphorous molecule of ATP during hydrolysis
D The breakdown of a pyrophosphate molecule which gives phosphate ions
Solution The solution is (A). Phosphorylation is the attachment of a high-energy phosphate
from ATP to another molecule to facilitate a chemical reaction.
43 If a chemical reaction could occur without an enzyme, why is it important to have one?
A Enzymes are important because they give the desired products only from the reaction.
B Enzymes are important because the products are obtained consistently with time.
C Enzymes are important because they do not disturb the concentration of the
products.
D Enzymes are important because energy remains conserved and no loss of energy
occurs.
Solution The solution is (A). Chemical reactions can occur without an enzyme catalyzing them,
but not at a rate consistent with life. Enzymes speed up reactions that allow
chemicals to be available for an organism’s growth and maintenance. However,
enzymes are also very specific to their host molecules. This ensures that only
reactions whose products are needed will be catalyzed.
44 How does enzyme feedback inhibition benefit a cell?
A Feedback inhibition benefits the cell by blocking the production of the products
by changing the configuration of enzymes. This will prevent the cells from
becoming toxic.
B Feedback inhibition benefits the cell by blocking the production of the reactants
by changing the configuration of enzymes. This will prevent the cells from
becoming toxic.
C Feedback inhibition benefits the cell by blocking the production of the products
by changing the configuration of reactants. This will prevent the cells from
becoming toxic.
D Feedback inhibition benefits the cell by blocking the production of the products
by reducing the reactants. This will prevent the cells from becoming toxic.
Solution The solution is (A). Feedback inhibition allows the cell to maintain a relatively
constant level of a product without having to degrade the enzyme. The products
bind to the allosteric site of the enzyme and change its configuration. The reaction
eventually stops. This way, overproduction of the product does not occur, which
would be toxic.

Advanced Placement Biology Instructor’s Solution Manual


6 | Metabolism 123

45 What type of reaction allows chemicals to be available for an organism’s growth and
maintenance in a timely manner?
A Enzymatically facilitated reactions
B Redox reactions
C Catabolic reactions
D Hydrolysis of ATP
Solution The solution is (A). Enzymatically facilitated reactions occur at a speed that brings
reactants together to provide the biochemical molecules needed for life. The
reactions would occur without the enzymes, but not fast enough to supply the
products needed by cells. Enzymes catalyze a reaction and accelerate the rate of
chemical reaction. Therefore, the products are made in timely manner and can be
available on time for an organism’s growth and maintenance.

TEST PREP FOR AP® COURSES


46 Cell metabolism is a complex process that uses many types of chemicals in a variety of
processes. Which statement is true?
A A loss of free nucleotides would result in cancer.
B A loss of assorted carbohydrates would result in mitosis.
C A loss of triglycerides would result in cell death.
D A loss of enzymes would result in cell death.
Solution The solution is (D). A lack of enzymes would result in an extreme slowdown of
metabolic pathways because reactions would only be able to occur spontaneously.
Since reactions occurring spontaneously do not necessarily occur quickly, the cell
would die as necessary processes slowed or halted.
47 Which pair of descriptors of chemical reactions go together?
A Anabolic and exergonic
B Exergonic and dehydration synthesis
C Endergonic and catabolic
D Hydrolysis and exergonic
Solution The solution is (D). A hydrolysis reaction involves addition of water to break bonds,
thus liberating energy.
48 What is the underlying principle that supports the idea that all living organisms share the
same core processes and features?
A All organisms must harvest energy from their environment and convert it to ATP to
carry out cellular functions.
B Plants produce their own energy and pass it on to animals.

Advanced Placement Biology Instructor’s Solution Manual


124 6 | Metabolism

C Herbivores, carnivores, and omnivores coexist for the survival of all.


D Glucose is the primary source of energy for all cellular functions.
Solution The solution is (A). All organisms must harvest energy from their environment and
convert it to ATP to carry out cellular functions. It is a requirement of life that
organisms metabolize energy.
49 It has been accepted that life on Earth started out as single-celled, simple organisms,
which then evolved into complex organisms. How did evolution proceed to produce such
a wide variety of living organisms from a simple ancestor?
A Prokaryotes produced the fungi, then the protists, which then branched to plants and
animals.
B Protists evolved first, then the prokaryotes, which branched into the fungi, plants, and
animals.
C Prokaryotes produced the protists, which branched into the fungi, plants, and animals.
D Prokaryotes produced the protists, then the fungi, which branched into plants and
animals.
Solution The solution is (C). Prokaryotes produced the protists, which branched into the
fungi, plants, and animals. Single-celled organisms that are the prokaryotes
appeared first on Earth. They produced multicellular organisms called protists. The
protists further branched into fungi, plants, and animals.
50 Plants make glucose through a pathway called photosynthesis. The amount of energy
captured from light can be expressed as the number of energy-containing molecules used
to make one molecule of glucose.
Which option best states the number of each molecule needed?
A 54 molecules of ATP and 18 molecules of NADPH
B 18 molecules of ATP and 12 molecules of NADPH

Advanced Placement Biology Instructor’s Solution Manual


6 | Metabolism 125

C 24 molecules of ATP and 18 molecules of NADPH


D 12 molecules of ATP and 18 molecules of NADPH
Solution The solution is (B). The amount of energy needed to make one molecule of glucose
from 6 molecules of carbon dioxide is 18 molecules of ATP and 12 molecules of
NADPH, or a total of 54 molecules of ATP.
51 What is an anabolic pathway? What is an example of an anabolic pathway used by cells in
their metabolism?
A Anabolic pathways involve the breakdown of nutrient molecules into usable forms. An
example is the harvesting of amino acids from dietary proteins.
B Anabolic pathways involve the breakdown of nutrient molecules into usable forms. An
example is the use of glycogen by the liver to maintain blood glucose levels.
C Anabolic pathways build new molecules out of the products of catabolic pathways. An
example is the separation of fatty acids from triglycerides to satisfy energy needs.
D Anabolic pathways build new molecules out of the products of catabolic pathways. An
example is the linkage of nucleotides to form a molecule of mRNA.
Solution The solution is (D). Anabolic pathways build new molecules out of the products
of catabolic pathways. An example is the linkage of nucleotides to form a molecule
of mRNA.
52 If glucose is broken down through aerobic respiration, a number of ATP molecules can be
made from the energy extracted. How many molecules of ATP are possible?
A 2 to 4
B 36 to 38
C 10 to 12
D 24 to 30
Solution The solution is (B). When glucose is broken down through aerobic respiration, 36–
38 molecules of ATP can be produced. Two molecules are obtained from the Krebs
cycle, and 34 molecules of ATP are produced from the electron transport chain.
53 Plants must have adequate resources to complete their functions. If they do not have
what they need, there are changes in the organism’s metabolism. What happens to the
metabolism of a plant that does not have adequate sunlight?
A Photosynthesis slows and less glucose is produced for energy use.
B The plant switches to anaerobic metabolism.
C The plant goes into a dormant state until the sunlight returns.
D The plant flowers quickly to reproduce while it can.

Advanced Placement Biology Instructor’s Solution Manual


126 6 | Metabolism

Solution The solution is (A). Photosynthesis slows and less glucose is produced for energy use.
Sunlight is an extremely important requirement for the plants to carry out the
photosynthesis reaction and manufacture their food. When the sunlight is not
adequate, the rate of photosynthesis decreases and, as a result, less glucose is
produced for energy use.
54 Water deficiency is arguably the easiest deficiency to detect in plants. This is because
plants that lack water will wilt since water within the plant’s cells helps to support the
plant’s weight. Plant cells become water deficient because their cells use the water for
metabolic processes.
What happens to the metabolism of a plant that does NOT have adequate water?
A Photosynthesis is inhibited, less glucose is produced, and water used by the cells is not
replaced.
B The plant increases its breakdown of glucose to create more water at the end of the
process.
C The plant will stop photosynthesizing for long periods of time until it has enough
water to do so.
D The cell will bring in more carbon dioxide to compensate for the lack of water,
allowing glucose synthesis to continue.
Solution The solution is (A). Photosynthesis is inhibited, less glucose is produced, and water
that leaves the cells is not replaced, causing wilting of the plant. When adequate
water is not present, photosynthesis will decrease or stop altogether. This is because
water is an important reactant in the photosynthesis reaction. As a result, the
product of the photosynthesis reaction, glucose, will decrease. Wilting of plants will
occur as a result of diminished water in the cells.
55 Enzymes facilitate chemical reactions that result in changes to a substrate. How does the
induced-fit model of enzymes and substrates explain their function?
A Both enzymes and substrates undergo dynamic changes, inducing the transitions state
of the substrate.
B The enzyme induces a change in the substrate but is not changed itself during the
reaction.
C The substrates attach to the enzyme, and the chemical reaction proceeds.
D The enzyme changes shape to fit the substrate, causing the transition state to occur.
Solution The solution is (A). Both enzymes and substrates undergo dynamic changes, inducing
the transition state of the substrate. When the enzyme and substrate come
together, their interaction causes a mild shift in the enzyme’s structure. This
confirms an ideal binding arrangement between the enzyme and the transition state
of the substrate.

Advanced Placement Biology Instructor’s Solution Manual


6 | Metabolism 127

56 Enzyme inhibitors play an important part in the control of enzyme functions, allowing
them to continue, or inhibiting them for a period of time. Which inhibitor affects the
initial rate, but does NOT affect the maximal rate?
A Allosteric
B Competitive
C Noncompetitive
D Uncompetitive
Solution The solution is (B). The effect of competitive inhibition is to increase the substrate
concentration required to achieve a given reaction speed. So, the rate of the
reaction will increase.

SCIENCE PRACTICE CHALLENGE QUESTIONS


6.1 Energy and Metabolism
57 Activation energy is required for a reaction to proceed, and it is lower if the reaction is
catalyzed. Sucrose (table sugar) is a disaccharide. When we eat sucrose, it is converted to
carbon dioxide and water, as with other carbohydrates.
1. Identify whether the breakdown of sucrose is endergonic or exergonic. Explain
the reasoning for your identification.
2. Based on your identification, explain whether cubes of sugar can be stored in a
sugar bowl, by creating a diagram similar to Figure 6.10.
3. If table sugar is placed in a spoon held over a high flame, the sugar is charred and
becomes a blackened mixture composed primarily of carbon. Create a visual
representation that includes a chemical equation to explain the role of the flame
in this process.
4. In terms of your answers to questions 1‒3, predict whether sugar cubes in a
bowl placed in a dish of water can be stored on a table, and justify your
prediction.
5. [Extension] The energy of activation of a chemical reaction can be determined
by measurement of the effect of temperature on reaction rate. The natural
logarithm of the reaction rate constant is a linear function of the inverse of the
temperature in Kelvin degrees. The negative of the slope of that graph is the
energy of activation divided by the universal ideal gas constant,
. Using the following data (R. Wolfenden and Yang Yean,
Journal of the American Chemical Society, 2008 Jun 18; 130(24): 7,548–7,549),
evaluate the energy of activation of the following reaction:

Advanced Placement Biology Instructor’s Solution Manual


128 6 | Metabolism

sucrose → fructose + glucose


Temperature (K) ln(rate)
440
423
403
388
A. Construct a graph of ln(rate) versus 1/T(K) and determine the energy of activation for
the uncatalyzed reaction.
B. Based on the data, explain the importance of enzymes for time scales characteristic of
living systems on Earth—that is to say, life as we know it.
The time scale required for half of the molecules of initial sucrose to remain can be
estimated. The relationship between the half-life and the activation energy is:

At a temperature of 300 K, approximately room temperature, RT is equal to 2,494 J/mole.


Solution Sample answer:
1. The breakdown of sucrose to fructose and glucose is an exergonic process.
One could calculate the free-energy change and find that it was negative.
One may also note that this is an important process in digestion in which
higher free energy is degraded to lower states, which are then converted to
even lower free-energy states; respiration is a cascade. While there are
chemical reactions within respiration that are endergonic, they must be
coupled to exergonic reactions. One may misconceive that a process such as
the degradation of sucrose, which in living systems is catalyzed by an
enzyme, is endergonic and coupled to the exergonic configuration changes in
the catalytic protein. However, the protein configuration is restored,
returning to its own initial free-energy state, cyclically. So, while the catalyst
changes the kinetics (the rate) of the reaction, it does not drive the reaction
forward.
2. This means that if the sugar cube can be isolated from biological systems that
could catalyze the conversion, the sugar cube would be stable. The diagram
in Figure 6.10 would then represent the free energy of the sucrose sugar
cube at the left and the glucose and fructose products at the right.
Separating these is a free-energy barrier.
3. If we place the sugar cube on a teaspoon and expose it to a flame, the energy
provided by the flame will allow the system to move to another, lower free-
energy state consisting of a mixture of carbon, carbon monoxide, carbon
dioxide, and water.

Advanced Placement Biology Instructor’s Solution Manual


6 | Metabolism 129

4. If we place the sugar cube in a glass of water and allow microbial life to
participate, then the sugar becomes a nutrient. The catalytic power of the
proteins expressed by the microbes will allow the system to move to the
lower state.
5. A graph can be constructed from the data according to the model suggested.
-3.00
0.0022 0.0023 0.0024 0.0025 0.0026
-3.50
1/T(K)
-4.00
f(x) = − 6845.9987564527 x + 11.7688497411029
-4.50
ln(rate)
-5.00
-5.50
-6.00
A. Multiplying the slope by 8.314 J/molK gives an energy of activation of 57 kJ/mole.
B. We can conclude from the data that this is a very large energy of activation for
transformation and that the time scale characteristic of life as we know it would not
be consistent with respiration conducted by uncatalyzed processes.
58 Physical exercise involves both anabolic and catabolic processes. For each process,
explain an expected outcome and describe an example of a specific exercise that can lead
to the expected outcome.
Solution Sample answer: Physical exercise can lead to weight loss through the respiration of
energy stored in tissues. Exercise can also lead to the synthesis of new tissue and
likely weight gain. The former process is catabolic, and the latter is anabolic.
6.2 Potential, Kinetic, Free, and Activation Energy
59 Explanations in science are often constructed by analogy. Explanations of the behavior of
a poorly understood phenomenon can often be constructed by analogy to a phenomenon
that is well understood. For each of the following cellular functions that require free
energy, describe a parallel human activity and identify a source of free energy for that
activity. For example, the synthesis of proteins can be expected to proceed as an
assembly of a small set of subcomponents, just as the construction of a building is
accomplished by gathering and joining materials. It is consistent with our analogy to
expect that there must be a free-energy resource that is consumed in the synthesis of
proteins, just as hydrocarbon fuels are a source of energy for the construction of a
building.

Advanced Placement Biology Instructor’s Solution Manual


130 6 | Metabolism

Solution Students might propose one of the following analogies to guide the understanding
of the behavior of a biological system:
 Storing and retrieving genetic information: We expect the storage of
biological information to be analogous to the storage of cultural data, such as
texts, in which information is contained in the arrangement of
subcomponents, such as letters, numbers, notes, or colors. Though the
subcomponents of the information may be few, the retrieval of cultural data
can lead to nearly infinite forms of expression, and the process of data
retrieval is complex and elegant. Likewise, arrangements of a very small
number of nucleotides can be translated into the diversity of life.
 Sending chemical messages: We expect the coordination of subsystems
within an organism to be accomplished by messaging. We expect the
subcomponents of the messages to be simple and highly redundant. We
expect the sequence and timing of messages to be as important as the
information provided by the message. All of these features are characteristic
of human communication systems and of the cell-cell signaling that
coordinates cell behavior and integrates tissue systems.
 Digesting food: Harvesting, preparing, cooking, and delivering food involves
multiple specialized actors in coordinated action. We expect metabolism to
be an analogous process of energy harvesting and delivery. The output from
each step has coevolved with its input. Some processes, such as hunting and
gathering, have relatively low efficiency, but are less demanding in terms of
specialization and coordination. In some processes, highly organized physical
structures are required by the system. By analogy, glycolysis has a lower
efficiency than the Krebs cycle, which involves the complex structure of the
mitochondria.
 Regulating transport: Transport of materials can involve containerized
passage through a system of locks in which free energy is consumed. This is
analogous to the conformational changes of a protein spanning a cell
membrane. At the other end, less complex materials can simply leak through
a barrier. And between these are the porins whose channels are like streams
that may be dammed.

Advanced Placement Biology Instructor’s Solution Manual


6 | Metabolism 131

6.3 The Laws of Thermodynamics


60 Each process in the figure shows examples of endergonic and exergonic processes.

1. Identify each process as endergonic or exergonic and provide reasoning for your
identification that includes your definition of the system.
2. For each process, does entropy increase or decrease? Explain your reasoning in
terms of changes in the amount of order within the system.
3. For each process, is there an input of energy? Explain your reasoning in terms of
(a) the source of the energy input into the system and (b) the interaction
between the system and its environment that provides that input of energy.
Solution There are many answers to these questions, and the answers depend on how one
defines the system.
The photo shows a compost pile. (1) If we consider the contents bound by the
wooden box to be the system and the process to be the decomposition of the
compost, then the system is exergonic. Although the process of photosynthesis is
endergonic through coupling to radiant energy, the respiration processes that
dominate the behavior of the system are all leading to the decomposition of
complex organic structures to produce much less ordered products—carbon dioxide
and water. (2) The entropy of the system increases as order decreases. (3) If the
compost pile is immersed in cold air for a sufficient period of time, respiration might
cease. Gardeners do work on the system to keep that from happening during the
winter. They frequently stir the compost pile to maintain conditions that support
microbial life. If the compost pile becomes too warm, it can also create unstable
conditions. So, without energy inputs, the compost pile may not be self-sustaining.
Even without the work done on the system by the gardener, there is an energy
exchange through heating between the compost pile and its surroundings.
The photo shows chicks hatching. (1) and (2) Considering the chick to be the system,
the system is strongly exergonic. The new tissue that is rapidly growing is at a much

Advanced Placement Biology Instructor’s Solution Manual


132 6 | Metabolism

higher free energy than the nutrient materials within the shell. Considering the shell
to be the system, the process is clearly endergonic; the shell must be broken by that
beak, and the highly symmetric shell is more ordered than the shell fragments.
Considering the chick and the shell to be the system, the system is exergonic; we
know that the process will advance as free energy of the shell and its nutrients are
coupled to the free energy of the growing chick. (3) However, without heating, the
system may not continue to function, so changes in S are not sufficient.
The photo shows sand art. (1) and (2) If we consider the ordered ridges relative to
the planar surface to be the system, then it is highly ordered, and, as we are told in
the caption, this order is in the process of degrading. In the transformation from the
ordered to the disordered state, the products have a lower free energy than the
reactants. The system is exergonic even if a barrier prevented wind, water, or other
environmental disruptions the system. The barrier would prevent kinetic and not
thermodynamic effects, and the system would ultimately degrade, though very
slowly. (3) The possibility of spontaneous disorder, even in the sealed system, shows
that no energy input is needed to drive the process.
The photo shows spheres rolling down a hill. Considering a sphere to be the system,
(1) and (2,) we see that the process is spontaneous only if (3) there is an input from
the gravitational work done on the sphere. So, for that system, the process of rolling
is endergonic. Considering a sphere and the Earth to be the system, we see that the
process is a spontaneous rearrangement of parts within the system; the center-of-
mass of the sphere-Earth system spontaneously decays from the slightly high free-
energy state of the sphere at the top of the hill. But (3) there is no work done on the
system after the sphere has been placed at the top of the hill.
61 Energy transfers occur constantly in daily activities. Think of two scenarios: cooking on a
stove and driving a car. For each scenario, describe the system and explain how the
second law of thermodynamics applies to the system in terms of energy input and change
in entropy.
Solution Sample answer: Cooking on a stove involves the transfer of energy as heating from
combustion of fuel to pan and then from the pan to contents of the pan. Considering
the pan and then the contents of the pan to be the system, energy is added to the
system from the surroundings through heating. In this process, the transport of heat
energy creates atomic-scale disorder as the speeds of atomic increase and the
correlations of their motions decrease. Driving a car is very different. Considering
the car to be the system, the entropy of the system decreases as the speed and
direction of the system become more ordered. The second law is consistent with
this behavior since the entropy of the surroundings increases though the
decomposition of high-energy fuels to low free-energy exhaust. In fact, we
have become aware that the entropy production of driving is a central challenge
of our era.

Advanced Placement Biology Instructor’s Solution Manual


6 | Metabolism 133

62 Consider a simple process that illustrates the change in entropy when energy is
transferred. Take a block of ice as a system with a temperature of 0 °C. This is water as a
solid, so it has a high structural order. This means that the molecules are in a fixed
position. As a result, the entropy of the system is low.
1. Take a block of ice as a system with a temperature of 0 °C. This is water as a
solid, so it has a high structural order. This means that the molecules are in a
fixed position. As a result, the entropy of the system is low.
2. Allow the ice to melt at room temperature. Describe changes in the motion and
interactions of water molecules before and after melting. Explain where the
energy came from whose transfer produced melting. Predict the effect of the
energy transfer on the entropy on the system, and justify your prediction.
3. Heat the water until the temperature reaches the boiling point. Explain what
happens to the entropy of the system when the water is heated.
4. Continue to heat the water at the constant temperature of the boiling point.
Describe changes in the motion and interactions of water molecules before and
after boiling. Predict the effect of the energy transfer on the entropy of the
system, and justify your prediction.
5. [Extension/Connection] Molecules of water have simple responses to heating:
The molecules move faster and interact less strongly with other neighboring
molecules. Consider the primary producers of an aquatic ecosystem in summer.
Describe the source of energy transfer to the system of photosynthetic plants
and algae. Predict changes in the system in response. Explain what happens to
the entropy of this trophic level when energy transfer occurs. Now consider the
primary producers and their aqueous environment as the system. Explain what
happens to the entropy of this system composed of photosynthetic organisms
and their abiotic environment.
6. Predict the change in entropy of the system when both autotrophs and their
abiotic environment are considered. Justify your prediction. Predict the signs of
the entropy changes in both biotic and abiotic components of this system.
Predict the relative magnitudes of these entropy changes, and justify your
prediction.
Solution Sample answer:
1. N/A
2. Heating causes the molecules to become less aligned and slightly decrease
the volume in which they are constrained to move; liquid water has a slightly
higher density that water ice. However, as melting occurs, there is no change
in average speed since melting occurs at constant temperature. The more
disordered system has a higher entropy and the increase is paid through
transfer of energy from the surrounding environment.

Advanced Placement Biology Instructor’s Solution Manual


134 6 | Metabolism

3. As the system continues to gain heat energy from the surroundings, the
speeds of the molecules and disorder increase, so the entropy of the system
increases.
4. When the boiling temperature is reached, further energy transport through
heating causes a second change of phase and the system is vaporized.
Increasing disorder and increasing volume increase the entropy of the
system. Collisions with other molecules broaden the distribution of kinetic
energies of molecules within the system, but the average temperature
remains unchanged until the system is entirely vaporized.
5. Energy is transferred to the photosynthetic system primarily as sunlight. The
response of the system is the synthesis of materials, reproduction, and
growth. Order increases, and so does entropy. If we include the abiotic
environment, water, and atmospheric gases, it must be that the entropy
decreases. This is a requirement of the second law of thermodynamics.
6. The magnitude of the entropy increase in the abiotic environment must be
larger than the magnitude of the entropy decrease in the biotic environment.
Otherwise, the behavior of the system overall violates the second law. Some
students may suggest that since it is a law and laws are made to be broken,
this provides insufficient evidence. Remind them that in a system, “law” has
a different meaning: laws are never broken, by definition. However, scientists
are always trying to find ways to break scientific laws and disprove theories.
When this occurs, a new theory that increases our understanding of the
world results.
6.4 ATP: Adenosine Triphosphate
63 The sodium-potassium pump is an example of free-energy coupling. The free energy
derived from exergonic ATP hydrolysis is used to pump sodium and potassium ions across
the cell membrane. The hydrolysis of one ATP molecule releases 7.3 kcal/mol of free
energy If it takes 2.1 kcal/mol of free energy to move one Na+
across the membrane how many sodium ions could be moved by
the hydrolysis of one ATP molecule? Show your calculations to provide reasoning for
your answer.
Solution Sample answer: Assume that only these two processes need to be considered.

The factors display the reasoning. If 7.3 kcal are available for each ATP and 2.1 kcal
are required to transport 1 sodium ion, the number of sodium ions transported per
ATP is the ratio.

Advanced Placement Biology Instructor’s Solution Manual


6 | Metabolism 135

64 Is the EA for ATP hydrolysis in cells likely relatively low or high compared to the EA for the
combustion of gasoline in an internal combustion engine?
1. Explain your reasoning in terms of the relative stabilities of ATP and gasoline
compared to air in which no catalysts are present.
2. Describe how the role of the enzyme ATPase in the hydrolysis of ATP in a cell
differs from a spark in the cylinder of an internal combustion engine.
3. Describe a strategy for collecting data that can be used to measure the energies
of activation (EA) of each of these two processes with instruments that can
measure concentrations of reactions produced in each system.
Solution Sample answer: The spark of the internal combustion engine is similar to a catalyst
in that the free energy of the original state is recovered through the generation of
electrical energy by the rotational motion produced by the spark. However, in the
absence of a spark, gasoline can spontaneously combust at temperatures above its
ignition temperature. The activation energy can be estimated as comparable with
RT, where R is the universal gas constant and T is the ignition temperature, roughly 5
kJ/mole. The activation energy of ATP hydrolysis is usually quoted as 57 kJ/mole.
Rather than look up the numbers, it would be useful to consider the likelihood that
ATP will spontaneously become ADP. Metabolic and cellular integrity would suffer
since these processes are regulated by the availability of ATP.
6.5 Enzymes
65 Vitamin B12 is a coenzyme involved in a wide variety of cellular processes. Synthesis of
vitamin B12 occurs only in bacteria; in animals, these bacteria populate anaerobic
environments in the gut. Consequently, vegan diets in developing nations and diets
common to developing nations provide no source of B12. Researchers (Ghosh et al.
http://dx.doi.org/10.3389/fnut.2016.00001) found that rats whose diets contained limited
(L) and no (N) B12 displayed symptoms that were not observed in the control group (C)
whose diet included B12 and was otherwise identical. Chemical analysis of adipocytokines
in the plasma after feeding periods of 4 and 12 weeks are shown in the following table.
Adipocytokines Feeding duration
Tissue of Origin (weeks) C L N
Leptin (pg/L) 4 5.7 ± 0.21 5.8 ± 0.25 6.1 ± 0.25
Adipose 12 5.8 ± 0.39 6.5 ± 0.36 9.9 ± 0.68
MCP-1 (mg/L) 4 43.0 ± 1.18 44.4 ± 1.95 46.9 ± 2.08
Monocytes 12 43.2 ± 2.47 45.3 ± 3.02 49.5 ± 1.27
IL-6 (mg/L) 4 150 ± 3.2 154 ± 4.5 184 ± 8.0
Monocytes 12 151 ± 6.7 176 ± 11.0 185 ± 8.2
The sample size for these data is small: n = 6 within each group. Also shown in the table
are cells in which these cytokine messages originate. Adipose cells store fats. Monocytes
are white blood cells of the immune system. Over the 12 weeks of feeding, the weights of

Advanced Placement Biology Instructor’s Solution Manual


136 6 | Metabolism

all three groups were equivalent, while the percent of body fat increased relative to the
control for the rats fed a diet of limited and no B12: 40 percent (N) and 20 percent (L),
respectively.
A. Identify which adipocytokines show significant increases relative to the control group,
after only 4 weeks of treatment. Justify your identification.
B. Identify which adipocytokines show only significant increases relative to the control
group, after 12 weeks of treatment. Justify your identification.
C. Identify which adipocytokines show significant increases relative to the control group,
after 4 weeks of treatment but no further increase after 12 weeks. Justify your
identification.
Adipocytokines are chemical messengers that regulate metabolism and blood vessel
production and dilation. High concentrations of adipocytokines are commonly found
among individuals with abnormal autoimmune response. Monocyte chemoattractant
protein 1 (MCP-1) is involved in the trafficking or guiding of monocytes to damaged tissue,
as in a wound. In mice, leptin receptors of cells in the hypothalamus suppress hunger.
Interleukin (IL-6) is released to initiate and then regulate inflammation in response to an
infection. (The mice in this study were not infected or wounded.)
D. Construct an explanation, with reasoning based on the evidence provided by these
data, for the observed variations in adipocytokines. Many noncommunicable diseases are
associated with abnormal autoimmune responses, and the number of diseases that
involve abnormal autoimmune response is increasing. Many autoimmune diseases, such
as diabetes and heart disease, occur in developed nations at a much higher frequency
than in developing nations.
E. Evaluate, based on these data concerning the effect of restrictions on the availability of
B12, the following question: Does the increased lack of exposure to pathogens in
developed nations lead to reduced or abnormal immune response?
Solution Sample answer:
A. Leptin and IL-6 are likely to be significantly different than the control group for
the N diet. The intervals of the measured values of these cytokines do not overlap
with those of the control measurements. With such a small sample size, the
conclusion of significance would be very similar to that obtained with a t-test. And
application of a t-test is out of scope for the AP Biology Exam.
B. After 12 weeks of treatment, both leptin and IL-6 are significantly different than
the control. Justification for this claim is the same as given in (A).
C. The increase in concentrations of leptin continues to rise during the longer
treatment period. However, the concentration of IL-6 stops increasing. Again, the
claim is based on a comparison of the intervals of measured values.
D. During the treatment, the percent body fat of the animals whose diets are
deficient in B12 increases relative to the control. The constancy of MCP-1 is

Advanced Placement Biology Instructor’s Solution Manual


6 | Metabolism 137

consistent with the absence of wounding that would evoke an inflammatory


response. The increase in leptin is consistent with the increase in adipose tissue. The
interleukin increase indicates that the physiological state of the animals deficient in
B12 have responded as if the system is infected. Adipose cells or their secretion
products are provoking an immune response from the host.
E. Students may need to reread the problem narrative to obtain information about
the source of B12. Populations in developed nations may be less exposed to
bacteria. Consequently, there may be a diminished supply of B12, or other dietary
supplements provided by a higher and more diverse population of gut bacteria. The
evaluation of this question should connect the role and origin of B12 with the
example of autoimmune diseases and obesity in developed nations. Good scientific
questions can be pursued experimentally, taking into account all of the available
data and are congruent with other good questions. This question is a good scientific
question and is referred to as the hygiene hypothesis.
66 Using an example, explain how enzyme feedback inhibition regulates a cellular process.
Solution Students might consider stability. A system under negative feedback remains at the
set point. They might consider efficiency. A system under negative feedback uses
resources on demand rather than consume free energy to produce redundant,
unused subcomponents. Students might focus on the enzyme part of this question
and respond that, without catalysis, the system must be greatly simplified or
operate on a time scale that is inconsistent with life. Students might focus on the
specific part of the question and point out that the investment in greater
information content allows more precise communication.

Advanced Placement Biology Instructor’s Solution Manual


138 7 | Cellular Respiration

7 | CELLULAR RESPIRATION
REVIEW QUESTIONS
1 What is the most important energy currency used by cells?
A ATP
B ADP
C AMP
D Adenosine
Solution The solution is (A). ATP has high-energy bonds between its phosphate molecules,
allowing energy to be stored by cells within ATP molecules.
2 What happens when a chemical is reduced during a reaction?
A The compound is reduced to a simpler form.
B An electron is added to the chemical.
C A hydrogen atom is removed from the substrate.
D It acts as a catabolic reaction.
Solution The solution is (B). Addition of an electron to a substrate causes its reduction, and
the donor is said to be oxidized.
3 Which molecules are oxidizing agents?
A FAD+ and NAD+
B FADH2 and NADH
C FAD and FADH2
D NAD+ and NADH
Solution The solution is (A). FAD+ and NAD+ have a tendency to accept 2H+ and 2e− to become
reduced and form FADH2 and NADH2, respectively.
4 Which reaction releases energy?
A AMP + phosphate → ADP + H2O
B ADP + phosphate → ATP + H2O
C ATP + H2O → ADP + phosphate
D AMP + H2O → ATP + phosphate
Solution The solution is (C). Hydrolysis of ATP releases energy and frees an inorganic
phosphate. This energy is used for various chemical reactions.

Advanced Placement Biology Instructor’s Solution Manual


7 | Cellular Respiration 139

5 During the second half of glycolysis, what occurs?


A ATP is used up.
B Fructose is split in two.
C ATP is produced.
D Glucose becomes fructose.
Solution The solution is (C). During the pay-off phase of glycolysis, substrate level
phosphorylation takes place twice, creating a net of 2 ATP molecules per glucose.
6 GLUTs are integral membrane proteins that assist in the facilitated diffusion of glucose
into and out of cells. What reaction in glycolysis prevents glucose from being transported
back out of the cell?
A Hexokinase dephosphorylates glucose using ATP, creating a glucose molecule that
can’t cross the hydrophilic portion of the plasma membrane.
B Hexokinase phosphorylates glucose using ADP, creating a glucose molecule that can’t
cross the hydrophobic interior of the plasma membrane.
C Hexokinase dephosphorylates glucose using ADP, creating a glucose molecule that
can’t cross the hydrophilic portion of the plasma membrane.
D Hexokinase phosphorylates glucose using ATP, creating a glucose molecule that can’t
cross the hydrophobic interior of the plasma membrane.
Solution The solution is (D). Hexokinase phosphorylates glucose using ATP. The negative
charge of the phosphate prevents the passage of phosphorylated glucose through
the hydrophobic interior of the plasma membrane.
7 How many ATP molecules are used and produced per molecule of glucose during
glycolysis?
A The first half of glycolysis uses 2 ATPs, and the second half of glycolysis produces
4 ATPs.
B The first half of glycolysis produces 2 ATPs, and the second half of glycolysis uses
4 ATPs.
C The first half of glycolysis uses 4 ATPs, and the second half of glycolysis produces
2 ATPs.
D The first half of glycolysis produces 4 ATPs, and the second half of glycolysis uses
2 ATPs.
Solution The solution is (A). The enzymes hexokinase and PFK utilize 2 ATPs each in the first
half of the cycle. The second half produces 4 ATPs by substrate-level
phosphorylation. This occurs twice.

Advanced Placement Biology Instructor’s Solution Manual


140 7 | Cellular Respiration

8 What is removed from pyruvate during its conversion into an acetyl group?
A Oxygen
B ATP
C Vitamin B
D Carbon dioxide
Solution The solution is (D). During the oxidation of pyruvate, a molecule of carbon dioxide
along with NADH is produced.
9 What do the electrons added to NAD+ do in aerobic respiration?
A They become part of a fermentation pathway.
B They go to another pathway for ATP production.
C They energize the acetyl group in the citric acid cycle.
D They are converted to NADP.
Solution The solution is (B). NADH undergoes oxidative phosphorylation and enters the
electron transport chain to produce ATP.
10 GTP, which can be converted to ATP, is produced during which reaction of the citric
acid cycle?
A Isocitrate into α-ketoglutarate
B Succinyl-CoA into succinate
C Fumarate into malate
D Malate into oxaloacetate
Solution The solution is (B). During the citric acid cycle, two oxidative decarboxylation
reactions occur, producing 2 CO2 and 2 NADH. In addition, two dehydrogenation
reactions producing 1 NADH and 1 FADH2 and 1 substrate-level phosphorylation
occur. GTP is produced during the fifth reaction of the citric acid cycle. This reaction
is catalyzed by the enzyme succinyl-CoA synthetase.
11 How many NADH molecules are produced on each turn of the citric acid cycle?
A One
B Two
C Three
D Four
Solution The solution is (C). The citric acid cycle produces 3 NADH with the help of 2 oxidative
decarboxylations and 1 dehydrogenation.
12 What compound receives electrons from NADH?

Advanced Placement Biology Instructor’s Solution Manual


7 | Cellular Respiration 141

A FMN
B Ubiquinone
C Cytochrome c1
D Oxygen
Solution The solution is (A). Complex I (NADH dehydrogenase) contains an FMN subunit that
immediately accepts electrons from NADH and passes them to CoQ.

Advanced Placement Biology Instructor’s Solution Manual


142 7 | Cellular Respiration

13 Chemiosmosis involves the movement of what? Where does it occur?


A Electrons across the cell membrane
B Hydrogen atoms across a mitochondrial membrane
C Hydrogen ions across a mitochondrial membrane
D Glucose through the cell membrane
Solution The solution is (C). Hydrogen ions produced during the electron transport chain are
pumped across the inner mitochondrial membrane. These ions help maintain the
correct pH gradient for the efficient production of ATP.
14 What is the function of an electron in the electron transport chain?
A To dephosphorylate ATP, producing ADP
B To power active transport pumps
C To reduce heme in complex III
D To oxidize oxygen
Solution The solution is (B). As electrons move across the electron transport chain, hydrogen
ions are pumped across the inner membrane of the mitochondria.
15 What would be the outcome if hydrogen ions were able to diffuse through the
mitochondrial membrane into the mitochondria without the need for integral membrane
proteins?
A ATP would not be produced.
B Pyruvate would not be produced.
C Citric acid would not be produced.
D Carbon dioxide would not be produced.
Solution The solution is (A). ATP is produced with the help of ATP synthase embedded as
integral proteins in the inner mitochondrial membrane. These structures allow a
sequential passage of H+ ions into the matrix, generating ATP.
16 Which fermentation method can occur in animal skeletal muscles?
A Lactic acid fermentation
B Alcohol fermentation
C Mixed acid fermentation
D Propionic fermentation
Solution The solution is (A). The animal muscle cells contain lactate dehydrogenase, the
enzyme responsible for the formation of lactic acid.

Advanced Placement Biology Instructor’s Solution Manual


7 | Cellular Respiration 143

17 Which molecules are produced in glycolysis and used in fermentation?


A Acetyl-CoA and NADH
B Lactate, ATP, and CO2
C Glucose, ATP, and NAD+
D Pyruvate and NADH
Solution The solution is (D). Each glucose molecule produces 2 pyruvate and 2 NADH
molecules. These are further used in the fermentation process to form either lactate
or ethanol.
18 What are the products of alcohol fermentation?
A Methane and NADH
B Lactic acid and FAD+
C Ethanol and NAD+
D Pyruvic acid and NADH
Solution The solution is (C). Ethanol and NAD+ are produced. Pyruvate first forms the
acetaldehyde that further utilizes NADH to form ethanol and NAD+.
19 In the first step of glycolysis, what is glucose transformed into?
A Glucose-6-phosphate
B Fructose-1,6-bisphosphate
C Dihydroxyacetone phosphate
D Phosphoenolpyruvate
Solution The solution is (A). Conversion of glucose into glucose-6-phosphate is the first step
of glycolysis.
20 What is beta-oxidation?
A The main process used to break down glucose
B The main process used to assemble glucose
C The main process used to break down fatty acids
D The main process used to remove amino groups from amino acids
Solution The solution is (C). Beta-oxidation involves breaking down fatty acids using two
carbon fragments from the carboxyl end of fatty acids. It occurs in the mitochondria,
and the carbon of fatty acids is oxidized.
21 Which statement about catabolic pathways is false?
A Carbohydrates can feed into oxidative phosphorylation.
B Glycerol can be broken down into glucose and feed into glycolysis.

Advanced Placement Biology Instructor’s Solution Manual


144 7 | Cellular Respiration

C Amino acids can feed into pyruvate oxidation.


D Fatty acids can feed into the citric acid cycle.
Solution The solution is (A). As carbohydrates eventually transform into glucose, they can be
fed into glycolysis. Oxidative phosphorylation is only performed by the electron
transport chain by oxidizing NADH.
22 What impact, if any, do high levels of ADP have on glycolysis?
A They increase the activity of enzymes involved with glycolysis.
B The high levels decrease the activity of enzymes involved with glycolysis.
C They have no effect on the activity of any enzymes involved with glycolysis.
D The high levels slow down all pathways involved with glycolysis.
Solution The solution is (A). ADP is an activator and a positive regulator of certain enzymes
like phosphofructokinase-1.
23 The control of which enzyme exerts the greatest control of glycolysis?
A Hexokinase
B Phosphofructokinase
C Glucose-6-phosphatase
D Aldolase
Solution The solution is (A). Hexokinase is responsible for phosphorylating glucose into
glucose-6-phosphate which traps and prevents its passage through the plasma
membrane. Therefore, hexokinase is needed for glycolysis to start.
24 What does NOT occur as ATP concentration increases relative to ADP?
A Decreased activity of phosphofructokinase
B Increased activity of pyruvate kinase
C Decreased activity of isocitrate dehydrogenase
D Slowdown of the electron transport chain
Solution The solution is (B). In high levels of ATP, the pyruvate kinase activity tends to
decrease because it is a negative regulator/inhibitor of pyruvate kinase. Acetyl-CoA
also shows this inhibition.

Advanced Placement Biology Instructor’s Solution Manual


7 | Cellular Respiration 145

CRITICAL THINKING QUESTIONS


25 Why is it beneficial for cells to use ATP rather than directly use the energy stored in the
bonds of carbohydrates to power cellular reactions? What are the greatest drawbacks to
harnessing energy from the bonds of several different compounds?
A ATP is readily available in the form of a single unit that provides a consistent,
appropriate amount of energy. The cell would need to tailor each reaction to each
energy source if it harvested energy from different compounds.
B ATP energy cannot activate the ROS-dependent stress response, whereas food
molecules are responsible for activating ROS.
C ATP is low in energy, but food molecules possess higher levels of energy that cells
can use.
D ATP is readily available to cells, unlike compounds that have to first be phosphorylated
to release their energy.
Solution The solution is (A). ATP provides the cell with a way to handle energy in an efficient
manner. The molecule can be charged, stored, and used as needed. Moreover, the
energy from hydrolyzing ATP is delivered in a consistent amount. Harvesting energy
from the bonds of several different compounds would result in energy deliveries of
different quantities.
26 What role does NAD+ play in redox reactions?
A NAD+, an oxidizing agent, can accept electrons and protons from organic molecules
and get reduced to NADH.
B NAD+, a reducing agent, can donate its electrons and protons to organic molecules.
C NAD+, an oxidizing agent, can accept electrons from organic molecules and get
reduced to NADH2.
D NAD+, a reducing agent, can donate its electrons and protons to inorganic molecules.
Solution The solution is (A). NAD+ is the oxidized form of the molecule. It is an oxidizing agent;
NAD+ can accept electrons from organic molecules. After NAD+ accepts two electrons
and a proton, it becomes NADH, which is the reduced form of the molecule. NAD +
has a tendency to accept electrons and protons to become reduced to NADH. This
reduced form can further act as a reducing agent. Therefore, NAD can play an
oxidizing as well as a reducing role in redox reactions.
27 In the following general reaction, how are electrons transferred? Explain the role of each
species. Remember that R represents a hydrocarbon molecule and RH represents the
same molecule with a particular hydrogen identified.
RH + NAD+ → NADH + R

Advanced Placement Biology Instructor’s Solution Manual


146 7 | Cellular Respiration

A RH acts as a reducing agent and donates its electrons to the oxidizing agent NAD +,
forming NADH and R.
B NAD+, the oxidizing agent, donates its electrons to the reducing agent RH, forming
R and NADH.
C RH acts as an oxidizing agent and donates electrons to the reducing agent NAD +,
producing NADH and R.
D NAD+, the reducing agent, accepts electrons from the oxidizing agent RH, producing
NADH and R.
Solution The solution is (B). NAD+ is the oxidizing agent, which can accept electrons from RH,
the reducing agent. This causes NAD+ to be reduced to NADH and RH to be oxidized
to R. RH, the reducing agent, has electrons to donate, and the oxidizing agent, NAD +,
has a tendency to accept electrons, which in turn produces NADH and R as products.
28 Nearly all organisms on Earth carry out some form of glycolysis. How does this fact
support or not support the assertion that glycolysis is one of the oldest metabolic
pathways?
A To be present in so many different organisms, glycolysis was probably present in a
common ancestor rather than evolving many separate times.
B Glycolysis is present in nearly all organisms because it is an advanced and recently
evolved pathway that has been widely used because it is so beneficial.
C Glycolysis is absent in a few higher organisms. This contradicts the assertion that it is
one of the oldest metabolic pathways.
D Glycolysis is present in some organisms and absent in others, which may or may not
support the assertion that it is one of the oldest metabolic pathways.
Solution The solution is (A). If glycolysis evolved relatively late, it likely would not be as
universal in organisms as it is. It probably evolved in very primitive organisms and
persisted, with the addition of other pathways of carbohydrate metabolism that
evolved later.
29 Red blood cells (RBCs) do not perform aerobic respiration, but they do perform glycolysis.
Why do all cells need an energy source, and what would happen if glycolysis were blocked
in a red blood cell?
A Cells require energy to perform certain basic functions. Blocking glycolysis in RBCs
causes imbalance in the membrane potential, leading to cell death.
B Cells need energy to perform cell division. Blocking glycolysis in RBCs interrupts the
process of mitosis, leading to nondisjunction.
C Cells maintain the influx and efflux of organic substances using energy. Blocking
glycolysis stops the binding of CO2 to the RBCs, causing cell death.
D Cells require energy to recognize attacking pathogens. Blocked glycolysis inhibits the
process of recognition, causing invasion of the RBCs by a pathogen.

Advanced Placement Biology Instructor’s Solution Manual


7 | Cellular Respiration 147

Solution The solution is (A). All cells must consume energy to carry out basic functions such as
pumping ions across membranes. A red blood cell would lose its membrane
potential if glycolysis were blocked, and it would eventually die.
30 What is the primary difference between a circular pathway and a linear pathway?
A The reactant and the product are the same in a circular pathway, but different in a
linear pathway.
B The circular pathway components get exhausted, whereas those of the linear pathway
do not and are continually regenerated.
C Circular pathways are not suited for amphibolic pathways, whereas linear pathways are.
D Circular pathways contain a single chemical reaction that is repeated, while linear
pathways have multiple events.
Solution The solution is (D). In a circular pathway, the final product of the reaction is also the
initial reactant. The pathway is self-perpetuating, as long as any of the intermediates
of the pathway are supplied. Circular pathways are able to accommodate multiple
entry and exit points, thus being particularly well suited for amphibolic pathways. In
a linear pathway, one trip through the pathway completes the pathway, and a
second trip would be an independent event.
31 Cellular respiration breaks down glucose and releases carbon dioxide and water. Which
step in the oxidation of pyruvate produces carbon dioxide?
A Removal of a carboxyl group from pyruvate releases carbon dioxide. The pyruvate
dehydrogenase complex comes into play.
B Removal of an acetyl group from pyruvate releases carbon dioxide. The pyruvate
decarboxylase complex comes into play.
C Removal of a carbonyl group from pyruvate releases carbon dioxide. The pyruvate
dehydrogenase complex comes into play.
D Removal of an acetyl group from pyruvate releases carbon dioxide. The pyruvate
dehydrogenase complex comes into play.
Solution The solution is (D). Two molecules of pyruvate are produced by glycolysis. A carboxyl
group is removed from pyruvate, releasing a molecule of carbon dioxide. This step
proceeds twice because there are two pyruvate molecules produced at the end of
glycolysis. A carboxyl group is released with the help of pyruvate dehydrogenase,
releasing carbon dioxide.
32 What three steps are included in the breakdown of pyruvate?
A Pyruvate dehydrogenase removes a carboxyl group from pyruvate, producing carbon
dioxide. Dihydrolipoyl transacetylase oxidizes a hydroxyethyl group to an acetyl group,
producing NADH. Finally, an enzyme-bound acetyl group is transferred to CoA,
producing a molecule of acetyl-CoA.

Advanced Placement Biology Instructor’s Solution Manual


148 7 | Cellular Respiration

B Pyruvate dehydrogenase oxidizes a hydroxyethyl group to an acetyl group, producing


NADH. It further removes a carboxyl group from pyruvate, producing carbon dioxide.
Finally, dihydrolipoyl transacetylase transfers an enzyme-bound acetyl group to CoA,
forming an acetyl-CoA molecule.
C Pyruvate dehydrogenase transfers enzyme-bound acetyl group to CoA, forming an
acetyl CoA molecule. It then oxidizes a hydroxyethyl group to an acetyl group,
producing NADH. Dihydrolipoyl transacetylase removes a carboxyl group from
pyruvate, producing carbon dioxide.
D Pyruvate dehydrogenase removes a carboxyl group from pyruvate, producing carbon
dioxide. Dihydrolipoyl dehydrogenase transfers enzyme-bound acetyl groups to CoA,
forming an acetyl-CoA molecule. Finally, a hydroxyethyl group is oxidized to an acetyl
group, producing NADH.
Solution The solution is (A). Step 1. A carboxyl group is removed from pyruvate, releasing a
molecule of carbon dioxide. The result is a two-carbon hydroxyethyl group bound to
the enzyme pyruvate dehydrogenase. Step 2. The hydroxyethyl group is oxidized to
an acetyl group, and the electrons are picked up by NAD+, forming NADH. Step three:
The enzyme-bound acetyl group is transferred to CoA, producing a molecule of
acetyl-CoA.
33 How do the roles of ubiquinone and cytochrome c differ from the other components of
the electron transport chain?
A CoQ and cytochrome c are mobile electron carriers, while NADH dehydrogenase and
succinate dehydrogenase are bound to the inner mitochondrial membrane.
B CoQ and cytochrome covalently bind electrons, while NADH dehydrogenase and
succinate dehydrogenase are bound to the inner mitochondrial membrane.
C CoQ and cytochrome c are bound to the inner mitochondrial membrane, while NADH
dehydrogenase and succinate dehydrogenase are mobile electron carriers.
D CoQ and cytochrome c covalently bind electrons, while NADH dehydrogenase and
succinate dehydrogenase are mobile electron carriers.
Solution The solution is (A). CoQ and cytochrome c are transport molecules and referred to as
small electron carriers. Ubiquinone (CoQ) is the only component of the electron
transport chain that is not a protein. Also, the other components of the electron
transport chain are large complexes anchored to the inner mitochondrial
membrane.
34 What accounts for the different number of ATP molecules that are formed through
cellular respiration?
A Transport of NADH from cytosol to mitochondria is an active process that decreases
the number of ATP produced.
B The ATPs produced are utilized in the anaplerotic reactions that are used for the
replenishment of the intermediates.

Advanced Placement Biology Instructor’s Solution Manual


7 | Cellular Respiration 149

C Most of the ATPs produced are rapidly used for the phosphorylation of certain
compounds found in plants.
D A large number of ATP molecules are used in the detoxification of xenobiotic
compounds produced during cellular respiration.
Solution The solution is (A). Few tissues except muscle produce the maximum possible
amount of ATP from nutrients. The intermediates are used to produce needed
amino acids, fatty acids, cholesterol, and sugars for nucleic acids. When NADH is
transported from the cytoplasm to the mitochondria, an active transport mechanism
is used, which decreases the amount of ATP that can be made. The electron
transport chain differs in composition between species, so different organisms will
make different amounts of ATP from their electron transport chains.
35 Which statement best describes complex IV in the electron transport chain?
A Complex IV consists of an oxygen molecule held between the cytochrome and copper
ions. The electrons flowing finally reach the oxygen, producing water.
B Complex IV contains a molecule of flavin mononucleotide and iron-sulfur clusters. The
electrons from NADH are transported here to coenzyme Q.
C Complex IV contains cytochrome b, c, and Fe-S. Here, the proton motive Q cycle
takes place.
D Complex IV contains a membrane-bound enzyme that accepts electrons from FADH 2
to make FAD. This electron is then transferred to ubiquinone.
Solution The solution is (A). The cytochromes in complex IV hold an oxygen molecule very
tightly between the iron and copper ions until the oxygen is completely reduced. The
reduced oxygen then picks up two hydrogen ions from the surrounding medium to
make water.
36 What is the primary difference between fermentation and anaerobic respiration?
A Fermentation uses only glycolysis, and its final electron acceptor is an organic
molecule, whereas anaerobic respiration uses glycolysis, TCA, and the ETC, but finally
gives electrons to an inorganic molecule.
B Fermentation uses glycolysis, TCA, and ETC, but finally gives electrons to an inorganic
molecule, whereas anaerobic respiration uses only glycolysis, and its final electron
acceptor is an organic molecule.
C Fermentation uses glycolysis, and its final electron acceptor is an inorganic molecule,
whereas anaerobic respiration uses glycolysis, TCA, and ETC, but finally gives electrons
to an organic molecule.
D Fermentation uses glycolysis, TCA, and ETC, but finally gives electrons to an organic
molecule, whereas anaerobic respiration uses only glycolysis, and its final electron
acceptor is an inorganic molecule.

Advanced Placement Biology Instructor’s Solution Manual


150 7 | Cellular Respiration

Solution The solution is (A). Fermentation uses glycolysis only. Anaerobic respiration uses all
three parts of cellular respiration, including the parts in the mitochondria like the
citric acid cycle and electron transport; it also uses a different final electron acceptor
instead of oxygen gas.
37 What type of cellular respiration is represented in the following equation, and why?
CO2 + H2 + NADH → CH4 + H2O + NAD+
A Anaerobic respiration, because the final electron acceptor is inorganic
B Aerobic respiration, because oxygen is the final electron acceptor
C Anaerobic respiration, because NADH donates its electrons to a methane molecule
D Aerobic respiration, because water is being produced as a product
Solution The solution is (A). NAD+ is regenerated from NADH using an inorganic molecule
(carbon dioxide) as a final electron acceptor.
38 Would you describe metabolic pathways as inherently wasteful or inherently economical,
and why?
A Metabolic pathways are economical due to feedback inhibition. Also, intermediates
from one pathway can be utilized by other pathways.
B Metabolic pathways are wasteful because they perform uncoordinated catabolic and
anabolic reactions that waste some of the energy that is stored.
C Metabolic pathways are economical due to the presence of anaplerotic reactions that
replenish the intermediates.
D Metabolic pathways are wasteful because most of the energy produced is utilized in
maintaining the reduced environment of the cytosol.
Solution The solution is (A). They are very economical. The substrates, intermediates, and
products move between pathways and do so in response to finely tuned feedback
inhibition loops that keep metabolism balanced overall. Intermediates in one
pathway may occur in another, and they can move from one pathway to another
fluidly in response to the needs of the cell.
39 What lipids are connected to glucose catabolism pathways, and how are they connected?
A Cholesterol and triglycerides can be converted to glycerol-6-phosphate that continues
through glycolysis.
B Glucagon and glycogen can be converted to 3-phosphoglyceraldehyde, which is an
intermediate of glycolysis.
C Chylomicrons and fatty acids get converted to 1,3-bisphosphoglycerate that continues
in glycolysis, forming pyruvate.
D Sphingolipids and triglycerides form glucagon that can be fed into glycolysis.

Advanced Placement Biology Instructor’s Solution Manual


7 | Cellular Respiration 151

Solution The solution is (A). Triglycerides can be both made and broken down through parts
of the glucose catabolism pathways. Glycerol can be phosphorylated to glycerol-3-
phosphate, which continues through glycolysis.
40 How does citrate from the citric acid cycle affect glycolysis?
A Citrate and ATP are negative regulators of phosphofructokinase-1.
B Citrate and ATP are negative regulators of hexokinase.
C Citrate and ATP are positive regulators of phosphofructokinase-1.
D Citrate and ATP are positive regulators of hexokinase.
Solution The solution is (A). Citrate from the citric acid cycle indicates that an alternative
energy source is available and thus inhibits phosphofructokinase-1. ATP is another
negative regulator of PFK-1.
41 Why might negative feedback mechanisms be more common than positive feedback
mechanisms in living cells?
A Negative feedback mechanisms maintain homeostasis, whereas positive feedback
drives the system away from equilibrium.
B Positive feedback mechanisms maintain a balanced amount of substances, whereas
negative feedback restricts them.
C Negative feedback turns the system off, making it deficient in certain substances.
Positive feedback balances out these deficits.
D Positive feedback brings substance amounts back to equilibrium, while negative
feedback produces excess amounts of the substance.
Solution The solution is (A). Negative feedback mechanisms control the process by turning
the system off at the appropriate step. Positive feedback accelerates the processes,
taking the system away from equilibrium.

TEST PREP FOR AP® COURSES


42 The table shows the amount of oxygen consumed (third column) by different animals
(first column) at different temperatures. This type of apparatus measures the change in
volume of air to detect the removal of oxygen. However, organisms produce carbon
dioxide as they take in oxygen. To provide accurate measurements, what would you need
to add to the setup?

Advanced Placement Biology Instructor’s Solution Manual


152 7 | Cellular Respiration

A A substance that removes carbon dioxide gas


B A plant that will add oxygen to allow an animal to breathe
C A glucose reserve
D A substance that adds carbon dioxide gas
Solution The solution is (A). A substance to remove carbon dioxide should be added, as the
respirometer measures the change in the gas volume. The animals produce carbon
dioxide that, if not removed, would bias the results.

43 According to the data, the crickets at 25 °C have greater oxygen consumption per gram of
tissue than do the crickets at 10 °C. This trend in oxygen consumption is the opposite of
that in mice.

The difference in trends in oxygen consumption among crickets and mice is due to what?
A Their difference in size
B Their mode of nutrition
C Their difference in metabolic heat production
D Their mode of ATP production
Solution The solution is (C). Crickets are ectotherms, meaning that their metabolic rate is
generally higher at higher ambient temperatures. This is why the crickets at 25 °C
have a higher metabolic rate, as evidenced by higher oxygen consumption, than
crickets at a lower temperature.
44 Where in a cell does glycolysis take place in both prokaryotes and eukaryotes?
A The cytosol

Advanced Placement Biology Instructor’s Solution Manual


7 | Cellular Respiration 153

B The mitochondria
C The plasma membrane
D The nucleus
Solution The solution is (A). Glycolysis occurs in the cytosol in eukaryotes as well as in
prokaryotes.
45 A new species of obligate anaerobe, a bacterium, has been found that lives in hot, acidic
conditions. While other pathways may also be present, which metabolic pathway is the
most likely to be present in this species?
A Aerobic respiration
B The citric acid cycle
C Oxidative phosphorylation
D Glycolysis
Solution The solution is (D). Glycolysis is a metabolic pathway that is common to almost all
organisms.
46 What evidence provides the strongest support that glycolysis is an older and more
conserved pathway than the citric acid cycle?
A Glycolysis is the primitive pathway, found in all three domains. It also occurs in
anaerobic conditions and in the cytosol.
B This pathway occurs in the cytosol, is found in all animals and plants, and does not
require oxygen.
C Glycolysis takes place in anaerobic conditions, can metabolize cholesterol and fatty
acids, and occurs even in methanogens.
D This pathway only occurs in the mitochondria. It is highly flexible because it is found in
almost all organisms.
Solution The solution is (A). Glycolysis is found in all three domains, while the citric acid cycle
is not. This suggests that glycolysis is the more primitive pathway.
47 What is Structure X in the graphic?

A The inner mitochondrial membrane

Advanced Placement Biology Instructor’s Solution Manual


154 7 | Cellular Respiration

B The mitochondrial matrix


C A eukaryotic plasma membrane
D The cytosol
Solution The solution is (A). The phospholipid bilayer membrane is the inner mitochondrial
membrane. The embedded structures depict the complexes of the electron
transport chain.
48 What would be the most direct result of blocking Structure Z in the graphic?

A Cytochrome c would not pass electrons from complex III to complex IV.
B Ubiquinone would not pass electrons from complex III to complex IV.
C NADH would not be converted to NAD+, and the electron transport chain would stop.
D No protons would be pumped across the membrane.
Solution The solution is (A). Structure Z is cytochrome c. If it gets blocked, then electrons
wouldn’t be able to pass from complex III to complex IV.

Advanced Placement Biology Instructor’s Solution Manual


7 | Cellular Respiration 155

49 Where do the electrons moving along the membrane in the figure come from, and where
do the electrons end up?

A The electrons are released by NADH and FADH2 and finally accepted by oxygen to form
water.
B The electrons are given off by water and finally accepted by NAD + and FAD+ to produce
the energy currencies NADH and FADH2.
C The electrons are emitted by ubiquinone and are, in turn, transferred from complex I
to complex II. Water finally accepts the electrons.
D The electrons are given out by NADH and FADH2 and are, in turn, finally accepted by
H2O.
Solution The solution is (A). The electrons come from NADH and FADH2, which are produced
during glycolysis and the citric acid cycle. The electrons from NADH and FADH join
with H+ and O2 to form water.
50 Glucose catabolism pathways are sequential and lead to the production of energy. What
is the correct order of the pathways for the breakdown of a molecule of glucose as shown
in the formula?
C6H12O6 + O2 → CO2 + H2O + energy
A Oxidative phosphorylation → citric acid cycle → oxidation of pyruvate → glycolysis
B The oxidation of pyruvate → citric acid cycle → glycolysis → oxidative phosphorylation
C Glycolysis → oxidation of pyruvate → citric acid cycle → oxidative phosphorylation
D Citric acid cycle → glycolysis → oxidative phosphorylation → oxidation of pyruvate
Solution The solution is (C). Glycolysis produces pyruvate, and then this pyruvate undergoes
oxidation to produce CO2 and acetyl-CoA, the latter of which enters the citric acid
cycle to produce more ATP, NADH, and FADH. NADH and FADH2 donate electrons to
the electron transport chain to produce ATP and water molecules through oxidative
phosphorylation.
51 Which statement most directly supports the claim that different species of organisms use
different metabolic strategies to meet their energy requirements for growth,
reproduction, and homeostasis?

Advanced Placement Biology Instructor’s Solution Manual


156 7 | Cellular Respiration

A During cold periods, pond-dwelling animals can increase the number of unsaturated
fatty acids in their cell membranes, while some plants make antifreeze proteins to
prevent ice crystal formation in their tissues.
B Bacteria lack introns, while many eukaryotic genes contain many of these intervening
sequences.
C Carnivores have more teeth that are specialized for ripping food, while herbivores
have more teeth specialized for grinding food.
D Plants generally use starch molecules for storage, while animals use glycogen and fats
for storage.
Solution The solution is (D). Having different mechanisms of storing energy (starch molecules
in plants, glycogen in animals) is a way to strategize the conservation and use of
energy. According to their requirements, these organisms can then use the stored
energy in a sustainable way.
52 Which statement best describes how the citric acid cycle relates to glycolysis, oxidative
phosphorylation, and chemiosmosis?
A Glycolysis produces pyruvate, which is converted to acetyl-CoA and enters the citric
acid cycle. This cycle produces NADH and FADH2, which donate electrons to the
electron transport chain to pump protons and produce ATP through chemiosmosis.
Production of ATP using an electron transport chain and chemiosmosis is called
oxidative phosphorylation.
B The citric acid produces pyruvate, which converts to glucose to enter glycolysis. This
pathway produces NADH and FADH2, which enter oxidative phosphorylation to
produce ATP through chemiosmosis.
C Citric acid produces NADH and FADH2, which undergo oxidative phosphorylation. This
produces ATP by pumping protons through chemiosmosis. The ATP produced is
utilized in large amounts in the process of glycolysis.
D Glycolysis produces pyruvate, which directly enters the citric acid cycle. This cycle
produces the energy currency that undergoes the electron transport chain to produce
water and ATP.
Solution The solution is (A). Pyruvate is produced by glycolysis during the catabolism of
glucose. Pyruvate is converted to acetyl-CoA, which enters the citric acid cycle and is
oxidized to CO2. The electrons extracted are picked up by NAD+ and FAD+, and then
carried by NADH and FADH2 to the electron transport chain of oxidative
phosphorylation. NADH and FADH2 transfer their electrons to the electron
transporters embedded in the inner mitochondrial membrane. As the electrons move
through the electron transport chain, the energy produced is used to pump protons
from the mitochondrial matrix to the intermembrane space. These protons then
diffuse back through the inner mitochondrial membrane through ATP synthase during
chemiosmosis. This facilitates the addition of a phosphate to ADP, forming ATP.

Advanced Placement Biology Instructor’s Solution Manual


7 | Cellular Respiration 157

SCIENCE PRACTICE CHALLENGE QUESTIONS


7.1 Energy in Living Systems
53 Combustion of carbohydrates, like in a fireplace, is a reduction-oxidation reaction in which
the carbon atom is oxidized and the oxygen atom is reduced, producing water and carbon
dioxide. Oxidative phosphorylation and glycolysis are also reduction-oxidation reactions
that produce the same products. Explain the differences and similarities among these
abiotic and biotic processes in terms of the changes in entropy and heat that contribute to
the free energy extracted from chemical bonds, the spontaneity of each, and the role of
catalysis.
Solution This question is very open ended and could be used as the basis for small group
discussion. Sample answers:
 A fire generates almost entirely heat and a large increase in entropy, so the
resulting free energy is negligible. Oxidative phosphorylation produces very little
heat, so the bond energy is captured as free energy. Glycolysis is less efficient,
and the heat produced is useful for thermoregulation.
 Both the abiotic and biotic processes are exergonic.
 The development of elegant catalysis and structural features for metabolism and
its regulation is very different from the unregulated and random fire. But all of
these processes have activation energies, including the spark.
7.3 Oxidation of Pyruvate and the Citric Acid Cycle
54 A. [Extension] Living systems require free energy to carry out cellular functions and
employ various strategies to capture, use, and store free energy. Explain the advantage
that the higher energy efficiency per kilogram of the Krebs cycle provides to you
compared to a metabolism based on glycolysis alone. Your explanation should make use
of all the following facts:
 for glycolysis is kJ/mole of glucose.
 for aerobic respiration is kJ/mole glucose.

 The basal metabolic rate of mammals is often represented as


 The molar mass of glucose is 180 g/mole.
B. Explain the bioenergetic difference between aerobic and anaerobic respiration in terms
of the difference between free-energy production and power. Your explanation should
make use of all the following facts:

Advanced Placement Biology Instructor’s Solution Manual


158 7 | Cellular Respiration

 Power is the rate of free-energy production.


 Cancer cells derive most of their free energy from glycolysis.
 Enzymes of the citric acid (Krebs) cycle form coordinate complexes on the
cytoskeleton within the mitochondria.
C. The life cycle of the human parasite Trypanosoma brucei is divided between the body
of the tsetse fly and the human blood stream. The parasite causes “sleeping sickness” in
Sub-Saharan Africa. Within the human bloodstream, the parasite depends on glycolysis,
with enzymes compartmentalized in a membrane-bound organelle called the glycosome.
In the insect host, the parasite utilizes glycolysis as well as substrate-level and oxidative
phosphorylation. Explain the advantage of a life cycle in the human host that employs
anaerobic respiration with a rate of free-energy production that is enhanced by
compartmentalization in the glycosome and a life cycle in the insect host that is aerobic.
D. Predict the advantages of a biological system that uses both glycolysis and oxidative
phosphorylation. Your prediction should make use of all the following facts:
 Signaling can be used to detect low-oxygen environments and to regulate
response.
 Some cells, such as muscle and blood cells, must function in both low- and high-
oxygen environments.
 Glycolysis is reversible.
 The citric acid cycle is not reversible.
 Thermoregulation is needed for homeostasis.
Solution It is very hard for students to learn to construct explanations. They need to
remember that it must be testable and account for all of the available facts.
Identifying the facts that must be included in an explanation helps them to develop
the habit of organizing the evidence. Sample answer:
A. Basal metabolic rate is calculated by the student to be between 4,000 and
7,000 kJ/day. Dividing that by the free energy conversion for glucose shows that
about 40 moles of glucose, or 7.6 kg or 17 lb, would need to be consumed per day.
In contrast, the same calculation gives about 1 lb/ day for aerobic respiration.

That’s about 1,500 teaspoons per day, or about 2 teaspoons per waking minute. One
measure of efficiency might be the ability to work other than feed one’s self.

Advanced Placement Biology Instructor’s Solution Manual


7 | Cellular Respiration 159

B. Thermodynamics is really thermostatics. Often the rate matters more. Cancer


cells are growing rapidly, so we know that the rate of energy consumption is high.
However, they are also primarily using glycolysis for this production. I read that
there are effects of mitochondrial damage and restricted blood flow (and so
hypoxic). The student should puzzle over this and read about it—there is a well-
known concept called the Warburg model that looks at the effect that a switch to
anaerobic respiration has on cancer cells.
On the other hand, we know that the Krebs cycle is highly coordinated and not
diffusion limited. The need for structural support is often discussed, and there is
work that shows the attachment of clusters of enzymes in a coordinated manner to
the cytoskeleton. However, the idea of a metabolon is of growing interest. The
glycolytic complex metabolon is now discussed.
C. A connection to compartmentalization can be made with Trypanosoma. And to
make it all as ambiguous as biology is, there is an order of organisms that have a
specialized organelle for glycolysis. The claim is made that Trypanosoma brucei
(sleeping sickness) has a metabolic rate 50 times higher than is common for
glycolysis. This compartmentalization compensates for the lower output of
anaerobic respiration. The advantage of anaerobic respiration in the human host is
that venal blood is depleted of oxygen. The insect host has no similar
inhomogeneous oxygen supply. Also, the fly’s hemolymph is rich in proline, and the
degradation of this amino acid can be used as input to the Krebs cycle.
D. The selective advantage is provided through a more flexible response to changes
in the environment. The facts here point to the ability to switch to anaerobic
respiration under oxygen depletion, to store fat through gluconeogenesis, and to
use low efficiency metabolism to generate heating.
7.4 Oxidative Phosphorylation
55 Dinitrophenol (DNP) was used in the manufacture of munitions in World War I. In the
1930s, it was used as a weight-loss drug. Use in the United States cannot be regulated by
the U.S. Food and Drug Administration (FDA) because DNP is considered a dietary
supplement. Attempts to ban the drug in the United Kingdom following the death of four
users in 2015 failed in Parliament. DNP is a small molecule that is soluble in the
mitochondrial inner membrane. The hydroxyl group reversibly dissociates a proton.

Advanced Placement Biology Instructor’s Solution Manual


160 7 | Cellular Respiration

A. Predict the effect of DNP on the electrochemical gradient across the inner
mitochondrial membrane.
B. Explain how DNP can be used to reduce weight.
C. The effects of DNP can be reversed by administering glucose. However, treatment with
a combination of glucose and 2-deoxyglucose, which is an inhibitor of glycolysis, does not
reverse the effects of DNP. Explain, in terms of the products of glycolysis, why this
reversal of the effects of DNP was unexpected. (Hint—It might be useful to review the
reactants and products of glycolysis.)
D. Obesity correlates with an epidemic of other health issues, such as elevated blood
pressure, heart disease, and diabetes type II. A slow-release form of DNP (CRMP) is
patented. With slow-release technology, a drug can be delivered in small doses over time
from a pill whose matrix limits solubility. A simple but nonscientific question that can be
raised is: Will a slow-release drug retard progress toward behavioral changes that can
reduce the magnitude of this epidemic? Scientific questions can be pursued by testing the
outcomes predicted by possible answers. Refine this question for discussion in small
groups. Be prepared to justify the merits of your question.
Solution Sample answer:
A. Because DNP makes the inner mitochondrial membrane “leaky” to protons, the
electrochemical gradient across the membrane is disrupted.
B. DNP produces weight loss by partial starvation. The hydroxyl group of DNP is
protonated on the intermembrane space side of the inner membrane where
hydrogen ion concentrations are high. Diffusing through the membrane, the proton
is dissociated in the matrix where the hydrogen ion concentrations are low. The net
effect is the transport of a proton through the membrane. Molecules that have this
effect are called protonophores or proton translocators. The production of ATP and
NADH in the electron transport chain is driven by the electrochemical gradient
produced by the relatively low concentration of protons in the matrix. The transport
of protons to the matrix lowers the electrochemical potential. The ETS is said to be
decoupled.
C. Flooding with glucose increases glycolysis. The fact that treatment with 2-
deoxyglucose, which inhibits glycolysis, along with glucose supports this implication.
However, for each glucose converted to two pyruvates, two hydrogen ions are
released. The increase in pH shifts the equilibrium in the equation shown further to
the left, which would increase the leakage of protons. The work that first reported
this finding (Nakamura et al., 1989, Cardiovasc. Res. Apr., 23(4):286–94) suggested
increased levels of ATP in the intermembrane space might be responsible. In the
1990s, discoveries of ATP-dependent potassium channels began to be reported.
Transport of potassium into the matrix would repair the electrochemical potential
difference, reversing the effects of DNP. Claims are not made on the Internet that,
when using DNP, you should also take a dose of insulin.

Advanced Placement Biology Instructor’s Solution Manual


7 | Cellular Respiration 161

D. Useful questions will produce predictions that can be tested. The following might
be among those that students pose:
 What data are available concerning the long-term success rates of dieting or
exercise as diabetes treatment?
 If CRMP is patented, will it be regulated as a prescription drug and not a
dietary supplement?
 Will mitochondria in all cells, particularly muscle and nerve tissue, be
affected, or can only tissues that have a prominent role in homeostasis, such
as liver or white adipose tissue, be targeted? (Inventors of CRMP use an
ether derivative of DNP that allows it to selectively target the liver.)
56 As shown in the figure, cyanide inhibits the electron transport chain by competing with O 2
molecules for the cytochrome c oxidase heme group. Carbon monoxide (CO) has a similar
effect. Both cyanide and carbon monoxide cause poisoning in victims of smoke inhalation.

A. Predict the effects of these poisons on the following properties of mitochondria just
after exposure: the pH of the intermembrane space, the concentration of NADH, and the
rate of production of ATP in the matrix. Justify your predictions.
B. Rotenone is a poison that blocks the transfer of electrons from Complex I of the
electron transport chain to ubiquinone. Methylene blue is a molecule with many uses
involving its reduction-oxidation properties. Recent studies show the effectiveness of
methylene blue in increasing the body’s metabolic rate and as a treatment for Alzheimer’s
patients. The oxidized form of methylene blue is reduced by NADH, and its reduced form
is oxidized by O2. Explain the use of methylene blue as an antidote for rotenone
poisoning.

Advanced Placement Biology Instructor’s Solution Manual


162 7 | Cellular Respiration

Solution Sample answer:


A. When the cytochrome is blocked, electrons cannot be transferred to O 2. The
concentration of molecular oxygen increases. The transfer of electrons to
cytochrome is also blocked, and so the NADH concentration increases. Hydrogen
ions cannot be transported over the membrane so the pH of the intermembrane
space drops.
B. Methylene blue is currently a hot supplement available on the web; athletes and
students want improved performance. Measurements of respiration rate indicate
that the effects are real. Since methylene blue is able to act as a redox surrogate for
Complex II and III, the effect of rotenone is counteracted by the transfer of electrons
from NADH to methylene blue to cytochrome.
7.5 Metabolism without Oxygen
57 E. coli are enteric (gut-dwelling) facultative anaerobic bacteria. (Facultative anaerobes can
grow with or without free oxygen. Obligatory anaerobes grow only in the absence of free
oxygen.) Researchers planned to grow cultures of E. coli under a range of conditions to
model the transition from strictly anaerobic to aerobic respiration.
The oxygen content of atmospheres at constant total pressure will be controlled by
volumes of nitrogen and oxygen gases.

Ratios of volume, between 0 and 0.25 of shaken growth flasks, can be


measured in terms of optical density, which is the percent of transmission of light through
a sample of the growing E. coli culture. A rule of thumb is that the range of strict
anaerobes is when and the boundary for aerobic respiration is when r = 0.05.
A large number of flasks that can be constantly shaken at fixed temperature, and
from which samples can be taken without atmospheric contamination, are available
for this study.
These results of the experiment will be used to infer growth rates of E. coli along the
entire 7.5-m length of the average human intestine (small intestine and large intestine),
where the oxygen content varies from atmospheric to anaerobic conditions. The retention
time of food in the small intestine, whose average length is 2.5 m, is approximately 4 h.
The retention time of food over the entire length of the intestine is between 24 and 72 h.
A. Describe and apply a mathematical model that can be used to represent the variation
of oxygen environments of a bacterium that is being transported with the food along the
length of the intestine.
B. Design the experimental sampling times in terms of growth intervals of interest in
this study: (i) the time when the bacteria are passing the small-large intestine boundary;
(ii) the time when the bacteria reach the end of the large intestine; and (iii) the time when
the bacteria reach facultative anaerobic conditions,

Advanced Placement Biology Instructor’s Solution Manual


7 | Cellular Respiration 163

C. Sketch a graph that predicts the distribution of aerobic, facultative anaerobic, and
obligatory anaerobic bacteria along the length of the entire intestine based on these
parameters. Keep in mind that anaerobes have a lower respiration rate.
Solution Sample answer:
A. The oxygen ratio must go from 0.25 to 0 over a distance of 7.5 m. To make
progress, begin by assuming the simplest form (you might also use 0.25e − ax since
the ratio cannot be negative)
rO2 = 0.25 – 0.03x,
where x is the distance along the intestine and rO2 is the oxygen ratio.
To answer Part B, we also need a model that involves the dependence of rO2 on
time. The speed of food through the intestine can be used to obtain the rO 2(t)
relationship. The range of total time is very broad (and that is good because it
introduces conditional modeling) So, the speed is between 0.3 and
1 m/h. Then since , we have a second relationship

where t is measured in hours.


B. The times of interest are (i) passing the small-large intestine boundary, x = 2m, r =
0.19, and ; (ii) reached the end of the large intestine, and
(iii) the boundary to facultative anaerobic conditions,
C. A reasonable sketch could look like the following:

7.6 Connections of Carbohydrate, Protein, and Lipid Metabolic Pathways


58 White snakeroot is a plant that contains chemicals that deactivate the enzyme lactate
dehydrogenase. Humans who consume milk from cows or goats that eat white snakeroot
can become ill. Symptoms of milk poisoning include vomiting, abdominal pain, and
tremors, which become worse after exercise. Beyond childhood, most people do not
express the enzyme lactase that catalyzes the breakdown of lactose into glucose and
galactose. Consumption of milk can produce symptoms similar to those of milk poisoning.

Advanced Placement Biology Instructor’s Solution Manual


164 7 | Cellular Respiration

After a period of consumption of dairy foods, though, prebiotic adaptation (changes in the
microbes in the intestine) imparts lactose tolerance. Since dairy foods are a valuable
source of calcium, proteins, and vitamin D, considerable research has been conducted to
characterize adaptation.
Explain the similarities and differences between the effect of milk poisoning by white
snakeroot and lactose intolerance, and the possibility of prebiotic adaptation for each.
Solution Sample answer: As described in the text, in the absence of an active form of lactate
dehydrogenase in the liver, lactic acid cannot be converted to pyruvate. Lactose,
another component of dairy food, cannot be metabolized without the enzyme
lactase. The human population recently originated in Europe, where dairy food
resources produced selection favoring the production of lactase by adults.
Populations recently originating in Africa and Asia do not express lactase as adults.
However, consumption of dairy products leads to changes in gut bacteria. Such a
prebiotic adaptation could not protect Midwestern residents of the 19th century
against milk poisoning. Both gut bacteria and their human host are susceptible to
disruption by white snakeroot.

Advanced Placement Biology Instructor’s Solution Manual


8 | Photosynthesis 165

8 | PHOTOSYNTHESIS
REVIEW QUESTIONS
1 Which component is NOT used by both plants and cyanobacteria to carry out
photosynthesis?
A Carbon dioxide
B Chlorophyll
C Chloroplasts
D Water
Solution The solution is (C). The chloroplast is not a component used by both plants and
cyanobacteria. Photosynthesis occurs in chloroplasts of eukaryotes, while in
prokaryotes like cyanobacteria, photosynthesis occurs within the membrane and
cytoplasm.
2 Why are chemoautotrophs NOT considered the same as photoautotrophs if they both
extract energy and make sugars?
A Chemoautotrophs use wavelengths of light not available to photoautotrophs.
B Chemoautotrophs extract energy from inorganic chemical compounds.
C Photoautotrophs prefer the blue side of the visible light spectrum.
D Photoautotrophs make glucose, while chemoautotrophs make galactose.
Solution The solution is (B). Chemoautotrophs extract energy from inorganic chemical
compounds. Chemoautotrophs are organisms that use inorganic molecules, rather
than sunlight, as energy sources to synthesize their own food. They are mostly
bacteria and archaea that live in hostile sea environments where they are the
primary producers.
3 In which compartment of the plant cell do the light-independent reactions of
photosynthesis take place?
A Mesophyll
B Outer membrane
C Stroma
D Thylakoid
Solution The solution is (C). The light-independent reactions take place in the stroma of the
chloroplast because ATP and NADPH are produced on the stroma side of thylakoid.
Also, Calvin cycle enzymes are found in the stroma, so light-independent reaction
takes place in stroma.

Advanced Placement Biology Instructor’s Solution Manual


166 8 | Photosynthesis

4 What is a part of grana?


A The Calvin cycle
B The inner membrane
C Stroma
D Thylakoids
Solution The solution is (D). The stack of the tube-like strands connecting thylakoids is called
granum.
5 What are two major products of photosynthesis?
A Chlorophyll and oxygen
B Oxygen and carbon dioxide
C Sugars/carbohydrates and oxygen
D Sugars/carbohydrates and carbon dioxide
Solution The solution is (C). The photosynthesis reaction involves the reactants carbon
dioxide and water. The products of the reaction are oxygen, carbohydrates, and
mainly glucose.
6 What is the primary energy source for cells?
A Glucose
B Starch
C Sucrose
D Triglycerides
Solution The solution is (A). Glucose is a ready source of energy since its carbon atoms are
easily oxidized to form carbon dioxide, releasing energy in the process. It can be
metabolized via aerobic (cellular respiration) and anaerobic (fermentation) means,
whereby the cell uses the by-products of these reactions to produce ATP, the cell’s
energy currency.
7 Which portion of the electromagnetic radiation originating from the sun is harmful to
living tissues?
A Blue
B Green
C Infrared
D Ultraviolet
Solution The solution is (D). Ultraviolet waves are high-energy waves that penetrate tissues
and damage cells and DNA. The resulting damage to DNA from ultraviolet radiation
can lead to the formation of thymidine dimers, a mutation that can potentially lead

Advanced Placement Biology Instructor’s Solution Manual


8 | Photosynthesis 167

to cancer if many are accumulated within a cell and are not repaired through the
cell’s DNA repair mechanisms.
8 The amount of energy in a wave can be measured using what trait?
A Color intensity
B Distance from trough to crest
C The amount of sugar produced
D Wavelength
Solution The solution is (D). The amount of energy is the distance measured by two
successive points in the wave that are characterized by same phase, called the
wavelength.
9 What portion of the electromagnetic radiation emitted by the sun has the least energy?
A Gamma
B Infrared
C Radio
D X-rays
Solution The solution is (C). Radio waves possess the least energy in the electromagnetic
spectrum and have long wavelengths.
10 What is the function of carotenoids in photosynthesis?
A They supplement chlorophyll absorption.
B They are visible in the fall during leaf color changes.
C They absorb excess energy and dissipate it as heat.
D They limit chlorophyll absorption.
Solution The solution is (D). Carotenoids absorb excess energy and dissipate it as heat. In
photosynthesis, carotenoids serve as an efficient molecule for the disposal of excess
energy. Excess energy can damage the leaf; therefore, carotenoids, which reside in
thylakoid membranes, absorb excess energy and dissipate that energy as heat.
11 Which structure is NOT a component of a photosystem?
A Antenna molecule
B ATP synthase
C Primary electron acceptor
D Reaction center
Solution The solution is (B). ATP synthase is not a component of photosystem. It is present in
thylakoid membranes of chloroplasts that create ATP during photosynthesis.

Advanced Placement Biology Instructor’s Solution Manual


168 8 | Photosynthesis

12 Which complex is NOT involved in producing the electrochemical gradient for ATP
synthesis?
A Antenna complex
B Cytochrome complex
C Photosystem I
D Photosystem II
Solution The solution is (C). Photosystem I is not involved in the establishment of conditions
for ATP synthesis because it captures energy to reduce NADP+ into NADPH.
13 What can be calculated from a wavelength measurement of light?
A A specific portion of the visible spectrum
B Color intensity
C The amount of energy of a wave of light
D The distance from trough to crest of the wave
Solution The solution is (C). The amount of energy of the wave can be calculated by its
wavelength. It is the distance between two consecutive points of a wave.
14 Which molecule must enter the Calvin cycle continually for the light-independent
reactions to take place?
A CO2
B RuBisCO
C RuBP
D 3-PGA
Solution The solution is (A). CO2 is required to be fixed and converted into organic form
(glyceraldehyde 3 phosphate to glucose) continually for use by plants. Stage 1 of
carbon fixation initiates in the presence of CO2, which is used to convert inorganic
carbon to an organic compound.
15 Which order of molecular conversions is correct for the Calvin cycle?
A RuBP + G3P → 3-PGA → sugar
B RuBisCO → CO2 → RuBP → G3P
C RuBP + CO2 → [RuBisCO]3-PGA → G3P
D CO2 → 3-PGA → RuBP → G3P
Solution The solution is (C). RuBP and CO2 are needed with the enzyme RuBisCO to produce
3-PGA molecules. ATP and NADPH are used to convert 3-PGA molecules into G3P.

Advanced Placement Biology Instructor’s Solution Manual


8 | Photosynthesis 169

16 Which statement correctly describes carbon fixation?


A The conversion of CO2 into an organic compound
B The use of RuBisCO to form 3-PGA
C The production of carbohydrate molecules from G3P
D The use of ATP and NADPH to reduce CO2
Solution The solution is (A). Carbon fixation is the term used to describe the conversion of
inorganic carbon into organic form. The conversion of CO2 (inorganic form) into an
organic compound (3-PGA) in stage 1 of the Calvin cycle is an example of carbon
fixation.
17 Which substance catalyzes carbon fixation?
A 3-PGA
B NADPH
C RuBisCO
D RuBP
Solution The solution is (C). RuBisCO is an enzyme that is used to catalyze the reaction
between RuBP molecules and CO2 to produce an organic molecule called G3P. This
comprises stage 1 of the Calvin cycle and is called carbon fixation.
18 Which pathway is used by both plants and animals?
A Carbon fixation
B Cellular respiration
C Photosystem II
D Photosynthesis
Solution The solution is (B). Both plants and animals perform cellular respiration. The
difference is that animals get their food by eating it, while plants make their own
food by photosynthesis.
19 Which organism is a heterotroph?
A Cyanobacterium
B Intestinal bacterium
C Kelp
D Pond algae
Solution The solution is (B). Heterotrophs are essentially organisms that cannot “make” their
own food, and thus must obtain their nutrients through the ingestion or absorption
of organic sources of carbon. Of the available answer choices, intestinal bacteria
such as E. coli are heterotrophic.

Advanced Placement Biology Instructor’s Solution Manual


170 8 | Photosynthesis

20 What is the role of ribulose 1,5, bisphosphate carboxylase/oxygenase, abbreviated


RuBisCO, in photosynthesis?
A It catalyzes the reaction between CO2 and ribulose bisphosphate (RuBP).
B It catalyzes the reaction that produces glyceraldehyde3-phosphate (G3P).
C It catalyzes the reaction that regenerates RuBP.
D It catalyzes the reaction utilizing ATP and NADPH.
Solution The solution is (A). RuBisCO catalyzes the reaction between CO2 and ribulose
bisphosphate (RuBP).
21 What is the product of the Calvin cycle?
A Glucose
B Glyceraldehyde-3-phosphate
C Phosphoglycerate (PGA)
D Sucrose
Solution The solution is (B). In addition to glyceraldehyde-3-phosphate, three molecules of
adenosine diphosphate (ADP) and 2 molecules of NADP are produced. However, the
latter two products are regenerated and are reused in the light-dependent reactions
of photosynthesis.

CRITICAL THINKING QUESTIONS


22 What are the roles of ATP and NADPH in photosynthesis?
A ATP and NADPH are forms of chemical energy produced from the light-dependent
reactions to be used in the light-independent reactions that produce sugars.
B ATP and NADPH are forms of chemical energy produced from the light-independent
reactions to be used in the light-dependent reactions that produce sugars.
C ATP and NADPH are forms of chemical energy produced from the light-dependent
reactions to be used in the light-independent reactions that produce proteins.
D ATP and NADPH are forms of chemical energy produced from the light-dependent
reactions to be used in the light-independent reactions that use sugars as reactants.
Solution The solution is (A). ATP and NADPH are forms of chemical energy produced from the
light-dependent reactions to be used in the light-independent reactions that
produce sugars. NADPH and ATP are produced during the light-dependent reactions.
The energy captured during photosynthesis is used to convert ADP to ATP and to
reduce NADP to NADPH. The NADPH and ATP thus produced enter the light-
independent reactions.
23 What is the overall outcome of the light reactions in photosynthesis?

Advanced Placement Biology Instructor’s Solution Manual


8 | Photosynthesis 171

A NADPH and ATP molecules are produced during the light reactions and are used to
power the light-independent reactions.
B NADPH and ATP molecules are produced during the light reactions and are used to
power the light-dependent reactions.
C Sugar and ATP are produced during the light reactions and are used to power the
light-independent reactions.
D Carbon dioxide and NADPH are produced during the light reactions and are used to
power the light-dependent reactions.
Solution The solution is (A). NADPH and ATP molecules are produced during the light
reactions and are used to power the light-independent reactions. NADPH and ATP
molecules are the end products of the light reactions. The free energy stored in ATP
and NADPH is used to power light-independent reactions. PSII captures energy to
create protein gradients to make ATP, while PSI captures energy to reduce NADP +
to NADPH.
24 How does the equation relate to both photosynthesis and cellular respiration?

A Photosynthesis uses energy to build carbohydrates, while cellular respiration


metabolizes carbohydrates.
B Photosynthesis uses energy to metabolize carbohydrates, while cellular respiration
builds carbohydrates.
C Photosynthesis and cellular respiration both use carbon dioxide and water to produce
carbohydrates.
D Photosynthesis and cellular respiration both metabolize carbohydrates to produce
carbon dioxide and water.
Solution The solution is (A). Photosynthesis uses energy to build carbohydrates, while cellular
respiration metabolizes carbohydrates. Energy is needed to fuel the production of
carbohydrates and for the anabolic reactions of metabolism. Photosynthesis absorbs
light energy to build carbohydrates in chloroplasts, and aerobic cellular respiration
releases energy by using oxygen to metabolize carbohydrates in mitochondria and
cytoplasm. In photosynthesis, plants absorb sunlight to build carbohydrates from

Advanced Placement Biology Instructor’s Solution Manual


172 8 | Photosynthesis

carbon dioxide and water. In cellular respiration, cells metabolize carbohydrates to


release energy. This released energy is used by cells for cellular activities.
25 How is the energy from the sun transported within chloroplasts?
A When photons strike photosystem (PS) II, pigments pass the light energy to
chlorophyll a molecules that excite an electron, which is then passed to the electron
transport chain. The cytochrome complex transfers protons across the thylakoid
membrane and transfers electrons from PS-II to PS-I. The products of the light-
dependent reaction are used to power the Calvin cycle to produce glucose.
B When photons strike PS I, pigments pass the light energy to chlorophyll, molecules
that excite electrons, which is then passed to the electron transport chain. The
cytochrome complex then transfers protons across the thylakoid membrane and
transfers electrons from PS-II to PS-I. The products of the light-dependent reaction are
used to power the Calvin cycle to produce glucose.
C When photons strike PS II, pigments pass the light energy to chlorophyll molecules
that in turn excite electrons, which are then passed to the electron transport chain.
The cytochrome complex transfers protons across the thylakoid membrane and
transfers electrons from PS-I to PS-II. The products of the light-dependent reaction are
used to power the Calvin cycle to produce glucose.
D When photons strike PS II, pigments pass the light energy to chlorophyll molecules
that excite electrons, which are then passed to the electron transport chain. The
cytochrome complex transfers protons across the thylakoid membrane and transfers
electrons from PS II to PS I. The products of the light-independent reaction are used to
power the Calvin cycle to produce glucose.
Solution The solution is (A). This is the most accurate description of the chemical processes
involved with photosynthesis. When a photon strikes PS II, pigments pass the light
energy to chlorophyll a molecules that excite an electron, which are then passed to
the electron transport chain. The cytochrome complex transfers protons across the
thylakoid membrane and transfers electrons from PS II to PS I. The products of the
light-dependent reactions are used to power the Calvin cycle to produce glucose.
26 Explain why X-rays and ultraviolet light wavelengths are dangerous to living tissues.
A UV and X-rays are high-energy waves that penetrate the tissues and damage cells.
B UV and X-rays are low-energy waves that penetrate the tissues and damage cells.
C UV and X-rays cannot penetrate tissues and damage the cells.
D UV and X-rays can penetrate tissues and thus do not damage the cells.
Solution The solution is (A). UV and X-rays are high-energy waves that penetrate the tissues
and damage cells. X-rays and UV light are higher in energy than the wavelengths
used for photosynthesis. These high-energy waves can penetrate tissues and
damage cells and DNA, explaining why both X-rays and UV rays can be harmful to
living organisms.

Advanced Placement Biology Instructor’s Solution Manual


8 | Photosynthesis 173

Ultraviolet rays and X-rays prove detrimental for plants’ cell function. They are high-
energy waves that penetrate the tissues and cause damage to the cells and DNA
of plants.
27 If a plant were to be exposed to only red light, would photosynthesis be possible?
A Photosynthesis would not take place.
B The rate of photosynthesis would increase sharply.
C The rate of photosynthesis would decrease drastically.
D The rate of photosynthesis would decrease and then increase.
Solution The solution is (B). When a plant is exposed to both red and far-red light, then a
phenomenon called the Emerson effect takes place. There is a sharp increase in the
rate of photosynthesis after chloroplasts are exposed to wavelengths of 670 nm (far-
red light for PS II) and 700 nm (red light for PS I). The Emerson effect means that
when a plant is exposed to both red light and far-red light, then there is a sharp
increase in the rate of photosynthesis.
28 Which statement describes the electron transfer pathway from photosystem II to
photosystem I in the light-dependent reactions?
A After splitting water in PS II, high-energy electrons are delivered through the
chloroplast electron transport chain to PS I.
B After splitting water in PS I, high-energy electrons are delivered though the chloroplast
electron transport chain to PS II.
C After the photosynthesis reaction, the released products like glucose help in the
transfer of electrons from PS II to PS I.
D After the completion of the light-dependent reactions, the electrons are transferred
from PS II to PS I.
Solution The solution is (A). After splitting water in PS II, high-energy electrons are delivered
though the chloroplast electron transport chain to PS I. The excited electron
removed from the photosystem must then be replaced. In PS II, the electron comes
from the splitting of water, which releases oxygen as a waste product. In PS I, the
electron comes from the chloroplast electron transport chain. The reaction center of
PS II (called P680) delivers its high-energy electrons, one at a time, to a primary
electron acceptor and through the electron transport chain to PS I. Photolysis of
water releases high-energy electrons and oxygen as a waste product. These high-
energy electrons are sent to PS I through the chloroplast electron transport chain
that reduces NADP+ to NADPH.
29 What will happen to a plant leaf that loses CO2 too quickly?
A There will be no effect on the rate of photosynthesis.
B Photosynthesis will slow down or possibly stop.

Advanced Placement Biology Instructor’s Solution Manual


174 8 | Photosynthesis

C Photosynthesis will increase exponentially.


D Photosynthesis will decrease and then increase.
Solution The solution is (B). Photosynthesis will slow down or possibly stop. Loss of gases,
especially of CO2, will affect photosynthesis in the leaf, and Calvin cycle will slow
down and eventually stop because of inadequate carbon to fix in the system. A
continuous supply of atmospheric CO2 is thus required.
30 Carbon in the form of CO2 must be taken from the atmosphere and attached to an existing
organic molecule in the Calvin cycle. Therefore, the carbon is bound to the molecule. The
products of the cycle only occur because of the added carbon.
What are the products of the Calvin cycle, and what is regenerated?
A The product of the Calvin cycle is glyceraldehyde-3 phosphate, and RuBP is
regenerated.
B The product of the Calvin cycle is glyceraldehyde-3 phosphate, and RuBisCO is
regenerated.
C The product of the Calvin cycle is a 3-PGA molecule, and glyceraldehyde-3 phosphate
is regenerated.
D The product of the Calvin cycle is glyceraldehyde-3 phosphate, and oxygen is
regenerated.
Solution The solution is (A). The product of the Calvin cycle is glyceraldehyde-3 phosphate,
and RuBP is regenerated. The carbon cycle is the cyclical use of carbon in organic
molecules for energy pathways and its release into the atmosphere after these
pathways have been completed. The carbon is then fixed from the atmosphere by
photosynthetic organisms to an organic molecule, and the cycle continues.
The Calvin cycle produces glyceraldehyde-3 phosphate. The cycle must be repeated
three times to yield the product. RuBP is generated from the G3P molecules that
remain in the cycle to fix more CO2.
31 How do desert plants prevent water loss from the heat, which would compromise
photosynthesis?
A By using CAM photosynthesis and by closing stomatal pores during the night
B By using CAM photosynthesis and by opening of stomatal pores during the night
C By using CAM photosynthesis and by keeping stomatal pores closed at all times
D By bypassing CAM photosynthesis and by keeping stomatal pores closed at night
Solution The solution is (B). Desert plants use CAM photosynthesis to reduce water loss. In
CAM photosynthesis, desert plants open their stomata at night to collect carbon
dioxide from the air. At night, temperatures are cooler, preventing a large amount of
water loss by transpiration. The collected carbon dioxide is then stored within the
leaves as malate. This allows the plants to close their stomata during the day,

Advanced Placement Biology Instructor’s Solution Manual


8 | Photosynthesis 175

reducing water loss by evapotranspiration while still having a supply of carbon


dioxide to use for photosynthesis.
32 Why are carnivores, such as lions, dependent on photosynthesis to survive?
A Because the prey of lions are generally herbivores, which depend on heterotrophs
B Because the prey of lions are generally smaller carnivorous animals, which depend on
nonphotosynthetic organisms
C Because the prey of lions are generally herbivores, which depend on autotrophs
D Because the prey of lions are generally omnivores, which depend only on autotrophs
Solution The solution is (A). The prey of lions are generally herbivores, which depend on
heterotrophs. These herbivores rely on the plants and other photosynthetic
organisms for food. Therefore, without photosynthetic organisms, the carnivore’s
prey would not be present. Lions generally prey on herbivorous animals, which are
dependent on photosynthetic organisms.
33 Why does it take three turns of the Calvin cycle to produce G3P, the initial product of
photosynthesis?
A To fix enough carbon to export one G3P molecule
B To fix enough oxygen to export one G3P molecule
C To produce RuBisCO as an end product
D To produce ATP and NADPH for fixation of G3P
Solution The solution is (A). In stage 3 of the Calvin cycle, RuBP, the molecule that starts the
cycle, is regenerated so that the cycle can continue. All of the extra G3P is used in
this regeneration process. Because the G3P exported from the chloroplast has three
carbon atoms, it takes three turns of the Calvin cycle to fix enough carbon to export
one G3P. G3P is produced after three turns of Calvin cycle because the carbon
dioxide molecule is incorporated one at a time, so the cycle must be completed
three times to produce a single three-carbon (G3P) molecule.

TEST PREP FOR AP® COURSES


34 Photosynthesis and cellular respiration are found throughout the eukaryotic world. They
are complementary to each other because they each use products of the other process.
What do the two pathways share?
A Chloroplasts and mitochondria
B Photosystems I and II
C The cytochrome complex
D Thylakoids

Advanced Placement Biology Instructor’s Solution Manual


176 8 | Photosynthesis

Solution The solution is (C). Photosynthesis and cellular respiration both require the
cytochrome complex for electron transport, but the roles in each process differ.
35 What evidence exists that the evolution of photosynthesis and cellular respiration support
the concept that there is a common ancestry for all organisms?
A All organisms perform cellular respiration using oxygen and glucose, which are
produced by photosynthesis.
B All organisms perform cellular respiration using carbon dioxide and glucose, which are
produced by photosynthesis.
C All organisms perform cellular respiration using oxygen and lipids, which are produced
by photosynthesis.
D All organisms perform cellular respiration using carbon dioxide and lipids, which are
produced by photosynthesis.
Solution The solution is (A). All organisms perform cellular respiration using oxygen and
glucose, which are produced by photosynthesis. The principal energy source used
throughout the planet is glucose or some form of carbohydrate. The ultimate source
of this chemical is photosynthesis, which is found in every organism that produces
glucose. The process of photosynthesis produces oxygen as a product that is used by
all organisms in cellular respiration. These two pathways form the foundation of
metabolism in nearly all of the organisms in existence. Primitive anaerobic
prokaryotic microorganisms are thought to be the earliest organisms developed,
with others following once photosynthesis allowed oxygen to exist in the
atmosphere. Cellular respiration then became possible and built on some of the
chemical reactions that already existed in photosynthesis.
36 What are the correct labels for the indicated parts of a chloroplast?

A A. stroma, B. outer membrane, C. granum, D. thylakoid, E. inner membrane


B A. outer membrane, B. stroma, C. granum, D. thylakoid, E. inner membrane
C A. outer membrane, B. stroma, C. granum, D. inner membrane, E. thylakoid
D A. stroma, B. outer membrane, C. inner membrane, D. granum, E. thylakoid

Advanced Placement Biology Instructor’s Solution Manual


8 | Photosynthesis 177

Solution The solution is (B). The correct labels are A. outer membrane, B. stroma, C. granum,
D. thylakoid, E. inner membrane.
37 What cellular features and processes are similar in both photosynthesis and cellular
respiration?
A Both processes are contained in organelles with single membranes, and both use a
version of the cytochrome complex.
B Both processes are contained in organelles with double membranes, and neither uses
a version of the cytochrome complex.
C Both processes are contained in organelles with double membranes and use a version
of the cytochrome complex.
D Both processes are contained in organelles with single membranes, and neither uses a
version of the cytochrome complex.
Solution The solution is (C). Both processes are contained in organelles with double
membranes (chloroplasts and mitochondria), and both use a version of the
cytochrome complex in their respective electron transport chains.
38 Why do the light-dependent reactions of photosynthesis take place in the thylakoid?
A Photosystem I is anchored to the membrane, but not photosystem II.
B The cytochrome complex requires a membrane for chemiosmosis to occur.
C The light-dependent reactions depend on the presence of carbon dioxide.
D Light energy is absorbed by the thylakoid membrane.
Solution The solution is (B). The cytochrome complex requires a membrane for chemiosmosis
to occur. The cytochrome complex in the thylakoid membrane of chloroplasts
facilitates ATP generation by the movement of hydrogen ions across the thylakoid
membrane. This ATP is further used to power the Calvin cycle.
39 Metabolic pathways both produce and use energy to perform their reactions. How does
the Calvin cycle help to harness, store, and use energy in its pathway?
A The Calvin cycle harnesses energy in the form of 6 ATP and 6 NADPH that are used to
produce fructose-3-phosphate (F3P) molecules. These store the energy captured from
photosynthesis. The cycle uses this energy to regenerate RuBP.
B The Calvin cycle harnesses energy in the form of 6 ATP and 6 NADPH that are used to
produce glyceraldehyde-3-phosphate (GA3P) molecules. These store the energy
captured from photosynthesis. The cycle uses this energy to regenerate RuBP.

Advanced Placement Biology Instructor’s Solution Manual


178 8 | Photosynthesis

C The Calvin cycle harnesses energy in the form of 3 ATP and 3 NADPH that are used to
produce glyceraldehyde-3-phosphate (GA3P) molecules. These store the energy
captured from photosynthesis. The cycle uses this energy to regenerate the RuBP.
D The Calvin cycle harnesses energy in the form of 6 ATP and 3 NADPH that are used to
produce glyceraldehyde-3-phosphate (GA3P) molecules. These store energy captured
from photosynthesis. The cycle uses this energy to regenerate RuBP.
Solution The solution is (B). The Calvin cycle harnesses energy in the form of the 6 ATP and 6
NADPH that are used to produce glyceraldehyse-3-phosphate (GA3P) molecules.
These store the energy captured from photosynthesis. The cycle uses this energy to
regenerate the RuBP that starts the cycle.
40 Based on the diagram, what would most likely cause a plant to run out of NADP?

A Missing the ATP synthase enzyme


B Exposure to light
C Lack of water preventing H+ and NADP+ from forming NADPH
D Not enough CO2
Solution The solution is (D). As you can see in the diagram, CO2 is the only listed option that is
directly involved in the Calvin cycle, which produces NADP+.
41 As temperatures increase, gases such as CO2 diffuse faster. As a result, plant leaves will
lose CO2 at a faster rate than normal. If the amount of light impacting on the leaf and the
amount of water available is adequate, predict how this loss of gas will affect
photosynthesis in the leaf.
A Loss of gases, mainly CO2 , will not affect photosynthesis in the leaf, as adequate
amounts of water and light are still present which will let the Calvin cycle run
smoothly.

Advanced Placement Biology Instructor’s Solution Manual


8 | Photosynthesis 179

B Loss of gases, mainly CO2 , will affect photosynthesis in the leaf, as the Calvin cycle will
become faster to compensate for the loss.
C Loss of gases, mainly CO2 , will not affect photosynthesis in the leaf, as stored
reservoirs of CO2 in the leaf can be utilized in such times.
D Loss of gases, mainly CO2 , will affect photosynthesis in the leaf, as the Calvin cycle will
slow down and possibly stop because of inadequate carbon to fix in the system.
Solution The solution is (D). Loss of gases, especially of CO2 , will affect photosynthesis in the
leaf as the Calvin cycle will slow down and eventually stop because of inadequate
carbon to fix in the system. A continuous supply of atmospheric CO2 is thus
required.
42 How do the cytochrome-complex components involved in photosynthesis contribute to
the electron transport chain?
A Photosystem I excites the electron as it moves down the electron transport chain into
photosystem II.
B Plastoquinone and plastocyanine perform redox reactions that allow the electron to
move down the electron transport chain into photosystem I.
C ATP synthase de-excites the electron as it moves down the electron transport chain
into photosystem I.
D RuBisCO excites the electron as it moves down the electron transport chain into
photosystem II.
Solution The solution is (B). Plastoquinone and plastocyanine perform redox reactions that
allow the electron to move down the electron transport chain into photosystem I.
The cytochrome complexes perform oxidation-reduction reactions that, in essence,
pass on the electron to the next stop on the ETC.
43 How do membranes in chloroplasts contribute to the organelles’ essential functions?
A The inner membrane contains the chemicals needed for the Calvin cycle and
components of the light-dependent reactions. The thylakoid membrane contains
photosystems I and II, as well as the enzyme NAD+ reductase.
B The inner membrane contains only the chemicals needed for the Calvin cycle. The
thylakoid membrane contains components of the light-dependent reactions,
photosystems I and II, and the enzyme NAD+ reductase.
C The inner membrane contains components of the light-dependent reactions as well as
photosystems I and II. The thylakoid membrane contains the chemicals needed for the
Calvin cycle and the enzyme NAD+ reductase.
D The inner membrane contains the chemicals needed for the Calvin cycle, components
of the light-dependent reactions, and photosystems I and II. The thylakoid membrane
contains the enzyme NAD+ reductase.

Advanced Placement Biology Instructor’s Solution Manual


180 8 | Photosynthesis

Solution The solution is (A). The inner membrane contains the chemicals needed for the
Calvin cycle and components of the light-dependent reactions. The thylakoid
membrane contains photosystems I and II, as well as the enzyme NAD+ reductase.
The inner membrane contains the chemicals needed for the Calvin cycle and
provides a home for the components of the light-dependent reactions. Photosystem
I and II are housed within the thylakoid membrane, as are the enzymes NAD +
reductase, which catalyze the reduction of NADP and ATP synthase, which plays an
important role in ATP production through the chemiosmosis process across the
membrane.
44 If the absorption spectrum of photosynthetic pigments was restricted to the green portion
of the spectrum, which pigment or pigments would be affected the least?
A Carotenoids
B Chlorophyll a
C Chlorophyll b
D Chlorophyll c
Solution The solution is (A). Carotenoids would not be affected if the absorption spectrum of
photosynthetic pigments were restricted to the green portion of the spectrum,
because carotenoids absorb violet-blue-green light.
45 How can the passage of energy from light until it is captured in the primary electron
acceptor be described?
A Chlorophyll molecules in the photosystems are excited and pass the energy to the
primary electron acceptor where the energy is used to excite electrons from the
splitting of water.
B Chlorophyll a molecules in the photosystems are excited and pass the energy to the
primary electron acceptor where the energy is used to excite electrons from the
splitting of water.
C Chlorophyll b molecules in the photosystems are excited and pass the energy to the
primary electron acceptor where the energy is used to excite electrons from the
splitting of water.
D Chlorophyll molecules in the photosystems absorb light and get excited in the primary
electron acceptor from where the energy is used to excite electrons from the splitting
of water.
Solution The solution is (A). The chlorophyll molecules in the photosystem are excited and
pass the energy to the primary electron acceptor where the energy is used to excite
electrons from the split of water. Excitation occurs in chlorophyll a molecules and
then energy is passed to the primary electron acceptor where the energy is used for
exciting electrons from the splitting of water.

Advanced Placement Biology Instructor’s Solution Manual


8 | Photosynthesis 181

SCIENCE PRACTICE CHALLENGE QUESTIONS


8.2 The Light-Dependent Reaction of Photosynthesis
46 On a hot, dry day, plants close their stomata to conserve water. Explain the connection
between the oxidation of water in photosystem II of the light-dependent reactions and
the synthesis of glyceraldehyde-3-phosphate (G3PA) in the light-independent reactions.
Predict the effect of closed stomata on the synthesis of G3PA, and justify the prediction.
Solution Sample answer: Feedback is used to maintain water within cells by closing stomata.
When water cannot be lost by vaporization, the redox couple between water and
NADP+ is broken: Electrons cannot be delivered to reduce NADP+. The light-
independent reactions then must stop since they are regulated by the supply of
NADPH. The product of the light-independent reactions is G3PA.
47 The emergence of photosynthetic organisms is recorded in layers of sedimentary rock
known as a banded iron formation. Dark-colored and iron-rich bands composed of
hematite (Fe2O3) and magnetite (Fe3O4) only a few millimeters thick alternate with light-
colored and iron-poor shale or chert. Hematite and magnetite can form precipitates from
water that have high concentrations of dissolved oxygen. Shale and chert can form under
conditions that have high concentrations of carbonates (CO3−2). These banded iron
formations appeared 3.7 billion years ago (and became less common 1.8 billion years
ago). Justify the claim that these sedimentary rock formations reveal early Earth
conditions.
Solution Sample answer: These systems indicate formation from waters that fluctuated over
time in the dominant dissolved oxygen-containing chemical species. Based on these,
it may be reasoned that the atmosphere was oscillating between carbon dioxide rich
and oxygen rich. Because oxygen is a product of photosynthesis that consumes
carbon dioxide, the formations can reveal the emergence on early Earth of
photosynthetic organisms. When a high-oxygen atmosphere eventually became
stable, the banded iron formation ceased.
48 The diagram summarizes the light reactions of photosynthesis.

Advanced Placement Biology Instructor’s Solution Manual


182 8 | Photosynthesis

The diagram shows light-dependent reactions of photosynthesis, including the reaction


centers, electron transport chains, and the overall reactions within each of these. The free
energy per electron is shown for the oxidation-reduction reactions.
The free change of the captured radiant energy is shown.

A. In the overall mass balance equation for the light reactions shown above, identify the
source of electrons for the synthesis of NADPH.
B. Calculate the number of electrons transferred in this reaction.
C. Using the free energies per electron displayed, calculate the free-energy change of the
light-dependent reactions.
D. Given that the free-energy change for the hydrolysis of ATP is kJ/mole and the
+
free-energy change for the formation of NADPH from NADP is 18 kJ/mole, calculate the
total production of free energy for the light reactions.
E. Using this definition of energy efficiency, calculate the efficiency of the light reaction of
photosynthesis: energy efficiency = free energy produced/energy input.
Solution Sample answer:
A. The source of the electrons in the light-dependent reactions of photosynthesis is
the oxygen atom in water.

Advanced Placement Biology Instructor’s Solution Manual


8 | Photosynthesis 183

B. Each oxygen atom loses two electrons, so four electrons are transferred in this
reaction.
C. The free energy per electron of the conversion of NADP+ to NADPH is identified
in the chart as kJ/electron. The free energy per electron of the conversion
of water to molecular oxygen is 77 kJ/electron. The change in free energy is
There are four electrons transferred, so the overall
free energy change is 420 kJ.
D. There are three ATP produced from three ADP with a free energy change of 94.5
kJ. There are two NADPH produced from two NADP+ for a total of 36 kJ. Then the
free energy captured in chemical bonds of ATP and NADPH is 131 kJ.
E. The total free-energy change of . You often see an
estimate of 20 percent efficiency. The radiant-energy input according to the diagram
is 2,800 kJ and . The true photosynthetic efficiency is more difficult
to determine; the number of photons captured is only approximately eight. The free
energies that are available from tables are for pH of 7 and standard temperature
and pressure.
49 Algae can be used for food and fuel. To maximize profit from algae production under
artificial light, researchers proposed an experiment to determine the dependence of the
efficiency of the process used to grow the algae on light intensity (“brightness”) that will
be purchased from the electric company.
The algae will be grown on a flat sheet that will be continuously washed with dissolved
carbon dioxide and nutrients. Light-emitting diodes (LEDs) will be used to illuminate the
growth sheet. Photodiodes placed above and below the sheet will be used to detect light
transmitted through and reflected from the algal mat. The intensity of light can be varied,
and the algae can be removed, filtered, and dried. The amount of stored energy in the
algal mats can be determined by calorimetry.

Advanced Placement Biology Instructor’s Solution Manual


184 8 | Photosynthesis

A. Identify a useful definition of efficiency for this study and justify your choice.
B. Frequencies of light emitted by the LEDs will not be variables but must be specified for
the construction of the apparatus. Identify the frequencies of light that should be used in
the experiment, and justify your choice.
C. Evaluate the claim that the experiment is based on the assumption that there is an
upper limit on the intensity of light used to support growth of algae. Predict a possible
effect on algal growth if light with too great an intensity is used, and justify the prediction.
D. Design an experiment by describing a procedure that can be used to determine the
relationship between light intensity and efficiency.
Solution Sample answer:
A. The product is biomass, and the component of the cost that is considered here is
the price paid for the brightness of light used to grow the algae. So, efficiency could
be defined as the value of the biomass divided by the cost of electrical energy. It
could be defined as the energy contained in the algae divided by the energy used to
produce light. It could be defined as the profit from the sale of the biomass (or its
derivatives) divided by the cost of the power.
B. LEDs with a distribution of wavelengths matched to the absorbance maxima for
chlorophyll a should be chosen.
C. Light with too great an intensity can damage tissue, denature enzymes, etc. Light
produces heat, and thermoregulation is required for homeostasis.
D. Seed the mat with a fixed volume at fixed concentration of algae. At fixed
frequency, irradiate algae for a fixed period of time. Remove the algal mat and
measure biomass. Repeat varying light intensity. Then repeat this sequence at fixed
light intensity, varying pulse frequency.
50 The classical theory of evolution is based on a gradual transformation, the accumulation
of many random mutations that are selected. The biological evidence for evolution is
overwhelming, particularly when one considers what has not changed: core conserved
characteristics.
A. Describe three conserved characteristics common to both chloroplasts and
mitochondria.
Some hypotheses that have been proposed to account for biological diversity are
saltatory, involving sudden changes, rather than gradualist. In defense of the classical
gradualist theory of evolution, nearly all biologists in the late 1960s rejected the theory of
endosymbiosis as presented by Lynn Margulis in 1967.

Advanced Placement Biology Instructor’s Solution Manual


8 | Photosynthesis 185

B. Suppose that you want to disprove the theory of endosymbiosis.


Explain how the following evidence could disprove the theory:
i. A “transitional species” with cellular features that are intermediate cells with
and without mitochondria
ii. A “transitional organelle” with some features, such as compartmentalized
metabolic processes, but not other features, such as DNA
Explain how the following evidence supports the theory of endosymbiosis:
iii. Bacteria live within your intestines, but you still have a separate identity.
iv. No one has directly observed the fusion of two organisms in which a single
organism results.
Solution Sample answer:
A. The following could be identified:
 DNA and DNA processing
 Inner/outer membrane
 ATP- and NADH-based energetics
 Chemiosmosis including ETS and multiple analogous proteins
B.
i. A transitional species would have intermediate structural and functional
features. The existence of other gradual variations of this transitional species
would mean that the simplest solution would be gradualist.
ii. Similarly, a transitional organelle—for example, something like a glycosome
or a metabolon—that, with a molecular phylogeny such as protein
homologues, would disprove endosymbiosis with the accumulation of
sufficient evidence.
iii. Endosymbiosis is not the same as symbiosis. In this context, the organism has
systems with integrated reproduction and growth in which critical, obligatory
functions are provided by one or the other.
iv. Evidence of evolution is often obtained by observation of the necessary
consequences of an event without observation of the event. So, absence of
direct evidence is not evidence of absence.
8.3 Using Light Energy to Make Organic Molecules
51 Discovering the carbon-fixation reactions (or light-independent reactions) of
photosynthesis earned Melvin Calvin a Nobel Prize in 1961. The isolation and
identification of the products of algae exposed to 14C revealed the path of carbon in
photosynthesis. 14C was fed to the algal culture in the form of a bicarbonate ion (HCO3−).

Advanced Placement Biology Instructor’s Solution Manual


186 8 | Photosynthesis

To agitate the culture, air, which contains CO2, was bubbled through the system, so there
were two sources of carbon.
Since Calvin’s experiment, research has focused on the way carbon from a solution
containing bicarbonate ions is absorbed by algae. In aqueous solutions, the bicarbonate
anion (HCO3−) is in equilibrium with dissolved CO2, as shown in the equation.

In a later experiment, Larsson and Axelsson (1999) used acetazolamide (AZ), a carbonate
anhydrase inhibitor, to inhibit enzymes that convert bicarbonate into carbon dioxide.
They also used disulfonate (DIDS), an inhibitor of the transport of anions, such as the
bicarbonate ion, through the plasma membrane.
A. Pose a scientific question that can be pursued with AZ and DIDS in terms of the path of
carbon in photosynthesis.
B. The plasma membrane is permeable to the nonpolar, uncharged carbon dioxide
molecule. However, the concentration of carbon dioxide in solution can be very small.
Explain how the enzyme carbonate anhydrase can increase the availability of carbon
dioxide to the cell.
C. Larsson Sand Axelsson conducted experiments in which the growth medium was fixed
at two different pH levels and determined the effects of AZ and DIDS on the rate of
photosynthesis by measuring oxygen concentrations at various times. The results are
shown in the side-by-side graphs. The arrows indicate the time points during which
HCO3−, AZ, and DIDS were added to each system.

In which system, A or B, is there a strong reliance on the bicarbonate ion as the


mechanism of carbon uptake by the cell? Justify your answer using the data.
D. If both systems are dosed with the same concentrations of bicarbonate ion, in which
system, A or B, is the pH higher? Justify your answer using the data and the bicarbonate-
carbon dioxide equilibrium equation.

Advanced Placement Biology Instructor’s Solution Manual


8 | Photosynthesis 187

Solution Sample answer:


A. If AZ and DIDS are introduced into the light-independent reactions of
photosynthesis in the middle of the reaction, how would the introduction of these
inhibitors impact the amount of carbon produced?
B. The enzyme resides on the exterior surface of the cell and releases carbon dioxide
that is nonpolar and penetrates the membrane. The charged bicarbonate ion does
not passively enter the cell.
C. The data show that as bicarbonate is added, the equilibrium concentration of
carbon dioxide increases and photosynthesis (detected by the production of oxygen)
increases. However, some of that increase is due to the action of the anhydrase
enzyme and the relative amounts of each pathway (entry as CO2 and entry as
HCO3−), which are indicated by the slopes of the curve before and after the addition
of AZ. In system A, the addition of AZ decreased the rate by a factor of almost 10;
thus, under the conditions for system A, most C entered the cell as CO2—produced
by the surface-bound enzyme. After the addition of AZ, system A gets CO2 from that
dissolved in solution. DIDS has no effect, so in system A there is almost no entry of
bicarbonate into the cell. In system B, carbonate is the predominate source of
carbon. After the addition of DIDS, photosynthesis shuts down.
D. The results described in (C) tell us that system A is at a higher pH where,
according to the equilibrium equation, the concentration of dissolved carbon
dioxides is high. System B is at a higher pH where the concentration of dissolved
carbon dioxide is small.

Advanced Placement Biology Instructor’s Solution Manual


188 9 | Cell Communication

9 | CELL COMMUNICATION
REVIEW QUESTIONS
1 Which property prevents the ligands of cell-surface receptors from entering the cell?
A The molecules bind to the extracellular domain.
B The molecules are hydrophilic and cannot penetrate the hydrophobic interior of the
plasma membrane.
C The molecules are attached to transport proteins that deliver them through the
bloodstream to target cells.
D The ligands are able to penetrate the membrane, directly influencing gene expression
upon receptor binding.
Solution The solution is (B). The molecules are hydrophilic and cannot penetrate the
hydrophobic interior of the plasma membrane. Hydrophobic ligands can easily cross
the lipid bilayer, bringing about changes in gene expression. Hydrophilic ligand
molecules cannot cross the plasma membrane and bind the cell-surface receptor.
2 The secretion of hormones by the pituitary gland is an example of which type of signaling?
A Autocrine signaling
B Direct signaling across gap junctions
C Endocrine signaling
D Paracrine signaling
Solution The solution is (C). The secretion of hormones by the pituitary gland is an example of
endocrine signaling helping in long-distance signaling.
3 Why are ion channels necessary to transport ions into or out of a cell?
A Ions are too large to diffuse through the membrane.
B Ions are charged particles and cannot diffuse through the hydrophobic interior of the
membrane.
C Ions bind to hydrophobic molecules within the ion channels.
D Ions bind to carrier proteins in the bloodstream, which must be removed before
transport into the cell.
Solution The solution is (B). Ions are charged particles and cannot diffuse through the
hydrophobic interior of the membrane. Ions are charged particles and therefore
cannot diffuse through the membrane and require ion channels for their transport.

Advanced Placement Biology Instructor’s Solution Manual


9 | Cell Communication 189

4 Why are endocrine signals transmitted more slowly than paracrine signals?
A The ligands are transported through the bloodstream and travel greater distances.
B The target and signaling cells are close together.
C The ligands are degraded rapidly.
D The ligands do not bind to carrier proteins during transport.
Solution The solution is (A). The ligands are transported through the bloodstream and travel
greater distances. In endocrine signaling, the ligands are transported at long
distances.
5 Aldosterone is a steroid hormone that regulates reabsorption of sodium ions in the kidney
tubular cells. What is the probable mechanism of action of aldosterone?
A It binds gated ion channels and causes a flow of ions in the cell.
B It binds cell surface receptors and activates synthesis of cAMP.
C It binds to cell surface receptors and activates a phosphorylation cascade.
D It binds to an intracellular receptor and activates gene transcription.
Solution The solution is (D). It binds an intracellular receptor and activates gene transcription.
Aldosterone is a steroid hormone that binds to intracellular receptors and promotes
gene expression.
6 The gas nitric oxide has been identified as a signaling molecule. Which mechanism of
action would you expect from a gaseous molecule?
A It binds to a G-protein-linked receptor.
B It binds to a receptor tyrosine kinase.
C It binds to a gated ion channel.
D It binds to an intracellular receptor.
Solution The solution is (D). It binds to an intracellular receptor. Nitric oxide is a gas that can
easily diffuse through the cell membrane. It binds to the intracellular receptors and
brings about effector functions.
7 How do DAG and IP3 originate?
A They are formed by phosphorylation of cAMP.
B They are ligands expressed by signaling cells.
C They are hormones that diffuse through the plasma membrane to stimulate protein
production.
D They are the cleavage products of the inositol phospholipid, PIP2.

Advanced Placement Biology Instructor’s Solution Manual


190 9 | Cell Communication

Solution The solution is (D). They are the cleavage products of the inositol phospholipid, PIP 2.
The enzyme phospholipase C cleaves PIP2 to form diacylglycerol (DAG) and inositol
triphosphate (IP3).
8 What property enables the residues of the amino acids serine, threonine, and tyrosine to
be phosphorylated?
A They are polar.
B They are nonpolar.
C They contain a hydroxyl group.
D They occur more frequently in the amino acid sequence of signaling proteins.
Solution The solution is (C). They contain a hydroxyl group. Serine, threonine, and tyrosine
are hydroxyl-group-containing amino acids, which enables them to be easily
phosphorylated.
9 Dopamine is a neurotransmitter in the brain that causes long-term responses in neurons
and binds to a G-protein-linked receptor. Which chemical would you expect to increase in
concentration after dopamine binds its receptor?
A ATP
B cAMP
C Calcium ions
D Sodium ions
Solution The solution is (B). When dopamine binds to the G-protein-coupled receptor, ATP is
hydrolyzed by adenylyl cyclase to produce cAMP, which acts as a second messenger.
10 The hormone insulin binds to a receptor tyrosine kinase on the surface of target cells.
Which step takes place before phosphorylation of tyrosine residues?
A A tyrosine kinase enzyme must be activated.
B GDP is exchanged for GTP.
C The receptor forms a dimer.
D The insulin molecule is internalized in the cytoplasm.
Solution The solution is (C). The receptor forms a dimer in response to ligand binding, which
activates the tyrosine kinase activity of the receptor, leading to auto-
phosphorylation of the tyrosine residues on the receptor.
11 What is the function of a phosphatase?
A A phosphatase removes phosphorylated amino acids from proteins.
B A phosphatase removes the phosphate group from phosphorylated amino acid
residues in a protein.

Advanced Placement Biology Instructor’s Solution Manual


9 | Cell Communication 191

C A phosphatase phosphorylates serine, threonine, and tyrosine residues.


D A phosphatase degrades second messengers in the cell.
Solution The solution is (B). A phosphatase removes the phosphate group from
phosphorylated amino acid residues in a protein. The phosphate group from a
phosphorylated amino acid is removed by a phosphatase that helps in altering the
activity of a protein.
12 How does NF-κB induce gene expression?
A A small, hydrophobic ligand binds to NF-κB, activating it.
B NF-κB is phosphorylated and is then free to enter the nucleus to bind DNA.
C NF-κB is a kinase that phosphorylates a transcription factor that binds DNA and
promotes protein production.
D Phosphorylation of the inhibitor IκB dissociates the complex between it and NF-κB,
allowing NF-κB to enter the nucleus and stimulate transcription.
Solution The solution is (D). Phosphorylation of the inhibitor Iκ-B dissociates the complex
between it and NF-κB, allowing NF-κB to enter the nucleus and stimulate
transcription. When Iκ-B is bound to NF-κB, this complex cannot enter the nucleus.
When Iκ-B is phosphorylated by PKC, it dissociates from the complex, releasing
NF-κB. NF-κB acts as a transcription factor that enters the nucleus and initiates
transcription.
13 Apoptosis can occur in a cell under what conditions?
A When a cell is infected by a virus
B When a cell is damaged
C When a cell is no longer needed
D All of the above
Solution The solution is (D). Apoptosis occurs when a cell is no longer needed, is damaged, or
is infected by a virus.
14 Cancer cells that continue to divide when defective often show changes in what cellular
function?
A Apoptosis
B Their mechanism of glycolysis
C The mechanism of protein biosynthesis
D Replication of DNA
Solution The solution is (A). Cancer cells show defects in apoptosis. Although they are
defective, they do not undergo cell suicide.

Advanced Placement Biology Instructor’s Solution Manual


192 9 | Cell Communication

15 Epinephrine mediates the fight-or-flight response of the body. One of the effects is to
increase the amount of glucose available to muscles.
What does the signaling pathway triggered by epinephrine cause to occur in liver cells?
A Activation of metabolism
B Cell division
C Inhibition of glucose metabolism by liver cells
D Synthesis of enzymes
Solution The solution is (A). Binding of epinephrine leads to increase in metabolic rate.
Epinephrine signaling results in breakdown of glycogen and an increase in
concentration of glucose.
16 Which type of molecule acts as a signaling molecule in yeasts?
A Autoinducer
B Mating factor
C Second messenger
D Steroid
Solution The solution is (B). When a haploid yeast cell is ready to mate, it releases a signaling
molecule called mating factor, which binds to the receptors present on the nearby
yeast cells. Binding of mating factor arrests the normal cell cycle of the nearby cell
and the cell enters the signaling cascade that includes protein kinases and GTP-
binding proteins that are similar to G-proteins.
17 When is quorum sensing triggered to begin?
A A sufficient density of bacteria is present.
B Bacteria release growth hormones.
C Bacterial protein expression is switched on.
D Treatment with antibiotics occurs.
Solution The solution is (A). A sufficient density of bacteria is present. When high cell density
is present, more autoinducers are present, which bind to receptors, leading to
enhanced gene expression and more production of autoinducers.
18 Yeast-releasing mating factor can be classified as which type of signal?
A Autocrine
B Endocrine
C Paracrine
D Gap junction

Advanced Placement Biology Instructor’s Solution Manual


9 | Cell Communication 193

Solution The solution is (C). Paracrine signaling helps communicate with nearby cells. Yeast
cells that are present in close proximity communicate through paracrine signaling.
19 The bioluminescent bacteria Vibrio fischeri produce luminescence only if the population
reaches a certain density. What is the advantage of an autoinducer?
A An autoinducer allows the producer to act independently of the presence of other
cells.
B An autoinducer does not diffuse away from the cell.
C An autoinducer allows a positive feedback loop, which increases the response in
proportion to the population size.
D An autoinducer presents no advantage for the cell.
Solution The solution is (C). An autoinducer allows a positive feedback loop, which increases
the response in proportion to the population size. Production of an autoinducer
allows a positive feedback loop for production of more autoinducer, which is directly
proportional to cell density.

CRITICAL THINKING QUESTIONS


20 What is the difference between intracellular signaling and intercellular signaling?
A Intracellular signaling occurs between cells of two different species. Intercellular
signaling occurs between two cells of the same species.
B Intracellular signaling occurs between two cells of same species. Intercellular signaling
occurs between cells of two different species.
C Intracellular signaling occurs within a cell. Intercellular signaling occurs between cells.
D Intracellular signaling occurs between cells. Intercellular signaling occurs within cells.
Solution The solution is (C). Intracellular signaling occurs within a cell. Intercellular signaling
occurs between the cells. Signaling between two cells is called intercellular signaling.
Communication that occurs within a cell is called intracellular signaling.

Advanced Placement Biology Instructor’s Solution Manual


194 9 | Cell Communication

21 What are the differences between internal receptors and cell-surface receptors?
A Internal receptors bind to ligands that are hydrophobic, and the ligand-receptor
complex directly enters the nucleus, initiating transcription and translation. Cell-
surface receptors bind to hydrophilic ligands and initiate a signaling cascade that
indirectly influences the making of a functional protein.
B Internal receptors bind to ligands that are hydrophilic, and ligand-receptor complex
directly enters the nucleus, initiating transcription and translation. Cell-surface
receptors bind to hydrophobic ligands and initiate a signaling cascade that indirectly
influences the making of a functional protein.
C Internal receptors bind to ligands that are hydrophobic and initiate the signaling
cascade that indirectly influences the making of a functional protein. Cell-surface
receptors bind to hydrophilic ligands, and a ligand-receptor complex directly enters
the nucleus, initiating transcription and translation.
D Internal receptors are integral membrane proteins that bind to hydrophobic ligands,
initiating a signaling cascade, which indirectly influences the making of a functional
protein. Cell-surface receptors bind to hydrophilic ligands, and the ligand-receptor
complex directly enters the nucleus, initiating transcription and translation.
Solution The solution is (A). Their ligands enter the cell to bind the receptor. The complex
formed by the internal receptor and the ligand then enters the nucleus, directly
affecting protein production by binding to the chromosomal DNA and initiating the
transcription of mRNA that codes for proteins. Cell-surface receptors, however, are
embedded in the plasma membrane; their ligands do not enter the cell. Binding of
the ligand to the cell-surface receptor initiates a cell-signaling cascade and does not
directly influence the making of proteins, though it may involve the activation of
intracellular proteins. Internal receptors bind to hydrophobic molecules. Receptor-
ligand complexes directly influence the synthesis of proteins by binding to DNA in
the nucleus. Cell-surface receptors bind to hydrophilic ligands, which cannot cross
the plasma membrane, and lead to a signaling cascade, indirectly influencing the
synthesis of functional proteins.
22 Cells grown in the laboratory are mixed with a dye molecule that is unable to pass
through the plasma membrane. If a ligand is added to the cells, the dye is observed
entering the cells.
What type of receptor did the ligand bind to on the cell surface?
A G-protein-linked R receptor
B Ligand-gated ion channel
C Voltage-gated ion channel
D Receptor tyrosine kinase
Solution The solution is (B). An ion channel-linked receptor opened a pore in the membrane,
which allowed the ionic dye to move into the cell. Binding of the ligand to the ligand-

Advanced Placement Biology Instructor’s Solution Manual


9 | Cell Communication 195

gated ion channel receptor will lead to conformational changes in the receptor,
leading to formation of a pore in the membrane that allows the dye to move in
along with the ligand.
23 The same second messengers are used in many different cells, but the response to second
messengers is different in each cell. How is this possible?
A Different cells produce the same receptor, which bind to the same ligands, but have
different responses in each cell type.
B Cells produce variants of a particular receptor for a particular ligand through
alternative splicing, resulting in different responses in each cell.
C Cells contain different genes, which produce different receptors that bind to same
ligand, activating different responses in each cell.
D Cells produce different receptors that bind to the same ligand or the same receptor
that binds to the same ligand with different signaling components, activating different
responses in each cell.
Solution The solution is (D). Different cells produce different proteins, including cell-surface
receptors and signaling pathway components. Therefore, they respond to the same
ligands differently because the second messengers activate different pathways.
Signal integration can also change the end result of signaling. Different receptors in
different cells bind to same ligand and produce different responses.
24 What would happen if the intracellular domain of a cell-surface receptor was switched
with the domain from another receptor?
A It would activate the pathway normally triggered by the receptor that contributed the
intracellular domain.
B It would activate the same pathway even after the intracellular domain is changed
with the domain from another receptor.
C The receptor will be mutated and become nonfunctional, not activating any pathway.
D The receptor will become mutated and lead to continuous cell signaling, even in the
absence of a ligand.
Solution The solution is (A). The binding of the ligand to the extracellular domain would
activate the pathway normally activated by the receptor donating the intracellular
domain. The intracellular domain determines the response.
25 How would a chemical that blocks the binding of EGF to the EGFR interfere with the
replication of cancerous cells that overexpress EGFR?
A It will activate the EGFR pathway.
B It will block the EGFR pathway.
C It will have no effect, and the EGFR pathway will continue normally.
D It will lead to overexpression of the EGFR pathway.

Advanced Placement Biology Instructor’s Solution Manual


196 9 | Cell Communication

Solution The solution is (B). The chemical would interfere with the activation of the EGFR
pathway. A chemical that blocks the binding EGF to the EGFR would block the EGFR
pathway.
26 How does the extracellular matrix control the growth of cells?
A Contact of receptors with the extracellular matrix maintains equilibrium of the cell and
provides optimal pH for the growth of the cells.
B Contact of the receptor with the extracellular matrix helps maintain concentration
gradients across membranes, resulting in the flow of ions.
C The extracellular matrix provides nutrients for the cell.
D The extracellular matrix connects the cell to the external environment and ensures
correct positioning of the cell to prevent metastasis.
Solution The solution is (D). Receptors on the cell surface must be in contact with the
extracellular matrix in order to receive positive signals that allow the cell to live. If
the receptors are not activated by binding, the cell will undergo apoptosis. This
ensures that cells are in the correct place in the body and helps prevent invasive cell
growth, such as occurs in metastasis of cancer cells.
27 Which options gives an example for each one of the following effects of a cell signal: on
protein expression, cellular metabolism, and cell division?
A Protein expression: binding of epinephrine (adrenaline) to a G-protein-linked receptor;
cellular metabolism: the MAP-kinase cascade; cell division: promoted by the binding of
the EGF to its receptor tyrosine kinase
B Protein expression: the MAP-kinase cascade; cellular metabolism: binding of
epinephrine (adrenaline) to a G-protein-linked receptor; cell division: promoted by the
binding of the EGF to its receptor tyrosine kinase
C Protein expression: binding of the EGF to its receptor tyrosine kinase; cellular
metabolism: the MAP-kinase cascade; cell division: FAS-RAS signaling
D Protein expression: RAS signaling; cellular metabolism: binding of the EGF to its
receptor tyrosine kinase promotes an increase; cell division: binding of epinephrine
(adrenaline) to a G-protein-linked receptor
Solution The solution is (B). Protein expression: The MAP-kinase cascade is initiated by the
binding of a signal to a receptor tyrosine kinase. In the fight-or-flight response
caused by the binding of epinephrine (adrenaline) to a G-protein-linked receptor,
whose signaling pathway activates the enzymes involved in gluconeogenesis,
glycogen breaks down. Binding of the epidermal growth factor to its receptor
tyrosine kinase will start a signaling cascade such as the RAS-RAF-MEK-ERK pathway,
which leads to cell proliferation. The MAPK pathway leads to protein expression.
Epinephrine results in increased metabolic rate in cases of fight-or-flight. Epidermal
growth factor promotes cell division and proliferation.

Advanced Placement Biology Instructor’s Solution Manual


9 | Cell Communication 197

28 The mitogen-activated protein (MAP) kinase cascade triggered by RTKs results in cell
division. What are a few possible scenarios of abnormalities in the MAPK pathway leading
to uncontrolled cell proliferation?
A Gain of function mutation in RAS protein, mutation in Iκ-B, loss of function mutation in
genes for MAPK kinase pathway, regulated phosphorylation cascade
B Loss of function mutation in RAS protein and gain of function mutation in RAF protein,
Iκ -B permanently bound to NF-κB, regulated phosphorylation cascade
C RAS protein unable to hydrolyze its bound GTP, loss of function mutation in Iκ-B, gain
of function mutation in genes for MAPK kinase pathway, unregulated phosphorylation
cascade
D Unregulated phosphorylation cascade, loss of function mutation in RAS and RAF
protein, mutation in genes for MAPK kinase pathway, regulated phosphorylation
cascade
Solution The solution is (C). If the RAS protein is locked in the on position because it cannot
hydrolyze GTP, proliferation is not stopped. The Iκ-B molecule cannot bind to NF-κB
and prevent its activity. Overexpression of the genes for the MAPK kinase pathway
components can lead to overgrowth. Every step of the phosphorylation cascade
must be tightly regulated to avoid uncontrolled cell proliferation. If the RAS protein
is unable to hydrolyze its bound GTP, it will remain permanently active, loss of
function mutation in Iκ-B will not allow Iκ-B to bind to NF-κB, rendering it
permanently active, gain of function mutation in genes for MAPK kinase pathway
will lead to overexpression of the pathway, unregulated phosphorylation cascade.
All these situations will lead to uncontrolled cell division.
29 What characteristics make yeast a good model for learning about signaling in humans?
A Yeasts are prokaryotes. They have a short life cycle, are easy to grow, and share
similarities with humans in certain regulating mechanisms.
B Yeasts are eukaryotes. They have a short life cycle, are easy to grow, and share
similarities with humans in certain regulating mechanisms.
C Yeasts are single-celled organisms. They have a short life cycle, are easy to grow, and
share similarities with humans in certain regulating mechanisms.
D Yeasts are single-celled organisms. They have a complex life cycle like that of humans
and share similarities in regulating mechanisms.
Solution The solution is (C). Yeasts are eukaryotes and have many of the same systems that
humans do. However, they are single celled, so they are easy to grow, have a short
generation time, and are much simpler than humans.

Advanced Placement Biology Instructor’s Solution Manual


198 9 | Cell Communication

30 Why is signaling in multicellular organisms more complicated than signaling in single-


celled organisms?
A Multicellular organisms coordinate between distantly located cells; single-celled
organisms communicate only with nearby cells.
B Multicellular organisms involve receptors for signaling; single-celled organisms
communicate by fusion of plasma membrane with the nearby cells.
C Multicellular organisms require more time for signal transduction than single-celled
organisms because they show compartmentalization.
D Multicellular organisms require more time for signal transduction than single-celled
organisms because they lack compartmentalization.
Solution The solution is (A). Multicellular organisms must coordinate many different events in
different cell types, which may be very distant from each other. Single-celled
organisms are only concerned with their immediate environment and the presence
of other cells in the area. Multicellular organisms take more time in signal
transduction than single-celled organisms because they coordinate between cells
that are distantly located.
31 Biofilms are a prominent danger in infectious disease treatment today because it is
difficult to find drugs that can penetrate the biofilm. What characteristics would a drug
have if it aimed to prevent bacteria from forming biofilms in the first place? Explain your
answer.
Solution A drug aimed at preventing biofilm formation would block the receptors of auto-
inhibitors. By blocking auto-inhibitors, quorum sensing would not occur in bacterial
populations, and these colonies would not turn on genes that form biofilms. These
drugs may cause bacterial populations to increase because the bacteria would
continue to divide even after they reach “quorum”. Therefore, antibacterial drugs
that inhibit bacterial reproduction would have to be used in addition to an auto-
inhibitor blocker.
32 Supports the hypothesis that signaling pathways appeared early in evolution and are well
conserved using the yeast mating factor as an example.
A Signaling in yeast uses the RTK pathway and is evolutionarily conserved, like
epinephrine signaling in humans.
B Signaling in yeast uses G-protein-coupled receptors for signaling and is evolutionarily
conserved, like epinephrine signaling in humans.
C Signaling in yeast uses an endocrine pathway and is evolutionarily conserved, like
epinephrine signaling in humans.
D Mating factor in yeast uses an autocrine signaling pathway and is evolutionarily
conserved, like epinephrine signaling in humans.

Advanced Placement Biology Instructor’s Solution Manual


9 | Cell Communication 199

Solution The solution is (B). Signaling pathways have been identified in many bacteria,
protists, and fungi. The mating factor signaling pathway in yeast uses a G-protein-
linked receptor, a G-protein that activates a MAPK kinase cascade. Yeast produces
mating factor, which binds to the receptors present on the surrounding yeast cells,
and initiates a cell-signaling cascade including protein kinases and GTP-binding
proteins, which are similar to G-proteins. G-proteins are evolutionarily conserved
since they are used in various signaling mechanism in humans.

TEST PREP FOR AP® COURSES


33 Upon ingestion of bacteria, white blood cells release a chemical messenger into the blood
stream that causes the synthesis of inflammation response proteins by liver cells. What is
this is an example of?

Advanced Placement Biology Instructor’s Solution Manual


200 9 | Cell Communication

A Autocrine signaling
B Endocrine signaling
C Paracrine signaling
D Synaptic signaling
Solution The solution is (B). The chemical messenger is a hormone secreted by the endocrine
system.
34 Molecules do not flow between the endothelial cells in the brain capillaries. The
membranes of the cells must be joined by what?
A Gap junctions
B Ligand-gated channels
C Synapses
D Tight junctions
Solution The solution is (D). Tight junctions prevent the flow of molecules between
adjacent cells.
35 What are the possible benefits of having autocrine signaling?
A Autocrine signaling helps to communicate with distantly located cells.
B Autocrine signaling connects nearby cells.
C Autocrine signaling helps to amplify the signal by inducing more signaling production
from the cell itself.
D Autocrine signaling is specific only for the cell that produced it.
Solution The solution is (C). Autocrine signals may induce a cell to produce more signal
molecules by inducing proliferation and increasing the signal—for example, helper T.
Autocrine signals coordinate responses by groups of identical cells. Cancer cells can
use autocrine signals to escape the normal cell proliferation control and multiply
abnormally. Autocrine signaling involves the action of the hormone on the same cell
from which it was produced and helps in increasing the signal for a better response.
36 If a chemical is an inhibitor of the enzyme adenylyl cyclase, which step in the G-protein
signaling pathway would be blocked?
A Activation of gene transcription
B Exchange of GTP for GDP
C Ligand-bound receptor activation of G-protein
D Synthesis of cAMP
Solution The solution is (D). Adenylyl cyclase hydrolyzes ATP to ADP to synthesize cAMP.
Therefore, a chemical that inhibits the activity of adenylyl cyclase will affect the
formation of cAMP.

Advanced Placement Biology Instructor’s Solution Manual


9 | Cell Communication 201

37 Thyroid hormone is a lipid-soluble signal molecule that crosses the membrane of all cells.
Why would a cell fail to respond to the thyroid hormone?
A The MAPK cascade leading to cell activation is defective in the target cells.
B The DNA sequence it binds to underwent a mutation.
C There is no intracellular receptor for thyroid hormone in the cell.
D The second messenger does not recognize the signal from the receptor.
Solution The solution is (C). There is no intracellular receptor for thyroid hormone in the cell.
The thyroid hormone triggers its signaling pathway only in the presence of its
intracellular receptor; it would fail if this receptor were not found.
38 The poison from the krait snake’s bungarotoxin binds irreversibly to acetylcholine
receptors, interfering with acetylcholine binding at the synapse. What is the effect of
bungarotoxin binding on the post synaptic cell?
A cAMP production is inhibited.
B Bungarotoxin G-proteins are not activated.
C Ion movement in the cell is inhibited.
D Phosphorylation cascade is inhibited.
Solution The solution is (C). Ion movement in the cell is inhibited. Bungarotoxin binds
irreversibly to acetylcholine receptors, interfering with acetylcholine binding at the
synapse. Therefore, it will directly inhibit the passage of ions across the postsynaptic
nerve and the signal is not transmitted.
39 In autoimmune lymphoproliferative syndrome (ALPS), lymphocytes that multiplied during
an infection persist in the body and damage tissue. The syndrome is caused by a mutation
in the FAS gene, which encodes a cell-surface receptor.
Which signaling pathway does the receptor initiate?
A Activated metabolism
B Apoptosis
C Cell division
D Cell differentiation
Solution The solution is (B). The lymphocytes persist because they do not undergo apoptosis;
therefore, it is the pathway affected.

Advanced Placement Biology Instructor’s Solution Manual


202 9 | Cell Communication

40 Place the following events in their sequential order:


1. Protein kinase A is activated.
2. Glycogen breaks down.
3. Epinephrine binds to G-protein-linked receptor.
4. G-protein activates adenylyl cyclase.
5. GTP is exchanged for GDP on the G-protein.
6. ATP is converted into cAMP.
A 1, 3, 5, 4, 6, 2
B 3, 5, 4, 1, 6, 2
C 3, 4, 5, 1, 6, 2
D 3, 5, 4, 6, 1, 2
Solution The solution is (D). Epinephrine binds to G-protein-linked receptor, resulting in the
activation of G-protein by the exchange of GDP with GTP. Activated G-protein
stimulates protein kinase A and leads to glycogen breakdown.
41 The RAS protein is a G-protein connected with the response to RTKs that initiates the
MAPK kinase cascade when GDP is released and GTP uploaded. Mutations in the RAS
protein that interfere with its GTPase activity are common in cancer.
What is the connection between the inability of RAS to hydrolyze GTP and uncontrolled
cell proliferation?
A RAS, when bound to GTP, becomes permanently inactive even in the presence of the
ligand and no longer regulates cell division.
B RAS, when bound to GTP, becomes permanently active even in the absence of the
ligand and no longer regulates cell division.
C RAS, when bound to GTP, forms a dimer after binding to the ligand and causes
uncontrolled division, but it remains inactive when the ligand is absent.
D RAS, when bound to GTP, does not form a dimer after binding to the ligand but
stimulates downstream signaling to occur and causes uncontrolled cell division.
Solution The solution is (B). RAS is the first major step in the MAPK kinase cascade. If it is
always active, the transcription signals keep promoting cell division. When RAS is
unable to hydrolyze its bound GTP, it remains permanently active even in the
absence of the ligand, and cell undergoes unregulated division.

Advanced Placement Biology Instructor’s Solution Manual


9 | Cell Communication 203

42 Common medications called blockers bind to G-protein-linked receptors in heart


muscles, blocking adrenaline. They are prescribed to patients with high blood pressure.
Can you formulate a hypothesis on their mechanism of action?

A Adrenaline has a stimulatory effect on heart rate and blood pressure. blockers are
antagonistic to adrenaline and produce an inhibitory effect.
B Adrenaline has both a stimulatory and an inhibitory effect on heart rate and blood
pressure. blockers bind to G-protein and stimulate the inhibitory effect of
adrenaline.

C Adrenaline has an inhibitory effect on heart rate and blood pressure. blockers have
a synergistic effect along with adrenaline, producing an inhibitory effect.
D Adrenaline has both a stimulatory and an inhibitory effect on heart rate and blood
pressure. blockers bind to G-protein and intervene with the inhibitory effect of
adrenaline.
Solution The solution is (A). Adrenaline promotes rapid heartbeat, increasing cardiac output
and blood pressure. Adrenaline increases heart rate. blockers are provided to
patients with high blood pressure because they bind to G-protein and produce an
inhibitory effect, decreasing the blood pressure.

SCIENCE PRACTICE CHALLENGE QUESTIONS


9.1 Signaling Molecules and Cellular Receptors
43 The figure shows a series of states for typical G-protein signal transduction.

Use this representation to describe the following stages in this signaling process:
A. Between A and B
B. Between B and C
C. Between C and D
D. Between D and E
E. Between E and A

Advanced Placement Biology Instructor’s Solution Manual


204 9 | Cell Communication

Advanced Placement Biology Instructor’s Solution Manual


9 | Cell Communication 205

Solution Teaching Tip: Prior Knowledge: Chapters 3‒7


Sample answer:
A. Between A and B, the signal ligand attaches to the protein and induces a
conformational change in the subunit.
B. Between B and C, the GDP dissociates from the protein.
C. Between C and D, the high concentration of GTP within the cell causes the
replacement of GDP with GTP.
D. Between D and E, the subunit activated by GTP dissociates from the protein.
E. Between E and A, GTP dephosphorylates, returning the subunit to its original
configuration which then re-associates with protein.
9.2 Propagation of the Signal
44 Tyrosine kinase receptors are pairs of proteins that span the plasma membrane. On the
extracellular side of the membrane, one or more sites are present that bind to signaling
ligands such as insulin or growth factors. On the intracellular side, the ends of peptide
chains on each protein phosphorylate the other member of the pair, providing active
docking sites that initiate cellular responses. The signal is switched off by dissociation of
the ligand. For each ligand-receptor system, the equilibrium constant, k, controls the
distribution of receptor-bound and unbound ligands. In systems with large values of k, a
site is likely to be occupied, even at low concentrations of ligand. When k is small, the
likelihood of binding is low, even when the concentration of ligand is high. To initiate a
new stimulus response cycle for the receptor, the ligand must dissociate. Larger values of
k mean that the receptor is more likely to be occupied and thus unavailable to bind
another ligand.
Some ligand-binding systems have multiple binding sites. For example, hemoglobin binds
four oxygen molecules, whereas myoglobin has only a single binding site. When multiple
binding sites are present, the presence of an already-bound ligand can cooperatively
affect the binding of other ligands on the same protein. For hemoglobin, the binding is
positively cooperative. The affinity of oxygen for heme increases as the number of bound
oxygen molecules increases.

Advanced Placement Biology Instructor’s Solution Manual


206 9 | Cell Communication

A. Describe the features in the graph for hemoglobin that demonstrate positive
cooperativity.

B. The insulin receptor (IR) is a tyrosine kinase receptor that has two sites to which insulin
can attach. IR is negatively cooperative. In the graph in (A), the dependence of the bound
fraction on available insulin is similar to the curve for with negative cooperativity.
Describe the features of this curve in the graph that demonstrate negative cooperativity.
C. When viewed from above the cell surface, the representation shows receptors with one
and two bound insulin molecules. Explain the negative cooperation for this receptor
based on the free energy of conformational changes in the receptor-peptide chains.
D.

E. Explain the advantages in terms of selection of two-site binding with negative


cooperation relative to one-site binding.
F. Three binding curves with negative cooperativity and different values of k are shown on
the graph. Describe conditions in which there is an advantage in having a low value of k
with negative cooperativity.
Solution Sample answer:
A. Positive cooperativity is shown by the slope of the curve. The slope rises and then
falls as the sites become saturated.
B. Negative cooperativity is shown by the slope of the curve. The slope falls
immediately.
C. Free-energy expenditure is required for changes in protein configuration (binding
of the ligand). The second configuration requires more free energy because the
protein is now constrained by the attached insulin.

Advanced Placement Biology Instructor’s Solution Manual


9 | Cell Communication 207

D. Two binding sites can increase the range of response of the ligand-receptor
system. Very high concentrations of ligand can be managed when necessary and low
concentrations can be accommodated with less free-energy expenditure. The
affinity for the second ligand will also be lower so that dissociation will be faster and
the receptor will increase the frequency of stimulation-response.
E. As described for (D), if k is large, then the frequency of response-cycle completion
will be smaller. The message sent by the next ligand is wasted unless that receptor
has been restored to the original configuration. A smaller value of k shifts the
receptor toward a condition to receive the next signal.
45 Organisms, including plants, have evolved chemical signaling pathways to direct
physiological responses to environmental changes. Stomata are pores, typically on the
underside of leaves, that regulate CO2, O2, and H2O exchange between plants and the
external environment. This interaction controls photosynthetic rate and transpiration
rate. The opening and closing of stomata are controlled by specialized guard cells that
surround the stomatal pore. The osmotic state within the guard cells determines their
turgor; when the guard cells are flaccid, stomata close. Turgor in the guard cells is
regulated by the active transport of several ions, including K+ and H+, across the plasma
membrane. Several environmental factors can cause stomatal closing: water deficit,
darkness, microbes, ozone, and sulfur dioxide and other pollutants. Intracellular carbon
dioxide concentration and light can trigger stomata to open.
The system is regulated by a phytohormone (plant hormone) called abscisic acid (ABA)
and the amino acid precursor of the synthesis of a second phytohormone called ethylene
(ACC). The second messengers NO and in the signal response to changes in the
concentrations of these hormones activate transcription factors that affect ion transport
across guard cell membranes. High CO2 levels and light also alter phytohormone
concentrations.
A. Explain why plants must regulate the opening and closing of stomata. Explain how this
response relates to the capture of free energy for cellular processes.
B. Construct an explanation in terms of the water potential, Y, for the efflux (outward
flow) of H+ during water stress (drought).
C. Consider a scenario involving environmental factors, such as water stress and daylight,
which have opposing effects on the opening and closing of stomata; stomata would be
signaled to close under drought conditions and to open during photosynthesis. Pose two
scientific questions regarding the response of the system, one involving the
phytohormones ABA and ACC, and the second involving the concentration of second
messengers.
D. The data shown in the figure were obtained by treating rockcress (Arabidopsis) with
doses of ABA, ACC, and ABA plus ACC. Using the terms and and or, describe the expected
and unexpected responses of the system just after 10 min and around 45 min, as
displayed by these data.

Advanced Placement Biology Instructor’s Solution Manual


208 9 | Cell Communication

E. Researchers are investigating the interactions among multiple signaling pathways, a


phenomenon referred to as “crosstalk.” The same second messengers, NO and Ca2+, are
used in many different signaling pathways. Construct an explanation by analogy to other
phenomena in which combining a small set of events (for example, 0 and 1 in a computer,
the musical scale, or the R, G, and B components of a color) can lead to a vast assortment
of outcomes.
Solution Sample answer:
A. The rate of conversion of carbon dioxide and water to oxygen and C3 or C4 sugars
determines the rate of free-energy production.
B. As ions are pumped through the plasma membrane, the water potential within
the cell increases and the cell swells.
C. Possible answers: Researchers in the field are asking these questions:
 If these hormones lead to the same cellular response, then isn’t this
redundant?
 If these hormones lead to opposing cellular responses, then isn’t this
something that should have been selected against?
 Are the hormones present in a Y/N manner, or are their “quorum-like”
evaluations of their equilibrium concentrations that are operating to allow an
integrated signal—if there are n and m discrete concentration levels for each,
then the information content for a pair of interacting hormones increases as
the number
 Is there time dependence in the response of the system to each receptor
that produces cycles that contain further information? This seems to be the
direction that this work is taking.

Advanced Placement Biology Instructor’s Solution Manual


9 | Cell Communication 209

 Does the secondary messenger have a target that is near and the lifetime of
the messenger is short?
 Does each signaling pathway operate independently or are there interactions
that lead to additional information? This seems to be the direction that the
field is going, crosstalk among pathways.
Teaching Tip: Like all “pose a question,” this is going to require guidance from the
instructor if good questions are to be produced. By visiting small group discussions
and making clear that the questions have value and can be either good or bad (not
all questions are good questions), students can be encouraged to think scientifically.
D. Clearly the combination of the two produces a surprising effect: just after 10 min,
the signal to close the stomata is sent by both ABA and ACC and, as expected, a
signal ABA and ACC amplifies the effect. However, around 45 min, the stomata close
if the signal is ABA or ACC. But ABA and ACC open the stomata.
E. With just 3 colors, 27 combinations can be made. If there are 16 bits (possible
values of 0 or 1 in the representation of a number on a computer), 65,536 integer
values can be expressed. Thinking of each bit as receptor state (ligand present or not
present) demonstrates the potential range of cell responses.
9.3 Response to the Signal
46 Construct a graphical representation of information as a function of time during the
transduction of a signal along a signaling pathway.
B. Annotate your representation for a specific signaling system, such as the effect of
epinephrine on the free energy released from glucose.
Solution

Advanced Placement Biology Instructor’s Solution Manual


210 9 | Cell Communication

9.4 Signaling in Single-Celled Organisms


47 Bacteria and fungi produce several extracellular chemicals, including antibiotics that affect
other organisms in the environment. Antibiotics are also produced industrially in large
bacteria-containing fermentation tanks. However, antibiotics that have been used by
humans to control microbes are now found at subinhibitory concentrations in the
environment. Low levels of antibiotics in the environment are mutagenic for bacteria and
promote the development of antibiotic resistance.
Bacteria produce chemical signals that detect population density and regulate gene
expression, a phenomenon called quorum sensing. Density is signaled by the extracellular
concentration of small amino acid derivatives. To combat antibiotic resistance, an
emerging strategy for the control of bacterial disease is quorum quenching.
A. Describe the advantage of antibiotics to the organisms that produce them.
B. Based on the name of the emerging strategy for controlling bacterial infections,
describe a possible mechanism by which bacteria determine their population density.
Justify the claim that quorum quenching may provide a more sustainable approach to
disease control than the use of antibiotics.
Solution Sample answer:
A. Antibiotics are extracellular chemical signals. They are used as communication
within a species (there are also examples of communication between species called
eavesdropping) and to provide a competitive advantage for resources.
B. The success of genes that produce resistance leads to the long-term selection of
resistant bacterial strains. Even if antibiotic use were better regulated, subinhibitory
concentrations are found to induce mutations, and even with greater restraint, the
long-term effect is the lost effectiveness of these materials in the treatment of
disease associated with bacterial infection. If a molecule controlled behavior rather
than killed bacteria, a selective pressure would less likely drive evolution of bacteria.
The quorum signal would be turned off by a “quorum quencher,” such as an enzyme
that modifies the molecule that is sensed in the detection of a quorum.
A. Use your graph to describe trends in the amount of information rather than the actual
magnitude. In sketching your graph, consider how the shape of the curve would change
during these events:
i. Extracellular first messenger
ii. Receptor binding and conformational changes
iii. Release of second messengers
iv. Cellular responses
v. Halt signal and degrade intermediates

Advanced Placement Biology Instructor’s Solution Manual


10 | Cell Reproduction 211

10 | CELL REPRODUCTION
REVIEW QUESTIONS
1 A diploid cell has how many times the number of chromosomes as a haploid cell?
A Four times
B Half
C One-fourth
D Twice
Solution The solution is (D). In a diploid cell, the haploid number doubles when an offspring
receives one set from the father and another set from the mother.
2 The first level of DNA organization in a eukaryotic cell is maintained by which molecule?
A Cohesin
B Condensin
C Chromatin
D Histone
Solution The solution is (D). The histones are the alkaline proteins present in the eukaryotic
cells that the DNA strands are wound around, thereby packaging the DNA strand
into structural units known as nucleosomes.
3 What inherited feature, in specific combinations, determines an organism’s traits?
A Cell membranes
B Genes
C Proteins
D RNA
Solution The solution is (B). Each gene is a factor that is inherited from the parents by
offspring and possesses alleles of a single trait.
4 What are identical copies of chromatin held together by cohesin at the centromere
called?
A Histones
B Nucleosomes
C Chromatin
D Sister chromatids
Solution The solution is (D). Sister chromatids are held together at the kinetochore region by
cohesin, which assists them in cell division.

Advanced Placement Biology Instructor’s Solution Manual


212 10 | Cell Reproduction

5 Chromosomes are duplicated during what stage of the cell cycle?


A G1 phase
B Prophase
C Prometaphase
D S phase
Solution The solution is (D). In the S phase, DNA replication and centrosome duplication
occur. The DNA replication results in the formation of identical pairs of DNA, known
as sister chromatids.
6 Which event does NOT occur during some stages of interphase?
A DNA duplication
B Increase in cell size
C Organelle duplication
D Separation of sister chromatids
Solution The solution is (D). Interphase includes G0, G1, S, and G2 phases, which in the end
produces a chromosome made up of sister chromatids. It is in metaphase of the
mitotic phase that separation of sister chromatids occurs.
7 Attachment of the mitotic spindle fibers to the kinetochores is a characteristic of which
stage of mitosis?
A Anaphase
B Prophase
C Prometaphase
D Metaphase
Solution The solution is (C). During prometaphase, the individual chromosomes bind to the
mitotic spindle at the kinetochore and arrange themselves at the metaphase plate.
8 The fusing of Golgi vesicles at the metaphase plate of dividing plant cells forms what
structure?
A Actin ring
B Cell plate
C Cleavage furrow
D Mitotic spindle
Solution The solution is (B). In the plant cells during cytokinesis, the Golgi vesicles, which are
transported to form the phragmoplast at metaphase plate, fuse to form a structure
called a cell plate.

Advanced Placement Biology Instructor’s Solution Manual


10 | Cell Reproduction 213

9 What would be the outcome of blocking S phase of interphase?


A The cell would enter karyokinesis.
B DNA replication would not occur.
C Centrosomes would be duplicated.
D The cytoskeleton would be dismantled.
Solution The solution is (B). S phase is responsible for the replication or synthesis of DNA,
which, if blocked, would not copy DNA into sister chromatids.
10 At which cell cycle checkpoint does external forces have the greatest influence?
A G1 checkpoint
B G2 checkpoint
C M checkpoint
D G0 checkpoint
Solution The solution is (A). The G1 checkpoint is influenced as it checks for the presence of
essential nutrients and enzymes. The growth hormones and factors are essential to
inform the cell about the local conditions.
11 If the M checkpoint is NOT cleared, what stage of mitosis will be blocked?
A Prophase
B Prometaphase
C Metaphase
D Anaphase
Solution The solution is (D). The M checkpoint occurs just before the anaphase. It checks if all
the sister chromatids are properly attached to the kinetochore because the cells will
undergo separation in the anaphase.
12 Which protein is a positive regulator that phosphorylates other proteins when activated?
A p53
B Retinoblastoma protein (Rb)
C Cyclin
D Cyclin-dependent kinase (Cdk)
Solution The solution is (D). Cdk’s are protein kinases that, with the help of cyclins, have the
ability to phosphorylate other proteins when activated.

Advanced Placement Biology Instructor’s Solution Manual


214 10 | Cell Reproduction

13 Which negative regulatory molecule can trigger apoptosis if vital cell cycle events do
NOT occur?
A p53
B p21
C Retinoblastoma protein (Rb)
D Cyclin-dependent kinase (Cdk)
Solution The solution is (A). p53 is the major negative regulatory molecule since it has the
capacity to put the cell on halt or induce apoptosis during any DNA damage or if the
damage cannot be repaired.
14 What is the main prerequisite for clearance at the G2 checkpoint?
A The cell has reached a sufficient size.
B The cell has an adequate stockpile of nucleotides.
C An accurate and complete DNA replication has occurred.
D Proper attachment of mitotic spindle fibers to kinetochores has occurred.
Solution The solution is (C). The G2 checkpoint follows S phase, and correctly duplicated
chromosomes will allow cell cycle progression at the G2 checkpoint.
15 What do you call changes to the order of nucleotides in a segment of DNA that codes for
a protein?
A Proto-oncogenes
B Tumor suppressor genes
C Gene mutations
D Negative regulators
Solution The solution is (C). Gene mutations are responsible for altering the sequence of
nucleotides. The shape and function of protein produced by these genes are likely
altered.
16 Human papillomavirus can cause cervical cancer. The virus encodes E6, a protein that
binds p53. Based on this fact and what you know about p53, what effect do you think E6
binding has on p53 activity?
A E6 activates p53.
B E6 protects p53 from degradation.
C E6 mutates p53.
D E6 binding marks p53 for degradation.
Solution The solution is (D). Binding of E6 to p53 leads to the degradation of tumor
suppressor protein, which in turn encourages the development of cancerous growth.

Advanced Placement Biology Instructor’s Solution Manual


10 | Cell Reproduction 215

17 What is a gene that codes for a positive cell cycle regulator called?
A Kinase inhibitor
B Oncogenes
C Proto-oncogenes
D Tumor suppressor genes
Solution The solution is (C). Proto-oncogenes are proteins that help in cell cycle progression.
If they get mutated, the cell progression may get stopped.
18 Which molecule is a Cdk inhibitor or is controlled by p53?
A Anti-kinase
B Cyclin
C p21
D Rb
Solution The solution is (C). If DNA damage is found in replication, p53 becomes active and
triggers synthesis of Cdk inhibitors, also known as p21.
19 Which eukaryotic cell cycle events are missing in binary fission?
A Cell growth
B DNA duplication
C Karyokinesis
D Cytokinesis
Solution The solution is (C). Prokaryotic cells do not possess nuclei, and, hence, they do not
undergo karyokinesis.
20 Which statement about binary fission is false?
A In both prokaryotic and eukaryotic cells, the outcome of cell reproduction is a pair of
daughter cells, which are genetically identical to the parent cell.
B Karyokinesis is unnecessary in prokaryotes because there is no nucleus.
C Replication of the prokaryotic chromosome begins at the origin of replication and
continues in both directions at once.
D The mitotic spindle draws the duplicated chromosomes to the opposite ends of the
cell, followed by formation of a septum and two daughter cells.
Solution The solution is (D). There is no need for a mitotic spindle in prokaryotic cells because
they do not undergo karyokinesis.

Advanced Placement Biology Instructor’s Solution Manual


216 10 | Cell Reproduction

21 The formation of what structure, which will eventually form the new cell walls of the
daughter cells, is directed by FtsZ?
A Contractile ring
B Cell plate
C Cytoskeleton
D Septum
Solution The solution is (D). The FtsZ triggers the accumulation of proteins at the center of
the cell to promote cell wall synthesis for division. This wall is known as the septum.

CRITICAL THINKING QUESTIONS


22 How would you compare and contrast a human somatic cell to a human gamete?
A Somatic cells have 46 chromosomes and are diploid, whereas gametes have half as
many chromosomes as found in somatic cells.
B Somatic cells have 23 chromosomes and are diploid, whereas gametes have half as
many chromosomes as are present in somatic cells.
C Somatic cells have 46 chromosomes and are haploid, whereas gametes have 23
chromosomes and are diploid.
D Somatic cells have 46 chromosomes with one sex chromosome. In gametes, 23
chromosomes are present with two sex chromosomes.
Solution The solution is (A). Human somatic cells have 46 chromosomes: 22 pairs and
2 sex chromosomes. This is the 2n, or diploid, condition. Human gametes have
23 chromosomes, one each of 23 unique chromosomes, one of which is a sex
chromosome. This is the n, or haploid, condition. The somatic cells have
46 chromosomes with 22 autosomes and 2 sex chromosomes. They are
considered diploid. But in gametes, 23 chromosomes are found with only
one sex chromosome; they are haploid.
23 Eukaryotic chromosomes are thousands of times longer than a typical cell. How can
chromosomes fit inside a eukaryotic nucleus?
A The genetic material remains distributed in the nucleus, mitochondria, and
chloroplast.
B The genome is present in a looped structure; thus, it fits the size of the nucleus.
C The DNA remains coiled around proteins to form nucleosomes.
D The genetic material remains bound to the nuclear envelope, forming invaginations.
Solution The solution is (C). The DNA double helix is wrapped around histone proteins to form
structures called nucleosomes. Nucleosomes and the linker DNA in between them
are coiled into a 30-nm fiber. During cell division, chromatin is further condensed by
packing proteins.

Advanced Placement Biology Instructor’s Solution Manual


10 | Cell Reproduction 217

24 Briefly describe the events that occur in each phase of interphase.


A G1: assessment for DNA damage; S: duplication of genetic material; G2: duplication and
dismantling organelles
B G1: duplication of organelles; S: duplication of DNA; G2: assessment of DNA damage
C G1: synthesis of DNA; S: synthesis of organelle genetic material; G2: assessment of DNA
damage
D G1: preparation for DNA synthesis; S: assessment of DNA damage; M: division of cell
Solution The solution is (A). During G1, the cell increases in size, the genomic DNA is assessed
for damage, and the cell stockpiles energy reserves as well as the components to
synthesize DNA. During the S phase, the chromosomes, centrosomes, and centrioles
(animal cells) duplicate. During the G2 phase, the cell recovers from the S phase,
continues to grow, duplicates some organelles, and dismantles other organelles.
25 Chemotherapy drugs, such as vincristine and colchicine, disrupt mitosis by binding to
tubulin (the subunit of microtubules) and interfering with microtubule assembly and
disassembly. Exactly what mitotic structure do these drugs target, and what effect would
that have on cell division?
A The drugs bind tubulin and inhibit the binding of spindle to the chromosome. This can
arrest the cell cycle.
B The drugs bind the tubulin, which leads to an error in the chromosome separation.
This could lead to apoptosis.
C The drugs bind the tubulin, thereby inhibiting their division in S phase. This inhibits cell
division.
D The drugs bind the spindle fiber and hinder the separation of chromatins. This
promotes the division spontaneously.
Solution The solution is (A). The mitotic spindle is formed of microtubules. Microtubules are
polymers of the protein tubulin; therefore, it is the mitotic spindle that is disrupted
by these drugs. Without a functional mitotic spindle, the chromosomes will not be
sorted or separated during mitosis. The cell will arrest in mitosis and die.
26 Why might a cell that has just completed cytokinesis enter the G0 phase instead of the
G1 phase?
A Some cells are physiologically inhibited from undergoing any division and remain in
the G0 phase to provide assistance to their neighboring cells.
B Some cells reproduce only under certain conditions and, until then, they remain in the
G0 phase.
C Suspected DNA damage can lead the cell to undergo the G0 phase.
D The lack of important components of cell division makes cells stay in the G 0 phase.

Advanced Placement Biology Instructor’s Solution Manual


218 10 | Cell Reproduction

Solution The solution is (B). Many cells temporarily enter G0 until they reach maturity. Some
cells are only triggered to enter G1 when the organism needs to increase that
particular cell type. Some cells only reproduce following an injury to the tissue.
Some cells never divide once they reach maturity.
27 Which general conditions must be met at each of the three main cell cycle checkpoints?
A G1 checkpoint: assessment of DNA damage; G2: assessment of new DNA;
M checkpoint: segregation of sister chromatids in anaphase
B G1 checkpoint: energy reserves for S phase; G2 checkpoint: assessment of new DNA;
M checkpoint: attachment of spindle to kinetochore
C G1 checkpoint: assessment of DNA damage; G2 checkpoint: energy reserves for
duplication; M checkpoint: attachment of spindle to kinetochore
D G1 checkpoint: energy reserves for S phase; S checkpoint: synthesis of DNA,
G2 checkpoint: assessment of new DNA
Solution The solution is (B). The G1 checkpoint monitors adequate cell growth, the state of
the genomic DNA, adequate stores of energy, and materials for S phase. At the
G2 checkpoint, DNA is checked to ensure that all chromosomes were duplicated and
there are no mistakes in newly synthesized DNA.
28 What is the role of the positive cell cycle regulators compared to that of the negative
regulators?
A Positive regulators promote the cell cycle, but negative regulators block the cell cycle.
B Positive regulators block the cell division in cancerous cells, but negative regulators
promote in such cells.
C Positive regulators promote the cell cycle, but negative regulators arrest the cell cycle
until certain events have occurred.
D Positive regulators show positive feedback mechanisms, but negative regulators show
negative feedback in the cell cycle.
Solution The solution is (C). Positive cell regulators, such as cyclin and Cdk, perform tasks that
advance the cell cycle to the next stage. Negative regulators such as Rb, p53, and
p21 block the progression of the cell cycle until certain events have occurred.
29 What occurs at the M checkpoint? What would happen if the M checkpoint failed?
A The M checkpoint checks for proper attachment of sister chromatids, and, if it fails,
then cells may undergo nondisjunction of chromosomes.
B The M checkpoint checks if the DNA is damaged and promotes its repair. If it fails,
then the daughters end up with damaged DNA.

Advanced Placement Biology Instructor’s Solution Manual


10 | Cell Reproduction 219

C The M checkpoint ensures the proper duplication of DNA, and, if it fails, the cells may
undergo nondisjunction of chromosomes.
D The M checkpoint ensures that all the components required for cell division are
available, and, if it fails, the cell cycle will be inhibited.
Solution The solution is (A). The M checkpoint determines whether all the sister chromatids
are correctly attached to the spindle microtubules. Normally, the cycle will not
proceed until the kinetochores of each pair of sister chromatids are attached to at
least two spindle fibers arising from opposite poles of the cell. This ensures that the
sister chromatids separate during anaphase and that each daughter cell contains the
proper amount of DNA.
If the M checkpoint fails, then it is possible that sister chromatids might not separate
during mitosis. If the spindle microtubules only attach to one sister chromatid and
the cell cycle is not halted by the M checkpoint, then both sister chromatids will be
pulled to one pole during anaphase (nondisjunction). The result will be one daughter
cell with too many chromosomes and one daughter cell with too few chromosomes
(aneuploidy).
30 Which regulatory mechanisms might be lost in a cell producing faulty p53?
A Assessment of damaged DNA, recruiting repair enzymes, and binding of spindle to
kinetochore
B Quality of DNA, triggering apoptosis, and recruiting repair enzymes
C Quality of DNA, binding of spindle to kinetochore, and assessment of DNA repair
D Triggering apoptosis, recruiting repair enzymes, and proper binding of spindle to
kinetochore
Solution The solution is (B). Regulatory mechanisms that might be lost include monitoring of
the quality of the genomic DNA, recruiting of repair enzymes, and the triggering of
apoptosis. DNA damage and quality are detected by p53. It triggers apoptosis in case
of damage, and it recruits enzymes.
31 p53 can trigger apoptosis if certain cell cycle events fail. How does this regulatory
outcome benefit a multicellular organism?
A The apoptosis helps in controlling the consumption of energy by the extra cells.
B The apoptosis inhibits the production of faulty proteins, which could be produced due
to the DNA damage.
C The process of apoptosis stops the invasion of viruses in the other cells.
D The cells are killed due to the production of reactive oxygen species produced, which
could harm the organism.
Solution The solution is (B). If a cell has damaged DNA, the likelihood of producing faulty
proteins is higher. The daughter cells of such a damaged parent cell would also
produce faulty proteins, which might eventually become cancerous. If p53

Advanced Placement Biology Instructor’s Solution Manual


220 10 | Cell Reproduction

recognizes this damage and triggers the cell to self-destruct, the damaged DNA is
degraded and recycled. No further harm comes to the organism. A healthy cell is
triggered to divide instead. Apoptosis is a vital mechanism, as p53 checks for the
DNA damage in the cell, which could produce faulty proteins when expressed. This
could prove fatal for a multicellular organism.
32 Which processes do eukaryotic cell division and binary fission have in common?
A DNA duplication, division of cell organelles, division of the cytoplasmic contents
B DNA duplication, segregation of duplicated chromosomes, and division of the
cytoplasmic contents
C Formation of a septum, DNA duplication, division of the cytoplasmic contents
D Segregation of duplicated chromosomes, formation of a septum, division of cell
organelles
Solution The solution is (B). The common processes of eukaryotic cell division and binary
fission are DNA duplication, segregation of duplicated chromosomes, and division of
the cytoplasmic contents.
33 The formation of what structure, that will eventually form the new cell walls of the
daughter cells, is directed by FtsZ?
A Contractile ring
B Cell plate
C Cytoskeleton
D Septum
Solution The solution is (D). The FtsZ triggers the accumulation of proteins at the center of
the cell to promote cell wall synthesis for division. This wall is known as the septum.

TEST PREP FOR AP® COURSES


34 Which statement cannot be inferred from the karyotype shown?

Advanced Placement Biology Instructor’s Solution Manual


10 | Cell Reproduction 221

A The cell contains DNA.


B The cell contains 46 chromosomes.
C The cell is diploid.
D The cell is prokaryotic.
Solution The solution is (D). The cell is prokaryotic. Prokaryotes have a single circular
chromosome, not multiple chromosomes.
35 How can DNA, which in humans measures approximately 2 m, fit inside a human cell that
is about 10 μm? How does the organization of the genetic material in eukaryotes differ
from that of prokaryotes?
A The DNA is found wrapped around histones to form nucleosomes, which further
compact and ultimately form linear chromosomes. The prokaryotic genome is found
as a loop and in eukaryotes as a double-stranded linear structure.
B The DNA is wrapped around the nucleosomes to show a compact structure. The
eukaryotes show a loop structure, and prokaryotes show a double-stranded linear
genome.
C The genetic material shows ringed heterochromatin structure. The prokaryotes show
multiple loops, and eukaryotes show a condensed chromatin.
D The genetic material is wrapped around histones. The prokaryotes have a condensed
structure in nucleoids, but eukaryotes have a double-stranded linear structure.
Solution The solution is (D). DNA is compacted to fit in a cell. In the first level of compaction,
short stretches of the DNA double helix wrap around a core of histone proteins,
forming a nucleosome that is linked by linker DNA. The next level of compaction
occurs as the nucleosomes and linker DNA are coiled into a 30-nm chromatin fiber.
In the third level of packing, a variety of fibrous proteins are used to pack the
chromatin.

Advanced Placement Biology Instructor’s Solution Manual


222 10 | Cell Reproduction

In eukaryotes, the genome consists of several double-stranded linear DNA molecules


forming chromosomes. In prokaryotes, the genome is composed of a single double-
stranded DNA molecule in the form of a loop or circle.
36 Which statement about structure 1 on the karyotype is NOT true?

A Structure 1 consists of homologous chromosomes.


B The two parts of structure 1 will have genes in different loci.
C The two parts of structure 1 originate from different parents.
D The two parts of structure 1 will have slightly different sequences of nucleotides.
Solution The solution is (B). The two parts of structure 1 will have genes in different loci.
Homologous chromosome will have genes in the same loci. However, the loci may
have different variations of the genes.
37 Based on the karyotype provided, the nondisjunction of which chromosome causes Down
syndrome?

Advanced Placement Biology Instructor’s Solution Manual


10 | Cell Reproduction 223

A Chromosome 21
B Chromosome 22
C X chromosome
D Y chromosome
Solution The solution is (A). The figure depicts three copies of chromosome 21, while the
other chromosomes have two chromatids. This condition is found in Down
syndrome and occurs during anaphase I when one pair of homologous
chromosomes fails to separate.
38 What is the sequence of mitotic cell cycle for one pair of chromosomes that is undergoing
normal mitotic division?
A Anaphase - metaphase - prophase - cytokinesis
B Anaphase - prophase - metaphase - cytokinesis
C Prophase - anaphase - metaphase - cytokinesis
D Prophase - metaphase - anaphase - cytokinesis

Advanced Placement Biology Instructor’s Solution Manual


224 10 | Cell Reproduction

Solution The solution is (D). The figure shows the sequence prophase – metaphase –
anaphase – cytokinesis.

There is a cell labeled prophase, with two chromosomes. During prophase, spindle
fibers emerge from centrosomes and the nuclear envelope breaks down. A cell
labeled metaphase shows the two chromosomes are lined up at the center of the
cell. A cell labeled anaphase shows one chromosome and a sister chromatid pulled
to one pole of the cell and just a sister chromatid pulled to the other pole. Finally, in
cytokinesis, two cells are depicted; one has a full chromosome and a sister
chromatid, the other has only a single sister chromatid.
39 In a study on cell division, researchers culture synchronously dividing human cells with
thymidine, which causes the cells to arrest at the G1 boundary. The cells are then placed in
medium lacking thymidine, which releases the block, and the cells begin to divide again.
Starting with Sample A and ending with Sample D, the DNA content of the cells is
measured at different times after thymidine is removed. Results for four samples (A–D)
are shown in the graph.

Which sample presents the expected results for cells in S phase?


A Sample A
B Sample B
C Sample C
D Sample D
Solution The solution is (B). Sample B shows relative content of DNA that has been doubled.
This doubling of the DNA content happens only in S phase, which immediately
follows G1.

Advanced Placement Biology Instructor’s Solution Manual


10 | Cell Reproduction 225

40 In a study on cell division, researchers culture synchronously dividing human cells with
thymidine. This causes the cells to arrest at the G1 boundary. The cells are then placed in
medium lacking thymidine, which releases the block, and the cells begin to divide again.
Starting with Sample A and ending with Sample D, the DNA content of the cells is
measured at different times after thymidine is removed. Results for four samples (A-D)
are shown in the graph. Explain what is happening in terms of the cell cycle and DNA
content in sample B.

A All the contents of the cell have been doubled.


B The DNA content of the cell has doubled.
C Two cells have been fused.
D The cells are showing the semiconservative mechanism of cell division
Solution The solution is (B). Sample B occurred after the S phase of the cell cycle, which
causes the DNA content of the cell to double after all DNA is copied.

41 Li-Fraumeni syndrome (LFS1) is a rare hereditary disorder that leads to a predisposition to


cancer. This hereditary disorder is linked to mutations in the tumor-suppressor gene
encoding the transcription factor p53. p53 acts at the G1 checkpoint. If damaged DNA is
detected, p53 halts the cell cycle. As p53 levels rise, the production of p21 is triggered.
p21 enforces the halt in the cell cycle. A variant of Li-Fraumeni, called LFS2, is thought to
occur due to a mutation of the CHK2 gene, which is also a tumor-suppressor gene. CHK2
regulates the action of p53. Which of the following cascades is most likely to occur in a
normal cell that does not contain the LFS mutation?
A 1. Cell cycle progression
2. p53
3. p21
4. CHK2
B 1. p53
2. p21

Advanced Placement Biology Instructor’s Solution Manual


226 10 | Cell Reproduction

3. CHK2
4. Cell cycle progression
C 1. p21
2. p53
3. CHK2
4. Cell cycle progression
D 1. CHK2
2. p53
3. p21
4. Cell cycle progression
Solution The solution is (D). CHK2 is a kinase that checks for the DNA damage and
phosphorylates the p53 to produce p21 (inhibitor of Cdk’s). After the repair process,
the cell cycle progresses.

Advanced Placement Biology Instructor’s Solution Manual


10 | Cell Reproduction 227

42 The insulin growth factor (IGF-1) promotes cell proliferation as shown in the diagram.

The expression of which protein in the diagram is controlled through negative feedback?
A Active Cdk4
B Cyclin D1
C Cyclin D1/Cdk4 complex
D IGF-1
Solution The solution is (C). The Cdk4 bound to cyclin becomes active, and this is responsible
for inhibiting the cyclin D1-Cdk4 complex.
43 Why are p53, p21, and CHK2 considered tumor-suppressor genes, NOT proto-oncogenes?
Give an example of a proto-oncogene.
A p53, p21, and CHK2 suppress the proteins that regulate the cell cycle, whereas proto-
oncogenes, like phosphorylated Rb, help in cell cycle progression.
B p53, p21, and CHK2 are negative cell cycle regulators, whereas Cdk’s are proto-
oncogenes, which could cause cancer when mutated.
C p53, p21, and CHK2 suppress the proteins that regulate the cell cycle, whereas Rb is
considered a proto-oncogene because it is the most primitive.
D The three proteins help stop the formation of tumors, whereas Cdk’s are called proto-
oncogenes because they are the most primitive of all.
Solution The solution is (A). A proto-oncogene is a segment of DNA that codes for one of the
positive cell cycle regulators. If such a gene becomes mutated, producing a hyper-
activated protein product, it is considered an oncogene. A tumor-suppressor gene is

Advanced Placement Biology Instructor’s Solution Manual


228 10 | Cell Reproduction

a segment of DNA that codes for one of the negative cell cycle regulators. If such a
gene becomes mutated such that the protein product becomes less active, the cell
cycle will run unchecked. A single oncogene can initiate abnormal cell division;
however, tumor suppressors lose their effectiveness only when both copies of the
gene are damaged. The Cdk gene is considered a proto-oncogene. The three
proteins are tumor suppressor proteins; they regulate the cell cycle and any
mutation in these will cause tumor. The Cdk gene, if it undergoes mutation, would
lead to cancer and is called a proto-oncogene as a result.

SCIENCE PRACTICE CHALLENGE QUESTIONS


10.2 The Cell Cycle
44 Many biological processes are synchronized with the 24-h rotational period of Earth.
Circadian (24-h) periodicity is common across phyla. One of these processes is the cell
cycle. The currently accepted explanation is that the low-oxygen atmosphere of early
Earth had no ozone layer to filter out the solar ultraviolet radiation that damages DNA.
Completing the S phase of the cell cycle at night provided a selective advantage. The
internal clock controlling the cell cycle and the circadian clock became synchronized.
Research has demonstrated that changes in one clock, either the circadian clock or
the cell cycle clock, disrupt timing in the other. The question was, which clock controls
the other?
Researchers have found that the circadian clock, which can be observed by fluorescent
markers on proteins that carry the circadian signal, can be disrupted by changes in light,
nutrition, or exposure to the steroid dexamethasone. Nutrition can also disrupt the cell
cycle clock. Rat fibroblasts (cells constantly undergoing mitosis) were cultured on medium
containing different levels of fetal bovine serum (FBS) with and without the addition of
dexamethasone. Confluence is a phenomenon that occurs in tissue culture when the
surface of the growth medium becomes covered with cells and the cells stop dividing. The
circadian and cell cycle periods were measured.
Dexa- Circadian Cell Cycle
Run FBS methasone Confluence Period (h) Period (h)
A 0% No No 24 ± 0.5 24 ± 0.5
B 10% No No 21.9 ± 1.1 21.3 ± 1.3
C 15% No No 19.4 ± 0.5 18.6 ± 0.6
D 10% Yes No 24.2 ± 0.5 20.1 ± 0.94
E 20%* Yes No 21.25 ± 0.36 19.5 ± 0.42
F 20% Yes No 29 ± 1.05 16.05 ± 0.48
G 10% Yes Yes 24 ± 0.5 n/a
*Subsets of samples with 20% FBS and dexamethasone were clustered around two means
for each measured period.

Advanced Placement Biology Instructor’s Solution Manual


10 | Cell Reproduction 229

A. Based on these data, describe the connections between the circadian period and the
cell cycle period for each of the experimental conditions.
B. Based on these data, justify the claim that, in cells that are actively dividing, the
circadian period is set by the cell cycle period rather than the reverse.
Solution Sample answer:
A. The periods of both clocks when dexamethasone is absent are initially identical.
Growth on FBS is known to accelerate the cell cycle. The circadian period is equal to
the cell cycle period, implying causation. Higher concentrations of the nutrient are
consistent with this. When the circadian rhythm is disrupted by dexamethasone, the
cell cycle period is unaffected, though the circadian period is lengthened (runs B and
D). This is consistent with only one (E) of the measurements made at higher FBS
levels. Finally, when the cell cycle stops, the circadian period returns to its original
value.
B. Conclusion: The circadian clock is always expressed, but during periods of rapid
cell division, the circadian period becomes synchronized with the cell cycle period;
although at much shorter cell cycle periods, the circadian period becomes erratic.
Teaching note—This is a very surprising result in this field. Studies suggest a
correlation between disruption of the circadian clock, for example a shift to late-
night working hours, and cancer. This would involve entrainment of the cell cycle by
the circadian cycle, and the opposite is shown by this work to be the case.
45 Cells in different tissues of a fully developed human show significant variations in the
length of time that they remain in the G0 phase of the cell cycle: muscle (lifetime), nerve
(lifetime), adipose (years), liver (years), erythrocyte (months), bone osteoclasts (weeks),
leukocyte (days), and epidermal (hours). For each of these types of tissues, propose a
reason based on internal and external factors and function that might account for the
differences among their longevities.
Solution Sample answer:
 Muscle: constrained by available space in a developed tissue
 Nerve: constrained by function because it is the intercellular connections
that imbue function
 Adipose, liver, erythrocyte: Replacement rate is proportional to stresses.
 Leukocyte: constrained by the demands of phagocytosis
 Epidermal: hours; includes skin, stomach, and lung—all tissues in contact
with a hostile chemical environment
46 Describe the essential components and results of mitosis and the activities that occur
during interphase to prepare the cell for mitosis.
Solution Sample answer: During G1, the cell increases in size, the genomic DNA is assessed for
damage, and the cell stockpiles energy reserves as well as the components to

Advanced Placement Biology Instructor’s Solution Manual


230 10 | Cell Reproduction

synthesize DNA. During the S phase, the chromosomes, the centrosomes, and the
centrioles (animal cells) duplicate. During the G2 phase, the cell recovers from the S
phase, continues to grow, duplicates some organelles, and dismantles other
organelles.
10.4 Cancer and the Cell Cycle
47 Cancer comprises many diseases with a common cause: uncontrolled cell growth. Cancer
is a complex response to a host of environmental mutagens as well as the accumulation
of random mutations. Since the “war on cancer” began in 1971, the death rate due to
cancer has changed very little despite the discovery of several tumor-suppressor genes,
including p53.
A. Briefly describe the multiple functions of p53, including the role of p53 in apoptosis.
B. A principle of biology is that “form follows function.” The protein p53, which has
multiple functions, is named for its molecular mass—approximately 53 kDa. This is not a
large polymer by comparison with other proteins; for example, ATP synthase, which has
only one function, has a molecular mass of approximately 550 kDa. Based on analogies to
processes involved in cellular signaling, create a model(s) to explain how so many
functions can be supported by a single, relatively simple structure.
C. Mutational signatures of p53 are shown in the figure (G.P. Pfeifer et al., Nature, 21(48),
2002) for the three types of cancer with the highest death rates in the United States: lung
(~225,000 deaths in 2016), breast (246,000), and colorectal (381,000).

These data can be obtained by sequencing the gene that encodes p53. Approximately
85 percent of lung cancers occur in smokers. Based on these data, calculate how many
deaths due to lung cancer among nonsmokers were reported in 2016. How much does
smoking increase the likelihood of death due to lung cancer?
D. As shown under each graph in the figure, particular transversions (replacement of a
pyrimidine by a purine of vice versa) or transitions (replacement of a purine or pyrimidine
by the alternative purine or pyrimidine) are features of specific mutational signatures.
Based on these data, identify the transversion or transition that seems to be induced by
cigarette smoke.

Advanced Placement Biology Instructor’s Solution Manual


10 | Cell Reproduction 231

E. Using your answer to (B), predict possible mechanisms—that is, transversion or


transition—for the different mutational signatures among lung cancers of smokers and
those of other cancers, and for the very similar mutational signatures of lung cancers of
nonsmokers and of breast and colorectal cancers. The partitioning of function along the
length of the protein can lead to functional and nonfunctional segments. It is believed that
the transversions due to smoking are caused by polyaromatic hydrocarbons. The hotspots
for these mutations lie in the segment that binds to DNA. The transition hotspots are in
segments that regulate apoptosis.
Solution Sample answer:
A. As described in the text, p53 detects DNA damage, stops the cell cycle at G 1/S,
activates DNA repair, and initiates apoptosis.
B. p53 achieves multifunctionality with several segments of amino acid sequence
responsible for different functions. Segments activate transcription factors that lead
to the expression of genes, much as a signal receptor can produce messengers that
lead to expression. These segments activate expression of enzymes that achieve the
other functions. In addition, a large segment binds to DNA to manage checks of
integrity and initiate repair.
C. If 85 percent of the 225,000 lung cancer deaths occur among smokers, then
deaths can be attributed to death due to lung cancer triggered by
smoking. Nonsmokers may also have lung cancer, and the ratio is 0.85/0.15, which is
a factor of between 5 and 6. Second-hand smoke is sometimes suggested as the
cause of the disease in nonsmokers, although other mutagens such as asbestos
fibers are thought to be causes.
D. The population of smokers with lung disease shows a significant increase in the
transversion. Guanine is a pyrimidine, and thymine is a purine. This
replacement creates kinks that are much less likely to be silent mutations than are
the transitions. Cancer is often modeled as the result of an accumulation of
mutations, and the dominant mutations among the nonsmoker lung, breast, and
colon-rectal cancers are transitions.
E. The partitioning of function along the length of the protein can lead to functional
and nonfunctional segments. It is believed that the transversions due to smoking are
caused by polyaromatic hydrocarbons. The hotspots for these mutations lie in the
segment that binds to DNA. The transition hotspots are in segments that regulate
apoptosis.

Advanced Placement Biology Instructor’s Solution Manual


232 11 | Meiosis and Sexual Reproduction

11 | MEIOSIS AND SEXUAL REPRODUCTION


REVIEW QUESTIONS
1 How many and what type of daughter cells does meiosis produce?
A Four haploid
B Four diploid
C Two haploid
D Two diploid
Solution The solution is (A). In meiosis, there are two rounds of nuclear division—meiosis I
and meiosis II. This results in four nuclei and four daughter cells, each with half the
number of chromosomes from the parents.
2 What structure is most important in forming the tetrads?
A Centromere
B Chiasmata
C Kinetochore
D Synaptonemal complex
Solution The solution is (D). The synaptonemal complex is a protein lattice that forms
between homologous chromosomes and is a key part in forming the tetrad and
maintaining the synapsis between the strands. It disassembles at the end of
prophase I.
3 At which stage of meiosis are sister chromatids separated from each other?
A Anaphase I
B Anaphase II
C Prophase I
D Prophase II
Solution The solution is (B). The sister chromatids are pulled apart by the kinetochore
microtubules and move toward opposite poles during anaphase II.
4 At metaphase I, homologous chromosomes are connected only at what structures?
A Chiasmata
B Kinetochores
C Microtubules
D Recombination nodules

Advanced Placement Biology Instructor’s Solution Manual


11 | Meiosis and Sexual Reproduction 233

Solution The solution is (A). Chiasmata—sites where crossing over has occurred—are formed
during prophase I and link the homologous chromosomes until they are separated in
anaphase 1.
5 What phase of mitotic interphase is missing from meiotic interkinesis?
A G0 phase
B G1 phase
C G2 phase
D S phase
Solution The solution is (D). The S phase, or synthesis phase, is where the DNA of the
chromosomes is replicated. It is not present in interkinesis, which is a brief interlude
between meiosis I and II in some species.
6 What part of meiosis is most similar to mitosis?
A Reduction division
B Interkinesis
C Meiosis I
D Meiosis II
Solution The solution is (D). Mitosis and meiosis II are similar processes since the number of
chromosomes is conserved in both and no genetic variation is introduced.
7 What is NOT true during crossing over?
A Chiasmata are formed.
B Nonsister chromatids exchange genetic material.
C Recombination nodules mediate cross-over events.
D Spindle microtubules guide the movement of chromosomal material.
Solution The solution is (D). Spindle microtubules are not directly involved with crossing over.
8 During which phase does the second round of genetic variation occur during meiosis?
A Anaphase I
B Metaphase I
C Prophase II
D Genetic variation only occurs during prophase I.
Solution The solution is (B). The random assortment of homologous chromosomes at the
metaphase plate is the second mechanism that introduces variation into the
gametes or spores.

Advanced Placement Biology Instructor’s Solution Manual


234 11 | Meiosis and Sexual Reproduction

9 Which type of life cycle has both a haploid and a diploid multicellular stage?
A Alternation of generations
B Asexual
C Diploid-dominant
D Haploid-dominant
Solution The solution is (A). Species with a life cycle that alternates alternate between
haploid and diploid multicellular stages.
10 What is a source of variation in asexual reproduction?
A Crossing over of chromosomes
B Mutation of DNA
C Random assortment of chromosomes
D There is no variation in asexual reproduction.
Solution The solution is (B). A mutation is a change that occurs in the nucleotides of the DNA.
It is a source of genetic variation, often the only source, in organisms that reproduce
asexually.
11 What is a likely evolutionary advantage of sexual reproduction over asexual reproduction?
A Sexual reproduction involves fewer steps.
B Sexual reproduction results in variation in the offspring.
C Sexual reproduction is more metabolically efficient.
D Sexual reproduction uses up fewer resources in a given environment.
Solution The solution is (B). Sexual reproduction increases genetic variability by shuffling
combinations of genes and chromosomes. This can provide an evolutionary
advantage.
12 What is a disadvantage of sexual reproduction over asexual forms of reproduction?
A Half the population is capable of carrying offspring.
B Identical offspring are not produced.
C Adaptation to rapidly changing environments is more difficult.
D Mutation rates are slower.
Solution The solution is (A). This is one disadvantage of sexual reproduction—that only half of
the population (the females) can carry offspring. Reproduction tends to be slower.

Advanced Placement Biology Instructor’s Solution Manual


11 | Meiosis and Sexual Reproduction 235

13 Fungi typically display which type of life cycle?


A Alternation of generations
B Asexual
C Diploid-dominant
D Haploid-dominant
Solution The solution is (D). Fungi have haploid-dominant life cycles. The haploid multicellular
stage produces specialized haploid cells that fuse to form a diploid zygote, which
immediately undergoes meiosis to produce haploid cells.
14 What is a haploid cell produced in a diploid-dominant organism by meiosis called?
A Gamete
B Gametophyte
C Spore
D Sporophyte
Solution The solution is (A). Gametes are unicellular haploid cells produced in diploid-
dominant organisms. They fuse with other gametes during fertilization to produce
diploid zygotes.

CRITICAL THINKING QUESTIONS


15 What happens to tetrads after they form?
A Prophase I of meiosis forms the tetrads. They line up at the midway point between the
two poles of the cell to form the metaphase plate. There is an equal chance of a
microtubule fiber encountering a maternally or a paternally inherited chromosome.
Orientation of each tetrad is independent of the orientation of other tetrads.
B Prophase II of meiosis forms the tetrads. They line up at the midway point between
the two poles of the cell to form the metaphase plate. There is an equal chance of
microtubule fiber encountering maternally or paternally inherited chromosome.
Orientation of each tetrad is independent of the orientation of other tetrads.
C Prophase I of mitosis forms the tetrads. They line up at the midway between the two
poles of the cell to form the metaphase plate. There is an equal chance of a
microtubule fiber encountering a maternally or a paternally inherited chromosome.
Orientation of each tetrad is independent of the orientation of other tetrads.
D Prophase I of meiosis forms the tetrads. They line up at the midway between the two
poles of the cell to form the metaphase plate. There is a chance of microtubule fiber
encountering a maternally inherited chromosome. Orientation of each tetrad is
independent of the orientation of other tetrads.
Solution The solution is (A). The tetrads line up at the midway point between the two poles of
the cell to form the metaphase plate. There is an equal chance of a microtubule fiber

Advanced Placement Biology Instructor’s Solution Manual


236 11 | Meiosis and Sexual Reproduction

encountering a maternally or a paternally inherited chromosome. Orientation of


each tetrad is independent of the orientation of other tetrads. In prophase I of
meiosis, the homologous chromosomes form the tetrads. In metaphase I, these pairs
line up at the midway point between the two poles of the cell to form the
metaphase plate. Because there is an equal chance that a microtubule fiber will
encounter a maternally or paternally inherited chromosome, the arrangement of the
tetrads at the metaphase plate is random. Any maternally inherited chromosome
may face either pole. Any paternally inherited chromosome may also face either
pole. The orientation of each tetrad is independent of the orientation of the other
tetrads. When the microtubules pull the tetrads apart, the sister chromatids remain
attached to each other.
16 What distinguishes metaphase I from metaphase II?
A Metaphase I occurs when chromosomes appear in homologous pairs on the spindle.
Metaphase II has a single line of chromosomes on the spindle. A pair of chromosomes
is pulled apart and migrates toward a pole in anaphase I, while in anaphase II, sister
chromatids separate. Telophase I reconstitutes the nucleus and loosens the
chromosomes, while telophase II mimics telophase I.
B Prophase I condenses the chromosomes and eliminates the nuclear membrane. The
microtubules arrange in a spindle. Prophase II mimics prophase I. Metaphase I occurs
when chromosomes appear in homologous pairs on the spindle. Metaphase II has a
single line of chromosomes on the spindle. Pairs of chromosomes are pulled apart and
migrate toward the poles during anaphase I, while in anaphase II, sister chromatids
separate. Telophase I reconstitutes the nucleus and condenses the chromosomes,
while telophase II mimics telophase I.
C Prophase I condenses the chromosomes and adds a nuclear membrane. The
microtubules arrange in a spindle. Prophase II mimics prophase I. Metaphase I occurs
when chromosomes appear in homologous pairs on the spindle. Metaphase II has a
single line of chromosomes on the spindle. A pair of chromosomes is pulled apart and
migrates toward the poles in anaphase I, while in anaphase II, sister chromatids
separate. Telophase I reconstitutes the nucleus and loosens the chromosomes, while
telophase II mimics telophase I.
D Prophase I condenses the chromosomes and eliminates the nuclear membrane. The
microtubules arrange in a spindle. Prophase II mimics prophase I. Metaphase I occurs
when chromosomes appear in homologous pairs on the spindle. During metaphase II,
the chromosomes line up in a double line across the spindle. Each pair of
chromosomes is pulled apart and migrates toward the poles in anaphase I, while in
anaphase II, sister chromatids separate. Telophase I reconstitutes the nucleus and
loosens the chromosomes, while telophase II mimics telophase I.
Solution The solution is (A). The difference in metaphase between meiosis I and II reflects
how meiosis I involves the separation of homologous chromosomes and meiosis II

Advanced Placement Biology Instructor’s Solution Manual


11 | Meiosis and Sexual Reproduction 237

represents the splitting of sister chromatids. This causes the differences in alignment
between metaphase I and II and the subsequent differences in later phases.
17 Though the stages of meiosis have the same names as the stages of mitosis, they exhibit
fundamental differences. What are the main differences between the two processes?
A Meiosis differs from mitosis in that the number of chromosomes is halved and genetic
variation is introduced in meiosis, but not in mitosis.
B Meiosis differs from mitosis in that the number of chromosomes is halved and genetic
variation is reduced in meiosis, but not in mitosis.
C Metaphase and telophase portions of meiosis and mitosis are the same. Meiosis and
mitosis are also the same, except for the number of chromosomes. Anaphase I and
anaphase II are different.
D Prophase and telophase portions of meiosis and mitosis are the same. Meiosis II and
mitosis are also the same and have the same number of chromosomes. Anaphase I
and anaphase II are different.
Solution The solution is (A). Meiosis cycles through two rounds of cell division, while mitosis
happens once. Meiosis I differs from mitosis in that the number of chromosomes is
halved, and crossing over (in prophase I) and independent assortment (in
metaphase I) adds genetic variation. Meiosis II is essentially the same as mitosis; the
number of chromosomes does not change, and no more genetic variation is
introduced. However, the end result is four haploid cells instead of the two diploid
cells that are produced in mitosis. Also, an S phase does not precede meiosis II as it
does in mitosis.
In meiosis, the number of chromosomes is halved in order to make haploid gametes,
rather than the diploid body cells produced during mitosis. Genetic variation is
introduced during meiosis through crossing over. In mitosis, the resulting cells are
genetic clones.
18 How does the orientation of homologous chromosomes during metaphase I of meiosis
contribute to greater variation in gametes?
A The random alignment of homologous chromosomes at the metaphase plate ensures
the random destination of the chromosomes in the daughter cells.
B Because homologous chromosomes dissociate from the spindle fibers during
metaphase I, they move randomly to the daughter cells.
C The homologous chromosomes are paired tightly during metaphase I and undergo
crossover as the synaptonemal complex forms a lattice around them.
D Recombination of maternal and paternal chromosomes occurs in metaphase I,
because the homologous chromosomes are not connected at their centromeres.

Advanced Placement Biology Instructor’s Solution Manual


238 11 | Meiosis and Sexual Reproduction

Solution The solution is (A). The pairs of homologous chromosomes that line up in
metaphase I are randomly oriented: maternal or paternal chromosomes may be on
either side of the cell’s equator and encounter a microtubule that will pull them
randomly to either daughter cell. So, the gametes made from meiosis contain a
mixture of chromosomes originally derived from both parents, but randomly
assorted.
19 How does the Red Queen’s catchphrase, “It takes all the running you can do to stay in the
same place,” describe coevolution between competing species?
A When a sexually reproducing species and an asexually reproducing species compete
for the same resources, they both “run [evolve] in the same place” because the
increased genetic variation in the sexually reproducing species balances the loss in
energy it uses to find and attract mates.
B When one species gains an advantage with a favorable variation, selection increases
on another species with which it competes. This species must also develop an
advantage or it will be outcompeted. The two species “run [evolve] to stay in the same
place.”
C When one species develops a mutation that decreases its ability to survive, a
competing species will become better able to survive even though it has not changed
in any way. In effect, this species “runs [evolves] to stay in the same place.”
D When two asexually reproducing species encounter rapid environmental change, the
species that is also able to reproduce sexually will outcompete the other. This way it
can “run [evolve] to stay in the same place.”
Solution The solution is (B). When one species gains an advantage with a favorable variation,
selection increases on another species with which it competes. This species must
also develop an advantage or it will be outcompeted. The two species “run [evolve]
to stay in the same place.” Each tiny advantage gained by favorable variation gives a
species an edge over close competitors, predators, parasites, or even prey. The only
method that will allow a coevolving species to maintain its own share of the
resources is to also continually improve its fitness. As one species gains an
advantage, this increases selection on the other species; it must also develop an
advantage or it will be outcompeted. The net effect is like running to stay in the
same place.
The evolution of two competing species is tightly connected. If one species has a
mutation that enables it to take advantage of its competitor in any way, the other
species will survive only if it also evolves a way to take the same or another
advantage. Neither one “wins”; each must “run in place” just to survive. The
variations that result from meiosis enable coevolution to happen, often quickly.
20 Which three processes lead to variation among offspring that have the same two parents?
A Genetic recombination, fertilization, meiosis
B Crossing over, random chromosome assortment, genetic recombination

Advanced Placement Biology Instructor’s Solution Manual


11 | Meiosis and Sexual Reproduction 239

C Meiosis, crossing over, genetic recombination


D Fertilization, crossing over, random chromosome assortment
Solution The solution is (D). Random (or independent) assortment mixes the chromosomes
derived from each parent. Crossing over creates a genetic combination that did not
exist prior to the exchange. Fertilization occurs randomly between an egg and a
large number of available sperm. All three produce genetic variation.
21 Compare the three main types of life cycles in multicellular organisms and give an
example of an organism that employs each.
A In a diploid dominant cycle, the multicellular diploid stage is present, as in humans.
Haploid-dominant life cycles have a multicellular haploid stage, as in fungi. In
alternation of generations, haploid-dominant and diploid-dominant stages alternate,
as in plants.
B In a diploid-dominant cycle, the unicellular diploid stage is present, as in humans. In a
haploid-dominant life cycle, a unicellular haploid stage is present, as in fungi. In
alternation of generations, haploid-dominant and diploid-dominant stages alternate,
as in plants.
C In a diploid-dominant cycle, a multicellular haploid stage is present, as in humans. In a
haploid-dominant life cycle, a multicellular diploid stage is present, as in fungi. In
alternation of generations, haploid-dominant and diploid-dominant stages alternate,
as in plants.
D In a diploid-dominant cycle, a multicellular diploid stage is present, as in algae. In a
haploid-dominant life cycle, a multicellular haploid stage is present, as in plants. In
alternation of generations, haploid-dominant and diploid-dominant stages alternate,
as in fungi.
Solution The solution is (A). In a diploid dominant cycle, the multicellular diploid stage is
present, as in humans. Haploid-dominant life cycles have a multicellular haploid
stage, as in fungi. In alternation of generations, haploid-dominant and diploid-
dominant stages alternate, as in plants. There are three main categories of life cycles
in multicellular organisms: (1) diploid-dominant, in which the multicellular diploid
stage is the most obvious life stage; humans are an example; (2) haploid-dominant,
in which the multicellular haploid stage is the most obvious life stage; examples
include fungi and some algae; and (3) alternation of generations, in which the two
stages are apparent to different degrees depending on the group, as with plants and
some algae.
In diploid-dominant life cycles, the multicellular diploid stage is present such as with
most animals, including humans; haploid-dominant, in which the multicellular
haploid stage is the most obvious life stage, such as with all fungi and some algae;
and alternation of generations, in which the two stages are apparent to different
degrees depending on the group, as with plants and some algae.

Advanced Placement Biology Instructor’s Solution Manual


240 11 | Meiosis and Sexual Reproduction

TEST PREP FOR AP® COURSES


22 Reproductive cells in most species are different from the cells that make up the rest of the
organism. What are the body cells called, and how are they different from the
reproductive cells?
A Body cells are called gametes and have half the number of chromosomes found in
reproductive cells.
B Body cells are called somatic cells and have the same number of chromosomes as
reproductive cells.
C Body cells are called somatic cells and have double the number of chromosomes
found in reproductive cells.
D Body cells are called gametes and have double the number of chromosomes found in
reproductive cells.
Solution The solution is (C). Body cells are called somatic cells and have double the number of
chromosomes found in reproductive cells. When the haploid reproductive cells
merge during fertilization, the diploid number of chromosomes is restored, and the
cells produced by subsequent mitotic divisions are somatic.
23 Spores are structures produced by some plants and all fungi. What is true about them?
A Spores are haploid reproductive cells that can produce haploid organisms through
mitosis.
B Spores are haploid precursors to gametes that give rise to gametes when
environmental conditions are favorable.
C Spores are haploid reproductive cells that can produce diploid cells without
fertilization.
D Spores are haploid cells formed only during asexual reproduction and so are not
formed by meiosis.
Solution The solution is (A). Spores are haploid reproductive cells that can form multicellular
haploid organisms after rounds of mitosis. They can also fuse with other spores to
produce diploid zygotes.
24 In prophase I, the homologous chromosomes are paired up and linked together. What
binds the chromosomes together and maintains their alignment?
A Cohesin proteins
B Tetrads
C The centromere
D Synaptonemal complex

Advanced Placement Biology Instructor’s Solution Manual


11 | Meiosis and Sexual Reproduction 241

Solution The solution is (D). The synaptonemal complex is a protein lattice that forms
between homologous chromosomes during prophase I when the chromosomes
undergo synapsis.
25 One of the ways that sexual reproduction enhances the diversity of offspring from the
same parents is through a process called crossing over. What entities does this occur
between during prophase I?
A Sister chromatids
B Tetrads
C Nonhomologous chromosomes
D Nonsister chromatids of homologous chromosomes
Solution The solution is (D). Exchange of genetic material between nonsister chromatids of
homologous chromosomes is called crossing over.
26 There are three sources of genetic variation in sexual reproduction. Which one is NOT
considered random?
A All are random.
B Crossing over
C Egg and sperm fertilization
D Tetrad alignment on the meiotic spindle
Solution The solution is (A). Fertilization, crossing over during prophase 1, and tetrad
alignment during metaphase I are all random events.
27 Which one of the three types of life cycles of sexually reproducing organisms does NOT
have a multicellular haploid stage?
A Alternation of generations
B Diploid-dominant
C Haploid-dominant
D They all have a multicellular haploid stage in their life cycles.
Solution The solution is (B). In diploid-dominant organisms, haploid cells are produced, and
fertilization occurs when the male and female gametes fuse. There is no
multicellular haploid life stage.
28 How are spores produced in haploid-dominant and alternation of generation life cycles?
A By gametophytes
B By germ cells
C Through mitosis
D Through meiosis

Advanced Placement Biology Instructor’s Solution Manual


242 11 | Meiosis and Sexual Reproduction

Solution The solution is (D). During sexual reproduction, specialized haploid cells form a
diploid zygote. The zygote undergoes meiosis to form four haploid cells called
spores.
29 What is one thing that is true of haploid-dominant life cycles but NOT of alternation of
generation life cycles?
A Meiosis
B (+) and (−) mating types
C Spores
D A free-living haploid stage
Solution The solution is (B). Haploid-dominant organisms, such as fungi and some algae,
spend the dominant part of their life cycles as multicellular haploids. During sexual
reproduction, specialized haploid cells from two individuals—designated the (+) and
(−) mating types—join to form a diploid zygote. The zygote immediately undergoes
meiosis to form four haploid cells, called spores. The spores undergo mitosis to form
the multicellular haploid individual. Organisms that alternate generations, like
plants, have both haploid and diploid multicellular life stages. In some organisms,
like ferns, both stages are free living. In others, only one stage is free living. Meiosis
occurs in both life cycles, so both exhibit genetic variation through crossing over,
chromosome assortment, and fertilization.

Advanced Placement Biology Instructor’s Solution Manual


12 | Mendel's Experiments and Heredity 243

12 | MENDEL'S EXPERIMENTS AND


HEREDITY
REVIEW QUESTIONS
1 Mendel performed hybridizations by transferring pollen to the female ova from what part
of the male plant?
A Anther
B Pistil
C Stigma
D Seed
Solution The solution is (A). Mendel performed hybridization by transferring the pollen from
the anther of the male plant.
2 What is one of the seven characteristics that Mendel observed in pea plants?
A Flower size
B Leaf shape
C Seed texture
D Stem color
Solution The solution is (C). One of the seven contrasting characteristics used by Mendel in
pea plants was seed texture.
3 Imagine you are performing a cross involving garden pea plants. What F1 offspring would
you expect if you cross true-breeding parents with green seeds and yellow seeds? Yellow
seed color is dominant over green.
A 100 percent yellow-green seeds
B 100 percent yellow seeds
C 50 percent yellow, 50 percent green seeds
D 25 percent green, 75 percent yellow seeds
Solution The solution is (B). A cross between true breeding plants with green and yellows
seeds would result in 100 percent yellow seeds because yellow is dominant over
green color.

Advanced Placement Biology Instructor’s Solution Manual


244 12 | Mendel's Experiments and Heredity

4 Consider a cross to investigate the pea pod texture trait, involving constricted or inflated
pods. Mendel found that the traits behave according to a dominant/recessive pattern in
which inflated pods were dominant.
If you performed this cross and obtained 650 inflated-pod plants in the F2 generation
bred from true-breeding stock, approximately how many constricted-pod plants would
you expect to have?
A 600
B 165
C 217
D 468
Solution The solution is (C). The phenotypic ratio of a monohybrid cross is 3 : 1. The predicted
number of constricted pods in the F2 generation will be 217.

5 The observable traits expressed by an organism are described as its —


A alleles
B genotype
C phenotype
D zygote
Solution The solution is (C). The observable traits of an organism are referred as its
phenotype.
6 A recessive trait will be observed in individuals that are what for that trait?
A Diploid
B Heterozygous
C Homozygous or heterozygous
D Homozygous
Solution The solution is (D). Homozygous individuals will express the recessive trait.
7 If black and white true-breeding mice are mated and the result is all gray offspring, what
inheritance pattern would this be indicative of?
A Codominance
B Dominance
C Incomplete dominance
D Multiple alleles

Advanced Placement Biology Instructor’s Solution Manual


12 | Mendel's Experiments and Heredity 245

Solution The solution is (C). The phenotype produced is a combination of the dominant and
recessive phenotypes. The offspring produced has an intermediate phenotype
between black and white and hence show incomplete dominance.
8 The ABO blood groups in humans are controlled by the IA, IB, and I alleles. The IA allele
encodes the A blood group antigen, IB encodes B, and I encodes O. Both A and B are
dominant to O. If a heterozygous blood type A parent (iAi) and a heterozygous blood type
B parent (iBi) mate, one-quarter of their offspring will have AB blood type (IAIB) in which
both antigens are expressed equally.
Therefore, the ABO blood groups are an example of —
A codominance and incomplete dominance
B incomplete dominance only
C multiple alleles and incomplete dominance
D multiple alleles and codominance
Solution The solution is (D). ABO blood groups in humans are expressed from the IA, IB, and I
alleles and therefore are examples of multiple alleles. Since both the alleles IA and IB
express simultaneously resulting in AB blood group, the ABO blood group is also an
example of codominance.
9 In a mating between two individuals that are heterozygous for a recessive lethal allele
that is expressed in utero, what genotypic ratio—homozygous dominant : heterozygous :
homozygous recessive—would you expect to observe in the offspring?
A 1:2:1
B 3:1:1
C 1:2:0
D 0:2:1
Solution The solution is (C). The genotypic ratio 1 : 2 : 0 will be obtained after mating
between two heterozygous individuals for a recessive lethal allele since the offspring
with the homozygous recessive allele will not survive in utero.
10 The forked line and probability methods make use of what probability rule?
A Monohybrid rule
B Product rule
C Sum rule
D Test cross
Solution The solution is (B). The values along each forked pathway can be multiplied because
each gene assorts independently.

Advanced Placement Biology Instructor’s Solution Manual


246 12 | Mendel's Experiments and Heredity

11 In pea plants, smooth seeds (S) are dominant to wrinkled seeds (s). The Punnett square
shows a genetic cross of two plants that are heterozygous for the seed shape trait.

What is the missing genotype?


A SS
B Ss
C sS
D ss
Solution The solution is (D). The ss genotype is obtained when the s allele of one parent
combines with s allele of another parent.
12 If the inheritance of two traits fully obeys Mendelian laws of inheritance, where may you
assume that the genes are located?
A On any autosomal chromosome or chromosomes
B On Y chromosomes
C On the same chromosome
D On separate chromosomes
Solution The solution is (D). If the inheritance of two traits fully follows the Mendelian laws of
inheritance, the genes assort independently, so they are located on separate
chromosomes.
13 How many different offspring genotypes are expected in a trihybrid cross between
parents heterozygous for all three traits? How many phenotypes are expected if the traits
behave in a dominant and recessive pattern?
A 64 genotypes; 16 phenotypes
B 16 genotypes; 64 phenotypes
C 8 genotypes; 27 phenotypes
D 27 genotypes; 8 phenotypes
Solution The solution is (D). There are 27 genotypes and 8 phenotypes expected in a trihybrid
cross. Review Table 12.5: General Rules for Multihybrid Crosses.

Advanced Placement Biology Instructor’s Solution Manual


12 | Mendel's Experiments and Heredity 247

14 Four-o’clock flowers may be red, pink, or white. In the crossing of true-breeding red and
true-breeding white plants, all the offspring are pink. What is the correct genotype of the
offspring if the red parent has genotype RR and the white parent has genotype rr? Use a
Punnett square.
A RR and Rr
B Rr and rr
C Rr only
D RR only
Solution The solution is (C). Rr is the only possible genotype of the offspring obtained from a
cross of true-breeding red and true-breeding white plants.
15 Which cellular process underlies Mendel’s law of independent assortment?
A Chromosomes align randomly during meiosis.
B Chromosomes can exchange genetic material during crossover.
C Gametes contain half the number of chromosomes of somatic cells.
D Daughter cells are genetically identical to parent cells after mitosis.
Solution The solution is (A). The law of independent assortment states that allele pairs
separate independently at the time of gamete formation. This means that traits are
transmitted to offspring independently of one another.
16 While studying meiosis, you observe that gametes receive one copy of each pair of
homologous chromosomes and one copy of the sex chromosomes. This observation is the
physical explanation of Mendel’s law of —
A dominance
B independent assortment
C random distribution of traits
D segregation
Solution The solution is (D). The law of segregation states that the two copies of each
hereditary factor segregate during the production of gametes. The offspring acquires
one factor from each parent as a result of segregation.
17 In some primroses, the petal color blue is dominant. A cross between a true-breed blue
primrose and a white primrose yields progeny with white petals. A second gene at
another locus prevented the expression of the dominant coat color.
What is this an example of?
A Codominance
B Hemizygosity

Advanced Placement Biology Instructor’s Solution Manual


248 12 | Mendel's Experiments and Heredity

C Incomplete dominance
D Epistasis
Solution The solution is (D). A cross between a true-breed blue primrose and a white
primrose which yields a progeny with white petals is an example of epistasis since
the white color allele masks or interferes with the expression of the blue color allele.
18 Purple flowers (P) are dominant over red flowers (p), and long pollen grains are dominant
over round pollen grains. When purple flowers and long pollen grain plants were crossed
with plants with white flowers and round pollen grains, all the F1 plants showed purple
flowers and long pollen grains. The F1 plants were crossed and the results are in the table.

What conclusions about the physical relationship between the traits can be drawn from
the experiment?
A The traits are probably linked.
B The traits follow the law of independent assortment.
C The traits are located on different chromosomes.
D There was epistasis.
Solution The solution is (A). The probability of having purple flowers and long pollen grains is
higher. Therefore, it is likely that the genes for purple flower color and long pollen
grains are in close proximity, and the probability for them to be inherited together
is higher.
19 When the expression of one gene pair masks or modifies the expression of another, what
do the genes show?
A Codominance
B Epistasis
C Incomplete dominance
D Partial linkage
Solution The solution is (B). The antagonistic interaction between two genes, such that one
gene masks or interferes with the expression of another gene, is called epistasis.

Advanced Placement Biology Instructor’s Solution Manual


12 | Mendel's Experiments and Heredity 249

CRITICAL THINKING QUESTIONS


20 Why is the garden pea an excellent system for studying inheritance?
A The garden pea has flowers that close tightly to promote cross-fertilization.
B The garden pea has flowers that close tightly to prevent cross-fertilization.
C The garden pea does not mature in one season and is a perennial plant.
D Male and female reproductive parts attain maturity at different times, promoting self-
fertilization.
Solution The solution is (B). The garden pea has flowers that close tightly during self-
pollination. This helped to prevent accidental or unintentional fertilizations that
could have diminished the accuracy of Mendel’s data. Garden peas naturally adopt
ways to promote self-fertilization and prevent cross-fertilization. Self-fertilization
leads to true breeding lines that avoid the appearance of unexpected traits in
offspring that might occur if the plants were not true breeding.
21 How would you perform a reciprocal cross to test stem height in the garden pea?
A First cross is performed by transferring the pollen of a heterozygous tall plant to the
stigma of a true-breeding dwarf plant. Second cross is performed by transferring the
pollen of a heterozygous dwarf plant to the stigma of a true-breeding tall plant.
B First cross is performed by transferring the pollen of a true-breeding tall plant to the
stigma of a true-breeding dwarf plant. Second cross is performed by transferring the
pollen of a true-breeding dwarf plant to the stigma of a true-breeding tall plant.
C First cross is performed by transferring the pollen of a true-breeding tall plant to the
stigma of a heterozygous dwarf plant. Second cross is performed by transferring the
pollen of a heterozygous dwarf plant to the stigma of a true-breeding tall plant.
D First cross is performed by transferring the pollen of a heterozygous tall plant to the
stigma of a heterozygous dwarf plant. Second cross is performed by transferring the
pollen of a heterozygous tall plant to the stigma of a heterozygous dwarf plant.
Solution The solution is (B). Two sets of P parents would be used. In the first cross, pollen
would be transferred from the anther of a true-breeding tall plant to the stigma of a
true-breeding dwarf plant. In the second cross, pollen would be transferred from the
anther of a true-breeding dwarf plant to the stigma of a true-breeding tall plant. For
each cross, F1 and F2 offspring would be analyzed to determine whether offspring
traits were affected according to which parent donated each trait.
22 Flower position in pea plants is determined by a gene with axial and terminal alleles.
Given that axial is dominant to terminal, what are the possible F1 and F2 genotypes and
phenotypes from a cross involving parents that are homozygous for each trait? Express
genotypes with conventional genetic abbreviations.

Advanced Placement Biology Instructor’s Solution Manual


250 12 | Mendel's Experiments and Heredity

A F1: all AA-axial; F2: AA-axial and aa-terminal


B F1: all aa-terminal; F2: AA-axial and Aa-terminal
C F1: AA-axial and Aa-terminal; F2: all AA-axial
D F1: all Aa-axial; F2: AA-axial, Aa-axial, and aa-terminal
Solution The solution is (D). Because axial is dominant, the gene would be designated A. F1
would be all heterozygous Aa with an axial phenotype. F2 would have possible
genotypes of AA, Aa, and aa; these would correspond to axial, axial, and terminal
phenotypes, respectively. F1 genotypes will all be hybrids (Aa) of both parents.
Therefore, all the flowers in F1 will be axial. F2 genotypes will segregate in the ratio
of 1AA : 2Aa : 1aa. Therefore, the F2 generation will contain both axial and terminal
flower in the ratio 3 : 1.
23 Use a Punnett square to predict the offspring in a cross between a dwarf pea plant
(homozygous recessive) and a tall pea plant (heterozygous). What is the phenotypic ratio
of the offspring?
A 1 tall : 1 dwarf
B 1 tall : 2 dwarf
C 3 tall : 1 dwarf
D 1 dwarf : 4 tall
Solution The solution is (A). The Punnett square would be 2 × 2 and will have t and t along the
top, and T and t along the left side. Clockwise from the top left, the genotypes listed
within the boxes will be Tt, Tt, tt, and tt. The phenotypic ratio will be 1 tall : 1 dwarf
after crossing a homozygous recessive dwarf pea plant and a heterozygous tall pea
plant.
24 Can a human male be a carrier of red-green color blindness?
A Yes, males can be the carriers of red-green color blindness since color blindness is
autosomal dominant.
B No, males cannot be the carriers of red-green color blindness since color blindness is
X-linked.
C No, males cannot be the carriers of red-green color blindness since color blindness is
Y-linked.
D Yes, males can be the carriers of red-green color blindness since color blindness is
autosomal recessive.
Solution The solution is (B). No, males either have colored vision or are color blind. They
cannot be carriers because an individual needs two X chromosomes to be a carrier.
This is an example of hemizygosity.

Advanced Placement Biology Instructor’s Solution Manual


12 | Mendel's Experiments and Heredity 251

25 What are the genotypes and genotypic proportions of a cross between AABBCc and
Aabbcc parents? Use the probability method for your calculations.
A Possible genotypes are AABBcc, AaBbCc, and AaBbcc, and the ratio is 1 : 2 : 1.
B Possible genotypes are AABbcc, AaBbCc, and AaBbcc, and the ratio is 1 : 3 : 1.
C Possible genotypes are AABbCc, AABbcc, AaBbCc, and AaBbcc, and the ratio is
1 : 1 : 1 : 1.
D Possible genotypes are AABbcc, AaBbCC, and AaBbcc, and the ratio is 1 : 1 : 1.
Solution The solution is (C). Considering each gene separately, the cross at A will produce
offspring of which half are AA and half are Aa; B will produce all Bb; C will produce

half Cc and half cc. Proportions then are or AABbCc; continuing

for the other possibilities yields AABbcc, AaBbCc, and AaBbcc. The
proportions therefore are 1 : 1 : 1 : 1.
26 How does the segregation of traits result in different combinations of gametes at the end
of meiosis?
A The chromosomes randomly align during anaphase I at the equator. Separation of
bivalent chromosomes occurs during metaphase I of meiosis I. Similarly, separation of
sister chromatids occurs at metaphase II of meiosis II. At the end of meiosis II, four
different gametic combinations are produced, each containing a haploid set of
chromosomes.
B The chromosomes randomly align during metaphase I at the equator, and separation
of homologous chromosomes occurs during anaphase I. Similarly, separation of sister
chromatids occurs at anaphase II of meiosis II. At the end of meiosis II, four different
gametic combinations are produced, each containing a haploid set of chromosomes.
C The chromosomes randomly align during prophase I at the equator, and separation of
sister chromatids occurs during metaphase I of meiosis I. Similarly, separation of
bivalent chromosomes occurs at metaphase II of meiosis II. At the end of meiosis II,
four different gametic combinations are produced, each containing a diploid set of
chromosomes.
D The chromosomes randomly align during prophase I at the equator, and separation of
bivalent chromosomes occurs during anaphase I of meiosis I. Similarly, separation of
homologous chromosomes occurs at metaphase II of meiosis II. At the end of meiosis
II, four different gametic combinations are produced, each containing a diploid set of
chromosomes.
Solution The solution is (B). The chromosomes randomly align during metaphase I at the
equator, and separation of homologous chromosomes occurs during anaphase I.
Similarly, separation of sister chromatids occurs at anaphase II of meiosis II. At the

Advanced Placement Biology Instructor’s Solution Manual


252 12 | Mendel's Experiments and Heredity

end of meiosis II, four different gametic combinations are produced, each containing
a haploid set of chromosomes. Four different combinations of gametes are
produced: RT, Rt, rT, and rt. Separation of bivalent chromosomes occurs at anaphase
I of meiosis I, resulting in two daughter cells. Separation of sister chromatids occurs
at anaphase II of meiosis II, resulting in four different gametic combinations each
containing a haploid set of chromosomes.
27 In Section 12.3 Laws of Inheritance, an example of epistasis was given for summer squash.
Cross white WwYy heterozygotes to demonstrate the phenotypic ratio of 12 white:
3 yellow : 1 green that was given in the text.
A Twelve offspring are white because the W gene is epistatic to the Y gene. Three
offspring are yellow, because w is not epistatic. The green offspring is obtained when
the recessive form of both genes (wwyy) is present.
B Twelve offspring are white because W gene is hypostatic to Y gene. Three offspring are
yellow because Y is epistatic to w. The green offspring is obtained when the dominant
form of both the genes (WWYY) is present.
C Twelve offspring are white because W gene is dominant. Three offspring are yellow
because Y is dominant and w is recessive. The green offspring is obtained when the
recessive form of both the genes (wwyy) is present, showing codominance.
D Twelve offspring are white because W is epistatic to Y gene. Three offspring are yellow
because Y is hypostatic to w. The green offspring is obtained when the recessive form
of both the genes (wwyy) are present, showing codominance.
Solution The solution is (C). The cross can be represented as a 4 × 4 Punnett square, with the
following gametes for each parent: WY, Wy, wY, and wy. For all 12 of the offspring
that express a dominant W gene, the offspring will be white. The three offspring that
are homozygous recessive for w but express a dominant Y gene will be yellow. The
remaining wwyy offspring will be green.

TEST PREP FOR AP® COURSES


28 The trait for widow’s peak can be considered a monoallelic dominant trait in humans. If a
man with a widow’s peak and a woman with a straight hairline have a child together,
what is the probability that the child will inherit the widow’s peak if the father’s mother
had a straight hairline?
A 0.25
B 0.50
C 0.75
D 1
Solution The solution is (B). The father is heterozygous dominant for the trait (Ww), and the
mother is homozygous recessive (ww). The offspring produced from such parents

Advanced Placement Biology Instructor’s Solution Manual


12 | Mendel's Experiments and Heredity 253

would be 50 percent Ww (widow’s peak) and 50 percent ww (straight hairline). The


probability of having a child with the dominant trait is 50 percent, or 0.50.

Advanced Placement Biology Instructor’s Solution Manual


254 12 | Mendel's Experiments and Heredity

29 Don’t like brussels sprouts? Blame your genes. The chemical PTC (phenylthiocarbamide),
which is nearly identical to a compound found in the cabbage family, tastes very bitter to
some people. Others cannot detect a taste. The ability to taste PTC is incompletely
dominant and is controlled by a gene on chromosome 7. A woman who finds brussels
sprouts mildly distasteful—in other words, who can taste PTC weakly—has a child with a
man who hates brussels sprouts—in other words, who can taste PTC strongly.
What is the probability that their son likes brussels sprouts—in other words, cannot
taste PTC?
A 0
B 0.25
C 0.50
D 1
Solution The solution is (A). The mother is a weak taster for PTC test as she does not like
brussels sprouts, which means she is heterozygous for the trait (Tt). The father is a
strong taster (TT) for PTC taste, so he hates brussels sprouts. Therefore, the
probability of their child (Tt-weak taster) liking brussels sprouts will be 0.
30 Tay-Sachs disease is an autosomal recessive disorder that causes severe problems in
neurons. Children who receive two copies of the gene rarely live beyond the age of five.
There is no cure for the disease. During a genetic screening, a couple is told that both
partners carry the recessive gene.
What kind of issue must the couple confront?
A Scientific
B Financial
C Ethical
D Educational
Solution The solution is (C). Parents will have to consider if it is right to undergo prenatal
screening and terminate a pregnancy, which may be unethical.
31 A couple has three daughters. What is the probability that the next child they have will be
a daughter?
A 0%
B 25%
C 50%
D 100%
Solution The solution is (C). The probability that the couple’s next child will be a daughter will
be 50 percent because there are equal chances of having a son or a daughter.

Advanced Placement Biology Instructor’s Solution Manual


12 | Mendel's Experiments and Heredity 255

32 What is the probability that a couple will have three daughters?

Solution The solution is (D). The probability of having a girl is Therefore, the probability

that a couple will have three daughters is


33 Petunias can be blue, red, or violet. When a blue flower is crossed with a red flower, all
the resulting flowers are violet. When a violet flower is crossed with a red flower, about
half of the flowers are violet and half are red.
How do you characterize the color trait?
A Complete dominance
B Codominance
C Incomplete dominance
D Sex linked
Solution The solution is (C). A cross between a blue (BB) flower and a red (RR) flower
produces all violet (BR) flowers as a result of incomplete dominance. Both alleles
when present produce intermediate inheritance, in which one allele for the trait is
not completely expressed over its paired allele.
34 Petunias can be blue, red, or violet. When a blue flower is crossed with a red flower, all
the resulting flowers are violet. Two violet petunias are crossed.
What is the most probable result of the cross?
A 75 percent blue and 25 percent red
B 50 percent blue and 50 percent red
C 75 percent red and 25 percent blue
D 25 percent blue, 50 percent violet, and 25 percent red
Solution The solution is (D). This is a case of incomplete dominance where one allele for
the trait is not completely expressed over its paired allele, resulting in an

Advanced Placement Biology Instructor’s Solution Manual


256 12 | Mendel's Experiments and Heredity

intermediate phenotype (violet). The cross between two violet petunia flowers
(BR) will produce 25 percent blue flowers (BB), 75 percent violet flowers, and
25 percent red flowers (RR).
35 Fruit flies (Drosophila melanogaster) with a wild-type phenotype have gray bodies and red
eyes. Certain mutations can cause changes to these traits. Mutant flies may have a black
body and/or cinnabar eyes. To study the genetics of these traits, a researcher crossed a
true-breeding wild-type male fly with a true-breeding female fly with a black body and
cinnabar eyes. All of the F1 progeny displayed a wild-type phenotype.
Which statement is correct about the traits observed?
A Gray body and cinnabar eyes are dominant.
B Eye color is sex linked.
C Body color is sex linked.
D Gray body and red eyes are dominant.
Solution The solution is (D). A cross between wild-type males (GGRR) and mutant females
(ggrr) produce all wild-type progenies (GgRr), showing that gray body is dominant
over black body, and red eyes are dominant over cinnabar eyes.
36 Female flies from the F1 generation were crossed with true-breeding male flies with black
bodies and cinnabar eyes. The table represents the predicted outcome and the data
obtained from the cross.

Which assumption led to the predicted numbers?


A The traits assort independently.
B The traits are located on the X chromosome.
C The traits are on the same chromosome.
D The female flies were homozygous for wild-type alleles.
Solution The solution is (A). All the phenotypes are observed in equal numbers (1 : 1 : 1 : 1).
Therefore, the traits follow the law of independent assortment.
37 Cats can be black, yellow, or calico (black and yellow patches). Coat color is carried on the
X chromosome.
What type of inheritance is color coat in cats?
A Codominance

Advanced Placement Biology Instructor’s Solution Manual


12 | Mendel's Experiments and Heredity 257

B Incomplete dominance
C Codominance, sex linked
D Incomplete dominance, sex linked
Solution The solution is (C). Coat color is governed by two alleles, black (B) and yellow (Y).
When both alleles are present, they exhibit codominance. The development of
colored patches requires two X chromosomes, and hence under normal conditions,
the calico phenotype is observed in females and not in males.
38 Cats can be black, yellow, or calico (black and yellow patches). Coat color is carried on the
X chromosome. A yellow cat is crossed with a black cat. Assume that the offspring are
both male and female.
What are the phenotypes of the offspring and in what proportions?
A All the cats are yellow.
B All the cats are black.
C All the cats are calico.
D There is not enough information to answer the question.
Solution The solution is (D). The question does not state the sex of either parent cat.
Information is missing.

SCIENCE PRACTICE CHALLENGE QUESTIONS


12.2 Characteristics and Traits
39 The gene SLC24A5 encodes an antiporter membrane protein that exchanges sodium for
calcium (R. Ginger et al., JBC, 2007). This process has a role in the synthesis of the
melanosomes that cause skin pigmentation. A mutation in this gene affecting a single
amino acid occurs in humans. The homozygous mutant gene is found in 99 percent of
humans with European origins. Both the wild type and mutant display codominance.
A. Representing the wild-type form of the gene as +/+ and the mutant form of the gene as
m/m for two homozygous parents, construct a Punnett square for this cross using the first
grid below. Annotate your representation to identify the phenotypes with high (H),
intermediate (I), and low (L) melanosome production. Use the second grid to represent an
F2 generation from the offspring of the first cross. Use annotation to show the phenotype.

Advanced Placement Biology Instructor’s Solution Manual


258 12 | Mendel's Experiments and Heredity

F1 m m
+ Blank

+ blank blank

F2 blank Blank

Blank Blank Blank

Blank Blank blank

B. Draw sister chromatids at anaphase II for both parents in the F1 generation, and
annotate your drawing to identify each genotype of the gametes using the cells of the
Punnett square.
C. Explain which of Mendel's laws is violated by codominance.
D. Suppose that these data were available to evaluate the claim that the wild-type and
mutant forms of SLC24A5 are codominant:
F2 blank blank

Phenotype Observed Expected


H 1,206 blank

I 2,238 blank

L 1,124 blank

Complete the table. Explain the values expected in terms of the genotype of the offspring.

E. Using a statistic at the 95 percent confidence level, evaluate the claim that the wild-
type and mutant forms of SCLO24A5 are codominant. The definition of the statistic

,
where is the chi-square test statistic, c is the significant level of the test (we will use
0.05), O is the observed value for variable i, and E is the expected value for variable i. The
chi-square statistic table is provided in the AP Biology Exam.
p 1 2 3 4 5 6 7 8
0.05 3.84 5.99 7.82 9.49 11.07 12.59 14.07 15.51
Degrees of Freedom

Advanced Placement Biology Instructor’s Solution Manual


12 | Mendel's Experiments and Heredity 259

Solution Sample answer:


A.
F1 m m
+ +/m I +/m I
+ +/m I +/m I

F2 m +
m m/m L +/m I
+ +/m I +/+ H
B. The drawing shows a pair of chromatids for each parent on which there only wild-
type on one pair and only mutant on the other.
C. Mendel thought that genes were either dominant or recessive. He did not
consider the idea that both traits can be dominant at once.
D. Heterozygotes express the both alleles.
E.
Phenotype Observed Expected (o − e)2/e
H 1,206 1,142 3.59
I 2,238 2,284 0.93
L 1,124 1,142 0.28
The sum of the last column is 4.80, which is less than 5.99 (using the table of critical
values provided at the Exam), so the claim of codominance cannot be rejected based
on these data.
40 Adrenoleukodystrophy (ALD) is a genetic disorder in which lipids with very high molecular
weights are not metabolized and accumulate within cells. Accumulation of these fats in
the brain damages the myelin that surrounds nerves. This progressive disease has two
causes: an autosomal recessive allele, which causes neonatal ALD, and a mutation in the
ABCD1 gene located on the X chromosome. A controversial treatment is the use of
Lorenzo’s oil, which is expensive; despite this treatment, neurological degradation persists
in many patients. Gene therapy as a potential treatment is currently in trials but is also
very costly.
An infant patient exhibits symptoms of neonatal ALD, which are difficult to distinguish
from the X-linked form of the disease. The infant’s physician consults electronic health
records to construct a pedigree showing family members who also presented symptoms
similar to ALD. The pedigree is shown in this diagram. The infant patient is circled.
Symbols for males (o) and females (m) are filled when symptoms are present.

Advanced Placement Biology Instructor’s Solution Manual


260 12 | Mendel's Experiments and Heredity

A. Using the pedigree, explain which form of ALD (neonatal or X linked) is present in
the infant.
B. Sharing of digital records among health providers is one method proposed to improve
the quality and reduce the cost of health care in the United States. The privacy of
electronic health records is a concern. Pose three questions that must be addressed in
developing policies that balance the costs of treatments and diagnoses, patient quality of
life, and risks to individual privacy.
Solution Sample answer:
A. The disease is not X linked. It might be neonatal. The great aunt disproves
sex linked.
Autosomal recessive:

X linked:

Advanced Placement Biology Instructor’s Solution Manual


12 | Mendel's Experiments and Heredity 261

B. Questions might arise from these types of considerations:


 The U.S. Department of Health and Human Services cited 380 breaches of
healthcare records involving 500 or more patients in 2011. Are these records
secure? Who is responsible for their security?
 Insurance companies have traditionally charged rates that are fixed by pre-
existing conditions. Access to care could be affected. Should rates be
adjusted to genome as they are now according to certain habits?
 Potential employers may have an interest in these records. Can employers
gain access? Will the precedent of mandatory drug testing be broadened?
 These records might have prosecutorial value that affects the rights of both
the accused and the accuser. My DNA is admissible for forensic purposes, so
will it also be used to infer guilt or innocence of intent or “natural
tendencies.”
 In some scenarios (the movie Gattaca, for example) the meaning of “health
records” could evolve with unintended consequences if the whole genome is
recorded. In the most efficient societies, your role is matched to your abilities
—can these be evaluated without my participation?
41 Two genes, A and B, are located adjacent to each other (linked) on the same
chromosome. In the original cross (P0), one parent is homozygous dominant for both traits
(AB), whereas the other parent is recessive (ab).
Characteristic Alleles Chromosome
Seed color Yellow (I)/green (i) 1
Seed coat and Colored (A)/white (a) 1
flowers
Mature pods Smooth (V)/wrinkled (v) 4
Flower stalk From leaf axils (Fa)/umbellate 4
at top of plant (fa)
Height >1m (Le)/−0.5 m (le) 4
Unripe pods Green (Gp)/yellow (gp) 5
Mature seeds Smooth (R)/wrinkled (r) 7
A. Describe the distribution of genotypes and phenotypes in F1.
B. Describe the distribution of genotypes and phenotypes when F1 is crossed with the
ab parent.
C. Describe the distribution of genotypes and phenotypes when F1 is crossed with the AB
parent.
D. Explain the observed non-Mendelian results in terms of the violation of the laws
governing Mendelian genetics.

Advanced Placement Biology Instructor’s Solution Manual


262 12 | Mendel's Experiments and Heredity

Solution Sample answer:


A. AB × ab where AB and ab can be treated as a single gene results in only AaBb.
B. The possible gametes from AaBb when treated as a single gene are AB and ab. So,
the result of xab is AaBb and aabb. The phenotype is 1 : 1 with equal probabilities
doubly dominant and doubly recessive.
C. The possible gametes from AaBb when treated as a single gene are AB and ab. So,
the result of xAB is AABB and AaBb. All offspring are dominant in both genes.
D. Mendelian genetics assumes that the genes assort independently. If they cannot
crossover—because they are adjacent—then the outcomes are as described and
non-Mendelian.
12.3 Laws of Inheritance
42 Gregor Mendel’s 1865 paper described experiments on the inheritance of seven
characteristics of Pisum sativum, shown in the first column in the table below. Many years
later, based on his reported outcomes and analysis of the inheritance of a single
characteristic, Mendel developed the concepts of genes, their alleles, and dominance.
These concepts are defined in the second column of the table, using conventional symbols
for the dominant allele for each characteristic. Even later, the location of each of these
genes on one of the seven chromosomes in P. sativum was determined, as shown in the
third column.
A. Before the acceptance of what Mendel called “factors” as the discrete units of
inheritance, the accepted model was that the traits of progeny were “blended” traits of
the parents. Evaluate the evidence provided by Mendel’s experiments in disproving the
blending theory of inheritance.
B. Mendel published experimental data and analysis for two experiments involving the
inheritance of more than a single characteristic. He examined two-character inheritance
of seed shape and seed color. He also reported three-character inheritance of seed shape,
seed color, and flower color. Evaluate the evidence provided by the multiple-character
experiments. Identify which of the following laws of inheritance depend upon these
multiple-character experiments for support:
i. During gamete formation, the alleles for each gene segregate from each other so
that each gamete carries only one allele for each gene.
ii. Genes for different traits can segregate independently during the formation of
gametes.
iii. Some alleles are dominant, whereas others are recessive. An organism with at
least one dominant allele will display the effect of the dominant allele.
iv. All three laws can be inferred from the single-character experiments.
C. As shown in the table, some chromosomes contain the gene for more than one of the
seven characteristics Mendel studied, for example, seed color and flowers. The table

Advanced Placement Biology Instructor’s Solution Manual


12 | Mendel's Experiments and Heredity 263

below shows, with filled cells above the dashed diagonal line, the combinations of
characteristics for which Mendel reported results. In the cells below the dotted diagonal
line, identify with an X each cell where deviations from the law or laws identified in part B
might be expected.

D. Explain the reasons for the expected deviations for those combinations of
characteristics identified in part C.
E. In one of the experiments reported by Mendel, deviations from the law identified in
part B might be expected. Explain how the outcomes of this experiment were consistent
with Mendel’s laws.
Solution Sample answer:
A. Characters are always in one of two forms for these experiments. There is no
progeny with intermediate characters.
B. The correct choice is ii. Independent assortment requires that genes are not
correlated. This can only be tested by considering the distribution of outcomes from
more than one gene.
C.

D. Deviations are expected if the genes are on the same chromosome.


E. Outcomes from the two-character experiment involving seed shape and color
showed independent assortment, although they are on the same chromosome.
Recombination through crossing over restores the independence.

Advanced Placement Biology Instructor’s Solution Manual


264 12 | Mendel's Experiments and Heredity

43 A dihybrid cross involves two traits. A cross of parental types AaBb and AaBb can be
represented with a Punnett square:

This representation clearly organizes all of the possible genotypes and reveals the
9 : 3 : 3 : 1 distribution of phenotypes and a 4 × 4 grid of 16 cells. Expressed as a fraction
of the 16 possible genotypes of the offspring, the phenotypic ratio describes the
probability of each phenotype among the offspring:
3 (AA, Aa, aA) × 3 (BB, bB, Bb)/16 = 9/16
3 (AA, Aa, aA) × 1 (bb) /16 = 3/16
1 (aa) × 3 (BB, bB, Bb) = 3/16
1 (aa) × 1 (bb) = 1/16
A. Using the probability method, calculate the likelihood of these phenotypes from each
dihybrid cross:
 Recessive in the gene with alleles A and a from the cross AaBb × aabb
 Dominant in both genes from the cross AaBb × aabb
 Recessive in both genes from the cross AaBb × aabb
 Recessive in either gene from the cross AaBb × aabb
A Punnett square representation of a trihybrid cross, such as the self-cross of AaBbCc,
is more cumbersome because there are eight columns and rows (2 × 2 × 2 ways to
choose parental genotypes) and 64 cells. A less tedious representation is to calculate the
number of each type of genotype in the offspring directly by counting the unique
permutations of the letters representing the alleles. For example, the probability of the
cross AaBbCc × AaBbCc is 3 (AA, Aa, aA) × 3 (BB, Bb, bB) × 3 (CC, Cc, cC)/64 = 27/64.

Advanced Placement Biology Instructor’s Solution Manual


12 | Mendel's Experiments and Heredity 265

B. Using the probability method, calculate the likelihood of these phenotypes from each
trihybrid cross:
 Recessive in all traits from the cross AaBbCc × aabbcc
 Recessive in the gene with alleles C and c and dominant in the other two traits
from the cross AaBbCc × AaBbCc
 Dominant in the gene with alleles A and a and recessive in the other two traits
from the cross AaBbcc × AaBbCc
C. The probability method is an easy way to calculate the likelihood of each particular
phenotype, but it doesn’t simultaneously display the probability of all possible
phenotypes. The forked line representation described in the text allows the entire
phenotypic distribution to be displayed. Using the forked line method, calculate the
probabilities in a cross between AABBCc and Aabbcc parents:
 All traits are recessive: aabbcc.
 Traits are dominant at each loci, A? B? C?
 Traits are dominant at two genes and recessive at the third.
 Traits are dominant at one gene and recessive at the other two.
Solution Sample answer:
A.
 Recessive in the gene with alleles A and a from the cross AaBb × aabb:
1 (aa) × 2 (bb or Bb) /4 = 1/2
 Dominant in both genes from the cross AaBb × aabb:
1 (Aa) × 0 (bb) /4 + 1 (Bb) × 0 (aa) /4 = 0
 Recessive in both genes from the cross AaBb × aabb:
1 (aa) × 1 (bb) /4 = 1/4
 Recessive in either gene from the cross AaBb × aabb:
[1 (aa) × 2 (Bb or bb) + 1 (Aa) × 2 (bb)]/4 = 1
B.
 Recessive in all traits from the cross AaBbCc × aabbcc:
1 (aa) × 1 (bb) × 1 (cc) /8 = 1/8
 Recessive in the gene with alleles C and c and dominant in the other two
traits from the cross AaBbCc × AaBbCc:
1 (cc) × 3 (AA, Aa, aA) ו 3 (BB, Bb, bB) = 9/64
 Dominant in the gene with allele A and a and recessive in the other two traits
from the cross AaBbcc × AaBbCc:
1 (bb) × 1 (cc) × 3 (AA, Aa, aA) /64 = 3/64

Advanced Placement Biology Instructor’s Solution Manual


266 12 | Mendel's Experiments and Heredity

C. We count the ways of particular phenotypes


 Having all recessive: aabbcc—just one way
 Having all dominant at each: A? B? C?—3 ways for each, such as AA, Aa, and
aA—27 ways
 Having dominant at two genes and recessive at the third: A?B?cc, A?bbC?,
and aaB?C? is 9 + 9 + 9 = 27
 Having dominant at one gene and recessive at the other two: A?bbcc, aaB?
cc, and aabbC? is 3 + 3 + 3 = 9
The probability of each phenotype is equal to the number of ways of making it from
the genotypes divided by the total number of genotypes:
All dominant: 27/64 = 0.42
Dominant A and recessive at B and C: 3/64 = 0.047
Dominant B and recessive at A and C: 3/64 = 0.047
Dominant C and recessive at A and B: 3/64 = 0.047
Dominant A and B and recessive at C: 9/64 = 0.14
Dominant A and C and recessive at B: 9/64 = 0.14
Dominant B and C and recessive at A: 9/64 = 0.14
All recessive: 1/64 = 0.0156
The sum of the probabilities of each outcome is equal to 1.
44 Construct a representation showing the connection between the process of meiosis and
the transmission of six possible phenotypes from parents to F2 offspring. The phenotypes
are labeled A, a, B, b and C, c. Expression of each phenotype is controlled by a separate
Mendelian gene. Your representation should show the proportion of every possible
combination of phenotypes (e.g., ABC, AbC, etc.) that will be present in the F2 offspring.

Advanced Placement Biology Instructor’s Solution Manual


12 | Mendel's Experiments and Heredity 267

Solution

Advanced Placement Biology Instructor’s Solution Manual


268 13 | Modern Understandings of Inheritance

13| MODERN UNDERSTANDINGS OF


INHERITANCE
REVIEW QUESTIONS
1 When comparing humans (or in Drosophila), are X-linked recessive traits observed more
frequently in males, in similar numbers between males and females, more frequently in
females, or is the frequency different depending on the trait?
A In more males than females
B In more females than males
C In males and females equally
D In different distributions depending on the trait
Solution The solution is (A). Affected males result from mothers who are affected
(homozygous) or who are carriers (heterozygous) for the X-linked recessive trait.
Males possess only one X chromosome; therefore, males will always have the
affected trait if obtained from a carrier or affected mother.
2 Which recombination frequency corresponds to perfect linkage and violates the law of
independent assortment?
A 0
B 0.25
C 0.5
D 0.75
Solution The solution is (A). The recombination frequency ranges from 0 percent (linked
genes) to 50 percent (unlinked genes). Zero represents genes in close proximity to
each other and segregated together, thus violating Mendel’s law of independent
assortment.
3 Which recombination frequency corresponds to independent assortment and the absence
of linkage?
A 0
B 0.25
C 0.5
D 0.75

Advanced Placement Biology Instructor’s Solution Manu


13 | Modern Understandings of Inheritance PAGE \* MERGEFORMAT 277275

Solution The solution is (C). The 0.50 recombination frequency corresponds to the absence of
linkage. Genes undergoing recombination assort independently of each other into
one of two gamete cells.
4 Based on the diagram, which statement is true?

A Recombination of the body color and red/cinnabar eye alleles will occur more
frequently than recombination of the alleles for wing length and aristae length.
B Recombination of the body color and aristae length alleles will occur more frequently
than recombination of red/brown eye alleles and the aristae length alleles.
C Recombination of the gray/black body color and long/short aristae alleles will not
occur.
D Recombination of the red/brown eye and long/short aristae alleles will occur more
frequently than recombination of the alleles for wing length and body color.
Solution The solution is (D).
Recombination of the red/brown eye and long/short aristae alleles will occur more
frequently than recombination of the alleles for wing length and body color because of
the greater map distance between the red/brown eye and long/short aristae.
5 Which code describes position 12 on the long arm of chromosome 13?
A 13p12
B 13q12
C 12p13
D 12q13

Advanced Placement Biology Instructor’s Solution Manu


270 13 | Modern Understandings of Inheritance

Solution The solution is (B). 13q12 code describes position 12 on the long arm of
chromosome 13.

Advanced Placement Biology Instructor’s Solution Manu


13 | Modern Understandings of Inheritance PAGE \* MERGEFORMAT 277275

6 Assume a pericentric inversion occurred in one of two homologs prior to meiosis. The
other homolog remains normal. During meiosis, what structure, if any, would these
homologs assume in order to pair accurately along their lengths?
A V formation
B Cruciform
C Loop
D A pairing would not be possible.
Solution The solution is (C). A loop is formed during a crossover between a chromosome with
pericentric inversion and its normal homologous chromosome. This results in the
formation of chromatids with deletions or entire segments missing.

CRITICAL THINKING QUESTIONS


7 Which statement best describes the Chromosomal Theory of Inheritance?
A The theory was proposed by Charles Darwin. It describes the units of inheritance
between parents and offspring, as well as the processes by which those units control
offspring development.
B The theory was proposed by Boveri-Sutton. It describes linkage, recombination, and
crossing over, and states that Mendelian genes have specific loci on chromosomes,
which undergo segregation and independent assortment.
C The theory was proposed by Charles Darwin. It states the Mendelian genes have two
alternate forms and undergo independent assortment. It helped increase the
understanding of linkage and recombination.
D The theory was proposed by Boveri-Sutton. It describes the units of inheritance
between parents and offspring as well as the processes by which those units control
offspring development.
Solution The solution is (B). The theory was proposed by Boveri-Sutton. It describes linkage,
recombination, and crossing over and states the Mendelian genes have specific loci
on chromosomes, which undergo segregation and independent assortment.
8 In a test cross for two characteristics (dihybrid cross), can the predicted frequency of
recombinant offspring be 60 percent? Why or why not?
A No, the predicted frequency of recombinant offspring ranges from 0 percent (for
linked traits) to 50 percent (for unlinked traits) because of both parental and
nonparental cases.
B Yes, the predicted frequency of recombinant offspring can be 60 percent if genes are
located very far from each other.

Advanced Placement Biology Instructor’s Solution Manu


272 13 | Modern Understandings of Inheritance

C Yes, the predicted frequency can be 60 percent if crossing over occurs during every
meiotic event.
D No, the predicted frequency can never be 60 percent due to the presence of
mutations such as deletions.
Solution The correct solution is (A). No. The predicted frequency of recombinant offspring
ranges from 0 percent (for linked traits) to 50 percent (for unlinked traits).
9 Which statement best describes how nondisjunction can result in an aneuploid zygote?

A Nondisjunction only occurs when homologous chromosomes do not separate during


meiosis I, resulting in the formation of gametes containing n + 1 and n − 1
chromosomes.
B Nondisjunction only occurs when sister chromatids do not separate in meiosis II,
resulting in the formation of gametes containing n + 1 and n − 1 chromosomes.
C Nondisjunction is the failure of homologous chromosomes to separate during meiosis I
or the failure of sister chromatids to separate during meiosis II, leading to the
formation of n + 1/n − 1/n chromosomes.
D Nondisjunction occurs when the sister chromatids fail to separate during mitosis II,
resulting in the formation of gametes containing n + 1 and n − 1/n chromosomes.

Advanced Placement Biology Instructor’s Solution Manu


13 | Modern Understandings of Inheritance PAGE \* MERGEFORMAT 277275

Solution The solution is (C). Nondisjunction is the failure of homologous chromosomes to


separate during meiosis I or the failure of sister chromatids to separate during
meiosis II, leading to the formation of n + 1/n − 1/n chromosomes.
10 Which answer correctly identifies the various chromosomal aberrations and their worst-
case negative consequences?
A Nondisjunction - aneuploid gametes; duplication - physical and mental abnormalities;
deletion - lethal to a diploid organism; inversion - chromosomal breaks in gene; and
translocations - effects depend on how positions of genes are altered
B Nondisjunction - physical and mental abnormalities; inversion - genetic imbalance;
duplication - aneuploid gametes; translocations - chromosomal breaks in the gene;
and deletion - effects depend on how positions of genes are altered
C Deletion - aneuploid gametes; translocations - physical and mental abnormalities;
duplication - effects depend on positions of genes; nondisjunction - causes genetic
imbalance lethal to a diploid organism; and aneuploidy - leads to various syndromes
D Nondisjunction - chromosomal breaks in gene; duplication - physical and mental
abnormalities; deletion - genetic imbalance lethal to a diploid organism; inversion -
aneuploid gametes; and translocations - effects depend on positions of genes
Solution The solution is (A). Nondisjunction - aneuploid gametes; duplication - physical and
mental abnormalities; deletion - lethal to a diploid organism; inversion -
chromosomal breaks in gene; translocations - effects depend on how positions of
genes are altered.

TEST PREP FOR AP® COURSES


11 The following figure represents a Drosophila linkage map for genes A–E. The numbers
between the gene loci are the relative map units between each gene.

Based on the linkage map, which two genes are most likely to segregate together?
A Genes A and B
B Genes B and C
C Genes C and D
D Genes D and E
Solution The solution is (C). Genes C and D are at minimum distance from each other and,
therefore, are most likely to segregate together.

Advanced Placement Biology Instructor’s Solution Manu


274 13 | Modern Understandings of Inheritance

12 A test cross was made between true-breeding EEWW flies and eeww flies. The resulting
F1 generation was then crossed with eeww flies. One hundred offspring in the F2
generation were examined, and it was discovered that the E and W genes were not linked.
Which is the correct genotype of the F2 offspring if the genes were linked and if the genes
were NOT linked?
A Linked: 50 percent EEWW and 50 percent eeww; not linked: 25 percent EeWw, 25
percent, Eeww 25 percent eeWw, and 25 percent eeww
B Linked: 25 percent Eeww, 50 percent eeWw; not linked: parental genotypes EeWw
and eeww
C Linked: genotypes EeWw and eeww, and recombinant genotypes Eeww and eeWw in
the F2 generation are nearly the same irrespective of their linkage
D Linked: mostly with parental genotypes, Eeww and eeWw; unlinked: 25 percent EeWw
and eeww with 75 percent Eeww and eeWw
Solution The solution is (A). The predicted genotypes are (parental): EEWW, eeww;
(recombinants): Eeww, eeWw. If the genes are not linked, each genotype should be
found in 25 progeny. If the genes were linked, there would be more progeny with
the parental genotypes than the recombinant genotypes.
13 A cross was made with true-breeding AABB flies and true-breeding aabb flies. The
resulting F1 generation then was crossed with true-breeding aabb flies. Based on the
linkage map, which F2 generation genotype ratio is most likely to be observed?

A Number observed: AaBb (46), Aabb (4), aaBb (4), Aabb (46)
B Number observed: AaBb (4), Aabb (46), aaBb (46), Aabb (4)
C Number observed: AaBb (25), Aabb (25), aaBb (25), Aabb (25)
D Number observed: AaBb (50), Aabb (0), aaBb (0), Aabb (50)
Solution The solution is (C). This is a case of test cross. Therefore, the ratio obtained will be
1 : 1 : 1 : 1 (25 : 25 : 25 : 25).

Advanced Placement Biology Instructor’s Solution Manu


13 | Modern Understandings of Inheritance PAGE \* MERGEFORMAT 277275

14 Which symptom is most likely associated with the disorder shown in the karyotype?

A Lethality
B Infertility
C Heart and bleeding defects
D Short stature and stunted growth
Solution The solution is (B). The karyotype represents the 47 XXY condition, known as
Klinefelter syndrome. This syndrome leads to infertility.
15 Which option describes the disorder shown in the karyotype and the social, ethical, or
medical issue related to the disorder?

Advanced Placement Biology Instructor’s Solution Manu


276 13 | Modern Understandings of Inheritance

A Down syndrome (47 XY +21) causes intellectual disability, vision problems, congenital
heart disease, and susceptibility to cancer. Healthcare providers often do not discuss
the positive aspects of raising a child with Down syndrome and often provide out-of-
date information.
B Klinefelter syndrome (47 XXY) causes intellectual disability, vision problems,
congenital heart disease, and susceptibility to cancer. Arguments often are made
against abortion of an affected fetus.
C Klinefelter syndrome (47 XXY) causes sterility and reduced testosterone production.
Arguments often are made against informing insurance companies about a diagnosis
of this disease.
D Down syndrome (47 XY +21) causes sterility and lower testosterone production.
Arguments often are made against informing insurance companies about a diagnosis
of this disease.
Solution The solution is (C). The karyotype shows Klinefelter syndrome, with 47 XXY. Medical
issues related to the disorder include sterility, and lower testosterone production.
An ethical issue concerned with the disorder is whether information about genetic
diagnosis should be given to insurance companies.
16 Which gene order is the most likely outcome of an inversion mutation in the chromosome
shown?

A RSTUV
B RRSTUV
C RSUV
D RTSUV
Solution The solution is (D). RTSUV is most likely the outcome of an inversion mutation.
Inversion leads to a 180-degree rotation of the chromosomal segment, leading to
abnormal gene order.

Advanced Placement Biology Instructor’s Solution Manu


13 | Modern Understandings of Inheritance PAGE \* MERGEFORMAT 277275

17 With the help of the diagram given, which statement most appropriately describes
nondisjunction and its genetic consequences?

A Nondisjunction occurs when a homologous pair is unable to separate during meiosis I,


resulting in the formation of gametes containing n + 1 and n − 1 chromosomes. This is
called aneuploidy.
B Nondisjunction occurs due to the inability of sister chromatids to separate during
meiosis II, resulting in the formation of gametes containing n + 1 and n − 1
chromosomes. This results in heart and bleeding defects.
C Nondisjunction is the failure of homologous chromosomes to separate during meiosis I
or failure of sister chromatids to separate during meiosis II. This results in aneuploid
gametes.
D Nondisjunction occurs when a pair of homologous chromosomes fails to segregate
during meiosis II resulting in the formation of gametes containing n + 1, n − 1, or n
numbers of chromosomes. This results in abnormal growth patterns.
Solution The solution is (C). Nondisjunction occurs when a pair of homologous chromosomes
fails to separate during meiosis I or sister chromatids fail to separate during meiosis
II. Nondisjunction leads to individuals containing aneuploid gametes.

Advanced Placement Biology Instructor’s Solution Manu


278 13 | Modern Understandings of Inheritance

18 Klinefelter syndrome is —
A more severe than Down syndrome due to gene deletions in Klinefelter syndrome
B more severe than Down syndrome due to trisomy in Klinefelter syndrome
C less severe than Down syndrome due to monosomy in Down syndrome
D less severe than Down syndrome due to X-inactivation in this disorder
Solution The solution is (D). Klinefelter syndrome is less severe as compared to Down
syndrome. In Klinefelter syndrome, the extra copy of the X chromosome is
inactivated. In Down syndrome, an extra copy of the chromosome 21 means
individuals have three copies of each chromosome 21 gene instead of two, making it
difficult for cells to properly control how much protein is made. Producing too much
or too little protein can have serious consequences.

SCIENCE PRACTICE CHALLENGE QUESTIONS


13.1 Chromosomal Theory and Genetic Linkages
19 Drosophila that are true breeding for the traits straight wings (S) and red eyes (R) are
crossed with flies that are true breeding for curved wings (s) and brown eyes (r). A test
cross then is made between the offspring and the true-breeding ssrr flies.
A. Use the symbols S, s, R, and r to construct a representation of the parental genotypes
in the test cross.
B. If these genes are located on different chromosomes, use a Punnett square to
construct a representation of the offspring of the test cross.
C. Predict the distribution of genotypes and phenotypes resulting from the test cross.
D. As it happens, these genes are both on chromosome II as shown below. Use the
symbols S, s, R, and r to construct a representation of the parental and recombinant
genotypes in the test cross.
E. Suppose that 500 flies are produced in the test cross. Apply mathematical methods to
calculate the expected number of recombinant offspring using the linear map units (LMU)
shown in the diagram.

Advanced Placement Biology Instructor’s Solution Manu


13 | Modern Understandings of Inheritance PAGE \* MERGEFORMAT 277275

Solution Sample answer:


A. The F1 generation is SSRR × ssrr to give SsRr for one parent in the test cross. The
other is ssrr.
B.
blank
sr Description
SR SsRr Long wing, red eye
Sr Ssrr Long wing, brown eye
sR ssRr Short wing, red eye
sr ssrr Short wing, brown eye
C. The genotypes and phenotypes will be 1 : 1 : 1 : 1.
D. The parentals are SsRr and ssrr. The recombinants are ssRr and Ssrr.
E. The distance in linkage map units is 29, so the recombinant frequency is 0.29. So,
145 recombinant offspring are expected.
20 Studies like the one described in question AP12.1 were carried out by Morgan and
Sturtevant beginning in 1911. The discovery of linkage was made by Bateson and Punnett
in 1900. They crossed a true-breeding purple (P) plant with long seeds (L) with a true-
breeding red (r) plant with round seeds (l). They then performed a self-cross between the
F1 generation. They obtained the F2 data shown in the table.
Phenotype Genotype(s) Observed Expected
Purple, long 4,831
Purple, round 390
Red, long 393
Red, round 1,338

Advanced Placement Biology Instructor’s Solution Manu


280 13 | Modern Understandings of Inheritance

Phenotype Genotype(s) Observed Expected


Total 6,952
A. Use the symbols P, p, L, and l to construct a representation of the F2 genotypes and
complete the second column in the table.
B. Complete the fourth column table above by recording values of the predicted numbers
of plants with each genotype.

C. Apply a test at the 95 percent confidence level to evaluate the claim that these
data confirm linkage. The definition of the statistic

and this table are provided at the AP Biology Exam.


P 1 2 3 4 5 6 7 8
0.05 3.84 5.99 7.82 9.49 11.07 12.59 14.07 15.51
0.01 6.64 9.32 11.34 13.28 15.09 16.81 18.48 20.09
Degrees of Freedom
D. At first Bateson and Punnett did not see that these genes are located on the same
chromosome and proceed to measure the linkage distance between them, taking the first
step toward creating a gene map. Justify a selection of data and the procedure from
which data could be collected that would have provided the necessary evidence to
confirm linkage and recombination.
Solution Sample answer:
A. and B.
Phenotype Genotype(s) Observed Expected
Purple, long PpLl, PpLL, PPLl, PPLL 4,831 3,910.5
Purple, round Ppll and PPll 390 1,303.5
Red, long ppLl and ppLL 393 1,303.5
Red, round ppll 1,338 434.5
Total 6,952
C. The result is clearly larger than the critical value of 7.82 (3 classes).

Advanced Placement Biology Instructor’s Solution Manu


13 | Modern Understandings of Inheritance PAGE \* MERGEFORMAT 277275

D. They needed to do a test cross of the F1 generation with the homozygous


recessive. Then they could have determined the fraction of the offspring with
parental (PpLl and ppll) and recombinant (ppLl and Ppll) genotypes. From this they
might have made the leap that young Sturtevant made.
21 Review the observations that provided researchers with evidence in support of the
Chromosomal Theory of Inheritance.
A. Evaluate the dependence of these observations on improvements in a critical
technology during the period from 1850 to 1940. Identify this technology and describe
how this technology allowed scientists to make the connection between chromosomes
and genes. (As a hint, the name chromosome is taken from the Greek word chroma,
which means “colored” or “stained.”)
B. Mendel’s laws of inheritance are explained by the chromosomal theory. Use these
observations to justify:
 The law of segregation
 The law of independent assortment
Solution Sample answer:
A. The evidence is microscopic. All of the observations identified in the text can only
be made with the microscope:
1. During meiosis, homologous chromosome pairs migrate as discrete
structures that are independent of other chromosome pairs.
2. The sorting of chromosomes from each homologous pair into pre-gametes
appears to be random.
3. Each parent synthesizes gametes that contain only half of their chromosomal
complement.
4. Even though male and female gametes (sperm and egg) differ in size and
morphology, they have the same number of chromosomes, suggesting equal
genetic contributions from each parent.
5. The gametic chromosomes combine during fertilization to produce offspring
with the same chromosome number as their parents.
B. Points (1) and (3) explain the law of segregation. Points (2) and (1) explain
independent assortment.
22 Errors in the transmission of genetic information to future generations are essential.
Otherwise, organisms could not evolve over time. Some errors in the synthesis of new
DNA during S phase in either meiosis or mitosis are not repaired. These errors usually
involve single nucleotides. Errors that occur during prophase I of meiosis that are not
corrected can involve the exchange of sequences between homologous chromosomes
(duplications) or even nonhomologous chromosomes (translocations). Duplications
l

Advanced Placement Biology Instructor’s Solution Manu


282 13 | Modern Understandings of Inheritance

usually are retained, and the organism remains viable without a change in phenotype.
Translocations are usually lethal or significantly alter phenotype. In eukaryotes,
duplications and the shuffling of parental genes through recombination are important
sources of variation.
Construct an explanation of the role of duplication as a source of raw material for future
mutations and selection and contrast this type of variation with recombination.
Solution Sample answer: Duplication of a gene allows a second copy to undergo mutation
independently of the original gene. As the two diverge, new functions result which
provides a gradual (but not so gradual as recombination) means of change. If only
recombination was the source of variation, then pairs of genes that are very close
together would remain parental and never vary.
23 Bacteria and Archaea reproduce asexually, and genetic material is in a closed loop. In both
domains, genetic material is transferred horizontally, and polyploidy is common.
Polyploidy is common in plants and occurs in invertebrate animals but is less common in
vertebrates. In all domains, multiple copies of genes (gene duplication) are common.
Based on this information, compare and contrast the mechanisms that provide genetic
variation in the three domains: Bacteria, Archaea, and Eukarya.
Solution Sample answer: Multiple copies of the genome—like duplication—provide raw
material for the development of an alternative set of proteins. The rapidity of
reproduction and development in unicellular organisms combined with multiple
copies of the genome increases the frequency of presentation of new genotypes for
selection. Horizontal gene transfer further increases the shuffling of information—
even originating with other species.

Advanced Placement Biology Instructor’s Solution Manu


14 | DNA Structure and Function PAGE \* MERGEFORMAError! Unknown switch argument.***

14 | DNA STRUCTURE AND FUNCTION


REVIEW QUESTIONS
1 Who was the first person to isolate the material that came to be known as nucleic acids?
A Frederick Griffith
B Friedrich Miescher
C James Watson
D Oswald Avery
Solution The solution is (B). DNA was first isolated from white blood cells by Friedrich
Miescher.
2 What is bacterial transformation?
A The transformation of a bacterium occurs during replication.
B It is the transformation of a bacterium into a pathogenic form.
C Transformation of bacteria involves changes in its chromosome.
D Transformation is a process in which external DNA is taken up by a cell, thereby
changing morphology and physiology.
Solution The solution is (D). Transformation is a process in which external DNA is taken up by
a cell, thereby changing morphology and physiology.
3 What type of nucleic acid material is analyzed the most frequently in forensics cases?
A Cytoplasmic rRNA
B Mitochondrial DNA
C Nuclear chromosomal DNA
D Nuclear mRNA
Solution The solution is (C). Forensics looks at the nuclear genetic material.
4 The experiments by Hershey and Chase helped confirm that DNA was the hereditary
material on the basis of the finding of what?
A Radioactive phages were found in the pellet.
B Radioactive cells were found in the supernatant.
C Radioactive sulfur was found inside the cell.
D Radioactive phosphorus was found in the cell.
Solution The solution is (D). Radioactive phosphorous was found in the heavier particles that
settled as pellets. The heavier bacterial cells settled down and formed pellets.

********************************************************
l

284 14 | DNA Structure and Functi


o
5 If DNA of a particular species was analyzed and it was found that it contains 27 percent A,
what would be the percentage of T?
A 23%
B 27%
C 30%
D 54%
Solution The solution is (B). Because A binds to T, there is the same proportion of A and T in
each DNA molecule.
6 If the sequence of the 5′ to 3′ strand is AATGCTAC, then the complementary sequence has
which sequence?
A 3′-AATGCTAC-5′
B 3′-CATCGTAA-5′
C 3′-TTACGATG-5′
D 3′-GTAGCATT-5′
Solution The solution is (C). A binds to T and C binds to G in DNA molecules.
7 The DNA double helix does NOT have what?
A Antiparallel configuration
B Complementary base pairing
C Major and minor grooves
D Uracil
Solution The solution is (D). Uracil is a nucleotide base that is present in RNA and not in DNA.
8 What is a purine?
A A double-ring structure with a six-membered ring fused to a five-membered ring
B A single six-membered ring
C A six-membered ring
D Three phosphates covalently bonded by phosphodiester bonds
Solution The solution is (A). A double-ring structure with a six-membered ring fused to a five-
membered ring.
9 What is the name of the method developed by Fred Sanger to sequence DNA?
A Dideoxy chain termination
B Double helix determination

********************************************************
14 | DNA Structure and Function PAGE \* MERGEFORMAError! Unknown switch argument.***
C Polymerase chain reaction
D Polymer gel electrophoresis
Solution The solution is (A). The Dideoxy Chain Termination method was developed by Fred
Sanger. It is a sequencing method based on the use of chain terminators.
10 What happens when a dideoxynucleotide is added to a developing DNA strand?
A The chain extends to the end of the DNA strand.
B The DNA stand is duplicated.
C The chain is not extended any further.
D The last codon is repeated.
Solution The solution is (C). If a ddNTP is added to a growing a DNA strand, the chain is not
extended any further because the free 3′OH group needed to add another
nucleotide is not available.
11 In eukaryotes, what is DNA wrapped around?
A Histones
B Polymerase
C Single-stranded binding proteins
D Sliding clamp
Solution The solution is (A). In eukaryotes, the DNA is wrapped around proteins known as
histones to form structures called nucleosomes.
12 Which enzyme is only found in prokaryotic organisms?
A DNA gyrase
B Helicase
C Ligase
D Telomerase
Solution The solution is (A). DNA gyrase helps to maintain the supercoiled structure in
prokaryotes.
13 Where is uracil found?
A Chromosomal DNA
B Helicase
C Mitochondrial DNA
D mRNA

********************************************************
l

286 14 | DNA Structure and Functi


o
Solution The solution is (D). Uracil is a nitrogenous base found in mRNA molecules. Its
complementary base pair is adenine.
14 What prevents the further development of a DNA strand in Sanger sequencing?
A The addition of DNA reductase
B The addition of dideoxynucleotides
C The elimination of DNA polymerase
D The addition of uracil
Solution The solution is (B). If a ddNTP is added to a growing DNA strand, the chain is not
extended any further because the free 3′OH group needed to add another
nucleotide is not available. ddNTP lack the 3′OH group on the five-carbon sugar.
15 What is NOT one of the proteins involved during the formation of the replication fork?
A Helicase
B Ligase
C Origin of replication
D Single-stranded binding proteins
Solution The solution is (C). The origin of replication is the point at which the DNA unwinds.
16 In which direction does DNA replication take place?
A 5′ to 3′
B 3′ to 5′
C 5′
D 3′
Solution The solution is (A). DNA polymerase adds nucleotides from 5′ to 3′ direction.
17 Meselson and Stahl’s experiments proved that DNA replicates by which mode?
A Conservative
B Converse
C Dispersive
D Semiconservative
Solution The solution is (D). The semiconservative mode of replication suggested that each of
the two parental DNA strands act as a template for new DNA to be synthesized.
After replication, each double-stranded DNA includes one parental or “old” strand
and one “new” strand.

********************************************************
14 | DNA Structure and Function PAGE \* MERGEFORMAError! Unknown switch argument.***

********************************************************
l

288 14 | DNA Structure and Functi


o
18 Which set of results was found in Meselson and Stahl’s experiments?
A The original chromosome was kept intact and a duplicate was made.
B The original chromosome was split and half went to each duplicate.
C The original chromosome was mixed with new material and each duplicate strand
contained both old and new.
D The original chromosome was used as a template for two new chromosomes and
discarded.
Solution The solution is (B). The semiconservative method suggested that each of the two
parental DNA strands act as a template for new DNA to be synthesized. After
replication, each double-stranded DNA includes one parental or “old” strand and
one “new” strand. It was found in Meselson and Stahl’s experiment.
19 Which enzyme initiates the splitting of the double DNA strand during replication?
A DNA gyrase
B Helicase
C Ligase
D Telomerase
Solution The solution is (B). Helicase opens the DNA helix by breaking hydrogen bonds
between the nitrogenous bases ahead of the replication fork.
20 Which enzyme is most directly responsible for the main process of producing a new DNA
strand?
A DNA pol I
B DNA pol II
C DNA pol III
D DNA pol I, DNA pol II, and DNA pol III
Solution The solution is (C). DNA polymerase III is the main enzyme in DNA replication that
adds nucleotides in 5’ to 3’ direction.
21 Which portion of a chromosome contains Okazaki fragments?
A Helicase
B Lagging strand
C Leading strand
D Primer

********************************************************
14 | DNA Structure and Function PAGE \* MERGEFORMAError! Unknown switch argument.***
Solution The solution is (B). The replication of the strand that occurs in a direction away from
the replication fork is known as lagging strand and contains Okazaki fragments.

********************************************************
l

290 14 | DNA Structure and Functi


o
22 What does the enzyme primase synthesize?
A DNA primer
B Okazaki fragments
C Phosphodiester linkage
D RNA primer
Solution The solution is (D). Primase synthesizes RNA primers to initiate synthesis by DNA
polymerase, which can add nucleotides only in the 5′ to 3′ direction.
23 The ends of the linear chromosomes are maintained by what?
A DNA polymerase
B Helicase
C Primase
D Telomerase
Solution The solution is (D). Telomerase maintains the linear end of the chromosomes. It
consists of a catalytic part and a built-in RNA template.
24 What is the difference in the rate of replication of nucleotides between prokaryotes and
eukaryotes?
A Eukaryotes are 50 times slower.
B Eukaryotes are 20 times faster.
C Prokaryotes are 100 times slower.
D Prokaryotes are 10 times faster.
Solution The solution is (D). The advantage in prokaryotes is that RNA and protein synthesis
occurs much more quickly than the eukaryotes. It is 10 times faster than eukaryotes.
25 What are Autonomously Replicating Sequences (ARS)?
A Areas of prokaryotic chromosomes that initiate copying
B Portions of prokaryotic chromosomes that can be transferred from one organism to
another
C Areas of eukaryotic chromosomes that are equivalent to the origin of replication in
E. coli
D Portions of eukaryotic chromosomes that replicate independent of the parent
chromosome
Solution The solution is (C). Areas of eukaryotic chromosomes that are equivalent to the
origin of replication in E. coli.

********************************************************
14 | DNA Structure and Function PAGE \* MERGEFORMAError! Unknown switch argument.***
26 What type of body cell does NOT exhibit telomerase activity?
A Adult stem cells
B Embryonic cells
C Germ cells
D Liver cells
Solution The solution is (D). Liver cells do not exhibit telomerase activity. Adult, embryonic,
and germ cells do.
27 During proofreading, which enzyme reads the DNA?
A DNA polymerase
B Helicase
C Topoisomerase
D Primase
Solution The solution is (A). DNA polymerase edits DNA by proofreading every newly
added base.
28 If a prokaryotic cell is replicating nucleotides at a rate of 100 per second, how fast would a
eukaryotic cell be replicating nucleotides?
A 1,000 per second
B 100 per second
C 10 per second
D 1 per second
Solution The solution is (C). Rate of replication in prokaryotes is 10 times than eukaryotes. If
replication in prokaryotes is 100 per second, then in eukaryotes it will be 10 per
second.
29 Which type of point mutation would have no effect on gene expression?
A Frameshift
B Missense
C Nonsense
D Silent
Solution The solution is (D). Silent mutation is the one in which a nucleotide is substituted,
but there is no effect on the protein sequence or gene expression.

********************************************************
l

292 14 | DNA Structure and Functi


o
30 Which type of point mutation would result in the substitution of a stop codon for an
amino acid?
A Frameshift
B Missense
C Nonsense
D Silent
Solution The solution is (C). Nonsense mutation introduces a stop codon in place of an
amino acid.
31 A woman has developed skin cancer and she is pregnant. She is worried that her child will
be born with the cancer she has while carrying the baby.
Should she be worried?
A Yes, the cancer can spread to the baby.
B No, the mutations causing the cancer are in somatic cells, not reproductive germ cells.
C Yes, the mutations can be passed on to the child through the placenta.
D No, UV light only affects adult, somatic cells.
Solution The solution is (B). The mutation causing the cancer does not occur in the germ cells
of the woman, but in the somatic cells. Therefore, it will not affect the baby.
32 What is the initial mechanism for repairing nucleotide errors in DNA?
A DNA polymerase proofreading
B Mismatch repair
C Nucleotide excision repair
D Thymine dimers
Solution The solution is (A). DNA polymerase is an efficient enzyme, but can make mistakes
while adding nucleotides during replication. It edits the DNA by proofreading every
newly added base.
33 Nucleotide excision repair often is employed when UV exposure causes the formation of
what?
A Phosphodiester bonds
B Purine conjugates
C Pyrimidine dimers
D Tetrad disassembly

********************************************************
14 | DNA Structure and Function PAGE \* MERGEFORMAError! Unknown switch argument.***
Solution The solution is (C). Thymine and cytosine are pyrimidine dimers. On long exposure of
UV rays, thymine dimers are formed, which puts people at higher risk of developing
skin cancer. Nucleotide excision repair is a DNA repair mechanism that excises the
thymine dimers in normal individuals.

CRITICAL THINKING QUESTIONS


34 Explain Griffith’s transformation experiments. What did he conclude from them?
A Two strains of S. pneumoniae were used for the experiment. Griffith injected a mouse
with heat-inactivated S strain (pathogenic) and R strain (nonpathogenic). The mouse
died and S strain was recovered from the dead mouse. He concluded that external
DNA is taken up by a cell that changed morphology and physiology.
B Two strains of Vibrio cholerae were used for the experiment. Griffith injected a mouse
with heat-inactivated S strain (pathogenic) and R strain (nonpathogenic). The mouse
died and S strain was recovered from the dead mouse. He concluded that external
DNA is taken up by a cell that changed morphology and physiology.
C Two strains of S. pneumoniae were used for the experiment. Griffith injected a mouse
with heat-inactivated S strain (pathogenic) and R strain (nonpathogenic). The mouse
died and R strain was recovered from the dead mouse. He concluded that external
DNA is taken up by a cell that changed morphology and physiology.
D Two strains of S. pneumoniae were used for the experiment. Griffith injected a mouse
with heat-inactivated S strain (pathogenic) and R strain (nonpathogenic). The mouse
died and S strain was recovered from the dead mouse. He concluded that mutation
occurred in the DNA of the cell that changed morphology and physiology.
Solution The solution is (A). Two strains of S. pneumoniae were used in Griffith’s
transformation experiments. The R strain is nonpathogenic. The S strain is
pathogenic and causes death. When Griffith injected a mouse with the heat-
inactivated S strain and a live R strain, the mouse died. The S strain was recovered
from the dead mouse. Thus, Griffith concluded that something had passed from the
heat-killed S strain to the R strain, transforming the R strain into S strain in the
process.
35 Which answer best explains why radioactive sulfur and phosphorus were used to label
bacteriophages in the Hershey and Chase experiments?
A Protein was labeled with radioactive sulfur and DNA was labeled with radioactive
phosphorus. Phosphorus is found in DNA, so it will be tagged by radioactive
phosphorus.
B Protein was labeled with radioactive phosphorus and DNA was labeled with
radioactive sulfur. Phosphorus is found in DNA, so it will be tagged by radioactive
phosphorus.

********************************************************
l

294 14 | DNA Structure and Functi


o
C Protein was labeled with radioactive sulfur and DNA was labeled with radioactive
phosphorus. Phosphorus is found in DNA, so DNA will be tagged by radioactive sulfur.
D Protein was labeled with radioactive phosphorus and DNA was labeled with
radioactive sulfur. Phosphorus is found in DNA, so DNA will be tagged by radioactive
sulfur.
Solution The solution is (A). Hershey and Chase labeled one batch of phage with radioactive
sulfur, 35S, to label the protein coat. Another batch of phage was labeled with
radioactive phosphorus, 32P. Because phosphorus is found in DNA, but not protein,
the DNA and not the protein would be tagged with radioactive phosphorus.
36 How can Chargaff’s rules be used to identify different species?
A The amount of adenine, thymine, guanine, and cytosine varies from species to species
and is not found in equal quantities. They do not vary between individuals of the same
species and can be used to identify different species.
B The amount of adenine, thymine, guanine, and cytosine varies from species to species
and is found in equal quantities. They do not vary between individuals of the same
species and can be used to identify different species.
C The amount of adenine and thymine is equal to guanine and cytosine and is found in
equal quantities. They do not vary between individuals of the same species and can be
used to identify different species.
D The amount of adenine, thymine, guanine, and cytosine varies from species to species
and is not found in equal quantities. They vary between individuals of the same
species and can be used to identify different species.
Solution The solution is (A). The content of DNA is different indifferent species and the
amounts of adenine, thymine, guanine, and cytosine are found in different
quantities. Therefore, the amounts of adenine, thymine, guanine, and cytosine are
consistent for a species and can be used to identify that species.
37 In the Avery, Macleod, and McCarty experiments, what conclusion would the scientists
have drawn if the use of proteases prevented the transformation of R strain bacteria?
Solution The conclusion would be that proteins are the heritable material in cells instead of
nucleic acids.
38 Describe the structure and complementary base pairing of DNA.
A DNA is made up of two strands that are twisted around each other to form a helix.
Adenine pairs up with thymine and cytosine pairs with guanine. The two strands are
antiparallel in nature; that is, the 3′ end of one strand faces the 5′ end of the other
strand. Sugar, phosphate, and nitrogenous bases contribute to the DNA structure.

********************************************************
14 | DNA Structure and Function PAGE \* MERGEFORMAError! Unknown switch argument.***
B DNA is made up of two strands that are twisted around each other to form a helix.
Adenine pairs up with cytosine and thymine pairs with guanine. The two strands are
antiparallel in nature; that is, the 3′ end of one strand faces the 5′ end of the other
strand. Sugar, phosphate, and nitrogenous bases contribute to the DNA structure.
C DNA is made up of two strands that are twisted around each other to form a helix.
Adenine pairs up with thymine and cytosine pairs with guanine. The two strands are
parallel in nature; that is, the 3′ end of one strand faces the 3′ end of the other strand.
Sugar, phosphate, and nitrogenous bases contribute to the DNA structure.
D DNA is made up of two strands that are twisted around each other to form a helix.
Adenine pairs up with thymine and cytosine pairs with guanine. The two strands are
antiparallel in nature; that is, the 3′ end of one strand faces the 5′ end of the other
strand. Only sugar contributes to the DNA structure.
Solution The solution is (A). DNA is made up of two strands that are twisted around each
other to form a right-handed helix. Base pairing takes place between a purine and
pyrimidine; namely, A pairs with T and G pairs with C. Adenine and thymine are
complementary base pairs, and cytosine and guanine are also complementary base
pairs. The base pairs are stabilized by hydrogen bonds; adenine and thymine form
two hydrogen bonds and cytosine and guanine form three hydrogen bonds. The two
strands are anti-parallel in nature; that is, the 3′ end of one strand faces the 5′ end
of the other strand. The sugar and phosphate of the nucleotides form the backbone
of the structure, whereas the nitrogenous bases are stacked inside.
39 Which answer provides a brief summary of the Sanger sequencing method?
A Frederick Sanger’s sequencing is a chain termination method that is used to generate
DNA fragments that terminate at different points using dye-labeled
dideoxynucleotides. DNA is separated by electrophoresis on the basis of size. The DNA
sequence can be read out on an electropherogram generated by a laser scanner.
B Frederick Sanger’s sequencing is a chain elongation method that is used to generate
DNA fragments that elongate at different points using dye-labeled dideoxynucleotides.
DNA is separated by electrophoresis on the basis of size. The DNA sequence can be
read out on an electropherogram generated by a laser scanner.
C Frederick Sanger’s sequencing is a chain termination method that is used to generate
DNA fragments that terminate at different points using dye-labeled
dideoxynucleotides. DNA is joined together by electrophoresis on the basis of size. The
DNA sequence can be read out on an electropherogram generated by a laser scanner.
D Frederick Sanger’s sequencing is a chain termination method that is used to generate
DNA fragments that terminate at different points using dye-labeled
dideoxynucleotides. DNA is separated by electrophoresis on the basis of size. The DNA
sequence can be read out on an electropherogram generated by a magnetic scanner.

********************************************************
l

296 14 | DNA Structure and Functi


o
Solution The solution is (A). In Frederick Sanger's dideoxy chain termination method, dye-
labeled dideoxynucleotides are used to generate DNA fragments that terminate at
different points. The DNA is separated by capillary electrophoresis on the basis of
size, and from the order of fragments formed, the DNA sequence can be read. The
DNA sequence readout is shown on an electropherogram that is generated by a
laser scanner.
40 Compare and contrast the similarities and differences between eukaryotic and prokaryotic
DNA.
A Eukaryotes have a single, circular chromosome, while prokaryotes have multiple,
linear chromosomes. Prokaryotes pack their chromosomes by super coiling, managed
by DNA gyrase. Eukaryote chromosomes are wrapped around histone proteins that
create heterochromatin and euchromatin, which is not present in prokaryotes.
B Prokaryotes have a single, circular chromosome, while eukaryotes have multiple,
linear chromosomes. Prokaryotes pack their chromosomes by super coiling, managed
by DNA gyrase. Eukaryote chromosomes are wrapped around histone proteins that
create heterochromatin and euchromatin, which is not present in prokaryotes.
C Prokaryotes have a single, circular chromosome, while eukaryotes have multiple,
linear chromosomes. Eukaryotes pack their chromosomes by super coiling, managed
by DNA gyrase. Prokaryote chromosomes are wrapped around histone proteins that
create heterochromatin and euchromatin, which is not present in eukaryotes.
D Prokaryotes have a single, circular chromosome, while eukaryotes have multiple,
linear chromosomes. Prokaryotes pack their chromosomes by super coiling, managed
by DNA gyrase. Eukaryote chromosomes are wrapped around histone proteins that
create heterochromatin and euchromatin, which is present in prokaryotes.
Solution The solution is (B). Prokaryotes have a single, circular chromosome, while
eukaryotes have multiple, linear chromosomes. Prokaryotes pack their
chromosomes into the cell using super coiling, managed DNA gyrase. Eukaryote
chromosomes are wrapped around histone proteins that create heterochromatin
and euchromatin that is not found in prokaryotic cells.
41 DNA replication is bidirectional and discontinuous. How can you explain your
understanding of those concepts?
A DNA polymerase reads the template strand in the 3′ to 5′ direction and adds
nucleotides only in the 5′ to 3' direction. The leading strand is synthesized in the
direction of the replication fork. Replication on the lagging strand occurs in the
direction away from the replication fork in short stretches of DNA called Okazaki
fragments.
B DNA polymerase reads the template strand in the 5′ to 3′ direction and adds
nucleotides only in the 5′ to 3′ direction. The leading strand is synthesized in the

********************************************************
14 | DNA Structure and Function PAGE \* MERGEFORMAError! Unknown switch argument.***
direction of the replication fork. Replication on the lagging strand occurs in the
direction away from the replication fork in short stretches of DNA called Okazaki
fragments.
C DNA polymerase reads the template strand in the 3′ to 5′ direction and adds
nucleotides only in the 5′ to 3′ direction. The leading strand is synthesized in the
direction away from the replication fork. Replication on the lagging strand occurs in
the direction of the replication fork in short stretches of DNA called Okazaki
fragments.
D DNA polymerase reads the template strand in the 5′ to 3′ direction and adds
nucleotides only in the 3′ to 5′ direction. The leading strand is synthesized in the
direction of the replication fork. Replication on the lagging strand occurs in the
direction away from the replication fork in long stretches of DNA called Okazaki
fragments.
Solution The solution is (A). DNA polymerase can add nucleotides only in the 5’ to 3’
direction. DNA polymerase recognizes the 3’OH end as its landing site; thus,
polymerase “reads” the template strand in the 3’ to 5’ direction and builds the new
DNA complementary DNA polymer in the 5’ to 3’ direction. One strand—called the
leading strand—is synthesized continuously in the direction of the replication fork
(the direction in which helicase is separating the two strands), with polymerase
adding new nucleotides one-by-one. However, replication of the other strand—
called the lagging strand—occurs in a direction away from the replication fork, in
short stretches of DNA known as Okazaki fragments.
42 How did the scientific community learn that DNA replication takes place in a
semiconservative fashion?
A Meselson and Stahl experimented with E. coli. DNA grown in 15N was heavier than
DNA grown in 14N. When DNA in 15N was switched to 14N media, DNA sedimented
halfway between the 15N and 14N levels after one round of cell division, indicating 50
percent presence of 14N. This supports the semiconservative replication model.
B Meselson and Stahl experimented with S. pneumonia. DNA grown in 15N was heavier
than DNA grown in 14N. When DNA in 15N was switched to 14N media, DNA sedimented
halfway between the 15N and 14N levels after one round of cell division, indicating 50
percent presence of 14N. This supports the semiconservative replication model.
C Meselson and Stahl experimented with E. coli. DNA grown in 14N was heavier than
DNA grown in 15N. When DNA in 15N was switched to 14N media, DNA sedimented
halfway between the 15N and 14N levels after one round of cell division, indicating 50
percent presence of 14N. This supports the semiconservative replication model.
D Meselson and Stahl experimented with S. pneumonia. DNA grown in 15N was heavier
than DNA grown in 14N. When DNA in 15N was switched to 14N media, DNA sedimented
halfway between the 15N and 14N levels after one round of cell division, indicating
complete presence of 14N. This supports the semiconservative replication model.

********************************************************
l

298 14 | DNA Structure and Functi


o
Solution The solution is (A). Meselson and Stahl experimented with E. coli grown first in heavy
nitrogen (15N) then in 14N. DNA grown in 15N is heavier than DNA grown in 14N, and
sediments to a lower level in cesium chloride solution in an ultracentrifuge. When
DNA grown in 15N is switched to media containing 14N, after one round of cell division
the DNA sediments halfway between the 15N and 14N levels, indicating that it now
contains 50 percent 14N. In subsequent cell divisions, an increasing amount of DNA
contains 14N only. These data support the semiconservative replication model.
43 Why is half of DNA replicated in a discontinuous fashion?
A Replication of the lagging strand occurs in the direction away from the replication fork
in short stretches of DNA, since access to the DNA is always from the 5′ end. This
results in pieces of DNA being replicated in a discontinuous fashion.
B Replication of the leading strand occurs in the direction away from the replication fork
in short stretches of DNA, since access to the DNA is always from the 5′ end. This
results in pieces of DNA being replicated in a discontinuous fashion.
C Replication of the lagging strand occurs in the direction of the replication fork in short
stretches of DNA, since access to the DNA is always from the 5′ end. This results in
pieces of DNA being replicated in a discontinuous fashion.
D Replication of the lagging strand occurs in the direction away from the replication fork
in short stretches of DNA, since access to the DNA is always from the ′' end. This
results in pieces of DNA being replicated in a discontinuous fashion.
Solution The solution is (A). Since access to the DNA strand is always from the 5′ end, the
replication of one strand, called the lagging strand, occurs in a direction away from
the replication fork, in short stretches of DNA. This results in pieces of DNA being
replicated in a discontinuous fashion. These pieces will be joined into a single strand
of DNA.
44 Explain the events taking place at the replication fork. If the gene for helicase is mutated,
what part of replication will be affected?
A Helicase separates the DNA strands at the origin of replication. Topoisomerase breaks
and reforms DNA’s phosphate backbone ahead of the replication fork, thereby
relieving the pressure. Single-stranded binding proteins prevent reforming of DNA.
Primase synthesizes RNA primer which is used by DNA polymerase to form a daughter
strand. If helicase is mutated, the DNA strands will not be separated at the beginning
of replication.
B Helicase joins the DNA strands together at the origin of replication. Topoisomerase
breaks and reforms DNA’s phosphate backbone after the replication fork, thereby
relieving the pressure. Single-stranded binding proteins prevent reforming of DNA.
Primase synthesizes RNA primer which is used by DNA polymerase to form a daughter

********************************************************
14 | DNA Structure and Function PAGE \* MERGEFORMAError! Unknown switch argument.***
strand. If helicase is mutated, the DNA strands will not be joined together at the
beginning of replication.

********************************************************
l

300 14 | DNA Structure and Functi


o
C Helicase separates the DNA strands at the origin of replication. Topoisomerase breaks
and reforms DNA’s sugar backbone ahead of the replication fork, thereby increasing
the pressure. Single-stranded binding proteins prevent reforming of DNA. Primase
synthesizes DNA primer which is used by DNA polymerase to form a daughter strand.
If helicase is mutated, the DNA strands will be separated at the beginning of
replication.
D Helicase separates the DNA strands at the origin of replication. Topoisomerase breaks
and reforms DNA’s sugar backbone ahead of the replication fork, thereby relieving the
pressure. Single-stranded binding proteins prevent reforming of DNA. Primase
synthesizes DNA primer which is used by RNA polymerase to form a parent strand. If
helicase is mutated, the DNA strands will be separated at the beginning of replication.
Solution The solution is (A). A replication fork is formed when helicase separates the DNA
strands at the origin of replication. The DNA tends to become more highly coiled
ahead of the replication fork. Topoisomerase breaks and reforms DNA’s phosphate
backbone ahead of the replication fork, thereby relieving the pressure that results
from this supercoiling. Single-strand binding proteins bind to the single-stranded
DNA to prevent the helix from re-forming. Primase synthesizes an RNA primer. DNA
polymerase III uses this primer to synthesize the daughter DNA strand.
If helicase is mutated and cannot function, the DNA strands will not be separated at
the beginning of replication.
45 What are Okazaki fragments, and how they are formed?
A Okazaki fragments are short stretches of DNA on the lagging strand, which is
synthesized in the direction away from the replication fork.
B Okazaki fragments are long stretches of DNA on the lagging strand, which is
synthesized in the direction of the replication fork.
C Okazaki fragments are long stretches of DNA on the leading strand, which is
synthesized in the direction away from the replication fork.
D Okazaki fragments are short stretches of DNA on the leading strand, which is
synthesized in the direction of the replication fork.
Solution The solution is (A). Replication of the lagging strand in DNA replication occurs in a
direction away from the replication fork in short stretches of DNA known as Okazaki
fragments.
46 Compare and contrast the roles of DNA polymerase I and DNA ligase in DNA replication.
A DNA polymerase I removes the RNA primers from the developing copy of DNA. DNA
ligase seals the ends of the new segment, especially the Okazaki fragments.
B DNA polymerase I adds the RNA primers to the already developing copy of DNA. DNA
ligase separates the ends of the new segment, especially the Okazaki fragments.

********************************************************
14 | DNA Structure and Function PAGE \* MERGEFORMAError! Unknown switch argument.***
C DNA polymerase I seals the ends of the new segment, especially the Okazaki
fragments. DNA ligase removes the RNA primers from the developing copy of DNA.
D DNA polymerase I removes the enzyme primase from the developing copy of DNA.
DNA ligase seals the ends of the old segment, especially the Okazaki fragments.
Solution The solution is (A). DNA polymerase I has exonuclease activity that removes RNA
primers from the developing copy while DNA ligase seals the ends of the new
segment, especially the Okazaki fragments.
47 If the rate of replication in a particular prokaryote is 900 nucleotides per second, how long
would it take to make two copies of a 1.2 million base pair genome?
A 22.2 min
B 44.4 min
C 45.4 min
D 54.4 min
Solution The solution is (B). The rate of replication equals 900 nucleotides per second.
For 1.2 million base pairs, the time taken for replication would be
(1,333.3 s). To make two copies, the time
taken will be
48 How do the linear chromosomes in eukaryotes ensure that their ends are replicated
completely?
A The ends of the linear chromosomes are maintained by the activity of the telomerase
enzyme.
B The ends of the linear chromosomes are maintained by the formation of a
replication fork.
C The ends of the linear chromosomes are maintained by the continuous joining of
Okazaki fragments.
D The ends of the linear chromosomes are maintained by the action of the polymerase
enzyme.
Solution The solution is (A). Telomerase enzyme replicates the ends of chromosomes by
attaching to the 3′ chromosomal end of the DNA strand. When the 3′ end is
elongated, then DNA polymerase adds complementary nucleotides to the end of
chromosomes.

********************************************************
l

302 14 | DNA Structure and Functi


o
49 What is the best way to compare and contrast prokaryotic and eukaryotic DNA
replication?
A A prokaryotic organism’s rate of replication is 10 times faster than that of eukaryotes.
Prokaryotes have a single origin of replication and use five types of polymerases, while
eukaryotes have multiple sites of origin and use 14 polymerases. Telomerase is absent
in prokaryotes. DNA pol I is the primer remover in prokaryotes, while in eukaryotes it
is RNase H. DNA pol III performs strand elongation in prokaryotes and pol δ and pol ε
do the same in eukaryotes.
B A prokaryotic organism’s rate of replication is 10 times slower than that of eukaryotes.
Prokaryotes have a single origin of replication and use five types of polymerases, while
eukaryotes have multiple sites of origin and use 14 polymerases. Telomerase is absent
in eukaryotes. DNA pol I is the primer remover in prokaryotes, while in eukaryotes it is
RNase H. DNA pol III performs strand elongation in prokaryotes and pol δ and pol ε do
the same in eukaryotes.
C A prokaryotic organism’s rate of replication is 10 times faster than that of eukaryotes.
Prokaryotes have five origins of replication and use a single type of polymerase, while
eukaryotes have a single site of origin and use 14 polymerases. Telomerase is absent
in prokaryotes. DNA pol I is the primer remover in prokaryotes, while in eukaryotes it
is RNase H. DNA pol III performs strand elongation in prokaryotes and pol δ and pol ε
do the same in eukaryotes.
D A prokaryotic organism’s rate of replication is 10 times slower than that of eukaryotes.
Prokaryotes have a single origin of replication and use five types of polymerases, while
eukaryotes have multiple sites of origin and use 14 polymerases. Telomerase is absent
in prokaryotes. DNA pol I is the primer remover in eukaryotes, while in prokaryotes it
is RNase H. DNA pol III performs strand elongation in prokaryotes and pol δ and pol ε
do the same in eukaryotes.
Solution The solution is (A). Prokaryotic organisms have a single origin of replication, while
eukaryotic ones have multiple sites. The rate of replication of prokaryotic cells is
approximately 10 times that of eukaryotes. There are five types of DNA polymerases
used by prokaryotes and 14 in eukaryotes. Telomerase functions in eukaryotic cells,
but not prokaryotic one. The RNA primer remover in prokaryotic organisms is DNA
pol I, but RNase H in eukaryotic cells. Strand elongation is performed by DNA pol III
in prokaryotes and by Pol δ, pol ε in eukaryotic organisms.

********************************************************
14 | DNA Structure and Function PAGE \* MERGEFORMAError! Unknown switch argument.***
50 What would be the consequence of a mutation in a mismatch repair enzyme? How would
this affect the function of a gene?
A Mismatch repair corrects the errors after the replication is completed by excising the
incorrectly added nucleotide and adding the correct base. Any mutation in a mismatch
repair enzyme would lead to more permanent damage.
B Mismatch repair corrects the errors during the replication by excising the incorrectly
added nucleotide and adding the correct base. Any mutation in the mismatch repair
enzyme would lead to more permanent damage.
C Mismatch repair corrects the errors after the replication is completed by excising the
added nucleotides and adding more bases. Any mutation in the mismatch repair
enzyme would lead to more permanent damage.
D Mismatch repair corrects the errors after the replication is completed by excising the
incorrectly added nucleotide and adding the correct base. Any mutation in the
mismatch repair enzyme would lead to more temporary damage.
Solution The solution is (A). Some errors are not corrected during replication, but instead are
corrected after replication is completed; this type of repair is known as mismatch
repair. The enzymes recognize the incorrectly added nucleotide and excise it; this
then is replaced by the correct base. If this remains uncorrected, it may lead to more
permanent damage.
51 A mutation has occurred in the DNA and in the mRNA for a gene. Which one would have a
more significant effect on gene expression? Why?
A Both will result in the production of defective proteins. The DNA mutation, if not
corrected, is permanent, while the mRNA mutation will affect only proteins made
from that mRNA strand. Production of defective protein ceases when the mRNA
strand deteriorates.
B Both will result in the production of defective proteins. The DNA mutation, if not
corrected, is permanent, while the mRNA mutation will not affect proteins made from
that mRNA strand. Production of defective protein continues when the mRNA strand
deteriorates.
C Only DNA will result in the production of defective proteins. The DNA mutation, if not
corrected, is permanent. Production of defective protein ceases when the DNA strand
deteriorates.
D Only mRNA will result in the production of defective proteins. The mRNA mutation will
affect only proteins made from that mRNA strand. Production of defective protein
ceases when the mRNA strand deteriorates.
Solution The solution is (A). Both will result in a defective protein produced from the gene
information. The DNA mutation, if not corrected, is permanent; the mRNA mutation
will affect only proteins made from that RNA strand. When the mRNA strand
deteriorates, the production of the defective protein ceases.

********************************************************
l

304 14 | DNA Structure and Functi


o
52 What are the effects of point mutations on a DNA strand?
A Mutations can cause a single change in an amino acid. A nonsense mutation can stop
the replication or reading of that strand. Insertion or deletion mutations can cause a
frame shift. This can result in nonfunctional proteins.
B Mutations can cause a single change in amino acid. A missense mutation can stop the
replication or reading of that strand. Insertion or deletion mutations can cause a frame
shift. This can result in nonfunctional proteins.
C Mutations can cause a single change in amino acid. A nonsense mutation can stop the
replication or reading of that strand. Substitution mutations can cause a frame shift.
This can result in nonfunctional proteins.
D Mutations can cause a single change in amino acid. A nonsense mutation can stop the
replication or reading of that strand. Insertion or deletion mutations can cause a frame
shift. This can result in functional proteins.
Solution The solution is (A). If one base is replaced by another base, but the coding for an
amino acid is not changed, there is no effect on the DNA strand and a silent
mutation has occurred. A missense mutation happens when a point mutation causes
a change in a single amino acid. A nonsense mutation causes a stop message to be
read, and the replication or reading of that strand is stopped at that point. Insertion
or deletion mutations cause a frame shift from that point on and a non-functional
protein will result.
53 What is the significance of mutations in tRNA and rRNA?
A Mutations in tRNA and rRNA would lead to the production of defective proteins or no
protein production.
B Mutations in tRNA and rRNA would lead to changes in the semiconservative mode of
replication of DNA.
C Mutations in tRNA and rRNA would lead to production of a DNA strand with a mutated
single strand and normal other strand.
D Mutations in tRNA and rRNA would lead to skin cancer in patients of Xeroderma
pigmentosa.
Solution The solution is (A). A mutation in a single type of tRNA will affect the transfer of one
amino acid. The result of this will be a decrease in proteins that require this amino
acid, as it will not be brought to the assembly points of proteins in adequate
amounts. A mutation in rRNA will affect the assembly of all proteins and lead to
significant deficiencies in these molecules.

********************************************************
14 | DNA Structure and Function PAGE \* MERGEFORMAError! Unknown switch argument.***

TEST PREP FOR AP® COURSES


54 What would Chase and Hershey have concluded if the supernatant contained radioactive
labeled-phosphorus?
A DNA was the primary source of heritable information.
B RNA was the primary source of heritable information.
C Protein was the primary source of heritable information.
D Phages were the primary source of heritable information.
Solution The solution is (C). Protein would have been identified as the primary source of
heritable information.
55 Which piece of evidence supports that the material Miescher discovered was DNA?
A The precipitate contained sulfur.
B The precipitate contained oxygen.
C The precipitate contained phosphorus.
D The precipitate contained protein.
Solution The solution is (C). The precipitate contained phosphorus, which is abundant in DNA.
56 How are forensic scientists able to use DNA analysis to identify individuals?
A Comparison of DNA from a known source or individual with analysis of the sequence
of an unknown sample of DNA allows scientists to find out if both of them are similar
or not.
B DNA from the unknown sample is sequenced and analyzed. The result of the analysis
then is matched with any random population. The matching individual then helps in
forensics.
C Comparison of DNA from a known source or individual with analysis of the sequence
of bases in strands of an unknown sample of RNA allows scientists to find out if both
of them are similar or not.
D Comparison of DNA from a known source or individual with analysis of the sugars and
phosphates in strands of an unknown sample of DNA allows scientists to find out if
both of them are similar or not.
Solution The solution is (A). Analysis of the sequence of bases in strands of DNA and their
comparison to DNA from a known source or individual allows scientists to state that
they are the same, or very similar, thus identifying the unknown source of the
sample of DNA.

********************************************************
l

306 14 | DNA Structure and Functi


o
57 What were the contributions of Francis Crick, James Watson, and Rosalind Franklin to the
discovery of the structure of DNA?
A Franklin used X-ray diffraction methods to demonstrate the helical nature of DNA,
while Watson and Crick formulated the double-stranded structural model of DNA.
B Franklin, Watson, and Crick first employed the technique of X-ray diffraction to
understand the storage of DNA. Since it did not work out, Watson and Crick then ran
experiments to ascertain the DNA structure.
C Watson and Crick used X-ray diffraction methods to demonstrate the helical nature of
DNA, while Franklin formulated the double-stranded structural model of DNA.
D Watson and Crick used X-ray diffraction methods to demonstrate the helical nature of
DNA, while Franklin formulated the double-stranded structural model of DNA.
Solution The solution is (A). Franklin was using X-ray diffraction methods to understand the
structure of DNA. Watson and Crick were able to piece together the puzzle of the
DNA molecule on the basis of Franklin's data.
58 What do RNA and DNA have in common?
A Both contain four different nucleotides.
B Both are usually double-stranded molecules.
C Both contain adenine and uracil.
D Both contain ribose.
Solution The solution is (A). Both contain four different nucleotides.
59 What would be a good application of plasmid transformation?
A To make copies of DNA
B To isolate a change in a single nucleotide
C To separate DNA fragments
D To sequence DNA
Solution The solution is (C). DNA fragments move through gel based on their negative charge.
60 How do the components of DNA fit together?
A DNA is composed of nucleotides, consisting of a five-carbon sugar, a phosphate, and a
nitrogenous base. DNA is a double-helical structure in which complementary base
pairing occurs. Adenine pairs with thymine and guanine pairs with cytosine. Adenine
and thymine form two hydrogen bonds and cytosine and guanine form three
hydrogen bonds. The two individual strands of DNA are held together by covalent
bonds between the phosphate of one nucleotide and sugar of the next. The two
strands run antiparallel to each other.

********************************************************
14 | DNA Structure and Function PAGE \* MERGEFORMAError! Unknown switch argument.***
B DNA is composed of nucleotides, consisting of a five-carbon sugar, a phosphate, and a
nitrogenous base. DNA is a double-helical structure in which complementary base
pairing occurs. Adenine pairs with cytosine and guanine pairs with thymine. Adenine
and cytosine form two hydrogen bonds and guanine and thymine form three
hydrogen bonds. The two individual strands of DNA are held together by covalent
bonds between the phosphate of one nucleotide and sugar of the next. The two
strands run antiparallel to each other.
C DNA is composed of nucleotides, consisting of a five-carbon sugar, a phosphate, and a
nitrogenous base. DNA is a double-helical structure in which complementary base
pairing occurs. Adenine pairs with cytosine and guanine pairs with thymine. Adenine
and cytosine form three hydrogen bonds and guanine and thymine form two
hydrogen bonds. The two individual strands of DNA are held together by covalent
bonds between the phosphate of one nucleotide and sugar of the next. The two
strands run antiparallel to each other.
D DNA is composed of nucleotides, consisting of a five-carbon sugar, a phosphate, and a
nitrogenous base. DNA is a double-helical structure in which complementary base
pairing occurs. Adenine pairs with cytosine and guanine pairs with thymine. Adenine
and cytosine form three hydrogen bonds and guanine and thymine form two
hydrogen bonds. The two individual strands of DNA are held together by covalent
bonds between the phosphate of one nucleotide and sugar of the next. The two
strands run parallel to each other.
Solution The solution is (A). DNA is made up of two strands that are twisted around each
other to form a right-handed helix. Base pairing takes place between a purine and
pyrimidine; namely, A pairs with T and G pairs with C. Adenine and thymine are
complementary base pairs, and cytosine and guanine are also complementary base
pairs. The base pairs are stabilized by hydrogen bonds; adenine and thymine form
two hydrogen bonds, and cytosine and guanine form three hydrogen bonds. The two
strands are anti-parallel in nature; that is, the 3′ end of one strand faces the 5′ end
of the other strand. The sugar and phosphate of the nucleotides form the backbone
of the structure, whereas the nitrogenous bases are stacked inside.
61 What is the best way to describe the Sanger DNA sequencing method used for the human
genome sequencing project?
A A DNA sample is denatured by heating and then put into four tubes. A primer, DNA
polymerase, and all four nucleotides are added. Limited quantities of one of the four
dideoxynucleotides (ddNTPs) are added to each tube respectively. Each one of them
carries a specific fluorescent label. Chain elongation continues until a fluorescent
ddNTP is added to the growing chain, after which chain termination occurs. Gel
electrophoresis is performed and the length of each base is detected by laser scanners
with wavelengths specific to the four different ddNTPs.
B A DNA sample is denatured by heating and then put into four tubes. A primer, RNA
polymerase, and all four nucleotides are added. Limited quantities of one of the four

********************************************************
l

308 14 | DNA Structure and Functi


o
dideoxynucleotides (ddNTPs) are added to each tube respectively. Each one of them
carries a specific fluorescent label. Chain elongation continues until a fluorescent
ddNTP is added to the growing chain, after which chain termination occurs. Gel
electrophoresis is performed and the length of each base is detected by laser scanners
with wavelengths specific to the four different ddNTPs.
C A DNA sample is denatured by heating and then put into four tubes. A primer, DNA
polymerase, and all four nucleotides are added. Limited quantities of one of the four
dideoxynucleotides (ddNTPs) are added to each tube respectively. Each one of them
carries a specific fluorescent label. Chain elongation continues until a fluorescent
ddNTP is removed from the growing chain, after which chain termination occurs. Gel
electrophoresis is performed and the length of each base is detected by laser scanners
with wavelengths specific to the four different ddNTPs.
D A DNA sample is denatured by heating and then put into four tubes. A primer, DNA
polymerase, and all four nucleotides are added. Limited quantities of one of the four
deoxynucleotides (dNTPs) are added to each tube respectively. Each one of them
carries a specific fluorescent label. Chain elongation continues until a fluorescent dNTP
is added the growing chain, after which chain termination occurs. Gel electrophoresis
is performed and the length of each base is detected by laser scanners with
wavelengths specific to the four different dNTPs.
Solution The solution is (B). The DNA sample to be sequenced is denatured or separated into
two strands by heating it to high temperatures. The DNA is divided into four tubes in
which a primer, DNA polymerase, and all four nucleotides (A, T, G, and C) are added.
In addition to each of the four tubes, limited quantities of one of the four
dideoxynucleotides are added to each tube respectively. The tubes are labeled as A,
T, G, and C according to the ddNTP added. For detection purposes, each of the four
dideoxynucleotides carries a different fluorescent label. Chain elongation continues
until a fluorescent dideoxy nucleotide is incorporated, after which no further
elongation takes place. After the reaction is over, electrophoresis is performed. Even
a difference in length of a single base can be detected. The sequence is read from a
laser scanner.
62 What process is illustrated in the figure?

********************************************************
14 | DNA Structure and Function PAGE \* MERGEFORMAError! Unknown switch argument.***

********************************************************
l

310 14 | DNA Structure and Functi


o
A Transcription
B Mutation
C Excision
D Translation
Solution The solution is (A). RNA is being made from DNA.
63 How does the model of DNA replication illustrate the function of topoisomerase?
A Topoisomerase relieves the pressure that results from supercoiling by breaking and
reforming DNA’s phosphate backbone ahead of the replication fork.
B Topoisomerase increases the pressure to increase supercoiling by breaking and
reforming DNA’s phosphate backbone ahead of the replication fork.
C Topoisomerase relieves the pressure that results from supercoiling by breaking and
reforming DNA’s nucleotide base pairs ahead of the replication fork.
D Topoisomerase relieves the pressure that results from separation of DNA strands by
breaking and reforming DNA’s phosphate backbone ahead of the replication fork.
Solution The solution is (A). The DNA tends to become more highly coiled ahead of the
replication fork. Topoisomerase breaks and reforms DNA’s phosphate backbone
ahead of the replication fork, thereby relieving the pressure that results from this
supercoiling.
64 Flamingos have genotypes for white feathers yet often appear with pink feathers within
the same population. What is most likely affecting the phenotype of some flamingos,
causing their feathers to turn pink in an isolated population?
A Weather variations
B Dietary changes
C DNA mutations
D Translation failure
Solution The solution is (A). Weather variations would affect the entire isolated population.
65 What can be the result of DNA failing to undergo repair after too much UV exposure?
A Second-degree burns
B A malignant melanoma
C A breakdown of deep layers of the skin
D A sunburn
Solution The solution is (B). UV light exposure can cause melanoma.

********************************************************
14 | DNA Structure and Function PAGE \* MERGEFORMAError! Unknown switch argument.***

********************************************************
l

312 14 | DNA Structure and Functi


o
66 What type of change can occur in the DNA of a chromosome that is termed a
chromosomal mutation?
A Substitution
B Translocation
C Missense
D Transversion
Solution The solution is (B). Sometimes a piece of DNA from one chromosome may get
translocated to another chromosome or to another region of the same
chromosome; this chromosomal mutation is known as translocation.
67 Why are patients with Xeroderma pigmentosa more prone to cancer than the rest of the
population?
A Xeroderma pigmentosa patients cannot employ the nucleotide excision repair
mechanism. When these patients are exposed to UV light, thymine dimers are formed
and they are not able to repair this defect. These dimers distort the structure of DNA
and cause them to have a high risk of contracting skin cancer.
B Xeroderma pigmentosa patients can employ the nucleotide excision repair
mechanism. When these patients are exposed to UV light, the thymine dimers are
formed and they are able to repair this defect. These dimers do not distort the
structure of DNA and they have moderate risk of contracting skin cancer.
C Xeroderma pigmentosa patients cannot employ the nucleotide excision repair
mechanism. When these patients are exposed to UV light, the adjacent adenine forms
dimers and they are not able to repair this defect. These dimers distort the structure
of DNA and they have high risk of contracting skin cancer.
D Xeroderma pigmentosa patients cannot employ the nucleotide excision repair
mechanism. When these patients are exposed to UV light, the adjacent thymine
cannot form thymine dimers and they are not able to repair this defect. The
nonformation of dimers distorts the structure of DNA and they have high risk of
contracting skin cancer.
Solution The solution is (D). Affected individuals have skin that is highly sensitive to UV rays
from the sun. When individuals are exposed to UV, pyrimidine dimers are formed;
people with Xeroderma pigmentosa are not able to repair the damage. These are
not repaired because of a defect in the nucleotide excision repair enzymes, whereas
in normal individuals, the thymine dimers are excised and the defect is corrected.
The thymine dimers distort the structure of the DNA double helix, and this may
cause problems during DNA replication. People with Xeroderma pigmentosa may
have a higher risk of contracting skin cancer than those who do not have the
condition.

********************************************************
14 | DNA Structure and Function PAGE \* MERGEFORMAError! Unknown switch argument.***
68 You are looking at two fragments of DNA. Both have the sequence CATTCTG on one
strand and GTAAGAC on the other. One of the fragments is exposed to UV light, the other
is not.
What will happen to the fragments and how might these mutations be repaired?
A The fragment exposed to UV light contains thymine dimers. Thymines lying adjacent
to each other can form thymine dimers when exposed to UV light. They can be
repaired by nucleotide excision.
B The fragment exposed to UV light contains adenine dimers. Adenines lying adjacent to
each other can form dimers when exposed to UV light. They can be repaired by
nucleotide excision.
C The fragment exposed to UV light contains thymine dimers. Thymines lying parallel to
each other can form thymine dimers when exposed to UV light. They can be repaired
by nucleotide excision.
D The fragment exposed to UV light contains thymine dimers. Thymines lying adjacent
to each other can form thymine dimers when exposed to UV light. They can be
synthesized by nucleotide excision.
Solution The solution is (A). Nucleotide excision repairs thymine dimers. When exposed to
UV, thymines lying adjacent to each other can form thymine dimers. In normal cells,
they are excised and replaced.
69 How can mutations increase variation within a population?
A Substitution mutations may cause a different amino acid to be placed at a specific
location, causing small changes in the protein. Frameshift mutations usually cause
multiple amino acid changes, increasing chances that a new protein will form, leading
to radically different characteristics in the offspring.
B Substitution mutations may cause multiple amino acid changes, increasing chances
that a new protein will form, leading to radically different characteristics in the
offspring. Frameshift mutations may cause a different amino acid to be placed at a
specific location, causing small changes in a protein.
C Substitution mutations may cause a different amino acid to be placed at a specific
location, resulting in major changes to the protein and leading to radically different
characteristics in the offspring. Frameshift mutations cause multiple amino acid
differences in a protein, leading to small changes in the protein.
D Substitution mutations result in a different amino acid being placed at a specific
position in a protein, causing small changes. Silent mutations could result in new
characteristics possessed by an offspring when a stop codon is substituted for an
amino acid.
Solution The solution is (A). Substitution mutations can cause a different amino acid to be
placed at a specific location in a protein that could alter its characteristics and

********************************************************
l

314 14 | DNA Structure and Functi


o
change the population over time. It might result in more resistance to bacterial
infections or even to direct sunlight. Frameshift mutations could, conceivably, make
an entirely different protein, giving the resulting offspring a new characteristic not
possessed previously.

SCIENCE PRACTICE CHALLENGE QUESTIONS


14.1 Historical Basis of Modern Understanding
70 The proof that DNA, not protein, is the carrier of genetic information involved a number
of historical experiments, including transformation or horizontal gene transfer (HGT),
which is the uptake and expression of extracellular DNA.

A. As described in the figure, transformation or HGT was first reported by Griffith in 1928
in an experiment in which the following occurred:
1. Heat-treated, pathogenic bacteria recovered their pathogenicity when incubated
with nonpathogenic bacteria.
2. Plasmids were transferred to nonpathogenic bacteria from pathogenic bacteria
through conjugation.
3. Nonpathogenic bacteria acquired pathogenicity when incubated in a broth
containing heat-treated, pathogenic bacteria.
4. Polysaccharide cell capsules from pathogenic bacteria were transferred to
nonpathogenic bacteria.
B. Griffith’s experiment, however, left undetermined the identity of the cellular
component that encoded genetic information. The identity of DNA as the carrier of
genetic information was resolved through the experiments by Martha Chase and Alfred
Hershey because they observed the following:
1. Injections with a serum containing chemically isolated polysaccharides and
nonpathogenic bacteria were not lethal.

********************************************************
14 | DNA Structure and Function PAGE \* MERGEFORMAError! Unknown switch argument.***
2. Pathogenic bacterial DNA that was radioactively labeled using a phosphorus
isotope was not present in mice that died.
3. Bacteriophages from a bacterial culture grown in a nutrient-containing medium
and radioactively labeled using a sulfur isotope transferred the label to bacteria
incubated in an unlabeled nutrient-containing medium.
4. Bacteriophages from a bacterial culture grown in a nutrient-containing medium
and radioactively labeled using a sulfur isotope did not transfer the label to
bacteria incubated in an unlabeled nutrient-containing medium.
C. Transformation and transduction increase variation within populations of bacteria and
archaebacteria by the following:
1. Transferring DNA among different species
2. Transferring free DNA across the cell membrane without energy expenditure
3. Transferring DNA between different strains of the same species of bacteria
4. Phagocytosis of bacteriophages
The evolution of antibiotic resistance via HGT poses a challenge to medical technology. On
the other hand, transformation often is assayed by incorporating an antibiotic-resistance
gene in the plasmid to be transferred into the host organism. In natural environments,
bacterial and archaebacterial cells become competent (able to transport DNA through the
cytoplasmic membrane) in response to stress such as UV radiation, high population
density, or heat shock. Such conditions are often difficult to model in the laboratory,
where competence can be induced by high concentrations of divalent cations, Ca 2+ or
Mg2+, or electrical shock. In either setting, extracellular DNA can be transported into the
cell, and (to a good approximation) uptake is proportional to the concentration of
extracellular DNA.
D. Identify a factor that might affect transformation or HGT. Then, design a plan to
evaluate the dependence of transformational efficiency (defined as the number of
transformations per gram of extracellular DNA) of plasmids that transfer antibiotic
resistance to a particular strain of Escherichia coli that is not resistant on that factor.
Solution Sample answer:
A. (3)
B. (4)
C. (1)
D. The factor is identified. It might be any environmental factor, such as
temperature, salinity, or pH. The method of control could be specified, such as heat
bath or buffer, but the target of assessment would be the identification of the factor
and the need to vary it measurably while fixing values of other factors. One of these
fixed values should be population density which, based on lab experiences, the

********************************************************
l

316 14 | DNA Structure and Functi


o
student should be able to identify as optical density. Assay could be fluorescent if
the student had prior experience with this method or the use of antibiotic
resistance. Cells that had been transformed would then be counted in terms of the
number of viable or fluorescent colonies. A test of the assumption that uptake is
proportional to extracellular DNA should be included in the design with variation in
DNA concentration at fixed values of other parameters.
71 Prior to the work of Hershey and Chase, scientists thought that inheritance involved
“nucleoproteins.”
The amount of information to be transmitted between generations did not seem
consistent with the chemical simplicity of the few nucleotides found in polymers of
deoxyribonucleic acids in comparison to the diversity of protein polymers. Briefly explain:
 The relationship between the structure of polymeric DNA and the information
stored
 The relationship between the interactions between base pairs on
complementary strands of the double helix and Chargaff’s observation on the
relative abundance of nucleotides in DNA
 The meaning of the statement from the Nature publication on the structure of
DNA by Watson and Crick: “It has not escaped our notice that the specific pairing
we have postulated immediately suggests a possible copying mechanism for the
genetic material.”
Solution Sample answer: Although the number of amino acids and the 3-D shape increases
the complexity of proteins, a very long, linear sequence of four bases stores
sufficient information.
The pairing is the AT and CG forms that are supported by the structure with a pair of
strands, each with corresponding bases. The authors glibly point to the creation of a
whole field, genomics, based on the weakness of the hydrogen bonding between the
complementary strands.
14.2 DNA Structure and Sequencing
72 In 1977, Fred Sanger developed a method to determine the order of nucleotides in a
strand of DNA. Sanger won a Nobel Prize for his work, and his method of sequencing
based on dideoxy chain termination (see figure) has been foundational to the rapid
development of more modern, rapid, and inexpensive methods of sequencing. The
challenge of the $1,000 in one-day sequencing of the human genome was achieved in
2016 by next-generation sequencing (NGS), a “catch-all” term describing several
sequencing methods.

********************************************************
14 | DNA Structure and Function PAGE \* MERGEFORMAError! Unknown switch argument.***

A. Using the diagrams for reference, explain the effect of the addition of
dideoxynucleotides on chain growth of the DNA strand that is copied during sequencing in
terms of the structures of dideoxyribose and deoxyribose.

B. Suppose that a single strand to be sequenced is 5′CGAGTACG3′. In the presence of each


of the four deoxynucleotides and the dideoxynucleotide ddCTP, describe the strands that
would be formed from this template. Include in your description an annotation indicating
the 3′ and 5′ ends of the fragments resulting from the procedure.
C. Next-generation sequencing makes termination technology very rapid and relatively
inexpensive. All babies born in the United States are currently screened by state-
mandated tests for several genetic conditions. The number of conditions tested ranges
from 29 (GA and KS) to 59 (IL and MS). It is proposed that whole-genome sequencing
should be mandatory for all newborns. The Genetic Information Nondiscrimination Act
(2008) prevents health insurers from denying coverage or increasing costs of premiums
based on genetic information. It also prohibits employers from making use of these data
for hiring, firing, or promotion. The act passed in the House with a vote of 420 to 3,
although it was lobbied against by organizations representing business (human resources,
health insurance, and manufacturers), including the U.S. Chamber of Commerce. The act
does not cover life, long-term care, or disability insurance. Pose three questions that are
relevant to the use of whole-genome data.
Solution Sample answer:
A. Without the OH at the 3’ position a Pi group cannot bond and extend the sugar
backbone.

********************************************************
l

318 14 | DNA Structure and Functi


o
B. Chain termination would occur at each G leading to the fragments 3′CTCATDC5′,
3′CATGC5′, and 3′C5′.
C. Questions could address ownership (ownership is a difficult question—is the DNA
on a licked postage stamp still owned?—similar to questions of permissible
searches), the relative benefits to society and perhaps to the individual of genomic
information could include reduced cost of insurance, reduced cost of medical
treatment, increased efficiency and effectiveness of medical treatment, the possible
creation of caste systems. A really good scientific question might be the extent to
which possibly identifiable, distinct phenotypes such as intelligence, athleticism,
beauty, artistic talent, etc., can be associated with a gene or an array of genes, as
opposed to environmental factors or behaviors. A search will show that the omission
of life insurance from GINA has led to some discussion and some possible concerns.
The student might question the reason that life insurance or disability insurance was
omitted. Questions might address whether or not courts have heard claims based on
GINA—there have been some and they are easily found. Questions about data
security will be asked.
14.3 Basics of DNA Replication
73 Our understanding of the mechanisms of DNA replication is important to research on
cancer and aging. Additionally, the molecular basis of Mendelian genetics was established.
A. The mechanism of DNA replication was investigated by Meselson and Stahl. The diagram
from their 1958 paper summarizes their findings. Describe how this representation
illustrates the manner in which DNA is copied for transmission between generations.

********************************************************
14 | DNA Structure and Function PAGE \* MERGEFORMAError! Unknown switch argument.***

B. During the synthesis of new strands of DNA from the parent strands, DNA polymerase
can only add nucleotides at the terminal 3’ of a growing strand. Using the diagram,
describe the similarities and differences between the DNA replication of both strands.

C. Shown at the left end of the upper parent strand is the six-base repeat sequence
TTAGGG. In humans, this is the repeated, telomeric sequence that is attached to the
telomere. The RNA primer in humans spans 10 base pairs, unlike in the drawing where it
spans only three. In somatic cells, an enzyme called telomerase no longer functions.
Explain the function of telomerase in the development of stem cells and cancer cells, and
the inhibition of telomerase in programmed cell death or apoptosis.
Solution Sample answer:
A. The semiconservative nature of replication is illustrated by this very effective
representation in which the newly synthesized strands are unshaded. The student

********************************************************
l

320 14 | DNA Structure and Functi


o
might be expected to describe the use of centrifugation to separate light and heavy
isotopes of nitrogen.
B. The description should include the breaking of hydrogen bonds by helicase, the
attachment of DNA polymerase to the leading strand, the creation of a template for
synthesis on the lagging strand with RNA primer and the synthesis of fragments as
required by the 3’ to 5’ direction of addition. Names of specific enzymes are not
expected to be memorized and this item illustrates how they would be provided but
that the information provided can be used.
C. Telomerase as assessable content is out of scope although it is described in the
text. What would be appropriate for assessment on the AP Biology Exam would be
the recognition that the RNA primer might not have a sufficient number of bases
remaining on the lagging strand as DNA pol approaches the telomere. This results in
a segment of the lagging side not being copied. Without telomerase to repair the
lost telomeric sequence, the number of repeats is reduced until eventually the cell
cannot divide, leading to cell death. Cancer cells activate telomerase and so avoid
cell death.
14.4 DNA Replication in Prokaryotes
74 The mitochondria of eukaryote cells contain their own circular DNA (mtDNA), consistent
with their origin according to the theory of endosymbiosis. The mitochondrial genome is
highly conserved in Eukarya. In humans, the 50 to 100 mitochondria in each of the cells in
most tissues have 5 to 10 copies of the genome. Each has 37 genes that primarily encode
proteins of the electron transport chain. Point mutations in which a single nucleotide is
incorrectly placed is not repaired because the error-checking provided by DNA
polymerase is not present in the mitochondria. The mutation rate for mtDNA is
approximately 100 times higher than the mutation rate for nuclear DNA. The
simultaneous existence of multiple alleles in each cell is likely, a condition called
heteroplasmy. In mammals, sperm mitochondria are destroyed prior to fertilization.
A. Explain how point mutations in mtDNA can result in a loss of function in critical cellular
components such as cytochrome c yet not be lethal to the cell.
B. Oocyte mitochondria are randomly segregated during meiosis, resulting in variation in
the frequency of mtDNA mutations in offspring relative to the parent. Explain how a loss
of function does not accumulate, lowering the metabolic performance from generation to
generation.
As described in the Evolution Connection in this chapter of the text, a fossil fingertip found
in a Siberian cave revealed an evolutionary link between Neanderthals and Denisovans.
Fossils from 28 individuals were located in the “pit of bones,” Sima de los Huesos, in
Spain, thousands of miles from the Siberian cave. In 2013, mtDNA from a femur of one of
these individuals was compared with mtDNA of Denisovans, Neanderthals, and modern

********************************************************
14 | DNA Structure and Function PAGE \* MERGEFORMAError! Unknown switch argument.***
humans. It was found that the Sima fossil shared many more alleles with Denisovans than
with either Neanderthals or modern humans. In 2016, the same group of scientists who
sequenced the mtDNA from the femur of one of the Sima fossils partially sequenced the
DNA from that fossil, showing a clear connection to Neanderthals.
C. Analyze these data to draw alternative conclusions regarding the relatedness of the
three fossils and support each with evidence.
D. Design a plan to differentiate or resolve these alternative conclusions.
Solution Sample answer:
A. Multiple copies of mtDNA allow expression even though some copies code for
proteins that do not function.
B. As mutations accumulate the cell become less efficient. When a cell line has less
than a threshold number of functional mitochondrial, the cell dies and that cell line
is deleted. This applies to both somatic cells and to gametes.
C. The mtDNA evidence indicates that the Sima people are more closely related to
the Denosivans. However, the DNA evidence indicates the opposite conclusion.
D. The recovery of both mtDNA and nuclear DNA was a major feat. However,
suppose that the mtDNA reveals a maternal ancestor of different lineage. Because of
the very high mutation rate of mtDNA large variations between family lines occur
while an ancestral line within a family remains very similar. mtDNA from other bones
at Sima might reveal other maternal lines and this would be consistent with the DNA
result, implying a common maternal ancestor related to Denosivans. Evidence that
would confirm this would be provided by a third, as yet undiscovered or unstudied,
population whose mtDNA could be sequenced.

********************************************************
**************************336***************************

15 | GENES AND PROTEINS


REVIEW QUESTIONS
1 What is the flow of information for protein synthesis according to the central dogma?
A DNA to mRNA to protein
B DNA to mRNA to tRNA to protein
C DNA to protein to mRNA to protein
D mRNA to DNA to mRNA to protein
Solution The solution is (A). DNA carries genetic information that is copied onto an mRNA
template to make a particular protein.
2 The DNA of virus A is inserted into the protein coat of virus B. The combination virus is
used to infect E. coli. The virus particles produced by the infection are analyzed for DNA
and protein.
What results would you expect?
A DNA and protein from B
B DNA and protein from A
C DNA from A and protein from B
D DNA from B and protein from A
Solution The solution is (B). DNA is the genetic material, not protein. So when virus A is
infected in the protein coat of virus B, the new virus produced will have DNA and
protein of virus A. Protein is not the genetic material and cannot be inherited.
3 The AUC and AUA codons in mRNA both specify isoleucine. What feature of the genetic
code explains this?
A Complementarity
B Degeneracy
C Nonsense codons
D Universality
Solution The solution is (B). The genetic code is a triplet code, with each DNA or RNA codon
consisting of three nucleotides that encode one amino acid. It is degenerate, as most
amino acids can be specified by more than one codon.

********************************************************
**************************************************335***

4 How many nucleotides are in 12 mRNA codons?


A 12
B 24
C 36
D 48
Solution The solution is (C). Twelve mRNA codons would have 36 nucleotides, since each
codon consists of three nucleotides.
5 Which molecule does NOT contain genetic information?
A DNA
B mRNA
C Protein
D RNA
Solution The solution is (C). Proteins are made up of amino acids and do not contain genetic
information.
6 Which molecule in the central dogma can be compared to a disposable photocopy of a
book kept on reserve in the library?
A DNA
B mRNA
C Protein
D tRNA
Solution The solution is (B). Messenger RNA is like a disposable photocopy of a book kept on
reserve in the library. It carries a copy of the instructions from the nucleus to other
parts of the cell and usually has a short half-life.
7 Which subunit of the E. coli polymerase confers specificity to transcription?
A

C
D
Solution The solution is (D). The subunit is involved in transcription initiation. It confers
transcriptional specificity so that the polymerase begins to synthesize mRNA from an
appropriate initiation site.

********************************************************
**************************336***************************

8 Why are the and regions of prokaryotic promoters called consensus sequences?
A They are identical in all bacterial species.
B They are similar in all bacterial species.
C They exist in all organisms.
D They have the same function in all organisms.
Solution The solution is (B). At the and regions upstream of the initiation site, there
are two promoter consensus sequences, or regions, that are similar across all
promoters and across various bacterial species.
9 The sequence that signals the end of transcription is called the —
A promoter
B stop codon
C TATA box
D terminator
Solution The solution is (D). Transcription continues until RNA polymerase reaches a stop or
terminator sequence at the end of a gene.
10 If the protein is missing, will a prokaryotic gene be terminated?
A It depends on the gene.
B No, the rho protein is essential.
C Transcription termination is not required.
D Yes, the rho protein is not involved in transcription.
Solution The solution is (A). It depends on the sequence of the gene as to whether the
transcription termination is rho dependent or independent. If the rho protein is
missing, then termination can occur if the sequence consists of short strings of
adenines.
11 Which feature of promoters can be found in both prokaryotes and eukaryotes?
A GC box
B Octamer box
C TATA box
D and sequences
Solution The solution is (C). The TATA box is a DNA sequence found in the promoter regions
of both prokaryotes and eukaryotes.

********************************************************
**************************************************335***

12 At what stage in the transcription of a eukaryotic gene would TFII factors be active?
A Elongation
B Initiation
C Processing
D Termination
Solution The solution is (B). The initiation of transcription in eukaryotes requires TFII. It binds
to the promoter region and then recruits RNA polymerase II.
13 Which polymerase is responsible for the synthesis of 5S rRNA?
A Polymerase I
B Polymerase II
C Polymerase III
D Ribonuclease I
Solution The solution is (C). Polymerase III transcribes a variety of structural RNAs that
includes the 5S pre-rRNA, transfer pre-RNAs (pre-tRNAs), and small nuclear
pre-RNAs.
14 What transcripts will be most affected by low levels of -amanitin?
A 18S and 28S rRNAs
B 5S rRNAs and tRNAs
C Other small nuclear RNAs
D Pre-mRNAs
Solution The solution is (D). Pre-mRNAs are transcribed by RNA polymerase II and are
extremely sensitive to low levels of -amanitin.
15 Which feature distinguishes eukaryotic transcription from bacterial transcription?
A Eukaryotic transcription does not start at a consensus sequence.
B Eukaryotic transcription does not require an initiation complex.
C Eukaryotic transcription and translation do not take place at the same time.
D Eukaryotic transcription does not require a termination sequence.
Solution The solution is (C). In prokaryotes, both transcription and translation takes place in
the cytoplasm, while in eukaryotes, the transcription takes place in the nucleus and
translation occurs in the cytoplasm.
16 A poly-A sequence is added at the —
A 5′ end of a transcript in the nucleus
B 3′ end of a transcript in the nucleus

********************************************************
**************************336***************************

C 5′ end of a transcript in the cytoplasm


D 3′ end of a transcript in the cytoplasm
Solution The solution is (B). At the 3′ end of the transcript, a poly-A sequence is added.
Polymerase A adds a string of approximately 200 A residues.
17 Which pre-mRNA processing step is important for initiating translation?
A Adding a poly-A tail
B RNA editing
C Splicing
D Adding the 7-methylguanosine cap
Solution The solution is (D). Capping occurs at the 5′ end while pre-mRNA synthesis is still
going on. A 7-methylguanosine cap is added by a phosphate linkage at the 5′ end of
the growing transcript.
18 Where are the RNA components of ribosomes synthesized?
A Cytoplasm
B Endoplasmic reticulum
C Nucleus
D Nucleolus
Solution The solution is (D). The nucleolus in the eukaryotic cell is a condensed region where
ribosomes are formed. The ribosomal subunits are synthesized in the nucleolus, and
then exported to the cytoplasm before use.
19 What processing step enhances the stability of pre-tRNAs and pre-rRNAs?
A Cleavage
B Methylation
C Nucleotide modification
D Splicing
Solution The solution is (B). Methylation is the addition of CH3 moiety for the stability of pre-
tRNAs and pre-rRNAs.
20 What are introns?
A DNA sequences to which polymerases bind
B Processed mRNA
C Translated DNA sequences in a gene
D Untranslated DNA sequences in a gene

********************************************************
**************************************************335***

Solution The solution is (D). Untranslated DNA sequences that are transcribed in pre-mRNAs
and consist of noncoding or intervening sequences are called introns. Until introns
are removed, translation does not occur.
21 What is often the first amino acid added to a polypeptide chain?
A Adenine
B Leucine
C Methionine
D Thymine
Solution The solution is (C). AUG on an mRNA, where translation begins, always specifies
methionine.
22 In any given species, there are at least how many types of aminoacyl tRNA synthetases?
A 20
B 40
C 100
D 200
Solution The solution is (A). Each tRNA molecule is linked to its correct amino acid by a group
of enzymes called aminoacyl tRNA synthetases. At least one type of aminoacyl tRNA
synthetase exists for each amino acid.
23 In prokaryotic cells, ribosomes are found in/on the —
A cytoplasm
B mitochondrion
C nucleus
D endoplasmic reticulum
Solution The solution is (A). Prokaryotic cells do not contain organelles. The ribosomes lie in
the cytoplasm of prokaryotes.
24 Peptide bond synthesis in prokaryotic translation is catalyzed by —
A a ribosomal protein
B a cytoplasmic protein
C mRNA itself
D ribosomal RNA
Solution The solution is (D). Peptidyl transferase is an RNA-based enzyme that is integrated
into the 50S ribosomal subunit and catalyzes the formation of peptide bonds.

********************************************************
**************************336***************************

25 What would happen if the 5′-methyl guanosine was NOT added to an mRNA?
A The transcript would be degraded when the mRNA moves out of the nucleus to the
cytoplasm.
B The mRNA molecule would be stabilized and start the process of translation within the
nucleus of the cell.
C The mRNA molecule would move out of the nucleus and create more copies of the
mRNA molecule.
D The mRNA molecule would not be able to add the poly-A tail onto its strand at the
5′ end.
Solution The solution is (A). Without 5′ capping, the pre-mRNA transcript would be degraded
and initiation of translation would be compromised.
26 Which option is associated with the docking of mRNA on a ribosome in eukaryotic cells?
A Kozak’s rules
B Poly-A sequence
C Shine-Dalgarno sequence
D TATA box
Solution The solution is (A). A Kozak consensus sequence is found in eukaryotic mRNA; the
following consensus sequence must appear around the AUG:
5′-GCC(purine)CCAUGG-3′

CRITICAL THINKING QUESTIONS


27 If mRNA is complementary to the DNA template strand and the DNA template stand is
complementary to the DNA non-template strand, why are the base sequences of mRNA
and the DNA non-template strand not identical? Could they ever be?
A No, they cannot be identical because the T nucleotide in DNA is replaced by the U
nucleotide in RNA, and AUG is the start codon.
B No, they cannot be identical because the T nucleotide in RNA is replaced by the U
nucleotide in DNA.
C They can be identical if methylation of the U nucleotide in RNA occurs, yielding a T
nucleotide.
D They can be identical if demethylation of the U nucleotide in RNA occurs, yielding a T
nucleotide.
Solution The solution is (A). DNA is different from RNA in that T nucleotides in DNA are
replaced by U nucleotides in RNA. The start codon, AUG, includes a U nucleotide.
Therefore, they could never be identical in base sequence.

********************************************************
**************************************************335***

28 Imagine if there were 200 commonly occurring amino acids instead of 20. Given what you
know about the genetic code, what would be the shortest possible codon length?
A 4
B 5
C 2
D 3
Solution The solution is (A). For 200 commonly occurring amino acids, codons consisting of
four types of nucleotides would have to be at least four nucleotides long, because
44 = 256. There would be much less degeneracy in this case.
29 What part of the central dogma is NOT always followed in viruses?
A The flow of information in HIV is from RNA to DNA, then back to RNA to protein.
Influenza viruses never go through DNA.
B The flow of information is from protein to RNA in HIV, while the influenza virus
converts DNA into RNA.
C The flow of information is similar, but nucleic acids are synthesized as a result of
translation in HIV and influenza viruses.
D The flow of information is from RNA to protein. This protein is used to synthesize the
DNA of the viruses in HIV and influenza.
Solution The solution is (A). The flow of information goes from RNA to DNA back to RNA to
protein in HIV. Other viruses such as the influenza virus never go through DNA.
30 Suppose a gene has the sequence ATGCGTTATCGGGAGTAG. A point mutation changes the
gene to read ATGCGTTATGGGGAGTAG. How would the polypeptide product of this gene
change?
Solution It would change from MRYRE to MRYGE.
31 Explain the initiation of transcription in prokaryotes. Include all proteins involved.
A In prokaryotes, the polymerase comprises five polypeptide subunits, two of which are
identical. Four of these subunits, denoted and compose the
polymerase core enzyme. The fifth subunit, is involved only in the initiation of
transcription. The polymerase, which comprises all five subunits, is called the
holoenzyme.

********************************************************
**************************336***************************

B In prokaryotes, the polymerase comprises four polypeptide subunits, two of which are
identical. These subunits, denoted and compose the polymerase core
enzyme. There is a fifth subunit that is involved in translation initiation. The
polymerase, which comprises all four subunits, is called the holoenzyme.
C In prokaryotes, the polymerase comprises five polypeptide subunits, two of which are
identical. Four of these subunits, denoted and compose the
polymerase holoenzyme. The fifth subunit, is involved only in transcription
initiation. The polymerase, which comprises all five subunits, is called the core
enzyme.
D In prokaryotes, the polymerase comprises five polypeptide subunits, two of which are
identical. Four of these subunits, denoted and compose the
polymerase core enzyme. The fifth subunit, is involved only in termination. The
polymerase, which comprises all five subunits, is called the holoenzyme.
Solution The solution is (A). Refer to Figure 15.7 and add the additional subunits of the
holoenzymes that are not in the figure.
32 How would you describe the difference between rho-dependent and -independent
termination of transcription in prokaryotes?
A Rho-dependent termination is controlled by the rho protein, and the polymerase stalls
near the end of the gene at a run of G nucleotides on the DNA template. In rho-
independent termination, when the polymerase encounters a region rich in C-G
nucleotides, the mRNA folds into a hairpin loop that causes the polymerase to stall.
B Rho-independent termination is controlled by the rho protein, and the polymerase
stalls near the end of the gene at a run of G nucleotides on the DNA template. In rho-
dependent termination, when the polymerase encounters a region rich in C–G
nucleotides, the mRNA folds into a hairpin loop that causes the polymerase to stall.
C Rho-dependent termination is controlled by the rho protein, and the polymerase
begins near the end of the gene at a run of G nucleotides on the DNA template. In rho-
independent termination, when the polymerase encounters a region rich in C–G
nucleotides, the mRNA creates a hairpin loop that causes the polymerase to stall.
D Rho-dependent termination is controlled by the rho protein, and the polymerase stalls
near the end of the gene at a run of G nucleotides on the DNA template. In rho-
independent termination, when the polymerase encounters a region rich in A–T
nucleotides, the mRNA creates a hairpin loop that causes the polymerase to stall.
Solution The solution is (A). Rho-dependent termination is controlled by the rho protein,
which tracks along behind the polymerase on the growing mRNA chain. Near the
end of the gene, the polymerase stalls at a run of G nucleotides on the DNA
template. The rho protein collides with the polymerase and releases mRNA from the
transcription bubble. Rho-independent termination is controlled by specific
sequences in the DNA template strand. As the polymerase nears the end of the gene

********************************************************
**************************************************335***

being transcribed, it encounters a region rich in C–G nucleotides. This creates an


mRNA hairpin that causes the polymerase to stall right as it begins to transcribe a
region rich in A–T nucleotides. Because A–U bonds are less thermostable, the core
enzyme breaks away.
33 What is the main structure that differentiates rho-dependent and -independent
termination in prokaryotes?
A Rho-independent termination involves the formation of a hairpin.
B Rho-dependent termination involves the formation of a hairpin.
C Rho-dependent termination stalls when the polymerase begins to transcribe a region
rich in A–T nucleotides.
D Rho-independent termination stalls when the polymerase begins to transcribe a
region rich in G nucleotides.
Solution The solution is (A). Refer to Figures 15.7 and 15.8. Add the termination step, which
may involve either the rho protein or a hairpin.
34 Which step in the transcription of eukaryotic RNA differs the most from its prokaryotic
counterpart?
A The initiation step in eukaryotes requires an initiation complex with enhancers and
transcription factors. Also, the separation of the DNA strand is different, as histones
are involved.
B The initiation step in prokaryotes requires an initiation complex with enhancers and
transcription factors. Also, the separation of the DNA strand is different, as histones
are involved.
C The elongation step in eukaryotes requires an initiation complex with enhancers and
transcription factors. Also, the separation of the DNA strand is different, as histones
are involved.
D The initiation step in eukaryotes requires an initiation complex with enhancers and
transcription factors. Also, the separation of the DNA strand is different, as histones
are not involved.
Solution The solution is (A). The initiation step in eukaryotes requires an initiation complex
with enhancers and transcription factors. The separation of the DNA strand is also
different as histones are involved.
35 Would you be able to determine which RNA polymerase you isolated from a eukaryotic
cell without analyzing its products?
A No, they have the same -amanitin sensitivity in all products.
B No, quantitative analysis of products is done to determine the type of polymerase.
C Yes, they can be determined as they differ in -amanitin sensitivity.
D Yes, they can be determined by the number of molecules that bind to the DNA.

********************************************************
**************************336***************************

Solution The solution is (C). Yes, RNA polymerases differ in their sensitivity to -amanitin.
36 Predict how alternative splicing may lead to an economy of genes. Do you need a
different gene for every protein that the cell can produce?
A Alternative splicing can lead to the synthesis of several polypeptides from a
single gene.
B Alternative splicing can lead to the synthesis of several forms of mRNA from a
single gene.
C Alternative splicing can lead to the synthesis of several forms of codons from a set
of genes.
D Alternative splicing can lead to the synthesis of several forms of ribosomes from a set
of genes.
Solution The solution is (A). It is possible to synthesize several polypeptides from a single
gene by alternative splicing.
37 What is the major challenge in the production of RNA in eukaryotes compared to
prokaryotes?
A Exporting the mRNA across the nuclear membrane
B Importing the mRNA across the nuclear membrane
C Keeping the mRNA inside the nuclear membrane
D Translating the mRNA into proteins within seconds
Solution The solution is (A). The major challenge is exporting the mRNA across the nuclear
membrane.
38 What would happen if the 5’ methyl guanosine was not added to an mRNA?
A The transcript would degrade when the mRNA moves out of the nucleus to the
cytoplasm.
B The mRNA molecule would stabilize and start the process of translation within the
nucleus of the cell.
C The mRNA molecule would move out of the nucleus and create more copies of the
mRNA molecule.
D The mRNA molecule would not be able to add the poly-A tail on its strand at the 5’
end
Solution The solution is (A). Without the 5’ capping, the pre-mRNA transcript would degrade
and initiation of translation would be compromised.
39 How should the following DNA sequence (non-template strand) be transcribed and
translated?
5′-ATGGCCGGTTATTAAGCA-3′

********************************************************
**************************************************335***

A The mRNA would be 5′-AUGGCCGGUUAUUAAGCA-3′ and the protein will be MAGY.


B The mRNA would be 3′-AUGGCCGGUUAUUAAGCA-5′ and the protein will be MAGY.
C The mRNA would be 5′-ATGGCCGGTTATTAAGCA-3′ and the protein will be MAGY.
D The mRNA would be 5′-AUGGCCGGUUAUUAAGCA-3′ and the protein will be MACY.
Solution The solution is (A). The mRNA would be 5′-AUGGCCGGUUAUUAAGCA-3′. The protein
would be MAGY. Even though there are six codons, the fifth codon corresponds to a
stop, so the sixth codon would not be translated

********************************************************
**************************336***************************

40 The RNA world hypothesis proposes that the first complex molecule was RNA, and it
preceded protein formation. Which major function of ribosomal RNA supports this
hypothesis?
A rRNA has catalytic properties in the large subunit, and it assembles proteins.
B rRNA is a protein molecule that helps in the synthesis of other proteins.
C rRNA is essential for the transcription process.
D rRNA plays a major role in posttranslational processes.
Solution The solution is (A). The rRNA in the large subunit has catalytic properties. It
synthesizes the peptide bond. The main idea is that the molecule that assembles
proteins cannot itself be a protein. In other words, a protein could not have
synthesized the first protein. RNA subunits can self-assemble.
41 A tRNA is chemically modified so that the bound amino acid is different than the one
specified by its anticodon. Which codon in the mRNA would the tRNA recognize: the one
specified by its anticodon or the one that matches the modified amino acid it carries?
A The anticodon will match the codon in mRNA.
B The anticodon will match with the modified amino acid it carries.
C The anticodon will lose the specificity for the tRNA molecule.
D The enzyme aminoacyl tRNA synthetase would lose control over the amino acid.
Solution The solution is (A). The anticodon matches the codon in mRNA.

TEST PREP FOR AP® COURSES


42 What characteristic of the genetic code points to a common ancestry for all organisms?
A The code is degenerate.
B The code contains 64 codons.
C The genetic code is almost universal.
D The code contains stop codons.
Solution The solution is (C). The code is universal.
43 What process transfers heritable material to the next generation?
A Replication
B Splicing
C Transcription
D Translation
Solution The solution is (A). Replication is the process by which two identical DNA molecules
are produced from one DNA molecule.

********************************************************
**************************************************335***

44 When comparing transcription of heritable information in prokaryotes and eukaryotes,


which events are the same?
A The transcription by polymerase, the recognition of a consensus sequence in the
promoter, and the termination by a hairpin loop are conserved.
B The translation by polymerase, the recognition of a consensus sequence in the
promoter, and the termination by a hairpin loop are conserved.
C The transcription by polymerase, the recognition of a highly variable sequence in the
promoter, and the termination by a hairpin loop are conserved.
D The transcription by polymerase, the recognition of a consensus sequence in the
promoter, and the elongation by a hairpin loop are conserved.
Solution The solution is (A). Transcription by a polymerase, recognition of a consensus
sequence in the promoter, and termination by a hairpin loop are conserved.
45 Which cell structure does NOT contain heritable information?
A Chloroplast
B Cytoplasmic membrane
C Mitochondria
D Nucleus
Solution The solution is (B). The cytoplasmic membrane separates the interior of the cell from
the outside environment.
46 How does the enzyme reverse transcriptase violate the central dogma of molecular
biology in HIV?
A The enzyme reverse transcriptase reverse-transcribes the RNA in the genome of HIV
into DNA.
B The enzyme reverse transcriptase translates the RNA of the HIV into protein and then
back into DNA.
C The enzyme reverse transcriptase transcribes the DNA straight into the protein
molecules.
D The enzyme reverse transcriptase transcribes DNA into RNA, then again into DNA.
There is no protein synthesis.
Solution The solution is (A). The genome of HIV is made of two molecules of RNA, which are
reverse transcribed to DNA in the host. The enzyme reverse transcriptase is virally
encoded and packaged in the virus particle.

********************************************************
**************************336***************************

47 Radioactive deoxythymidine triphosphate is supplied to the protist Euglena. After an


interval of time, the cells are homogenized, and different fractions are analyzed for
radioactivity content in large nucleic acid molecules. Which fraction will not be labeled?
A Nucleus
B Mitochondrion
C Chloroplast
D Plasma membrane
Solution The solution is (D). The plasma membrane does not contain DNA.
48 You sequence a gene of interest and isolate the matching mRNA. You find that the mRNA
is considerably shorter than the DNA sequence. Why is that?
A There was an experimental mistake. The mRNA should be the same length as
the gene.
B The mRNA should be longer than the DNA sequence because the promoter is also
transcribed.
C The processed mRNA is shorter because introns were removed.
D The mRNA is shorter because the signal sequence to cross the nuclear membrane was
removed.
Solution The solution is (C). Introns are noncoding regions of an RNA transcript, which are
spliced out before the RNA is translated into protein. Splicing decreases the length
of mRNA in comparison to DNA molecule.
49 A mutation in the promoter region of the gene for beta-globin can cause beta-
thalassemia, a hereditary condition that causes anemia. Why would mutations in the
promoter region lead to low levels of hemoglobin?
A The globin chains produced are too long to form functional hemoglobin.
B The globin chains are too short to form functional hemoglobin.
C Fewer globin chains are synthesized because less mRNA is transcribed.
D Globin chains do not fold properly and are nonfunctional.
Solution The solution is (D). Protein folding is determined by the amino acid sequence.
Mutations in the promoter region do not affect the amino acid sequence of the
protein.
50 You are given three mRNA sequences:
1. 5′-UCG-GCA-AAU-UUA-GUU-3
2. 5′-UCU-GCA-AAU-UUA-GUU-3′
3. 5′-UCU-GCA-AAU-UAA-GUU-3′

********************************************************
**************************************************335***

Using the table, write the peptide encoded by each of the mRNA sequences.
Codon on mRNA Amino Acid
GCA alanine
AAG lysine
GUU valine
AAU asparagine
UGC cysteine
UCG serine
UCU serine
UUA leucine
UAA stop
A (i) Serine-alanine-asparagine-leucine-valine; (ii) serine-alanine-asparagine-leucine-
valine; (iii) serine-alanine-asparagine(-stop)
B (i) Serine-phenylalanine-asparagine-leucine-valine; (ii) serine-alanine-asparagine-
leucine-valine; (iii) serine-alanine-asparagine(-stop)
C (i) Serine-alanine-asparagine-leucine-valine; (ii) serine-alanine-asparagine(-stop); (iii)
serine-alanine-asparagine-leucine-valine
D (i) Serine-alanine-asparagine-leucine-valine; (ii) serine-arginine-asparagine-leucine-
valine; (iii) serine-alanine-asparagine(-stop)
Solution The solution is (A). The three mRNA sequences are (i) serine-alanine-asparagine-
leucine-valine; (ii) serine-alanine-asparagine-leucine-valine; (iii) serine-alanine-
asparagine(-stop).
51 Refer to the table.
Codon on mRNA Amino Acid
GCA alanine
AAG lysine
GUU valine
AAU asparagine
UGC cysteine
UCG serine
UCU serine
UUA leucine
UAA stop

********************************************************
**************************336***************************

You are given three mRNA sequences:


1. 5′-UCG-GCA-AAU-UUA-GUU-3
2. 5′-UCU-GCA-AAU-UUA-GUU-3′
3. 5′-UCU-GCA-AAU-UAA-GUU-3′
Using the peptide encoded by each sequence, compare the three peptides obtained. How
are peptides 2 and 3 different from 1? What would the consequence be for the cell in
each case?
A There is a silent mutation in peptide 2, and peptide 3 has a stop codon due to a
mutation.
B There is a silent mutation in peptide 3, and peptide 2 has a stop codon due to a
mutation.
C There is a different amino acid in peptide 2, and peptide 3 has a stop codon due to a
mutation.
D There isn’t a mutation in peptide 2, and peptide 3 has a stop codon due to a mutation.
Solution The solution is (A). In the case of peptide 2, there is a silent mutation. Although the
base has been substituted, the amino acid is the same because the code is
degenerate. In peptide 3, the mutation introduced a stop codon. Translation stops at
the asparagine.

SCIENCE PRACTICE CHALLENGE QUESTIONS


15.1 The Genetic Code
52 Gamow (1954) proposed that the structure of DNA deduced by Watson and Crick (1953)
could be interpreted as a way of forming roughly 20 “words”" of the common amino acids
from the four “letters” A, T, C, and G that represent DNA nucleotides.
Crick and coworkers (1961) used a method developed by Benzer to induce mutations in
the DNA of a virus by the insertion of a single nucleotide. The mutant could not infect the
bacterium Escherichia coli and neither could viruses with a second insertion of a second
DNA nucleotide. However, a third nucleotide insertion restored the ability of the virus to
infect the bacterium.
In 1961, Nirenberg and Matthaei conducted a series of experiments to better understand
the flow of genetic information from gene to protein. They discovered that in solutions
containing the contents of ruptured E. coli bacterial cells from which DNA had been
removed, polymers containing only one repeating amino acid, phenylalanine, would be
synthesized if synthetic mRNA composed of only the single nucleotide, uracil (U), was
added to the solution in which phenylalanine was also present. In solutions containing
mRNA with only adenine (A) or cytosine (C) and the amino acids lysine or proline,
polymers containing only these amino acids would be synthesized. The researchers found

********************************************************
**************************************************335***

that when ribosomes were removed by filtration, these polymers did not form. Nirenberg
and Leder (1964) extended this work to include other nucleotides.
A. Summarize the conclusions regarding the encoding and decoding of heritable
information supported by these studies. Explain how these studies provided evidence to
support the triplet code.
Khorana (1960) developed a technique for synthesizing RNA composed of predictable
distributions of repeated pairs or triplets of nucleotides. He found, for example, that RNA
synthesized when A and U were present in relative concentrations of 4:1, respectively, will
produce RNA sequences with these distributions determined by their relative
probabilities: AAU:AAA, AUA:AAA, and UAA:AAA; [calculated as
follows: (i) 4/5 of the bases are A, so the likelihood of selecting A is 0.8; (ii) the selection is
repeated to determine the second letter of the three-letter codon; (iii) the likelihood of
selecting a U is 1 in 5; (iv) the probability of selecting the set AUU is the product;
(v) similarly, the probability of AAA is (4/5)3; and (vi) the ratio of these probabilities is
their relative likelihood]: AUU:AAA, UUA:AAA, and UAU:AAA;
and UUU:AAA; .
B. Based on Khorana’s findings, calculate the relative distributions of the following ratios
of concentrations of RNA triplet sequences from mixtures in which the relative
concentrations of guanine and cytosine, G:C, are 5:1.
Ratio Relative Probabilities
GGC:GGG
GCG:GGG
CGG:GGG
GCC:GGG
CGC:GGG
CCG:GGG
CCC:GGG
C. Based on the work of Nirenberg, Matthaei, Leder, and Khorana, the following table was
constructed (taken from Khorana's Nobel Prize address):

********************************************************
**************************336***************************

A solution containing the amino acids shown in the table and equal concentrations of the
two nucleotides C and G is prepared. Predict the proteins that can be synthesized from
this mixture in terms of each possible codon and their relative concentrations in terms of
their amino acid repeat sequences.
D. Describe the effects of the codons UAA, UAG, and UGA on protein synthesis.
Solution Sample answer:
A. Gamow (a physicist) proposed that a sequence of three nucleotide letters could
be used to make a single amino acid word. Crick and colleagues found that the
length of the word was always three letters. Nirenberg, Matthaei, and Leder showed
that DNA could be synthesized in the absence of a living cell and confirmed Gamow's
idea.
B.
Ratio Relative Probabilities
GGC:GGG
GCG:GGG
CGG:GGG
GCC:GGG
CGC:GGG
CCG:GGG
CCC:GGG

C. pro (CCC), ser (CCG), gly (CGG), arg (CGC), gly (GGG), gly (GGC), ala (GCC), and
gly (GCG)
So, concentrations of alanine, proline, arginine, and serine are equal and each in
concentrations that are one-quarter of the concentration of glycine. The specific
names of the amino acids are not in scope.
D. These are the stop codons and do not encode amino acids.

********************************************************
**************************************************335***

15.2 Prokaryotic Transcription


53 The yeast life cycle is usually dominated by haploid cells, each with a single set of
unpaired chromosomes. The cell propagates asexually, and the genetic material is
replicated through mitosis. Cell division occurs every 2–4 h, leading to 60–100 generations
in a single day. Yeast also reproduces sexually, particularly under adverse environmental
conditions. When two haploid cells—with DNA containing complementary mating-type
alleles—conjugate, a diploid zygote results. The diploid zygote can then complete the
sexual segment of the life cycle through meiosis. After meiosis, four haploid spores are
produced, which can germinate.
Researchers can grow yeast easily on nutrient-containing plates. Because both asexual
and sexual reproduction is rapid, yeast has become an important organism for the
experimental investigation of mutagenesis and evolution among eukaryotes.
Environmental factors, such as chemicals or radiation, induce mutations. High-energy UV-
c radiation of less than 1 min in duration will result in many mutated yeast cells. UV-c can
be used to mutate a strain of yeast in which the synthesis of adenine is blocked. This
mutation is observable because the ade-2 mutant has a red color when cultured on
nutrient-containing plates. Exposure to UV-c also can result in additional mutations. In
particular, one mutant, ade-7, changes the color of the ade-2 mutant to white.
A. You have a UV-c lamp, culture plates, and growth chambers at 23 °C and 37 °C. You
also have available known haploid strains that are (ade-2,+,+), where + denotes the wild
type. Design a plan to determine the rate of UV-c-induced mutations in nutrient-
containing plates inoculated with yeast.
Earth's ozone layer removes high-energy ultraviolet radiation, UV-c, from the solar
radiation received at the surface. Lower-energy ultraviolet radiation, UV-b, strikes Earth’s
surface. Damage to DNA induced by ultraviolet radiation occurs with the formation of
bonds between an adjacent pair of pyrimidine nucleotides, thymine and cytosine, on the
same strand of DNA. A repair enzyme, photolyase, which is activated by visible light, is
present in plants and most animals, but not in humans. In characterizing the relationship
between environmental mutagens and cell damage, a useful assumption is often made
and referred to as the linear hypothesis. This assumption states that the extent of damage
is proportional to the amount of radiation received.
Mutation rates for a strain (preac) that does not produce photolyase and a wild-type (+)
strain were studied. Cultures of the two strains of yeast were diluted, and nutrient-
containing plates were inoculated in triplicate at 23 °C. The plates were exposed to bright
sunlight for varying time intervals. After exposure, the plates were incubated in the dark
at 23 °C. After incubation between 1 and 8 h, data shown in the table were collected by
counting the density of living cells relative to the control, and averaging these among
replicates.
B. Using the data table, graph the average survival fraction, relative to the wild-type
control. Predict the number of mutations in a sample of 1,000 cells of the preac type that
are exposed to bright sunlight for 15 s.

********************************************************
**************************336***************************

Incubation 10-s 20-s 30-s 40-s 50-s


Time (h) Exposure Exposure Exposure Exposure Exposure
1 0.83 0.58 0.33 0.17 0.08
2 1.00 0.43 0.17 0.09 0.04
3 0.92 0.38 0.12 0.03 0.01
4 0.75 0.35 0.08 0.01 0.00
5 0.99 0.49 0.11 0.01 0.00
6 0.81 0.42 0.12 0.01 0.00
7 0.80 0.32 0.09 0.01 0.00
8 1.05 0.59 0.11 0.01 0.00
Mean 0.89 0.45 0.14 0.04 0.02
Standard 0.11 0.10 0.08 0.06 0.03
Deviation
Yeast can also be used to study sexual reproduction, a somewhat puzzling phenomenon.
Cloning of cells through mitosis is molecularly much less complex than meiosis, consumes
less energy, and is less risky. Two alternative explanations for the evolution of sexual
reproduction are popular. In one model, through assortment of genes, meiosis leads to an
increase in the frequency of beneficial mutations. In the second model, detrimental
mutations are purged from a population through sex. Studies using yeast (Gray and
Goddard, Evol. Biol., 2012 and McDonald et al., Nature, 2012) have provided a mechanism
to study these models. The graph shows the fitness (defined as the log of the ratio of the
number of cells in successive generations) of yeast as a function of number of mitotic
reproductions in yeast grown in low- and high-stress environments, and with and without
alternating induction of sexual reproduction.

********************************************************
**************************************************335***

C. Based on these data, evaluate the merits of the alternative theories of the adaptive
advantage provided by sexual reproduction.
Solution Sample answer:
A. The main point should be that population measurements at multiple times after
inoculation should be made with replicas at each time. Also, low to high dilutions
will allow less uncertainty in the counts. Measurements of population density could
count colonies or use a hemocytometer. It is essential that measurements of
number are made. Triplets for each dilution are exposed to UV-c light for intervals
such as 0, 10, 20, 30, 40, and 50 s. After exposure, each is incubated at 23° in the
dark. Students should choose low temperature so yeasts will avoid sexual
reproduction.
At intervals of time (guided by the statement that cloning occurs every few hours)
such as 1, 2, 3, 4, 5 h, and so on, the plates are removed from the incubator and
counts are made. In this first experiment, we are counting the number of cells that
are red. Mutations in the ade-2 will leave the cell white. Because we have a control
and will be forming a fraction relative to the control Nred, exposed and
mutated/Nred, no exposure we count either red or non-red.
Students who have done this or a similar lab may know that a mutation in ade-4 will
counteract the effect of ade-7. They might then go on to propose a test to confirm
that the mutation was at ade-2. In this case, after the plates have been used to
obtain population density, a test cross should be made with (ade-2, +, ade-7) and
perhaps also with (+, ade-4, +). This can be done by streaking a colony of non-red
cells on a line down the middle of the plate. Then a test strain should be streaked

********************************************************
**************************336***************************

onto the plate in a series of lines perpendicular to the first streak. Where the streaks
nearly intersect, there will be a region of sexually reproduced cells after they are
shifted to incubation at the higher temperature. Observation of incubated cells is
made over the next two days. If the products of sexual reproduction of the (ade-2, +,
ade-7) test are red, we know that the ade-2 allele was mutated. If the products of
sexual reproduction of the (+, ade-4, +) test are red, we know that the ade-7 allele
was mutated.
To determine the mutation rate, the ratio to the control is formed. This ratio should
be approximately constant among samples taken at different times during
incubation after exposure. Taking an average of this ratio (and standard deviation)
and plotting that average against the period of exposure yields an approximately
straight line (or so we assume) whose slope is the mutation rate in units of
B. This task is going to be daunting for students with weak backgrounds in math, or
who are unfamiliar with graphing programs. Excel produced the graph shown below.
Students should also have labels on the axes.

C. These results show that under low-stress conditions, there is no additional gain in
fitness due to sexual reproduction. However, under stress, there is a dramatic gain in
fitness.

********************************************************
**************************************************335***

15.3 Eukaryotic Transcription


54 A. Describe the storage and retrieval of genetic information with the following model. Use
the list to fill in the blanks with the letter corresponding to the correct term.
A. amino acid F. translation
B. tRNA G. protein
C. DNA H. RNA polymerase
D. transcription I. rRNA
E. mRNA
Within the cytoplasm, __ is synthesized from __ bound to __ in a sequence that
blank blank blank

corresponds to information provided by __. This process is called __.


blank blank

Within the nucleus, information originating in __ is encoded as a sequence of bases in __,


blank blank

which is synthesized by the enzyme __. This process is called __.


blank blank

B. During development, cell differentiation occurs, and the expression of genes is


permanently switched off. Using the model in (A), explain where information flow is most
effectively blocked.
C. A chemical message is received by the cell regulating the timing of events controlled by
gene expression. Using the model in (A), explain where information flow is most
effectively managed.
Solution Sample answer:
A. Within the cytoplasm G is synthesized from A bound to B in a sequence that
corresponds to the information provided by E. This process is called F.
Within the nucleus, information originating in C is encoded as a sequence of bases in
I that is synthesized by the enzyme H. This process is called D.
B. Permanently blocking gene expression is accomplished by the irreversible
modification of DNA, stopping transcription. This is often accomplished by
methylation, a concept that is out-of-scope for the AP Biology Exam.
C. Posttranscriptional regulation is most efficient since the feedback loop has signals
that vary in time, and the already transcribed information is available quickly.
55 Structure and function in biology result from both the presence of genetic information
and the expression of that information. Some genes are continually expressed, whereas
the expression of most genes is regulated, commonly at the level of transcription. At the
initiation of transcription, the TATA-binding protein (TBP) provides access to the DNA
strand to be transcribed. The 5ʹTATAAA3’ sequence called the TATA box is found in
prokaryotes, archaebacteria, and eukaryotes. Even among eukarya, when the TATA box is
not present among eukaryotes, the initiation of transcription involves TBP. Scientists
attribute this common characteristic to the relative thermostability of the A-T interaction.
Hydrogen bonds hold the two strands of the DNA double helix together. This type of bond

********************************************************
**************************336***************************

has the smallest interaction energy of all intermolecular forces; as temperature increases,
these bonds are broken.
A. Explain the advantage, in terms of the energy required, which is provided by an AT-rich
region in the sequence where transcription is initiated.
B. The fact that the TATA box or the associated TBP are common to all domains provides
evidence of common ancestry among all life. Pose a scientific question that would need
to be addressed by a valid alternative explanation of this fact.
C. A whole-genome survey of prokaryotes (Zheng and Wu, BMC Bioinformatics, 2010)
showed that the relative amounts of guanine and cytosine in DNA poorly predicted the
temperature range conditions that are suitable for an organism. Refine the question
posed in (B), taking this result into account.
Solution Sample answer:
A. Because A and T interact with only two hydrogen bonds (rather than three in the
C-G interaction), the energy required to open the helix is smaller in the AT-rich
region.
B. Is the difference between the A-T and G-C interactions large enough that the
TATA box would be the outcome for any DNA structure?
C. How can we know if an initiation region with CG-rich regions could not transcribe
DNA? This result shows that the difference between CG-rich and AT-rich regions is
small enough not to provide a robust prediction of temperature range. So the
difference should not be expected to be sufficient to allow multiple evolutionary
lines to all adopt the TATA box solution. There must be a common ancestor.
15.5 Ribosomes and Protein Synthesis
56 Only a fraction of DNA encodes proteins. The noncoding portion of a gene is referred to as
the intron. The intron fraction depends upon the gene. Introns are rare in prokaryotic and
mitochondrial DNA; in human nuclear DNA, this fraction is about 95 percent. The intron is
transcribed into mRNA, but this noncoding mRNA is edited out before translation of the
coding portion, or exon, of a gene. The edited exon segments are then spliced together by
a spliceosome, a very large and complex collection of RNAs and proteins.
Although introns do not encode proteins, they have functions. In particular, they amplify
the expression of the exon, although the mechanism is unknown. When introns are very
long, which is common among mammalian genes with roles in development, they can
significantly extend the time required to complete transcription. Analysis of genes
common to different plant and animal species shows many shared intronic positions and
base sequences, although in some organisms, such as yeast, many introns have been
deleted. Because introns do not encode proteins, mutations can remain silent and
accumulate.
A. Introns are ancestral remnants that are replicated because they do not disadvantage
the organism. Consider the claim that introns are “junk DNA.” Evaluate the claim with

********************************************************
**************************************************335***

supporting evidence.
B. Introns may be retained during transcription. Explain how the retention of a
transcribed intron between two transcribed exons within a gene could do the following:
 Block expression of one polypeptide sequence.
 Increase expression of a polypeptide.
 Alter the polypeptide expressed.
Solution Sample answer:
A. The fact that such a significant burden is imposed and the risk (such as splicing
errors) is so great imply that there has to be an advantage in maintaining the
noncoding sequences. Reasonable arguments can be made that the introns are just
artifacts and have been for the four decades since the discovery of the intron. The
assessment here is of the student’s ability to provide supporting evidence. This
evidence is summarized in the stem.
B. Consider a linear sequence with exons on either side of a retained intron. If the
intron contains a stop code the second exon will not translate to protein. If both
exons encode the same protein, expression will be increased. If the intron has a
length that is not a multiple of three, it can cause the mRNA to be translated with
nonfunctional proteins at either or both of the exons.

********************************************************
**************************360************************

16 | GENE REGULATION
REVIEW QUESTIONS
1 Control of gene expression in eukaryotic cells occurs at which level(s)?
A Only the transcriptional level
B Epigenetic and transcriptional levels
C Epigenetic, transcriptional, and translational levels
D Epigenetic, transcriptional, translational, and post-translational levels
Solution The solution is (D). Eukaryotes exhibit a tightly regulated mechanism of gene
expression. Control of gene expression in eukaryotic cells occurs at the epigenetic,
transcriptional, translational, and post-translational levels.
2 What do Figures X and Y in the figure illustrate?

A Transcription and translation in a eukaryotic cell (Figure X) and a prokaryotic cell


(Figure Y)
B Transcription and translation in a prokaryotic cell (Figure X) and a eukaryotic cell
(Figure Y)

********************************************************
***********************************************359***

C Transcription in a eukaryotic cell (Figure X) and translation in a prokaryotic cell


(Figure Y)
D Transcription in a prokaryotic cell (Figure X) and translation in a eukaryotic cell
(Figure Y)
Solution The solution is (B). Figure X depicts coupled transcription and translation in
prokaryotes. Eukaryotes show compartmentalization within their cells; therefore,
transcription occurs inside the nucleus and translation occurs in the cytosol. Figure Y
shows transcription and translation in eukaryotes.
3 If glucose is absent but lactose is present, the lac operon will be —
A activated
B repressed
C partially activated
D mutated
Solution The solution is (A). Lactose acts as an inducer of the lac operon. When glucose is
present, catabolite repression occurs and the lactose operon is repressed.
4 What would happen if the operator sequence of the lac operon contained a mutation that
prevented the repressor protein from binding the operator?
A In the presence of lactose, the lac operon will not be transcribed.
B In the absence of lactose, the lac operon will be transcribed.
C The cAMP-CAP complex will not increase RNA synthesis.
D The RNA polymerase will not bind the promoter.
Solution The solution is (B). If the repressor cannot bind to the operator, the lac operon will
be transcribed when lactose is absent, since the RNA polymerase can bind to the
promoter and initiate transcription.
5 What would happen if the operator sequence of the trp operon contained a mutation that
prevented the repressor protein from binding to the operator?
A In the absence of tryptophan, the genes trpA–E will not be transcribed.
B In the absence of tryptophan, only genes trpE and trpD will be transcribed.
C In the presence of tryptophan, the genes trpA–E will be transcribed.
D In the presence of tryptophan, the trpE gene will not be transcribed.
Solution The solution is (C). If the operator sequence of the trp operon contained a mutation
that prevented the repressor protein from binding to the operator, constitutive
expression of the trp operon would occur regardless of the presence or absence of
the tryptophan in the medium.

********************************************************
**************************360************************

6 What are epigenetic modifications?


A Addition of reversible changes to histone proteins and DNA
B Removal of nucleosomes from the DNA
C Addition of more nucleosomes to the DNA
D Mutation of the DNA sequence
Solution The solution is (A). Epigenetic modifications do not change the sequence of the DNA
and only bring about reversible changes in histone, proteins, and DNA.
7 Which statement about epigenetic regulation is false?
A Histone protein charge becomes more positive when acetyl groups are added.
B DNA molecules are modified within CpG islands.
C Methylation of DNA and histones causes nucleosomes to pack tightly together.
D Histone acetylation results in the loose packing of nucleosomes.
Solution The solution is (A). Acetylation has the effect of changing the overall charge of the
histone tail from neutral to negative. Acetylation disrupts the association between
nucleosomes and DNA, leading to weaker binding of the nucleosomal components.
8 What is true of epigenetic changes?
A They only allow gene expression.
B They allow movement of histones.
C They change the DNA sequence.
D They are always heritable.
Solution The solution is (B). Epigenetic modifications result in the movement of histones to
open or close a chromosomal region. Open chromosomal regions can be
transcribed, while closed chromosomal regions cannot be transcribed.
9 The binding of what is required for transcription start?
A A protein
B DNA polymerase
C RNA polymerase
D A transcription factor
Solution The solution is (C). The binding of RNA polymerase is required for transcription
initiation.

********************************************************
***********************************************359***

10 What would be the outcome of a mutation that prevented DNA-binding proteins from
being produced?
A Decreased transcription because transcription factors would not bind to transcription-
binding sites
B Decreased transcription because enhancers would not be able to bind to transcription
factors
C Increased transcription because repressors would not be able to bind to promoter
regions
D Increased transcription because RNA polymerase would be able to increase binding to
promoter regions
Solution The solution is (B). Enhancer regions are the binding sites for the transcription
factors. When a DNA-binding protein binds to DNA, the shape of the DNA changes.
This shape change allows activators bound to the enhancer regions to interact with
transcription factors bound to the promoter region and the RNA polymerase.
11 What will result from the binding of a transcription factor to an enhancer region?
A Decreased transcription of an adjacent gene
B Increased transcription of a distant gene
C Alteration of the translation of an adjacent gene
D Initiation of the recruitment of RNA polymerase
Solution The solution is (B). Enhancers are the DNA sequences that influence the rate of
transcription by up-regulating the gene expression.
12 What is involved in post-transcriptional control?
A Control of RNA splicing
B Ubiquitination
C Proteolytic cleavage
D Phosphorylation
Solution The solution is (A). Post-transcriptional control includes the control of RNA splicing
after transcription.
13 Gene A is thought to be associated with color blindness. The protein corresponding to
gene A is isolated. Analysis of the protein recovered shows there are actually two
different proteins that differ in molecular weight that correspond to gene A.
What is one reason that two proteins may correspond to the gene?

********************************************************
**************************360************************

A One protein had a 5’ cap and a poly-A tail in its mRNA, and the other protein did not.
B One protein had a 5’ UTR and a 3’ UTR in its RNA, and the other protein did not.
C The gene was alternatively spliced.
D The gene produced mRNA molecules with differing stability.
Solution The solution is (C). The alternative splicing of any gene can lead to the formation of
proteins varying in their molecular weights.
14 Binding of an RNA-binding protein will change the stability of the RNA molecule in
what way?
A Increase
B Decrease
C Neither increase nor decrease
D Either increase or decrease
Solution The solution is (D). Binding of an RNA-binding protein (RBP) will either increase or
decrease the stability of the RNA molecule depending on the specific RBP that binds.
15 A mutation in the 5’UTR that prevents any proteins from binding to the region will —
A increase or decrease the stability of the RNA molecule
B prevent translation of the RNA molecule
C prevent splicing of the RNA molecule
D increase or decrease the length of the poly-A tail
Solution The solution is (A). The binding of RBP’s to the 5′UTR can increase or decrease the
stability of an RNA molecule, depending on the specific RBP that binds. Any
mutation in the 5′UTR can increase or decrease the stability of the RNA molecule.
16 What can post-translational modifications of proteins affect?
A mRNA splicing
B 5’ capping
C 3’ polyadenylation
D Chemical modifications
Solution The solution is (D). Chemical modifications occur post-translationally and can affect
protein function.

********************************************************
***********************************************359***

17 A mutation is found in eIF-2, which impairs the initiation of translation. The mutation
could affect all but one of the following functions of eIF-2.
Which function would NOT be affected?
A The mutation prevents eIF-2 from binding to RNA.
B The mutation prevents eIF-2 from being phosphorylated.
C The mutation prevents eIF-2 from binding to GTP.
D The mutation prevents eIF-2 from binding to the 40S ribosomal subunit.
Solution The solution is (B). When eIF-2 is wildtype, it does not usually undergo
phosphorylation and translation occurs. Mutation can lead to phosphorylation of
eIF-2. Phosphorylated eIF-2 undergoes a conformational change and cannot bind to
GTP. Therefore, the initiation complex cannot form properly and translation cannot
occur.
18 What does the addition of an ubiquitin group to a protein do?
A Increases the stability of the protein
B Decreases translation of the protein
C Increases translation of the protein
D Marks the protein for degradation
Solution The solution is (D). The addition of an ubiquitin group to a protein marks the protein
for degradation.
19 What are cancer-causing genes called?
A Transformation genes
B Tumor suppressor genes
C Oncogenes
D Proto-oncogenes
Solution The solution is (C). Cancer-causing genes are called oncogenes.
20 Targeted therapies are used in patients with a certain gene expression pattern. A targeted
therapy that prevents the activation of the estrogen receptor in breast cancer would be
beneficial to what type of patient?
A Patients who express the EGFR receptor in normal cells
B Patients with a mutation that inactivates the estrogen receptor
C Patients with over-expression of ER alpha in their tumor cells
D Patients with over-expression of VEGF, which helps in tumor angiogenesis
Solution The solution is (C). A targeted therapy can prove to be beneficial for patients
showing over-expression of estrogen receptors.

********************************************************
**************************360************************

21 In a new cancer treatment, a cold virus is genetically modified so that it binds to, enters,
and is replicated in cells, causing them to burst. The modified cold virus cannot replicate
when wildtype p53 protein is present in the cell.
How does this treatment treat cancer without harming healthy cells?
A The modified virus only infects and enters cancer cells.
B The modified virus replicates in normal and cancer cells.
C The modified virus only infects and enters normal cells.
D The modified virus replicates only in cancer cells.
Solution The solution is (D). The treatment can treat cancer when the modified virus
replicates only in cancer cells.
22 A drug designed to switch silenced genes back on in cancer cells would result in what?
A Methylation of DNA and deacetylation of histones
B Methylation of DNA and acetylation of histones
C Deacetylation of DNA and methylation of histones
D Acetylation of DNA and demethylation of histones
Solution The solution is (A). Preventing methylation of DNA and acetylation of histones can
switch on the silenced genes.
23 What are positive cell-cycle regulators that can cause cancer when mutated called?
A Transformation genes
B Tumor suppressor genes
C Oncogenes
D Mutated genes
Solution The solution is (C). Positive cell-cycle regulators that can cause cancer when mutated
are called oncogenes.

CRITICAL THINKING QUESTIONS


24 What best distinguishes prokaryotic and eukaryotic cells?
A Prokaryotes possess a nucleus whereas eukaryotes do not, but eukaryotes show
greater compartmentalization that allows for greater regulation of gene expression.
B Eukaryotic cells contain a nucleus whereas prokaryotes do not, and eukaryotes show
greater compartmentalization that allows for greater regulation of gene expression.

********************************************************
***********************************************359***

C Prokaryotic cells are less complex and perform highly regulated gene expression,
whereas eukaryotes perform less-regulated gene expression.
D Eukaryotic cells are more complex and perform less-regulated gene expression,
whereas prokaryotic cells perform highly regulated gene expression.
Solution The solution is (B). Eukaryotic cells contain a nucleus whereas prokaryotes do not,
allowing greater regulation of gene expression in eukaryotes.
25 Which statement is correct regarding the distinction between prokaryotic and eukaryotic
gene expression?
A Prokaryotes regulate gene expression at the level of transcription, whereas eukaryotes
regulate at multiple levels including epigenetic, transcriptional, and translational.
B Prokaryotes regulate gene expression at the level of translation, whereas eukaryotes
regulate at the level of transcription to manipulate protein levels.
C Prokaryotes regulate gene expression with the help of repressors and activators,
whereas eukaryotes regulate expression by degrading mRNA transcripts, thereby
controlling protein levels.
D Prokaryotes control protein levels using epigenetic modifications, whereas eukaryotes
control protein levels by regulating the rate of transcription and translation.
Solution The solution is (A). Because prokaryotes perform transcription and translation at the
same time, they regulate gene expression at the transcription level whereas
eukaryotes regulate gene expression at multiple levels.
26 All the cells of one organisms share the genome. However, during development, some
cells develop into skin cells while others develop into muscle cells. How can the same
genetic instructions result in two different cell types in the same organism? Thoroughly
explain your answer.
Solution Different genetic programs are turned on or off when cells differentiate into
different cell types (e.g. skin cells, muscle cells, etc.) As a result, cells express genes
needed for the tissue in which they are located.

27 Which statement describes prokaryotic transcription of the lac operon?


A When lactose and glucose are present in the medium, transcription of the lac operon
is induced.
B When lactose is present but glucose is absent, the lac operon is repressed.
C Lactose acts as an inducer of the lac operon when glucose is absent.
D Lactose acts as an inducer of the lac operon when glucose is present.
Solution The solution is (C). Environmental stimuli can increase or induce transcription in
prokaryotic cells. In this example, lactose in the environment will induce the

********************************************************
**************************360************************

transcription of the lac operon, but only if glucose is not available in the
environment.
28 The lac operon consists of regulatory regions such as the promoter as well as the
structural genes lacZ, lacY, and lacA, which code for proteins involved in lactose
metabolism. What would be the outcome of a mutation in one of the structural genes of
the lac operon?
A Mutation in structural genes will stop transcription.

B Mutated lacY will produce an abnormal galactosidase protein.

C Mutated lacA will produce a protein that will transfer an acetyl group to
galactosidase.
D Transcription will continue but lactose will not be metabolized properly.
Solution The solution is (D). A mutation in one of the structural genes of the lac operon will
not prevent transcription of the operon. However, depending on the type of
mutation in the gene, an abnormal protein may be produced, which could prevent
metabolism of lactose.
29 In some diseases, alteration to epigenetic modifications turns off genes that are
normally expressed. Hypothetically, how could you reverse this process to turn these
genes back on?
Solution Sample answer: To turn these genes back on, you would have to reverse the
epigenetic modifications. For example, if the genes are turned off due to
methylation, removing the methyl groups should turn the genes back on.
30 Flowering Locus C (FLC) is a gene that is responsible for flowering in certain plants.
FLC is expressed in new seedlings, which prevents flowering. Upon exposure to cold
temperatures, FLC expression decreases and the plant flowers. FLC is regulated
through epigenetic modifications.
What type of epigenetic modifications is present in new seedlings and after cold
exposure?
A In new seedlings, histone acetylations are present; upon cold exposure, methylation
occurs.
B In new seedlings, histone deacetylations are present; upon cold exposure, methylation
occurs.
C In new seedlings, histone methylations are present; upon cold exposure, acetylation
occurs.
D In new seedlings, histone methylations are present; upon cold exposure, deacetylation
occurs.

********************************************************
***********************************************359***

Solution The solution is (A). Methylation of DNA causes nucleosomes to pack tightly together.
Transcription factors cannot bind the DNA, and genes are not expressed. Histone
acetylation results in loose packing of nucleosomes. Transcription factors can bind
the DNA, and genes are expressed. Since the FLC gene is expressed in new seedlings,
the corresponding DNA likely has histone acetylation. Since gene expression of the
FLC gene decreases upon cold exposure, the corresponding DNA is likely methylated
in response to cold temperatures.
31 A mutation within the promoter region can alter gene transcription. How can this
happen?
A Mutated promoters decrease the rate of transcription by altering the binding site for
the transcription factor.
B Mutated promoters increase the rate of transcription by altering the binding site for
the transcription factor.
C Mutated promoters alter the binding site for transcription factors to increase or
decrease the rate of transcription.
D Mutated promoters alter the binding site for transcription factors and thereby cease
transcription of the adjacent gene.
Solution The solution is (C). A mutation in the promoter region can change the binding site
for a transcription factor that normally binds to increase transcription. The mutation
could either decrease the ability of the transcription factor to bind, thereby
decreasing transcription, or it can increase the ability of the transcription factor to
bind, thus increasing transcription.
32 What could happen if a cell had too much of an activating transcription factor present?
A The transcription rate would increase, altering cell function.
B The transcription rate would decrease, inhibiting cell functions.
C The transcription rate decreases due to clogging of the transcription factors.
D The transcription rate increases due to clogging of the transcription factors.
Solution The solution is (A). If too much of an activating transcription factor were present,
then transcription would be increased in the cell. This could lead to dramatic
alterations in cell function.
33 The wnt transcription pathway is responsible for key changes during animal development.
The transcription pathway shown in the figure uses arrows to represent activation and
perpendicular symbols to represent repression of wnt gene products.
Based on the pathway, how would blocking wnt gene expression affect the production
of Bar-1?

********************************************************
**************************360************************

Solution Sample answer: Bar-1 production would decrease. If wnt production is blocked, then
MIG-1, LIN-17 are not produced, which results in MIG-5 not being produced either.
When MIG-5 is not produced, PRY-1 is produced because there is no inhibition from
MIG-5. Once PRY-1 is produced, it will repress BAR-1 production.
34 How can RBPs prevent miRNAs from degrading an RNA molecule?
A RBPs can bind first to the RNA, thus preventing the binding of miRNA, which
degrades RNA.
B RBPs bind the miRNA, thereby protecting the mRNA from degradation.
C RBPs methylate miRNA to inhibit its function and thus stop mRNA degradation.
D RBPs direct miRNA degradation with the help of a DICER protein complex.
Solution The solution is (A). RNA-binding proteins (RBP) bind to the RNA and can either
increase or decrease the stability of the RNA. If they increase the stability of the RNA
molecule, the RNA will remain intact in the cell for a longer period of time than
normal. Since both RBPs and miRNAs bind to the RNA molecule, RBP can potentially
bind first to the RNA and prevent the binding of the miRNA that will degrade it.
35 How can external stimuli alter post-transcriptional control of gene expression?
A UV rays can alter methylation and acetylation of proteins.
B RNA-binding proteins are modified through phosphorylation.
C External stimuli can cause deacetylation and demethylation of the transcript.
D UV rays can cause dimerization of the RNA-binding proteins.

********************************************************
***********************************************359***

Solution The solution is (B). External stimuli can modify RNA-binding proteins (i.e., through
phosphorylation of proteins) to alter their activity.
36 Protein modifications can alter gene expression in many ways. How can phosphorylation
of proteins alter gene expression?
A Phosphorylation of proteins can alter translation, RNA shuttling, RNA stability, or post-
transcriptional modification.
B Phosphorylation of proteins can alter DNA replication, cell division, pathogen
recognition, and RNA stability.
C Phosphorylated proteins affect only translation and can cause cancer by altering the
p53 function.
D Phosphorylated proteins affect only RNA shuttling, RNA stability, and post-
translational modifications.
Solution The solution is (A). Because proteins are involved in every stage of gene regulation,
phosphorylation of a protein (depending on the protein that is modified) can alter
accessibility to the chromosome; can alter translation (by altering the transcription
factor binding or function); can change nuclear shuttling (by influencing
modifications to the nuclear pore complex); can alter RNA stability (by binding or not
binding to the RNA to regulate its stability); can modify translation (increase or
decrease); or can change post-translational modifications (add or remove
phosphates or other chemical modifications).
37 Changes in epigenetic modifications alter the accessibility and transcription of DNA. How
could environmental stimuli, such as ultraviolet light exposure, modify gene expression?
A UV rays could cause methylation and deacetylation of the genes that could alter the
accessibility and transcription of DNA.
B UV rays could cause phosphorylation and acetylation of the DNA and histones, which
could alter the transcriptional capabilities of the DNA.
C UV rays could cause methylation and phosphorylation of the DNA bases, which could
become dimerized, rendering no accessibility of DNA.
D UV rays can cause methylation and acetylation of histones, making the DNA more
tightly packed and leading to inaccessibility.
Solution The solution is (A). Environmental stimuli, such as ultraviolet light exposure, can
alter the modifications to the histone proteins or DNA. Such stimuli may change an
actively transcribed gene into a silenced gene by removing acetyl groups from
histone proteins or by adding methyl groups to DNA.
38 New drugs are being developed that decrease DNA methylation and prevent the removal
of acetyl groups from histone proteins. How could these drugs affect gene expression to
help kill tumor cells?

********************************************************
**************************360************************

A These drugs maintain the demethylated and the acetylated forms of the DNA to keep
transcription of necessary genes on.
B The demethylated and the acetylated forms of the DNA are reversed when the
silenced gene is expressed.
C The drug methylates and acetylates the silenced genes to turn them back on.
D Drugs maintain DNA methylation and acetylation to silence unimportant genes in
cancer cells.
Solution The solution is (A). These drugs will keep the histone proteins and the DNA
methylation patterns in the open chromosomal configuration so that transcription is
feasible. If a gene is silenced, these drugs could reverse the epigenetic configuration
to re-express the gene.
39 How can understanding the gene expression pattern in a cancer cell tell you something
about that specific form of cancer?
A Understanding gene expression patterns in cancer cells will identify the faulty genes,
which is helpful in providing the relevant drug treatment.
B Understanding gene expression will help diagnose tumor cells for antigen therapy.
C Gene profiling would identify the target genes of the cancer-causing pathogens.
D Breast cancer patients who do not express EGFR can respond to anti-EGFR therapy.
Solution The solution is (A). Understanding which genes are expressed in a cancer cell can
help diagnose the specific form of cancer. It also can help identify treatment options
for that patient. For example, if a breast cancer tumor expresses EGFR in high
numbers, it might respond to specific anti-EGFR therapy. If that receptor is not
expressed, it would not respond to that therapy.
40 What is personalized medicine? How can it be used to treat cancer?
A Personalized medicines would vary based on the type of mutations and the gene’s
expression pattern.
B The medicines are given based on the type of tumor found in an individual’s body.
C The personalized medicines are provided based only on the symptoms of the patient.
D The medicines tend to vary depending on the severity and the stage of the cancer.
Solution The solution is (A). The design of drugs on the basis of gene expression patterns
within individual tumors is the basis of personalized medicine. With an increased
understanding of gene regulation and gene function, medicines can be designed
specifically to target diseased cells without harming healthy cells. Cancer is a
heterogeneous disease with many different mutations and gene-signaling pathways,
leading to the development and progression of the disease. By identifying the gene
expression patterns in individuals and within individual tumors, treatments can be

********************************************************
***********************************************359***

developed and prescribed to target only those genes and pathways that are
abnormal.

TEST PREP FOR AP® COURSES


41 What is found in both prokaryotes and eukaryotes?
A 3’ poly-A tails
B 5’ caps
C Promoters
D Introns
Solution The solution is (C). In prokaryotic as well as in eukaryotic genes, regulation of
transcription takes place with the help of promoters.
42 The enzyme polyadenylate polymerase catalyzes the addition of adenosine
monophosphate to the 3’ ends of mRNAs to form a poly-A tail. If the enzyme were
blocked so that it could not function, the result would be —
A increased mRNA stability in eukaryotes and decreased mRNA stability in prokaryotes
B decreased mRNA stability in eukaryotes and no effect in prokaryotes
C no effect in eukaryotes and increased mRNA stability in prokaryotes
D no effect in eukaryotes and decreased mRNA stability in prokaryotes
Solution The solution is (B). In eukaryotes, poly-A tails provide mRNA with stability and
protection against random endonucleases. In prokaryotes, this modification is
absent.
43 What are two ways in which gene regulation differs and two ways in which it is similar in
prokaryotes and eukaryotes?
A Prokaryotes show co-transcriptional translation, whereas eukaryotes perform
transcription prior to translation; in both cell types, regulation occurs through the
binding of transcription factors, activators, and repressors.
B Prokaryotes perform transcription prior to translation, whereas eukaryotes show co-
transcriptional translation—that is, the processes occur in the same organelle.

********************************************************
**************************360************************

C Prokaryotes show co-transcriptional translation that is regulated prior to translation,


whereas eukaryotes perform transcription prior to translation that is regulated only at
the level of transcription. In both domains, transcription factors, activators, and
repressors provide regulation.
D Prokaryotes show cotranscriptional translation that occurs in the nucleus whereas
eukaryotes show transcription prior to translation. In both cell types, regulation occurs
using transcription factors, activators, and repressors.
Solution The solution is (A). In prokaryotes, RNA transcription and protein formation occur
almost simultaneously. In eukaryotes, RNA transcription occurs prior to protein
formation and takes place in the nucleus. Translation of RNA to protein occurs in the
cytoplasm. In prokaryotes, gene expression is regulated primarily at the
transcriptional level, for example through operons. In eukaryotes, gene expression is
regulated at many levels (epigenetic, transcriptional, post-transcriptional,
translational, and post-translational). In both prokaryotes and eukaryotes, gene
expression can be regulated through transcription-factor binding of promoters. In
both prokaryotes and eukaryotes, repressors can suppress transcription and
activators can increase transcription in response to external stimuli.

44 Lactose digestion in E. coli begins with its hydrolysis by the enzyme galactosidase.
The gene encoding galactosidase, lacZ, is part of a coordinately regulated operon
containing other genes required for lactose utilization.
Which figure correctly depicts the interactions at the lac operon when lactose is NOT
being utilized?

A D

B
Solution The solution is (D). The correct configuration when lactose is not being utilized is
RNA polymerase on promoter and repressor protein bound to lactose, not bound
to DNA.
45 What would be the result of a mutation in the repressor protein that prevented it from
binding lactose?
A The repressor will bind to lactose when it is removed from the operator.
B The repressor will bind the operator in the presence of lactose.
C The repressor will not bind the operator in the presence of lactose.

********************************************************
***********************************************359***

D The repressor will not bind the operator in the absence of lactose.
Solution The solution is (B). The active repressor normally binds to lactose if it is present. If
the repressor is mutated, then it will not be able to bind with lactose and will, in
turn, bind to the operator site and suppress the operon and RNA synthesis.
46 What type of modification might be observed in the GR gene in all newborn rats?
A The DNA will have many methyl molecules.
B The DNA will have many acetyl molecules.
C The DNA will have few methyl groups.
D The histones will have many acetyl groups.
Solution The solution is (A). DNA, when methylated at many locations, suppresses the
expression of the gene. Therefore, the GR genes would not be expressed in the
newborn rats.
47 What type of modification will be observed in the GR gene in the highly nurtured rats?
A The DNA will have many methyl molecules.
B The DNA will have many acetyl molecules.
C The DNA will have few methyl groups.
D The histones will have few acetyl groups.
Solution The solution is (C). The highly nurtured pups will show a greater amount of GR gene
expression, thereby showing very few methylated molecules in the DNA. The low
methylation would be responsible for the higher expression of the GR gene.
48 The level of transcription of a gene is tested by creating deletions in the gene as shown in
the illustration. These modified genes are tested for their level of transcription: (++)
normal transcription levels; (+) low transcription levels; (+++) high transcription levels.

Which deletion is in an enhancer involved in regulating the gene?

********************************************************
**************************360************************

A Deletion 1
B Deletion 2
C Deletion 3
D Deletion 4
Solution The solution is (C). Deletion 3 is an enhancer involving in regulating the gene.
There are deletions in the gene sequence in Deletion 3 which reduces the
transcription level.
49 In the figure, which deletion is in a repressor involved in regulating the gene?

A Deletion 1
B Deletion 2
C Deletion 3
D Deletion 4
Solution The solution is (A). Deletion 1 is in a repressor, as there is a sudden increase in the
level of transcripts when it is induced. If it were not a repressor, then the level of
transcript would be lower, proving that is it a repressor.
50 The diagram shows different regions (1–5) of a pre-mRNA molecule, a mature-mRNA
molecule, and the protein corresponding to the mRNA.

A mutation in which region is most likely to be damaging to the cell?

********************************************************
***********************************************359***

A Region 1
B Region 2
C Region 3
D Region 5
Solution The solution is (B). Region 2 seems to be encoding a gene. Any mutation in this
region would likely produce a nonfunctional protein, damaging the cell.
51 What do regions 1 and 5 correspond to?

A Exons
B Introns
C Promoters
D Untranslated regions
Solution The solution is (D). The untranslated regions are useful for post-transcriptional
regulation. The 5’UTR (leader sequence) contains the ribosome-complex binding site
and 3’UTR (trailer sequence) contains binding sites for regulatory proteins.
52 What are regions 1 through 5 in the diagram?

A Regions 1, 3, and 5 are exons; regions 2 and 4 are introns.


B Regions 2 and 4 are exons; regions 1, 3, and 5 are introns.
C Regions 1 and 5 are exons; regions 2, 3, and 4 are introns.
D Regions 2, 3, and 4 are exons; regions 1 and 5 are introns.
Solution The solution is (A). Regions 1, 3, and 5 are the exons. These are the DNA regions
encoding a useful gene. In between every exon, larger introns are present,
corresponding to regions 2 and 4 in the diagram.

********************************************************
**************************360************************

53 A mutation results in the formation of the mutated mature-mRNA as indicated in the


diagram. What type of mutation occurred, and what is the likely outcome of the
mutation?

A Mutation in the GU-AG sites of introns produced a nonfunctional protein.


B A transversion mutation in the introns led to alternative splicing, producing a
functional protein.
C A transversion mutation in the GU-AG site mutated this mRNA, producing a
nonfunctional protein.
D Transition mutations in the introns could produce a functional protein.
Solution The solution is (A). The mutation caused a failure in recognition of the intron 2 end
and instead appears to have recognized the end of the next intron (intron 4). This
caused excision of introns 2, 3, and 4. The most likely outcome of this mutation is a
truncated protein that will be nonfunctional.
54 The diagram illustrates the role of p53 in response to UV exposure. What would be the
result of a mutation in the p53 gene that inactivates it?

********************************************************
***********************************************359***

A Skin will peel in response to UV exposure.


B Apoptosis will occur in response to UV exposure.
C No DNA damage will occur in response to UV exposure.
D No peeling of skin will occur in response to UV exposure.
Solution The solution is (D). Mutation in p53 would restrict its function, causing no activation
of p21 and also no apoptosis, leading to no peeling of the skin.
55 What will NOT occur in response to UV exposure if a p53 mutation inactivates the p53
protein?
A Damage to DNA, p53 activation, and p21 activation
B p21 activation and apoptosis
C p21 activation
D p53 activation, p21 activation, and apoptosis
Solution The solution is (C). If p53 inactivates due to a mutation, then p21 will get inactivated
too. As a result, apoptosis will not take place.
56 What happens when tryptophan is present?

A The repressor binds to the operator, and RNA synthesis is blocked.


B RNA polymerase binds to the operator, and RNA synthesis is blocked.
C Tryptophan binds to the repressor, and RNA synthesis proceeds.
D Tryptophan binds to RNA polymerase, and RNA synthesis proceeds.
Solution The solution is (A). In the trp operon, the tryptophan binds to the inactive apo-
repressor and makes it active. This active repressor would bind to the operator site,
blocking RNA synthesis.
57 What happens in the absence of tryptophan?

A RNA polymerase binds to the repressor.


B The repressor binds to the promoter.
C The repressor dissociates from the operator.
D RNA polymerase dissociates from the promoter.
Solution The solution is (C). The absence of tryptophan would inactivate the repressor,
dissociating it from the operator. This apo-repressor now remains in the cell in an
inactive form, allowing the operon to synthesize RNA.

********************************************************
**************************360************************

58 Anabaena is a simple multicellular photosynthetic cyanobacterium. In the absence of


fixed nitrogen, certain newly developing cells along a filament express genes that code for
nitrogen-fixing enzymes and become non-photosynthetic heterocysts. The specialization
is advantageous because some nitrogen-fixing enzymes function best in the absence of
oxygen. Heterocysts do not carry out photosynthesis, but instead provide adjacent cells
with fixed nitrogen and receive fixed carbon and reduced energy carriers in return. As
shown in the diagram, when there is low fixed nitrogen in the environment, an increase in
the concentration of free calcium ions and 2-oxyglutarate stimulates the expression of
genes that produce two transcription factors (NtcA and HetR) that promote the
expression of genes responsible for heterocyst development. HetR also causes production
of a signal, PatS, that prevents adjacent cells from developing as heterocysts.

Based on your understanding of the ways in which signal transmission mediates cell
function, which prediction is most consistent with the information given?
A In an environment with low fixed nitrogen, treating the Anabaena cells with a calcium-
binding compound should prevent heterocyst differentiation.
B A strain that overexpresses the PatS gene should develop many more heterocysts in a
low nitrogen environment.
C In an environment with abundant fixed nitrogen, free calcium levels should be high in
all cells, preventing heterocysts from developing.
D In environments with abundant fixed nitrogen, loss of the HetR gene should induce
heterocyst development.
Solution The solution is (A). As increased calcium stimulates the expression of heterocyst
development genes, providing Anabaena cells with calcium-binding compound
inhibits heterocyst development.

********************************************************
***********************************************359***

59 Which statement about Anabaena is false?

A Decreasing the concentration of free calcium ions will prevent heterocyst


development.
B In the presence of fixed nitrogen, NtcA will not be expressed.
C Low fixed nitrogen levels result in increased PatS levels.
D A mutation in NtcA that makes it nonfunctional also will allow adjacent cells to
develop as heterocysts.
Solution The solution is (D). The NtcA gene is responsible for heterocyst development
whereas the HetR gene promotes PatS gene, which controls the heterocyst
development in adjacent cells.

SCIENCE PRACTICE CHALLENGE QUESTIONS


16.4 Eukaryotic Transcriptional Gene Regulation
60 The operon model describes expression in prokaryotes. Describe this model and the
essential difference in the way in which expression is regulated in eukaryotes.
Solution Sample answer: An operon is a cluster of genes involved in the same biochemical
pathway. The genes are transcribed together, and are all under the control of the
same promoter. In eukaryotes, genes are not clustered into operons, and a different
promoter regulates each gene.
An operon includes structural genes involved in a single biochemical pathway which
are under the same control and transcribed into a single mRNA. Either the operon is
active and all structural genes are transcribed or the operon is off and none of the

********************************************************
**************************360************************

genes are transcribed. Upstream of the structural genes are located the regulatory
sequences that include a promoter where RNA polymerase binds and operators
where repressors bind. The genes encoding the repressor proteins are usually not
part of the operon. Activator sequences can be located upstream of the promoter.
Operons are found in bacteria with few exceptions in some organisms such as yeast
and C. elegans.
In eukaryotic cells, genes are generally transcribed individually. Each gene is
preceded by its own promoter upstream (at the 5′ end) and a transcription
terminator at the 3′ end. Genes encoding proteins involved in the same pathway are
nevertheless expressed individually.

********************************************************
**********************************************************379***

17 | BIOTECHNOLOGY AND GENOMICS


REVIEW QUESTIONS
1 How are GMOs created?
A Introducing recombinant DNA into an organism by any means
B In vitro fertilization methods
C Mutagenesis
D Plant breeding techniques
Solution The solution is (A). Recombinant DNA is DNA that has been genetically modified in
the laboratory. A genetically modified organism (GMO) is created by introducing
recombinant DNA into an organism.
2 Which technique used to manipulate genetic material results in a significant increase in
DNA or RNA fragments?
A Gel electrophoresis
B Nucleic acid extraction
C Nuclear hybridization
D Polymerase chain reaction (PCR)
Solution The solution is (D). PCR is a method used to make many copies of DNA or RNA
fragments from a small number of copies.
3 What is the role of plasmids in molecular cloning?
A They are used to create clones.
B They are used as vectors to insert genes into bacteria.
C They are a functional part of binary fission.
D They contain the circular chromosome of prokaryotic organisms.
Solution The solution is (B). Plasmids are vectors that can be used to insert genes into
bacteria.
4 What is meant by a recombinant DNA molecule?
A Chimeric molecules
B Bacteria transformed into another species
C Molecules that have been through the PCR process
D The result of crossing over during cell reproduction
Solution The solution is (A). Recombinant DNA is a chimeric molecule, as it has been
genetically modified in the laboratory.

********************************************************
**************************380***********************************

5 What is Bt toxin is considered to be?


A A gene for modifying insect DNA
B An organic insecticide produced by bacteria
C A nerve toxin in humans
D A strain of genetically modified tomatoes
Solution The solution is (B). Bt, or Botulinum toxin, is lethal to insects and is, therefore, an
organic insecticide produced by a bacterium.
6 What is one trait of the Flavr Savr Tomato?
A Has a better shelf life
B Is not a variety of vine-ripened tomato in the supermarket
C Was not created to have better flavor
D Undergoes soft rot
Solution The solution is (A). The Flavr Savr Tomato was genetically modified to resist rot and
to ripen more slowly, which gives it a better shelf life.
7 What is first step in isolating DNA?
A Generating genomic DNA fragments with restriction endonucleases
B Introducing recombinant DNA into an organism by any means
C Overexpressing proteins in E. coli
D Lysing the cells in the sample
Solution The solution is (D). The first step in isolating DNA is cell lysis, a process in which the
cell membrane is broken.
8 What is genomics?
A Genomics is the study of entire genomes, including the complete set of genes, their
nucleotide sequence and organization, and their interactions within a species and with
other species.
B Genomics is the process of finding the locations of genes on each chromosome.
C Genomics is an illustration that lists genes and their location on a chromosome.
D Genomics is a genetic marker is a gene or sequence on a chromosome that co-
segregates (shows genetic linkage) with a specific trait.
Solution The solution is (A). Genomics is the study of entire genomes, including the complete
set of genes, their nucleotide sequence and organization, and their interactions
within a species and with other species.

********************************************************
**********************************************************379***

9 What is required in addition to a genetic linkage map to build a complete picture of


the genome?
A A genetic marker
B A physical map
C Linkage analysis of chromosomes
D Plasmids
Solution The solution is (B). A physical map, which shows the arrangement of genes on the
chromosome, is necessary to build a complete picture of the genome.
10 Genetic recombination occurs by which process?
A Crossing over
B Chromosome segregation
C Independent assortment
D Sister chromatids
Solution The solution is (A). Genetic recombination occurs when homologous chromosomes
exchange material in a process called crossing over.
11 Individual genetic maps in a given species are —
A genetically similar
B genetically identical
C genetically dissimilar
D not useful in species analysis
Solution The solution is (A). Individual genetic maps in a given species are genetically similar
but not identical.
12 What procedure uses information obtained by microscopic analysis of stained
chromosomes?
A Cytogenetic mapping
B Radiation hybrid mapping
C RFLP mapping
D sequence mapping
Solution The solution is (A). A cytogenetic map is the visual appearance of a chromosome
that has been stained.

********************************************************
**************************380***********************************

13 What is true about linkage analysis?


A It is used to create a physical map.
B It is based on the natural recombination process.
C It involves breaking and re-joining of DNA artificially.
D It requires radiation hybrid mapping.
Solution The solution is (B). Genes far apart on a chromosome are separated by natural
recombination more frequently than genes that are close together. Linkage analysis
assesses the relative order of genes based on this natural recombination frequency.
14 What does the chain termination method of DNA sequencing use to terminate
polynucleotide elongation?
A Labeled dideoxynucleotides
B Unlabeled dideoxynucleotides
C Labeled deoxynucleotides
D Unlabeled deoxynucleotides
Solution The solution is (A). Chain termination occurs when a dideoxynucleotide is introduced
into the DNA strand. Each dideoxynucleotide is labeled so that the strand can be
visualized.
15 What sequencing technique is used to identify regions of similarity between cell types or
species?
A Dideoxy chain termination
B Proteins, DNA, or RNA sequence alignment
C Shotgun sequencing
D Whole-exome sequencing
Solution The solution is (B). Similarity among cell types or species can be assessed by aligning
the sequence of proteins, DNA, or RNA.
16 Whole-genome sequencing can be used for advances in what field?
A Bioinformatics
B Iron industry
C Multimedia
D The medical field
Solution The solution is (D). Whole genome sequencing yields genetic information that can
lead to medical advances.

********************************************************
**********************************************************379***

17 Sequencing an individual person’s genome —


A is currently impossible
B helps identify genetic mutations associated with certain diseases
C will not lead to legal issues regarding discrimination and privacy
D will not help make informed choices about medical treatment
Solution The solution is (B). Sequencing an individual’s genome yields information about
genetic mutations associated with disease.
18 Genomics can be used in agriculture to do what?
A Generate new hybrid strains
B Improve disease resistance
C Improve yield
D Improve yield, resistance, and generate hybrids
Solution The solution is (D). Genomics yields a wide range of information that can lead to
improved yield, resistance to disease, and the generation of new hybrid strains.
19 What are the uses of metagenomics?
A Identification of biofuels
B Testing for multiple drug susceptibility in a population
C Use in increasing agricultural yields
D Identifying new species more rapidly and analyzing the effect of pollutants on the
environment
Solution The solution is (D). Metagenomics is the study of genetic material recovered from
the environment. This information can be used to identify new species and, over
time, to assess the effect of pollutants on the environment.
20 Genomics can be used on a personal level to do what?
A Determine the risks of genetic diseases for an individual’s children
B Increase transplant rejection
C Predict the career success of a person
D Produce antibodies for an antigen
Solution The solution is (A). Genomics can be used to determine whether an individual has
genes that might cause genetic disease in his or her children.

********************************************************
**************************380***********************************

21 What is the percentage of single gene defects causing disease in developed countries?
A 0.05
B 0.1
C 0.2
D 0.4
Solution The solution is (A). Approximately 0.05, or 5 percent, of single gene defects cause
disease in persons living in developed countries.
22 The rapid identification of new species and the analysis of the effect of pollutants on the
environment is a function of what?
A Metagenomics
B Linkage analysis
C Genomics
D Shotgun sequencing
Solution The solution is (A). Metagenomics involves the study and analysis of genetic material
obtained from the environment.
23 The risks of genetic diseases for an individual’s children can be determined through —
A metagenomics
B linkage analysis
C genomics
D shotgun sequencing
Solution The solution is (C). Genomics involves analyzing genes, which can determine the
chances of genetic diseases appearing in an individual’s children.
24 What is a biomarker?
A The color coding of different genes
B A protein uniquely produced in a diseased state
C A molecule in the genome or proteome
D A marker that is genetically inherited
Solution The solution is (B). A biomarker is a protein only produced when disease is present.
25 What is a metabolome?
A A provisional listing of the genome of a species
B A unique metabolite used to identify an individual

********************************************************
**********************************************************379***

C A method used for protein analysis


D The complete set of metabolites related to the genetic makeup of an organism
Solution The solution is (D). A metabolome is complete set of metabolites related to the
genetic makeup of an organism.
26 How would you describe a set of proteins with altered expression levels?
A A group of biomarkers
B A protein signature
C The result of a defect in mRNA transcription
D The results of crossing over during cell replication
Solution The solution is (B). A protein signature is a unique set of proteins present in a
disease state.
27 What is a protein signature?
A A protein expressed on the cell surface
B A unique set of proteins present in a diseased state
C The path followed by a protein after it is synthesized in the nucleus
D The path followed by a protein in the cytoplasm
Solution The solution is (B). A protein signature is a unique set of proteins present in a
diseased state
28 What describes a protein that is uniquely produced in a diseased state?
A A genomic protein
B A genetic defect
C A chimeric molecule
D A biomarker
Solution The solution is (D). A biomarker is a protein that indicates the occurrence of a
particular disease.
29 What are the metabolites that result from the anabolic and catabolic reactions of an
organism called?
A Genetic metabolic profile
B Metabolic signature
C Metabolome
D Metagenomic
Solution The solution is (C). Metabolome refers to the metabolites that are produced by
catabolic and anabolic biochemical reactions.

********************************************************
**************************380***********************************

CRITICAL THINKING QUESTIONS


30 What is the process of Southern blotting?
A Southern blotting is used to find particular DNA sequences. Fragments are separated
on gel, incubated with probes to check for sequence of interest, and transferred to
nylon membrane.
B Southern blotting is used to find particular DNA sequences. Fragments are separated
on gel, transferred to nylon membrane, and incubated with probes to check for
sequence of interest.
C Southern blotting is used to find particular RNA sequences. Fragments are separated
on gel, transferred to nylon membrane, and incubated with probes to check for
sequence of interest.
D Southern blotting is used to find particular RNA sequences. Fragments are separated
on gel, incubated with probes to check for sequence of interest, and transferred to
nylon membrane.
Solution The solution is (B). Southern blotting is used to find particular DNA sequences.
Fragments are separated on gel, transferred to nylon membrane, and incubated
with probes to check for sequence of interest.
31 A researcher wants to study cancer cells from a patient with breast cancer. Is cloning the
cancer cells an option?
A The cancer cells should be cloned along with a biomarker for better detection
and study.
B The cells should be screened first in order to assure their carcinogenic nature.
C The cancer cells, being the clones of each other already, should directly be grown in a
culture media and then studied.
D The cancer cells should be extracted using the specific antibodies.
Solution The solution is (C). Cancer cells are by definition clones of each other. All
the researcher needs to do is grow the patient’s cell through cell culture and
study them.
32 What are the uses of genome mapping?
A Genome mapping is useful in identifying human disease-causing genes, developing
microbes to clean up pollutants, and increasing crop yield.
B Genome mapping is directly required to produce recombinants, in FISH detection, and
detecting the methylated parts of genetic material.
C Genome mapping is useful for knowing the pedigree of diseases in humans and tracing
the movement of transposons in plants.
D Genome mapping identifies human disease-causing genes only.

********************************************************
**********************************************************379***

Solution The solution is (A). Human genome maps help researchers in their efforts to identify
human disease-causing genes and can be used in a variety of other applications,
such as using live microbes to clean up pollutants or even prevent pollution.
Research involving plant genome mapping may lead to producing higher crop yields
or developing plants that better adapt to climate change.
33 If you had a chance to get your genome sequenced, what are some questions you might
be able to have answered about yourself?
A You can determine the drugs that can rectify the disease, symptoms of the disease,
and its severity.
B You can determine the ancestry and genetic origin of diseases and their susceptibility
to drugs.
C You can predict the symptoms of disease, the vectors to be used in gene therapy, and
the causal organism of the disease.
D You can determine the pedigree of a disease, produce recombinants, and detect the
presence of extracellular genes using FISH.
Solution The solution is (B). It would be possible to determine ancestry, tendency to develop
some diseases that are of genetic origin, or susceptibility to drugs.
34 What is an example of a genomic mapping method?
A The radiation mapping method is an example which uses radiations to break the DNA
and is affected by the change in recombination frequency.
B Cytogenetic mapping obtains information from microscopic analysis of stained
chromosomes. It can estimate the approximate distance between markers.
C In restriction mapping, the DNA fragments are cut by using the restriction enzymes
and then stained fragments are viewed on gel.
D Cytogenetic mapping obtains information from microscopic analysis of stained
chromosomes. It can estimate the exact base pair distance between markers.
Solution The solution is (B). Cytogenetic mapping uses information obtained by microscopic
analysis of stained sections of the chromosome. It is possible to determine the
approximate distance between genetic markers using cytogenetic mapping, but not
the exact distance (number of base pairs).
35 What are three methods of gene sequencing?
A Chain termination method – automated sequencers are used to generate sequences
of short fragments; Shotgun sequencing method – incorporation of ddNTP during DNA
replication; Next-generation sequencing – cutting DNA into random fragments,
sequencing using chain termination, and assembling overlapping sequences
B Chain termination method – incorporation of ddNTP during DNA replication; Shotgun
sequencing method – cutting DNA into random fragments, sequencing using chain

********************************************************
**************************380***********************************

termination, and assembling overlapping sequences; Next-generation sequencing –


automated sequencers are used to generate sequences of short fragments
C Chain termination method – incorporation of ddNTP during DNA replication; Shotgun
sequencing method – automated sequencers are used to generate sequences of short
fragments; Next-generation sequencing – cutting DNA into random fragments,
sequencing using chain termination, and assembling overlapping sequences
D Chain termination method – automated sequencers are used to generate sequences
of short fragments; Shotgun sequencing method – cutting DNA into random
fragments, sequencing using chain termination, and assembling overlapping
sequences; Next-generation sequencing – incorporation of ddNTP during DNA
replication
Solution The solution is (B). The basic sequencing technique used in all modern-day
sequencing projects is the chain termination method (also known as the dideoxy
method), which was developed by Fred Sanger in the 1970s. The chain termination
method involves DNA replication of a single-stranded template with the use of a
primer and a regular dideoxynucleotide (ddNTP), which is a monomer of DNA. In the
shotgun sequencing method, several copies of a DNA fragment are cut randomly
into many smaller pieces (somewhat like what happens to a round shot cartridge
when fired from a shotgun). All of the segments are then sequenced using the chain-
sequencing method. Next-generation sequencing is a group of automated
techniques used for rapid DNA sequencing. These automated, low-cost sequencers
can generate sequences of hundreds of thousands or millions of short fragments (25
to 500 base pairs) in the span of one day.
36 What is the greatest challenge facing genome sequencing?
A The lack of resources and use of chemicals for the sequencing of the DNA fragments
B The ethical issues such as discrimination based on person’s genetics
C The use of chemicals during the sequencing methods could incorporate mutations
D The scientific issues such as conserving the human genome sequences
Solution The solution is (B). The ethical issues surrounding genome sequencing are the
most challenging. Humans have a responsibility to use this knowledge wisely.
Otherwise, it could be easy to misuse the power of such knowledge, leading to
discrimination based on a person's genetics, human genetic engineering, and other
ethical concerns. This information also could lead to legal issues regarding health
and privacy.

********************************************************
**********************************************************379***

37 How is shotgun sequencing performed?


A The DNA is cut into fragments, sequencing is done using chain termination method,
fragments are analyzed to see the overlapping sequences, and the entire fragment is
reformed.
B The DNA is cut into fragments, overlapping sequences are analyzed using computer,
sequencing is done using chain termination method, and the DNA fragment is
reformed.
C The DNA is cut into fragments, stained with fluorescent dye, and sequenced using the
chain termination method; the fragments are analyzed to see the overlapping
sequences; and the entire DNA fragment is reformed.
D The DNA is cut into fragments, sequencing is done using the chain termination
method, the DNA is stained with fluorescent dye, and a computer is used to analyze
and reform the entire DNA fragment.
Solution The solution is (A). In the shotgun sequencing method, several copies of a DNA
fragment are cut randomly into many smaller pieces (somewhat like what happens
to a round shot cartridge when fired from a shotgun). All of the segments are
sequenced using the chain-sequencing method. Then, with the help of a computer,
the fragments are analyzed to see where their sequences overlap. By matching up
overlapping sequences at the end of each fragment, the entire DNA sequence can be
reformed.
38 Coumadin is a drug frequently given to prevent excessive blood clotting in stroke or heart
attack patients, which could lead to another stroke or heart attack. Administration of the
drug also can result in an overdose in some patients, depending on the liver function of a
patient.
How could pharmacogenomics be used to assist these patients?
A Pharmacogenomics could provide a counteracting drug to decrease the effect of
Coumadin.
B Pharmacogenomics could test every patient for their sensitivity to the drug.
C Pharmacogenomics will not be able to provide any help to patients highly sensitive to
the drug.
D Pharmacogenomics could provide an overdose to each patient to test for the
symptoms of the drug.
Solution The solution is (B). Pharmacogenomics allows each patient to be tested for genotype
associated sensitivity to drugs, thereby identifying patients who might experience an
overdose of drugs prior to administration.

********************************************************
**************************380***********************************

39 Why is so much effort being poured into genome mapping applications?


A Genome mapping is necessary to know the base pair difference between the markers.
B The mapping would help scientists understand the role of proteins in specific
organelles.
C The mapping technique identifies the role of transposons.
D Genome mapping helps identify faulty alleles, which could cause diseases.
Solution The solution is (D). A genetic map of the human genome for multiple individuals
could identify alleles of genes susceptible to cancer causing agents. The mapping
could also identify allele variations resistant to changes resulting in cancer, thereby
offering the opportunity for genetic therapy for the disorders.
40 What is the reason for studying mitochondrial genomics that is most directly important
for humans?
A Mitochondria evolved from bacteria; therefore, their genome is important to study.
B Mitochondria undergo rapid mutation; and it is essential that this pattern be studied.
C Mitochondria contain DNA, and it is passed on from mother to the offspring, which
renders it helpful in tracing genealogy.
D Mitochondria are the only ATP-producing organelles of the cell, thus their genome is
important.
Solution The solution is (C). Mitochondria are intracellular organelles that contain their own
DNA. Mitochondrial DNA mutates at a rapid rate and often is used to study
evolutionary relationships. Another feature that makes studying the mitochondrial
genome interesting is mitochondrial DNA in most multicellular organisms only is
passed on from the mother. For this reason, mitochondrial genomics often is used to
trace genealogy.
41 How can proteomics complement genomics?
A The genes are responsible for producing proteins, which implies that proteomics
complements genomics.
B Genomics is responsible for deciding the structure of the proteins and, thereby, the
result of proteomic studies.
C The genome is constant, but proteome is dynamic as different tissues possess the
same genes but express different genes, thereby complementing genomics.
D The study of genes is incomplete without the study of their respective proteins, thus
they complement each other.

********************************************************
**********************************************************379***

Solution The solution is (C). Proteomics complements genomics and is useful when scientists
want to test their hypotheses that were based on genes. Even though all cells of a
multicellular organism have the same set of genes, the set of proteins produced in
different tissues is different and dependent on gene expression. Thus, the genome is
constant, but the proteome varies and is dynamic within an organism.
42 How could a proteomic map of the human genome help find a cure for cancer?
A A genetic map could help identify genes that could counteract the cause of cancer.
B Metabolomics can be used to study the genes producing metabolites during cancer.
C Proteomics detects biomarkers whose expression is affected by the disease process.
D The mapping helps analyze the inheritance of cancer-causing genes.
Solution The solution is (C). Proteomic approaches are being used to improve screening and
early detection of cancer. This is achieved by identifying proteins whose expression
is affected by the disease process. An individual protein is called a biomarker,
whereas a set of proteins with altered expression levels is called a protein signature.
43 What contributions have been made through the use of microbial genomics?
A Microbial genomics has provided various tools to study psychological behaviors of
organisms.
B Microbial genomics has been useful in producing antibiotics, enzymes, improved
vaccines, disease treatments, and advanced cleanup techniques.
C Microbial genomics has contributed resistance in other bacteria by horizontal and
lateral gene transfer mechanisms.
D Microbial genomics has contributed to fighting global warming.
Solution The solution is (B). Microorganisms are used to create products, such as enzymes
that are used in research, antibiotics, and other anti-microbial mechanisms.
Microbial genomics is helping develop diagnostic tools, improved vaccines, new
disease treatments, and advanced environmental cleanup techniques.

TEST PREP FOR AP® COURSES


44 In separating DNA for genomic analysis, it is important to consider RNA contaminating
the sample during the cell lysis step of a DNA extraction, which is likely to cause what
to occur?
A DNA separates into the supernatant.
B The protease destroys the DNA.
C RNA does not affect the DNA.
D DNA combines with the RNA.
Solution The solution is (C). DNA is unaffected by the RNA.

********************************************************
**************************380***********************************

45 There are many techniques for investigating human genomic disorders. Western blotting
looks for protein, eastern blotting looks for post-translational changes, northern blotting
looks at mRNA, and Southern blotting looks at DNA.
If you were to look at sickle cell anemia, a disorder affecting hemoglobin produced in red
blood cells, which technique would be the most useful in detecting a polymorphism in a
sample?
A Northern blotting
B Southern blotting
C Western blotting
D Eastern blotting
Solution The solution is (B). The polymorphism results in change (mutation) in the sequence
of a gene, so the analyses of DNA will be most useful.
46 A population of insects was formally distinguished by a mix of colors on their thorax and
legs. This population now appears to be split into two subgroups, purple legged and
orange legged. Researchers hypothesize that the purple-legged group may be increasing
resistance to the Bt (Bacillus thuringiensis) toxin.
Which idea supports this observation?
A Transgenesis
B Natural selection
C Hybridization
D Recombination
Solution The solution is (B). The resistance to certain environmental pressure is result
of mutations that produce organisms able to survive and reproduce, transmitting the
favorable trait, which in turn will increase the number of organisms carrying
the trait.
47 Which statement describes the process of molecular cloning?
A The foreign DNA and plasmid are cut with the same restriction enzyme, and DNA is
inserted within the lacZ gene, whose product metabolizes lactose. The foreign DNA
and vector are allowed to anneal. The vector is transferred to a bacterial host that is
ampicillin sensitive, and those with a disrupted lacZ gene show an inability to
metabolize X-gal.
B The foreign DNA and plasmid are denatured using high heat, and DNA is inserted
within the lacZ gene, whose product metabolizes glucose. The foreign DNA and vector
are allowed to anneal. The vector is transferred to a bacterial host that is ampicillin
sensitive, and the disrupted lacZ gene will metabolize X-gal.

********************************************************
**********************************************************379***

C The foreign DNA and plasmid are cut with the same restriction enzyme, and DNA is
inserted randomly in the plasmid. The foreign DNA and vector are allowed to anneal.
The vector is transferred to a bacterial host that is ampicillin sensitive, and the
disrupted lacZ gene shows an inability to synthesize X-gal.
D The foreign DNA and plasmid are cut with the same restriction enzyme, and DNA is
inserted within the lacZ gene, whose product metabolizes lactose. The foreign DNA
and vector are allowed to anneal. The vector is transformed into a viral host that is
ampicillin sensitive, and the disrupted lacZ gene shows an inability to synthesize X-gal.
Solution The solution is (A). The foreign DNA and plasmid are cut with the same restriction
enzyme, which recognizes a particular sequence of DNA called a restriction site. The
restriction site occurs only once in the plasmid, and is located within the lacZ gene, a
gene necessary for metabolizing lactose. The restriction enzyme creates sticky ends,
which allow the foreign DNA and cloning vector to anneal. Ligase, an enzyme, binds
the annealed fragments together. The ligated cloning vector is transformed into a
bacterial host strain that is ampicillin sensitive and missing the lacZ gene from its
genome. Bacteria are grown in a medium containing ampicillin and X-gal, a chemical
metabolized by the same pathway as lactose. The ampicillin kills bacteria without
the plasmid. Plasmids lacking the foreign insert have an intact lacZ gene and are able
to metabolize X-gal, releasing a dye that turns blue. Plasmids with an insert have a
disrupted lacZ gene and produce white colonies.
48 There are three methods of creating maps to evaluate genomes: cytogenetic (staining
chromosomes), radiation hybrid maps (fragments with X-rays), and sequence maps
(comparing DNA sequences). Which option accurately describes the three methods?
A Cytogenetic mapping – stained sections of chromosomes are analyzed using
microscope, the distance between genetic markers can be found; Radiation hybrid
mapping – breaks DNA using radiation and is affected by recombination frequency;
Sequence mapping – DNA sequencing technology used to create physical maps
B Cytogenetic mapping – stained sections of chromosomes are analyzed using
microscope, the approximate distance between genetic markers can be found;
Radiation hybrid mapping – breaks DNA using radiation and is unaffected by
recombination frequency; Sequence mapping – DNA sequencing technology used to
create physical maps
C Cytogenetic mapping – stained sections of chromosomes are analyzed using
microscope, the distance in base pairs between genetic markers can be found;
Radiation hybrid mapping – breaks DNA using radiation and is unaffected by
recombination frequency; Sequence mapping – DNA sequencing technology used to
create physical maps
D Cytogenetic mapping – stained sections of chromosomes are analyzed using a
telescope, the distance between genetic markers can be found; Radiation hybrid
mapping – breaks DNA using radiation and is affected by recombination frequency;
Sequence mapping – DNA sequencing technology used to create physical maps

********************************************************
**************************380***********************************

Solution The solution is (B). Cytogenetic mapping uses information obtained by microscopic
analysis of stained sections of the chromosome. It is possible to determine the
approximate distance between genetic markers using cytogenetic mapping, but not
the exact distance (number of base pairs). Radiation hybrid mapping uses radiation,
such as X-rays, to break the DNA into fragments. The amount of radiation can be
adjusted to create smaller or larger fragments. This technique overcomes the
limitation of genetic mapping and is not affected by increased or decreased
recombination frequency. Sequence mapping resulting from DNA sequencing
technology allowed for the creation of detailed physical maps with distances
measured in terms of the number of base pairs.
49 How many cells with different genetic variations are possible after a single round of
meiosis?
A Two
B Three
C Four
D Eight
Solution The solution is (C). Complete meiosis produces four gametes which are genetically
different due to events in meiosis I (recombination during prophase I, and the
random alignment of the homologous chromosomes during metaphase I).

SCIENCE PRACTICE CHALLENGE QUESTIONS


17.1 Biotechnology
50 Prokaryotes have an adaptive strategy to identify and respond to viral infections. This
strategy uses segments of the cyclic DNA called CRISPRs and genes coding for CRISPR-
associated (cas) proteins. When a virus enters the cell, a strand of the viral DNA is excised
by a cas protein and inserted into the bacterial DNA in a CRISPR region. When the same
viral DNA is encountered subsequently, this foreign DNA is targeted by cas proteins that
carry RNA markers transcribed from the inserted segment. The cas proteins cleave the
viral DNA. The bacteria “remember” the infectious agent, providing a form of immunity.
A. Use the diagram to identify the components of a transcript-based response of bacteria
to the presence of viral DNA by placing the corresponding number next to each feature of
the diagram.

blank

********************************************************
**********************************************************379***

___ viral DNA ___ cas protein


blank

___ cellular DNA


blank ___ cas protein–RNA complex
blank

___ excised viral DNA


blank ___ cell membrane
blank

___ cas protein–RNA–viral DNA complex


blank ___ stored viral DNA template
blank

___ degraded viral DNA


blank

The CRISPR system was discovered in cultures of yogurt in 2002. Subsequently,


researchers developed a technology based on manipulation of this system. The code for
the prokaryotic CRISPR/cas system is highly conserved and is found in the human genome.
DNA sequences are known that encode proteins responsible for many heritable diseases.
CRISPR/cas is a technology that allows DNA to be cleaved at the boundaries of a
nucleotide sequence, making the protein dysfunctional. The break in the strand is then
recognized and replaced with code for the functional protein. If the editing is done with
zygote-forming cells, the change is inherited. Not only the patient, but all progeny of the
patient is cured. This technology is the first to easily make genomic modifications of a
germ line. In the words of a prominent molecular biologist, this technology, which was
recognized as the Breakthrough of 2015 in the journal Science, “democratizes genetic
engineering.” Just as PCR became a standard tool that is widely used, any molecular
biology lab is now able to apply this technology.
B. Pose three questions—whose pursuit would require an understanding of genetics—
regarding the ethical and social issues that accompany the use of this medical technology.
C. Explain the value of genetic variation within a population. Predict a possible effect that
this technology could have, if unregulated, on human genetic variation.
Solution Sample answer:
A.
4 viral DNA 9 degraded viral DNA
3 cellular DNA 2 cas protein
5 excised viral DNA 6 cas Protein–RNA complex
8 cas protein–RNA–viral DNA 1 cell membrane
complex 7 stored viral DNA template
B. The nature of the problem elicits predictions of future events which cannot be
tested now. However, the question should discriminate between those that are
likely to eventually be answerable and those that will not be. For example, should
the use of the technology be restricted or should the use depend on the ability to
pay, are questions that will be decided without scientific reasoning. There are many
possible questions that do require reasoning. Sample questions:

********************************************************
**************************380***********************************

 How will disease be defined and differentiated from a phenotypic variation?


 What is the uncertainty associated with environmental factors that may or
may not be required for expression?
 What is the likelihood of developing a disease?
 How do we weight the induction of expression due to environmental factors
that may be avoided?
 If the gene in question is in a gene network, what is the relative importance
of the targeted gene?
 At the time when a decision regarding the use of the technology is needed,
how will expertise needed to answer these questions be accessed?
C. Phenotypic variation within a population increases likelihood of survival of genes
within that population. Genome editing will cause a reduction in variation. If
unregulated, the genome could behave as if the population were small, since
effective population size is a measure of diversity. Small populations have greater
risk of extinction.
51 Gel electrophoresis of polymers and polymer fragments is an important element in many
investigations. Samples of a solution are pipetted onto a gel. The gel is placed in a solution
that maintains a constant pH, and an electric field is applied over the length of the gel.
Separated components are placed on a substrate where they can be visualized and
identified by comparison with samples of standards. Application of this method to DNA is
called a Southern blot, named for the inventor of the technology. Application to RNA is
called a northern blot, another demonstration that biologists have fun (there are also
western, eastern, and far-eastern blots, but these techniques are not named for their
inventor).
A. Consider the three amino acids shown in the figure and explain how when placed on a
gel in an electric field the amino acids would move, how the amino acids would be
separated as they moved, and which would move further.

B. A biologist wants to determine whether a new protocol is successful in constructing


and amplifying a molecular clone of a segment of DNA introduced as a plasmid. After the

********************************************************
**********************************************************379***

procedure is complete, the bacterial cells containing plasmid with the inserted segment
are lysed, and a gel is run on which spots of the lysate and the sequence to be cloned have
been pipetted. Use the data displayed in the developed gel shown in the figure to
evaluate the question of whether or not the protocol was successful.

C. Design a plan to answer the question of whether the new DNA has been incorporated
in the DNA of the host organism.
Solution Sample answer:
A. The electric field exerts a force on a charged molecule. Higher molecular mass
molecules migrate less far, as do molecules with smaller charge and so the
components will be separated and in this case it is uncertain whether histidine, with
a larger charge but also a greater mass, will travel further than serine. Certainly
alanine will travel further than serine. The rubric will award points for reasoning, not
for correct answer.
B. The sequence is indicted in the run of the lysate. However, this does not mean
that the sequence is incorporated into the genome.
C. To determine whether the sequence is heritable, the examination of lysate would
be repeated after the passage of a few generations. So, the plan would be grow the
cells on nutrient media, dilute, plate, grow, and repeat this sequence a few times.
Then lyse the cells and run a gel.
17.3 Whole-Genome Sequencing
52 Genetic engineering can be applied to heritable information to produce what is referred
to as a “knockdown organism.” Biotechnology also can be applied to produce non-
heritable changes in a “knockdown gene.” Post-transcriptional strategies target the mRNA
product of a gene. One such strategy uses the conserved genes that encode RNA
interference (RNAi) proteins for the regulation of level of mRNA transcription.
Some viral RNA is double stranded (dsRNA). A cell responds to the presence of double
stranded RNA by attachment of the enzyme DICER which cuts dsRNA into short fragments.
One strand of the fragment is transferred to the RNA induced silencing complex (RISC)
which searches for mRNA with a sequence matching that of the fragment strand. When
detected, this mRNA is degraded.

********************************************************
**************************380***********************************

A. Common in cancer cells is a mutation of the gene that codes for the protein p53, whose
role is to detect and repair errors in DNA and, if repairs cannot be made, initiate
apoptosis. Create a visual representation to explain how the DICER-RISC system within
the cell can be used to suppress the translation of a mutated form of the gene coding for
p53, potentially destroying a tumor.
B. Whole genome sequences provide a library of potentially expressed proteins, but they
do not provide information on the functions of each protein. In an approach called
reverse genetics, investigations attempt to determine the function of the gene, often by
silencing the gene using RNAi technology. Assume that you have the ability to synthesize
dsRNA from a DNA segment taken from an organism whose whole genome has been
determined. Design a plan for collecting data that could be used to assign a function to
the protein encoded by this sequence. (Hint—Do not worry about the number of
experiments that might need to be conducted to implement your plan. An automated
technique called high throughput screening robotically supports thousands of
simultaneous experiments.)
Solution Sample answers:
A. Because the sequence coding for the mutant p53 is known, a dsRNA
complementing this template can be constructed and introduced with a viral vector.
When the dsRNA is recognized, the DICER-RISC machinery will repress the mutant.
This technology was applied to this problem in 2014.
B. A dsRNA is constructed using the desired sequence as the template. Cells
(probably using a surrogate such as yeast) are exposed. Any suspected function can
be evaluated by presenting the appropriate substrate and looking for reaction
products or growing the cells on nutrient media and looking for the absence of
product.

********************************************************
****************************************************************405****

18 | EVOLUTION AND ORIGIN OF SPECIES


REVIEW QUESTIONS
1 Which scientific concept did Charles Darwin and Alfred Wallace independently discover?
A Mutation
B Natural selection
C Overbreeding
D Sexual reproduction
Solution The solution is (B). Darwin and Wallace both independently described the process of
natural selection, the mechanism by which evolutionary change is produced.
2 Which statement about a natural principle that points to the inevitability of natural
selection is false?
A Most characteristics of organisms are inherited.
B Offspring vary among each other in regard to their characteristics.
C Some generations of offspring do not need to compete for resources.
D Certain traits will be better represented in the next generation.
Solution The solution is (C). After reading the work of economist Thomas Malthus, Darwin
recognized that all species over-reproduce. He reasoned that at each and every
generation, then, there is competition for resources to survive.
3 What is the best definition of adaptation?
A A trait or behavior that aids an organism’s survival and reproduction
B A heritable trait or behavior that aids an organism’s survival and reproduction
C A trait or behavior that aids a population’s survival and reproduction
D A heritable trait or behavior that aids a population’s survival and reproduction
Solution The solution is (B). An adaptation is a heritable characteristic that aids an organism’s
survival and reproduction.
4 What is an example of an adaptation?
A The better nutrition of a human helps her grow taller.
B The webbed feet of a duck help it swim.
C The urban location of a raccoon helps it find food.
D The large leaves of a desert plant require more water.
Solution The solution is (B). An adaptation is a heritable trait or behavior, like the webbed
feet of a duck, which helps it swim.

********************************************************
***************************404*****************************************

5 Which process is divergent evolution?


A Groups of organisms evolve in different directions from a common point.
B A new species develops rapidly when an event cuts off a portion of a population.
C Groups of organisms independently evolve to similar forms.
D A species evolves when a few members move to a new geographical area.
Solution The solution is (A). When two species evolve in diverse directions from a common
point, divergent evolution occurs.
6 Which situation is most likely an example of convergent evolution?
A Some fish that live in total darkness have eyes.
B Hawks and other birds have feathers.
C Worms and snakes both move without legs.
D Flowers that look very different have the same reproductive organs.
Solution The solution is (C). Worms and snakes do not share a recent common ancestry, yet
they move similarly. This is likely an example of convergent evolution.
7 What are homologous structures?
A Physical structures that have no apparent function
B Parallel structures in diverse organisms
C Physical structures that are used only occasionally
D Similar structures in diverse organisms
Solution The solution is (B). Scientists call synonymous parts in diverse species homologous
structures.
8 What are two examples of vestigial structures?
A Gills in fish and parts of the throat in humans
B Butterfly wings and dragonfly wings
C Hind leg bones in whales and leaves on some cacti
D Shark fins and dolphin fins
Solution The solution is (C). Vestigial structures are those with minimal to no utility for an
organism. Whales do not use hind leg bones and leaves are of minimal use to cacti.
They represent remnants of features useful to the organisms’ ancestors.

********************************************************
****************************************************************405****

9 Which statement best describes the relationship between the theory of evolution and the
origin of life?
A The theory includes an explanation of life’s origins.
B The theory cannot explain the origin of life.
C The theory does not try to explain the origin of life.
D The theory does not contribute understanding to pre-life processes.
Solution The solution is (C). The theory of evolution describes how populations change over
time and how life diversifies the origin of species; it does not try to explain life’s
origins.
10 Which statement best describes what happens when an antibiotic is applied to a
population of bacteria?
A The bacteria develop resistance to the antibiotic in direct response to its application.
B The bacteria’s genetic material mutates in response to the antibiotic, resulting in
resistance.
C A gene for resistance, already present in the population, decreases in frequency.
D A gene for resistance, already present in the population, increases in frequency.
Solution The solution is (D). A gene for resistance is in the population already. The antibiotic
kills bacteria without the resistant gene, strongly selecting resistant individuals. The
gene for resistance thus increases in frequency in the gene pool.
11 Which option is the best definition of species?
A A group of individual organisms with significant genetic similarities
B A group of individual organisms with significant genetic similarities that share external
and internal characteristics
C A group of individual organisms that interbreed
D A group of individual organisms that interbreed and produce viable, fertile offspring
Solution The solution is (D). The best definition is a group of individual organisms that
interbreed and produce viable, fertile offspring.
12 What do scientists focus on to distinguish between species?
A Ecological niches
B Morphological differences
C Reproductive barriers
D Genetic changes

********************************************************
***************************404*****************************************

Solution The solution is (C). Biological species concept is widely used to distinguish between
species. It states that species are groups of interbreeding natural populations that
are reproductively isolated from other such groups.
13 What are two primary sources of genetic variation?
A Mutations and sexual reproduction
B Isolation and sexual reproduction
C Sexual reproduction and asexual reproduction
D Migration and sexual reproduction
Solution The solution is (A). The accumulation of mutations and the genetic shuffling of sexual
reproduction are two primary sources of genetic variation.
14 Which statement best describes the relationship between genetic variation and
speciation?
A Without genetic variation, speciation would occur more slowly.
B Without genetic variation, speciation would not be possible.
C Genetic variation influences sympatric speciation, but not allopatric speciation.
D There is no relationship between genetic variation and any form of speciation.
Solution The solution is (B). Two basic mechanisms of evolutionary change—natural selection
and genetic drift—cannot operate without genetic variation. Other mechanisms of
evolutionary change, migration and mutation, produce genetic variation. Genetic
variation makes evolutionary change possible.
15 Which statement about postzygotic barriers is false?
A They occur after fertilization.
B They include hybrids that are sterile.
C They include hybrid organisms that do not survive the embryonic stage.
D They include reproductive organ incompatibility.
Solution The solution is (D). Reproductive organ incompatibility prevents reproduction from
taking place, so it is a prezygotic barrier rather than a postzygotic barrier.
16 Which situation is an example of a prezygotic barrier?
A Two species of fish produce sterile offspring.
B Two species of flowers attract different pollinators.
C Two species of insects mate, but the zygote does not survive.
D Two species of lizards mate, but the offspring dies before reproducing.
Solution The solution is (C). A postzygotic barrier takes place after zygote formation. An
organism does not survive the embryonic stage or is born sterile.

********************************************************
****************************************************************405****

17 Which situation would most likely lead to allopatric speciation?


A A flood causes the formation of a new lake.
B A storm causes several large trees to fall down.
C A mutation causes a new trait to develop.
D An injury causes an organism to seek out a new food source.
Solution The solution is (A). Allopatric speciation occurs when a population becomes
geographically isolated. The formation of a new lake is one way in which such
isolation could occur.
18 What is the main difference between an autopolyploid individual and an allopolyploid
individual?
A Number of extra chromosomes
B Functionality of extra chromosomes
C Source of extra chromosomes
D Number of mutations in the extra chromosomes
Solution The solution is (C). In autopolyploids, the source of the extra chromosomes is the
individual itself. A diploid parent produces a polyploid offspring. In allopolyploids,
different species combine to produce polyploid offspring.
19 What is unique about speciation due to adaptive radiation?
A It leads to multiple species forming from one parent species.
B It only occurs on or around island archipelagos.
C It requires a population to disperse from its parent species.
D It is a special kind of sympatric speciation.
Solution The solution is (A). Adaptive radiation occurs when many adaptations evolve from a
single point of origin. This is a unique quality, as multiple speciations fuel greater
evolutionary change.
20 What is least likely to be a factor that increases the probability of speciation by adaptive
radiation?
A There are vacant ecological niches nearby.
B Genetic drift in a population increases.
C There are isolated regions with suitable habitats.
D There are few competitor species.
Solution The solution is (B). Genetic drift is not likely to be a factor in speciation by adaptive
radiation.

********************************************************
***************************404*****************************************

21 In a hybrid zone, in addition to interacting, what else do two closely related species do?
A Compete
B Reproduce
C Transition
D Fuse
Solution The solution is (B). Two closely related species that interact and reproduce to form
hybrids do so in an area called a hybrid zone.
22 Which situation means reinforcement is more likely to occur in the hybrid zone?
A The hybrid offspring are more fit than the parent species.
B Reproductive barriers weaken.
C The hybrid offspring are about as fit as the parent species.
D Reproductive barriers strengthen.
Solution The solution is (D). When reproductive barriers strengthen, hybrids are generally not
as fit as the parent species, and reinforcement of speciation takes place.
23 Which statement is false?
A Gradual speciation and punctuated equilibrium both result in the divergence of
species.
B Punctuated equilibrium is most likely to occur in a large population in a stable
environment.
C In the punctuated equilibrium model, gradualism is not excluded.
D In the gradual speciation model, traits change incrementally.
Solution The solution is (B). Punctuated equilibrium is most likely to occur in a smaller
population and in an environment undergoing change.
24 Which component of speciation would be least likely to be a part of punctuated
equilibrium?
A A division in populations
B A change in environmental conditions
C Ongoing gene flow
D A number of mutations occurring at once
Solution The solution is (C). Continued gene flow is likely to slow down the rate of speciation.
This is less likely to be a part of the evolutionary change described by the model of
punctuated equilibrium.

********************************************************
****************************************************************405****

CRITICAL THINKING QUESTIONS


25 What conclusions can you draw about the relationship between the way in which the
present-day theory of evolution developed and the credibility of the theory? Explain your
thinking.
A When the theory of evolution was first proposed, it was met with a lot of criticism and
disbelief, but it is widely supported today. Theories that have withstood a larger
amount of criticism are more credible than those that are accepted easily.
B The theory of evolution has its foundation in both biological and geological
observations, making it a more credible theory because it can explain more about
the world.
C The theory of evolution relies on the heritability of traits, but the mechanism of this
inheritance was not understood when the theory was developed. This reduces the
credibility of the theory because the people who created it did not understand how
it worked.
D It is meaningful that two naturalists working independently from each other offered
the same explanation for the same set of phenomena. When two people
independently look at the same evidence and come to the same conclusion, this
reinforces the credibility of that conclusion.
Solution The solution is (D). I can conclude that the way in which the theory was developed
says something about its credibility. It is meaningful that two naturalists who
worked independently from each other offered the same explanation for the same
set of phenomena. When two people independently look at the same (or similar)
body of evidence and come to the same conclusion, this reinforces the credibility of
that conclusion. This same idea is part of today’s scientific process, in which studies
must be replicable to be considered valid.
26 How does an adaptation, such as better running speed, relate to natural selection?
A Natural selection produces beneficial adaptations, such as better running speed, in
individuals that run more frequently.
B Natural selection randomly mutates individuals’ genetic code until it produces
beneficial adaptations, such as better running speed.
C Natural selection produces adaptations, such as better running speed, to help
individuals survive and reproduce.
D Natural selection reproduces individuals with favorable genetic traits, such as the
adaptation of better running speed, over time.
Solution The solution is (C). An adaptation is a heritable trait that allows an individual
organism to better survive and reproduce in its present environment. An animal that
runs faster, for example, is better suited to catching prey and/or avoiding predators,
and thus more likely to survive and pass on its ability to run quickly to its offspring.

********************************************************
***************************404*****************************************

Natural selection is the reproduction of individuals with favorable genetic traits—


such as the adaptation of better running speed—over time.
27 What is an example of convergent evolution? How does it support the theory of evolution
by natural selection?
A An example of convergent evolution is the development of the same function,
swimming, in organisms that live in different parts of the globe, such as Arctic beluga
whales and Antarctic right whales. The fact that organisms that do not come in
contact with each other have developed the same traits suggests that natural
selection can produce similar adaptations in organisms who share a similar
environment.
B An example of convergent evolution is the set of adaptations, such as better running
speed or more efficient hunting, developed by a species in response to competition
with a new species that moves into the same region. The fact that a species adapts
after it comes into contact with a competitor suggests that natural selection works
more quickly with higher selective pressures.
C An example of convergent evolution is the development of an ancestral structure, a
limb, into two different modern structures, such as a hand and a flipper. The fact that
natural selection can cause a structure to develop down two different pathways due
to different environmental conditions supports the theory of evolution.
D An example of convergent evolution is the development of the same function, flying,
in organisms that do not share a recent common ancestry, such as insects and birds.
The fact that wings that allow flight have developed from very different original
structures suggests that the process of natural selection can produce similar
adaptations in two very different types of organisms who share a similar environment.
Solution The solution is (D). An example of convergent evolution is the development of the
same function, flying, in organisms that do not share a recent common ancestry,
such as insects and birds. The fact that wings that allow flight have developed from
very different original structures suggests that the process of natural selection can
produce similar adaptations in two very different types of organisms who share a
similar environment.
28 Why do scientists consider vestigial structures evidence for evolution?
A Vestigial structures are the result of convergent evolution, so they are good evidence
that natural selection act similarly in similar environmental conditions.
B Vestigial structures are the result of common ancestry, so they are good evidence that
different populations of organisms evolved from a common point.
C Vestigial structures are the result of convergent evolution, so they are good evidence
for an end goal to evolution.
D Vestigial structures are the result of common ancestry, so they are good evidence for
a common origin of all life.

********************************************************
****************************************************************405****

Solution The solution is (A). Vestigial structures are, in fact, homologies—underlying patterns
in form (if not exactly function) that are the result of common ancestry. Whether or
not they are useful, the fact that anatomical similarities exist even in very different
organisms is good evidence that different populations of organisms evolved from a
common point. Furthermore, the actual uselessness of vestigial structures points to
ongoing evolution—these structures are no longer part of the adaptive features of
an organism, but likely once were.
29 Reproduction in sexually-reproducing organisms occurs when two sex cells, or gametes,
fuse. In fish, this occurs when sperm swim through the water to find the ovum. In flowers,
pollen is dispersed through the air and carried to another flower. Explain what
evolutionary adaptations for reproduction occur in humans, based on the fact that we are
land-based animals.
Solution Because humans are land-based animals, the female must provide an internal, fluid
environment in which sperm will not be desiccated and can move. Thus, the male
sperm is deposited directly into the internal reproductive tract of the female, which
creates an insulated environment in which sperm can survive and travel to the ovum.
30 While examining the human genome, you find a gene that is not homologous to any other
organisms known to man. You conclude that this gene must be unique to the human
species and could not have evolved from another organism. Would this discovery suggest
that humans do not share a common ancestor with all other organisms on Earth? Explain
your answer.
Solution No. Although all organisms descended from other organisms, it is possible for a new
gene to form in the human genome, or any organism, through processes such as
mutation or crossing-over during meiosis. Such a gene would not need to have been
inherited from an ancestor and humans would still likely share many other genes with
other Earth organisms.
31 Mutations in the glucose 6-phosphate dehydrogenase (G6PD) gene can cause a rare
anemia when inherited. However, homozygotes with this mutation are less prone to
malaria infection, a disease that historically was the most widespread deadly disease
among humans. Predict how this mutation would affect the fitness of individuals living in
countries where malaria is endemic.
Solution Despite the ability of G6PD to cause deadly anemia, the mutation is favored and will
remain in the population because it causes resistance to malaria, a much more
widespread disease among humans.
32 How does the scientific meaning of theory differ from the common vernacular meaning?
A A scientific theory is a hypothesis that needs to be tested, whereas people often use
theory to mean a simple guess.
B A scientific theory is a statement that has been proven correct, while people often use
it to mean a statement that has not yet been verified.

********************************************************
***************************404*****************************************

C A scientific theory is a thoroughly tested set of explanations for a body of observations


of nature, while people often use it to mean a guess or speculation.
D A scientific theory is a random guess, while people often use it to mean a statement
that is somewhat based in fact.
Solution The solution is (B). In science, a theory is a body of well tested and verified
explanations for a set of observations about the natural world. Its use connotes
rigorous examination of evidence and general consensus by experts in the field. In
common vernacular, theory is used to mean a guess or suggested explanation. It
does not connote rigorous examination of evidence. It is much more like the
scientific term hypothesis, which is a reasoned explanation but has not yet been
tested and verified.
33 Why is having a way of defining species and distinguishing between them important for
the study of evolution?
A A distinction between species allows scientists to understand the common origin of
all species.
B A common definition of species allows scientists to agree on all aspects of the theory
of evolution.
C Divergence can only occur at the species level: It does not occur to larger taxa.
Therefore, it is important to know which groups are distinct species.
D In the study of evolution, the species is the unit over which change is measured.
Solution The solution is (A). Studying evolution means studying change among life forms over
time. It is always important to have a unit, or benchmark, for measuring change. In
the study of evolution, the species is that unit or benchmark.
34 If a population stopped reproducing sexually, but still reproduced asexually, how would its
genetic variation be affected over time? Could speciation occur in this situation?
A Genetic variation would increase, and speciation would be possible.
B Genetic variation would increase, and speciation would not be possible.
C Genetic variation would decrease, and speciation would be possible.
D Genetic variation would decrease, and speciation would not be possible.
Solution The solution is (C). Over time, its genetic variation would probably decrease over all,
since sexual reproduction is a primary means of variation. There would still be the
possibility of mutations introducing genetic variation, and depending on the species,
migration could introduce different genes into the population. Some genetic
variation is necessary for any evolutionary change to happen. Over time, speciation
would still be possible, especially if a portion of the population was isolated from the
rest and mutations occurred.
35 What role do prezygotic and postzygotic barriers play in speciation?

********************************************************
****************************************************************405****

A Prezygotic and postzygotic barriers allow for the formation of less-fit hybrids that
reinforces speciation.
B Prezygotic and postzygotic barriers prevent interbreeding of species such that there is
no gene flow between them.

********************************************************
***************************404*****************************************

C Prezygotic and postzygotic barriers prevent migration of the two species, causing
them to remain in contact with each other and begin to interbreed.
D Prezygotic and postzygotic barriers are present only in newly formed species, allowing
scientists to identify the time of divergence of the species.
Solution The solution is (B). Speciation is the formation of two separate species from one
original species. Inherent in the definition of speciation is the development of
reproductive isolation—the inability to interbreed. Both prezygotic barriers and
postzygotic barriers prevent interbreeding of species such that there is no gene flow
between them.
36 A population of flowers was separated into two subpopulations when a new river cut
through the plain in which they were growing. The number of interbreeding events per
year for the two subpopulations of flowers is shown in the graph below. Twenty-four
years after they were separated, can you conclude that the two subpopulations of flowers
have become new species? Why or why not?

Solution No; just because interbreeding does not occur does not mean the flowers are
incapable of interbreeding.
37 Which type of speciation, allopatric or sympatric, is more common? Why?
A Allopatric speciation is more common because it prevents gene flow between
the species.
B Allopatric speciation is more common because it involves stronger prezygotic barriers.

********************************************************
****************************************************************405****

C Sympatric speciation is more common because it prevents gene flow between the
species.
D Sympatric speciation is more common because it involves stronger prezygotic barriers.
Solution The solution is (A). Allopatric speciation is more common than sympatric speciation.
Geographic isolation (the defining characteristic of allopatric speciation) greatly
reduces gene flow between populations, so speciation is more likely to occur.
Continued gene flow is more likely in a sympatric situation, so this type of speciation
is less common.
38 Using adaptive radiation, how can the diversification of the finches Darwin observed in
the Galapagos be explained?
A The finches likely shared a common ancestor when they came to the island, but
exhibited different traits. Each species of finch settled the island where its particular
traits would be the most adaptive.
B The finches likely originated as one parent species, but over time mutations caused
them to develop reproductive barriers and separate into different species. To reduce
competition, the species then radiated out to inhabit different islands.
C The finches likely dispersed from one parent species, and natural selection based on
different food sources in differing habitats led to adaptive changes, evidenced in the
different beak shapes of the different species, each suited to a different food type.
D It is likely that a series of cataclysmic events caused an original finch species to diverge
into the many finch species that inhabited the islands when Darwin observed them.
The different species then radiated out to the different islands and adapted to the
different conditions on each.

********************************************************
***************************404*****************************************

Solution The solution is (D). Adaptive radiation is speciation that occurs when one population
radiates out and forms several new species, each in a different niche or isolated
habitat. Island archipelagos, like the Galapagos, are ideal contexts for this type of
speciation, because the water provides the geographical isolation for speciation to
occur. Darwin observed finches with different beak shapes and related these shapes
to different food sources. These finches likely dispersed from one parent species,
and evolution in response to natural selection based on different food sources in
differing habitats led to behavioral, and eventually genetic, changes. These changes
are evidenced in the different beak shapes of the different species—each suited to a
different food type.
39 In which situation would hybrid reproduction cause two species to fuse into one?
A Separate species cannot interbreed, so hybrid reproduction does not occur in nature.
B If the hybrid offspring are more fit than the parents, reproduction would likely
continue between both species and the hybrids, eventually bringing all organisms
under the umbrella of one species.

********************************************************
****************************************************************405****

C Two species that have recently diverged from each other can reproduce with each
other, creating hybrid individuals that belong to the species of the parents’ common
ancestor.
D If two species occupy the same niche in the same area, they can either compete or
they can collaborate and reproduce with each other, eventually fusing into a single
species.
Solution The solution is (D). If two closely related species continue to produce hybrids that
are as fit as the parent species, or even more fit, the species would be more likely to
fuse. If there was an increased pressure for that fusion, such as an environmental
change that pushed the two species to share habitat or food sources more closely,
this might increase the possibility of fusion.
40 What do both rate of speciation models have in common?
A Both models ignore the influence of gene flow for simplicity’s sake.
B Both models apply only to island chains.
C Both models require the influence of cataclysmic events which precipitate rapid
adaptation and speciation.
D Both models conform to the rules of natural selection and the influences of gene flow,
genetic drift, and mutation.
Solution The solution is (A). Both rate of speciation models result in the divergence of one
species into two species. Both rely on the usual mechanism of change, natural
selection, and both are influenced by factors such as environmental changes, rates
of mutation, and population size.
41 In which situation would hybrid reproduction would two species to continue divergence?
A If two closely related species continue to produce hybrids, the hybrids will compete
with both species, causing them to find new niches that will further their divergence.
B If two closely related species continue to produce hybrids, they will develop
reproductive barriers to prevent production of hybrids, to ensure that they remain
separate species.
C If two closely related species continue to produce hybrids that are less fit than the
parent species, there would be reinforcement of divergence.
D If two closely related species continue to produce hybrids they will always converge
into a single species.
Solution The solution is (B). If two closely related species continue to produce hybrids that
are less fit than the parent species, there would be reinforcement of divergence.
And, if there were an increased division between the two species, such as an
environmental change that pushed the two species to move farther apart or
increased reproductive barriers that decreased hybrid fitness even further, this
would increase the possibility of divergence even more.

********************************************************
***************************404*****************************************

TEST PREP FOR AP® COURSES


42 Prior to 1800 in England, the typical moth of the species Biston betularia (peppered moth)
had a light pattern. Dark-colored moths were rare. By the late nineteenth century, the
light-colored moths were rare, and the moths with dark patterns were abundant.
The cause of this change was hypothesized to be selective predation by birds (Tutt, 1896).
During the Industrial Revolution, soot and other wastes from industrial processes killed
tree lichens and darkened tree trunks. Thus, prior to the pollution of the Industrial
Revolution, dark moths stood out on light-colored trees and were vulnerable to predators.
With the rise of pollution, however, the coloring of moths vulnerable to predators
changed to light.
Which aspect of Darwin’s theory of evolution does the story of the peppered moth most
clearly illustrate?
A There is competition for resources in an overbred population.
B There is great variability among members of a population.
C There is differential reproduction of individuals with favorable traits.
D The majority of characteristics of organisms are inherited.
Solution The solution is (C). Through natural selection, moths with favorable coloring survived
to reproduce and pass on their coloring.
43 Prior to 1800 in England, the typical moth of the species Biston betularia (peppered moth)
had a light pattern. Dark-colored moths were rare. By the late nineteenth century, the
light-colored moths were rare, and the moths with dark patterns were abundant.
The cause of this change was hypothesized to be selective predation by birds (Tutt, 1896).
During the Industrial Revolution, soot and other wastes from industrial processes killed
tree lichens and darkened tree trunks. Thus, prior to the pollution of the Industrial
Revolution, dark moths stood out on light-colored trees and were vulnerable to predators.
With the rise of pollution, however, the coloring of moths vulnerable to predators
changed to light. In the late 1900s, England cleaned up its air, and pollution decreased.
The bark of trees went from dark to light.
Which outcome to the populations of peppered moth would you expect, given this
environmental change?
A An increase in the number of dark moths and a decrease in the number of light moths
B An increase in the number of moths overall
C An approximately equal number of light moths and dark moths
D An increase in the number of light moths and a decrease in the number of dark moths
Solution The solution is (D). Natural selection would allow for differential reproduction of the
more favorable trait. With decreased pollution and lighter trees, this would mean
the light moth.

********************************************************
****************************************************************405****

44 Prior to 1800 in England, the typical moth of the species Biston betularia (peppered moth)
had a light pattern. Dark-colored moths were rare. By the late nineteenth century, the
light-colored moths were rare, and the moths with dark patterns were abundant.
The cause of this change was hypothesized to be selective predation by birds (Tutt, 1896).
During the Industrial Revolution, soot and other wastes from industrial processes killed
tree lichens and darkened tree trunks. Thus, prior to the pollution of the Industrial
Revolution, dark moths stood out on light-colored trees and were vulnerable to predators.
With the rise of pollution, however, the coloring of moths vulnerable to predators
changed to light.
Commonly used in biology text books, the peppered moth is a classic example of
evolutionary change in action. The example describes changes in a population’s allele
frequencies: a small-scale change, evolutionarily speaking. The presence of both light and
dark forms within the gene pool is demonstrated by the story, but the peppered moth
stays a peppered moth.
Which scenario, if it were to occur, would be a model for large-scale evolutionary change?
A Conditions change such that the dark form of the moth is favored and the light form is
diminished in the population due to predation. Conditions change again, the dark form
is vulnerable, and the light form returns to prevalence.
B Conditions change such that the dark form of the moth is favored and the light form is
eradicated in the population due to predation. Conditions change again, the dark form
is vulnerable, and the dark form is eradicated due to predation.
C Conditions change such that dark form of the moth is favored and the light form is
diminished in the population due to predation. Conditions change again, and both
forms have equal prevalence.
D Conditions change such that dark form of the moth is favored and the light form is
eradicated in the population due to predation. Conditions change again, the dark form
is vulnerable. It develops an adaptation that shields it from predation.
Solution The solution is (B). Conditions change such that the dark form of the moth is favored
and the light form is eradicated in the population due to predation. Conditions
change again, the dark form is vulnerable, and the dark form is eradicated due to
predation.
45 Given your understanding of evolutionary theory and the relationship between evolution
and the genetic makeup of populations, which statement is false?
A Homologous characteristics that have evolved more recently are shared only within
smaller groups of organisms.
B The genetic code is a homologous characteristic shared by all species because they
share a common ancestor in the deep past.

********************************************************
***************************404*****************************************

C DNA sequence data would likely support any evolutionary tree drawn from anatomical
data sets.
D The degree of relatedness between groups of organisms is only sometimes reflected in
the similarity of their DNA sequences.
Solution The solution is (D). The degree of relatedness between groups of organisms is only
sometimes reflected in the similarity of their DNA sequences.
46 Each of the following observations comes from a different scientific discipline. Which
observation best supports Darwin’s concept of descent with modification?
A Geologists provide evidence that earthquakes reshape life by causing mass
extinctions.
B Botanists provide evidence that South American temperate plants have more in
common with South American tropical plants than temperate plants from Europe.
C Zoologists provide evidence that fewer animal species live on islands than on nearby
mainlands.
D Ecologists provide evidence that species diversity increases closer to the equator.
Solution The solution is (B). Botanists provide evidence that South American temperate
plants have more in common with South American tropical plants than temperate
plants from Europe.
47 Paleontologists have recovered a fossil for an organism named Archaeopteryx. It has
many features in common with reptiles, but, like birds, shows evidence of feathers.
For what aspect of evolutionary theory does this piece of evidence suggest support?
A Modern species are distinct natural entities.
B Modern species are not currently evolving.
C Modern species share a common ancestor.
D Modern species have both convergent and divergent traits.
Solution The solution is (C). A fossil that demonstrates common characteristics of now
divergent groups suggests common ancestry among living things, both specifically
and generally.
48 Which piece of evidence illustrates evolution as an ongoing process?
A Some genes from the bacterium E. coli have sequences that are similar to genes found
in humans.
B Marsupial mammals live in just a few places in the world today: Australia, South
America, and part of North America.

********************************************************
****************************************************************405****

C The fossil record shows that Rodhocetus, an aquatic mammal related to whales, had a
type of ankle bone that is otherwise unique to a group of land animals.
D In the 1940s, infections by the bacterium Staphylococcus aureus could be treated with
penicillin; today populations exist that are completely resistant.
Solution The solution is (A). Ethnic Tibetans are unique among humans for having
physiological mechanisms that help them live in places with low oxygen levels.
49 The process of mutation, which generates genetic variation, is random. Thus, life has
evolved, and continues to evolve, randomly. Which statement is an appropriately
evidence-based refinement of the above?
A The process of mutation, which generates genetic variation, is random. However, the
process of natural selection, which results in adaptations like the fit between a flower
and its pollinator, favors variants which are better able to survive and reproduce.
Natural selection is not random, so the overall process of evolution is not random,
either.
B The process of mutation, which generates genetic variation, is random. However, the
process of migration, which results in gene flow between populations, also generates
genetic variation. Migration is not random, so the overall process of evolution is not
random, either.
C The process of mutation, which generates genetic variation, is random. However, the
process of sexual reproduction, which also introduces genetic variance, is not random.
Because sexual reproduction is not random, the overall process of evolution is not
random, either.
D The process of mutation, which generates genetic variation, is random. Whether
mutations have a positive, negative, or neutral effect in terms of selective advantage is
also random. Mutations and their effects are random, so the overall process of
evolution is random.
Solution The solution is (A). The process of mutation, which generates genetic variation, is
random. However, the process of natural selection, which results in adaptations like
the fit between a flower and its pollinator, favors variants which are better able to
survive and reproduce. Natural selection is not random, so the overall process of
evolution is not random, either.
50 The selective breeding of plants and animals that possess desired traits is a process called
artificial selection. For example, broccoli, cabbage, and kale are all vegetables that have
been selected from one species of wild mustard.
How is artificial selection both similar to and different from Darwin’s conception of natural
selection? Does artificial selection provide evidence for evolution by natural selection?
Explain.

********************************************************
***************************404*****************************************

A Both artificial selection and natural selection are the differential reproduction of
individual organisms with favored traits. In artificial selection, humans have actively
modified plants and animals by selecting and breeding individuals with traits deemed
desirable. In natural selection, the most successful individuals in a species are selected
by the species to reproduce.
B Both artificial selection and natural selection are processes that result in better-
adapted individuals within a species. In artificial selection, humans have actively
modified plants and animals by selecting beneficial genes from other organisms and
inserting them into the target organisms. In natural selection, natural processes such
as mutations and viruses introduce new genes to a population.
C Both artificial selection and natural selection are processes that cause organisms to be
better adapted over time. In artificial selection, humans have trained animals to be
more successful in completing tasks that the humans want completed. In natural
selection, organisms train the functions that they will need to survive and reproduce.
D Both artificial selection and natural selection are the differential reproduction of
individual organisms with favored traits. In artificial selection, humans have actively
modified plants and animals by selecting and breeding individuals with traits deemed
desirable. In natural selection, individuals are selected naturally as its traits deem it
more fit for survival and reproduction.
Solution The solution is (A). Both artificial selection and natural selection are the differential
reproduction of individual organisms with favored (selected) traits. Each process
works by acting on heritable variations in a population. In artificial selection, humans
have actively modified plants and animals by selecting and breeding individuals with
traits deemed desirable. In natural selection, individuals are selected naturally as its
traits deem it more fit for survival and reproduction. The “more fit” individuals pass
on these “more fit” traits to their offspring; over time, the population’s traits change
in accordance with its environment. Artificial selection provides a good model for
understanding how natural selection works. Although it brings about dramatic
change in a relatively short period of time, it provides evidence for how small
changes multiplied over many generations can result in a species that bears little
resemblance to its ancestors.
51 Genes important in the embryonic development of animals have been relatively well
conserved during evolution. This means they are more similar among different species
than many other genes. What explains this genetic conservation across animal species?

********************************************************
****************************************************************405****

A Changes in the genes that are important to embryonic development have been
relatively minor because there are no selective pressures on an individual before it
is born.
B Changes in the genes that are important to embryonic development have been
relatively minor because not much time has elapsed since the divergence of the
various animal taxa.
C Changes in the genes that are important to embryonic development have been
relatively minor because early embryos are very fragile and even small mutations can
result in death.
D Changes in the genes that are important to embryonic development have been
relatively minor because mutational tweaking in the embryo has magnified
consequences in the adult.
Solution The solution is (D). Early embryonic development of animal species follows a similar
pattern. For example, structures such as tails and slits such as gills are present in all
vertebrate such as embryos, though they develop differently or disappear altogether
in later forms. Since anatomical development is the result of genetic material and
the machinery of DNA replication and expression, it makes sense that the genetic
information that produces these basic embryonic structures would be the same or
very similar in very different species. The differences between species are evidence
of evolutionary changes in their genetic makeup. Changes in the genes that are
important to embryonic development have been relatively minor, as mutational
tweaking in the embryo has magnified consequences in the adult.
52 The upper forelimbs of humans and cats have fairly similar structures. In contrast, the
upper forelimbs of whales (their flippers) have bones with a different shape and
proportion from both cats and humans. Interestingly, genetic data suggest that all three
organisms have a common ancestor from about the same point in time.
What is a likely explanation for these data?
A Cats and humans are more closely related to each other than either is to whales.
B The shape of the whale forelimb arose a result of disadvantageous mutations.
C The whale flipper is an adaptive characteristic unique to its water environment.
D The whale flipper is a vestigial structure.
Solution The solution is (C). Given the fact that all three organisms diverged from a common
ancestor at about the same point in time, and that evolutionary change is spurred by
environmental change, it seems that the whale flipper is an adaptive characteristic
unique to its water environment. Both humans and cats live on land, whereas
whales live in water. It makes good sense that natural selection in an aquatic
environment resulted in the differences we see in whale forelimb anatomy.

********************************************************
***************************404*****************************************

53 Biogeography is the study of biological species as they relate to geographical space


and geological time. The fossil record shows that dinosaurs originated about 200 to
250 million years ago. Would you expect the geographic distribution of early dinosaur
fossils to be broad (on many continents) or narrow (on one or a few continents)? Explain.
A Broad because dinosaurs originated before the breakup of Pangaea
B Broad because some dinosaurs could fly between continents
C Narrow because they went extinct too quickly to disperse very far
D Narrow because they lived so long ago that the fossils have mostly broken down or
disappeared
Solution The solution is (A). Given that the dinosaurs originated before or about the same
time as the breakup of the supercontinent Pangaea (about 200 million years ago), I
would expect that the geographic distribution of early dinosaur fossils to be quite
broad. It is likely that extant species of dinosaurs lived on many parts of Pangaea,
and it is also likely that when Pangaea broke apart, the fossils of dinosaurs would
have moved with the rocks as the continent broke apart and moved.
54 The term microevolution describes evolution on its smallest scale: the change in allele
frequencies in a population over generations. DDT is a pesticide that was widely in use in
the United States from the 1940s until 1972. The table summarizes a particular allele
frequency in laboratory strains of the common fruit fly, Drosophila melanogaster.
Strains Collected from Strains Collected from
Flies in the Wild in the Flies in the Wild in the
1930s 1960s
Frequency of Allele 0% 40%
Conferring DDT
Resistance
Using this information, in which model did natural selection improve the match between
D. mealanogaster and its environment through microevolution?
A DDT killed off a large proportion of the population, and the alleles present in the
surviving fruit flies differed from those in the original population.
B Mutations from the application of DDT caused the allele conferring DDT resistance to
appear in the population.
C Female mosquitoes chose to mate with male mosquitoes that had the allele conferring
DDT resistance because it would make their offspring more fit.
D The wide use of DDT meant that fruit flies with DDT resistance were more
evolutionarily fit than their counterparts without DDT resistance.
Solution The solution is (B). The allele conferring resistance to the pesticide DDT either arose
by mutation or was present in the population in the 1930s very rarely. The wide use
of DDT meant that fruit flies with DDT resistance were more evolutionarily fit than

********************************************************
****************************************************************405****

their counterparts without DDT resistance. The number of individuals with DDT
resistance in the population of fruit flies grew due to differential reproduction
associated with the increased evolutionary fitness of these individuals. DDT is a
pesticide—a poison—and natural selection strongly selected for individuals who had
the resistant allele. Thus, from the 1930s to the 1960s, the frequency of the allele
conferring DDT resistance increased in the fruit fly population.
55 In 1795, a Scottish geologist named Charles Hutton suggested that Earth’s geologic
features could be explained by gradual processes that were still operating. This was in
direct contrast to other scientific thought at the time, which included well-accepted
proposals that geologic layers were representative of catastrophic events caused by
processes no longer operating in the present time. Hutton proposed geologic features as
the result of slow and consistent change, such as valleys formed by rivers wearing through
rock. Hutton’s ideas were incorporated in the work of Charles Lyell, a geologist working in
Darwin’s time. Lyell advocated a principle called uniformitarianism, the consistency of
mechanisms of change over time. In other words, Lyell argued that the same geologic
processes operating in the present had operated in the past, and at the same rate.
The ideas of Hutton and Lyell influenced the work of Charles Darwin. How do Hutton’s and
Lyell’s ideas connect to and provide support for Darwin’s theory of evolutionary change?
A The idea that the same processes that operate in the present also operated in the
past, and at the same rate, supported Darwin’s hypothesis of natural selection
because humans could select for desirable traits and produce change very rapidly, so
natural selection would also be fast enough to produce the full range of diversity in
living organisms.
B The idea that the same processes that operate in the present also operated in the
past, and at the same rate, connects to Darwin’s hypothesis of natural selection
because he had observed it happening in the present.
C The idea that geologic change is the result of slow, continuous processes rather than
sudden, substantial change connects to Darwin’s support of gradualism rather than
punctuated equilibrium as the process that guided evolution.
D The idea that geologic change is the result of slow, continuous processes rather than
sudden, substantial change connects directly to Darwin’s hypothesis that, given
enough time, slow and subtle processes could produce substantial biological change.
Solution The solution is (C). The idea that geologic change is the result of slow, continuous
processes rather than sudden, substantial change connects directly to Darwin’s
hypothesis that, given enough time, slow and subtle processes could produce
substantial biological change. Further, the thesis that the Earth’s geologic features
result from such slow and steady change suggests a very old Earth. A theory of a
much, much older Earth supports Darwin’s theory of evolutionary change—the Earth
would have to be much older than commonly thought at Darwin’s time to be old
enough for life to have evolved from a single common ancestor.

********************************************************
***************************404*****************************************

56 The human immunodeficiency virus (HIV) reproduces very quickly. A single virus can
replicate itself a billion times in one 24-h period. In a hypothetical treatment situation, a
patient’s HIV population consists entirely of drug-resistant viruses after just a few weeks
of treatment.
How can this treatment result best be explained? How does this explanation illustrate that
evolution is an ongoing process?
A The resistant viruses passed their genes to the nonresistant viruses so that
100 percent of the viruses became resistant. This illustrates evolution as an
ongoing process because the genes of the population changed in real time.
B The nonresistant viruses died, and the resistant ones survived and rapidly reproduced.
This illustrates evolution as an ongoing process because the change in the HIV
population is the result of natural selection.
C The viruses developed resistance to the drug after repeated exposure to it. This
illustrates evolution as an ongoing process because the viruses were able to adapt to
changing conditions.
D The drug-resistant viruses were more fit than their nonresistant counterparts to begin
with, and over time they dominated the population. This illustrates evolution as an
ongoing process because natural selection favored one phenotype over another.
Solution The solution is (B). The treatment result is best explained as the result of natural
selection. A few drug-resistant viruses were in the original HIV population (at the
start of treatment) and natural selection increased their frequency to 100 percent.
The non-resistant viruses died, and the resistant ones survived and rapidly
reproduced. This explanation illustrates evolution as an ongoing process because the
change in the HIV population in the explanation is the result of evolutionary
processes. Natural selection strongly selected for the drug-resistant virus in the
presence of the drug. There was differential replication of the drug-resistant virus
with the accumulation of favorable traits (drug resistance) over generations.
57 A friend says: “Natural selection is about the survival of the very fittest in a population.
The fittest are those that are strongest, largest, and fastest.”
Would you agree with that statement? What evidence from scientific disciplines can you
offer to support your agreement or your disagreement?
A The statement is true. If an organism is not strong and fast, it will not survive long
enough to reproduce and pass on its genes, and if it is not large and fitter than the
other individuals around it, it will not be able to compete for a mate. Many seal
species, for example, have only a single male who gets to mate. He must be the very
fittest seal to win all the females.
B The very fittest organisms are not necessarily the ones that survive. Sometimes it is
the least fit organisms that survive and reproduce. For example, in one generation, the
mice that are bad at foraging for seeds may reproduce prolifically and dominate the

********************************************************
****************************************************************405****

mice that are good at foraging. In this case, natural selection will select for the less-fit
phenotype and spread it in the population.
C The definition of fitness is not correct. The strongest and fastest organisms are more
fit than the weaker and slower ones, but large individuals are often at a disadvantage
to smaller ones because they are easily spotted by predators. For example, a large
rabbit will stick out on a field more than a small one and will get eaten by a hawk.
D What is meant by fittest is not necessarily strong, large, and fast. Fitness, as defined in
evolutionary terms, has to do with survival and the reproduction of genetic material.
For example, a small but showy male bird may be selected by female birds to
reproduce, while a large but less colorful one is not.
Solution The solution is (D). Although survival of the fittest is a commonly used catchphrase
when discussion natural selection and evolution, the phrase is a bit misleading. It is
not only the very fittest that survive—many organisms that seem less fit than others
may survive and reproduce. Further, what is meant by fittest is not necessarily
strong, large, and fast. Fitness, as defined in evolutionary terms, has to do with both
survival and the reproduction of genetic material. A small but showy male bird may
survive and reproduce while a large but less colorful one does not. A thin, short-
living plant may yield plentiful seeds while a larger, longer-living plant does not.
Some of the largest organisms to walk Earth, the dinosaurs, were fit for a time. Not
many organisms of that size exist today—and certainly not in those numbers. What
is fit at one point in time is not what is fit in another—and some of the most
successful and prolific organisms ever are found among the microorganisms and
invertebrates.
58 A student placed 20 tobacco seeds of the same species on moist paper towels in each of
two petri dishes. Dish A was wrapped completely in an opaque cover to exclude all light.
Dish B was not wrapped. The dishes were placed equidistant from a light source set to a
cycle of 14 h of light and 10 h of dark. All other conditions were the same for both dishes.
The dishes were examined after seven days, and the opaque cover was permanently
removed from dish A. Both dishes were returned to the light and examined again at
14 days. The following data were obtained:

********************************************************
***************************404*****************************************

What best supports the hypothesis that the difference in leaf color is genetically
controlled?
A The number of yellow-leaved seedlings in dish A on day 7
B The number of germinated seeds in dish A on days 7 and 14
C The death of all the yellow-leaved seedlings
D The existence of yellow-leaved seedlings as well as green-leaved ones on day 14 in
dish B
Solution The solution is (D). The existence of yellow-leaved seedlings as well as green-leaved
ones on day 14 in dish B.
59 A student placed 20 tobacco seeds of the same species on moist paper towels in each of
two petri dishes. Dish A was wrapped completely in an opaque cover to exclude all light.
Dish B was not wrapped. The dishes were placed equidistant from a light source set to a
cycle of 14 h of light and 10 h of dark. All other conditions were the same for both dishes.
The dishes were examined after seven days, and the opaque cover was permanently

********************************************************
****************************************************************405****

removed from dish A. Both dishes were returned to the light and examined again at
14 days. The following data were obtained:

What best describes the usefulness of the yellow-leaved phenotype as a variation subject
to natural selection?
A The yellow-leaved phenotype can germinate in environments without light.
B The germination of the yellow-leaved phenotype is unaffected by light intensity.
C The germination of the yellow-leaved phenotype is accelerated as compared to the
green-leaved phenotype.
D The yellow-leaved phenotype cannot germinate in environments with light.
Solution The solution is (A). The yellow-leaved phenotype can germinate in environments
without light.

********************************************************
***************************404*****************************************

60 A student placed 20 tobacco seeds of the same species on moist paper towels in each of
two petri dishes. Dish A was wrapped completely in an opaque cover to exclude all light.
Dish B was not wrapped. The dishes were placed equidistant from a light source set to a
cycle of 14 h of light and 10 h of dark. All other conditions were the same for both dishes.
The dishes were examined after seven days, and the opaque cover was permanently
removed from dish A. Both dishes were returned to the light and examined again at
14 days. The following data were obtained:

Yellow-leaved seedlings are unable to convert light energy to chemical energy. Which
observation is most likely to be made on day 21?

********************************************************
****************************************************************405****

A A few yellow-leaved seedlings alive in dish A, but none in dish B


B A few yellow-leaved seedlings alive in dish B, but none in dish A
C No yellow-leaved seedlings alive in dish A or dish B
D A few yellow-leaved seedlings alive in dish A and dish B
Solution The solution is (C). If the seedlings are unable to convert light energy to chemical
energy, none will survive.
61 Populations of a nocturnal toad live along a long river. On the other side of a band of
territory that is about 10 km wide, there are populations of a toad that appear similar.
Which data would provide compelling evidence that the two populations represent
different species?
A The populations of toads on the other side of the banded territory are not completely
nocturnal.
B Fertile hybrid populations of toads are found between the two other populations.
C There appear to be some hybrid toads between the two populations, but they are few
and frail.
D The two populations of toads enact very different mating behaviors.
Solution The solution is (C). There appear to be some hybrid toads between the two
populations, but they are few and frail.
62 A group of students summarized information on five great extinction events. The students
are sampling a site in search of fossils from the Devonian period. Based on the chart, what
would be the most reasonable plan for the students to follow?

A Searching horizontal rock layers in any class of rock and trying to find those that
contain the greatest number of fossils
B Collecting fossils from rock layers deposited prior to the Permian period that contain
some early vertebrate bones

********************************************************
***************************404*****************************************

C Looking in sedimentary layers next to bodies of water in order to find marine fossils of
bivalves and trilobites
D Using relative dating techniques to determine the geological ages of the fossils found
so they can calculate the rate of speciation of early organisms
Solution The solution is (B). The most reasonable plan to follow would be collecting fossils
from rock layers deposited prior to the Permian period that contain some early
vertebrate bones.
63 Populations of a plant species have been found growing in the mountains at altitudes
above 2,500 m. Populations of a plant that appears similar, with slight differences, have
been found in the same mountains at altitudes below 2,300 m.
Which plan for collecting two kinds of data could provide a direct answer to the following
question: Do the populations growing above 2,500 m and the populations growing below
2,300 m represent a single species?
A Scientists could take the genetic code of a plant from each altitude and determine
whether the two sets of DNA are identical. They could also insert genes from one
plant into the cells from the other and see if the cells survive
B Scientists could look in the fossil record to find the plants’ most recent common
ancestor. They could also check the surrounding mountains to determine if the most
recent common ancestor is still living.
C Scientists could breed the two groups in the same environment and observe whether,
over several generations, they begin to look more similar. They could also switch the
groups, growing the high-altitude plants at low altitude and the low-altitude plants at
high altitude, and observe whether the former begin to look like low-altitude plants
and the latter begin to look like high-altitude plants.
D Scientists could collect seeds and test whether they might be cross-pollinated to
produce fertile offspring. They also could investigate the area between 2,500 m and
2,300 m to see if fertile hybrid populations might be found living between the two
other populations of plants.
Solution The solution is (D). Scientists could collect seeds and test whether they might be
cross-pollinated to produce fertile offspring. They could also investigate the area
between 2,500 m and 2,300 m to see if fertile hybrid populations might be found
living between the two other populations of plants.
64 Populations of a plant species have been found growing in the mountains at altitudes
above 2,500 m. Populations of a plant that appears similar, with slight differences, have
been found in the same mountains at altitudes below 2,300 m.
How do the two types of data from the previous exercise provide a direct answer to the
question of whether speciation has taken place?

********************************************************
****************************************************************405****

A If the plants become more similar when grown in the same environment, or if the
high-altitude plants respond to low altitude in the same way that low-altitude plants
have, and low-altitude plants respond to high altitude the same way that high-altitude
plants have, then the two groups have the same underlying genetic structure and
belong to one species.
B If the seeds from the plants can be cross-fertilized and developed into fertile offspring,
the two populations are not yet reproductively isolated and remain one species. If
hybrid forms are found, the two populations are not reproductively isolated and
hybrids are both viable and successful.
C If the genetic codes of the two plants are identical, then they must belong to the same
species. Also, if genes transplanted between the plants function successfully, then the
plants must be similar enough to each other to belong to the same species.
D If scientists are able to find the common ancestor of the two groups in the fossil
record or in neighboring communities, then they can determine whether the plants
have diverged into separate species or remain a single species.
Solution The solution is (B). Both of these data are consistent with definition of biological
species. If the seeds from the plants can be cross-fertilized and developed into fertile
offspring, the two populations are not yet reproductively isolated and remain one
species. If hybrid forms are found this provides the same direct answer: the two
populations are not reproductively isolated and hybrids are both viable and
successful.
65 Assuming a population that has genetic variation and is under the influence of natural
selection, place the following events in the order in which they would occur:
 Genetic frequencies within the population change.
 A change occurs in the population’s environment.
 Phenotypic variations shift.
 Individuals who are well-adapted leave more offspring than individuals who are
poorly adapted.
 Individuals who are poorly adapted do not survive at the same rate as individuals
who are well adapted.
A 1. A change occurs in the population’s environment.
2. Individuals who are poorly adapted do not survive at the same rate as individuals
who are well adapted.
3. Individuals who are well adapted leave more offspring than individuals who are
poorly adapted.
4. Genetic frequencies within the population change.
5. Phenotypic variations shift.

********************************************************
***************************404*****************************************

B 1. A change occurs in the population’s environment.


2. Genetic frequencies within the population change.
3. Phenotypic variations shift.
4. Individuals who are poorly adapted do not survive at the same rate as individuals
who are well adapted.
5. Individuals who are well adapted leave more offspring than individuals who are
poorly adapted.
C 1. Phenotypic variations shift.
2. A change occurs in the population’s environment.
3. Genetic frequencies within the population change.
4. Individuals who are poorly adapted do not survive at the same rate as individuals
who are well adapted.
5. Individuals who are well adapted leave more offspring than individuals who are
poorly adapted.
D 1. Individuals who are well adapted leave more offspring than individuals who are
poorly adapted.
2. Individuals who are poorly adapted do not survive at the same rate as individuals
who are well adapted.
3. Phenotypic variations shift.
4. Genetic frequencies within the population change.
5. A change occurs in the population’s environment.
Solution The solution is (A). The events in order of occurrence are as follows:
1. A change occurs in the population’s environment.
2. Individuals who are poorly adapted do not survive at the same rate as
individuals who are well adapted.
3. Individuals who are well adapted leave more offspring than individuals who
are poorly adapted.
4. Genetic frequencies within the population change.
5. 5. Phenotypic variations shift.
66 A biologist studies a population of voles for 20 years. During almost the entire research
period, the population stays between 50 and 75 individuals. Additionally, fewer than half
of the voles born do not survive to reproduce, due to predation and competition for food.
Then, in one generation, 80 percent of the voles born live to reproduce. The population
increases to 110 individuals.
What inferences about food and predation can you make for the singular generation in
which 80 percent of offspring survived? What prediction can you make about the genetic
and phenotypic variation of future populations for this group of voles?

********************************************************
****************************************************************405****

A Either there was less food available or the degree of predation increased. The future
generations of this group of voles should evidence less genetic variation.
B Either there was less food available or the degree of predation increased. The future
generations of this group of voles should evidence greater genetic variation.
C Either there was more food available or the degree of predation decreased. The future
generations of this group of voles should evidence less genetic variation.
D Either there was more food available or the degree of predation decreased. The future
generations of this group of voles should evidence greater genetic variation.
Solution The solution is (D). It is reasonable to infer that for the generation of voles in which
80 percent live and reproduce, there was more food available or the degree of
predation decreased. The future generations of this group of voles should evidence
greater genetic variation because there are more individuals surviving and passing
on their genetic information. If that variation persists (the population stays larger or
increases in size), it is likely that future populations will evidence a different variety
of phenotypes, as well.
67 There are years of drought in a small, relatively isolated community. During the
drought, small seeds with thin shells become rare. Large seeds with hard cases become
increasingly common. The large, tough seeds are successfully eaten by birds with large
and broad beaks.
Assuming that the drought continues and the population of birds in the community stays
isolated, what predictions for the population can you make under the influence of natural
selection?
A The birds with small, thin beaks will grow larger, broader beaks to be able to eat the
larger seeds. This will result in subsequent generations having a higher percentage of
birds with large, broad beaks.
B There will be more birds with small, thin beaks dying and more birds with large, broad
beaks surviving. Differential reproduction of birds with large, broad beaks will result in
subsequent generations having a higher percentage of birds with large, broad beaks.
C The species will diverge into two species, one with small, thin beaks and one with
large, broad beaks. The two species will then compete for resources.
D There will be neither phenotypic nor genotypic changes in the population.
Solution The solution is (B). There will be more birds with small, thin beaks dying and more
birds with large, broad beaks surviving. Differential reproduction of birds with large,
broad beaks will result in subsequent generations having a higher percentage of
birds with large, broad beaks. If the drought and isolated conditions persist, it is
possible that the genetic variation of the population will shift such that the
configuration of the small, thin beaked bird is increasingly rare or even no longer
present in the gene pool.

********************************************************
***************************404*****************************************

68 At one time, avian researchers in the Sulawesi region of Indonesia described the
Flowerpecker populations on the mainland and the Wakatobi archipelago as one species.
A recent reassessment of the Wakatobi populations resulted in the suggested
reclassification of these populations as a distinct species, the Wakatobi Flowerpecker.
Which piece of evidence, if true, would be cause for this reclassification?
A The populations have become dependent on the island food sources.
B The populations have become morphologically distinct from the mainland species.
C The populations have become adapted to the island habitat.
D The populations have become reproductively isolated from the mainland species.
Solution The solution is (D). The populations have become reproductively isolated from the
mainland species.
69 What pattern in the fossil record would you expect to see to support the model of gradual
speciation? How would you expect this pattern to differ from a pattern in the fossil record
that supports the model of punctuated equilibrium? Explain.
A In the case of gradual speciation, the fossil record would show only a few hybrid
individuals, followed by individuals of the two distinct species. For the case of
punctuated equilibrium, the fossil record would show many hybrid individuals
persisting through several geological layers.
B In the case of gradual speciation, the fossil record would show the parent species in a
single location, such that the newly diverged species remained in contact with each
other. For the case of punctuated equilibrium, the fossil record would show a
geographic divide within the parent species that caused it to diverge into multiple new
species.
C In the case of gradual speciation, the fossil record would show many intermediate
forms. For the case of punctuated equilibrium, the fossil record would show new
forms that persist essentially unchanged through several geological layers, then
disappear just as a new form appears.
D Gradual speciation would be undetectable in the fossil record. For the case of
punctuated equilibrium, the fossil record would show a steady progression of
distinct forms.
Solution The solution is (C). In the case of gradual speciation, the fossil record would show
many intermediate forms. The changes between them would be notable, but they
would be gradual and perhaps span many, many geological layers. In contrast, for
the case of punctuated equilibrium, the fossil record would show the sudden
appearance of a new form. This new form would persist essentially unchanged
through several geological layers, then disappear just as another new form appears.

********************************************************
****************************************************************405****

70 Until recently, these three species of short-tailed pythons, Python curtus, Python
brongersmai (middle), and Python breitensteini were considered one species. However,
due to the different locations in which they are found, they have become three distinct
species. What is this an example of?
A Divergent evolution
B Sympatric speciation
C Allopatric speciation
D Variation
Solution The solution is (C). This refers to speciation that occurs through geographic
separation.
71 Consider two species of birds that diverged while separated geographically but resumed
their contact before reproductive isolation was complete. Which option describes the first
step in what would happen over time if the two species mated extensively and their
hybrid offspring survived and reproduced more poorly than offspring from intraspecies
mating?
A Natural selection would cause prezygotic barriers to reproduction between the parent
species to strengthen over time.
B The production of unfit hybrids would increase and the speciation process would
complete.
C The extensive mating between the species would continue to produce large numbers
of hybrids.
D The gene pools of the parent species would fuse over time, reversing the speciation
process.
Solution The solution is (A). Natural selection would cause prezygotic barriers to reproduction
between the parent species to strengthen over time.

SCIENCE PRACTICE CHALLENGE QUESTIONS


18.1 Understanding Evolution
72 In addition to biology, evidence drawn from many different disciplines, including
chemistry, geology, and mathematics, supports models of the origin of life on Earth. In
order to determine when the first forms of life likely formed, the rate of radioactive decay
can be used to determine the age of the oldest rocks (see optional parts C and D) exposed
on Earth’s surface. These are found to be approximately 3.5 billion years old. The age of
rocks can be correlated to fossils of the earliest forms of life.

********************************************************
***************************404*****************************************

A. The graph compares times of divergence from the last common ancestor based on the
fossil record with a “molecular time” constructed by comparing sequences of conserved
proteins to determine a mutation rate (after Hedges and Kumar, Trends in Genetics,
2003). Explain how such a molecular clock could be refined to infer time for the evolution
of prokaryotes.

B. Using a molecular clock constructed from 32 conserved proteins, Hedges and


colleagues (Battistuzzi et al., BMC Evol. Biol., 2004) estimated the times during which key
biological processes evolved. A diagram based on their work is shown. Connect the time
of the origin of life inferred from this diagram with the age of the oldest fossil
stromatolites and the age of the oldest exposed rock to show how evidence from different
scientific disciplines provides support for the concept of evolution. Evaluate the legitimacy
of claims drawn from these different disciplines (biology, geology, and mathematics)
regarding the origin of life on Earth.

********************************************************
****************************************************************405****

The oldest known rocks are exposed at three locations: Greenland, Australia, and
Swaziland. The following application of mathematical methods provides the essential
evidence of the minimum age of Earth. The mathematics is appropriate for students who
have completed a second year of algebra. However, it is not illustrative of the type of item
that could appear on the AP Biology Exam.
The exposed rocks contain a radioactive isotope of rubidium, 87Rb, which decays into a
stable isotope of strontium, 87Sr. An 87Rb atom with 37 protons and 50 neutrons decays
when a proton is converted into a neutron to produce an atom, 87Sr, with 36 protons and
51 neutrons. As time passed, the number of each isotope changed from its initial value.
When a crystal containing 87Rb atoms formed from the molten surface of the hot, early
Earth during the Hadean eon, the number of these atoms at that initial time can be
represented as N87Rb,0. As time passed, the number of atoms of this isotope changed to
N87Rb.
C. Justify the relationship between the number of each isotope at any time and the
number of each at the time that the molten rock solidified (denoted by the subscript 0):

The decay of unstable radioisotopes is exponential with a half-life of T1/2, which for 87Rb is

This can be used to replace the initial number of 87Rb atoms, which cannot be measured,
with the present-day value:

When the measurements of the numbers of 87Rb and 87Sr were made (Moorbath et al.,
Nature, 1972), measurements of a second stable isotope of strontium, 86Sr, also were
made. The ratio of the initial number of 87Sr and 86Sr atoms is the same as today, since the
isotopes are both stable. The value of this ratio is 0.71.

This is a linear equation in the form where a is the term in parenthesis


containing the half-life of 87Rb. If is graphed versus , the slope
can be used to determine the time, t, that has passed since the rock formed from melting:

so

********************************************************
***************************404*****************************************

D. Data on the rubidium and strontium isotopes at Isua in Greenland are provided in the
table. Analyze these data to obtain the age of formation of these rocks.
N87Rb/N86Sr N87Sr/N86Sr
0.212 0.711
0.214 0.711
0.223 0.712
0.259 0.714
0.268 0.714
0.267 0.715
0.290 0.716
0.394 0.720
0.434 0.723
The solidification of the molten surface of Earth at the end of the Hadean eon (4 to
4.6 billion years ago) and the condensation of liquid oceans provided a medium from
which life emerged. The most ancient fossils are colonial, photosynthetic cyanobacteria
called stromatolites. As climate change melted the perennial snow covering Greenland,
new geologic evidence of the time of that origin was obtained (Nutman et al., Nature,
2016) with the discovery of the most ancient stromatolites. These fossils record
communities of photosynthetic bacteria embedded in Isua sediments 3.7 billion years ago.
Worldwide stromatolite fossils show a decline between 1 and 1.3 billion years ago.
Solution Sample answer:
A. By comparing sequence differences in conserved proteins among prokaryotes,
using such a molecular clock, the time separating last common ancestors among
prokaryote groups can be inferred.
B. At the extrapolation of these estimates for the emergence of biological innovation
lies the ultimate biological innovation, life. A reasonable interval is between 4.5 and
4.0 bya. Interestingly, this is significantly older than the oldest surface rocks, leading
many to conjecture that life emerged during the Hadean. The oldest surface rocks
only provide an upper bound on the formation of solid Earth since these are just
those that are exposed. The proliferation of cyanobacteria is unlikely to be a very
early phenomenon, so the 3.7 bya fossils. So, all three of these disciplines have
produced results which are complementary and providing evidence of both the
concept of evolution and time of the origin of life on Earth.
C. The left-hand side of this equality is the number of 87Sr atoms in a sample of rock.
These were either present initially, 87Sr0, or were formed from the decay 87Rb. The
number of 87Rb that have decayed is the difference between the initial number,
87
Rb0, and the current number of 87Rb.

********************************************************
****************************************************************405****

D. A graph should be constructed such as the one below except that axes should be
labeled. The independent variable is the ratio of 87Rb to 86Sr. The slope is the factor
that depends on the time interval that has elapsed since the solidification of the
melt to form these rocks. Using the half-life of 87Rb, the age of the rocks can be
determined.

0.724
0.722 f(x) = 0.0513512544963425 x + 0.70049882635943
0.72
0.718
0.716
0.714
0.712
0.71
0.708
0.706
0.704
0.2 0.25 0.3 0.35 0.4 0.45

73 In 1952, the Miller-Urey experiment showed that an electrical discharge in a gas-phase


mixture of ammonia, hydrogen, methane, and water produced five amino acids. When
the experiment was conducted, evidence indicated that this mixture was representative
of the Hadean (early Earth) atmosphere. The experiment was repeated in the presence of
jets of hot steam, simulating Hadean volcanic eruptions and producing an even larger
variety of amino acids.
A. Consider the following criticisms of the “organic soup” model and justify the selection
of data that other experiments might provide regarding the origin of life on Earth.
 Biopolymers on Earth have a left-hand symmetry at the carbon adjacent to the
carboxylic acid carbon, and these experiments produced mixtures of both left-
and right-hand symmetries.
 No peptide bonds between amino acids were observed.
 Early Earth’s atmospheric oxygen concentration is known to have been very low,
implying the absence of an ozone layer to filter high-energy ultraviolet (UV)
radiation.
 Ammonia decomposes when it absorbs high-energy UV radiation, but diatomic
nitrogen does not.
Models of the abiotic synthesis of biomolecules suffer from a “chicken and egg” dilemma.
Proteins are needed to synthesize DNA and RNA, and DNA and RNA are needed to
synthesize proteins. Which molecules came first?

********************************************************
***************************404*****************************************

B. In light of the following observations, evaluate the hypothesis that nucleotides arose
from a prebiotic mixture.
 Nuclei acids are not found in experiments like those of Miller and Urey.
 Purines and pyrimidines decompose at high temperature, and Earth was
bombarded by meteors and comets during the Hadean eon.
 Bonds in the purine and pyrimidine rings of nucleic acids are broken by high-
energy UV radiation.
 Carl Sagan and colleagues synthesized ATP from a mixture of adenosine, ribose,
and phosphate when exposed to UV radiation.
 Ribose has never been synthesized in experiments like those conducted by Miller
and Urey.
 Ribose has a left/right symmetry, and the right-handed form occurs in Earth
organisms.
Continuing with the analogy, if neither the chicken nor the egg came first, then both must
have arisen together. Some regard simultaneous innovations in both catalysis and
information storage and retrieval as too improbable. In samples of meteorites, both
amino acids and nucleic acids have been found. The amino acids are mixtures of left- and
right-handed symmetries, although some have shown a significant bias toward the left-
handed form (J. Elisa et al., ACS Central Science, 2016). The arrival from space of the seeds
of biomolecules is called panspermia. Carl Sagan (1966) and Francis Crick (1973), one of
the first to describe the structure of DNA, regarded panspermia as the only plausible
origin of life on Earth. In fact, their belief was in directed panspermia, the intentional
seeding by intelligent aliens.
C. Describe the questions that must be addressed for panspermia to be a scientific
hypothesis about the origin of life on Earth and describe the reasons for the directed
panspermia revision of this hypothesis.
To avoid the conflicting chicken-and-egg claims that “protein catalyst was first” and “DNA
information storage was first,” two alternatives have emerged regarding the origin of life
on Earth. Consider two simple ideas: (1) water blocks UV radiation, and cracks in the
ocean floor (hot vents) provide a temperature difference that generates a source of
entropy; and (2) ribosomes are composed of RNA.
D. Describe one of the following as a hypothesis concerning the origin of life on Earth:
 Reactions among molecules in the vicinity of hot vents became organized in
space and time, eventually developing structures that foreshadow the proton
gradient upon which metabolism is based. This alternative is the basis for what is
referred to as the metabolism-first hypothesis.

********************************************************
****************************************************************405****

 The catalytic properties of the ribosome reflect the self-catalytic polymerization


of nucleotides with sequential structures conserved in modern DNA, the catalytic
properties conserved in proteins, and the catalytic properties of the ribosome
whose core structure is RNA. This alternative is the basis for what is referred to
as the RNA-first hypothesis.
Solution Sample answer:
A. N2 is even more likely to be the predominant nitrogen molecule in the Hadean
atmosphere and less likely to react to form amino acids. The experiment should be
conducted with N2 and perhaps even more energetic radiation, such as X-rays. In the
gas phase, there is no mechanism to bias the result toward either left- or right-
handed products. If a surface—such as clay, mica, or metals—were present then the
left-handed form might be favored. Also, experimental designs need to probe the
way that surfaces might lead to polymerization of the amino acids.
B. Polymerization of nucleotides, once they formed, would a very low probability
event and that means very long times would be required. Whenever an impact
occurred, the clock would be reset. Somehow the purine and pyrimidine products
would need to be hidden from the UV radiation—though a report
(http://dx.doi.org/10.1016/j.jphotochemrev.2008.12.001) has suggested that the
five nucleic acids found in DNA and RNA are products of selection under pressure
from UV radiation. Ribose has a left/right symmetry so synthesis in solution is not
likely to favor the right-handed form. Sagan (1973) suggested that prokaryotes
protected the bases from radiation by sinking in the primordial oceans while pre-
eukaryotes remained near the surface in protective cages of radiation-absorbing
nucleic acids. This “geographic separation” led to the division. This probably was
dinner conversation between Sagan and his wife, Lynn Margulis, who later published
the accepted endosymbiotic explanation.
C. To be a valid scientific hypothesis panspermia needs to be testable. Or one must
restate the model in a way that can be tested. For example, one could make the
claim that nucleic acids found in meteorites can be extracted under pre-biotic
conditions and, given a pathway to the synthesis of ribose, can form nucleotides that
polymerize in the presence of phosphates. This claim, given the challenges
summarized in this problem, is not likely to be tested in the near future. So, those of
us who want answers to questions beyond the scope of science in our day
sometimes pose conjectures that satisfy this need, such as directed panspermia.
D. The assessment rubric would require that they state the hypothesis and sketch
the means by which the hypothesis would be tested.
Metabolism-first: The free energy source of a hot vent creates a resource: the flow
of energy through a system can lead to structure. This is similar to other ecosystems
but in the pre-biotic world the components of the ecosystem are molecular. The
proton gradient is the essential structure for all life on Earth—both in the synthesis
and degradation of carbohydrates. It could be claimed that under laboratory

********************************************************
***************************404*****************************************

conditions with a mixture representative of a deep Hadean ocean reduction


oxidation reaction pairs coupled by proton movement could be observed.
RNA-first: RNA plays the dual role of information storage and catalysis. So rather
than the simultaneous emergence of both functions, RNA specializes to fill each role
with new structures. It could be claimed that given a mixture composed of an RNA
sequence, the RNA core of a ribosome, the DNA bases as nucleotides, and amino
acids but no protein catalysts the protein coded for by the RNA could be produced.
74 The radiant energy emitted by a star gradually increases after its birth. During the Hadean
eon, while the molten Earth cooled and life emerged, the sun provided approximately
25 percent less radiant energy than it does now. Ignoring effects due to differences in the
composition of Earth’s atmosphere between then and now, this means that the average
surface temperature of the surface would be about 25 °C below the freezing temperature
of water. Evidence of liquid water on Earth during the Hadean eon is provided by geologic
structures known only to form in liquid water, such as lava pillows and the stromatolites
that are the fossilized layers of photosynthetic cyanobacteria.
Pose a scientific question that guides inquiry into early Earth conditions that supported
the innovation of photosynthesis.
Solution Sample answer: Just as the Earth’s temperature is currently increasing as we
increase the concentration of carbon dioxide in the atmosphere, could the
composition of the early atmosphere be largely carbon dioxide and water, two
greenhouse gases that trapped energy, elevating the temperature?
75 Connect the techniques of radiometric measurement, anatomy, and molecular biology to
the supporting evidence of the theory of evolution provided.
Solution Sample answer: The AP Biology Curriculum Framework is very specific about the
scope of essential content knowledge regarding three techniques that provide
evidence of evolution:
 Fossils can be dated by a variety of methods that provide evidence for
evolution. These include the age of the rocks where a fossil is found, the rate
of decay of isotopes including carbon-14, the relationships within
phylogenetic trees, and the mathematical calculations that take into account
information from chemical properties and/or geographical data. The details
of these methods are beyond the scope of this course and the AP Exam.
 Morphological homologies represent features shared by common ancestry.
Vestigial structures are remnants of functional structures, which can be
compared to fossils and provide evidence for evolution.
 Biochemical and genetic similarities, in particular DNA nucleotide and protein
sequences, provide evidence for evolution and ancestry.

********************************************************
****************************************************************405****

76 Describe reasons for the revision of scientific hypotheses of the origin of life on Earth.
Solution Sample answer: There are few places in the AP Biology Curriculum Framework
where the scope of content targeted by a declarative claim (describe) is not spelled
out for you. In this case, there is no reference to any particular theory and it would
be out of scope to assume prior knowledge of, for example, the Aristotelian model.
And there is no help provided in the description in the Framework of enduring
understanding 1D. And there is no supporting discussion of the nature of science
that would clarify the meaning or general features of “scientific hypothesis.” Some
alternative hypotheses regarding RNA worlds, panspermia, and directed panspermia
were described in problem 2 in this chapter.
An alternative interpretation that might lead to an Exam item could be implied by
this task: Describe reasons for the development of hypotheses of the origin of life on
Earth that are scientific. This would assess the student’s understanding that
evidence is required to justify a claim. That is explicit in the description of Science
Practice 6 in the Framework.
77 Directed evolution is an inquiry strategy that is usually used to investigate gene
expression or the function of proteins that are expressed. The investigator imposes a
selection pressure and observes the evolution of a population. In one investigation,
unicellular yeast was allowed to sediment in a column that contained nutrients at its
bottom. Yeast that reached the nutrients at the bottom was removed, weighed, and
examined under the microscope. After 60 generations, it was found that all of the
removed yeast was multicellular. To test the claim that their selection pressure favored
multicellularity, the investigators performed another experiment. In one column, they
provided a strong selection pressure, in which they only allowed 5 min for yeast to settle
before removing what had traveled the farthest toward the bottom. In a second column,
they provided a weak selection pressure, where they allowed 25 min for the yeast to
settle before removal. Strong selection resulted in more massive clusters of multicellular
yeast among the removed cells. Weak selection resulted in less massive multicellular
clusters among the removed cells.
A. Evaluate the claim that the use of both a strong and weak selection demonstrates that
evolution is an ongoing process that, under artificially imposed conditions, led to the
emergence of multicellularity in a single-celled organism.
B. In this directed evolution study, the selection pressure imposed by the investigators led
to a new phenotype. Consider a situation in which there is a vertical variation in the
density of nutritional resources. Analyze the advantages and disadvantages of
cooperative behavior, including changes in the likelihood of replication of the individual
and population genomes.

********************************************************
***************************404*****************************************

Solution Sample answer:


A. Yes, because the results of the second experiment demonstrate that
multicellularity in yeast responded differently to the selection pressure when it was
strong versus weak. In both experiments, the highest mass multicellular clusters will
settle fastest and, therefore, be favored by selection because they reach the
nutrients fastest. In the second experiment, when yeast is allowed only 5 min to
settle, the yeast removed will only be the largest multicellular clusters. When yeast
is allowed 25 min to settle, smaller multicellular colonies and possibly single cells will
have enough time to settle and be removed. Therefore, the yeast responded to
strong versus weak selection pressure, suggesting that the selection pressure was
favoring multicellularity.
B. If the density of nutrients is highest at the bottom of the column, multicellularity
will be favored because the most massive multicellular colonies will reach the
nutrients first. This will increase the frequency of genes that cause cells to engage in
multicellularity within this population of yeast. If the density of nutrients is highest
on the top of the column, multicellularity will not be favored because cells that sink
will not reach the nutrients. This will decrease the frequency of genes that cause
cells to engage in multicellularity within this population of yeast.
18.2 Formation of New Species
78 Selection processes in changing and unchanging environments differ. Connect the effects
of negative and positive selection pressures to changes in the environment.
Solution Sample answer: The concept of homeostasis is strongly emphasized in the AP
Biology Curriculum as is the theory of evolution. Classroom activities will have
supported the idea that many homeostatic mechanisms are conserved through
evolution and environmental changes select for mutations that have an advantage
as these changes occur. An item targeting this LO would provide an opportunity for
the student to summarize an illustrative example from those activities.
79 In biology, the word “race” is rarely used. It could be imagined to be synonymous with a
subspecies. Species is well defined, at least when horizontal gene transfer is not taken
into account, by reproductive isolation. Speciation may arise through geographic isolation.
A. Aside from geographic isolation leading to reproductive isolation, predict two other
mechanisms of speciation in a population and suggest how these mechanisms could lead
to a scientific definition of a subspecies.
B. If individuals of a species become separated by a natural disaster, they could become
increasingly different. Assuming that they could still interbreed, what would you predict
about the consequences if females, but no males, from one population (population A)
were introduced to the other population (population B)? What would be the effects on
autosomal, X, and Y chromosomes?

********************************************************
****************************************************************405****

Solution Sample answer:


A. In addition to geographic isolation, other factors that cause individuals to stop
interbreeding could cause speciation. For example, disruptive selection favoring
individuals that forage in the morning and those that forage at night could
eventually result in two populations that do not come into contact to breed. Over
time, they could become sufficiently different to be considered subspecies or
eventually species. The same thing could happen through strong sexual selection if
different groups preferred different traits.
B. Females cannot introduce Y chromosomes, so all of the Y chromosomes in the
newly mixed population B would be from the original population B. The autosomal
and X-linked characteristics of population A would be mixed into population B.

********************************************************
**************************440*************************************

19 | THE EVOLUTION OF POPULATIONS


REVIEW QUESTIONS
1 What is the ultimate source of all variation in and among populations?
A Genetic mutations that result in viable offspring
B Natural selection
C Diverse habitats
D Factors in the environment that may affect development
Solution The solution is (A). Genetic mutations lead to changes in alleles and the frequency of
alleles, contributing greatly to variation in and among populations.
2 When male lions reach sexual maturity, they are thrown out of their group, or pride, and
must live on their own or with other males until they can take over their own pride. This
can alter the allele frequencies of the population through which mechanism?
A Natural selection
B Gene flow
C Random mating
D Genetic drift
Solution The solution is (B). Gene flow is the movement of alleles into (or out of) a population
when an individual moves into (or out of) a population. When a male lion moves into
a pride, it brings in new alleles to that population.
3 Which population has violated the conditions of Hardy-Weinberg equilibrium?
A An infinitely large population
B A population in which the allele frequencies do not change over time
C A population in which the Hardy-Weinberg equation is equal to 1
D A population undergoing natural selection
Solution The solution is (D). During natural selection, allele frequencies of a population
change. A population in equilibrium is in a stable, nonevolving state.
4 What is the difference between micro- and macroevolution?
A Microevolution describes the evolution of small organisms, such as insects, while
macroevolution describes the evolution of large organisms, such as people and
elephants.
B Microevolution describes the evolution of microscopic entities, such as molecules and
proteins, while macroevolution describes the evolution of whole organisms.

********************************************************
************************************************************439***

C Microevolution describes the evolution of organisms in populations, while


macroevolution describes the evolution of species over long periods of time.
D Microevolution describes the evolution of organisms over their lifetimes, while
macroevolution describes the evolution of organisms over multiple generations.
Solution The solution is (C). Microevolution describes how a population changes over time,
whereas macroevolution describes how new species arise.
5 What would be supported by Lamarck?
A Natural selection leads to changes in organisms over time.
B The strong arms of a gorilla are the result of its parents constantly climbing, lifting, and
fighting.
C Lack of resources led to the death of three of four fox cubs.
D The founder effect is when a few individuals in a population are separated from the
original population.
Solution The solution is (B). Lamarck believed that traits that were acquired by a parent (such
as strong arms) would be passed along to their offspring.
6 What is population variance influenced by?
A Genetic structure
B Environment
C Diet composition
D All of the above
Solution The solution is (D). Genetic structure and environment are both factors that
influence population variance. Genetic structure determines how many genes are
present in the population and at what frequencies, while the environment creates
natural selection pressure that changes genetic variance.
7 What is genetic variance?
A The change in a population’s genetic structure
B The effect of chance on a population’s gene pool
C The diversity of alleles and genotypes within a population
D The magnification of genetic drift as a result of natural events or catastrophes
Solution The solution is (C). This is the definition of genetic variance, which often enters into
discussions of breeding. In order to prevent inbreeding, breeders try to increase a
population’s genetic variance.

********************************************************
**************************440*************************************

8 When closely related individuals mate with each other, or inbreed, the offspring are often
not as fit as the offspring of two unrelated individuals. Why?
A Inbreeding causes normally silent alleles to be expressed.
B The DNA of close relatives reacts negatively in the offspring.
C Inbreeding can bring together rare, deleterious mutations that lead to harmful
phenotypes.
D Close relatives are genetically incompatible.
Solution The solution is (C). Inbreeding depression occurs when two carriers of a deleterious
recessive mutations mate. Related individuals are more likely to carry the same
deleterious mutation. Carriers of the mutation do not phenotypically express the
mutation, but when both parents have the mutation, it may be expressed in the
offspring who will not be as fit as the parents.
9 What could cause genetic drift to occur within a population?
A Accidental deaths
B Predators
C Disease
D Lack of gene flow
Solution The solution is (A). Genetic drift is the change in allele frequencies due to chance.
Accidental death is a random, chance event, whereas deaths due to predators or
disease are not random.
10 What is the evolutionary mechanism that alters allele frequencies by chance called?
A Genetic drift
B Natural selection
C Inbreeding
D Migration
Solution The solution is (A). Genetic drift is the effect of chance on a population’s gene pool.
11 What is assortative mating?
A When individuals mate with those who are similar to themselves
B When individuals mate with those who are dissimilar to themselves
C When individuals mate with those who are most fit in the population
D When individuals mate with those who are least fit in the population
Solution The solution is (A). Assortative mating refers to an individual’s preference to mate
with partners who are phenotypically similar to himself or herself.

********************************************************
************************************************************439***

12 What is an example of a cline?


A A random fluctuation in a species’ gene frequencies
B A mutation that spreads across the ecological range of a species
C The females of a species preferring males that are orange in coloration instead
of white
D Species having greater cold tolerance in the colder parts of its range than in the
warmer parts of its range
Solution The solution is (D). A cline is a type of geographical variation where a species varies
gradually across an ecological gradient. Temperature is an ecological gradient and
species subject to colder temperatures are more cold tolerant.
13 Which type of selection results in greater genetic variance in a population?
A Stabilizing selection
B Directional selection
C Diversifying selection
D Positive frequency-dependent selection
Solution The solution is (C). Diversifying selection is a type of selection that favors more than
one distinct phenotype, resulting in increased genetic variance as the population
becomes more and more diverse.
14 What types of phenotypes does negative frequency-dependent selection favor?
A Advantageous
B Rare
C Common
D Disadvantageous
Solution The solution is (B). Negative frequency-dependent selection favors rare phenotypes.
It is a type of selection that increases the genetic variance within a population
through the selection of rare phenotypes.
15 The good genes hypothesis is a theory that explains what?
A Why more fit individuals are more likely to have more offspring
B Why alleles that confer beneficial traits or behaviors are selected for by natural
selection
C Why some deleterious mutations are maintained in the population
D Why individuals of one sex develop impressive ornament traits

********************************************************
**************************440*************************************

Solution The solution is (D). The good genes hypothesis states that males show off their
healthy genes through the development of ornaments. For example, male lions with
darker, fuller manes are selected by females over males with lighter, thinner manes.
The males with dark, full manes have displayed that they are in good health and are
able to father healthy offspring.
16 Which option describes when males and females of a population look or act differently?
A Sexual selection
B Diversifying selection
C Sexual dimorphism
D A cline
Solution The solution is (C). Sexual dimorphism concerns differences between the sexes, such
as the difference in color of male and female birds.

CRITICAL THINKING QUESTIONS


17 What is natural selection? Give an example of natural selection at work in a population.
A The process in which genes flow from one population to another; the beak size of
Darwin’s finches changing as the availability of different-sized seeds changes.
B The process in which genes flow from one population to another; the founder effect
occurring among humans immigrating to a new country.
C The process in which better-adapted organisms are able to survive and reproduce; the
beak size of Darwin’s finches changing as the availability of different-sized seeds
changes.
D The process in which better-adapted organisms are able to survive and reproduce; the
founder effect occurring among humans immigrating to a new country.
Solution The solution is (C). Natural selection occurs when individuals with favorable genetic
traits are better able to survive and reproduce. Natural selection leads to
evolutionary change. Natural selection of Darwin’s finches occurred because
individuals with beaks better adapted to different-sized seeds were selected for and
so different species arose.
18 Imagine you are trying to test whether a population of flowers is undergoing evolution.
You suspect there is selection pressure on the color of the flower: bees seem to cluster
around the red flowers more often than the blue flowers. In a separate experiment, you
discover that blue flower color is dominant to red flower color. In a field, you count 600
blue flowers and 200 red flowers.
What would you expect the genetic structure of the flowers to be?

********************************************************
************************************************************439***

A You would expect 300 homozygous dominant blue flowers, 300 heterozygous blue
flowers, and 200 homozygous recessive red flowers.
B You would expect 200 homozygous dominant blue flowers, 400 heterozygous blue
flowers, and 200 homozygous recessive red flowers.
C You would expect 100 homozygous dominant red flowers, 100 heterozygous red
flowers, and 600 homozygous recessive blue flowers.
D You would expect 14 homozygous dominant red flowers, 186 heterozygous blue
flowers, and 600 homozygous recessive blue flowers.
Solution The solution is (B). Red is recessive, so all red flowers would be homozygous
recessive (bb). The blue flowers would be either BB or Bb.
19 What must occur in order for a new trait to appear in a population and then reach a
steady, high frequency within that population?
A New traits appear through gene mutations or through genetic drift. In order to reach a
steady, high frequency in the population, there must be many mutagens, such as UV
radiation, in the environment to produce many mutations.
B New traits appear through gene mutations or through genetic drift. In order to reach a
steady, high frequency in the population, there must be a consistent source of
immigrant individuals with the allele.
C New traits appear through gene mutations or through evolution. In order to reach a
steady, high frequency in the population, the allele must code for a favorable
adaptation.
D New traits appear through gene mutations or through gene flow. In order to reach a
steady, high frequency in the population, the trait associated with the gene must be
favored by either natural or sexual selection.
Solution The solution is (D). New traits appear through gene mutation and also through gene
flow, which is the introduction of new alleles into a population through immigration
of individuals or gametes. These traits are then incorporated into the population
only if they are selected for by natural or sexual selection.
20 What is population variation? Identify an example.
A Population variation is a description of the diversity of different forms of life. An
example of population variation would be the different forms and functions of
prokaryotes versus eukaryotes.
B Population variation is the geographic distribution of different phenotypes in a
population. An example of population variation would be the fact that warm-blooded
mammals that live near the poles tend to be larger than their southern counterparts
to conserve heat.

********************************************************
**************************440*************************************

C Population variation is the distribution of phenotypes in a population. An example of


population variation would be the many different fur colors and patterns found in
domestic dogs.
D Population variation is the distribution of genotypes in a population. An example of
population variation would be Mendel’s pea plants that were homozygous dominant,
heterozygous, and homozygous recessive for various traits.
Solution The solution is (C). Population variation is the distribution of phenotypes (how
individuals look) within a population. Domestic dogs, with their different colors and
patterns of fur, have different phenotypes and are therefore examples of population
variation.
21 People who breed domesticated animals try to avoid inbreeding even though most
domesticated animals are indiscriminate. Why this is a good practice?
A A breeder would not allow close relatives to mate because inbreeding increases the
likelihood of fatal mutations in offspring.
B A breeder would not allow close relatives to mate because inbreeding prevents gene
flow which can bring new, successful genes into the population.
C A breeder would not allow close relatives to mate because inbreeding causes
diversifying selection, which dilutes the breeder’s desired genes in the population.
D A breeder would not allow close relatives to mate because inbreeding can bring
together deleterious recessive mutations that can cause abnormalities and
susceptibility to disease.
Solution The solution is (D). A breeder would want to avoid inbreeding depression.
Inbreeding depression occurs when two closely related individuals who carry the
same recessive, deleterious mutation mate. This can result in offspring that have
abnormalities or are more susceptible to disease.
22 What is the founder effect? Identify an example.
A The founder effect is an event that isolates part of a population, generating an allele
frequency which is not typical of the original population. An example of the founder
effect is the Amish population. The Amish population was established from about 200
German immigrants. Individuals of this founding population carried gene mutations
that cause inherited disorders such as Ellis-van Creveld syndrome. This form of
dwarfism is found in a large concentration in the Amish population today because the
immigrants that established the population had a high concentration of the disorder in
a very small population.
B The founder effect is an event that kills off a significant proportion of a population,
generating an allele frequency which is not typical of the original population. An
example of the founder effect is the Northern elephant seal. At one point, hunting of
these seals had reduced their numbers to less than 50 individuals. The population has

********************************************************
************************************************************439***

since rebounded, but still contains less genetic variation than the related Southern
elephant seal due to the loss of some alleles.
C The founder effect is when only a few males within a population are selected by
females to reproduce, generating an allele frequency which is different from the
original population. An example of the founder effect is the reproductive pattern of
mountain gorillas. Mountain gorillas tend to have a single dominant male, the
silverback, who gets the vast majority of the mating in the population. This leads to
the next generation expressing mainly genes from the silverback and very few genes
from the other males, changing the genetic structure of the population.
D The founder effect occurs when the selective pressure on a trait varies depending on
the alleles expressed within the population, generating varying allele frequencies
based on the genetic makeup of the original population. An example of the founder
effect is the cyclical dominance of three throat-color patterns in side-blotched lizards.
Solution The solution is (A). The founder effect is an event that causes a change in the genetic
structure of the population. The Amish are a good example of this because the small
founding population of 200 people had a gene mutation that is now in higher
frequency than in other populations.
23 What is a cline? Give an example.
A A cline is a type of geographic variation that is seen in populations of a given species
that vary gradually across an ecological gradient. For example, endothermic animals
tend to have larger bodies in the cooler climates closer to Earth’s poles, allowing them
to better conserve heat.
B A cline is a change in ecological conditions over a geographic distance. For example, a
latitudinal cline is the decrease in temperature towards Earth’s poles, and an
altitudinal cline is the decrease in temperature with increase in altitude.
C A cline is the specific set of traits in a population of a given species that have been
influenced by the local environment. For example, a population of warm-blooded
animals that lived in a cooler climate closer to the North Pole would have larger
bodies, allowing them to better conserve heat.
D A cline is the specific set of ecological conditions in a geographic region. For example,
towards the North Pole it is cold and there is little precipitation. This will influence the
traits of the organisms that live there.
Solution The solution is (A). A cline is a type of geographical variation in which the
populations of a species change gradually over an ecological gradient. For example,
the bodies of endothermic animals tend to be larger in colder climates than in
warmer climates.

********************************************************
**************************440*************************************

24 The table below shows data for a small population of mice. The mice are either brown or
white. Based on the data, is the population experiencing genetic drift? Explain.
Generation Brown Mice Black Mice
1 14 32
2 20 26
3 24 22
4 21 28
5 19 30
6 24 29
Solution No. The frequency of both phenotypes oscillate around the same numbers through
six generations.
25 The large alpha male elephant seal is constantly fending off the advances of medium-sized
males. Small males are then able to sneak copulation with females and successfully pass
on their genes.
What is this an example of? Explain.
A This is an example of sexual selection. The females are selecting the small males over
the large male.
B This is an example of genetic drift. Because there are so many medium-sized males to
compete with the large alpha male, the small males are able to mate and cause the
gene pool to shift toward smaller individuals.
C This is an example of positive frequency-dependent selection, which is selection that
favors phenotypes that are either common or rare. The sneaky males (rare) are
favored in this case.
D This is an example of directional selection. Because only the smallest males are
mating, the next generation will have a higher proportion of alleles for small size,
making the seals smaller over time.
Solution The solution is (C). This is an example of positive frequency-dependent selection.
This is selection that favors phenotypes that are either common or rare. The sneaky
males (rare) will allow genetic variance to increase so the gene pool is not
dominated by medium sized and large males.

********************************************************
************************************************************439***

26 Why is there no perfect organism despite natural selection?


A Because natural selection works on a geographic level
B Because natural selection works in a random manner like mutations
C Because of limitations due to a population’s existing variation in genes
D Because natural selection is limited to sexual dimorphism
Solution The solution is (C). Natural selection acts on the existing variation within a
population. It does not create new genetic variation. Therefore, natural selection
selects the fittest organisms, but it is limited by what natural genetic variation there
is in the population.
27 A new predator invades the habitat of a population of field mice. Individuals with larger
body size are easier for the predator to capture then individuals with smaller body size.
Draw a histogram of body sizes with two plot lines, one showing the former population
and another showing the new population that indicates how this population will likely
evolve. On your histogram, also indicate what type of natural selection is occurring here.
Solution

28 Quinine is an antimalarial drug that is used to treat malaria in the Western Hemisphere.
Scientists have noticed that this drug has become less effective over time. Based on the
data below, what type of selection is being exerted on the malaria population?

********************************************************
**************************440*************************************

Solution Directional selection.

TEST PREP FOR AP® COURSES


29 A scientist is studying the genetics of a population of plants that she suspects is
undergoing natural selection. After examining samples of the population’s DNA over
several years, she reports the following data:

Do these data provide evidence of natural selection in this population? Why or why not?
A No, because the genotype frequencies, not allele frequencies, have to change for
evolution to occur
B No, because the allele frequencies are changing randomly, suggesting that genetic
drift is occurring, not natural selection
C Yes, because it shows that the previously favorable or neutral allele A is now being
selected against in favor of allele B
D Yes, because it is showing that the frequency of both alleles is changing over time
Solution The solution is (C). Natural selection influences the allele frequency in a population.
In this example, allele B is being selected for and allele A is being selected against.
30 A scientist is studying two large populations of deer that are centralized in nearby forests.
She takes blood samples from all of the deer in each population and records in how many
individuals she finds allele A. She then computes the frequency of the alleles in each
population. The frequencies observed over r years are shown in the tables.

********************************************************
************************************************************439***

Which forms of evolution are most likely occurring in populations A and B? Explain your
answer.
A In population A, genetic drift is likely occurring, causing allele A to become more
prevalent than allele B. In population B, mutation apparently occurred, introducing
allele A to population B. Allele A also appears to be increasing due to genetic drift in
population B.
B In population A, natural selection is likely occurring, with allele A being favored over
allele B. In population B, gene flow apparently occurred, allowing allele A to become
established in population B. Allele A also appears to be favored by selection in
population B.
C In population A, gene flow apparently occurred, allowing allele B to become
established in population A. Allele A also appears to be favored by selection in
population A. In population B genetic drift is likely occurring, causing allele A to
become more prevalent than allele B.
D In population A, mutation apparently occurred, introducing allele B to population A.
Allele A also appears to be increasing due to genetic drift in population A. In
population B natural selection is likely occurring, with allele A being favored over
allele B.

********************************************************
**************************440*************************************

Solution The solution is (A). In population B, allele A was not present in year 1. There may
have been a mutation in year 3 that introduced the allele to the population and
genetic drift increased the frequency of the allele over time. In population A, genetic
drift is likely causing the frequencies of allele to change.
31 A land manager mows a section of annual grass at the end of July. Over the years, he
recorded the date of flowering from the mown field as well as a similar grass field that
was not mown. What is the most likely explanation for this trend?
Mowed Field Unmowed Field
Year Flowering Date Flowering Date
2010 7/29 7/28
2011 7/30 7/26
2012 7/31 8/1
2013 7/8 7/29
2014 7/1 8/2
2015 6/29 7/26
A Mowing stimulates flowering, so the grass adapts by flowering after mowing occurs.
B Mowing stabilizes the flowering time, which follows a steady trend in the mowed field
but not in the unmowed field.
C The mowing is preventing the grass from reproducing, causing the mowed grass to
adapt by flowering earlier.
D The grass typically flowers earlier and earlier every year as it becomes older with each
passing year.
Solution The solution is (C). Mowing is selecting for earlier-flowering individuals. Over time,
earlier-flowering phenotypes will dominate the population. This is an example of
directional selection.
32 A scientist observed two populations of insects for 10 years. They took data on the length,
in millimeters, of the insects’ mouthparts. Their data are shown in the graphs.

********************************************************
************************************************************439***

How is this population evolving and what agent of evolution is most likely at work?
A Inbreeding, because the gene distributions are becoming less similar among the
population
B Genetic drift, as the distribution of traits has become more random
C Gene flow, as the population has likely gained new mouthpart traits through
immigration
D Natural selection, as insects that have mid-sized mouthparts are being favored
Solution The solution is (D). Natural selection is occurring as insects with mid-sized
mouthparts are being selected for and are therefore becoming more prevalent in
the population.
33 Researchers believe that in a fish species, individuals with the recessive genotype aa are
predisposed to disease. Homozygous dominant (AA) individuals and heterozygous (Aa)
individuals are not believed to be susceptible to this disease. A pond was stocked with
100 fish of the AA genotype and 100 fish of the aa phenotype, and the fish were allowed
to breed. In the next generation, 35 percent of the fish had the dominant (AA) phenotype.
What does this result indicate?
A The homozygous dominant phenotype is higher than expected, indicating that
evolution has occurred.
B The homozygous dominant phenotype is lower than expected, indicating that
evolution has occurred.
C The homozygous dominant phenotype is higher than expected, indicating that
evolution has not occurred.
D The homozygous dominant phenotype is lower than expected, indicating that
evolution not occurred.

********************************************************
**************************440*************************************

Solution The solution is (A). In the parent population, 50/100 fish have the AA genotype, and
50/100 fish have the aa phenotype. This means that the frequency of each allele in
the population is 0.5, so p = 0.5 and q = 0.5. If the population is in Hardy-Weinberg
equilibrium, the frequency of homozygous dominant individuals in the offspring is
expected to equal p2, or 0.25. In other words, 25 percent of the offspring are
expected to have the AA genotype. The actual frequency is higher (35 percent),
indicating that an evolutionary change has occurred.
34 Heterozygote advantage is a condition in which heterozygotes are favored by natural
selection. How would the value of 2pq likely change if the population was undergoing
heterozygote advantage?
A It would remain in equilibrium because the value of p and q would remain the same.
B It would remain in equilibrium because the value of 2pq would remain the same.
C It would not remain in equilibrium because the value of 2pq would likely increase.
D It would not remain in equilibrium because the value of 2pq would likely decrease.
Solution The solution is (C). The value of 2pq, which is the predicted frequency of
heterozygous offspring, would increase because it is at a selective advantage.
35 The graph shows the change in gene frequency of the two alleles of a gene: A and B. The
population being studied has no emigration or immigration. Which type of evolution is
likely occurring here and is the allele selected for, neutral, or selected against by natural
selection?

A Nonrandom mating; Both alleles are favored.


B Gene flow; Allele A is favored.
C Genetic drift; Both alleles are neutral.
D Natural selection; Allele A is not favored.
Solution The solution is (C). The allele frequencies fluctuate up and down. Genetic drift is the
random change in a population. In this population, the frequencies of alleles A and B
are changing randomly over time, not due to an advantage of either allele.

********************************************************
************************************************************439***

36 The figure shows the change in gene frequency of the two alleles: A and B. These alleles
are located on separate genes that do not influence each other in any way. The
population being studied has no emigration or immigration. Which type of evolution is
likely occurring here, if at all? Explain how you know.

Solution Sample answer: This graph shows that allele A is not evolving because its gene
frequency is steady and unchanging over time. Allele B is undergoing genetic drift
due to the random ups and down of the allele frequency.
37 The graph shows the current frequencies of two genotypes of the same gene: AA and aa.
What would most likely happen to the frequencies of A and a if heterozygous individuals
were favored by natural selection?

A Both AA and aa will drop in frequency by the same amount.


B Both AA and aa will drop, but aa will drop more.
C AA will increase in frequency and aa will drop in frequency.
D Genotype aa will increase in frequency and AA will drop in frequency.
Solution The solution is (A). The frequency of both AA and aa would decrease by the same
amount.

********************************************************
**************************440*************************************

38 The diagram shows the frequency of alleles on two species of wind-pollinated plants, as
well as the prevailing wind direction. These frequencies have been fairly stable for around
10 years. However, climate change has created a new prevailing wind direction, as shown
in the diagram. How will the populations likely evolve in the future?

A Natural selection will cause the frequency of B to increase in population 1.


B Gene flow will cause the frequencies of A and B to drop in population 3.
C Genetic drift will cause the frequencies of A and C to increase in populations 1 and 2.
D Inbreeding will reduce the frequency of allele B in populations 2 and 3.
Solution The solution is (B). The new wind direction will introduce genes from population 1
that have lower frequencies of alleles A and B. When individuals from population 1
mate with individuals from population 3, the frequencies of alleles A and B will
therefore decrease.
39 The diagram shows two populations of a species that have been long separated by a river,
which prevents interbreeding. The two populations differ in coloration, as shown in the
diagram. Recent human activity has caused the river to dry, however, resulting in the two
populations shown in the lower diagram. What is the most likely explanation for this
change?

********************************************************
************************************************************439***

A An increase in gene flow between the two populations


B A decrease in gene flow between the two populations
C An increase in nonrandom mating between the two populations
D A decrease in nonrandom mating between the two populations
Solution The solution is (A). Gene flow is increased because the river is no longer a solid
barrier between the populations. As a result, genes for blue individuals have been
introduced to population 2.
40 Antibiotics are medicines that are designed to kill disease-causing organisms, or
pathogens. However, some pathogens evolve antibiotic resistance, where they gain traits
that allow them to survive in the presence of antibiotics. The ability of bacteria to adapt to
antibiotics so quickly has created a huge concern over whether antibiotics are being
overused.
What form of evolution is antibiotic resistance an example of, and why?

********************************************************
**************************440*************************************

A Gene flow because the bacteria are passing on the resistant trait within their
populations
B Natural selection because the bacteria is adapting to a new environmental condition:
the presence of the antibiotic
C Genetic drift because medical workers cannot follow the randomly fluctuating gene
frequencies of bacterial populations
D Mutation because each bacterium must mutate to an antibody-resistant form in order
to survive
Solution The solution is (B). Natural selection is selecting for individuals that are resistant to
antibiotics. As a result, the presence of antibiotic resistance increases in the bacteria
population.

SCIENCE PRACTICE CHALLENGE QUESTIONS


19.1 Population Evolution
41 Consider a polymorphic gene with three alleles: A, B, and C.
A. If the frequencies of the alleles A and B are 0.2 and 0.3, the frequency of allele C is
closest to —
A 0.25
B 0.5
C 0.2
D 0.3
Consider a gene with only two alleles: dominant A and recessive a. In a population of
1,000 organisms, the fraction expressing the homozygous recessive phenotype is 0.37.
B. The calculated allele frequencies p and q have values that are closest to —
A 0.69 and 0.31
B 0.31 and 0.69
C 0.37 and 0.63
D 0.63 and 0.37
C. The calculated number of individuals in this population that are heterozygotes is closest
to —
A 240
B 230
C 430
D 476

********************************************************
************************************************************439***

Mountain pine beetles (Dendroctonus ponderosae) were collected from a 1-acre tract of
lodge pole pine trees (Pinus contorta) in a region of British Columbia where the forests are
under temperature stress. The beetles were crushed, and a cellulase enzyme was
extracted. Three polymorphs of the enzyme were observed when separated by gel
electrophoresis. The three proteins observed correspond to alleles labeled C1, C2, and C3.
The numbers of beetles with each allele are shown in the table.
Genotype C1 C1 C2 C2 C3 C3 C1 C2 C1 C3 C2 C3 Total
Observed 120 230 112 175 198 165 1,000
D. The calculated allelic frequencies pC1, pC2, and pC3 are closest to —
A pC1 = 0.57, pC2 = 0.57, pC3 = 0.59
B pC1 = 0.29, pC2 = 0.29, pC3 = 0.42
C pC1 = 0.61, pC2 = 0.80, pC3 = 0.59
D pC1 = 0.31, pC2 = 0.40, pC3 = 0.29
E. To investigate the presence of selection at the cellulase locus due to changing
temperature, what should a biologist do?
A The biologist should calculate the values of the sums pC1 + pC2 + pC3 and
(pC1 + pC2 + pC3)2. If these numbers are not equal to 1, the gene is not in Hardy-
Weinberg equilibrium, and the gene is evolving.
B The biologist should return next year and repeat this examination of the enzyme,
calculating frequencies of each allele each year. Then calculate the values of the
sums
pC1 + pC2 + pC3 and (pC1 + pC2 + pC3)2. If these numbers are not the same each year,
the gene is not in Hardy-Weinberg equilibrium, and the gene is evolving.
C The biologist should return each year for several years and repeat this
examination of the enzyme, calculating frequencies of each allele each year. If
the allele frequencies are changing, the gene is not in Hardy-Weinberg
equilibrium, and temperature is exerting a selection pressure.
D The biologist should return each year for several years and repeat this
examination of the enzyme, calculating frequencies of each allele each year. If
the allele frequencies are changing, the gene is not in Hardy-Weinberg
equilibrium. Analysis of the dependence of allele frequencies on temperature
could indicate selection.
Solution A. The solution is (B).

********************************************************
**************************440*************************************

B. The solution is (B). The fraction of the population with the homozygous recessive
genotype, 0.37, is equal to q2. Taking the square root, q = 0.61 Since p + q = 1,
p = 1 – 0.61 or 0.39.

C. The solution is (D). The number of heterozygous individuals is expected to equal


to 2pq. The value of p is 0.39 and the value of q is 0.61, so
D. The solution is (D). Two copies of the C1 allele are present in the 120 C1C1
individuals, for 240 alleles total. One copy of the C1 allele is present in the 175 C1C2
individuals, and 165 copies are present in C1C3 individuals. Thus, the total number of
C1 alleles is equal to 240 + 175 + 198, or 613. A total of 2,000 alleles are present in
the 1,000 beetles tested, so the frequency of the C1 allele equals 613 divided by
2,000, as shown in equation 1. The frequency of the C2 allele is calculated in
equation 2, and the frequency of C3 is calculated in equation 3.

E. The solution is (D). A change in allele frequency indicates that selection is


occurring, but selection could occur for many reasons. Scientists must assess
whether the change in allele frequency depends on temperature to determine if a
temperature change is driving the change in allele frequency.
19.2 Population Genetics
42 Calamus finmarchicus is the dominant copepod in the Gulf of Maine. The polymorphic
aminopeptidase locus, Lap-1, has been shown to be useful for the genetic differentiation
of populations of this organism. By examining the population dynamics of copepods, the
dynamics of the fin fish on which they feed can be predicted. The map shows a landmass
separating two coastal estuarine habits, the mud flats of Egypt Bay and the Mount Desert
Narrows. For the past 40 years, transport between the two habits has been hindered by
a dam over the Carrying Place Inlet. However, small volumes of water occasionally crest
the dam.

********************************************************
************************************************************439***

To evaluate the geographic isolation of invertebrate populations in these two habitats,


copepods are sampled at the points labeled 1 and 2 on the photograph. These points lie at
either ends of the Carrying Place Inlet. Enzymes encoded by three alleles, labeled A, B,
and C, were determined by gel electrophoresis of equal numbers of the organisms
collected at the two sites. Numbers of each genotype are given in the table.
Site AA AB AC BB BC CC Total
1 82 114 102 75 98 30 500
2 96 108 92 54 110 40 500
A. Calculate the frequencies, f, of each allele and complete the table.
Site f(A) f(B) f(C)
1 blank blank Blank

2 blank blank Blank

B. Using a test, evaluate these data to determine whether the aminopeptidase gene in
these two populations is evolving. State your conclusion as claims supported by evidence
at both the 95 percent and 99 percent confidence levels. The formula for the test is
provided on the AP Biology Exam.

********************************************************
**************************440*************************************

This table of critical p values is also provided on the AP Biology Exam.


P 1 2 3 4 5 6 7 8
0.05 3.84 5.99 7.82 9.49 11.07 12.59 14.07 15.51
0.01 6.64 9.32 11.34 13.28 15.09 16.81 18.48 20.09
Degrees of Freedom
C. Based on these data, predict, with justification, changes over time in the
aminopeptidase enzyme for these populations.
D. The B form of this aminopeptidase is slightly more efficient at extracting nutritional
leucine from a protein than the A and C forms but slightly less efficient at extracting valine
and serine. Describe an investigation of the two habitats that could suggest a causal
relationship between changes in allele frequency and characteristics of the environment.
E. Single-nucleotide mutations are neutral when they encode changes in proteins that
result in no significant differential selection. If differences in environmental factors
between sites 1 and 2 are not observed, predict what other factors could result in
departures from Hardy-Weinberg equilibrium for aminopeptidase.
Solution Sample answer:

A. and so on

B.
C. This result says that we must reject the statement that the population is changing
if our criterion is 99 percent confidence. However, at the 95 percent level we can
accept the claim.
D. Differences between the amino acid content of the copepod diet in the two
habits should be investigated. Copepods eat phytoplankton but that is not in-scope.
Only that variation is diet may be suspected as the mechanism in the possible
selective pressure is needed.
E. The frequencies of the alleles of a gene that is in Hardy-Weinberg equilibrium are
constant. The conditions of HW equilibrium are no migration, no mutation, random
mating, no selection and no genetic drift. If we suppose that the mutations are
negligible so that there is no selection, then we are left with genetic drift as the
possible explanation of a changing allele frequency.
43 Bioluminescence is an example of convergent evolution; 30 distinct lineages have
acquired this characteristic, and all involve some form of a class of molecules called
luciferins. Sexual selection pressures are strong for light-emitting organisms. Ellis and
Oakley (Curr. Biol., 2016) examined the number of species that lack luminosity in groups
of closest evolutionary relation (sister linear) with those species that are luminous.

********************************************************
************************************************************439***

Similarly, scientists made the same comparison between groups that use luminosity for
concealment (counter-illumination) and their sister lineages. The graphs summarize their
results, comparing the natural logarithm of the number of species in each lineage.

Based on the data shown in the graphs, describe a model that can account for the
increased speciation of bioluminescent lineages, including the mechanism of speciation.
Solution Sample answer: The data show speciation increases when luminosity is a component
of mating selection, but when luminosity is not a component of mating selection,
there is no effect on speciation. The evidence for the former claim is that there is a
positive slope for each lineage. The evidence for the latter claim is that the slopes
are both positive (speciation increases) and negative (speciation decreases). A likely
mechanism is that strong selection during mating leads to reproductive isolation and
this leads to speciation.

********************************************************
**************************440*************************************

44 A biologist is using a simulation to model populations of African hornbills (Bycanistes spp.


and Ceratogymna spp.), a keystone species of the savanna. Populations of the birds are
declining due to habitat loss. The hornbill’s diet consists primarily of termites and fruit. A
critical component of termite digestion is chitin deacetylase, an enzyme whose mutation
rate is a model parameter. The other model parameter is population size, N. In the results
of the simulation study shown in the graphs, there is no selection, and the mutation rate
is fixed. Although both population size and mutation rate are fixed, randomness results in
the five different outcomes shown in each graph.

A. Select the graph displaying the results that are closer to Hardy-Weinberg equilibrium.
Justify the selection of the graph.
B. Based on these simulations, predict the future heterozygosity, 2pq, of the smaller
populations, as shown in graph (a).
C. Justify the use of a simulation study with no selection under environmental conditions
in which the availability of both termites and fruit is high.
D. If a change in the environment occurs suddenly, such as an increase in average
temperature, where fruit production declines, analyze the effect of the change on allele
frequency in the large and small populations.
Solution Sample answer:
A. The graph at the right closely approximates a constant allele frequency and so is
closer to HW equilibrium.
B. The simulation with smaller population size shows a drift toward fixation of one or
the other allele. In either case, the heterozygosity tends to zero.
C. When both resources are in abundance, the mutation is neutral. This is equivalent
to the absence of selection.
D. With suddenly increasing temperature, a small population of hornbills may or
may not survive, depending on whether the functional chitin-deacetylase is fixed.

********************************************************
***************************************************************************459***

20 | EVOLUTIONARY RELATIONSHIPS OF
LIFE ON EARTH
REVIEW QUESTIONS
1 Who devised a commonly used classification system?
A Carl Linnaeus
B Darwin
C Plato
D Aristotle
Solution The solution is (A). Carl Linnaeus, a Swedish botanist, zoologist, and physician
created the taxonomic classification system.
2 What uses a hierarchical model to classify organisms?
A Analogy
B Taxonomic classification system
C Order
D Systematics
Solution The solution is (B). The taxonomic classification system uses a hierarchical model to
classify organisms.
3 Correctly list the hierarchy of taxonomy.
A Kingdom, Domain, Phylum, Order, Class, Family, Genus, species
B Domain, Kingdom, Class, Phylum, Order, Family, Genus, species
C Domain, Kingdom, Phylum, Class, Order, Family, Genus, species
D Domain, Kingdom, Class, Phylum, Order, Family, Genus, species
Solution The solution is (C). The correct taxonomy order is Domain, Kingdom, Phylum, Class,
Order, Family, Genus, species.
4 Which category, below the level of Kingdom, would have the next largest number of
organisms?
A Order
B Phylum
C Family
D Class

********************************************************
**************************458****************************************************

Solution The solution is (B). Each sublevel in the taxonomic classification system moves
toward specificity with other organisms, causing the diversity of organisms to
narrow. The Kingdom category has the widest diversity of organisms, and because
the Phylum category is a subcategory of the Kingdom, it would have the next widest
diversity of organisms.
5 How is systematics related to phylogeny?
A Systematics provides guidelines that scientists use to describe the relationships of
organisms.
B Scientists use systematics programs to put together the phylogeny of an organism.
C In systematics, scientists use combined data based on evolutionary relationships from
many sources to put together the phylogeny of an organism.
D Systematics is a process used to put together the phylogeny of an organism.
Solution The solution is (C). Systematics is a field of organizing and classifying organisms
based on evolutionary relationships. Combined data from many sources is used to
put together the phylogeny of an organism. Scientists use combined data based on
evolutionary relationships from many sources to put together the phylogeny of an
organism.
6 What is the best explanation of what systematists do?
A Scientists in the field of systematics organize organisms by characteristics.
B Scientists in the field of systematics provide information on how organisms are similar
or different.
C Scientists in the field of systematics contribute to building, updating, and maintaining
the tree of life.
D Scientists in the field of systematics collect data from fossils.
Solution The solution is (A). Systematics is a field of organizing and classifying organisms
based on evolutionary relationships, which scientists use to put together the
phylogeny of an organism.
7 What is the purpose of a phylogenetic tree?
A To organize and name organisms into specific categories
B To show the evolutionary pathways and connections among organisms
C To show geographic or behavioral factors
Solution The solution is (B). The purpose of a phylogenetic tree is to show the evolutionary
pathways and connections among organisms. Scientists use a tool called a
phylogenetic tree to show the evolutionary pathways and relationships among
organisms.

*******************************************************
***************************************************************************459***

8 What does the term rooted mean on a phylogenetic tree diagram?


A Relationships among species are not shown.
B All organisms represented in the diagram relate to a single ancestral lineage.
C A single lineage evolved into a distinct new one.
D A lineage evolved early from the root and remains unbranched.
Solution The solution is (B). On a phylogenetic tree diagram, the term rooted means all
organisms represented in the diagram relate to a single ancestral lineage. Scientists
call trees rooted when there is a single ancestral lineage (typically drawn from the
bottom or left) to which all organisms represented in the diagram relate.
9 Phylogeny is important to everyday life in human society. How did the research team in
China use phylogeny as a guide to discover new plants that can be used to benefit people?
A The research team used DNA to uncover phylogenetic relationships in the legume
family, and they found a compound in the plant that is effective in treating cancer.
B The research team used DNA to uncover phylogenetic relationships in the legume
family, and then they identified a newly discovered plant as Dalbergia sissoo.
C The research team used DNA to uncover phylogenetic relationships in the legume
family, and they found a DNA marker that can be used to screen for plants with
potential medicinal properties.
D The research team searched all the relatives of the newly discovered plant Dalbergia
sissoo to find antifungal properties.
Solution The solution is (C). The research team used DNA to uncover phylogenetic
relationships in the legume family, and they found a DNA marker that can be used to
screen for plants with potential medicinal properties. The research supports the idea
that DNA markers can be used to screen for plants with potential medicinal
properties.
10 Which animals in the figure belong to a clade that includes animals with hair? Which
evolved first, hair or the amniotic egg?

********************************************************
**************************458****************************************************

A Rabbits have hair, which evolved before the amniotic egg.


B Rabbits and lizards have hair, which evolved after the amniotic egg.
C Rabbits have hair, which evolved after the amniotic egg.
D Rabbits and lizards have hair, which evolved before the amniotic egg.
Solution The solution is (C). Rabbits have hair, as indicated on the phylogeny, and the
amniotic egg evolved before hair, because it evolved in the common ancestor of
lizards and rabbits. Hair is a derived trait that only evolved in rabbits in this
phylogeny.
11 What is the largest clade in the diagram of those listed?

A Animals, Fungi, and Plants


B Fungi
C Diplomonads
D Flagellates
Solution The solution is (A). All the organisms within a clade stem from a single point on the
tree. The clade of Animals, Fungi, and Plants contains multiple groups.
12 Why is it important for scientists to distinguish between homologous and analogous
characteristics before building phylogenetic trees?
A Phylogenetic trees are based on evolutionary connections, so scientists can use the
analogous characteristics to build the phylogenetic trees.
B Phylogenetic trees are based on evolutionary connections, so scientists can use the
homologous characteristics to build the phylogenetic trees.
C Phylogenetic trees are based on similar functions, so scientists can use the
homologous characteristics to build the phylogenetic trees.
D Phylogenetic trees are based on similar functions, so scientists can use the
homologous characteristics to build the phylogenetic trees.

*******************************************************
***************************************************************************459***

Solution The solution is (B). Scientists consider phylogenetic trees to be a hypothesis of the
evolutionary past since one cannot go back to confirm the proposed relationships.
As such, phylogenetic trees are often constructed based on evidence drawn from
multiple disciplines to illustrate evolutionary pathways and connections among
organisms.
13 Which option describes an analogous structure?
A A penguin and a seagull both have wings. The penguin uses wings to swim while the
seagull uses wings to fly.
B Lizards and whales have similar skeleton structures, but they have a different habitat
and lifestyle.
C Birds and butterflies have wings with similar characteristics for flight, even though
their wings do not share an evolutionary relationship.
D The bone structure in leg of a cat is very similar to the bone structure in the arm of a
human, but the functions of the limbs are very different.
Solution The solution is (A). A penguin and a dolphin each use fins to navigate through water.
Even though the penguin (a bird) is not related to a dolphin (a mammal), they both
use fins for the same function. This is an example of an analogous structure because
they both use the fins for the same function.
14 What is the ring of life?
A A phylogenetic model where all three domains of life evolved from a pool of primitive
prokaryotes
B An evolutionary history and relationship of an organism or group of organisms
C A phylogenetic model that attempts to incorporate the effects of horizontal gene
transfer on evolution
D A field of organizing and classifying organisms based on evolutionary relationships
Solution The solution is (A). This is the definition of the ring of life model. It suggests that all
three domains-Archaea, Bacteria and Eukarya-evolved from a single pool of gene-
swapping prokaryotes.
15 Some individual prokaryotes were responsible for transferring the bacteria that caused
mitochondrial development to the new eukaryotes, whereas other species transferred the
bacteria that gave rise to chloroplasts. This statement best describes which model?
A Ring of life
B Tree of life
C Branches of life
D Web of life

********************************************************
**************************458****************************************************

Solution The solution is (D). This statement best describes the web of life. The web of life
model tries to incorporate the effects of horizontal gent transfer on evolution as
described in the statement.
16 Explain why the classic tree model needs to be revised.
A The model is unable to incorporate DNA evidence.
B The model is erroneously based on many analogous traits, which have no basis in
evolutionary relationships.
C The model cannot be experimentally verified.
D The model does not consider the possibility that genes could be transferred between
unrelated species.
Solution The solution is (D). Classical thinking on prokaryotic evolution suggests that species
evolve through random mutations causing descent into the variety of species as it
branches from extinct species to modern-day species. Sexual reproduction in
eukaryotes makes this view complicated, but genetic variation is still ultimately
thought to be the result of mutation. What this way of thinking does not consider is
the possibility that genes could be transferred from unrelated species, causing
genetic variation in that manner.
17 What are three different ways that eukaryotic cells may have evolved?
A Some hypotheses propose that mitochondria were acquired first. Others propose that
the nucleus evolved first. Still others hypothesize that prokaryotes descended from
eukaryotes by the loss of genes and complexity.
B Some hypotheses propose that eukaryotic cells are a combination of bacterial and
archaeal cells. Others propose that eukaryotic cells are a combination of bacterial and
fungal cells. Still others hypothesize that eukaryotic and prokaryotic cells developed
independently.
C Some hypotheses propose that mitochondria developed from bacterial cells. Others
propose that they developed from archaeal cells. Still others hypothesize that bacteria
developed from mitochondria that had been released from eukaryotic cells.
D Some hypotheses propose that eukaryotic cells developed from gram-negative
bacteria. Others propose that they developed from gram-positive bacteria. Still others
hypothesize that both gram-positive and gram-negative bacteria contributed to the
eukaryotic genome through horizontal gene transfer.
Solution The solution is (C). Some hypotheses propose that mitochondria were acquired first,
followed by the development of the nucleus. Others propose that the nucleus
evolved first and that this new eukaryotic cell later acquired the mitochondria. Still
others hypothesize that prokaryotes descended from eukaryotes by the loss of
genes and complexity.
18 Explain the ring of life model.

*******************************************************
***************************************************************************459***

A The ring of life model is a phylogenetic model where the three domains of life started
as distinct groups that could swap genes horizontally with each other in all directions.
B The ring of life model is a phylogenetic model where all three domains of life are said
to have developed from a pool of primitive prokaryotes.
C The ring of life model is a phylogenetic model where bacterial and archaeal cells fused
to form eukaryotic cells.
D The ring of life model is a phylogenetic model where there is only a single domain of
life due to modern DNA analysis.
Solution The solution is (D). The ring of life model is a phylogenetic model where all three
domains of life said to have developed from a pool of primitive prokaryotes. This
model proposes that Archaea, Bacteria, and Eukarya evolved from this pool of gene-
swapping prokaryotes. This model takes into account horizontal gene transfer and
genomic fusion to explain genetic variation and the evolution of the three domains
of life.
19 In a transformation experiment, a sample of E. coli bacteria was mixed with a plasmid
containing the gene for resistance to the antibiotic ampicillin (ampr). Plasmid was not
added to the second sample. Samples were plated on nutrient agar plates, some of which
were supplemented with the antibiotic ampicillin. The results of E. coli growth are
summarized in the figure. The shaded area represents extensive growth of bacteria; dots
represent individual colonies of bacteria.

Which plates have only ampicillin-resistant bacteria?


A Plate I only
B Plate III only
C Plate IV only
D Plates I and II
Solution The solution is (C). Colonies formed on plate IV containing ampicillin. They are due to
resistant bacteria that could grow in the presence of ampicillin.

********************************************************
**************************458****************************************************

CRITICAL THINKING QUESTIONS


20 How are organisms classified in the taxonomic classification system?
A The taxonomic classification system uses a hierarchical model to organize living
organisms. At each sublevel, the organisms are more similar.
B The taxonomic classification system uses a hierarchical model to organize living
organisms. At each sublevel, the number of organisms increases.
C The categories in the taxonomic classification system are organized from smaller,
more specific categories to larger categories.
D In the hierarchal model for the taxonomic classification system, from the point of
origin, the groups become less specific.
Solution The solution is (A). Scientists historically organized Earth’s millions of species into a
hierarchical taxonomic classification system from the most inclusive category to the
most specific: Domain, Kingdom, Phylum, Class, Order, Family, Genus, and species.
21 What is the correct way to format a two-word scientific name?
A Italicize both words. Both words are lowercase.
B Italicize both words. The first word should be capitalized. The second word should be
lowercase.
C Italicize both words. Capitalize both words.
D Underline both words. Capitalize both words.
Solution The solution is (B). Scientists generally refer to an organism only by its genus and
species, which is its two-word scientific name, also referred to the as binomial
nomenclature. The first name of this binomial, the genus name should always be
written first, with only the first letter capitalized. The second name, the species, is
written last and all letters are lowercase. Both names should be typed in italics.
22 Some organisms that appear very closely related may NOT actually be closely related.
Why is this?
A There are cases where organisms used to be closely related but diverged from each
other and no longer look closely related.
B There are cases where organisms can interbreed making them look like a single
species, when in fact they are not closely related at all.
C There are cases where organisms evolved through convergence and appear closely
related but are not.
D There are cases when extremely distant taxa can recombine into a single group.
Solution The solution is (C). In most cases, organisms that appear closely related actually are;
however, there are cases where organisms evolved through convergence and
appear closely related but are not.

*******************************************************
***************************************************************************459***

23 How does a phylogenetic tree relate to the passing of time?


A A phylogenetic tree relates to the passing of time because species branch off from
each other at regular time intervals.
B A phylogenetic tree is not related to the passing of time because speciation is based
on geographic changes.
C The phylogenetic tree only shows the order in which things took place.
D A phylogenetic tree relates to the passing of time when the diagram also shows how
long ago the divergence from the common ancestor occurred.
Solution The solution is (D). A phylogenetic tree relates to the passing of time when the
diagram also shows how long ago the divergence from the common ancestor
occurred. Phylogenetic trees approximate the passing of time by the lengths of their
branches. Longer branches mean that more time has passed since the organisms
shared a common ancestor.
24 Judeo-Christian religious texts explain that the Earth and all the organisms on Earth were
created in seven days. Why is this not a scientific hypothesis?
Solution Sample answer: This is not a good hypothesis because it cannot be tested. There is
no way to gather scientific evidence that deities exist or that they created humans in
the distant past.
25 Scientists use the cladistics system to organize what?
A Homologous traits
B Homoplasies
C Analogous traits
D Monophyletic groups
Solution The solution is (A). After the homologous and analogous traits are sorted, scientists
often organize the homologous traits using a system called cladistics. This system
sorts organisms into clades: groups of organisms that descended from a single
ancestor.
26 How does a clade relate to a monophyletic group?
A Clades vary in size depending on the number of branches.
B All the organisms within a clade stem from a single point on the phylogenetic tree.
C A clade shows branches that do not share a single point.
D A clade shows groups that diverge at a different branch point.
Solution The solution is (B). Clades must include all of the descendants from a branch point.
As such, all of the organisms in the clade or monophyletic group stem from a single
point on the phylogenetic tree.
27 Scientists apply the concept of maximum parsimony to do what?

********************************************************
**************************458****************************************************

A Describe phylogenies accurately.


B Eliminate analogous traits.
C Identify mutations to DNA codes.
D Locate homoplasies.
Solution The solution is (A). Scientists apply the concept of maximum parsimony to describe
phylogenies accurately. The maximum parsimony concept is used to aid in the
tremendous task of describing phylogenies accurately.
28 You discover a new species of animal in the rainforest. What characteristics could you use
to distinguish this organism from the other organisms in the same clade based on Figure
11?
Solution Organisms on a cladogram are distinguished by morphological or molecular features.
One could analyze the sequence of DNA in the organism’s genome to understand
how it relates to other organisms in its clade.
29 Based on the phylogenetic tree below, is the Jungle cat likely closer related to a tiger or a
cougar? Why?

Solution The Jungle cat is separated from the Cougar by two common ancestors on the tree
and from the Tiger by five common ancestors on the tree. Therefore, the Jungle cat
is likely more closely related to the Cougar then the Tiger.
30 Two cultures of bacteria are separated by a filter that blocks the movement of cells but
allows free exchange of anything smaller than a bacterial cell. On one side of the filter, a
sample of penicillin-resistant cells in culture broth is added, and on the second side of the
tube, a culture of penicillin-sensitive cells in culture is added. After 24 h, resistant cells
appear on the side with the cells sensitive to penicillin. Which three genetic mechanisms
can account for appearance of the penicillin-resistant cells?
A Transformation, transduction, and conjugation

*******************************************************
***************************************************************************459***

B Transformation, transduction, and mutation


C Transformation, conjugation, and mutation
D Transduction, conjugation, and mutation
Solution The solution is (A). Transformation and transduction are mechanism that explain the
appearance of resistant cells because the filter is permeable to viruses and free DNA.
Conjugation is not possible because cells cannot cross the filter. The third
mechanism is spontaneous mutation.

TEST PREP FOR AP® COURSES


31 What evolutionary question is better addressed by the fig-shaped evolutionary tree (a) as
opposed to the more typical, single-trunk phylogenetic tree (b)?

(a)

(b)
A What was the single organism from which all other forms of life on Earth arose?

********************************************************
**************************458****************************************************

B Did animals evolve from fungi?


C In which species of eukaryote did chloroplasts first appear?
D Were chloroplasts and mitochondria transferred to eukaryotic cells through horizontal
gene transfer?
Solution The solution is (D). The addition of multiple trunks and horizontal interconnections
in the fig-shaped tree of life best answers the question “Were chloroplasts and
mitochondria transferred to eukaryotic cells through horizontal gene transfer?” This
is because the fig-shaped tree shows interconnections and multiple starting points
that are missing from a single-trunk phylogenetic tree.
32 Which question, relating to the endosymbiotic hypothesis and the evolution of
eukaryotes, is NOT answered by the eukaryote-first hypothesis, based on the figures?

A Which evolved first, the nucleus or prokaryotes?


B Which evolved first, mitochondria or prokaryotes?
C How and when did the nucleus evolve in eukaryotes?
D How and when did prokaryotes evolve?
Solution The solution is (C). The eukaryote-first hypothesis does not answer the question,
“How and when did the nucleus evolve in eukaryotes?”
33 The phylogeny shows the evolution of traits in vertebrates.

*******************************************************
***************************************************************************459***

Based on this phylogeny, a student asks, “Does this mean that lizards, frogs and rabbits all
possessed hair and an egg with amnion at some point in their evolution?” Based on the
phylogeny, how should you respond to the student?
A Hair and an amniotic egg were both possessed by all three species at some point in
their evolution.
B Hair is only a characteristic found in the rabbit evolutionary history. The amniotic egg
was possessed by both the rabbit and lizard, but not frogs, at some point in their
evolutionary history.
C Hair is a characteristic only found in the rabbit evolutionary history. The amniotic egg
was possessed by all three species at some point in their evolutionary history.
D Hair was possessed by all three species at some point in their evolutionary history. The
amniotic egg was possessed by both the lizard and frog, but not the rabbit at some
point in their evolutionary history.
Solution The solution is (B). The best response would be to show the student that “Hair” is a
derived trait only possessed by the rabbit on this tree. The amniotic egg is possessed
by both the rabbit and lizard, but not frogs.

********************************************************
**************************458****************************************************

34 The tree shows the phylogenetic relationships between four species.

A scientist wishes to perform a genetic analysis on all four species in which she
determines the number of genetic similarities among all four species. What would she
likely find regarding the genetic similarities among species A, B, D, and E?
A Species D and E would share more genetic similarities with each other than with
species A and B, and vice versa.
B Species A and E would share more genetic similarities with each other than with
species B and D, and vice versa.
C Species D and A would share more genetic similarities with each other than with
species A and B, and vice versa.
D Species D and B would share more genetic similarities with each other than with
species A and E.
Solution The solution is (D). Species D and B have fewer nodes between them than A and E
and thus are more related.
35 What is the aim of scientists applying the maximum parsimony concept when creating
phylogenetic trees?
A The scientists spend more time creating the phylogenetic table.
B Scientists find the shortest tree with the smallest number of changes.
C A complex, detailed phylogenetic tree diagram is created.
D The scientists spend more time researching the data for evolutionary connections.
Solution The solution is (B). Scientists look for the most obvious and simple order of
evolutionary events, and doing so, they find the shortest tree with the smallest
number of changes.

*******************************************************
***************************************************************************459***

36 Dolphins and fish have similar body shapes. Is this feature more likely a homologous or
analogous trait? Explain your answer.
A Analogous: Dolphins are mammals and fish are not, thus their evolutionary paths are
quite separate. They have similar body shapes because of their similar environment.
B Analogous: Dolphins and fish are both vertebrates, thus they share an evolutionary
history, causing them to have similar body shapes.
C Homologous: Dolphins and fish are both vertebrates, thus they share an evolutionary
history, causing them to have similar body shapes.
D Homologous: Dolphins are mammals and fish are not, thus their evolutionary paths
are quite separate. They have similar body shapes because of their similar
environment.
Solution The solution is (A). Dolphins are mammals and fish are not, which means that their
evolutionary paths (phylogenies) are quite separate. Dolphins probably adapted to
have a similar body plan after returning to an aquatic lifestyle, and, therefore, this
trait is probably analogous.
37 What effect has the advancement of DNA technology had on determining phylogeny?
A Morphologic and molecular information often disagree.
B Scientists are struggling with molecular systematics.
C Information is not reliable because organisms appear to be closely related when they
are not.
D Computer programs help determine relatedness using DNA sequencing, and
morphologic and molecular information is more effective in determining phylogeny.
Solution The solution is (D). Computer programs help determine relatedness using DNA
sequencing, and molecular data. Molecular information is more effective in
determining phylogeny. New computer programs confirm earlier classified
organisms and also uncover previously made errors.
38 What is maximum parsimony used for in evolutionary biology?
A Maximum parsimony hypothesizes that organisms that share the most traits are the
most likely to share a common ancestor.
B Maximum parsimony hypothesizes that organisms that share a common ancestor are
more likely to have many traits in common.
C Maximum parsimony hypothesizes that events occurred in the simplest, most obvious
way, and the pathway of evolution probably includes the fewest major events that
coincide with the evidence at hand.
D Maximum parsimony hypothesizes that organisms that display homologous structures
are closely related, while organisms that display analogous structures must have
diverged much farther in the past.

********************************************************
**************************458****************************************************

Solution The solution is (C). Maximum parsimony hypothesizes that events occurred in the
simplest, most obvious way. To aid in defining phylogenies accurately, scientists
decipher the pathway of evolution using the fewest major events that coincide with
the evidence at hand.
39 The emu in Australia and ostrich in Africa are flightless birds that look similar. One
proposed hypothesis was the birds descend from an early common ancestor that
spread when the continents were connected. DNA analysis shows that emus and
ostriches share more genetic homology with flying birds, which live in the same region
than with each other.
What is the best explanation for these findings?
A This is an example of an early shared ancestor
B This is an example of convergent evolution.
C This is an example of random DNA homology
D This is an example of divergent evolution.
Solution The solution is (B). This is an example of convergent evolution because the emu and
ostrich, despite their physical similarities, likely do not share a recent common
ancestry because of their isolation on different continents.
40 A scientist decides to investigate the evolutionary connection between closely related
bacteria. Which gene would be a good choice to use for establishing relatedness, a very
well conserved gene or a poorly conserved sequence? Explain your reasoning.

a. A very well conserved gene would be a good choice, because well conserved genes undergo
sufficient changes during relatively short times, which allows for the study of recent
evolutionary events. Well-conserved genes do not undergo changes during short durations.
b. A poorly conserved gene would be a good choice, because poorly conserved genes show
sequence similarity, which is used as evidence of evolutionary relationships between
sequences.
c. A poorly conserved gene would be a good choice, because poorly conserved genes undergo
sufficient changes during relatively short times, which allows for the study of recent
evolutionary events.
d. A very well conserved gene would be a good choice, because well conserved genes show
sequence similarity, which is used as evidence of evolutionary relationships between
sequences.
Solution Sample answer: When recent evolutionary events are being studied, regions or
genes that evolve much more rapidly are needed because they have undergone
sufficient changes during this relatively short time.

*******************************************************
***************************************************************************459***

41 In a hypothetical population of beetles, there is a wide variety of color, matching the


range of coloration of the tree trunks on which the beetles hide from predators. The
graphs illustrate four possible changes in the beetle population as a result of a change in
the environment due to pollution that darkened the tree trunks.

What would be the most likely change in the coloration of the beetle population after
pollution and why?
A The coloration range shifted toward more light-colored beetles, as in diagram I. The
pollution helped the predators find the darkened tree trunks.
B The coloration in the population split into two extremes, as in diagram II. Both
the light-colored and the dark- colored beetles were able to hide on the darker
tree trunks.
C The coloration range became narrower, as in diagram III. The predators selected
beetles at the color extremes.
D The coloration in the population shifted toward more dark-colored beetles, as in
diagram IV. The light-colored beetles were found more easily by the predators than
were the dark-colored beetles.
Solution The solution is (D). Selection pressure will favor the dark-colored beetles which are
well camouflaged and less likely to be seen and eaten by predators.
42 A population of rodents settles on the shore of an island close to the Arctic Circle. The
landscape consists mainly of rocks. If the individuals are too large, they cannot hide in
crevices to escape hawks. On the other hand, small bodies do not maintain internal
temperature in cold weather. Show diagrammatically the change in the population and
explain what selective pressures took place.
Solution

********************************************************
**************************458****************************************************

*******************************************************
***************************************************************************459***

43 Five new species of bacteria were discovered in Antarctic ice core samples. The
nucleotides (base sequences of rRNA subunits) were determined for the new species. The
table shows the number of nucleotide differences between the species.
Species 1 2 3 4 5
1 – 3 19 18 27
2 Blank
– 19 18 26
3 Blank blank
– 1 27
4 Blank blank

blank
27
Which phylogenetic tree is most consistent with the data?

A C

D
B
Solution The solution is (C). Species 5 has the most nucleotide changes compared to the
other four species. Species 1 and 2 have three nucleotide differences between them,
whereas there are 18 and 19 between 1 or 2 and the second group, species 3 and 4.
There is only 1 nucleotide difference between 3 and 4.
44 Draw the phylogenetic tree for the species in the table. Identify where on the tree each
feature evolved.
Amniotic
Species Egg Endotherm Feathers Lungs Vertebrae Notochord
Snake Y N N Y Y Y
Ostrich Y Y Y Y Y Y
Shark N N N N Y Y
Frog N N N Y Y Y
Lancelet N N N N N Y

********************************************************
**************************458****************************************************

A The ostrich branched off first, followed by the snake, then the frog, then the shark,
and then the lancelet.
B The shark branched off first, followed by the lancelet, then the frog, then the ostrich,
and then the snake.
C The lancelet branched off first, followed by the shark, then the frog, then the snake,
and then the ostrich.
D The lancelet branched off first, followed by the shark, then the ostrich, then the snake,
and then the frog.
Solution The solution is (C). The organism with the fewest most common features branched
off early and is the least related to the other organisms. The ostrich possesses a
notochord (embryonic stage), vertebrae, lungs, amniotic eggs, feathers, and
constant body temperature (warm-blooded is not used any longer to describe
endothermic.)

45 Barbara McClintock discovered transposons while working on maize genetics. What are
the transposons composed of when they are able to shift from one location to another?
A Segments of RNA
B Plasmids
C Segments of DNA
D Proteins
Solution The solution is (C). Transposons are segments of DNA that can shift their location
and can enter a cell with the assistance of a plasmid.
46 What is horizontal gene transfer (HGT)?
A The proposal that eukaryotes developed a nucleus first, and then their mitochondrion
B The transmission of genetic material from one species to another through
mechanisms other than from parent to offspring
C The fusion of two prokaryotic genomes
D The division of kingdom in the taxonomic classification

*******************************************************
***************************************************************************459***

Solution The solution is (B). HGT is the introduction of genetic material from one species to
another species by mechanisms other than the vertical transmission from parent(s)
to offspring. These transfers allow even distantly related species to share genes,
influencing their phenotypes.
47 What is referred to as the transfer of genes by a mechanism that does not involve asexual
reproduction?
A Web of life
B Meiosis
C Gene fusion
D Horizontal gene transfer
Solution The solution is (D). Horizontal gene transfer is the introduction of genetic material by
mechanisms other than parent to offspring transfer as in sexual and asexual
reproduction.
48 Which options describes small, virus-like particles that act as a mechanism of gene
transfer between prokaryotes?
A Gene transfer agents
B Horizontal gene transfer
C Vertical gene transfer
D Basal taxon
Solution The solution is (A). Gene transfer agents are bacteriophage-like particle that
transfers random genomic segments from one species of prokaryote to another.

SCIENCE PRACTICE CHALLENGE QUESTIONS


20.1 Organizing Life on Earth
49 The figure shows a plot of the fraction, as a percentage, of families of marine organisms
living at a particular point in time that became extinct (vanished from the fossil record) in
the next geological moment. These mass extinctions mark the ends of geologic periods.
For example, the Triassic period ended around 213 million years ago (Mya).

********************************************************
**************************458****************************************************

A. The fact that evolution is an ongoing process is illustrated by these data. Whether the
process displays a pattern involving regular intervals is a question that has been raised. Of
those who believe periodicity is present, a period of 26 million years is favored. A wave
with this periodicity is drawn on the figure. Evaluate the evidence provided in terms of
agreement and disagreement with the marine extinction record.
The Cretaceous and Jurassic were periods of warm landmasses covered by a shallow sea.
The ends of these periods are known to be due to asteroids that left a sedimentary trace.
At the end of the Triassic, there is no evidence of an asteroid impact. Instead, there was
massive volcanism associated with the opening of the Atlantic Ocean, a four-fold increase
in carbon dioxide, and a 3–6 °C temperature rise (A. Marzoli et al., Science, 1999).
Macrofossil, spore, and pollen data show that marine animal species declined much more
than marine plant species (L. Mander et al., Proc. Natl. Acad. Sci., 2010). The cause of the
end of the Permian period is less uncertain, but an 8°C temperature rise has been
established (McElwain and Punyasena, Trends in Ecology and Evolution, 2007). Both
terrestrial and marine taxa were affected.
B. The graph estimates the number of distinct families, including both marine and
terrestrial, as a function of time before the present. Note that the time scale for this graph
is much longer than that of the other graph.

*******************************************************
***************************************************************************459***

Analyze this graph by


 Identifying times of mass extinctions
 Posing a question regarding any difference between the graph of extinctions of
marine life and the graph of family level diversity
 Explaining the slope of the graph of family diversity following a mass extinction
event
Solution Sample answer:
A. End-Permian, end-Triassic, end-Jurassic, and end-Cretaceous all fit the period.
However, during the Jurassic and Tertiary the fit is poor. If there is periodicity, no
other wavelength is better than the one suggested. Aside: A possible explanation
involving the effect of dark matter on the frequency of asteroid collisions has been
presented (Rampino, MNRAS, 2014).
B. The Permian, Triassic, and Cretaceous boundaries show drops for family diversity,
and these lines up with marine extinctions. After these extinction events, there is a
rapid increase in speciation as shown by the steep slopes following these events. At
the boundary marking the end of the Jurassic and beginning of the Cretaceous, there
is no drop in family diversity, though there is a mass extinction of marine life. Is the
increase in speciation in the warm, shallow seas of these periods compensating for
the extinction of marine life? Did the increase in temperature and/or carbon dioxide
have a more profound effect on marine species because of drops of oxygen

********************************************************
**************************458****************************************************

concentration or acidification of oceans than they had on terrestrial species,


particularly plants?
20.2 Determining Evolutionary Relationships
50 Lactate dehydrogenase, an enzyme involved in glycolysis, from several species is
compared using a Southern blot technique in the figure: (A) yeast; (B) snail; (C) mouse; (D)
rat; and (E) human (after K. Webster, Journal of Experimental Biology 2003).

A. Justify the claim that these data provide evidence that supports glycolysis as a
conserved core property.
There are three forms of lactate dehydrogenase whose roles vary within an organism:
LDH-A, -B, and -C. The question arises as to the origins of each. The cladogram (after
S. Tsuji et al., Proc. Natl. Acad. Sci., 1994) displays a proposed relatedness of variations in
LDHs over many species.

*******************************************************
***************************************************************************459***

B. Based on these data, describe the differences between the likely evolutionary
sequences of LDH-A, -B, and -C in the African frog and in humans.
C. Evaluate the claim that in both species (African frog and human), LDH-C is the most
recently evolved form of the enzyme.
Solution Sample answer:
A. Each of the species shows the same two DNA sequences. Since this enzyme is a
component of glycolysis, we see that this is a core conserved process.
B. The African frog LDH-A and LDH-B represent a divergence and the modification of
the nucleotide sequence in LDH-B led to LDH-C. In the human LDH-B and LDH-A have
been derived from LDH-C.
C. LDH-C probably came last in the frog and first in the human. The evidence is the
larger number of nucleotide substitutions.

********************************************************
**************************458****************************************************

51 Nucleotide-repeat sequences often occur within the intron, and sequence variation is
neutral; that is, there is no selection. For example, the nucleotide-repeat sequence
labeled A might be ACGGGC, and the repeat sequence labeled B might be ACTGGT.
Repeat sequences evolve by single-step duplication, deletion, and inversion, rather than
by single nucleotide substitution. Because these repeat sequences can be used to infer
phylogeny, a phylogenetic tree can be hypothesized based on the principle of parsimony
—the simplest explanation is the best explanation. Consider the repeat sequences A, B, C,
D, and E shown in the list in which only inversions have occurred among five different
species.
 Species 1: ABCDE
 Species 2: ADCBE
 Species 3: BACED
 Species 4: DACBE
 Species 5: ABCED
 Species 6: DEBCA
A. Pose three questions that can be used to infer the evolution of these five species.
B. Draw lines between nearest relatives to construct a cladogram that displays the
relationships inferred by answers to your questions.

Carson (Drosophila Genetics 1983) used inversions in intron-repeat sequences of the fruit
fly to infer evolution among the Hawaiian Drosophila. He further assigned the
chronological sequence of islands on which the flies appeared by assuming
(parsimoniously) that the geologically oldest of the volcanic islands was home to the
oldest fly ancestor. When a fly or flies arrived on a newer island, speciation occurred,
which, after a time, stabilized until another island hop occurred.
C. Evaluate Carson’s reasoning for speciation and ongoing evolution.
D. Pose two questions whose pursuit could provide additional evidence of Carson’s
hypothesized evolutionary sequence.

*******************************************************
***************************************************************************459***

Solution Sample answer:


A. Which species are most closely related?
Which species are connected by a single inversion?
Which sequences cannot be connected by a single inversion?
How can I get from species 1 to 2, 1 to 3, 1 to 4, 1 to 5, 2 to 3, 2 to 4, 2 to 5, etc.?
B.

C. The concept is that introduction to a new island introduces new selection acting
on mutations. Over time, the speciation that results undergoes a convergence to a
stable population. This species then migrates to a new island and the process is
repeated.
D. Do other proteins or gene sequences display the same phylogeny? Where did the
population on the oldest island originate? How can we be sure that only a single
inversion occurred between island hops? The answer to the first question is yes, and
Carson’s simple reasoning is regarded as very eloquent in this time of more brute-
force, next-generation sequencing to infer phylogeny. The answer to the second
question was South America. The answer to the third is that we cannot; the frequent
use of the word “hypothesis” is more common amongst evolutionary biologists than
among other sub-disciplines, say, biochemists. Other evidence is always desirable
until, as Stephen Jay Gould said, “It is pernicious to withhold consent.”

********************************************************
**************************498****************

21 | VIRUSES
REVIEW QUESTIONS
1 Viruses were first discovered after the development of the porcelain filter, called the
Chamberland-Pasteur filter. How did the porcelain filter enable scientists to discover
viruses?
A The porcelain filter removed diseases from a liquid sample.
B The porcelain filter removed virions from a liquid sample.
C The porcelain filter removed bacteria from a liquid sample.
D The porcelain filter removed a disease from tobacco plants.
Solution The solution is (C). The porcelain filter could remove all bacteria visible in the
microscope from any liquid sample, while allowing the viruses to pass through
the filter.
2 In the late 1930s, scientists got their first good view of viruses. How did this happen?
A The development of the light microscope helped scientists discover many viruses of all
types of living organisms.
B The development of the viral receptor helped scientists discover many viruses of all
types of living organisms.
C The development of the porcelain filter helped scientists discover many viruses of all
types of living organisms.
D The development of the electron microscope helped scientists discover many viruses
of all types of living organisms.
Solution The solution is (D). When the electron microscope was developed in the late 1930s,
it provided greater magnification and helped scientists see the structure of many
viruses of all types of living organisms.
3 Determining the origins of viruses is challenging. Which hypothesis proposes to explain
the origin of viruses by suggesting that viruses evolved from free-living cells?
A Escapist or the progressive
B System of self-replication
C Devolution or the regressive
D Virus molecular systematics
Solution The solution is (C). When the electron microscope was developed in the late
1930s, the development of the electron microscope helped scientists discover
many viruses of all types of living organisms based on viral adaptations to surviving
inside of host cells.

********************************************************
***************************************499***

4 Which statement best describes what biologists know about the evolution of viruses?
A Scientists can look at fossil records and similar historic evidence.
B Much about virus origins and evolution remains unknown.
C Biologists have accumulated a significant amount of knowledge about how viruses
originated.
D Biologists know exactly when viruses emerged and from where they came.
Solution The solution is (B). Much about virus origins and evolution remains unknown.
Although biologists have accumulated a significant amount of knowledge about how
present-day viruses evolve, much less is known about how viruses originated in the
first place.
5 What is an individual virus particle outside a host cell that consists of a nucleic acid core,
an outer protein coating, and sometimes an outer envelope?
A A capsid
B A virion
C A capsomere
D A viral receptor
Solution The solution is (B). A virion is a viral particle in its extracellular (outside of the cell)
state. It consists of the nucleocapsid (combination of viral nucleic acid and the
protenacious-capsid coat).
6 For many viruses to penetrate the cell membrane and complete their replication inside
the cell, the virus must attach to their host cells. How does a virus attach to a host cell?
A A virus uses its cellular structure to attach to a host cell.
B A virus uses a plasma membrane to connect to a host cell.
C A virus uses matrix proteins to attach to a host cell.
D Viruses use viral receptors to attach to a host cell.
Solution The solution is (B). Viruses exhibit high specificity for select host cells. Nonenveloped
or enveloped viruses may enter cells in different ways. Proteins present in the viral
capsid for nonenveloped viruses or on the envelope for enveloped viruses, binds to
the receptor for which it is specific, in much the same manner as a “key” fits in a
specific “lock.”
7 _____ means that the genomic RNA can function as mRNA.
blank

A Double-stranded
B Negative polarity
C Positive polarity
D Replica intermediates

********************************************************
**************************498****************

Solution The solution is (C). Positive polarity is an ssRNA virus with a genome that contains
the same base sequences and codons found in their mRNA.
8 Viruses often are classified based on the type of genetic material and its structure. In the
Baltimore classification scheme, which virus has a single-stranded RNA (–) genome?
A Human immunodeficiency virus (HIV)
B Rabies (rhabdovirus)
C Canine parvovirus (parvovirus)
D Common cold (pircornavirus)
Solution The solution is (B). The rabies (rhabdovirus) virus is in Group V, single-stranded
RNA (–).
9 What must scientist use to get a visual look at the internal structure of virions?
A A scanning electron microscope
B A transmission electron microscope
C A porcelain filter
D A light microscope
Solution The solution is (B). The internal structures of a virus can be observed in images from
a transmission electron microscope. A beam of electrons is transmitted and an
image is formed from the interaction of the electrons transmitted through the
specimen.
10 Which statement about the viral replication cycle is accurate?
A The viral replication cycle does not affect the structure of the host cell.
B The viral replication cycle cannot affect genetic material of the host cell.
C The viral replication cycle has seven basic steps.
D The viral replication cycle can change cell functions or even destroy the host cell.
Solution The solution is (D). The viral replication cycle can produce dramatic biochemical and
structural changes in the host cell, which may cause cell damage.
11 What happens in the lysogenic cycle of viral replication?
A During the budding process, virions leave the host cell individually.
B During the budding process, the host cell bursts.
C During the budding process, the virus connects with a permissive host cell.
D During the budding process, the host cell dies immediately.
Solution The solution is (A). Budding is a method of exit from the cell where virions leave the
host cell individually.

********************************************************
***************************************499***

12 In which cycle does the virus replicate and burst out of the host cell?
A Lytic
B Lysogenic
C Cytopathic
D Latency
Solution The solution is (A). The lytic cycle is a type of virus replication in which virions are
released through lysis, or bursting, of the cell.
13 How is the lytic cycle different from the lysogenic cycle?
A The phage infects a cell in the lytic cycle.
B The lytic cycle contains the formation of a prophage.
C In the lytic cycle, new phages are produced; immediately in the lysogenic cycle phage
DNA is merged into the host genome.
D The phages move on to infect other cells in the lysogenic phase.
Solution The solution is (C). In the lytic cycle, new phages are produced in recurring cycles
and can lead to host death; in the lysogenic cycle phage DNA is merged into the host
genome and remains dormant.
14 Which statement is false?
A Enveloped viruses and naked viruses both may enter cells using the fusion method.
B Many enveloped viruses enter the cell by receptor-mediated endocytosis.
C Naked viruses enter the cell by receptor-mediated endocytosis.
D Undergoing shape changes and creating channels in the host cell membrane is an
alternative method of cell penetration used by naked viruses.
Solution The solution is (A). Naked viruses cannot enter cells using the fusion method. Fusion
only occurs with enveloped virions.
15 An apple tree has yellow splotches on the leaves. What is this is a symptom of?
A Cell necrosis
B Discoloration
C Hyperplasia
D Hypoplasia
Solution The solution is (D). Hypoplasia, or decreased cell growth, causes thin, yellow areas to
appear in the leaves of plants.

********************************************************
**************************498****************

16 What happens during the release step in the lytic or lysogenic cycle of replication?
A During the release step, genetic information is transferred through the lytic and
lysogenic cycles.
B During the release step, DNA is transcribed to messenger RNA.
C During the release step, the nucleic acid is released from the viral capsid or envelope.
D During the release step, the new virions are able to infect adjacent cells and repeat
the replication cycle.
Solution The solution is (D). In the release step, the new virions leave the host cell and are
able to infect adjacent cells and repeat the replication cycle.
17 Why does the HIV virus use reverse transcriptase in the replication process?
A The HIV virus uses reverse transcriptase to replicate cells and build proteins.
B The HIV virus uses reverse transcriptase to erase mutated virions.
C The HIV virus uses reverse transcriptase because it is a retrovirus.
D The HIV virus uses reverse transcriptase because it has a DNA genome.
Solution The solution is (C). The HIV virus uses reverse transcriptase because it is a retrovirus.
Reverse transcriptase converts the viral RNA into double-stranded DNA.
18 What are the symptoms of the herpes simplex virus?
A The herpes simplex virus causes eye infections.
B The herpes simplex virus causes pneumonia.
C The herpes simplex virus causes pancreatitis.
D The herpes simplex virus can cause septicemia.
Solution The solution is (A). The herpes simplex virus, adenovirus, and cytomegalovirus all
cause eye infections.
19 Which statement accurately describes the measles virus?
A The measles virus causes nasal and lung infections.
B The measles virus causes pancreas and liver infections.
C The measles virus causes mouth and gum infections.
D The measles virus causes brain and skin infections.
Solution The solution is (D). The measles virus infects the respiratory system and can cause
brain and skin disorders.

********************************************************
***************************************499***

20 Which statement best describes vaccines?


A Vaccines kill viruses.
B Vaccines stimulate an immune response against future infections.
C Vaccines inhibit the virus by blocking the action of key viral proteins.
D Vaccines control and reduce symptoms.
Solution The solution is (B). Vaccines stimulate an immune response against future infections.
Viral vaccines are designed to give protective immunity against future infections.
21 Which kind of therapy attacks a stage of the virus replicative cycle?
A Phage therapy
B Antiretroviral therapy
C Gene therapy
D Cancer therapy
Solution The solution is (B). Antiretroviral therapy attacks a virus at different stages of its
replicative cycle.
22 Which virus causes parotitis?
A Measles virus
B Norovirus
C HIV
D Mumps virus
Solution The solution is (D). The mumps virus causes parotitis, which is inflammation of a
parotid gland.
23 Which statement about prions is true?
A Prions are larger than viruses.
B Prions contain DNA and RNA.
C The PrPC is the normal form of the protein.
D The PrPSC is folded abnormally.
Solution The solution is (D). The PrPSC, a proteinaceous infectious particle, is the abnormally
folded from of PrPC.

********************************************************
**************************498****************

24 Kuru is a prion disease that affects both humans and animals. How is kuru spread?
A Kuru disease is spread between cattle.
B Kuru is passed from person to person.
C Kuru is passed from cows with BSE to humans.
D Kuru is a viroid that infects plants.
Solution The solution is (B). Kuru is spread by the consumption of meat, nervous tissue, or
internal organs between members of the same species. Kuru, native to humans in
Papua New Guinea, was spread from human to human via ritualistic cannibalism.
25 Which statement about viroids is true?
A Viroids are single-stranded RNA particles.
B Viroids reproduce only outside of the cell.
C Viroids produce proteins.
D Viroids affect both plants and animals.
Solution The solution is (A). Viroids are small, single-stranded RNA particles. They are known
to infect plants such as potatoes, cucumbers, tomatoes, chrysanthemums, avocados,
and coconut palms. This results in crop failures and the loss of millions of dollars in
agricultural revenue each year.
26 On which industry can viroids have a severe impact?
A Dairy
B Poultry
C Avocado
D Livestock
Solution The solution is (C). Viroids can have a severe impact on the avocado industry
because viroids are known to infect plants, including avocados.
27 Which statement best explains how infected prions cause disease?
A Infected prions cause disease by transmitting nucleic acids to normal prion proteins.
B Infected prions cause disease by converting DNA to RNA in normal prion proteins.
C Infected prions cause disease by converting the shapes of normal proteins.
D Infected prions cause disease by replicating the normal form of the protein.
Solution The solution is (C). Infected prions cause disease by converting the shapes of normal
proteins. Prions convert the normal protein shape to an abnormal form.

********************************************************
***************************************499***

CRITICAL THINKING QUESTIONS


28 How did the development of a porcelain filter, called the Chamberland-Pasteur filter, help
scientists discover viruses?
A After filtering a liquid plant extract, the scientists could see the virions using the light
microscope.
B After filtering a liquid plant extract, the disease was still transferred to a healthy plant.
C After filtering a liquid plant extract, the virus cells multiplied.
D After filtering a liquid plant extract, scientists were able to trace historical footprints.
Solution The solution is (B). After filtering a liquid plant extract, the disease was still
transferred to a healthy plant. Years later, it was proven that the infectious agents
were a new type of very small, disease-causing particle.
29 Scientists have a few hypotheses about virus origins. Why might they develop and refine
further hypotheses to explain the origin of viruses?
A Advances in technology provide historic evidence.
B Biochemical and genetic information provide historic evidence.
C Advances in technology provide new information for scientists.
D Advances in technology have proven that viruses have a single common ancestor.
Solution The solution is (C). New technology provides new avenues for investigation and
stimulates new ideas.
30 Why don’t dogs catch the measles?
A Measles is a DNA virus, and DNA viruses cause human diseases.
B Dogs do not have glycoproteins.
C The virus can’t attach to dog cells.
D Dogs do not get RNA viruses.
Solution The solution is (C). The virus cannot attach to dog cells, because dog cells do not
have the receptors for the virus and/or there is no cell within the dog that is
permissive for viral replication.
31 The Baltimore classification system groups viruses according to how the mRNA is
produced. When classified this way, the viruses in each group —
A behave in a similar manner
B look very similar
C connect with living things
D are based on the type of disease they cause

********************************************************
**************************498****************

Solution The solution is (A). Each of the seven groups has a different replication strategy.
32 Researchers have been able to develop a variety of anti-HIV drugs, such as the drug AZT.
How does the drug AZT work?
A AZT blocks the enzyme called HIV protease, which the virus uses to reproduce itself.
B AZT blocks the HIV integrase enzyme, which the virus uses to insert its viral DNA into
the DNA of the host cell.
C AZT prevents reverse transcriptase and HIV protease enzyme from functioning inside
the body.
D AZT prevents reverse transcriptase from making DNA from the viral RNA genome.
Solution The solution is (D). AZT is a nucleic acid synthesis inhibitor that functions as an
analog of thymidine. It works by preventing the viral enzyme reverse transcriptase
from making DNA from the viral RNA genome. Without this step, HIV cannot
replicate.
33 Compare the lytic and lysogenic cycles. Which cycle has the potential to produce the most
virions?
A The lytic cycle can theoretically produce more virions as the viral genome takes over
the host cell, resulting in the large-scale release of virions.
B The lysogenic cycle can theoretically produce more virions as the reproductive cycle of
viruses undergoing lysogeny is much faster than the reproductive cycle of viruses
following lytic cycle.
C The lysogenic cycle can theoretically produce more virions as the viral genome is
incorporated into the host cell’s genome replicating along with the host cell.
D The lytic cycle can theoretically produce more virions as the prophage following
lysogenic cycle ultimately gets excised from the host cell’s genome and enter the
lytic cycle.
Solution The solution is (C). The lysogenic cycle can theoretically produce more virions than
the lytic cycle, because in the lysogenic cycle the viral genome is incorporated into
the genome of the host cell and the virus continues to live. The virus’s nucleic acid is
replicated as the host cell multiplies. The virus can live and replicate inside a host for
a long time.
34 Would a person who has never been in contact with the varicella-zoster virus be at risk of
developing chickenpox or shingles if they come in close contact with a person with
shingles? Explain your reasoning.
A The person is at risk of developing chickenpox. Chickenpox is the first infection with
the virus before it enters latency in the host.
B The person is at risk of developing shingles. Shingles is the first infection with the virus
before it enters latency in the host.

********************************************************
***************************************499***

C The person is at risk of developing chickenpox. Chickenpox is the first infection with
the virus that is already latent in the body.
D The person is at risk of developing shingles. The virus enters the person and gets
activated when a person with shingles comes in close contact.
Solution The solution is (A). Because this is the first time the person comes in contact with
the varicella-zoster virus, they risk developing chickenpox. Chickenpox is the first
infection with the virus before it enters latency in the host. Shingles is a secondary
infection from virus that is already latent in the body.
35 Which step in the replication cycle of viruses do you think is most critical for the virus to
infect cells? Explain why.
A The attachment step is the most critical, as infection cannot begin if the virus does not
attach to the host cell.
B The replication step is the most critical, as this step directs protein synthesis.
C The assembly step is the most critical, because new virions are assembled to
infect cells.
D The entry step is the most critical, as nucleic acid of the virus needs to enter the host
cell naked, leaving the capsid outside.
Solution The solution is (A). The attachment step is the most critical step in the replication
cycle, because if the virus does not attach to the host cell, the infection process
cannot begin.
36 For most people, the measles virus does not cause a serious illness. Symptoms include
fever and a rash, but the symptoms are usually gone in about a week. However, for some,
the measles virus can be much more serious.
How can the measles virus cause a potentially fatal illness?
A Measles can cause meningococcal disease, which causes severe headaches, seizures,
and in severe cases, can be life-threatening.
B Measles can cause variant Creutzfeldt-Jakob disease, which causes severe headaches,
seizures, and in severe cases, can be life-threatening.
C Measles can cause encephalitis/meningitis, which causes severe headaches, seizures,
and in severe cases, can be life-threatening.
D Measles can cause Legionnaires’ disease, which causes severe headaches, seizures,
and in severe cases, can be life-threatening.
Solution The solution is (C). Measles is a contagious respiratory system virus that can cause
encephalitis/meningitis, which is inflammation of the brain. Severe cases of
encephalitis can be life-threatening.

********************************************************
**************************498****************

37 Why is immunization after being bitten by a rabid animal so effective and why aren’t
people vaccinated for rabies like dogs and cats are?
A It takes a month for the virus to travel from the site of the bite to the central nervous
system. People are not vaccinated beforehand as routine vaccination of domestic
animals makes it unlikely that humans will contract rabies from an animal bite.
B It takes a week for the virus to travel from the site of the bite to the peripheral
nervous system. People are not vaccinated beforehand as routine vaccination
of domestic animals makes it unlikely that humans will contract rabies from an
animal bite.
C It takes a week for the virus to travel from the site of the bite to the central nervous
system. People are not vaccinated beforehand as routine vaccination of domestic
animals makes it unlikely that humans will contract rabies from an animal bite.
D It takes a week for the virus to travel from the site of the bite to the central nervous
system. People are not vaccinated beforehand, as routine vaccination of domestic
animals makes it fully sure that humans will contract rabies from an animal bite.
Solution The solution is (C). The rabies vaccine works after a bite because it takes week for
the virus to travel from the site of the bite to the central nervous system, where the
most severe symptoms of the disease occur. Adults are not routinely vaccinated for
rabies for two reasons: first, because the routine vaccination of domestic animals
makes it unlikely that humans will contract rabies from an animal bite; second, if one
is bitten by a wild animal or a domestic animal that one cannot confirm has been
immunized, there is still time to give the vaccine and avoid the often fatal
consequences of the disease.
38 Why don’t dogs and cats catch human colds from humans?
A As cats and dogs have different proteins than humans, the virus that causes colds in
humans cannot find receptors in dogs and cats.
B As cats and dogs have different receptors than humans, the virus that causes colds in
humans cannot find receptors in dogs and cats.
C Cats’ and dogs’ immune systems attack the virus unlike humans’ immune systems, so
the virus that causes colds in humans cannot find receptors in dogs and cats.
D As natural killer cells of cats and dogs attack the virus, the virus that causes colds in
humans cannot find receptors in dogs and cats.
Solution The solution is (B). To survive and reproduce, the viruses need a host of living cells.
Cats and dogs have different DNA than humans, so the virus that causes colds in
humans cannot find receptors in dogs and cats.

********************************************************
***************************************499***

39 Prions are responsible for variant Creutzfeldt-Jakob Disease (vCJD). How has this disease
been documented to spread from human to human?
A Surgery with instruments previously used in a patient with vCJD that were not
adequately sterilized and contaminated pineal growth hormones taken from human
pineal glands from infected cadavers.
B Through human consumption of infected meat and contaminated pituitary growth
hormones taken from human pituitary glands from infected cadavers.
C Surgery with instruments previously used in a patient with vCJD that were not
adequately sterilized and contaminated pituitary growth hormones taken from human
pituitary glands from unwell individuals.
D Surgery with instruments previously used in a patient with vCJD that were not
adequately sterilized and contaminated pituitary growth hormones taken from human
pituitary glands from infected cadavers.
Solution The solution is (B). This prion-based disease is most often transmitted through
human consumption of infected meat. However, humans can contract the disease
following surgery with instruments previously used in a patient with vCJD that were
not adequately sterilized. Also, transmission has been linked to contaminated
pituitary growth hormones taken from human pituitary glands from infected
cadavers.
40 What characteristics do viroids and viruses have in common?
A They both replicate within a host cell and contain nucleic acids.
B They both replicate within a host cell and do not contain nucleic acids.
C They both replicate within a host cell and contain proteins.
D They both replicate within a host cell and contain only RNA.
Solution The solution is (A). These noncellular infectious agents both replicate within a host
cell, and they both contain nucleic acid.
41 Why is the transmission of a prion NOT reliant upon genes made of DNA or RNA?
A DNA or RNA, though present, is not transmitted when a prion causes infection.
B The prion does not contain DNA or RNA.
C Only parts of DNA or RNA are transmitted in a prion.
D More of protein and less of DNA or RNA is transmitted.
Solution The solution is (B). Prions are infectious acellular particles that consist of abnormally
folded proteins that replicate without DNA or RNA.

********************************************************
**************************498****************

TEST PREP FOR AP® COURSES


42 The table shows the Baltimore Classification used to classify viruses based on their genetic
material.

What is the difference between how Group I and Group III viruses reproduce?
A In Group I, RNA is transcribed from an RNA genome while in Group III, RNA is
transcribed from a DNA genome.
B In Group I, RNA is transcribed from a DNA genome while in Group III, RNA is
transcribed from an RNA genome.
C In Group I, DNA is transcribed from a DNA genome while in Group III, RNA is
transcribed from an RNA genome.
D In Group I, DNA is transcribed from an RNA genome while in Group III, RNA is
transcribed from a DNA genome.
Solution The solution is (B). The Group I contains double-stranded DNA which requires the
RNA being transcribed from a DNA genome while in Group III, which has double-
stranded RNA, the RNA is transcribed from an RNA genome.

********************************************************
***************************************499***

43 The table shows the Baltimore Classification used to classify viruses based on their genetic
material.

What is similar or different between the genome of Group I and Group VI, as well as how
the two virus types reproduce?
A Group I and VI viruses use RNA as their genome. Group I viruses reproduce by
transcribing RNA from their DNA genome, while Group VI viruses first synthesize their
RNA genome using reverse transcriptase before they can reproduce.
B Group I and VI viruses use DNA as their genome. Group I viruses reproduce by
transcribing RNA from their DNA genome while group VI viruses first synthesize their
DNA genome using reverse transcriptase before they can reproduce.
C Group I and VI viruses use DNA as their genome. Group I viruses reproduce by
transcribing RNA from their DNA genome, while group VI viruses first synthesize RNA
genome using reverse transcriptase before they can reproduce.
D Group I viruses use DNA as their genome while group VI use RNA. Group I viruses
reproduce by transcribing RNA from their DNA genome while group VI viruses
synthesize DNA from RNA using reverse transcriptase before they can reproduce.
Solution The solution is (B). Group I and Group VI viruses both use DNA as their genome,
however, Group I viruses reproduce by transcribing RNA from their DNA genome,
while Group VI viruses have to first synthesize their DNA genome using reverse
transcriptase before they can reproduce.

********************************************************
**************************498****************

44 The diagram shows the stages during which a virus infects a host cell.

During which of the numbered steps does the amount of viral genetic material begin to
change within the host cell?
A Step 1. Virus enters the cell.
B Step 2. Virus RNA enters the nucleus.
C Step 3. New viruses assemble within the cell.
D Step 4. Viruses leave the cell.
Solution The solution is (B). As viral RNA enters the host’s nucleus in step 2, the virus hijacks
the host cell, causing the host to make more viral RNA.

********************************************************
***************************************499***

45 The diagram shows the stages during which a virus infects a host cell.

How could the influenza virus change the function of a host cell?
A Because it replicates its DNA within the cell and reproduces, which could interfere
with cell processes
B Because it replicates RNA within the cell and reproduces, which could interfere with
cell processes
C Because it attacks the immune system of the host cell, which in turn would interfere
with cell processes
D Because it replicates its protein within the cell and reproduces, which could interfere
with cell processes.
Solution The solution is (B). The influenza virus could change the function of the host cell
because it replicates its RNA within the cell and reproduces. This could interfere with
cell processes as more cell resources are used to make viruses instead of used to
keep the cell alive.

********************************************************
**************************498****************

46 The diagram models the lytic and lysogenic reproductive cycles of viruses.

Which cycle would maintain the DNA of the virus over several generations, and why?
A Lysogenic, because the viral DNA can be excised from the host cell’s DNA when
under stress
B Lytic, because the viral DNA can be excised from the host cell’s DNA when under stress
C Lytic, because the viral DNA can be passed on when the host cell replicates
D Lysogenic, because the viral DNA can be passed on when the host cell replicates
Solution The solution is (D). Lysogenic, because the viral DNA can be passed on when the host
cell replicates.

********************************************************
***************************************499***

47 The diagram models the lytic and lysogenic reproductive cycles of viruses.

Based on the diagram, is the following statement is true or false? Explain.


The lysogenic cycle allows viruses to preserve their genome during unfavorable
conditions.
A True, because when the host cell experiences unfavorable conditions, it stops dividing
and stays in the same state
B True, because the host cell in both the replication stage and during unfavorable
conditions stays in the lysogenic cycle as it is more preferable over the lytic cycle.
C False, because when the host cell experiences unfavorable conditions, the prophage
exits the genome and enters the lytic cycle
D False, because when the host cell experiences unfavorable conditions, the virus enters
latency period
Solution The solution is (C). The lysogenic cycle does allow viruses to preserve their genome
during host cell replication. However, when the host cell experiences unfavorable
conditions, the prophage exits the genome and enters the lytic cycle.

********************************************************
**************************498****************

SCIENCE PRACTICE CHALLENGE QUESTIONS


21.1 Viral Evolution, Morphology, and Classification
48 Influenza A virus is the most pathogenic of the human influenza viruses. Its envelope
encloses a protein complex (vRNP) and eight, single-stranded, negative RNA (the
complement of a positive RNA strand that can be transcribed by a ribosome) segments
(vRNA). Each segment encodes one or two proteins that support viral replication. On the
outer surface of the envelope are proteins that recognize and bind to host receptors.
A. Annotate the representation to briefly describe each process associated with a
numbered label.

B. Describe influenza A viral replication as a process regulated by either positive or


negative feedback, and justify your selection.
C. The human acquired immunodeficiency syndrome (AIDS) and many cancers are cause
by double-stranded RNA retroviruses.
Contrast the processes of viral replication of HIV and influenza A virus.
D. Explain the difference in the effects of infection by HIV and influenza A virus on host
genetic variability.
E. Measured mutation rates for influenza A virus and HIV are nearly identical (Sanjuan et
al., Jour. Virology, 2010). Explain this observation even though host error-checking
operates in one of these replication modes.
Solution Sample answer:
A.
1. Attachment/endocytosis
2. Envelope dissolution
3. Transport over nuclear membrane
4. Synthesis of RNA complement
5. Transport to cytoplasm
6. Budding to release progeny

********************************************************
***************************************499***

B. A virion infects the cell and the cell amplifies, creating multiple copies. The
mechanism of this one-to-many positive feedback is that the code for the ribosome-
protein complex vRNP increases the transcriptive capacity of the cell.
The human acquired immunodeficiency syndrome and many cancers are caused by
double-stranded RNA retroviruses.
C. The details of HIV replication are defined as “in-scope” by the Framework, and so
the student should be able to describe the process of retrovirus replication. In the
HIV there is a double-stranded RNA that is used to produce a strand of DNA through
action of reverse transcriptase that accompanies the viral contents. The DNA strand
is then incorporated into the host DNA, and the host cell DNA polymerase is used to
synthesize new copies of the virus RNA. As described in A, there is no comparable
hijacking of cell replication apparatus in the case of influenza A. Consequently, the
flu is an acute disease and HIV is a chronic disease.
D. As described above the host DNA is altered by the retrovirus, while the host DNA
is unaltered by the single strand RNA virus.
E. The host error-checking apparatus (spliceosome) is used in replication of
retrovirus, and there is no error checking in the single strand RNA virus. However,
there is no error checking on the reverse transcriptase step, so the net mutation
rates are comparable.
49 A. Three-dimensional (3-D) structures, or folding, of proteins have been shown to contain
more information about evolutionary relationships than the sequences of DNA
nucleotides that encode the proteins. Amino acid sequences of rabbit skeletal muscle
actin (375 amino acids) and bovine ATPase (386 amino acids) have only 39 locations in
common. However, the 3-D structure of these proteins is nearly identical (Flaherty et al.,
Proc. Natl. Acad. Sci. USA, 1991). As information about the 3-D folding of proteins and the
number of sequenced whole genomes has increased, folding has been shown to be an
evolutionarily conserved property.
Analyze these data to refine the following model: The evolutionary history of life on Earth
can be inferred from variations over time of the nucleotide sequence of a gene.

********************************************************
**************************498****************

B. By applying a classification scheme based on protein folding, Nasir and Caetano-Anollés


(Sci. Adv., 2015) have determined the number of folding families that viruses share with
the three domains. Approximately 60 percent of the folding patterns found in viruses
were common to all three domains, as shown in the figure. Fewer than 10 percent were
unique to viruses.

Viruses are acellular, and, consequently, they lie outside of the three domains of cellular
life. However, their exclusion is increasingly challenged. Since 2012, several very large
viruses have been discovered, each a double-stranded DNA virus with more than one
million bases, with some encoding nucleotides and amino acids. However, none encode
ribosomes, so these viruses are still dependent on a marine bacteriovore (amoeba or
flagellate) host for replication.
Hypotheses regarding the origin of life on Earth need to account for the relationship
between proteins and genetic information. Proteins are required to read and write
genetic information, but genetic information is required to synthesize proteins. Which of
these systems evolved first, and if neither came first, how could they evolve
simultaneously? The RNA-first model is based on the idea that ribosomal RNA both
encodes and synthesizes proteins.
Describe a hypothesis for the origin of life on Earth that combines the dual functionality of
RNA and the function of retroviral reverse transcriptase to propose a mechanism leading
to an ancient, acellular lineage of very large, double-stranded DNA viruses and a first DNA-
based cellular life form.
C. Like viruses, the nucleus of a eukaryote uses the machinery of the cell to transcribe
DNA and synthesize proteins. Evaluate the possibility of the origin of Eukarya by
specialization of a very large double-stranded DNA virus.

********************************************************
***************************************499***

Solution All are areas of rapid development and conjecture. They can be traced from the
citations provided in the problem. Sample answer:
A. The evolutionary history of life on Earth can be inferred from the variation over
time of the molecular phenotype of a gene that includes the 3-D shape of the
protein expressed.
B. An RNA world is consistent with a precursor to the retrovirus that acquired an
alternative (and more stable) information storage strategy through the construction
of DNA using reverse transcriptase. The formation of complementary strands of DNA
led to double stranded viruses. The accumulation of large genomes may indicate an
ancient origin. The nature of the host (single celled marine eukaryotes) suggests a
selective advantage when the information is enclosed within a lipid membrane. A
test of the hypothesis would be the self-assembly of double-stranded DNA from a
system composed of retrovirus and the formation of lipid vesicles containing these
strands.
C. Engulfment of a large double stranded virus by a prokaryote could lead to a
specialized information storage and retrieval function for this virus within the cell—
which continues to use the prokaryote systems for protein synthesis.
21.2 Virus Infections and Hosts
50 Viruses evolve but leave no fossil evidence that can be used to construct phylogenies.
However, viral DNA, especially that of retroviruses, is commonly found in the host
genome. By comparing sequences from the same virus integrated at different points in
time, the evolutionary history of the virus can be constructed. The viral genomes are
typically found incomplete, in segments, and interrupted by stop codons. In jawed
vertebrates, retroviral sequences or sequences that have been derived from them are a
significant fraction of the whole genome.
A. Explain why retroviral DNA, rather than the genomes of single-stranded or double-
stranded DNA or single-stranded RNA viruses, is found in host DNA.
B. Exaptation occurs when gene expression provides a function that is independent of the
selection pressures that have acted on the gene. For example, a pigment that provided
selective advantage by reducing damage from solar radiation becomes an element of
mating behavior. Feathers that evolved under selection to prevent heat loss become a
means of flight.
In a study of viral evolution within host genomes of primates, Katzuorakis and Gifford
(PLOS Genetics, 2010) found that viral genomes within the host were surprisingly stable;
with computer simulation, they estimated the probability of such constancy at 1 in
100,000.
Explain in terms of selection how viral genetic information that no longer replicates the
virus is maintained by the host.

********************************************************
**************************498****************

C. Distemper is an incurable disease of cats, dogs, and their sister lineages caused by a
parvovirus. The virus exploits the host’s transferrin, a membrane-bound protein used for
iron transport, to attach to the cell. The phylogeny of the Parvoviridae family has been
constructed (J. Kaebler, PLOS Pathogens, 2012). That study revealed the evolution of both
the virus and the host protein through selection to resist infection. About 54 million years
ago when the lineage of cats (Feliformia) diverged from that of dogs (Caniformia), the
parvovirus envelope diverged as well, conforming to changes in the host’s transferrin. In
1978, a worldwide disease in dogs due to a parvovirus suddenly appeared.
Explain how this pandemic could have originated in the cat population.
Solution Sample answer:
A. Single- and double-strand DNA viruses and single-strand RNA viruses do not
integrate their DNA into the host DNA, while retroviruses do and then use the host’s
DNA transcription system for replication. This leaves the viral DNA in the host
genome. If the cell becomes a zygote, the viral DNA is inherited.
B. Since the viral segments are retained, there must be a negative selection pressure
—their loss would disadvantage the organism relative to some selection. Estimates
of the amount of retroviral DNA in the human genome are between 1 and 8 percent.
This is an area of active research where not much is known yet.
C. Mutations of the code for the envelope have been selected by the changing
protein configuration of the receptor site—an arms race between host and virus.
Small differences in transferring between the two divergent lineages arose. The
highly pathogenic form of the virus that created the pandemic could have been a
mutation of the code for the envelope in the Feliformia lineage. That is the
conclusion drawn in the Kaebler paper.
51 A. A simple calculation of the rate of spread of a pox virus (virion) led researchers at
Imperial College London to a new insight. Virions communicate with other virions. The
researchers observed that the radius of an approximately circular plaque of infected cells
grew to 1.45 mm in just 3 days. They measured the distance between adjacent cells to be
0.037 mm to obtain the apparent time for the lytic cycle (from infection to lysis). They
compared this time to the actual rate at which new virions are formed: 5 to 6 h.
Predict the radius of infection if the infection process involved a sequence of entry,
replication, lysis, and infection of an adjacent cell.
B. To account for this discrepancy between observed and predicted growth rates, the
researchers examined the viral entry process and discovered that the actin protein on the
host cell’s surface that provided the viral receptor was modified by attachment. They then
found a mutant virus that did not modify the cell surface protein. The dependence of the
growth of plaque radius on time for the wild type and mutant are shown in the graph.

********************************************************
***************************************499***

Analyze these data and compare the infection rates calculated with those predicted in (A).
C. Use the results of this experiment to support the claim that responses to information
and communication of information affect natural selection.
Solution Sample answer:
A. A virus jumps a distance of 0.037 mm every 5 to 6 h. The distance traveled in 72 h
then is between 0.4 and 0.5 mm in 3 days. The researchers observed a distance
between three and four times greater than this.
B. The rate for the wild type is approximately 0.35 mm/12 h or 0.029 mm/h. The
rate for the mutant is 0.0063 mm/h. The rate per hour calculated in (A) is between
0.006 and 0.007 mm/h, which is comparable to the rate observed for the mutant.
C. These data show that the virus is not infecting adjacent cells if that cell has
already been infected. The signaling is accomplished by leaving a modified actin, to
which a virion subsequently responds. This is communication that has evolved by
the positive selection of increased replication rates.
52 Describe how viral replication introduces genetic variation in the viral population.
Solution Sample answer: This declarative learning objective can be clarified by looking at the
specific aspects of viral replication that create replication errors that are identified
as assessable on the AP Biology Exam:
 Virus replication allows for mutations to occur through usual host pathways.
 RNA viruses lack replication error-checking mechanisms, and thus have
higher rates of mutation.
 Related viruses can combine/recombine information if they infect the same
host cell.

********************************************************
**************************536*****************************************

22 | PROKARYOTES: BACTERIA AND


ARCHAEA
REVIEW QUESTIONS
1 Which is the best evidence that prokaryotes evolved about 3 billion years ago?
A Scientists believe photosynthesis evolved about 3.0 billion years ago.
B There is fossil evidence of mammalian forms going back about 4.0 billion years.
C Earth and its moon are thought to be about 4.5 billion years old.
D There is fossil evidence of microbial mats—large multilayered sheets of prokaryotes—
starting about 3.5 billion years ago.
Solution The solution is (D). There is fossil evidence of microbial mats starting about 3.5
billion years ago. Microbial mats are thought to represent the earliest forms of life
on Earth.
2 Which statement describing the environment of early Earth is false?
A The atmosphere contained much less molecular oxygen.
B Strong volcanic activity was common.
C It was subject to mutagenic radiation from the sun.
D There was little to no geologic activity.
Solution The solution is (D). There was little to no geologic activity. Due to fossil evidence
indicating the presence of microbial mats approximately 3.5 million years ago,
scientists believe that hot springs and hydrothermal vents may have been the
environments in which life began, thereby suggesting that life on early Earth did
have geologic activity.
3 Which type of extremophile grows optimally at temperatures of −15 to 10 °C or lower?
A Alkaliphiles
B Thermophiles
C Hyperthermophiles
D Psychrophiles
Solution The solution is (D). Psychrophiles are microbes that survive in extremely cold
environments. Therefore, the optimal conditions needed for growth include
temperatures of −15–10 °C (5–50 °F) or lower.

********************************************************
*****************************************************************535***

4 What is an example of a relatively moderate environmental condition to which some


prokaryotes are adapted and can survive as spores?
A Extremely low temperature
B Hypersalinity
C High doses of radiation
D Normal drought
Solution The solution is (D). Some soil bacteria are able to form endospores that are heat-
resistant and drought-resistant, which thereby allows for them to survive until
favorable conditions improve or reoccur.
5 More than how much bacteria and archaea cannot be successfully cultured in a laboratory
setting?
A 9%
B 19%
C 91%
D 99%
Solution The solution is (D). Over 99 percent of bacteria and archaea are unculturable in a
laboratory setting due to a lack of knowledge about the special nutritional
requirements for growth that are needed by these organisms. Additionally, some
bacteria cannot be cultured because they are obligate intracellular parasites and
cannot be grown outside a host.
6 The most substantial difficulty in culturing prokaryotes in laboratory settings is related
to —
A the lack of knowledge about their needs for growth
B growth requirements that are too difficult to meet
C inefficient methods for resuscitation of viable-but-nonculturable (VBNC) organisms
D the expense of techniques such as polymerase chain reaction (PCR)
Solution The solution is (A). The most substantial difficulty in culturing prokaryotes in
laboratory settings is related to the lack of knowledge about their needs for growth.
7 What represents the earliest forms of life on Earth?
A Hydrothermal vent
B Microbial mat
C Meteorite
D Stromatolite

********************************************************
**************************536*****************************************

Solution The solution is (B). A microbial mat, or large multi-layered sheet of prokaryotes, may
represent the earliest forms of life on Earth. Fossil evidence of their presence on
Earth dates back approximately 3.5 billion years ago.
8 Which statement best summarizes the conditions of early Earth at the time that life first
evolved?
A The atmosphere of early Earth was very different from today’s atmosphere, but
most other conditions (such as geologic upheaval and volcanic activity) were very
much the same.
B The atmosphere of early Earth was very much like today’s atmosphere, but many
other conditions (such as geologic upheaval and volcanic activity) were very different.
C Early Earth had a very different atmosphere, was subject to extreme radiation, and
had a lot of geologic upheaval and volcanic activity.
D Early Earth had a very different atmosphere and was subject to extreme radiation, but
there was very little geologic upheaval or volcanic activity.
Solution The solution is (C). Early Earth had a very different atmosphere, consisting of less
molecular oxygen than is present in current times. The planet was also subject to
strong radiation and volcanic activity due to geologic upheaval.
9 Halophiles prefer conditions in which there is a —
A high sugar concentration
B salt concentration of at least 0.2 M
C pH of 3 or below
D high level of radiation
Solution The solution is (B). Halophiles have an absolute dependence on salt. As such, a salt
concentration of at least 0.2 M is needed for their survival. These salt-tolerant
microbes can form salt-tolerant bacterial mats such as those found in the Dead Sea.
10 The presence of a membrane-enclosed nucleus is a characteristic of —
A prokaryotic cells
B eukaryotic cells
C all cells
D viruses
Solution The solution is (B). Eukaryotic cells possess nuclei, the large membrane-bound
organelle present within the cell, which serves as the location of their DNA. This
organelle is absent within prokaryotes, which lack membranous intracellular
structures such as organelles.

********************************************************
*****************************************************************535***

11 All prokaryotic and eukaryotic cells have four structures in common: the plasma
membrane, the cytoplasm, nucleic acid, and —
A the cell wall
B ribosomes
C the nucleus
D organelles
Solution The solution is (B). Ribosomes are nonmembranous organelles found in both cell
types. The function of these structures is to serve as the site of protein synthesis for
the cell.
12 Which statement comparing the prokaryotes Bacteria and Archaea is false?
A The cytoplasm of both bacterial and archaean prokaryotic cells has a high
concentration of dissolved solutes.
B Osmotic pressure in both types of prokaryotic cells is relatively high.
C The domains Bacteria and Archaea differ in the use of fatty acids versus phytanal
groups in their cell membranes.
D The domains Bacteria and Archaea have very similar cell wall structure.
Solution The solution is (D). Unlike the cell walls of eubacteria in Domain Bacteria, the
composition of cell walls for Domain Archaea bacteria do not include peptidoglycan.
There are four different types of Archaean cell walls, one type of which is composed
of pseudopeptidoglycan, which is similar to peptidoglycan in morphology but
contains different sugars in the polysaccharide chain.
13 Pseudopeptidoglycan is a characteristic of the walls of some —
A eukaryotic cell
B bacterial prokaryotic cell
C archaean prokaryotic cells
D bacterial and archaean prokaryotic cells
Solution The solution is (C). Pseudopeptidoglycan is a glycoprotein found in Archaean cell
walls. It is similar to peptidoglycan in morphology but contains different sugars in
the polysaccharide chain.
14 The cell wall, a feature of most prokaryotes, is —
A interior to the cell membrane
B exterior to the cell membrane
C a part of the cell membrane
D interior or exterior, depending on the particular cell

********************************************************
**************************536*****************************************

Solution The solution is (B). The cell wall is located outside the cell membrane and prevents
osmotic lysis. The chemical composition of cell walls varies between the two major
bacterial Domains.
15 Which statement summarizes what is known about macronutrient needs of prokaryotes?
A Boron is required in small amounts by some prokaryotic organisms.
B Manganese is required in small amounts by some prokaryotic organisms.
C Iron is required in small amounts by some prokaryotic organisms.
D Sulfur is needed in large amounts by prokaryotic organisms. It is part of the structure
of some amino acids and is also present in some vitamins and coenzymes.
Solution The solution is (D). Macronutrients are nutrients that are required in large amounts.
Only a few elements are considered to be macronutrients. Examples include carbon,
hydrogen, oxygen, nitrogen, phosphorus, and sulfur.
16 Which statement about the importance of particular nutrients is false?
A Carbon is a macronutrient and major element in all macromolecules.
B Nitrogen is a macronutrient and necessary component of proteins and nucleic acids.
C Hydrogen is a macronutrient and key component of many organic compounds,
including water.
D Iron is a macronutrient necessary for the function of cytochromes.
Solution The solution is (D). This statement is false. Iron is not a macronutrient. While it is
needed for the proper function of cytochromes, it is a micronutrient, which means
that prokaryotes require it in small amounts.
17 What are prokaryotes that obtain their energy from chemical compounds called?
A Phototrophs
B Autotrophs
C Chemotrophs
D Heterotrophs
Solution The solution is (C). Chemotrophs, also referred to as chemosynthetic organisms,
obtain their energy from chemical compounds. Chemotrophs that can use organic
compounds as energy sources are called chemoorganotrophs, while those that can
also use inorganic compounds as energy sources are called chemolithotrophs.
18 What uses organic compounds as both an energy source and as a carbon source?
A Chemolithotrophs
B Photoautotrophs

********************************************************
*****************************************************************535***

C Photoheterotrophs
D Chemoorganotrophs
Solution The solution is (D). Chemotrophs that can use organic compounds as both an energy
source and a carbon source are called chemoorganotrophs.
19 A primary role of many prokaryotes in the carbon cycle is that of —
A producers
B decomposers
C fixers
D synthesizers
Solution The solution is (B). The most important contributor of carbon dioxide to the
atmosphere is microbial decomposition of dead material, such as dead animals,
plants, and humus, that undergo respiration. As decomposers, bacteria, along with
fungi, are responsible for carrying out the decomposition process of the
aforementioned materials.
20 Ammonification is the process by which —
A ammonia is released during the decomposition of nitrogen-containing organic
compounds
B ammonium is converted into nitrite and nitrate in soils
C nitrate from soil is transformed to gaseous nitrogen compounds
D gaseous nitrogen is fixed to yield ammonia
Solution The solution is (A). Ammonification is the process by which ammonia is released
during the decomposition of nitrogen-containing organic compounds. Ammonia
released to the atmosphere, however, represents only 15 percent of the total
nitrogen released; the rest is as N2 and N2O.
21 Which option is a macronutrient needed by prokaryotes?
A Phosphorus
B Iron
C Chromium
D Boron
Solution The solution is (A). Phosphorus is needed for the synthesis of nucleotides and
consequently DNA and RNA, as well as for post-translational modification of
proteins.

********************************************************
**************************536*****************************************

22 A disease that is constantly present in a population is called —


A pandemic
B endemic
C emerging
D reemerging
Solution The solution is (B). An endemic disease is a disease that is constantly present, usually
at low incidence, in a population.
23 Which set of terms names diseases caused by bacteria?
A Diphtheria, bubonic plague, yellow fever
B Yellow fever, dengue fever, bubonic plague
C Bubonic plague, diphtheria, cholera
D Cholera, diphtheria, dengue fever
Solution The solution is (C). All three terms refer to diseases that are caused by bacteria.
Specifically, diphtheria is caused by the bacterium Corynebacterium diptheriae,
bubonic plague is caused by the bacterium Yersina pestis, and cholera is caused by
the bacterium Vibrio cholerae.
24 Which health issue is caused by biofilm colonization?
A Dental plaque
B Dry scalp
C Skin rash
D Prosthetic discomfort
Solution The solution is (A). A biofilm is a microbial community held together in a gummy-
textured matrix that consists primarily of polysaccharides secreted by the organisms,
together with some proteins and nucleic acids. Biofilms grow attached to surfaces,
such as teeth, where they can produce dental plague and eventually lead to tooth
decay, if left untreated.
25 Which statement about the loci of biofilm-related disease is false?
A Biofilms are related to foodborne illnesses because they colonize food surfaces and
food-processing equipment.
B In healthcare environments, biofilms grow on ventilators, shunts, and other medical
equipment.
C Biofilms tend to colonize medical devices such as prostheses, contact lenses, and
catheters.
D Biofilms do not form in open wounds, burned tissue, or internal medical devices such
as pacemakers.

********************************************************
*****************************************************************535***

Solution The solution is (D). Biofilms do not form in open wounds, burned tissue, or internal
medical devices such as pacemakers.
26 Which statement best describes the crisis related to antibiotics?
A It is becoming too expensive to manufacture effective antibiotics.
B It takes too much time to develop effective antibiotics; infections spread before
treatment is available.
C Bacteria are increasingly resistant to antibiotics used to treat and eradicate infections.
D People are increasingly allergic to antibiotics commonly used in treatment.
Solution The solution is (C). As result of mutations in bacteria some become resistant to
certain antibiotic, and as such will be able to reproduce regardless of the presence of
that antibiotic.
27 Which statement about the cause of resistant bacteria is false?
A The excessive use of antibiotics has resulted in the natural selection of resistant forms
of bacteria.
B Antibiotics are used by patients with colds or the flu, the treatment for which
antibiotics are useless.
C There is excessive use of antibiotics in livestock and in animal feed.
D Antibiotics are used by patients of different ages and the fact that their ages differ
increases resistance.
Solution The solution is (D). One of the main causes of resistant bacteria is the abuse of
antibiotics. The imprudent and excessive use of antibiotics has resulted in the
natural selection of resistant forms of bacteria. The antibiotic kills most of the
infecting bacteria, and therefore only the resistant forms remain. These resistant
forms reproduce, resulting in an increase in the proportion of resistant forms over
non-resistant ones. Another major misuse of antibiotics is in patients with colds or
the flu, for which antibiotics are useless. Another problem is the excessive use of
antibiotics in livestock. The routine use of antibiotics in animal feed promotes
bacterial resistance as well.
28 Which statement about diseases is false?
A An epidemic is a disease that occurs in a high number of individuals in a population at
a time.
B A pandemic is a widespread, usually worldwide, epidemic.
C An endemic disease is a disease that is constantly present, usually at high incidence, in
a population.
D An emerging disease is a disease that has appeared in a population for the first time.

********************************************************
**************************536*****************************************

Solution The solution is (C). This statement is false. An endemic disease is a disease that is
constantly present, usually at low incidence, not high incidence, in a population.
29 Which statement best explains which organisms need nitrogen fixation and why?
A Prokaryotes cannot use gaseous nitrogen to synthesize macromolecules, so it must be
converted into ammonia.
B Prokaryotes cannot use ammonia to synthesize macromolecules, so it must be
converted into gaseous nitrogen.
C Eukaryotes cannot use ammonia to synthesize macromolecules, so it must be
converted into gaseous nitrogen.
D Eukaryotes cannot use gaseous nitrogen to synthesize macromolecules, so it must be
converted into ammonia.
Solution The solution is (D). Eukaryotes cannot use gaseous nitrogen to synthesize
macromolecules, so it must be converted into ammonia. The largest pool of nitrogen
available in the terrestrial ecosystem is gaseous nitrogen from the air, but this
nitrogen is not usable by eukaryotes, such as plants. Gaseous nitrogen is
transformed, or fixed into more readily available forms such as ammonia through
the process of nitrogen fixation. Ammonia can be used by plants or converted into
other forms.
30 Which statement about nitrogen fixation is false?
A It can be accomplished abiotically, as a result of lightning.
B It can be accomplished abiotically, as a result of industrial processes.
C It can be accomplished biologically, by algae.
D It can be accomplished biologically, by cyanobacteria.
Solution The solution is (C). It can be accomplished biologically, by algae. Biological nitrogen
fixation (BNF) is carried out exclusively by prokaryotes such as soil bacteria,
cyanobacteria, and Frankia spp., filamentous bacteria interacting with actinorhizal
plants such as alder, bayberry, and sweet fern. As eukaryotes, algae are unable to
carry out nitrogen fixation.
31 Which three foods use prokaryotes in their processing?
A Cheese, yogurt, and milk
B Cheese, yogurt, and bread
C Wine, bread, and butter
D Milk, wine, and beer
Solution The solution is (B). Food products such as cheese, bread, and yogurt are produced
via the use of bacteria and other microbes, such as yeast, a fungus. The use of such
microbes for the use of food production is referred to as biotechnology.

********************************************************
*****************************************************************535***

32 What was the initial benefit for humans in processing foods with prokaryotes?
A The foods taste better.
B Nutrients are preserved.
C The food is less stable.
D Nutrients were safer.
Solution The solution is (B). Scientists believe that the production of certain food products
such as cheese began approximately 4,000–7,000 years ago. Fermentation, the
anaerobic process whereby cheese is produced by microbes, results in the
preservation of nutrients: Milk will spoil relatively quickly, but when processed as
cheese, it is more stable.
33 Which option best defines bioremediation?
A The use of microbial metabolism to clean up oil spills
B The use of microbial metabolism to ferment food
C The use of microbial metabolism to remove pollutants
D The use of microbial metabolism to fix nitrogen
Solution The solution is (C). Microbial bioremediation is the use of prokaryotes, via their
metabolic processes, to remove pollutants from the environment. Bioremediation
has been used to remove contaminants such as those arising from agricultural
chemicals (i.e., pesticides, fertilizers).
34 Which statement about bioremediation is false?
A It includes removing agricultural chemicals.
B It includes removing industrial by-products.
C It includes cleaning up oil spills.
D It includes cleaning up ammonia in soil.
Solution The solution is (D). Bioremediation has been used to remove agricultural chemicals
—pesticides, fertilizers—that leach from soil into groundwater and the subsurface.
Certain toxic metals and oxides, such as selenium and arsenic compounds, also can
be removed from water by bioremediation.
35 Nitrogen is an essential element that is widely available in the atmosphere. Because
eukaryotes cannot use nitrogen in its gaseous form, they benefit from prokaryotes’
conversion of gaseous nitrogen to —
A nitrates, a form of nitrogen they can use
B phosphate, a different essential element they can use
C ammonia, a form of nitrogen they can use
D hydrogen, a different essential element they can use

********************************************************
**************************536*****************************************

Solution The solution is (C). Ammonia is a form of nitrogen they can use, unlike nitrates, a
form of nitrogen they cannot use, and phosphate and hydrogen, which are not
sources of nitrogen.

CRITICAL THINKING QUESTIONS


36 What is the relationship between Earth’s ancient atmosphere and the evolution of some
of the first life-forms on Earth? Use the terms anaerobic and phototrophic, and explain
the effect of cyanobacteria on the atmosphere.
A Phototrophic organisms appeared during the first 2 billion years of Earth’s existence.
Anaerobic organisms appeared within 1 billion years of Earth’s formation. From these
organisms evolved the cyanobacteria that produce oxygen as a by-product of
photosynthesis, leading to the oxygenation of the atmosphere.
B For the first 2 billion years of Earth’s existence, the atmosphere had no molecular
oxygen. Thus, the first organisms were anaerobic. Cyanobacteria appeared within 1
billion years of Earth’s formation. From these evolved the phototrophic organisms
that produce oxygen as a by-product of photosynthesis, leading to the oxygenation of
the atmosphere.
C For the first 2 billion years of Earth’s existence, the atmosphere had no molecular
oxygen. Thus, the first organisms were anaerobic. Phototrophic organisms appeared
within 1 billion years of Earth’s formation. From these organisms evolved the
cyanobacteria, which produce oxygen as a by-product of photosynthesis, leading to
the oxygenation of the atmosphere.
D For the first 2 billion years of Earth’s existence, the atmosphere had no molecular
oxygen. Thus, the first organisms were anaerobic. Cyanobacteria that produce oxygen
as a by-product of photosynthesis, leading to the oxygenation of the atmosphere,
appeared within 1 billion years of Earth’s formation. From these organisms evolved
phototrophic organisms.
Solution The solution is (C). For the first 2 billion years of Earth’s existence, the atmosphere
had no molecular oxygen. Thus, the first organisms were anaerobic, which means
that they grow without oxygen. Phototrophic organisms are organisms that can
convert solar energy into chemical energy. These types of organisms appeared
within 1 billion years of Earth’s formation. From these organisms evolved the
cyanobacteria, also known as blue-green algae. Blue-green algae photosynthesize,
and since oxygen is a by-product of photosynthesis, the success of these organisms
led to the oxygenation of the atmosphere.

********************************************************
*****************************************************************535***

37 Extremophiles are considered an important area for research in the development of


therapeutic drugs or industrial applications. Why do you think this is so?
A Extremophiles can be altered genetically in vitro to allow them to live in extreme
conditions and this capability of alteration can be used to help humans. For example,
some water-resistant prokaryotes have developed DNA repair mechanisms. Also, they
could be developed and used in the treatment of human disease.
B Extremophiles have specialized adaptations that allow them to live in extreme
conditions. These adaptations can be mobilized to help humans. For example, some
water-resistant prokaryotes have developed DNA repair mechanisms. Also, they could
be developed and used in the treatment of human disease.
C Extremophiles can be altered genetically in vitro to allow them to live in extreme
conditions and this capability of alteration can be used to help humans. For example,
some radiation-resistant prokaryotes have developed DNA repair mechanisms. Also,
they could be developed and used in the treatment of human disease.
D Extremophiles have specialized adaptations that allow them to live in extreme
conditions. These adaptations can be mobilized to help humans. For example, some
radiation-resistant prokaryotes have developed DNA repair mechanisms. Also, they
could be developed and used in the treatment of human disease
Solution The solution is (D). Research might find ways to mobilize the adaptations of
extremophiles to help humans. For example, some radiation-resistant prokaryotes
have developed DNA repair mechanisms. If these could be better understood,
perhaps they could be developed and used in the treatment of human disease.
38 How would you detect the presence of a nonculturable prokaryote in an environmental
sample?
A Recombinant DNA techniques are used to detect the presence of a nonculturable
prokaryote in an environmental sample. Polymerase chain reaction is used to amplify
selected portions of prokaryotic DNA.
B Molecular biology techniques are used to detect the presence of a nonculturable
prokaryote in an environmental sample. Electrophoresis is used to amplify selected
portions of prokaryotic DNA.
C Molecular biology techniques are used to detect the presence of a nonculturable
prokaryote in an environmental sample. Polymerase chain reaction is used to amplify
selected portions of prokaryotic DNA.
D Recombinant DNA techniques are used to detect the presence of a nonculturable
prokaryote in an environmental sample. Electrophoresis is used to amplify selected
portions of prokaryotic DNA.
Solution The solution is (C). Since I cannot culture the prokaryote to find and grow colonies of
it, I would use molecular biology techniques to detect its presence. I would use
polymerase chain reaction (PCR) to amplify selected portions of prokaryotic DNA.

********************************************************
**************************536*****************************************

39 Why do scientists believe that the first organisms on Earth were extremophiles?
A Earth’s early environment was full of extreme places with much oxygen in the
atmosphere, no ozone to shield Earth’s surface from mutagenic radiation, much
geologic upheaval, and volcanic activity. Extremophiles are bacteria and archaea that
are adapted to grow in extreme environments.
B Earth’s early environment was full of extreme places with little oxygen in the
atmosphere, no ozone to shield Earth’s surface from mutagenic radiation, much
geologic upheaval, and volcanic activity. Extremophiles are bacteria and archaea that
are adapted to grow in extreme environments.
C Earth’s early environment was full of extreme places with little oxygen in the
atmosphere, no ozone to shield Earth’s surface from mutagenic radiation, less
geologic upheaval, and volcanic activity. Extremophiles are bacteria and archaea that
are adapted to grow in extreme environments.
D For the first two billion years of Earth’s existence, the atmosphere had no molecular
oxygen.
Solution The solution is (B). Extremophiles are bacteria and archaea that are adapted to grow
in extreme environments. Extreme environments are those with harsh conditions.
This means places with very high or low temperatures, places with extremely salty,
deep, or hot water, places without light, places that are dry, places subject to lots of
radiation, and so on. Earth’s early environment was full of extreme places. There
was little oxygen in the atmosphere, which means there was no ozone to shield
Earth’s surface from mutagenic radiation. There was much geologic upheaval and
volcanic activity, as well. The first organisms had to be organisms (prokaryotes) that
could withstand an environment of one or more of these extremes (temperature,
pressure, radiation, upheaval).
40 Which statement describes a typical prokaryotic cell?
A It has a cell wall enclosing cell membrane, cytoplasm, ribosomes, and nucleoid region
with genetic material. It may have a protective capsule, flagellum, pili, and plasmids.
B It has a cell wall enclosing cell membrane, cytoplasm, ribosomes, and nucleus
containing genetic material. It may have a protective capsule, flagellum, pili, and
plasmids.
C It has a cell wall enclosing nuclear membrane, cytoplasm, ribosomes, and nucleoid
region with genetic material. It may have a protective capsule, flagellum, pili, and
plasmids.
D It has a cell wall enclosing nuclear membrane, cytoplasm, mitochondria, vacuoles, and
nucleoid region with genetic material. It may have a protective capsule, flagellum, pili,
and plasmids.
Solution The solution is (A). A typical prokaryotic cell has a cell wall that encloses a cell
membrane. It has cytoplasm, ribosomes, and a nucleoid region with genetic material

********************************************************
*****************************************************************535***

(chromosome). It may have a protective capsule, a flagellum for locomotion, and pili
for attachment to surfaces. It may also have plasmids.
41 Explain the statement that both Archaea and Bacteria have the same basic structures, but
these structures are built from different chemical components.
A Typical cells in Archaea and Bacteria contain a cell wall, cell membrane, nucleoid
region, ribosomes, and often a capsule, flagellum, and pili. However, these are
sometimes made from different chemical compounds. Cell walls of Bacteria contain
peptidoglycan while Archaea do not. Plasma membrane lipids of Bacteria are fatty
acids while those of Archaea are phytanyl groups.
B Typical cells in Archaea and Bacteria contain a cell wall, cell membrane, nucleoid
region, and often a capsule, flagellum, and pili, but in some instances, different
chemical compounds make them. Cell walls of Bacteria contain peptidoglycan while
Archaea do not. Bacteria contain 70S ribosomes while Archaea contain 80S ribosomes.
C Typical cells in Archaea and Bacteria contain a cell wall, nuclear membranes, nucleoid
region, and often a capsule, flagellum, and pili, but in some instances, different
chemical compounds make them. Cell walls of Bacteria contain peptidoglycan while
Archaea do not. Plasma membrane lipids of bacteria are fatty acids, while the plasma
membrane lipids of Archaea are phytanyl groups.
D Typical cells in Archaea and Bacteria contain a cell wall, cell membrane, nucleoid
region, and often a capsule, flagellum, and pili, but in some instances, different
chemical compounds make them. Cell walls of Bacteria contain peptidoglycan while
Archaea do not. Plasma membrane lipids of Bacteria are phytanyl groups, while the
plasma membrane lipids of Archaea are fatty acids.
Solution The solution is (A). As prokaryotes, typical cells in both Archaea and Bacteria have
the same basic structures: cell wall, cell membrane, nucleoid region, ribosomes, and
often a capsule, flagellum, and pili. However, in a couple of significant instances,
these structures are built from different chemical components. The cell walls of
bacteria contain peptidoglycan, while the cell walls of archaea do not. The plasma
membrane lipids of bacteria are fatty acids, while the plasma membrane lipids of
archaea are phytanyl groups.
42 Three basic prokaryotic categories are cocci, spirilli, and bacilli. What are the basic
structural features of each category?
A These three prokaryote groups have similar basic structural features. They typically
have cell walls enclosing nuclear membranes, cytoplasm, ribosomes, mitochondria,
and nucleoid region with genetic material. They may have a protective capsule,
flagellum, pili, and plasmids.
B Cocci and spirilli have similar basic structural features. They typically have cell walls
enclosing cell membranes, a flagellum for locomotion, and pili for attachment. Bacilli
are rod shaped which contain ribosomes and a nucleoid region with genetic material.

********************************************************
**************************536*****************************************

C These three prokaryote groups have similar basic structural features. They typically
have cell walls enclosing cell membranes, cytoplasm, ribosomes, and a nucleoid region
with chromosomes. They may have a protective capsule, flagellum, pili, and plasmids.
D Bacilli and spirilli have similar basic structural features. They typically have cell walls
enclosing nuclear membranes, a flagellum for locomotion, and pili for attachment.
Cocci are spherical, containing ribosomes and a nucleoid region with genetic material.
Solution The solution is (C). These three categories are based on shape: cocci are spherical,
spirilli are spiral-shaped, and bacilli are rod-shaped. In terms of basic structure,
however, these prokaryotes are the same. They typically have cell walls that enclose
cell membranes. They all have cytoplasm, ribosomes, and a nucleoid region with
genetic material (chromosome). They may have a protective capsule, a flagellum for
locomotion, and pili for attachment to surfaces. They may have extra-chromosomal
DNA in plasmids, too.
43 Which macronutrient do you think is most important? What evidence can you offer to
support your choice?
A Carbon represents 12 percent of the total dry weight of a typical cell and is a
component of all macromolecules.
B Oxygen is necessary and is a major component for all macromolecules. It also
accounts for 50 percent of the total composition of a cell.

********************************************************
*****************************************************************535***

C Carbon is necessary and is a major component for all macromolecules. It also accounts
for 50 percent of the total composition of a cell.
D Nitrogen is necessary and is a major component for all macromolecules. It also
accounts for 50 percent of the total composition of a cell.
Solution The solution is (C). Carbon is the most important macronutrient because it is not
only necessary for all macromolecules, it is the major component of them. This
includes proteins, carbohydrates, nucleic acids, and lipids. Moreover, carbon
accounts for about 50 percent of the total composition of a cell.
44 A bacterium requires only a particular amino acid as an organic nutrient and lives in a
completely lightless environment. What mode of nutrition—free energy and carbon—
does it use? Justify your response.
A Chemoheterotroph, as it must rely on chemical sources of energy living in a lightless
environment and a heterotroph if it uses organic compounds for its carbon source
B Chemoorganotroph, as it must rely on chemical sources of energy living in a lightless
environment and an organotroph if it uses organic compounds other than carbon
dioxide for its carbon source
C Chemolitoautotroph, as it must rely on chemical sources of energy living in a lightless
environment and an autotroph if it uses organic compounds other than carbon dioxide
for its carbon source
D Chemoheterotroph, as it must rely on chemical sources of energy living in a lightless
environment and a heterotroph if it uses organic compounds other than carbon
dioxide for its carbon source
Solution The solution is (D). This bacterium is a chemoheterotroph. It must rely on chemical
sources of energy if it lives in a lightless environment and it must be a heterotroph if
it uses organic compounds other than carbon dioxide for its carbon source.
45 Assuming that you could synthesize all of the nitrogen-containing compounds needed if
you had nitrogen, what might you eat for a typical meal if you could fix nitrogen like some
prokaryotes?
A My meal might be fruits or vegetables, bread, and water, as nitrogen is present in the
highest amount in water.
B My meal might be fruits or vegetables, water, bread, and air, as atmospheric nitrogen
could be simply absorbed.
C My meal might be fruits or vegetables, cheese, meat, water, bread, and air, as
atmospheric nitrogen could be simply absorbed.
D My meal might be cheese or meat, water, bread, and air, as atmospheric nitrogen
could be simply absorbed.

********************************************************
**************************536*****************************************

Solution The solution is (B). If I could fix nitrogen, I would use atmospheric nitrogen to build
proteins, so I would not need to eat foods high in proteins. I would simply absorb
nitrogen! However, I would still need other nutrients, including a source of carbon
(carbohydrates would suffice), essential minerals, and water. So, a typical meal
might be fruits or vegetables, water, bread, and air (for nitrogen).
46 Which are more important: macronutrients or micronutrients? Explain your reasoning.
A Neither are important, as cells can survive as well as carry out essential functions
without both types of nutrients.
B Micronutrients, even though they are required in lesser amounts, without them cells
cannot survive and carry out functional processes.
C Macronutrients, as they are required in larger amounts by cells and thus are more
essential than micronutrients.
D Neither is more important as both types of nutrients are absolutely necessary for
prokaryotic cell structure and function.
Solution The solution is (D). Neither macronutrients or micronutrients are more important.
While macronutrients are needed in larger amounts and micronutrients are needed
in smaller amounts, both types of nutrients are absolutely necessary to prokaryotic
cell structure and function. Without any of the necessary macronutrients or
micronutrients, the cell could not survive, let alone carry out functional processes.
47 Identify and discuss a bacterial disease that caused a historically important plague or
epidemic. What is the modern distribution of this disease?
A Bubonic plague, caused by Yersinia pestis, was a pandemic that occurred in the
14th century. In modern times, there are only about 100 cases of bubonic plague each
year. The bacterium responds well to modern antibiotics.
B Bubonic plague, caused by Yersinia enterocolitica, was a pandemic that occurred in
the 14th century. In modern times, there are about 1,000 to 3,000 cases of bubonic
plague each year. The bacterium responds well to modern antibiotics.
C Pneumonic plague, caused by Yersinia pestis, was a pandemic that occurred in the
14th century. In modern times, there are about 1,000 to 3,000 cases of pneumonic
plague each year. The bacterium responds well to modern antibiotics.
D Bubonic plague, caused by Yersinia pestis, was a pandemic that occurred in the
14th century. In modern times, there are about 1,000 to 3,000 cases of bubonic
plague each year. The bacterium responds well to modern antibiotics.
Solution The solution is (A). The Black Death, or bubonic plague, was a pandemic that
occurred in the 14th century. Believed to have been caused by the bacterium
Yersinia pestis, it wiped out some 100 million people in the Mediterranean and
Europe. In modern times, the disease continues to be studied. There are about 1,000
to 3,000 cases of bubonic plague each year. The bacterium responds well to modern
antibiotics.

********************************************************
*****************************************************************535***

48 Have foodborne illnesses related to biofilms changed over time? Explain.


A Yes, better sterilization and canning procedures have reduced the incidence of
botulism. Most cases of foodborne illness now are related to small-scale food
production.
B No, better sterilization and canning procedures have reduced the incidence of
botulism. Most cases of foodborne illness now are related to small-scale food
production.
C No, better sterilization and canning procedures have increased the incidence of
botulism. Most cases of foodborne illnesses now are related to large-scale food
production.
D Yes, better sterilization and canning procedures have reduced the incidence of
botulism. Most cases of foodborne illnesses now are related to large-scale food
production.
Solution The solution is (A). In the past, foodborne illnesses were commonly related to
Clostridium botulinum, a bacterium that causes the fatal disease botulism. The
common sources for this bacterium were canned foods, homemade pickled goods,
and processed meats. Better sterilization and canning procedures have reduced the
incidence of botulism. Most cases of foodborne illness now are related to produce
and large-scale food production.
49 What is the relationship between MRSA and the problem of antibiotic resistance?
A Indiscriminate use of antibiotics results in the population of resistant bacteria like
MRSA to grow.
B There should be a comma after MRSA in the second sentence. Virulence is misspelled.
C MRSA affects a range of populations and is especially common in younger populations
living in compromised environments where infections are common. The fact that
young individuals are susceptible to MRSA indicates its highly virulent status.
D Indiscriminate use of antibiotics results in the bacterial population to reproduce and
grow rapidly
Solution The solution is (A). MRSA stands for methicillin-resistant Staphyloccoccus aureus, a
strain of the bacteria, Staphylococcus aureus, that is resistant to many commonly
used antibiotics, including methicillin, amoxicillin, penicillin, and oxacillin. This strain
has developed because of the imprudent use of antibiotics. When antibiotics are
used, most of the infecting bacteria is killed, and only the resistant forms remain.
These resistant forms reproduce, resulting in an increased population of resistant
bacteria. When the use of antibiotics occurs indiscriminately, the populations of
resistant bacteria simply grow in response.
50 What was the Plague of Athens? What is the modern distribution of this disease?

********************************************************
**************************536*****************************************

A The Plague of Athens was a disease caused by Yersinia pestis that killed one-quarter of
Athenian troops in 430 B.C. Between 10 and 15 million cases of typhoid fever occur
today, resulting in over 10,000 deaths annually.
B The Plague of Athens was a disease caused by Salmonella entericaserovar typhi that
killed one-quarter of Athenian troops in 430 B.C. Between 5 and 10 million cases of
typhoid fever occur today, resulting in over 20,000 deaths annually.
C The Plague of Athens was a disease caused by Yersinia pestis that killed one-quarter of
Athenian troops in 430 B.C. Between 16 and 33 million cases of typhoid fever occur
today, resulting in over 200,000 deaths annually.
D The Plague of Athens was a disease caused by Salmonella entericaserovar typhi that
killed one-quarter of Athenian troops in 430 B.C. Between 16 and 33 million cases of
typhoid fever occur today, resulting in over 200,000 deaths annually.
Solution The solution is (D). The Plague of Athens was a disease that killed one-quarter of
Athenian troops in 430 B.C. This disease is believed to have been caused by the
same bacterium that causes typhoid fever. Between 16 and 33 million cases of
typhoid fever occur today, resulting in over 200,000 deaths annually. It is considered
a reemerging disease and is monitored by the World Health Organization.
51 Identify three beneficial results of symbiotic nitrogen fixation.
A Plants benefit from an endless supply of nitrogen; soils benefit from being naturally
fertilized; and bacteria benefit from using potassium from plants.
B Plants benefit from a limited supply of nitrogen; soils benefit from being naturally
fertilized, and bacteria benefit from using photosynthates from plants.
C Plants benefit from an endless supply of carbon; soils benefit from being naturally
fertilized; and bacteria benefit from using photosynthates from plants.
D Plants benefit from an endless supply of nitrogen; soils benefit from being naturally
fertilized; and bacteria benefit from using photosynthates from plants.
Solution The solution is (D). The plant benefits from an endless supply of nitrogen; the soil
benefits from being naturally fertilized; bacteria benefit from using the
photosynthates from the plant.
52 Why is the processing of foods with prokaryotes considered an example of early
biotechnology?
A Prokaryotes have been used to only make specific food products like cheese, wine,
bread, beer, and yogurt since before the term biotechnology was coined.
B Prokaryotes have been used to make and alter specific food products like cheese,
wine, single cell proteins, beer, and yogurt since before the term biotechnology was
coined.
C Prokaryotes have been used to make and alter specific food products like cheese,
wine, bread, beer, and yogurt since before the term biotechnology was coined.

********************************************************
*****************************************************************535***

D Prokaryotes have been used to alter specific food products like cheese, wine, bread,
beer, and yogurt since before the term biotechnology was coined.

********************************************************
**************************536*****************************************

Solution The solution is (C). Biotechnology is defined as any technological application that
uses biological systems or living things to make or modify products for specific use.
The use of prokaryotes (living things) to make and alter specific food products—
alternative and useful food sources—is a clear example of early use of biological
systems and living things for technological purposes.
53 On what does the success of bioremediation of oil spills depend?
A Success depends on the presence of only aromatic and highly branched hydrocarbon
chain compounds and the temperature.
B Success depends on the presence of less nonvolatile and more aromatic and highly
branched hydrocarbon chain compounds and the temperature.
C Success depends on the type of oil compounds, the presence of naturally occurring oil-
solubilizing prokaryotes in the ocean, and the type of water body.
D Success depends on the type of oil compounds, the presence of naturally occurring oil-
solubilizing prokaryotes in the ocean and the temperature.
Solution The solution is (D). Success depends on the type of oil compounds (nonvolatile ones
are easier to remove) and the presence of naturally occurring oil-solubilizing
prokaryotes in the ocean. Temperature is also a factor.
54 Why is the relationship between sustainable agriculture and nitrogen fixers called a
mutualism?
A Due to agrobacterium, which are nitrogen fixers, plants benefit from an endless supply
of nitrogen; soils benefit from being naturally fertilized; and bacteria benefit from
using photosynthates from plants.
B Due to rhizobia, which are nitrogen fixers, plants benefit from an endless supply of
nitrogen; soils benefit from being naturally fertilized; and bacteria benefit from using
photosynthates from plants.
C Due to rhizobia, which are nitrogen fixers, plants benefit from an endless supply of
nitrogen; soils benefit from being naturally fertilized; and bacteria benefit from using
potassium from plants.
D Due to rhizobia, which are nitrogen fixers, plants benefit from a limited supply of
nitrogen; soils benefit from being naturally fertilized; and bacteria benefit from using
potassium from plants.
Solution The solution is (B). Rhizobia are soil bacteria that fix nitrogen in the nodules of
legumes, plants in which the bacteria live symbiotically. Because the plant benefits
from an endless supply of nitrogen, the soil benefits from being naturally fertilized,
and the bacteria benefit from using the photosynthates from the plant, the process
results in rhizobia as sustainable biofertilizers.

********************************************************
*****************************************************************535***

TEST PREP FOR AP® COURSES


55 Which piece of evidence is the best support for the alternative scenario of early life
formation, in which organic compounds on early Earth formed near submerged
volcanoes?
A Some prokaryotes that live near deep-sea vents today use hydrogen as an energy
source.
B Fossilized stromatolites that are 3.5 billion years old are found near deep-sea vents.
C Extremophiles that exist today live in a variety of extreme environments, including
those that are high in salinity.
D The chemical composition of water around deep-sea vents is the same as it was on
early Earth.
Solution The solution is (B). Fossilized stromatolites that are 3.5 billion years old are found
near deep-sea vents. Fossilized microbial mats represent the earliest record of life
on Earth. Stromatolites are sedimentary structures formed when minerals are
precipitated out of water by prokaryotes in a microbial mat.
56 When Miller and Urey repeated their experiment without the electrical discharge, no
organic compounds were found. What might explain this result? Consider your answer in
the context of the conditions of early Earth.
A The lack of organic compounds without the sparks indicates that organic components
are formed from biotic components.
B The first trial of the experiment must have been done incorrectly.
C Abiotic molecules can only develop into organic molecules in the presence of oxygen,
so oxygen should be added.
D Lightning, or some form of energy, is needed for the inorganic molecules in the
atmosphere to interact with each other. This indicates that a similar energy source
was present on early Earth, which stimulates the interaction and development.
Solution The solution is (D). The lack of organic compounds without the sparks indicates that
lightning, or some form of energy, is needed for the inorganic molecules in the
“atmosphere” to interact with each other. If organic compounds formed from
abiotic components as this experiment shows, then it follows that early Earth would
have had to have lightning or some other, similar energy source stimulating that
interaction and development.
57 Laboratory experiments have demonstrated that the abiotic synthesis of organic
molecules in condition similar to those of early Earth is possible. Which statement
provides additional support for the idea of abiotic synthesis of organic compounds?
A Analysis of the chemical composition of meteorites sometimes yields amino acids.
B A hydrothermal vent in the Sea of Cortez releases hydrogen sulfide and iron sulfide.

********************************************************
**************************536*****************************************

C Researchers have dripped solutions of amino acids onto hot surfaces to produce
amino acid polymers.
D Some present-day prokaryotes live and reproduce in very extreme and unforgiving
environments, such as the Arctic.
Solution The solution is (B). The first microbial mats likely obtained their energy from
chemicals found near hydrothermal vents. A hydrothermal vent is a breakage or
fissure in Earth’s surface that releases geothermally heated water. With the
evolution of photosynthesis about 3 billion years ago, some prokaryotes in microbial
mats came to use a more widely available energy source—sunlight—whereas others
were still dependent on chemicals from hydrothermal vents for energy and food.
58 Which cell type does the diagram illustrate?

A Plant cell
B Animal cell
C Bacterial cell
D Fungal cell
Solution The solution is (C). This is a bacterial cell because there is no true nucleus. The
chromosomal DNA is condensed in an area called a nucleoid which is not surrounded
by a membrane.
59 Which option best describes the function and presence of cell organelles among
prokaryotes and eukaryotes?
A Ribosomes are the sites of protein synthesis found in prokaryotic and eukaryotic cells.
The cell wall is a protective layer, present in most prokaryotic cells and in some
eukaryotes. Chromosomal DNA, the genetic material of the cell, is present in a
nucleoid region in prokaryotes while enclosed in a nucleus in eukaryotes.
B Ribosomes are the sites of protein synthesis found in prokaryotic and eukaryotic cells.
The cell wall is a protective layer found in a few prokaryotic and eukaryotic cells.
Chromosomal DNA is the genetic material of the cell that is present in a nucleoid
region in prokaryotes while in eukaryotes, it is enclosed in a nucleus.

********************************************************
*****************************************************************535***

C Ribosomes are sites of ATP production found in both prokaryotic and eukaryotic cells.
The cell wall is a protective layer, typically found in prokaryotic cells and in some
eukaryotes. Chromosomal DNA is present in a nucleoid region while enclosed in a
nucleus in eukaryotes. It is the genetic material of the cell.
D Ribosomes are the sites of protein synthesis found in prokaryotic and eukaryotic cells.
The cell wall is a protective layer, typically found in prokaryotic cells and in some
eukaryotes. Chromosomal DNA, the genetic material of the cell is present in a nucleus
in prokaryotes, while it is enclosed in a nucleoid region in eukaryotes.
Solution The solution is (A). Ribosomes are found in both prokaryotic and eukaryotic cells.
They are sites of protein synthesis. The cell wall is a protective layer characteristic of
prokaryotic cells and in some eukaryotes (plant cells, fungal cells). Chromosomal
DNA is the genetic material of the cell. In prokaryotes this material is present in a
more diffuse nucleoid region (not enclosed in a membrane). In eukaryotes, DNA is
enclosed in a membrane-bound nucleus.
60 A nonpathogenic bacterium acquires resistance to antibiotics. Which scenario describing
how this strain could pose a health risk to people is false?
A Genes for antibiotic resistance are transferred from the nonpathogenic bacterium to a
pathogenic bacterium via transduction.
B Genes for antibiotic resistance are transferred from the nonpathogenic bacterium to a
pathogenic bacterium via transformation.
C Genes for antibiotic resistance are transferred from the nonpathogenic bacterium to a
pathogenic bacterium via conjugation.
D Genes for antibiotic resistance are transferred from the nonpathogenic bacterium to a
pathogenic bacterium via binary fission.
Solution The solution is (D). Genes for antibiotic resistance are transferred from the
nonpathogenic bacterium to a pathogenic bacterium via conjugation or
transformation. Binary fission does not provide an opportunity for genetic
recombination or genetic diversity. Prokaryotes can share genes by three other
mechanisms—transduction, conjugation, or transformation.
61 In a rapidly changing environment, which prokaryotic population would you hypothesize
likely to be more successful: one that included individuals capable of conjugation or one
that did not?
A A population including individuals capable of conjugation would be more successful,
because all of its members would form recombinant cells having new gene
combinations advantageous in a new environment.
B A population including individuals capable of conjugation would be more successful,
as some members could form recombinant cells having new gene combinations
advantageous in a new environment.

********************************************************
**************************536*****************************************

C A population including individuals not capable of conjugation would be more


successful, as the members undergoing conjugation would form new recombinant
cells having gene combinations lethal in the new environment.
D A population including individuals not capable of conjugation would be more
successful, because conjugation will result in an increase in genetic diversity of the
prokaryotic population, which will be disadvantageous in a new population.
Solution The solution is (B). A population that included individuals capable of conjugation
would probably be more successful because some of its members could form
recombinant cells. These recombinant cells could have new gene combinations that
were advantageous in a new environment. In conjugation, DNA is transferred from
one prokaryote to another by means of a pilus, which brings the organisms into
contact with one another. The DNA transferred can be in the form of a plasmid or as
a hybrid, containing both plasmid and chromosomal DNA. This transfer of DNA could
lead to resistance plasmids and thus genes being acquired, which confer survival
advantages to a microbe that would be advantageous in a new environment.
62 Plates that have only ampicillin-resistant bacteria growing include which of the following?

A I only
B III only
C IV only
D I and II
Solution The solution is (C). Plate IV represents the growth of ampicillin-resistant bacteria.
63 Given your understanding of the experiment and of bacterial genetic recombination,
explain why there are fewer colonies on plate IV than on plate III.
A All E. coli cells were not successfully transformed on plate IV.
B The nutrient agar medium inhibited the growth of some bacteria on plate IV.
C All E. coli cells were successfully transformed on plate IV.
D The bacteria in plate III were naturally resistant to ampicillin.
Solution The solution is (A). It seems likely that not all E. coli cells were successfully
transformed. That is, not all bacteria took in the plasmid that confers ampicillin

********************************************************
*****************************************************************535***

resistance, so not as many bacteria grew on the plate to which ampicillin was
applied as on the plate to which it was not.
64 Which labeled structure in the diagram allows you to positively identify the cell as a
prokaryote?

A Structure A – circular DNA


B Structure B – ribosome

********************************************************
**************************536*****************************************

C Structure C – cell wall


D Structure D – cytoplasm
Solution The solution is (A). Circular DNA is the most frequent structure in prokaryotes, as
eukaryotes possess DNA that is divided into several separate, linear chromosomes.
65 A bacterial species that is a methanogen is discovered. If you wanted to build on this
discovery to better understand the evolution of mechanisms related to the ability to
capture, store, and use free energy in prokaryotes, which question would you pose to
answer?
A Have metabolic pathways evolved separately in Bacteria and Archaea?
B Should all methanogens be classed as Archaea in evolutionary phylogeny?
C Have methanogens evolved to live in both moderate and extreme environments?
D Did the methanogenic bacteria species also evolve as a strict anaerobe?
Solution The solution is (A). Since Archaea and Bacteria separated evolutionarily many billions
of years ago, it makes sense to research whether metabolic pathways evolved twice,
once in each domain.
66 What is another question you might pose to learn more about the structural features that
allow for the capture, storage, and use of free energy by archaean methanogens?
A Do archaean methanogens differ from other Archaea structurally, and if so, in what
way? Is one or more of these structural differences related to these methanogens’
ability to use H2 to oxidize CO2?
B Do archaean methanogens differ from other Bacteria structurally, and if so, in what
way? Is one or more of these structural differences related to these methanogens’
ability to use CO2 to oxidize H2?
C Do archaean methanogens differ from other Archaea structurally, and if so, in what
way? Is one or more of these structural differences related to these methanogens’
ability to use CO2 to oxidize H2?
D Do archaean methanogens differ from other Archaea structurally, and if so, in what
way? Is one or more of these structural differences related to these methanogens’
ability to use H2O to oxidize H2?
Solution The solution is (C). Posing questions regarding the structural differences of Archean
methanogens will allow for further examination of these microbes on a molecular
level which can thus help to explain their functions and subsequent characteristics
resulting in their ability to CO2 to oxidize H2.
67 Which set of phrases related to nutritional and metabolic adaptations best fits the
organisms described?
A Chemoautotrophs, obligate anaerobes
B Chemoautotrophs, faculative anaerobes

********************************************************
*****************************************************************535***

C Chemoheterotrophs, faculative anaerobes


D Chemoheterotrophs, obligate anaerobes
Solution The solution is (A). Chemoautotrophs obtain energy from oxidation of electron
donors from their environment so they do not need oxygen. Oxygen presence will
interfere with their ability to obtain energy.
68 In an experiment, researchers grew plant seedlings in soils to which one of two strains
of bacteria were added. A control group had no bacteria added to the soil. The seedlings’
uptake of the nutrient potassium increased dramatically in the soil with Strain 1 and
decreased dramatically in the soil with Strain 2. What specific and broad inferences
about the relationship between the bacteria, the seedlings, and available nutrients can
you make?
A The Strain 2 bacteria increased the availability of potassium in the soil and this
nutrient was needed and used by the seedlings in the soil. The Strain 1 bacteria
decreased the availability of potassium in the soil.
B The soil with Strain 1 bacteria must have had more potassium in comparison to soil
with Strain 2 bacteria. The seedlings took up more potassium in Soil 1 than in 2 due
to this.
C The Strain 1 bacteria increased the availability of potassium in the soil and this
nutrient was needed and used by the seedlings in the soil. The Strain 2 bacteria
decreased the availability of potassium in the soil.
D The Strain 1 bacteria decreased the availability of potassium in the soil and this
nutrient was needed and used by the seedlings in the soil. The Strain 2 bacteria
increased the availability of potassium in the soil.
Solution The solution is (C). The Strain 1 bacteria increased the availability of the nutrient
potassium in the soil, and this nutrient was needed and used by the seedlings in the
soil. The Strain 2 bacteria decreased the availability of the nutrient potassium in the
soil. In general, prokaryotes can increase or decrease the availability of nutrients
that plants require for growth.
Use the following information for Questions 69–72:
In a hypothetical research situation, scientists discover bacterial endospores in silt at the
bottom of a marsh that has been contaminated with a pollutant for 25 years. The silt of the
marsh was deposited in annual layers. The age of the endospores can be estimated, therefore,
by identifying the layer of silt in which the endospores are found. In flask A, researchers place
20-year-old endospores along with growth medium and the pollutant. In flask B, researchers
place 100-year-old endospores along with growth medium and the pollutant
69 Which statement describes the results you would expect to see in the growth of the
flasks?
A The growth in flask A will exceed that of flask B.

********************************************************
**************************536*****************************************

B The growth in flask B will exceed that of flask A.


C The growth each flask will be about equal.
D There will be little to no growth in each flask.
Solution The solution is (A). Flask A has more recent bacteria in it than flask B, and the more
recent bacteria are more likely to be resistant to the pollutant and will likely grow
better than the older bacteria.

********************************************************
*****************************************************************535***

70 Why you would expect to see more growth in one particular flask than in the other?
A Because endospores formed 20 years ago would be more dormant compared to
endospores formed 100 years ago, before the marsh was polluted
B Because endospores formed 20 years ago would be less adapted to polluted
conditions compared to endospores formed 100 years ago, before the marsh was
polluted
C Because endospores formed 20 years ago would be more adapted to polluted
conditions compared to endospores formed 100 years ago, before the marsh was
polluted
D Because endospores formed 20 years ago would be less dormant compared to
endospores formed 100 years ago, before the marsh was polluted
Solution The solution is (C). Because prokaryotic populations evolve rapidly in response to
their environment, it is likely that bacteria from endospores that formed 20 years
ago would already be adapted to the polluted conditions. So, these bacteria would
likely grow better than bacteria from endospores formed 100 years ago before the
marsh was polluted.
71 Suppose the researchers observe the flasks for a while, continuing to replenish growth
medium and pollutant as necessary. Which statement describes the results you would
expect to see in the growth of the flasks after some time?
A The growth in flask A will continue to exceed that of flask B.
B The growth in flask B will exceed continue to exceed that of flask A.
C Eventually, the difference in the growth in each flask will lessen.
D Eventually, will be little to no growth in each flask.
Solution The solution is (C). It is likely that the bacteria in flask B will evolve resistance to the
pollutant and grow prolifically. Therefore, the difference in population size between
the flasks would lessen over time.
72 Suppose the researchers observe the flasks for a while, continuing to replenish growth
medium and pollutant as necessary. If the difference in the growth in each flask lessened
after some time, which statement explains why?
A Because the endospores formed 100 years ago, before the marsh was polluted, they
would evolve resistance to the pollutant fairly quickly. The bacteria in flask B would
then grow more prolifically, and the difference in population size of each flask would
lessen.
B Because the endospores formed 20 years ago would lose their resistance to the
pollutant. The bacteria in flask A would die, and the difference in population size of
each flask would lessen.

********************************************************
**************************536*****************************************

C Because the endospores formed 100 years ago, before the marsh was polluted, they
would lose their resistance to the pollutant. The bacteria in flask B would then grow
more prolifically, and the difference in population size of each flask would lessen.
D Because the endospores formed 20 years ago would evolve resistance to the pollutant
fairly quickly. The bacteria in flask A would die, and the difference in population size of
each flask would lessen.
Solution The solution is (A). Prokaryotic populations evolve rapidly in response to their
environment. So, endospores that formed 100 years ago before the marsh was
polluted would evolve resistance to the pollutant fairly quickly.
73 How does resistance spread in bacteria?
A By undergoing genetic recombination through conjugation, transduction, and
transformation
B By undergoing reproduction through binary fission
C By undergoing genetic recombination through conjugation and transformation only
D Reproduction among bacteria through any mechanism results in the spread of
antibiotic resistance genes.
Solution The solution is (A). Bacteria reproduce quickly through binary fission, in which the
daughter cells are identical to the parent. By itself, binary fission would not be
associated with the spread of antibiotic resistance. However, bacteria can undergo
genetic recombination through conjugation, transduction, and transformation.
These mechanisms of recombination allow antibiotic resistance to spread rapidly.
74 Which best explains this result?
A The wrong course of antibiotics was used on the patient, so the infection was never
treated.
B Not all of the bacteria were killed, and the remaining ones reproduced and bring back
the symptoms of infection.
C The antibiotics were not prescribed for a long enough time to treat the infection.
D The infection was actually viral in nature, and so the antibiotics were a useless
treatment
Solution The solution is (B). f the patients stopped treatment once symptoms abated, but
bacteria were still present, these most resistant bacteria would reproduce and bring
back infection
75 Human intestines are home to hundreds of species of bacteria. One of these, Bacteriodes
thetaiotaomicron, has the capability of digesting complex plant materials that human
enzymes cannot digest. Its presence in the human guts makes a significant contribution to
human metabolic processes.
Which term best describes the relationship between humans and B. thetaiotaomicron?

********************************************************
*****************************************************************535***

A Commensalistic
B Mutualistic
C Parasitic
D Pathogenic
Solution The solution is (B). In a mutualistic relationship, both members of the microbial/host
relationship benefit. In the example stated in this question, Bacteriodes
thetaiotaomicron derives benefits from living as a part of the normal gut flora of the
human intestine while the human host, derives benefit from the bacterium due to
its ability to metabolize complex plant materials. Neither member in the relationship
is harmed as a result of their association with each other under normal
circumstances.
76 If you suddenly and dramatically changed your diet, how might this affect the diversity of
prokaryotic species that live in your intestine?
A The diversity would not get altered and would remain the same.
B Species abundance and relative distribution may increase.
C Species abundance and relative distribution may get affected.
D Species abundance and relative distribution may decrease.
Solution The solution is (C). Different prokaryotic species rely on different energy sources and
metabolic pathways to survive. A change in diet might affect which species grow and
survive, which would ultimately affect species abundance and relative distribution.
This change might also affect your health and well-being—some prokaryotic
populations might assist your digestive processes, while others might not. So you
could feel very good or very bad, accordingly.

77 More than 100 bacterial species live on the surface of the human body. Bacteria
cover portions of human skin in concentrations of up to 8 million cells per square
centimeter. In particular, human sebaceous glands support the growth of the bacterium
Propionibacterium acnes, which uses oil from the glands for food. Two strains of P. acnes
are associated with the development of acne on human skin, but other strains are
associated with healthy skin.
Which statement best describes the relationship between humans and P. acnes?
A In some cases, it is mutualistic and in others it is parasitic
B In some cases, it is mutualistic and in others it is commensalistic.
C It is almost always parasitic.
D It is almost always mutualistic.

********************************************************
**************************536*****************************************

Solution The solution is (A). In some cases, it is mutualistic and in others it is parasitic.
Propionibacterium acnes is associated with the development of acne on human skin,
thus suggesting a parasitic relationship in which some harm is incurred by one of the
members of the relationship, which in this case would be the human host. However,
the fact that other strains of this bacterium is associated with healthy skin, suggests
that there is a mutualistic relationship, due to the fact that humans derive benefit
from this association via having healthier skin. Likewise, the bacterium derives
benefit from having oil produced from human sebaceous glands that supports
their growth.

SCIENCE PRACTICE CHALLENGE QUESTIONS


22.1 Prokaryotic Diversity
78 That the uniformity of cell size in prokaryotes is independent of the conditions of cell
growth has long been a puzzle. Suppose that cells grew for a random period of time and
then divided. The largest and smallest, by sometimes dividing to make even larger or
smaller cells, would be expected to broaden the distribution of cell sizes, as shown in
the diagram below for a time, t2, after a time t1. Competing claims are made to explain
the fact, however, that the distribution does not broaden: (1) There is a “timer” that
initiates cell division; and (2) There is a volume threshold that, when reached, initiates
cell division. Recently (Amir, Phys. Rev. Lett, 2014), a third model was suggested: From
the end of the last cell division to the next, the cell volume increases by a constant
value.

A. Justify the claim of the third model by (1) Rejecting the two alternative claims, using
the fact that growth rate depends on the availability of resources and considering that
regulation of expression at a critical volume would require measurement of total volume
by the cell; and (2) Arguing that adding a constant volume before each cell division would
narrow the cell size distribution.
B. Design a plan to test both the most recent model and the timer model.
Solution Sample answer:

********************************************************
*****************************************************************535***

A. Since growth rate can depend on the environment, a fixed time between cell divisions
would lead to larger cells when resources allow. Since signaling in a cell is molecular and
local and there is no sensory system that has been described that can acquire data on the
non-local state of the cell volume, this explanation lacks congruence with other successful
explanations. However, by adding a constant volume at each cell division, any deviation in
initial cell size would make an increasingly smaller contribution to the overall variation,
decreasing as one over the number of cell divisions.
B. A reasonable response would identify the parameter to be measured as change in
volume as a function of time. Microscopic observations could be recorded digitally and
later analyzed to determine total volume change. This would provide the measurement
needed. Also, this technique could be applied to systems with varying rates of growth to
support rejection of the timer model. Amir and his group have performed these
experiments to test both models.
79 Gram-negative bacteria have an inner cytoplasmic membrane separated by a
peptidoglycan layer from a second outer membrane. In addition, transport proteins called
efflux pumps span this double membrane and actively eliminate chemicals such as
antibiotics that pass through porins on the outer membrane. These efflux pumps can
confer multi-drug resistance, a situation that is threatening human health.
A. Explain how combining a drug that disrupts ATP synthesis in bacteria with antibiotics is
a possible strategy for the treatment of bacterial infections caused by antibiotic-resistant
gram-negative bacteria. ATP synthesis in prokaryotes is accomplished by a protein that
connects the extracellular space to the cytoplasm. In gram negative bacteria, the proton
gradient that supplies the free energy to convert ADP into ATP is established across the
inner membrane.
B. Predict differences in the interactions of eukaryotic and prokaryotic cells with a drug
molecule that successfully targets ATP synthesis and provide reasoning for your
predictions. In gram-positive bacteria, ATP synthesis is accomplished by a protein that
spans the single membrane and the outer cell wall. During the production of yogurt and
wine, which rely on gram-positive bacteria, the pH is controlled. Sodium bicarbonate
secretions from the pancreas maintain the pH of the human intestine, where many
beneficial methanogens are gram-positive bacteria.
C. Explain why homeostasis for gram-positive bacteria requires control of extracellular
pH.
Solution Sample answer:
A. If the efflux pump is disabled, and then antibiotics that pass through the outer
membrane are not expelled by the pump. This is one of the reasons why
combinations of drugs may be used in an attempt to deal with the horizontal
distribution of code for efflux pumps. This is one of the causes of antibiotic
resistance.

********************************************************
**************************536*****************************************

B. ATP synthesis in the eukaryotic cell occurs in mitochondria. If the drug cannot
access the interior of mitochondria, then it will have no effect.
C. The extracellular space determines the proton concentration at one end of the
gradient. As pH increases, the difference between the intercellular and extracellular
proton concentration becomes smaller and the free energy available for ATP
synthesis is reduced.
80 Cyanobacteria are single-celled organisms with the capacity to fix nitrogen, N 2. Some
cyanobacteria cooperatively aggregate as filaments, and heterocysts may form at intervals
along the filament between a pair of vegetative (actively growing) cells. Heterocysts are
specialized cells that express certain genes when nitrogen becomes limiting. The
nitrogenase complex converts the nitrogen in N2 into NH3 (ammonia). This enzyme
functions only under anaerobic conditions that are, in part, enforced by an O 2 barrier
surrounding the cytoplasm of the heterocyst, as shown in the figure.

A. Other modifications displayed in the diagram maintain an anaerobic state and


synthesize ammonia from N2. Identify four modifications of vegetative cells, either by their
addition to or omission from the heterocyst. Refine the representation by drawing a line
between each of the three numbered circles and the feature.
B. Further refine the representation by providing a brief description of the role of each
modification in either regulating oxygen or synthesizing ammonia.
The Krebs cycle in prokaryotes and eukaryotes differs. In prokaryotes, the Krebs cycle
occurs in the cytoplasm and the intermediate 2-oxoglutarate ( ketoglutarate) is absent.
C. Construct a representation of the regulation of genes encoding the nitrogen fixation
system using the elements below. The irregular shapes are either metabolites or
transcription factors, NtcA, HetR, and PatS. In your representation, label each shape using
the names on the left in the figure below. Your representation must account for these
observations:
 When nitrogen is limiting, 2-oxoglutarate concentration in the cytoplasm
increases.

********************************************************
*****************************************************************535***

 HetR is transcribed when 2-oxoglutarate concentrations are low.


 PatS is transcribed when 2-oxoglutarate concentrations are low.
 Nitrogenase is transcribed when HetR concentrations are high and PatS
concentrations are low.
 When PatS concentrations are high, nifX genes are not transcribed.

D. Heterocysts form along the filament separated by a fixed number of vegetative cells.
Based on your model of the regulation of heterocyst development, make and support a
claim that accounts for this pattern.
Solution Sample answer:
A. Lines are drawn to identify the systems described in (B).
B. Further refine the representation by providing a brief description of the role of
each modification in either regulating oxygen or synthesizing ammonia.
1. Glycolysis is the only respiration process. Since oxygen is unavailable, the
Krebs cycle is switched off.
2. Only Photosystem I is functioning because oxygen is produced as a product
of Photosystem II and that would poison the nitrogenase system.
3. Hydrogenase is expressed to produce hydrogen for the formation of
ammonia.
4. Nitrogenase is expressed to produce nitrogen for the formation of ammonia.
C.

D. The transcription factor PatS diffuses from the heterocyst, through adjacent
vegetative cells. As distance from the heterocyst increases, the concentration of PatS
decreases. When the concentration is sufficiently low, PatS no longer blocks HetR
and the nitrogenase system is expressed.
81 Escherichia coli Strain A is able to grow in a minimal medium only when supplemented
with methionine and biotin. Strain B is able to grow in a minimal medium only when

********************************************************
**************************536*****************************************

supplemented with threonine, leucine, and thiamine. The two strains are incubated
together in a medium containing each supplement. They are then transferred to a
minimal medium with no supplements, and each strain is able to grow under these
conditions.
A. Describe the evidence that supports information exchange between Strain A and
Strain B, and the mechanisms that can account for this behavior are demonstrated by
Lederberg and Tatum (Nature, 1946).
Colistin is regarded as a last-resort antibiotic in the treatment of multi-drug-resistant,
gram-negative bacteria. The MCR-1 gene that confers colistin resistance was recently
detected in a plasmid found in E. coli from the intestines of human patients (Liu et al.,
Lancet Infect. Dis., 2016). Colistin is cheap to produce; often is used as a feed supplement
for domesticated animals (12,000 metric tons per year in 2015); and its use is increasing.
Colistin is also unstable in water (Healan et al., Antimicrob. Agents Chemother., 2012).
B. Describe the possible biological consequences of an immediate ban on the use of
colistin in agriculture.
Solution Sample answer:
A. Transformation (uptake of naked DNA), conjugation (exchange through direct
contact), and transduction (exchange by bacteriophage). The identification of these
three processes is within the scope of the Framework. Further details are out of
scope. Evidence of the exchange of information is provided because progeny are
able to synthesize each of the molecules after incubation. This is regarded as the
work leading to the discovery of conjugation.
B. Unlike environmental contaminants like CFCs, DDT, and PCBs that when input to
the environment is banned, the biological damage from the use of colistin is in the
spread of the MCR-1 gene that replicates rather than degrades. There would be no
biological consequences though there may be economic consequences.
82 Life on Earth is sustained by four processes that are unique to prokaryotes:
(1) Methanogens reduce hydrogen or carbon atoms to produce methane;
(2) Methanotrophs combine methane with oxygen to form formaldehyde;
(3) Nitrogen fixation converts N2 into ammonia; and (4) Nitrification converts ammonia
into nitrates. These processes recycle matter, maintaining the carbon (1 and 2) and
nitrogen (3 and 4) cycles.
Methanogens are strictly anaerobic. Estimates of global fluxes of methane from major
sources (Kirschke, Nature Geoscience, 2013, in units of 1012 g C/year) are shown in the
figure. Agricultural sources are predominately the microbiomes of ruminants (cows, goats,
etc.) and rice cultivated in shallow ponds where anoxic compost and crop residues
promote methanogen growth on roots. Other major human activities that contribute to
atmospheric methane levels are landfills and natural gas drilling.

********************************************************
*****************************************************************535***

The fate of this methane also is shown. Most reacts with OH in the lower atmosphere to
make formic acid, which then decomposes into carbon dioxide and water. Methanotrophs
consume the remaining methane.
Methane is a component of the carbon cycle, but it is much less significant than carbon
dioxide, whose major fluxes are shown in units of 1015 g C/year (NASA, 2015). Oceanic
uptake and loss of CO2 are primarily abiotic. Prokaryotic marine organisms account for
approximately 50 percent of the biotic exchanges.

A. Compare quantitatively the rates of carbon cycling as methane between the biosphere
and atmosphere. Calculate the percentage of methane production that is anthropocentric
(due to human actions).
B. Assuming that the rates of carbon dioxide exchange shown in the diagram are accurate,
analyze these data to identify a missing contribution to the carbon budget.
Recently, it was discovered that ruminants fed nitrooxypropanoic acid reduced their
methane release from digestion by approximately 50 percent and increased the rate of
meat production by as much as 80 percent (E. Duin et al., Proc. Natl. Acad. Sci., 2016).
C. Since methane is a greenhouse gas, its release into the atmosphere further increases
global temperatures. It has been claimed that a feed supplement program will reduce the
effects of climate change. Predict the consequences of such a program and provide
reasoning for your prediction.
A vertical profile of methane and oxygen below the surface of a rice paddy are shown in
the graph below (Lee et al., Front. Microbiol., 25, 2015). Also shown are estimates of the
relative abundance of all genera of methanotrophs (red line) and methanogens (blue line)

********************************************************
**************************536*****************************************

as a function of depth. Rice paddies are the largest contributor to agricultural methane
production. The estimates were based on extraction and analysis of ribosomal RNA from
the soil.

D. Justify the selection of these measurements of the concentrations of two types of


microbes and the gases that are consumed or produced to the development of a
quantitative understanding of the habitat range of both groups and the control of
methane release from rice fields.
Solution Sample answer:
A. All CH4 produced is consumed and 60 percent is anthropogenic.
B. There are 2 Pg C/year missing. This missing sink has been much researched and
the consensus is that terrestrial soils and deep storage below deserts are the missing
sinks (Li et al., Geophysical Research Letters, 42, 2015). A model has suggested that
soil is near capacity (He et al., Science, 353, 2016).
C. Increased biomass in the ruminant population is an increase in fixed carbon that
will be consumed by humans and so converted to carbon dioxide. Since methane
predominantly is converted to carbon dioxide through reactions with OH in the
troposphere, the feed supplement will have essentially no consequence—methane
is just a pass through to carbon dioxide. Another effect might be more significant.
The OH radicals that react with methane would otherwise be involved in maintaining
the ozone layer.
D. By measuring oxygen and carbon dioxide profiles the scientists obtain a measure
of the range of depths that provide habitats for these microorganisms. By
sequencing rRNA, they obtain an understanding of the distribution of microbial
species. Methanogens consume CO2 and produce methane. Methanotrophs
consume methane and produce CO2. By comparing the two abundance curves they
obtain a measure of the interaction.

********************************************************
*****************************************************************535***

22.5 Beneficial Prokaryotes


83 The human gut provides a habitat for approximately 100 trillion bacteria. Some sources
claim that the surface area of the cells lining the small and large intestines is between
150 and 300 m2 and compare this area to that of a tennis court. Recent measurements,
however, show that the surface area of the gut is closer to that of a studio apartment
(Helander and Fandriks, Jour. Gastro., 2014) and is roughly 50 m2.
A. Calculate the cellular surface area of the 100 trillion (1014) microbes in the typical
human gut, assuming that the cells are spherical with an average radius of 0.001 mm. Use
this calculated surface area to predict the relative rates of procurement of nutrients by
both microbes and the host cells lining the large and small intestines.
Humans compete with microbes for nutrients, but the relationship is mutually beneficial.
Between 10 and 30 percent of ingested food remains undigested before reaching the
large intestine. Some microbial waste products, particularly H2 and CH4, are not resources
for the host. But short-chain fatty acids like acetic, propionic, and butyric acids are
resources that microbes extract from the undigested fraction. The large intestine of the
adult human has a length of approximately 1.5 m with a volume between 6 and 7 L. The
total volume of gut microbes is just a few hundred milliliters.
B. Predict the length of a large intestine with equivalent recovery of resources and the
same transit times through the bowel if, rather than 100 trillion organisms with a total
volume of 1 L, there were 100 billion (1011) organisms, each with a volume of 10–8 mL (the
approximate volume of the epithelial cells lining the intestine).
The relationship between gut microbes and their host is more complex than simple
resource recovery, as shown in the figure of the microbiome below. PYY is a hormone that
works with the enteric nervous system lining the intestinal wall to cause changes in the
period of contractions of muscles (motility) that push material through the intestine.
C. Based on the diagram, summarize the regulation of appetite by the microbiome and
the elimination of waste by the host in terms of feedback loops and chemical signaling.

********************************************************
**************************536*****************************************

The microbial population of the intestine is referred to as the microbiome. Undernutrition


and obesity are both symptoms malnutrition, and populations of the microbiome vary
with the type of malnutrition (Brown et al., Nutr. Clin. Pract., 2012). The microbiome of
humans can be transplanted into germ-free (GF) mice to observe the effects of diet in a
controlled experiment of relatively short duration. The microbiomes of healthy and
undernourished six-month-old children were transplanted into GF mice whose growth is
graphed below. Growth in both length and weight were reduced when the source of the
microbiome was the undernourished child (after Blanton et al., Science, 2016). Both
groups of mice were provided with the same nutritional resources.

D. Pose two scientific questions that, when investigated, could lead to a solution for the
stunting of growth caused by undernourishment in early infancy that affects millions of
children. Human growth hormone stimulates the release of insulin-like growth factor 1
(IGF-1). IGF-1 is a messenger that activates the production of bone cells called osteocytes.
The data (after Schwarzer et al., Science, 2016) show concentrations of this growth factor
in mice with no microbiome (GF), wild-type mice whose microbiome and growth provide a
control (WT), and mice whose microbiome population is composed entirely of
Lactobacillus plantarum (two strains labeled L1 and L2). Lactobacillus is one of many
hundred genera of microbial inhabitants of a healthy human intestine.

********************************************************
*****************************************************************535***

E. Analyze these data in terms of the potential for disruption of human bone growth due
to loss or reduction in diversity of the microbiome.
Solution Sample answer:
A. The surface area of so many microbes is very large even though they are each
very small.

Assuming uptake is proportional to surface area (in scope in the Framework,) the
relative rates of nutrient uptake is 25 times greater for the microbes than for the
cells lining the intestine.
B. Length is rate multiplied by time. So if the rate is proportional to the surface area
and surface area is proportional to the number of microbes, then reducing the
number by 103 increases the time by the same factor. If the transit time through the
bowel is fixed, then the length must increase by a factor of 1,000.
C. Appetite is suppressed by the release of PYY. PYY is expressed when transcription
is amplified by the intracellular signal initiated when a small chain fatty acid attaches
to the receptor. This feedback loop reduces input until resources have been cleared.
A second feedback loop occurs when the enteric nerve activated contraction is
slowed. This causes an increase in the transit time, increasing the recovery of
resources.

********************************************************
**************************536*****************************************

D. Can the microbiomes (species and number of species) of the undernourished and
healthy children be characterized? Can colonization by healthy microbiomes during
a period of starvation in infancy support recovery without stunting if nutrition is
provided and if so, for how long? Which species of gut microbes are most effective?
What is the mechanism of this effect?
E. Complete loss (GF) clearly shows diminished levels of IGF-1 through the entire
period of observation. So if bone growth is stimulated by IGF-1 then bone growth is
diminished. What is striking is that (1) L1 shows early IGF-1 levels in excess of the
control; and (2) A different strain of the same species shows much smaller effect.
The diversity of the microbiome is critical to bone growth and development.
Replacement of one strain of Lactobacillus by another diminishes bone growth
(assuming proportionality) by 50 percent.

********************************************************
*********************************************************561***

23 | PLANT FORM AND PHYSIOLOGY


REVIEW QUESTIONS
1 Students are sketching diagrams of the shoot system of angiosperms for a plant anatomy
class. These lists describe diagrams made by four students. Which diagram represents the
shoot system incorrectly?
A Leaves, stem, fruit, flowers
B Stem, fruit, leaves, branches
C Flowers, leaves, branches, stem
D Stem, hair roots, leaves, flowers, branches
Solution The solution is (D). While leaves, stem, fruit, flowers, and branches are all part of the
shoot system, hair roots are associated with roots and part of the root system even
when above ground.
2 An herbicide causes roots to shrivel and die. What is the most direct consequence for a
plant treated with the herbicide?
A The plant will grow normally but will not bloom.
B The plant will dry out because water is not reaching all its organs.
C New leaves will form to compensate for the dying of roots.
D The plant will grow normally but will not produce fruit.
Solution The solution is (B). The plant will dry out because water is not reaching all its organs.
3 Scientists label cells in the lateral meristem of a sapling with a dye to follow the
developmental fate of the cells. After several weeks, sections are prepared from the
sapling and observed under the microscope.
Which tissues are most likely to be stained by the dye that was injected into the lateral
meristem?
A Vascular tissue to transport nutrients and water
B The tip of plant to promote growth of plant
C Secondary xylem to increase girth of stem
D Epidermis to cover the plant
Solution The solution is (C). The lateral meristem produces secondary xylem that increases
the girth of the stem, so the dye would appear in secondary xylem.

********************************************************
**************************562**********************************

4 A lab technician is looking for a slide that shows an example of permanent tissue. Which
slide is the best choice?
A A slide of the apical bud of a stem
B A slide obtained from the intercalary meristems
C Lateral meristem in the vascular cambium
D Secondary xylem
Solution The solution is (D). Secondary xylem is permanent because the cells have die after
reaching maturity, leaving the tissue permanently in place.
5 Which region of a plant is most likely to contribute to an increase in its length?
A Tip of leaves
B Dermal layer
C Vascular bundles
D Tip of the root
Solution The solution is (D). The root elongates at the tip.
6 You are measuring the effect of a new fertilizer on the growth of lawns. Which tissue
should be the target of the fertilizer?
A Apical meristem
B Lateral meristem
C Intercalary meristem
D Vascular bundle
Solution The solution is (C). The intercalary meristem is typically the zone in grasses where
elongation of the shoot and leaves takes place.
7 What does the dermal tissue of a plant provide for the plant?
A Transport of water
B Transport of minerals
C Support
D Protection
Solution The solution is (D). Dermal tissue is the outer layer, which provides protection for
the plant.

********************************************************
*********************************************************561***

8 A branch of celery is soaked in a glass of water containing food dye. Soon, the tough fibers
in celery branch are colored. What tissue do the tough fibers contain?
A Dermal tissue
B Xylem
C Phloem
D Ground tissue
Solution The solution is (B). Xylem transports water and solutes.
9 A plant biologist is examining sections of plant tissue under the microscope. The slides are
not labeled and the biologist is interested in simple tissues. Which slide is a sample of a
simple tissue?
A Cells dividing rapidly in a stem
B Root cambium showing different types of cells
C Parenchyma showing only one type of cell
D Leaf displaying the vascular bundle where diverse types of cells are involved in
transport
Solution The solution is (C). A tissue sample from parenchyma showing only one type of cell is
a sample of a simple tissue.
10 Students are asked to sort tissue slides into simple and complex tissues. How should they
recognize a complex tissue through the microscope?
A Complex tissue has a variety of cell types that fulfill different functions.
B Only complex tissue is observed in adult plants.
C Complex tissue appears only in lateral roots and branches.
D Complex tissues contain cells that are strikingly different in appearance but perform
the same function.
Solution The solution is (A). Complex tissue has a variety of cell types that fulfill different
functions. The cells have strikingly different appearances because they fulfill
different functions.
11 Students are sketching diagrams of the reproductive system of angiosperms for a plant
anatomy class. These lists describe diagrams made by four students. Which diagram
represents the reproductive system correctly?
A Hair roots, lateral roots, and taproot
B Stem, branches, and leaves
C Flowers and fruit
D Leaves, petioles, and branches

********************************************************
**************************562**********************************

Solution The solution is (C). Flowers and fruit are reproductive organs of plants. Flowers
produce pollen that pollinators can transport to other flowers so that sperm from
the pollen can fertilize the ovule. The seed develops within fruit, which aids in
dispersal.
12 Plant scientists are interested in isolating meristematic tissue for an experiment. They
sample several regions of a plant. Which sample is most likely to contain meristematic
tissue?
A The thin epidermis that covers an onion bulb
B A sample of fruit tissue
C A sample of actively dividing cells located at the tip of an onion root
D A region of the mesenchyme
Solution The solution is (C). A region of actively dividing cells located at the tip of an onion
root is most likely to contain meristematic tissue, which contributes to the
lengthening of the root.
13 Which sketch of a stem correctly labels the region to which leaves are attached?

A C

B D
Solution The solution is (C). The petiole is the structure that connects the leaf to the stem at
the node.
14 A student examines a plant part and concludes that it is part of a stem. The presence of
_____ fully justifies the student’s conclusion.
blank

A vascular tissue
B nodes and internodes
C epidermal layer
D stored carbohydrates
Solution The solution is (B). Nodes and internodes are part of the stem. Leaves grow from
nodes and internodes make up the space between nodes.

********************************************************
*********************************************************561***

15 A student reported vascular tissue while inspecting a cross-section of a plant stem under
the microscope. Which cells would allow the student to identify vascular tissue?
A Tracheids, vessel elements, sieve-tube cells, and companion cells
B Cells actively dividing at the apex of the stem
C Parenchyma cells at the center of the section
D Cells covered by a cuticle at the outside edge of the section
Solution The solution is (A). Vascular tissue contains xylem (which consists of tracheids, vessel
elements, and xylem parenchyma) and phloem (which consists of sieve-tube cells,
companion cells, phloem parenchyma, and phloem fibers).
16 While using a microscope to observe a stem section stained with a dye that binds lignin,
a student notices that some cells with thick cell walls and large hollow centers are
preferentially stained. He concludes that those cells belong to the —
A meristematic tissue
B vascular tissue
C ground tissue
D dermal tissue
Solution The solution is (B). Xylem contains lignin in cell walls, making them relatively thick.
The lignin reduces permeability to water and the large, hollow centers allow liquid to
pass through.
17 Scientists are cataloguing slides of plant cross-sections. They are interested in finding
examples of secondary growth. Which example contributes to secondary growth?
A Apical meristem, which contributes to increase in length
B Vascular cambium, which contributes to increase in thickness or girth
C Root region, which shows an increase in root hairs
D Stems, which show an increase in number of leaves
Solution The solution is (B). Vascular cambium contributes to increases of girth in a plant,
meaning that it produces secondary growth. Primary growth would result in an
increase in length.
18 Where is the vascular cambium located in an established woody plant?
A Between the primary xylem and the primary phloem
B Between the secondary xylem and the primary phloem
C Between the secondary xylem and the secondary phloem
D Between the primary xylem and the secondary phloem

********************************************************
**************************562**********************************

Solution The solution is (C). The vascular cambium divides, adding both to the secondary
phloem and secondary xylem. So the vascular cambium is located between the
secondary xylem and the secondary phloem.
19 Dendrochronology is the science of dating the age of a tree by counting the annual rings
in a tree trunk. If scientists are determining the age of a tree by dendrochronology, what
tissue are they looking at?
A Primary xylem
B Secondary xylem
C Primary phloem
D Vascular cambium
Solution The solution is (B). Secondary xylem is the tissue that contains rings. It is produced
during the year but differs according to the season.
20 While examining the stump of a recently cut tree, you count four thick rings alternating
with four rings that are much narrower and appear denser. From this observation, you
should conclude that the tree is —
A two years old, because each ring corresponds to a season
B three years old, because the first ring you observe is the primary xylem
C four years old, because secondary xylem grows only in the spring and fall of each year
D eight years old, because there are eight rings in all
Solution The solution is (C). The secondary xylem grows in the fall and spring of each year,
meaning that these rings correspond to changes in growth each fall and spring (with
one thick ring and one narrower ring per year).
21 Many forms of modified organs exist in plants. What is a rhizome?
A An underground stem with fleshy leaves modified for food storage as in onions
B A solid, underground stem covered with scales formed by some plants such as
crocuses
C An above ground stem with buds as seen in strawberry plants
D A modified horizontal stem that grows underground as seen in irises
Solution The solution is (D). A rhizome is a modified horizontal stem that grows underground.
22 Modified organs are part of survival strategies of plants. Which plant has a flattened,
photosynthetic stem that could be mistaken for a leaf?
A Fern
B Cactus

********************************************************
*********************************************************561***

C Potato
D Iris
Solution The solution is (B). A cactus has evolved to conserve water in a dry
environment. It can have leaves that are modified as spines, providing
protection and conserving water.
23 Analyzing cross-sections of different parts of a plant in a plant anatomy class, students
categorized the most frequently encountered types of cells in plant tissues. Which student
gave the most accurate report?
A Student A reported that meristematic cells were the most abundant.
B Student B tallied mostly collenchyma cells.
C Student C noticed mostly sclerenchyma cells.
D Student D observed that parenchyma cells were the most abundant.
Solution The solution is (D). Parenchyma cells are the most common tissue type. Student D
observed that parenchyma cells were the most abundant.
24 A carrot is an example of a taproot. Which option can also be classified as a taproot?
A The large network of superficial roots of a cactus
B A dandelion anchored by a long main root that penetrates deep into the soil
C A banyan tree’s system of roots that dangle from the branches
D A round organ that stores carbohydrates
Solution The solution is (B). A taproot is a long, main root that penetrates deep into the soil.
25 Some weeds are anchored by taproots. They cause problems to gardeners because
they are —
A easy to pull up because the root system is shallow
B difficult to pull up because their taproots penetrate deep into the soil
C difficult to pull up because they are anchored by an extensive network of roots
D easy to pull up because there is not a large network to anchor the plant
Solution The solution is (B). Taproots extend deep into the soil and provide a solid anchor
from an extensive network of roots. This makes them more difficult to extract from
the soil than a plant with a shallower root system.
26 One of the major concepts of biology is that form follows function. If that is so, what can
be deduced from the shape and location of the root cap?
A It provides protection to the root tip.
B It absorbs water and minerals.

********************************************************
**************************562**********************************

C It acts as a storage tissue.


D It replicates actively to elongate the root.
Solution The solution is (A). The root cap covers the tip of the root to provide protection.
27 A technician is preparing microscope slides that will display the different stages of mitosis
from root samples. He compares sections from several areas of the root. Which statement
is the best prediction of his observation?
A The technician will see mostly mitotic cells in the root cap.
B The technician will observe mitotic figures in the meristematic tissue below the cap.
C The technician will observe cell division in the elongation zone.
D The technician will see that most mitotic cells are in the maturation zone.
Solution The solution is (B). The technician will observe mitotic figures in the meristematic
tissue above the cap. The meristematic tissue in the root is located above the root
cap and contains mitotic cells.
28 Selective uptake of minerals in the root is measured and the results are analyzed. If you
analyze the data, what should you see?
A Pericycle is the tissue where selectivity takes place.
B The endodermis acts as a selective barrier for minerals taken up by the root.
C The epidermis acts as a selective barrier for minerals.
D The root cap functions as a selective barrier for minerals taken up by the root.
Solution The solution is (B). The endodermis is a selective barrier for minerals that are taken
up through the root because the Casparian strip forces water to pass through the
endoderm, where selectivity takes place.
29 Sudan Red dye stains primarily waxy, hydrophobic material. A root is soaked in Sudan Red
and analyzed for stain retention. What is a scientist observing sections of the root under a
microscope likely to see?
A The cells in the cortex show the deepest stain.
B The tracheids in the xylem contain mostly lipid droplets stained with Sudan Red.
C The Casparian strip will show the deepest coloring.
D The sieve elements in the phloem show staining with Sudan Red because of
transported oil droplets.
Solution The solution is (C). The Casparian strip will show the deepest coloring because it is
made of waxes, which are hydrophobic and would be stained by Sudan Red.

********************************************************
*********************************************************561***

30 In environments where light is scarce, some plants grow on other plants to reach light.
Which root system would best support this mode of life?
A Epiphytic root system in the air
B Prop roots that support the trees to stand in muddy soil
C Adventitious roots that grow above ground
D Taproots that penetrate the soil
Solution The solution is (A). Plants growing on other plants would function best with
epiphytic roots in the air (allowing them to be farther from the ground).
31 A section of buttercup root is stained with iodine, which stains starch blue. Where would
you expect to find the blue granules indicative of starch?
A Parenchymal cells of the cortex
B Cells of phloem
C Cells of the epidermis
D Cells of the endodermis and pericycle
Solution The solution is (A). Starch may be stored in parenchymal cells within the cortex.
32 Which of the following best describes a fibrous root system?
A Covers a limited surface and contains few roots
B Consists of a single main root with adjacent smaller roots
C Covers a large area and contains an extensive network of roots
D Contains several major, interconnected roots
Solution The solution is (C). A fibrous root system is relatively shallow, covers a large area,
and contains an extensive network of roots.
33 Ethylene promotes the fall of leaves by triggering the death of cells and abscission. What
region of the leaf responds to ethylene?
A the lamina, where photosynthesis takes place
B the vein, which carries nutrients and water in and out of the leaf
C the petiole, which attaches the leaf to the stem
D the margin, which is serrated and may be sharp
Solution The solution is (C). Petioles attach leaves to stems; ethylene must affect them to
cause leaves to fall.
34 A horticulture student is classifying plants as dicots or monocots according to their leaf
structure. How is a dicot leaf recognizable?
A It does not have stipules.

********************************************************
**************************562**********************************

B The veins form a network pattern.


C The veins are parallel.
D The veins form forks and fan out.
Solution The solution is (B). Dicot leaves have veins in networks, while monocot leaves have
parallel veins.
35 Multiple leaves attached to the same node are fairly unusual. One example is found on
the macadamia nut tree. The leaf arrangement in the macadamia tree is best
characterized as —
A whorled
B opposite
C tripled
D alternate
Solution The solution is (A). Macadamia nut tree leaves are attached to the same node,
making a whorled leaf arrangement.
36 You picked leaves while on a hike. One specimen appears to show an opposite
arrangement. On closer inspection, you notice that those are not leaves, but leaflets
attached to a midrib vein.
What type of leaf arrangement are you observing?
A Palmately compound
B Pinnately compound
C Simple whorled
D Simple spiral
Solution The solution is (B). Leaflets are attached to a midrib vein represent a pinnately
compound leaf.
37 Chlorophyll, the primary photosynthetic pigment, emits light in the red region of the
visible spectrum. The presence of chlorophyll correlates with photosynthetic capacity.
Under a fluorescent microscope, what part of a leaf would fluoresce in the red region of
the spectrum?
A Vascular bundle
B Epidermis

********************************************************
*********************************************************561***

C Mesophyll
D Cuticle
Solution The solution is (C). The mesophyll is the main site of photosynthesis.
38 A pulse of radioactive carbon dioxide is provided to isolated leaves. In which tissue would
you expect to see radioactive glucose appear first?
A In the cells of the mesophyll
B In the sieve elements of the phloem
C Epidermis
D Vessels of the xylem
Solution The solution is (A). Glucose, produced from G3P from the Calvin cycle or light
independent reactions, appears first in the cells which have chloroplasts. Sucrose,
not glucose, is the sugar that is transported in the plant.
39 Which adaptation is most likely to be found in a desert environment?
A Broad leaves to capture sunlight
B Spines instead of leaves
C Needle-like leaves
D Wide, flat leaves that can float
Solution The solution is (B). Plants that live in desert areas tend to have reduced or modified
leaves to conserve water. Some have spines that serve a defensive function while
also reducing water loss compared to larger leaves.
40 In the collection of a botanical garden, plants are classified according to the environments
in which they thrive. What plant would have large leaves covered with a thick upper
cuticle and wide flat blades, and possess large air spaces (chambers) within its mesophyll
tissue?
A A water lily floating on water
B A pine tree growing in the cold and dry taiga
C A cactus growing in a hot, sunny, and dry environment
D An orchid hanging from a tree in a tropical forest
Solution The solution is (A). A water lily would have waxy upper cuticle to repel water and
keep the stomata open, large air chambers/spaces and wide blades to help it float.
41 If a gardener trims leaves off of the stem of a rose, which part of the leaf is cut?
A Petiole
B Lamina

********************************************************
**************************562**********************************

C Stipule
D Midrib
Solution The solution is (A). The petiole attaches the leaf to the stem.
42 On a field trip, students collect a few samples to analyze back in their classroom. One
student picks a blade of grass in the field and identifies it as a dicot leaf, but his partner
thinks it is a monocot.
Which explanation supports his partner’s opinion?
A The leaf displays a thin lamina.
B There is no petiole.
C The margins are serrated.
D The venation is parallel.
Solution The solution is (D). Monocots have parallel veins while dicots have network patterns
of veins. If your lab partner identified the grass as a dicot but the leaves were
parallel, then that would be a good reason to disagree and to argue that the grass
was a monocot.
43 Which physical component of the total water potential cannot be manipulated by the
plant because it represents the interaction between water and hydrophilic molecules
lining the vessels and tracheids?
A Pressure
B Solute concentration
C Gravity
D Matric potential
Solution The solution is (D). The matric potential reaches zero when seeds take up water and
therefore cannot be manipulated by plants and is ignored in well-watered stems.
44 If the concentration of solute increases in a cell, the water potential will _____ inside the
blank

cell and water will move _____ the cell.


blank

A increase; out of
B increase; into
C decrease; into
D decrease; out of
Solution The solution is (C). Water potential becomes more negative as more solute
molecules bind water molecules. Osmotic pressure increases in the cell and water
moves into the cell.

********************************************************
*********************************************************561***

45 Plants can modify their water potential by opening and closing their stomata to modulate
the rate of respiration according to environmental conditions. Which environmental
condition would cause the stomata to close?
A Increased temperature
B High oxygen concentration
C High relative humidity
D High light levels
Solution The solution is (A). Warm temperatures accelerate the loss of water from the leaves.
When the stomata close, water loss is prevented, although photosynthesis is also
inhibited by a lack of carbon dioxide intake.
46 Plants regulate their internal water potential by opening and closing stomata. Which
events take place when stomata open?
A Water vapor is lost to the external environment, increasing the rate of transpiration.
B Water vapor is lost to the external environment, decreasing the rate of transpiration.
C Water vapor enters the spaces in the mesophyll, increasing the rate of transpiration.
D The rate of photosynthesis drops when stomata open.
Solution The solution is (A). When stomata open, water vapor is lost to the external
environment and transpiration rates increase.
47 A pulse of sugars labeled with a fluorescent dye is supplied to leaves of young plants.
After a brief interval, tissue sections are obtained from the plant and examined under the
fluorescence microscope. Tissues are scored for the presence of fluorescence and ranked
from very high to low fluorescence.
Which cells would contain the most fluorescence?
A Xylem
B Companion cells
C Sieve elements
D Epidermis
Solution The solution is (C). The most fluorescence would occur in sieve elements are they
are involved in sugar transport.
48 Sugars produced in the leaf are distributed throughout the plant body. An experimenter
supplies plants with a pulse of radiolabeled carbon dioxide in a control chamber. The
movement of radioactively labeled sugar is monitored in the plant by analyzing different
cells' content over time.
Where will the radiolabeled sugar appear immediately after detection in the leaf cells?

********************************************************
**************************562**********************************

A Tracheids and vessel elements


B Tracheids and companion cells
C Vessel elements and companion cells
D Sieve-tube elements and companion cells
Solution The solution is (D). The radiolabeled sugar would appear in sieve-tube elements and
companion cells because these are involved in sugar transport.
49 Solute potential decreases when solutes are added to a cell. The consequence is to draw
water into the cell. Which term corresponds to solute potential?
A Water potential
B Pressure potential
C Osmotic potential
D Negative potential
Solution The solution is (C). Osmotic potential corresponds to solute potential and influences
water movement in and out of the cell.
50 Plants have many light responses, including photosynthesis, photoperiodism, and
phototropism (growing toward a light source). Specific wavelengths of light absorbed
by different photoreceptors trigger responses. This table shows some of the most
common photoreceptors and pigments and the major regions of the spectrum in which
they are active.

Research shows that plants bend toward blue light. Even mutant plants that lack
carotenoids will bend toward blue light. The photoreceptor is likely —
A phytochrome
B chlorophyll
C phototropin
D carotenoids

********************************************************
*********************************************************561***

Solution The solution is (C). Phototropin and carotenoids are most active in blue light.
Because bending occurs even when carotenoids are absent, phototropin is
responsible.
51 Plant flowering is an example of photoperiodism, the response to the length of nights or
periods of darkness. A plant that responds to short nights followed by increasingly longer
nights will most likely flower in —
A spring
B summer
C autumn
D winter
Solution The solution is (C). During autumn, nights are increasingly longer. So this type of
plant is likely to flower in autumn.
52 Gravitropism is plant growth in response to gravity. A dahlia stem is toppled by the wind
and is lying on the ground. After a few days, you would likely notice that —
A the stem is growing by curving toward the roots
B the stem is growing by trailing on the ground
C the stem is growing by curving upward
D the plant is wilting
Solution The solution is (C). Because the stem is sensitive to gravity, it will grow upward, away
from the ground.
53 Plants most likely detect gravity by sensing the direction in which some components
respond to gravity. A mutant plant has roots that grow in all directions. Which organelle
would you expect to be missing in the cell?
A Mitochondria
B Amyloplast
C Chloroplast
D Nucleus
Solution The solution is (B). Specialized amyloplasts are involved in sensing gravity; they are
denser than cytoplasm.

********************************************************
**************************562**********************************

54 In an experiment to release seeds from dormancy, several hormones were applied to


seeds and germination rates were computed. The table lists the hormones.

Which plate likely showed the highest rate of germination?


A Abscisic acid
B Cytokinin
C Ethylene
D Gibberellic acid
Solution The solution is (D). Gibberellic acid can trigger germination.
55 Green bananas or unripe avocadoes can be kept in a brown bag to ripen faster. What
hormone is involved?
A Cytokinin
B Abscisic acid
C Ethylene
D Gibberellic acid
Solution The solution is (C). Ethylene is a gas that becomes trapped by the brown bag and
stimulates fruit ripening.
56 A lab teacher wants to demonstrate thigmonastic behavior of a plant. Which experiment
is the best choice?
A Observe flowering of a plant after a brief red light irradiation in the middle of a dark
period.
B Observe whether seedlings bend towards blue light.
C Observe whether a tree grows bent in the direction of the prevailing wind.
D Touch the plant Mimosa pudica and observe the closing of the leaflets.
Solution The solution is (D). Thigmonastic behavior occurs when plants respond to touch, so
touching a mimosa plant and observing the closing of the leaflets is the best
example.

********************************************************
*********************************************************561***

57 A lab teacher wants to demonstrate thigmotropic behavior of a plant. Which experiment


is the best choice?
A Roots growing downward
B A Venus flytrap snapping on an insect
C A seedling germinating under a stone and growing upward and away from the stone
D A plant growing toward a shaded area
Solution The solution is (C). Thigmotropism is a response to physical contact, so a seedling
that grows away from a stone is the best example.
58 What is a protection against microbial pathogens?
A Thorns and spines
B Cutin and suberin
C Neurotoxic compounds
D Bitter-tasting alkaloids
Solution The solution is (B). Cutin and suberin found on the surface serve as protection
against microbes, helping to form a barrier.
59 Many secondary alkaloids are poisonous to the nervous system. What organisms are
targeted by the alkaloids?
A Bacteria
B Herbivores
C Fungi
D Viruses
Solution The solution is (B). Herbivores are animals that eat plants, so they have nervous
systems that can be poisoned
60 Red light converts phytochrome red (Pr) to —
A an inactive form of Pr
B a breakdown product
C the far red light absorbing form called Pfr
D cryptochrome
Solution The solution is (C). Red light converts Pr to Pfr, which absorbs far red light.

********************************************************
**************************562**********************************

61 Circadian rhythm refers to a pattern of behavior that recurs on a daily schedule in the
absence of an external stimulus. Flowers open and close according to a circadian rhythm.
If a plant is transferred to a dark environment, what will happen?
A Flowers will stay closed.
B Flowers will stay open.
C Flowers will open and close every day at the same time.
D Flowers will open and close at random times.
Solution The solution is (C). Without cues from the environment, flowers will open and close
at approximately the same time every day.

CRITICAL THINKING QUESTIONS


62 Why are plants with shallow roots more easily damaged by some herbivores?
A Shallow roots do not anchor the plant to the ground and can be easily uprooted. Once
the plant is no longer in the ground, the roots are unable to grow back.
B Plants with shallow roots do not anchor the plant to the ground; meristems can be
easily damaged and cannot grow back when not in the ground.
C Shallow roots do not anchor the plant to the ground and can be easily uprooted. Once
the plant is no longer in the ground, roots take a long time to grow back.
D Shallow roots anchor the plant to the ground strongly but can be easily uprooted, and
they grow back very slowly.
Solution The solution is (A). Because herbivores can easily pull plants with shallow roots from
the ground compared to plants with deep roots, there is a greater risk of the plant
being unable to grow back. Once plants are completely pulled from the ground, the
roots are not in the soil to allow the plant to grow back.
63 A researcher intends to test the effects of several growth factors on the differentiation of
plant tissue. What would be the best choice of experimental tissue?
A Dermal tissue
B Meristematic tissue
C Vascular tissue
D Ground tissue
Solution The solution is (B). Meristematic tissue is the site of growth in plants.

********************************************************
*********************************************************561***

64 How do the locations and the functions of the three types of meristematic tissues
compare?
A Apical meristems found in the tip of stems and roots promote growth by elongation;
lateral meristems found at nodes and bases of leaf blades promote increase in length;
and intercalary meristems found in the vascular and cork cambia promote increase
in girth.
B Apical meristems found at nodes and bases of leaf blades promote growth by
elongation; lateral meristems found in the vascular and cork cambia promote increase
in girth; and intercalary meristems found in the tip of stems and roots promote
increase in length.
C Apical meristems found in the tip of stems and roots promote growth by elongation;
lateral meristems found in the vascular and cork cambia promote increase in girth;
and intercalary meristems found at nodes and bases of leaf blades promote increase
in length.
D Apical meristems found in the tip of stems and roots promote growth by elongation;
lateral meristems found in the vascular and cork cambia promote increase in length;
and intercalary meristems found at nodes and bases of leaf blades promote increase
in length.
Solution The solution is (C). Apical meristems found in the tip of stems and roots promote
growth; lateral meristems found in the vascular and cork cambia cause woody plants
to increase in girth and are seen in well-established plants.
65 In an experiment on transport in plants, seedlings are exposed to radiolabeled minerals.
In a second experiment, plants are provided with carbon dioxide that is labeled with 14C.
At the end of each experiment, tissue slices are analyzed for the presence of radiolabeled
minerals and radioactive sucrose.
Which plant tissue would show the presence of labeled minerals and which would show
the presence of radioactive sucrose?
A Phloem tissue would show the presence of labeled minerals, and xylem tissue would
show the presence of radioactive sucrose.
B Xylem tissue would show the presence of labeled minerals, and phloem tissue would
show the presence of radioactive sucrose.
C Parenchyma would show the presence of labeled minerals, and sclerenchyma would
show the presence of radioactive sucrose.
D Sclerenchyma would show the presence of labeled minerals, and parenchyma would
show the presence of radioactive sucrose.
Solution The solution is (B). Xylem tissue would show the presence of labeled minerals and
phloem tissue would show the presence of radioactive sucrose.

********************************************************
**************************562**********************************

66 How could the morphology of cells observed microscopically indicate that the specimen is
probably simple tissue?
A Simple tissue is made of cells that have different shapes, so the specimen will show
oval, polygonal, and other shapes.
B Simple tissue is made of cells that have intercellular spaces, so the specimen will
contain spaces.
C Simple tissue is made of cells that are elongated and tapered, so the specimen will
show elongated cells.
D Simple tissue is made of cells that are morphologically similar, so the specimen will
appear uniform.
Solution The solution is (D). Simple tissue is made of cells that are morphologically similar.
The tissue will appear uniform.
67 What are two advantages of a taproot?
A It anchors the plant, so that it is not easily uprooted by predators or wind. It is a sink
for proteins that is protected from herbivores by being underground.
B It anchors the plant, so that it is not easily uprooted by predators or wind. It is a
source of starches that is protected from herbivores by being underground.
C It anchors the plant, so that it cannot be uprooted by predators or wind. It is a sink for
starches that is protected from herbivores by being underground.
D It anchors the plant, so that it is not easily uprooted by predators or wind. It is a sink
for starches that is protected from herbivores by being underground.
Solution The solution is (D). It anchors the plant, so it is not easily uprooted by predators or
wind. It is a sink for starches that is protected from herbivores by being
underground.
68 Students observe several slides of tissue cross-sections under the microscope. They are
asked to develop a key system to classify the slides as coming from either monocot or
dicots. What key system should the students develop?
A In monocots, the vascular bundles form a distinct ring. In dicots, the vascular bundles
are scattered in the ground tissue.
B In monocots, the vascular tissue forms a characteristic X shape in the center. In dicots,
the phloem and xylem cells are scattered in the pith.
C In monocots, the vascular bundles are scattered in the ground tissue. In dicots the
vascular bundles form a distinct ring.
D In monocot roots, the pith is absent or very small. In dicots, the pith is large and well
developed.
Solution The solution is (C). In monocots, the vascular bundles are scattered in the ground
tissue. In dicots, the vascular bundles form a distinct ring.

********************************************************
*********************************************************561***

69 What are the functions of stomata and guard cells, and what would happen to a plant if
these cells did not function correctly?
A Guard cells allow carbon dioxide to enter and exit the plant. Stomata regulate the
opening and closing of guard cells. If the cells didn’t function, photosynthesis and
transpiration would cease, which would interfere with the necessary continuous flow
of water upward from roots to leaves.
B Stomata allow oxygen to enter and exit the plant. Guard cells regulate the opening
and closing of stomata. If the cells didn’t function, photosynthesis would continue but
transpiration would cease, which would interfere with the necessary continuous flow
of water upward from roots to leaves.
C Guard cells allow carbon dioxide to enter and exit the plant. Stomata regulate the
opening and closing of guard cells. Transpiration and in turn, photosynthesis, would
not occur, which is necessary to maintain a continuous flow of water upwards from
the roots to the leaves.
D Stomata allow gases to enter and exit the plant. Guard cells regulate the opening and
closing of stomata. Photosynthesis and, in turn, transpiration, would not occur, which
is necessary to maintain a continuous flow of water upwards from the roots to the
leaves.
Solution The solution is (D). Stomata allow gases to enter and exit the plant. Guard cells
regulate the opening and closing of stomata. If these cells did not function correctly,
a plant could not get the carbon dioxide needed for photosynthesis, nor could it
release the oxygen produced by photosynthesis or perform transpiration, necessary
to maintain a continuous flow of water upwards from the roots to the leaves.
70 An herbicide is developed that impairs the function of the cork cambium in woody plants.
Which changes in the plant should be monitored to gauge the effectiveness of the
herbicide?
A Cork will not be produced and the plant will not increase in girth.
B Excess cork will be produced and annual rings will not be formed.
C Cork will not be produced and the plant will not be able to exchange gases.
D Excess cork will be produced and the plant will not increase in girth.
Solution The solution is (A). Cork will not be produced and the plant will not increase in girth.
In woody plants, the cork cambium is the outermost lateral meristem; it produces
new cells towards the interior, which enables the plant to increase in girth. The cork
cambium also produces cork cells towards the exterior, which protect the plant from
physical damage while reducing water loss. If the herbicide does affect the cork
cambium, cork will not be produced and the plant will not increase in girth.

********************************************************
**************************562**********************************

71 Besides the age of a tree, what additional information can annual rings reveal?
A Annual rings can also indicate the height of the tree.
B Annual rings can also indicate the climatic conditions that prevailed during each
growing season.
C Annual rings can also indicate in which season the tree was sown.
D Annual rings can also give an estimation of how long a particular tree is going to live.
Solution The solution is (B). Annual rings can also indicate the climatic conditions that
prevailed during each growing season.
72 Modified stems give an advantage to plants. What advantage do rhizomes, stolons, and
runners provide? What advantages do corms, tubers, and bulbs provide?
A Rhizomes, stolons, and runners give rise to new plants that are the clones of the
parents and they store food. Corms, tubers, and bulbs can also produce new plants.
B Rhizomes, stolons, and runners give rise to new plants that are the different from the
parents. Corms, tubers, and bulbs can also produce new plants as well as store food.
C Rhizomes, stolons, and runners give rise to new plants that are the clones of the
parents. Corms, tubers, and bulbs can also produce new plants as well as store food.
D Rhizomes, stolons, and runners give rise to new plants that are similar to the parents
but show genetic variability. Corms, tubers, and bulbs can also produce new plants as
well as store food.
Solution The solution is (C). Rhizomes, stolons, and runners can give rise to new plants. These
new plants are clones and have the same genetic advantages of the parent plant.
Furthermore, these new plants are formed without dependence on the hazards
linked to sexual reproduction (pollen dissemination, fertilization). On the other
hand, there is no genetic variability. Corms, tubers, and bulbs can also produce new
plants as well as store food. Corms, tubers, and bulbs are underground structures
where they are protected from most herbivores and drastic changes in
environmental conditions. Moisture and temperatures are less variable in the soil
than in the air.
73 A time course is developed to follow the fate of the vascular bundles in the stem of dicots.
Sections along the stem are fixed, stained, and observed under a microscope. What
happens to the vascular bundles in the stem of a dicot as the plant matures?
A The vascular bundles join to form growth rings.
B The vascular bundles divide into primary xylem and primary phloem.
C The vascular bundles divide into secondary xylem and primary phloem.
D The vascular bundles die out.
Solution The solution is (A). The vascular bundles join to form growth rings.

********************************************************
*********************************************************561***

74 Which description correctly compares a taproot system with a fibrous root system?
A A taproot system, such as that of carrots, has a single main root that grows down. A
fibrous root system, such as that of wheat, forms a dense network of roots that is
closer to the soil surface. Fibrous root systems are found in monocots, and taproot
systems are found in dicots.
B A fibrous root system, such as that of a carrot, has a single main root that grows down.
A taproot system, such as that of wheat, forms a dense network of roots that is closer
to the soil surface. Fibrous root systems are found in monocots, and taproot systems
are found in dicots.
C A taproot system, such as that of rice, has a single main root that grows down. A
fibrous root system, such as that of a carrot, forms a dense network of roots that is
closer to the soil surface. Fibrous root systems are found in monocots, and taproot
systems are found in dicots.
D A taproot system, such as that of a carrot, has a single main root that grows down. A
fibrous root system, such as that of wheat, forms a dense network of roots that is
closer to the soil surface. Taproot systems are found in monocots, and fibrous root
systems are found in dicots.
Solution The solution is (A). A taproot system has a single main root that grows down. A
fibrous root system forms a dense network of roots that is closer to the soil surface.
An example of a taproot system is a carrot. Grasses such as wheat, rice, and corn are
examples of fibrous root systems. Fibrous root systems are found in monocots;
taproot systems are found in dicots.
75 What is the advantage of a root cap covering the apical meristem of a root?
A It provides protection and helps in absorption.
B It increases the surface area of root for absorption of water and minerals.
C It protects meristem against injury and provides lubrication for the growing root to dig
through soil.
D It protects the meristem against injury and helps in absorption.
Solution The solution is (C). The cap protects the actively dividing region against injury and
provides lubrication for the growing root to dig through soil.
76 How does selective uptake of water and minerals take place in a root?
A Water and minerals must follow entirely a path between cells, where selectivity
occurs.
B Water and minerals must follow entirely a path between cells, where no selectivity
occurs.
C Water and minerals must cross the endodermis.
D Water and minerals must cross the tracheids of the xylem.

********************************************************
**************************562**********************************

Solution The solution is (C). Water and minerals must cross the endodermis rather than
follow entirely a path between cells where no selectivity occurs.
77 What are the advantages to a plant of storing a food reserve underground?
A Food reserves are more nutritious underground. The soil conditions make these food
reserves abundant.
B Food reserves underground are hidden from potential predators. The soil conditions
make these food reserves abundant.
C Food reserves are more nutritious underground. The soil conditions such as moisture
and temperature are less variable.
D Food reserves underground are hidden from potential predators. Soil conditions such
as moisture and temperature are less variable.
Solution The solution is (D). Food reserves underground are hidden from potential predators.
Soil conditions of moisture and temperature are less variable.
78 Some desert plants have taproots that extend up to 20–30 ft underground. Others have
fibrous root systems that cover wide areas. What are the advantages of a deep taproot
and the advantages of a fibrous root system in a desert?
A A deep taproot can reach the deeper soil regions that stay moist after several rainfalls.
A shallow fibrous system provides additional support to anchor the plant in the desert.
B A deep taproot provides additional support to anchor the plant in the desert. A
shallow fibrous system increases the amount of water that can be absorbed after a
light rainfall when the soil dries quickly in the desert.
C A deep taproot increases the amount of water that can be absorbed after a light
rainfall when the soil dries quickly in the desert. A shallow fibrous system can reach
the deeper soil regions that stay moist after several rainfalls.
D A deep taproot can reach the deeper soil regions that stay moist after several rainfalls.
A shallow fibrous system increases the amount of water that can be absorbed after a
light rainfall when the soil dries quickly in the desert.
Solution The solution is (D). A taproot extends deep enough to tap underground water. It has
the advantage of reaching the deeper soil regions that stay moist after several
rainfalls. A shallow fibrous system increases the amount of water that can be
absorbed after a light rainfall when the soil dries quickly in the desert.
79 Samples of leaves from monocots and dicots are piled on the table in a laboratory and
students are sorting the leaves. What information will help them know which leaves to
identify as monocots?
A Cork cambium cells are usually absent from monocots, whereas they are present on
the upper epidermis of dicot leaves.
B Monocots have leaves with parallel venation, and dicot leaves have reticulate, net-like
venation.

********************************************************
*********************************************************561***

C Bilateral symmetry is observed in monocot leaves, whereas isobilateral symmetry is


observed in dicot leaves.
D Monocots have leaves with reticulate, net-like venation, and dicot leaves have parallel
venation.
Solution The solution is (B). Monocots have leaves with parallel venation. They look different
from the leaves of dicots, which have reticulate, net-like venation.
80 How does a compound leaf give a selective advantage to avoid herbivory?
A Compound leaves produce certain types of chemical compounds that are harmful to
herbivores.
B It is more efficient for large herbivores to eat large, simple leaves.
C Compound leaves are thicker than simple leaves.
D It is more efficient for large herbivores to eat the small leaflets of compound leaves.
Solution The solution is (B). It is more efficient for herbivores to eat large, simple leaves. If
one leaflet is eaten, it does not affect the health of neighboring leaflets whereas a
simple leaf may be damaged beyond repair if it is chewed.
81 Stomata are usually found in higher numbers on the abaxial or bottom surface of a leaf.
What is the advantage of such an arrangement?
A Presence of stomata on the abaxial or bottom surface ensures that no, or very little,
water is lost due to guttation.
B The abaxial or bottom surface receives more sunlight, and water evaporates faster by
transpiration.
C Herbivores do not prefer to eat leaves with stomata on the abaxial or bottom surface.
D The adaxial or upper surface receives more sunlight, and water evaporates faster by
transpiration.
Solution The solution is (D). The adaxial or upper surface receives more sunlight and water
evaporates faster by transpiration.
82 Which plants have leaves that are adapted to cold temperatures?
A Conifers such as spruce, fir, and pine have oval-shaped leaves with sunken stomata,
helping to reduce water loss.
B Succulents such as aloes and agaves have waxy cuticles with sunken stomata, helping
to reduce water loss.
C Conifers such as spruce, orchids, and pine have needle-shaped leaves with sunken
stomata, helping to reduce water loss.
D Conifers such as spruce, fir, and pine have needle-shaped leaves with sunken stomata,
helping to reduce water loss.

********************************************************
**************************562**********************************

Solution The solution is (D). Conifers such as spruce, fir, and pine have needle-shaped leaves
with sunken stomata, helping to reduce water loss.
83 How is a leaf different from a leaflet?
A A leaf petiole attaches directly to the stem at a bud node, whereas a leaflet petiole is
attached to the main petioleor the midrib, not the stem.
B A leaf has reticulate venation, whereas leaflets show parallel venation.
C A leaf petiole attaches to the main petiole or the midrib, not the stem, whereas a
leaflet petiole attaches directly to the stem at a bud node.
D A leaf has parallel venation, whereas leaflets show reticulate venation.
Solution The solution is (A). A leaf petiole attaches directly to the stem at a bud node,
whereas a leaflet petiole is attached to the main petiole or the midrib, not the stem.
84 Scientists on a new project to restore a damaged salt marsh are investigating several
plants that could be introduced. Plant X is considered a possible candidate. Before the
decision is made, the following data are examined. Assume that the contribution of
gravity and matric potential are negligible and can be ignored. Recall that the overall
water potential for a system is represented by the equation

.
Overall of the soil is the solute potential of the plant’s cell contents is

and the pressure potential of the plant’s cells and


Is Plant X a good candidate for introduction to the salt marsh?
A Yes, because the overall water potential of the plant is less negative than the water
potential of the soil
B No, because the overall water potential of the plant is less negative than the water
potential of the soil
C Yes, because the overall water potential of the plant is more negative than the water
potential of the soil
D No, because the overall water potential of the plant is more negative than the water
potential of the soil
Solution The solution is (C). The overall water potential of the plant is It is
more negative than the water potential of the soil. Water will move from the soil
to plant X, making it a good candidate.
85 What organs in humans are similar in function to the vascular tissues of vascular plants?
Solution Arteries, veins, and capillaries transport water and other nutrients throughout the
bodies of humans, similar to the function of xylem and phloem in vascular plants.

********************************************************
*********************************************************561***

86 Apoptosis, or programmed cell death, is an important step in the development of xylem.


How does apoptosis contribute to xylem development?
Solution Xylem cells are thin and elongated during development. When these cells undergo
apoptosis, the cells die and the cells cytoplasm in organelles eventually degrade. This
leaves the cell walls, which form hollow tubes that are ideal for transporting water.
87 A florist decided to paint the leaves of poinsettia with a gold paint to embellish them. The
plant soon wilted and the leaves drooped. What explains this damage?
A The paint clogged the stomata. Without photosynthesis, the plant could not pull water
from the soil.
B The paint clogged the stomata. Without transpiration, the plant could not pull water
from the soil.
C The paint clogged the hydathodes. Without transpiration, the plant could not pull
water from the soil.
D The paint clogged the stomata. Without guttation, the plant could not pull water from
the soil.
Solution The solution is (B). The paint clogged the stomata. Without transpiration, the plant
could not pull water from the soil.
88 The process of bulk flow transports fluids in a plant. What are the two main bulk flow
processes?
A Movement of water up the xylem and movement of solutes up and down the phloem
B Movement of water up the phloem and movement of solutes up and down the xylem
C Movement of water up and down the xylem and movement of solutes up the phloem
D Movement of solutes up the xylem and movement of water up and down the phloem
Solution The solution is (A). The process of bulk flow moves water up the xylem and moves
photosynthates (solutes) up and down the phloem.
89 During a severe drought, the soil becomes dry and its water potential decreases. Many
plants will wilt in such an environment. Consider that the overall water potential for a
system is represented by the equation

.
What is one reason that plants are unable to draw water from the soil?
A The water potential of the soil becomes lower than the water potential of the plants.
B The water potential of the soil becomes lower than the solute potential of the plants.
C The water potential of the soil becomes higher than the water potential of the plants.
D The solute potential of the soil becomes lower than the water potential of the plants.

********************************************************
**************************562**********************************

Solution The solution is (A). The water potential of the soil becomes lower than the water
potential of the plants. Water cannot move from the soil to the plants.
90 A botanist compares the number of stomata between two plants. One plant, a eucalyptus,
has stomata equally distributed on both sides of the leaf. The other plant has most of its
stomata on the underside of the leaf.
What does the positioning of the stomata indicate about which leaf surfaces on the two
plants receive light in their natural environment?
A The first plant receives light only on the upper surface of the leaves, whereas the
leaves of the second plant are equally exposed to sunlight.
B The first plant receives light only on the lower surface, whereas the second plant
receives light only on the upper surface.
C The first plant receives light only on the upper surface, whereas the second plant
receives light only on the lower surface.
D The first plant has leaves that are equally exposed to sunlight, whereas the second
plant receives light only on the upper surface.
Solution The solution is (D). The first plant has leaves that hang vertically and are equally
exposed to sunlight whereas the second plant displays horizontal leaves. In the
latter, plant, the upper surfaces of the blades receive most of the sunlight.
91 In the Northern Hemisphere, owners and managers of plant nurseries have to plan
lighting schedules for a long-day plant that will flower in February. What lighting periods
and color will be most effective?
A Long periods of illumination with light enriched in the red range of the spectrum
B Short periods of illumination with light enriched in the red range of the spectrum
C Long periods of illumination with light enriched in the far-red range of the spectrum
D Short periods of illumination with light enriched in the far-red range of the spectrum
Solution The solution is (A). A long-day plant needs a higher proportion of the Pfr form to Pr
form of phytochrome. The plant requires long periods of illumination with light
enriched in the red range of the spectrum that convert Pr to Pfr.
92 Why do plants that cannot detect gravity show stunted growth with tangled roots and
trailing stems?
A Without gravitropism, both roots and seedlings would grow upward.
B Without gravitropism, roots would grow in all directions and seedlings would grow
upward.
C Without gravitropism, roots would grow upward but seedlings would not grow
upward toward the surface.

********************************************************
*********************************************************561***

D Without gravitropism, roots would grow in all directions but seedlings would not grow
upward toward the surface.
Solution The solution is (D). Gravitropism will allow roots to dig deep into the soil to find
water and minerals, whereas the seedling will grow towards light to enable
photosynthesis. Without gravitropism, roots would grow in all directions. Seedlings
would not grow upwards towards the surface.
93 Storage facilities for fruits and vegetables are usually refrigerated and well ventilated.
Why are these conditions advantageous?
A Refrigeration slows chemical reactions, including fruit ripening. Ventilation adds the
ethylene gas that speeds up fruit maturation.
B Refrigeration slows chemical reactions, including fruit maturation. Ventilation removes
the ethylene gas that reduces fruit ripening.
C Refrigeration slows chemical reactions, including fruit maturation. Ventilation removes
the ethylene gas that speeds up fruit ripening.
D Refrigeration removes the ethylene gas that speeds up fruit ripening. Ventilation slows
chemical reactions, including fruit maturation.
Solution The solution is (C). Refrigeration slows chemical reactions, including fruit
maturation. Ventilation removes the ethylene gas that speeds up fruit ripening.
94 A Venus flytrap has a very low sensitivity threshold, yet it can tell the difference between
the light touch of an insect and a drop of rainwater or wind. How can the Venus flytrap
differentiate between a random stimulus and an actual prey?
A Hair-like appendages on the surface of the leaves respond to repeated contact.
B Hair-like appendages on the surface of the leaves respond to a single contact.
C Hair-like appendages on the surface of the leaves respond to chemical stimulus from
the insect.
D Hair-like appendages on the surface of the leaves respond to the electrical stimulus
from the insect.
Solution The solution is (A). Hair-like appendages on the surface of the leaves respond to
repeated contact in several places as would be expected from a small insect
wandering across the surface of the leaf.
95 Stomata close in response to bacterial infection. This response is a defense mechanism
because it _____, and the hormone involved is _____.
blank blank

A restricts the entry of O2; gibberellin


B restricts the entry of CO2; abscisic acid
C prevents further entry of pathogens; aux
D prevents further entry of pathogens; abscisic acid

********************************************************
**************************562**********************************

Solution The solution is (D). To prevent further entry of pathogens, stomata close, even if
they restrict entry of carbon dioxide. Some pathogens secrete virulence factors that
inhibit the closing of stomata. Abscisic acid is the stress hormone responsible for
inducing closing of stomata.
96 Why is shade avoidance an important survival mechanism for plants? Would you expect
seeds with large energy storage to display as strong a response of shade avoidance as
small seeds with limited reserves?
A A seedling growing in the shade of a mature plant will not have enough light to
promote meristematic growth. A seed with large storage will be able to sustain growth
until its seedling can reach enough light for photosynthesis.
B A seedling growing in the shade of a mature plant will not have enough light to
promote photosynthesis. Small seeds with limited reserves will be able to sustain
growth until seedlings can reach enough light for photosynthesis.
C A seedling growing in the shade of a mature plant will not have enough light to
promote photosynthesis. A seed with large storage will be able to sustain growth until
its seedling can reach enough light for photosynthesis.
D A seedling growing in the shade of a mature plant will not have enough light to
promote respiration. Small seeds with limited reserves will be able to sustain growth
until their seedlings can reach enough light for photosynthesis.
Solution The solution is (C). A seedling growing in the shade of a mature plant will not have
enough light to promote photosynthesis because the important wavelengths will be
absorbed by the leaves in the mature plant. A seed with large storage will be able to
sustain growth until its seedling can reach enough light for photosynthesis.

TEST PREP FOR AP® COURSES


97 A plant has a measured pressure potential and a solute potential
. The soil is saturated with water because it rained. How will the water
move? After three months of dry weather, the soil has dried out. How will the water
potential of the soil compare to the water potential measured immediately before the
rain? How will the stomata respond to the change in weather?
A The water will move from the plant to the soil. Dry soil has a lower water potential
than wet soil. Under drought conditions, the stomata close to conserve water, and
leaves may also be shed if the drought continues.
B The water will move from the soil to the plant. Dry soil has a higher water potential
than wet soil. Under drought conditions, the stomata close to conserve water, and
leaves may also be shed if the drought continues.
C The water will move from the soil to the plant. Dry soil has a lower water potential
than wet soil. Under drought conditions, the stomata open its pores wider in order to
perform a better rate of transpiration.

********************************************************
*********************************************************561***

D The water will move from the soil to the plant. Dry soil has a lower water potential
than wet soil. Under drought conditions, the stomata close to conserve water, and
leaves may also be shed if the drought continues.
Solution The solution is (D). The water potential of soil soaked after rain is close to zero. The
water potential of the plant is lower: Dry soil has a lower water
potential than wet soil. Under drought conditions, the stomata close to conserve
water. If the drought continues, leaves may be shed altogether to conserve water.
The water will move from the soil to the plant. Dry soil has a lower water potential
than wet soil. Under drought conditions, the stomata close to conserve water and
leaves may also be shed if the drought continues.
98 Plants lose water from their aboveground surfaces in the process of transpiration. Most of
this water is lost from stomata. Excess loss of water has severe consequences and may be
fatal for the plant. The table shows data collected on a sunny day.

What is the best explanation for the transpiration rates leveling off and declining at
temperature higher than 27 °C?
A The plant ran out of water.
B The plant needs less water as temperature increases, so transpiration slows down to
limit water uptake by the roots.

********************************************************
**************************562**********************************

C Stomata close to conserve water, slowing down transpiration.


D The amount of water in the leaves decreases at high temperature and less is available
for evaporation.
Solution The solution is (C). At higher temperatures, stomata close in order to conserve water
and therefore the transpiration rate decreases.
99 Humidity is an environmental factor that affects transpiration rate. Which statement
accurately explains the shape of the curve obtained when increasing humidity is plotted
against constant temperature to find the rate of transpiration?
A Increasing humidity leads to reduced evaporation rates due to increased difference in
water vapor pressure between leaf and atmosphere.
B Increasing humidity leads to reduced evaporation rates due to decreased difference in
water vapor pressure between leaf and soil.
C Increasing humidity leads to reduced evaporation rates due to decreased difference in
water vapor pressure between leaf and atmosphere.
D Increasing humidity leads to increased evaporation rates due to decreased difference
in water vapor pressure between leaf and atmosphere.
Solution The solution is (C). Increasing humidity leads to reduced evaporation rates due to
decreased difference in water vapor pressure between leaf and atmosphere.
100 Plants sense drought through the decrease in water potential in the ground. This graph
shows concentrations of several hormones that were measured during a drought period
and plotted versus time. According to the data in the graph, which hormone shows the
strongest response to drought?

A Auxin
B Abscisic acid
C Cytokinin
D Gibberellins

********************************************************
*********************************************************561***

Solution The solution is (B). Abscisic acid increases the most over time, showing the greatest
increase in response to drought.
101 When drought conditions are forecast, fields are sprayed with a hormone that will
promote a stress response. According to the graph, which hormone should be sprayed
and why?

A Gibberellins, to promote plant growth before the plants are damaged


B Abscisic acid, to promote plant growth before the plants are damaged
C Abscisic acid, to promote protective response to drought before the plants are
damaged
D Gibberellins, to promote protective response to drought before the plants are
damaged
Solution The solution is (C). Abscisic acid is sprayed on the field because it promotes the
protective response to drought before the plants are damaged by drought.
102 Seeds were germinated in the dark on three plates. Plate A was irradiated with a short
pulse of red light; plate B was irradiated with a short pulse of red light followed by a pulse
of far-red light; and plate C was the control and was maintained in the dark. After three
days, the plates were scored for percentage of germination, as shown in this table.

What conclusion can be drawn from the experiment?


A Darkness inhibits germination.
B Red light promotes germination.

********************************************************
**************************562**********************************

C Far-red light promotes germination.


D Germination is independent from light irradiation.
Solution The solution is (B). Based on this experiment, red light promotes germination
because seeds irradiated with red light (with no other treatment) had the
best growth.
103 Seeds were germinated in the dark on three plates. Plate A was irradiated with a short
pulse of red light; plate B was irradiated with a short pulse of red light followed
immediately by a pulse of far-red light; plate C was irradiated by a short pulse of red light
followed one hour later by a pulse of far-red light; and plate D was the control and was
maintained in the dark. After three days, the plates were scored for percentage of
germination, as shown in this table.

What hypothesis, regarding the mechanism of action of red light, is best supported by
the results?
A Red light converts the phytochrome to its active form Pr, which can be converted to
the inactive form Pfr by far red light. After 1 h, the cascade of events initiated by Pfr
has already begun promoting germination; hence, it cannot be reversed even by the
pulse of far light.
B Red light converts the phytochrome to its active form Pfr, which can be converted to
the inactive form Pr by far red light. After 1 h, the cascade of events initiated by Pr has
already begun promoting germination; hence, it cannot be reversed even by the pulse
of far light.
C Far red light converts the phytochrome to its active form Pfr, which can be converted
to the inactive form Pr by red light. After 1 h, the cascade of events initiated by Pr has
already begun promoting germination; hence, it cannot be reversed even by the pulse
of far light.
D Red light converts the phytochrome to its active form Pfr, which can be converted to
the inactive form Pr by far red light. After 1 h, the cascade of events initiated by Pfr
has already begun promoting germination; hence, it cannot be reversed even by the
pulse of far light.
Solution The solution is (B). Red light converts the phytochrome to its active form Pfr. The
active form can be converted to the inactive form Pr by far red light. After 1 h, the
cascade of events initiated by Pr has already begun promoting germination. The
pulse of far red light cannot reverse activation at this point.

********************************************************
*********************************************************561***

104 After branches of woody saplings were trimmed, half of the cuts were covered with a
sealant and the other half were left untouched. The plants with sealed cuts fared much
better after several weeks.
What is the likely reason?
A The sealant stopped evaporation.
B The plants with sealed cuts grew new branches.
C The plants with unsealed cuts were infected by pathogens that entered through the
cuts.
D The plants with unsealed cuts lost photosynthates through bleeding of sap.
Solution The solution is (C). Unsealed cuts allowed pathogens to enter the plant because
the protective outer tissues were damaged and no substitute protectant (sealant)
was used.
105 Jasmonate is produced in plants as a response to injury. Researchers compared the
response to infection of mutant plants that were unable to produce jasmonate (Ja −) with
the response of normal plants (Ja+) from the same species. Leaves were inoculated with
spores from pathogenic molds. The size of the wounds was examined 48 h after
application. The plants were also infected with moths and the weight of the larvae was
determined after 48 h. This table shows the results.

According to the results of the experiment, what conclusion can the researchers draw
about the specificity of jasmonate protection?
A Jasmonate protects against infection from a variety of pathogens.
B Jasmonate protects against infection from one pathogen.
C Jasmonate cannot provide protection against infection.
D Jasmonate provides specific defense in winters and the defense is nonspecific in
summers.
Solution The solution is (A). Jasmonate protects against infection from a variety of pathogens
and is part of the nonspecific defense of the host.

********************************************************
**************************562**********************************

106 In the Northern Hemisphere, a florist grows shrubs of the same species of woody plant
under two different light schedules for three weeks. The first set is maintained under 15 h
of light and 9 h of dark daily. The second set is maintained under 9 h of light followed by
14 h of dark daily. The first set of plants does not form flowers, but the second set of
plants blooms.
What can you conclude about these plants?
A This species of shrub does not flower if the day is short.
B They bloom early in the year around February.
C They bloom mid-summer around June.
D The critical dark period is 9 h.
Solution The solution is (B). These plants must bloom in February because the second set of
plants, subjected to a lighting schedule similar to that observe in February, bloomed.
107 Heliotropism is the description of a response to the light of the sun. Seedlings of
sunflowers were exposed to sunlight for 15 days. Following the 15 days of exposure to
sunlight, the seedlings were transferred to complete darkness and their movement was
monitored. This graph plots the movement of the seedlings in the dark versus time.

What conclusion can be drawn about the light dependence of the movement of
sunflowers from the graph?
A The movement does require light once it is set but it will eventually slow down,
suggesting that a clock molecule is degraded over time.
B The movement does not require light once it is set and it will keep showing this
upward and downward trend in the same manner.
C The movement does not require light once it is set and it will eventually slow down,
suggesting that a clock molecule never degrades.
D The movement does not require light once it is set and it will eventually slow down,
suggesting that a clock molecule is degraded over time.

********************************************************
*********************************************************561***

Solution The solution is (D). The movement does not require light once it is set. The
movement will eventually dampen, suggesting that a clock molecule is degraded
over time.
108 A student randomly chose 40 tobacco seeds of the same species from a packet. He placed
20 seeds on moist paper towels in each of two petri dishes. He wrapped dish A completely
in an opaque cover to exclude all light. He did not wrap dish B. He placed the dishes
equidistant from a light source set to a cycle of 14 h of light and 10 h of dark. All other
conditions were the same for the two dishes. He examined the dishes after 7 days, and
permanently removed the opaque cover from dish A. This table shows the student’s data.

What is the most probable cause for the difference in mean stem length between plants
in dish A and plants in dish B?
A Shortening of cells in the stem in response to the lack of light
B Elongation of the stem in response to the lack of light
C Enhancement of stem elongation by light
D Genetic differences between the seeds
Solution The solution is (B). Plants in dish A lacked light and experienced stem elongation
as a result.
109 Groups of 20 seedlings from the same plant species were treated with gibberellins. Each
group received a different concentration of hormone. The seedlings were grown under
the same environmental conditions. After 15 days of growth, the internode distances
between the first and second sets of leaves were measured in each group of seedlings. On
this graph, the mean internode distance for each group is plotted against the
concentration of gibberellins that the group received.

********************************************************
**************************562**********************************

According to the results, why is this effect of gibberellins on internode length used in
agriculture to spray grapes with oversized fruit?
A To lengthen the internode distance and accommodate larger fruit
B To shorten the internode distance and accommodate larger fruit
C To lengthen the internode distance and accommodate more flowers
D To shorten the internode distance and accommodate smaller fruit
Solution The solution is (A). Gibberellins are sprayed on grapes to lengthen the internode
distance and accommodate larger fruit.

SCIENCE PRACTICE CHALLENGE QUESTIONS


23.1 The Plant Body
110 A. The net photosynthetic production rate (NPP) is the difference between the rate of
carbon fixation by photosynthesis and the respiration rate. Each of these rates can be
expressed in units of grams of carbon per day (gC/d). Vascular plants convert fixed carbon
that is not released as carbon dioxide into biomass with a growth rate.
Draw areas within the box to represent the rates of growth (G) and respiration (R) to
show the limit of each on the overall growth rate. The area of the box represents the rate
of photosynthesis (P).

********************************************************
*********************************************************561***

B. When the dependences on temperature of photosynthetic and respiration rates of a


vascular plant are measured, the results depend on the species but have the general form
shown in the figure. In these measurements, the temperature is maintained for several
hours. The plant is then returned to 25 °C for several hours before the next set of
measurements is made at a slightly higher temperature.

Evaluate these data to approximately predict the quantitative effect on the NPP and free
energy availability in a deciduous forest ecosystem with a 3–5 °C increase in temperature.
This is the expected temperature increase by the year 2100. Assume the current average
summer temperature of the forest ecosystem is 25 °C.
C. In other experiments, rather than returning the plants to 25 °C, the plant is grown for
several days at a constant higher temperature. Under these conditions, the maximum
photosynthetic rate shifts toward the temperature of the new growing conditions.
However, there is little change in the temperature dependence of respiration rate. This is
referred to as temperature acclimation, an effect of great importance to predictions of
future climate change.
Pose two scientific questions whose pursuit could lead to either an improved
understanding of the mechanisms of temperature acclimation or improvements in models
of atmospheric carbon dioxide concentrations that control temperature.
According to the graph, growth is predicted to increase when acclimation is taken into
account and the average temperature of Earth’s surface increases by the expected 3–5 °C.
Growth enhancement may be reduced, however, if respiration increases more rapidly
than photosynthesis, particularly under periods of drought and stress. Thus, climate
warming may result in positive, negative, or potentially no effect on the free energy
availability in forest ecosystems.

********************************************************
**************************562**********************************

D. In the figure, the response to temperate change in terms of the rates of photosynthesis
and respiration are sketched as a function of time from the very short-term (seconds) to
the longer-term (decades) changes. Acclimation in the laboratory occurs in days. Analyze
the graphs; in the box bounded by a dashed line, sketch curves for responses of both
processes beyond the acclimation observed in the laboratory that are consistent with a
neutral effect on free energy availability and provide your reasoning.

E. Analyze the long-term effect of a rate of respiration that exceeds the rate of
photosynthesis in terms of dynamic homeostasis.
Solution Sample answer:
A.

B. The photosynthetic rate falls by 5 to 10 percent and the respiration rate


increases by between 10 and 15 percent to reduce the available free energy by
10 to 15 percent (12 percent based on the data shown).
C. Since this process occurs over a period of few hours the change is probably due to
post-transcriptional regulation of photosynthetic and respiration enzymes or due to
structural changes associated with transport of carbon dioxide or water. The
research in this field focuses largely on the effect of temperature on the RuBisCO
system, but that is outside of the scope of the AP Biology Exam. Good questions
should address mechanisms and be testable. For example,

********************************************************
*********************************************************561***

 Does increasing temperature cause stomata closure to conserve water and in


doing so limit carbon dioxide supply?
 Does increasing temperature increase activity of enzymes involved in
respiration?
 Does increasing temperature affect the permeability of membranes in the
chloroplast to carbon dioxide or water?
D. The sketch should show that a neutral effect will restore the positive gap between
P and R so that G remains unchanged.

E. Growth will stop when P = R and for situations in which , the systems will
unsustainably rely on stored energy reserves.
23.5 Transport of Water and Solutes in Plants
111 A disruption of dynamic homeostasis in the relationship between vascular plants and
insects is occurring as global climate changes. The reduction in the yield of soybeans is
plotted against leaf area removed by two insects, beetles and aphids. Soybean blooms
begin to develop in the week of 13 July. Prior to that time, there is no effect of leaf
removal on yield, even with complete loss of leaves. In the week of 18 August, plants are
beginning to form seeds, and loss of leaves can be devastating.

********************************************************
**************************562**********************************

A. One observed effect of climate change is the shift toward earlier development in many
insects. Quantitatively describe the worst possible consequences for yield, assuming plant
developmental timing is not altered by warming temperatures, if the peak abundance of
Japanese beetles is shifted from 18 July to 13 July, and 80 percent of leaf area is lost.
B. The expression of genes involved in seed development is temperature dependent,
unlike the scenario suggested in (A). More than 90 percent of soybean seeds planted in
2015 in the soybean-corn ecosystem of the central United States are the herbicide-
resistant, genetically modified “Roundup Ready” variety. The seed has a patented
genome. It produces seeds that are sterile and must be purchased each spring from the
patent holder.
Predict how the use of Roundup Ready seeds affects the selection of expression regulated
in response to increasing temperature. Roundup is an herbicide whose active chemical
component is glyphosphate. This molecule disrupts the synthesis of phenylalanine,
tyrosine, and tryptophan. By inserting a gene from Agrobacteria, a Roundup Ready seed
can synthesize these amino acids in the presence of the herbicide.
C. Pose two scientific questions that must be considered to estimate the long-term
effectiveness of this strategy for weed management.
Solution Sample answer:
A. This shift will reduce reduction in seed yield. With the earlier maximum
emergence the worst case (100 percent leaf area removed) seed yield is reduced by
approximately 25 percent. With a shift to a maximum emergence in the week of
13 July, the worst case loss is between 10 and 15 percent.
B. This process is operating outside of pressures imposed by a changing climate. As
temperature increases, blooms will set earlier and the advantage described in part A
is lost.

********************************************************
*********************************************************561***

C. For this differential response to glyphosphate to continue the plants that are
affected must remain susceptible. This means that there can be no mutation or
horizontal transfer that could lead to changes in the genomes of targeted plants. In
fact, selection has resulted in just a few years of “super weeds” that are not
susceptible to Roundup. Some studies have suggested adverse health and
environmental effects from Roundup in fresh water. In the meantime the patent has
expired and genetically modified seed that are not sterile are available. Roundup use
continues to increase. A shift in public acceptance of GMOs is having a small effect
and earlier strategies of weed management such as weeding by hand and deep
tillage are becoming more popular. These practices also bear a price, such as
immigration of farm workers and soil erosion.
Good questions may or may not be informed by the social and technological
contexts. Good questions target a possible relationship that is testable. For example,
 How will the populations be affected?
 How quickly will Roundup Ready fungal and microbial populations emerge?
 Will populations that have beneficial effect on plant growth and derive a
benefit from the host receive an advantage?
 How soon will the Agrobacterium gene appear in weeds that are resistant?
 What is the fate of Roundup in the environment?
 Will the costs of remediation reduce the effectiveness of the herbicide?
 How does one determine the effectiveness of a material for which neither
the costs nor the date when these costs must be paid are known?
112 A. By increasing the photosynthetic surface area, a plant increases the rate of capture of
free energy. For every carbon atom fixed into carbohydrates, between 200 and 400 water
molecules are released through stomata to the atmosphere. A simple geometric model
can be used to estimate the minimum number of leaves on a tree, as shown.

Identify and justify the data needed to describe the relationship between the free energy
captured and the water transpired by a tree with dimensions D, L, and W. Use these data

********************************************************
**************************562**********************************

to construct a mathematical model of the relationship between transpiration rate and


the rate of free energy captured when a single carbon atom is fixed.
B. The diversity of vascular plants decreases with increasing latitude. Equatorial
ecosystems have greater plant diversity than do ecosystems further south or north. One
of several explanations offered to account for this observation is the energy equivalence
model—as free energy increases, population size increases. As population size increases,
mutations increase. One bit of evidence for the energy-equivalence model is the
correlation of family-level diversity with actual evapotranspiration, the sum of water
transferred by both transpiration and evaporation of surface water. This property is
reported in mm of water per square meter of surface area.
Explain the relationship between free energy exchange and latitude that is the basis of
the energy-equivalence model.
C. Shared ancestry is indicated by taxonomic classification in which a family of organisms
contains many genera, and within each genus there are many species. A survey of tree
flora (Latham and Ricklefs, Oikos, 67, 1993) at comparable latitudes in a temperate
eastern Asia forest ecosystem (729 species in 177 genera and 67 families) and an eastern
North America forest ecosystem (253 species in 90 genera and 46 families) had no species
in common, but there were 20 common genera and 40 common families. Actual
evapotranspiration for the two ecosystems are 850 ± 200 mm (eastern North America)
and 730 ±160 mm (eastern Asia).
Analyze these data to test the validity of the energy-equivalence model.
Solution Sample answer:
A. We have an interval of water transpired per stomata per carbon fixed. Now we
need to convert carbon fixed to a quantity of energy captured to obtain a
relationship between water transpired per stoma and energy captured. To obtain a
relationship to the number of leaves, we need data on the average number of
stomata per leaf.
Letting Ecarbon be the rate of energy captured per carbon fixed, cwater, the carbon fixed
per water molecule transpired, wstomata be the number of water transpired per stoma,
sleaf the number of stomata per leaf and Nleaves is the number of leaves.

Given D, L, and W and estimates of the other parameters, this model predicts
transpiration rate.
B. Evapotranspiration has two pieces: (1) transpiration from the plant which
increases with the amount of radiation received and (2) evaporation from the
surface which increases with the amount of radiation received. Since radiation is
determined by latitude, so is evapotranspiration. Since we have connected
evapotranspiration to energy captured, we have connected latitude to energy
captured.

********************************************************
*********************************************************561***

C. These data do not support the energy-equivalence model. Family-level diversity is


smaller where evapotranspiration is higher. A model in which species-level diversity
is predicted to be proportional to energy is similarly rejected.
113 The evolution of vascular plants followed the colonization of terrestrial habitats by
ancestors of Chlorophyta, green algae, during the Devonian period (which began about
400 million years ago). The three most significant structural innovations in that process
are responses to selection through the availability of water resources: (1) the cuticle—a
waxy covering of the epidermis that retains water; (2) stomata—openings that penetrate
the cuticle through which water and carbon dioxide are transported; and (3) a vascular
system—plant tissues through which water moves.
Measurements of gases trapped in ice cores provide atmospheric concentrations of the
distant, as well as the recent, past. Life must adapt to changes in the environment.
Woodward examined samples from the Cambridge herbarium of several trees (Nature,
327, 1987) to determine the stomatal index (percentage of epidermal cells that contain a
stoma). In 1720, when the herbarium samples were collected, the carbon dioxide
concentration in Earth’s atmosphere was 225 ppm. In the year of the study, 1987, it was
340 ppm (it is 370 ppm in 2016). The following table presents some of Woodward’s
reported results.
Tree Genus CO2 (ppm) Stomatal Index (%)
Acer 225 14.9 ± 0.8
Blank
340 6.7 ± 1.1
Quercus 225 17.4 ± 1.1
Blank
340 9.6 ± 1
Rumex 225 15.5 ± 0.7
blank
340 11.8 ± 0.9
Teng and coworkers (PLoS ONE, 2009) followed the dependence of Arabidopsis, a
member of the Brassica family of vascular plants, grown under a range of elevated carbon
dioxide concentrations for 15 generations. They found elevated stomatal densities for
each generation that were not heritable.
Engineer and coworkers (Nature, 513, 2014) discovered a mutant Arabidopsis in which
stomatal density increases as carbon dioxide concentration increases. Measurements of a
component of the set of mRNA molecules for epidermal patterning factor 2 (EPF2),
responsible for stomatal density, are shown for plants grown in low and high carbon
dioxide concentrations.

********************************************************
**************************562**********************************

A. Analyze these data in terms of the likelihood that the effect of carbon dioxide
concentration on stomatal density involves negative feedback at the level of (i)
translation, (ii) post-transcription, or (iii) changes in genotype.
B. Changes in precipitation patterns are expected to accompany an increase in
atmospheric carbon dioxide. Predict the effect on a forest where trees that have matured
over decades are suddenly under drought stress. Justify your prediction in terms of
positive or negative feedback where stomatal density is high and a drought occurs.
C. In a favorable environment, trees continue to accumulate biomass and increase in
height until the flow rate of water through the xylem (plant vascular tissue that transports
water and minerals from roots to shoots) is no longer sufficient to support the negative
water potential at the interface between root and soil. Fluid dynamic models predict that
increasing the diameter, d, of the xylem greatly increases the rate of flow of water,
leading to greater productivity when water is abundant. Under conditions of drought
stress, the water potential is reduced, and an air bubble can disrupt the flow of water in
that vessel entirely. A larger stem diameter permits a larger number of small vessels.
Describe a model of the evolution of xylem in trees in terms of selection under conditions
of unlimited and limited water resources.
D. Olson and Rosell (New Phytologist, 197, 2013) investigated the question of whether
xylem diameter was determined by water availability or by plant height and,
consequently, stem diameter. A summary of their data is shown on the left with lines of
best fit through data with the corresponding color.

********************************************************
*********************************************************561***

Analyze these data and summarize the pattern that addresses their question. Note: As x
increases, log(x) increases.
Solution Sample answer:
A. Stomata are structures that are produced during development and the
dependence on carbon dioxide concentration shows control during development.
The study of Arabidopsis suggests that stomatal density is not fixed in the genome.
So the reduction is either through transcription factors that regulate expression of
the genome or at the post-transcription level. The data from Engineer et al. show
that the stomatal density response to carbon dioxide concentration is transcriptional
control since mRNA is depressed at lower levels and the mRNA levels are reversed
for the mutant.
B. If stomatal density is set during a period of low carbon dioxide, a relatively sudden
(nearly a factor of two in 300 years) change in carbon dioxide concentrations will,
neglecting acclimation, result in a high rate of photosynthesis. With no, or a small,
increase in respiration, there will be an increase in growth. However, if there is also
an increase in drought stress, having additional stomata will increase water loss
when those stomata are open. Therefore, the effect of drought can create positive
feedback, as a greater number of stomata will cause increased water loss when they
are opened to take in carbon dioxide.
C. The data provided by the problem description point to a positive selection for
increasing xylem diameter where water is abundant. In that case, increasing height
leads to increasing productivity and over many generations the xylem diameter

********************************************************
**************************562**********************************

would increase. And the data points to a negative selection for increasing xylem
diameter where water scarcity occurs. However, to achieve comparable flux of
water there would be a positive selection for increasing stem diameter. So both
models lead to increasing stem diameter with increasing height, but dry conditions
would lead to smaller xylem than do wet conditions.
D. These data are consistent with vessel diameter increasing with height. However,
they are inconsistent with vessel diameter under the control of water resources.
Wet and dry conditions lead to nearly identical relationships and relatively dry
environment (savanna) has vessel diameters that are larger than those of relatively
wet environments.
23.6 Plant Sensory Systems and Responses
114 Like the animal intestine, the organ system principally responsible for nutrient and water
uptake, the plant root system, is home to a microbiome upon which the host depends.
One important role for the root microbiome is innate immunity. Wheat take-all is a
disease caused by the fungus Gaeumannomyces graminis that attacks plant roots and
blocks root water channels. When a major outbreak occurs in a wheat field, susceptibility
remains high in the following year. But after four to six continued crops of wheat in the
same field, susceptibility to the disease declines. This resistance can be transferred with
the soil. Burning the soil surface or rotation with another crop returns susceptibility to the
next wheat crop. The Fusarium (a fungus) wilt disease of strawberries and potato scab
caused by Streptomyces scabies (a bacterium) show a similar disease progression and
transferability of resistance (Weller, Ann. Rev. Plant Phytopath, 26, 1988).
A. Plants, like animals, have immune defenses that may involve cooperative interactions
between organisms. Describe a model of immune response that accounts for these
behaviors.
In plants, the first line of defense is the cell wall. Animal cells lack this protective barrier.
Adaptive immunity of vertebrates to pathogens uses specific defenses that are
transportable within the organism, such as T-cells, and retains information about earlier
infections using memory T-cells. Unlike adaptive immunity, the innate responses of plants
are much less effective in defending against necrotrophic (colonizing dead tissue) than
against biotrophic (infecting living tissue) pathogens. In animal tissue, the response to
infection is inflammation, the recruitment of resources to protect the tissue. In plant
tissue, the response is apoptosis.
B. Describe contrasting models of defense strategies for plants and animals that express
each of these differences in terms of these strategies: cell boundary, immunological
memory, and tissue repair.
Solution Sample answer:
A. The transferability of immunity is analogous to the first organism-level
observation of immunity conferred by immunization. The populations of microbes in
the soil that present disease suppression have increased. This might be a

********************************************************
*********************************************************561***

competitive exclusion of the fungus, but it turns out that a bacterial secretion is anti-
fungal. This model is consistent with the effect of burning the field or rotation, both
of which disrupt the populations that confer disease resistance.
B. In plants, some defenses can be circulated or coordinated through the vascular
tissue. In many cases, however, a plant cell defends itself and in the event that the
defenses fail the cell destroys itself. This table summarizes the models and
corresponding evidence.
Defense Level of Protection
Cell boundary A physical barrier to the entry of pathogens is present in the
plant cell (the cell wall) that is not present in the animal cell. The
strategy for the plant is therefore not to let the pathogen enter
while the animal cell admits, recognizes, and responds to the
pathogen. Since the response in an animal is quickly distributed
by the circulatory system in minutes, the cell responds in a way
that quickly benefits the whole organism. Plant cells also produce
messages in response to infection that reduce further infection in
other parts of the plant. However, this process is generally slower
than in animals, again, due to the faster-moving animal
circulatory system.
Immunological It has been shown that plant cell defenses are effective against
memory biotrophs (pathogens specialized to invade a narrow range of
hosts) but not against necrotrophs (pathogens specialized to
invading a wide range of hosts). This suggests that the defense of
the cell is initiated by the response within the cell and not by
other cells. In many cases, plant cells respond to the malfunction
of cell processes that are key targets for pathogens. Rather than
responding to the pathogen (which would work for necrotrophic
pathogens), the plant defense is a response to an attack on a
functioning subsystem. More recent research has shown that this
is not true in all cases, however, as plant cells can recognize
pathogen associated molecular patterns, such as peptidoglycans
in pathogen capsules, flagellin within the flagella of pathogens,
etc. Plant cells also produce messages in response to infection
that reduce further infection in other parts of the plant.
Tissue repair The animal cell increases temperature and nutrition of the
affected tissue, etc. These are strategies that are aimed at cell
survival. The plant cell has a threshold beyond which no defense
is mounted. Rather the plant cell initiates apoptosis in an attempt
to quarantine the affected cells.

********************************************************
604 24 | The Animal Body: Basic Form and Function

24 | THE ANIMAL BODY: BASIC FORM AND


FUNCTION
REVIEW QUESTIONS
1 The pleural cavity is part of which cavity?
A Dorsal
B Thoracic
C Abdominal
D Pericardial
Solution The solution is (B). The pleural cavity is part of the thoracic cavity, which is part of
the ventral cavity.
2 Which plane divides an animal into dorsal and ventral portions?
A Sagittal
B Midsagittal
C Frontal
D Transverse
Solution The solution is (C). A frontal plane divides a body into dorsal and ventral portions.
3 What is the term for a hard covering or shell that provides protection and muscle
attachment?
A Apodeme
B Fusiform
C Exoskeleton
D Endotherm
Solution The solution is (C). An exoskeleton provides protection from predators and water
loss as well as attachment for muscles.
4 Which organism has a fusiform shape?
A Elephant
B Dolphin
C Spider
D Human
Solution The solution is (B). A dolphin has a fusiform shape because it is tubular with
tapered ends.

********************************************************
24 | The Animal Body: Basic Form and Function 605

5 Which type of animal maintains a constant internal body temperature?


A Endotherm
B Ectotherm
C Poikilotherm
D Fusiform
Solution The solution is (A). An endotherm is an animal that is able to maintain a relatively
constant body temperature, typically by having some form of insulation.
6 Smaller endothermic animals have _____ surface area for their mass compared with
blank

larger endothermic animals.


A equal
B greater
C less
D no
Solution The solution is (B). Smaller endothermic animals have greater surface area relative
to their mass, and therefore lose heat at a faster rate and require more energy to
maintain a constant internal temperature.
7 What is the term for epithelial cells that are composed of multiple layers?
A Simple
B Stratified
C Squamous
D Transitional
Solution The solution is (B). Stratified epithelia are epithelial cells that are arranged in
multiple layers.
8 Which type of epithelial cell is best adapted to aid diffusion?
A Squamous
B Cuboidal
C Columnar
D Transitional
Solution The solution is (A). Squamous epithelial cells facilitate diffusion in tissues when they
are flat and arranged in a single layer.

********************************************************
606 24 | The Animal Body: Basic Form and Function

9 Why do osteoclasts need to break down bone?


A To deposit bone material into the bone matrix
B To facilitate osteoclast persistence without using excess energy
C To provide access to calcium in the tissue
D To facilitate compact bone structure
Solution The solution is (C). Osteoclasts break down bone, which provides access to calcium
in the tissues.
10 Plasma is the —
A fibers in the blood
B matrix of the blood
C cell that phagocytizes bacteria
D cell that functions in response to antigens
Solution The solution is (B). The fluid portion, or matrix, of the blood is referred to as
the plasma.
11 Why is it necessary for most muscle cells to be under voluntary control?
A To facilitate response to local conditions of tissues
B To facilitate movement of bone
C To speed up or slow down the autonomic nervous system
D To facilitate movement of internal organs
Solution The solution is (B). Most muscle cells must be under voluntary control to facilitate
the movement of bone.
12 Cardiac muscle contains specialized regions along the plasma membrane called
intercalated discs. What is the role of intercalated discs?
A To efficiently pass electrical impulses between cardiac cells
B To facilitate immune response to foreign antigens
C To cushion body organs from damage
D To keep blood vessels in place
Solution The solution is (A). The intercalated disc is a line in cardiac muscle that assists in
efficiently passing electrical impulses between cells.

********************************************************
24 | The Animal Body: Basic Form and Function 607

13 Which part of the neuron contains the nucleus?


A Axon
B Dendrite
C Cell body
D Oligodendrocyte
Solution The solution is (C). The cell body of a neuron contains the nucleus.
14 Schwann cells or oligodendrocytes manufacture a lipid called myelin. Which statement
best describes the function of this lipid?
A Regulates the chemical environment
B Sends input
C Receives input
D Improves signal transfer efficiency
Solution The solution is (D). Oligodendrocytes insulate the axon to improve the efficiency of
electrical impulse transfer.
15 Animals maintain an overall steady state of internal conditions by —
A ectothermy
B homeostasis
C basal metabolic rate
D standard metabolic rate
Solution The solution is (B). Homeostasis is the maintenance of dynamic equilibrium that
controls proper body functions.
16 To what does the term equilibrium refer in the context of organismal homeostasis?
A Control mechanisms that amplify a response
B Control mechanisms that increase or decrease a stimulus
C The target point in homeostasis
D Maintenance of body functions within a given range
Solution The solution is (D). Homeostasis maintains a dynamic equilibrium, or given range, to
maintain appropriate body functions.
17 What type of feedback loop pushes an organism’s physiology further away from its
normal set points?
A Positive-feedback loop
B Negative-feedback loop

********************************************************
608 24 | The Animal Body: Basic Form and Function

C Set point
D Receptor
Solution The solution is (A). Positive-feedback loops amplify a stimulus and may push an
organism further out of homeostasis.
18 When faced with a sudden drop in environmental temperature, an endothermic animal
will —
A experience a substantial drop in its body temperature
B find a warm rock on which to bask
C increase muscle activity to generate heat
D increase fur or fat to increase insulation
Solution The solution is (C). Endothermic animals increase muscle activity to generate heat
when faced with a sudden drop in environmental temperature.
19 Which type of feedback loops primarily control homeostasis?
A Positive
B Negative
C Acclimatization
D Receptor
Solution The solution is (B). Negative-feedback loops increase or decrease a stimulus to
maintain equilibrium.
20 What is an example of negative feedback?
A Lowering of blood glucose after a meal
B Blood clotting after an injury
C Lactation during nursing
D Uterine contractions during labor
Solution The solution is (A). After eating a large meal, increased glucose leads to insulin
production. Insulin production leads to glucose in the blood being stored.
21 Which method of heat exchange occurs during direct contact between the source and the
animal?
A Radiation
B Evaporation
C Convection
D Conduction

********************************************************
24 | The Animal Body: Basic Form and Function 609

Solution The solution is (D). Conduction is when heat is conducted from one surface to
another.
22 Which strategy may be employed by an ectotherm to immediately increase body
temperature?
A Consume more food to increase fat as insulation
B Increase amount of vasodilation
C Increase amount of muscle contraction
D Sit on a warm rock
Solution The solution is (D). The body temperature of ectotherms is controlled by their
external environment, and sitting on a warm rock would be an immediate way for an
ectotherm to increase body temperature.

CRITICAL THINKING QUESTIONS


23 How does an asymmetrical body plan differ from radial or bilateral body plans?
A Asymmetrical organisms can produce equal halves if cut along a certain plane,
whereas radially and bilaterally symmetric organisms have no distinct pattern.
B Asymmetrical organisms have no distinct pattern, whereas radially and bilaterally
symmetric organisms can produce equal halves if cut along a certain plane.
C Asymmetrical organisms produce equal halves if cut along a certain plane with no
definite right or left side, whereas radially and bilaterally symmetric organisms can
produce equal halves.
D Asymmetrical organisms produce equal halves if cut along a certain plane with definite
right and left sides, whereas radially and bilaterally symmetric organisms can produce
equal halves.
Solution The solution is (B). Asymmetrical organisms have no distinct pattern or symmetry.
Radially and bilaterally symmetric organisms can produce equal or similar halves if
cut along a certain plane.
24 Why are most organisms with exoskeletons relatively small?
A Increases in body weight increase body size by a factor of eight, and the chitin
thickness of the exoskeleton has to significantly decrease to accommodate increase in
body size.
B Doubling of body size increases body weight by a factor of eight, and the chitin
thickness of the exoskeleton has to significantly decrease to accommodate weight
increase.

********************************************************
610 24 | The Animal Body: Basic Form and Function

C Increases in body weight increase body size by a factor of eight, and the chitin
thickness of the exoskeleton has to significantly increase to accommodate increase in
body size.
D Doubling of body size increases body weight by a factor of eight, and the chitin
thickness of the exoskeleton has to significantly increase to accommodate weight
increase.
Solution The solution is (D). Doubling of body size increases body weight by approximately a
factor of eight. In response to this, the chitin thickness of the exoskeleton has to
proportionately increase to accommodate weight increase.
25 What is the relationship between basal metabolic rate (BMR) and body size? Why?
A BMR decreases with body size, because larger animals require more energy to
maintain their size. However, smaller animals have relatively higher BMRs per body
weight because they have greater surface area.
B BMR increases with body size, because smaller animals require more energy to
maintain their size. However, larger animals have relatively higher BMRs per body
weight because they have greater surface area.
C BMR increases with body size, because larger animals require more energy to
maintain their size. However, smaller animals have relatively higher BMRs per body
weight because they have greater surface area for their mass.
D BMR decreases with body size, because smaller animals require more energy to
maintain their size. However, larger animals have relatively higher BMRs per body
weight because they have greater surface area for their mass.
Solution The solution is (C). BMR increases with absolute body size. This is true because larger
animals require more energy to maintain their size. However, smaller animals have
greater surface area for their mass, which means that they expend more energy to
maintain body temperature.
26 Radial symmetry is typically found in aquatic organisms. What is radial symmetry and why
is it advantageous to certain aquatic organisms?
A Radially symmetric means that a plane cut from the front to back of the organism
produces distinct left and right sides that are mirror images of each other. It helps
certain aquatic organisms to extract food from surrounding environments.
B Radially symmetric means that a plane cut from the front to back of the organism
produces distinct left and right sides that are mirror images of each other. It helps
certain aquatic organisms to perform photosynthesis.

********************************************************
24 | The Animal Body: Basic Form and Function 611

C Radially symmetric means that a plane cut along its longitudinal axis will produce
equal halves, and there is no distinct left or right. It helps certain aquatic organisms to
perform photosynthesis.
D Radially symmetric means that a plane cut along its longitudinal axis to produce equal
halves, and there is no distinct left or right. It helps certain aquatic organisms to
extract food from surrounding environments.
Solution The solution is (D). Organisms with radial symmetry have an up-and-down
orientation. Cutting along any longitudinal plane produces similar halves, and there
is no distinct left or right. Radial symmetry is typically found in organisms that attach
themselves to a base to extract food from surrounding environments.
27 Columnar epithelial cells, which are typically found in a single-layer arrangement, are
found along the digestive tract. What is the role of columnar epithelial cells in digestion?
A Columnar epithelial cells absorb material from the lumen of the digestive tract to
prepare the material for entry into the body.
B Columnar epithelial cells release mucus for lubrication as well as antimicrobial agents
in the digestive tract.
C Columnar epithelial cells secrete enzymes like salivary amylase which aid in digestion
by the breakdown of carbohydrates in the body.
D Columnar epithelial cells help in the propulsion of food by peristalsis in the digestive
tract of the body.
Solution The solution is (A). Columnar epithelial cells are specialized to absorb material from
the lumen of the digestive tract and transport material into the body.
28 In vertebrates, cartilage is found in fetal bones, ears, and intervertebral discs, whereas
bone is found in the skeleton. What are the similarities between cartilage and bone?
A Both are types of connective tissue in the body and cells of both are known as
chondrocytes.
B Both are types of connective tissue in the body and have non-vascular organic matrix
material that provides strength and flexibility.
C Both are types of connective tissue in the body and have organic matrix material that
provides strength and flexibility.
D Both consist of bone marrow and have organic matrix material that provides strength
and flexibility.
Solution The solution is (C). Cartilage and bone are both types of connective tissue in the
body. Both cartilage and bone have organic matrix material that provides strength
and flexibility.

********************************************************
612 24 | The Animal Body: Basic Form and Function

29 A friend sneaks up behind you and scares you, speeding up your heart rate. How and why
did this event influence cardiac muscle contraction?
A Muscle contraction speed increases as the enteric nervous system responds to local
conditions and makes muscle contraction speed up or slow down.
B Muscle contraction speed increases as the autonomic nervous system responds to
local conditions and makes muscle contraction speed up or slow down.
C Muscle contraction speed increases as the somatic nervous system responds to local
conditions and makes muscle contraction speed up or slow down.
D Muscle contraction speed increases as the central nervous system responds to local
conditions and makes muscle contraction speed up or slow down.
Solution The solution is (B). The autonomic nervous system responds to local conditions and
makes muscle contraction speed up or slow down. In the case of being frightened, if
your heart rate sped up, the muscle contraction speed increased.
30 Neurons have several specialized structures, including dendrites. What might happen if an
individual has malformed dendrites?
A The individual’s neurons would not be able to receive input properly.
B The individual’s neurons would not be able to synthesize proteins.
C The individual’s neurons would not be able to communicate with target neurons.
D The individual’s neurons would not be able to carry nerve signals.
Solution The solution is (A). Dendrites are cell body projections of a neuron and they are
specialized for receiving input. Therefore, if an individual has malformed dendrites,
their neurons would not be able to receive input properly.
31 How can squamous epithelia, which have a high surface area-to-volume ratio, both
facilitate diffusion and prevent damage from abrasion?
A Single layers of squamous epithelia facilitate gas, nutrient, or waste exchange,
whereas stratified layers provide protection but are not replaceable following damage.
B Stratified layers of squamous epithelia facilitate gas, nutrient, or waste exchange,
whereas single layers provide protection and are replaceable following damage.
C Single layers of squamous epithelia facilitate gas, nutrient, or waste exchange whereas
stratified layers provide protection and are replaceable following damage.
D Single layers of squamous epithelia facilitate only exchange of gases by diffusion,
whereas stratified layers provide protection and are replaceable following damage.
Solution The solution is (C). Thin thinness and great surface area of single layers of squamous
epithelia facilitate gas, nutrient, or waste exchange. Stratified layers provide
protection due to the tissue thickness and the ability to replace layers following
damage.

********************************************************
24 | The Animal Body: Basic Form and Function 613

32 What is homeostasis and how does it help maintain equilibrium of various body functions
throughout the body?
A Homeostasis is the process of achieving stability, which occurs through behavioral
changes. Equilibrium is maintained by ensuring that body functions remain within a
certain range.
B Homeostasis is the process by which constant adjustments to changes in the body
occur, and equilibrium is maintained by ensuring that body functions remain within a
certain range.
C Homeostasis is the process that prevents blood loss from circulation when a blood
vessel is ruptured, and equilibrium is maintained by ensuring that circulation of blood
is kept within a normal range.
D Homeostasis is the process by which constant adjustment to changes in the body
occurs, and equilibrium is maintained as body functions remain within a certain range
without any fluctuations.
Solution The solution is (B). Homeostasis is the process by which constant adjustment to
changes in the body occur using negative feedback. Equilibrium is a set-point
maintained by ensuring that body functions remain within a certain range.
33 How can an environmental change result in an alteration of gland secretion?
A A receptor detects change and sends a signal to the control center, which sends a
signal to the gland to inhibit the gland secretions.
B A receptor detects change and sends a signal to the control center, which sends a
signal to the gland to increase the secretions of the gland.
C A receptor detects change and sends a signal to the effector directly, which in this case
is the gland.
D A receptor detects change and sends a signal to the control center, which in turn
sends a signal to the effector, which in this case is the gland.
Solution The solution is (D). A receptor detects change in response to stimuli. Alteration of
the receptor sends a signal to the control center, which in turn sends a signal to the
effector, which in this case is the gland.
34 How is a condition such as diabetes a good example of the failure of a set point in
humans?
A A negative-feedback loop cannot proceed in diabetic individuals, as they do not
produce enough functional insulin to lower blood sugar.
B Negative-feedback loop cannot proceed in diabetic individuals, as they do not produce
enough functional insulin to increase the blood sugar.

********************************************************
614 24 | The Animal Body: Basic Form and Function

C Positive-feedback loop cannot proceed in diabetic individuals, as they do not produce


enough functional insulin to lower blood sugar.
D Positive-feedback loop cannot proceed in diabetic individuals, as they do not produce
enough functional insulin to increase the blood sugar.
Solution The solution is (A). The set point is the homeostasis target. Blood sugar decreases
after eating a meal because insulin is produced when sugar levels increase; in
response, sugar is used as energy stored in cells, which reduces blood sugar level.
Diabetic individuals do not produce enough functional insulin to lower blood sugar.
Therefore, the negative-feedback loop cannot proceed.
35 What are the roles of vasodilation and vasoconstriction in maintaining body temperature?
A Vasodilation allows for radiation and evaporative heat loss, and vasoconstriction
brings blood to the core to conserve heat by vital organs.
B Vasodilation brings blood to the core to conserve heat by vital organs, and
vasoconstriction results in radiation and evaporative heat loss.
C Vasodilation results in the formation of an insulating layer between skin and internal
organs, causing heat conservation and brings blood to the core to conserve heat.
D Vasodilation results in radiation and evaporative heat loss, and vasoconstriction
transfers heat from arteries to veins to warm blood returning to the heart.
Solution The solution is (B). Vasodilation allows more blood to go to the body’s surface, which
results in radiation and evaporative heat loss. Vasoconstriction reduces blood flow,
which brings blood to the core to conserve heat by vital organs.

TEST PREP FOR AP® COURSES


36 Maintaining body heat is important for maintaining body functions in animals. Which
statement provides an example of how an animal can actively generate body heat?
A Triglycerides are used to store energy for later use.
B An animal produces metabolic waste energy in the form of heat.
C An animal has insulation, which helps it maintain a constant body temperature.
D An animal eats a large amount of high-fat foods to produce adipose tissue.
Solution The solution is (B). Various activities of an animal, such as muscle movement or
mitochondrial activity, produce metabolic waste energy in the form of heat.

********************************************************
24 | The Animal Body: Basic Form and Function 615

37 Ectotherms and endotherms have different strategies for generating and maintaining
body heat. Why are ectotherms more dependent on the environment for body heat than
endotherms, and how are endotherms able to generate and maintain body temperature?
A Ectotherms use external thermal heat whereas endotherms use metabolically
generated heat to help regulate and maintain body temperatures.
B Ectotherms use external heat to help regulate and maintain body temperatures
whereas endotherms have constantly varying internal temperatures.
C Ectotherms use metabolically generated heat to maintain a constant body
temperature whereas endotherms use metabolically generated heat to regulate body
temperature within a wider range.
D Ectotherms use external thermal energy to help regulate and maintain body
temperatures whereas endotherms maintain a constant body temperature.
Solution The solution is (A). Ectotherms use external thermal energy, such as sunlight, to help
regulate and maintain body temperatures, whereas endotherms use metabolically
generated energy, such as muscle contraction, to maintain body temperatures.
38 Which statement most directly supports the claim that different species of organisms use
different metabolic strategies to meet their energy requirements for growth,
reproduction, and homeostasis?
A During cold periods, pond-dwelling animals can increase the number of unsaturated
fatty acids in their cell membranes, while some plants make antifreeze proteins to
prevent ice crystal formation in tissues.
B Bacteria lack introns, while many eukaryotic genes contain many of these intervening
sequences.
C Carnivores have more teeth that are specialized for grinding food.
D Plants generally use starch molecules for storage while animals use glycogen and fats
for storage.
Solution The solution is (D). Plants generally use starch molecules for storage while animals
use glycogen and fats for storage.
39 The body sizes of organisms vary and tend to be correlated with the region in which the
organisms are found. Why do organisms at different latitudes tend to have different body
sizes, and what is the relationship between heat loss and body size in an organism?
A Temperature varies by latitude, and body size affects heat retention and loss. Smaller
organisms lose heat at a slower rate than larger organisms, because they have a
smaller surface area for their mass.
B Temperature varies by latitude, and body size affects heat retention and loss. Smaller
organisms lose heat at a faster rate than larger organisms, because they have a
greater surface area for their mass.

********************************************************
616 24 | The Animal Body: Basic Form and Function

C Temperature varies by latitude, and body size affects heat retention and loss. Larger
organisms lose heat at a faster rate than smaller organisms, because they have a
greater surface area for their mass.
D Temperature varies by latitude, and body size affects heat retention and loss. Smaller
organisms lose heat at a faster rate than larger organisms, because they have a
smaller surface area for their mass.
Solution The solution is (B). Temperature varies by latitude, and body size affects heat
retention and loss. Smaller organisms lose heat at a faster rate than larger organisms
because they have a greater surface area for their mass.
40 If an American alligator has been basking but gets too hot, how might the alligator
cool itself?
A Increase vasodilation
B Sweat
C Move into shade
D Increase metabolic rate
Solution The solution is (C). The alligator might move into shade so air currents remove heat
from the body.
41 During torpor, arctic ground squirrels reduce their energy requirements by reducing their
core body temperature and metabolic rate, as shown in the graph.

Why would an active ground squirrel’s ATP synthesis also increase in proportion to
metabolic rate when temperatures fall below 0 °C?
A Colder temperatures cause ATP to degrade.
B ATP is synthesized through cellular respiration, which provides body heat.
C ATP synthesis is needed to provide more oxygen to the cells.
D ATP is consumed by the cells to generate body heat.

********************************************************
24 | The Animal Body: Basic Form and Function 617

Solution The solution is (A). Energy is needed to increase metabolic rates to prevent the body
from freezing.
42 Why is hibernation NOT a good option for small animals such as hummingbirds to help
reduce their metabolic rate and conserve their need for food?
A Hummingbirds have a fast metabolic rate and a large surface area to volume ratio.
B Hummingbirds are unable to lower their metabolic rate and body temperature to
enter hibernation.
C Hummingbirds migrate south for the winter.
D Hummingbirds live a short life.
Solution The solution is (A). Hummingbirds undergo nightly torpor to save energy that would
otherwise be used to maintain body temperature. Hummingbirds are not able to eat
enough to maintain their body temperature. Thus, they would not have sufficient
energy to maintain such a long state of hypothermia.
43 How does hibernation differ in small animals such as ground squirrels and larger animals
such as bears?
A Smaller animals can engage in torpor, while larger animals cannot.
B Larger animals can engage in torpor, while smaller animals cannot.
C Smaller animals cannot remain inactive throughout the entire winter, while larger
animals can.
D Larger animals cannot remain inactive throughout the entire winter, while smaller
animals can.
Solution The solution is (C). The student should mention that ground squirrels store food in
their burrow so that when they awake every 7–10 days, they are able to feed. Bears
must fatten up before hibernation and sleep for most of the hibernation period
without eating. Ground squirrels are capable of reducing their body temperature
near 0 °C, while bears must maintain their body temperatures near 37 °C.

********************************************************
618 24 | The Animal Body: Basic Form and Function

44 In the data table, BM is body mass; CD, cool-down time; WU, warm-up time; NBT, normal
body temperature; and BTH, body temperature during hibernation.

What can you conclude from the data collected on five different animals as shown in
the table?
A The time it takes for animals to change body temperature is directly related to
body size.
B The time it takes for animals to change their body temperature is indirectly related to
their size.
C Larger animals hibernate for longer periods of time.
D Smaller animals hibernate for shorter periods of time.
Solution The solution is (A). This is shown by the reduced warm-up time as body mass
increases.

********************************************************
24 | The Animal Body: Basic Form and Function 619

45 In the data table, BM is body mass; CD, cool-down time; WU, warm-up time; NBT, normal
body temperature; and BTH, body temperature during hibernation.

What can you conclude from about the time it takes to cool down versus the time it takes
to warm up?
A Larger animals consume more energy to maintain their body temperatures.
B Smaller animals can survive hibernation with less food reserves than larger animals.
C Smaller animals require more time to alter their body temperature.
D Larger animals require more time to alter their body temperature.
Solution The solution is (C). This is shown by the reduced warm-up and cool-down time as
body mass increases.
46 The endocrine system incorporates feedback mechanisms that maintain homeostasis.
Which example demonstrates negative feedback by the endocrine system?
A During labor, the fetus exerts pressure on the uterine wall, inducing the production of
oxytocin, which stimulates uterine wall contraction. The contractions cause the fetus
to further push on the wall, increasing the production of oxytocin.
B After a meal, blood glucose levels become elevated, stimulating beta cells of the
pancreas to release insulin into the blood. Excess glucose is then converted to
glycogen in the liver, reducing blood glucose levels.

********************************************************
620 24 | The Animal Body: Basic Form and Function

C At high elevation, atmospheric oxygen is scarcer. In response to signals that oxygen is


low, the brain decreases an individual’s rate of respiration to compensate for the
difference.
D A transcription factor binds to the regulating region of a gene, blocking the binding of
another transcription factor required for expression.
Solution The solution is (B). After a meal, blood glucose levels become elevated, stimulating
beta cells of the pancreas to release insulin into the blood. Excess glucose then is
converted to glycogen in the liver, reducing blood glucose levels.
47 This figure depicts the process of calcium homeostasis.

How is blood calcium control an example of a negative-feedback loop?


A Cells in the parathyroid gland sense calcium decrease, causing parathyroid hormone
release and stimulating calcium absorption. Bone also may break down to release
calcium.
B Cells in the parathyroid gland sense calcium decrease, causing calcitonin release and
stimulating calcium absorption. Bone also may break down to release calcium.
C Cells in the thyroid gland sense calcium decrease, causing calcitonin release and
stimulating calcium absorption. Bone also may break down to release calcium.
D Cells in the parathyroid gland sense calcium increase, causing parathyroid hormone
release and stimulating calcium absorption. Bone also may break down to release
calcium.

********************************************************
24 | The Animal Body: Basic Form and Function 621

Solution The solution is (A). Calcium decreases, which cells in the parathyroid gland sense. In
turn, parathyroid hormone is released, which stimulates calcium absorption. Bone
also may break down to release calcium. Therefore, the stimulus changed direction
and this is a negative-feedback loop.
48 In organisms, homeostasis of various bodily processes, such as body temperature, blood
glucose levels, and blood calcium levels, is essential for the maintenance of proper body
functions. What role does insulin play in homeostasis?
A When a fetus pushes against the uterine wall, insulin is released by the brain to
stimulate uterine contractions.
B In the presence of decreased blood glucose levels, insulin is produced by the
parathyroid to increase calcium absorption.
C Insulin activation activates other clotting factors until a fibrin clot is produced.
D Insulin is secreted by the pancreas in response to elevated blood glucose levels to
remove glucose from the blood.
Solution The solution is (D). In response to increased blood glucose levels, insulin is secreted
and removes glucose from the blood.
49 Proper blood glucose levels are necessary to maintain cellular function, because glucose is
fuel for cells. Glucagon is an important component of blood glucose homeostasis, which is
maintained by a negative-feedback loop.
What is the role of glucagon in blood glucose homeostasis?
A When blood sugar is low, glucose and ATP produce glycogen. Excess blood sugar
stimulates the release of glucagon, which in turn stimulates glycogen release to
increase blood glucose levels.
B When there is excess blood sugar, excess glucose and ATP produce glucagon. A drop in
blood glucose level stimulates the release of glycogen, which in turn stimulates
glycogen release to increase blood glucose levels.
C When there is excess blood sugar, the excess glucose and ATP produce glycogen. A
drop in blood glucose level stimulates the release of glucagon, which in turn
stimulates the release of glycogen to increase blood glucose levels.
D When there is excess blood sugar, the excess glucose and ATP produce glycogen. A
drop in blood glucose level stimulates the release of glucagon, which in turn releases
more glucagon to increase blood glucose levels.
Solution The solution is (C). When there is excess blood sugar, the excess glucose and ATP
stimulate the production of glycogen. A drop in blood glucose level stimulates the
release of glucagon, which in turn stimulates the release of glycogen to increase
blood glucose levels.

********************************************************
622 24 | The Animal Body: Basic Form and Function

50 One process that is under the control of a negative-feedback loop is red blood cell
production. These cells carry oxygen to all of the body cells, and remove some carbon
dioxide.
What would most likely happen if an individual had a sufficient number of red blood cells?
A The individual would have increased red blood cell production.
B The individual’s body would start destroying the red blood cells.
C The individual’s body would cease production of new red blood cells.
D The individual would produce the same amount of red blood cells.
Solution The solution is (C). Negative-feedback loops change the direction of a stimulus;
therefore, once a sufficient number of red blood cells is achieved, production ceases.
51 Diabetes results when either insulin cannot be produced or does not function properly.
Consequently, diabetes can produce complications such as blindness, heart disease, and
kidney disease. To help manage diabetes, a patient can get insulin injections.
How do insulin injections promote a negative-feedback loop to help maintain blood
glucose production?
A Insulin injections allow transport and storage of glucose to increase blood glucose
levels after consuming a large or high-glucose meal.
B Insulin injections allow only storage of glucose to decrease blood glucose levels after
consuming a large or high- glucose meal.
C Insulin injections allow transport and storage of glucose to increase blood glucose
levels before consuming a meal.
D Insulin injections allow transport and storage of glucose to decrease blood glucose
levels after consuming a large or high-glucose meal.
Solution The solution is (D). Insulin secretion lowers blood glucose levels. Therefore, insulin
injections allow transport and storage of glucose to decrease blood glucose levels
after consuming a large or high-glucose meal.
52 Positive-feedback loops amplify processes in organisms. Which statement describes the
role of the hormone oxytocin in a positive-feedback loop for childbirth?
A Oxytocin halts uterine contractions when the fetus pushes on the uterine wall.
B Oxytocin maintains pain levels as the child is pushed through the birth canal.
C Oxytocin stimulates uterine contractions when the fetus pushes on the uterine wall.
D Oxytocin decreases pain levels as the child is pushed through the birth canal.
Solution The solution is (C). Positive-feedback loops amplify a stimulus. Uterine contractions
send a signal to the brain, which releases oxytocin, which in turn stimulates more
uterine contractions.

********************************************************
24 | The Animal Body: Basic Form and Function 623

53 Birth is one of the few positive-feedback loops observed in humans and is essential for the
proper delivery of babies. How does a baby pushing against a pregnant woman’s cervix
stimulate a positive-feedback loop?
A Stretching stimulates nerve impulses to be sent to the brain, which releases oxytocin
from the pituitary, which in turn causes uterine contractions.
B Stretching stimulates nerve impulses to be sent to the brain, which releases estrogen
from the pituitary, which in turn causes uterine contractions.
C Stretching stimulates nerve impulses to be sent to the brain, which releases oxytocin
from the parathyroid gland, which in turn causes uterine contractions.
D Stretching stimulates nerve impulses to be sent to the brain which releases
progesterone from the pituitary, which in turn causes uterine contractions.
Solution The solution is (A). A positive-feedback loop occurs when a stimulus is amplified. The
stretching stimulates nerve impulses to be sent to the brain, which releases oxytocin
from the pituitary, which in turn causes uterine contractions.
54 Negative-feedback mechanisms are far more prevalent in the human body than positive-
feedback loops because they help regulate homeostasis. However, there are some
instances of positive-feedback loops that can be observed in animals.
Regulation of which option is an example of a positive-feedback loop?
A When body temperature gets too high, signals are sent to reduce body temperature.
B Increased blood glucose levels stimulate insulin production, which in turn sequesters
glucose from the blood.
C Decreased calcium levels stimulate increased calcium absorption.
D Activation of one clotting factor stimulates production of other clotting factors until a
fibrin clot is produced.
Solution The solution is (D). The clotting response involves the enzymatic conversion of
soluble protein to an insoluble protein. Activation of one clotting factor stimulates
production of other clotting factors until a fibrin clot is produced.
55 Both negative- and positive-feedback loops are essential for maintaining proper
body functions. Blood calcium and blood clotting are under the control of different
feedback loops.
Which process is maintained by a positive-feedback loop and why?
A Blood clotting is maintained by a positive-feedback loop, as clotting is amplified in
response by increasing the amount of clotting factors when clotting factors are
present.
B Blood clotting is maintained by a positive-feedback loop, as clotting factors are
maintained in a specific range and a positive loop helps return the conditions to the
set point.

********************************************************
624 24 | The Animal Body: Basic Form and Function

C Blood calcium is maintained by a positive-feedback loop, as calcium levels are


amplified in response by increasing the amount of calcium levels when calcium is
present.
D Blood calcium is maintained by a positive-feedback loop, as calcium levels are
maintained in a specific range and a positive-feedback loop helps return the
conditions to the set point.
Solution The solution is (A). Blood clotting is under the control of a positive-feedback loop,
because clotting is amplified in response by increasing the amount of clotting factors
when clotting factors are present. However, blood calcium is under the control of a
negative-feedback loop, because it is essential for blood calcium levels to be
maintained in a specific range, and a negative-feedback loop helps return the
conditions to the set point if they change.

********************************************************
25 | Animal Nutrition and the Digestive System 625

25 | ANIMAL NUTRITION AND THE


DIGESTIVE SYSTEM
REVIEW QUESTIONS
1 When you eat an apple, it is first physically broken down into smaller fragments. What is
the term for this process?
A Elimination
B Absorption
C Mastication
D Peristalsis
Solution The solution is (C). When you eat an apple, you first physically break down the apple
into smaller fragments by chewing, or masticating; mastication is the first step of
digestion in humans.
2 Which statement is true?
A The majority of water is reabsorbed by the small intestines.
B Elimination is a process that occurs via diffusion.
C Absorption is the process that chemically breaks down food.
D The small intestines absorb nutrients.
Solution The solution is (D). Absorption occurs as nutrients diffuse across the epithelial cell
lining of the small intestines; this process allows the nutrients to enter the
bloodstream.
3 Ruminants and pseudo-ruminants are both able to digest plant materials, but they have
different mechanisms for doing so. Which animal is a pseudo-ruminant?
A Cow
B Goat
C Crow
D Camel
Solution The solution is (D). A camel is a pseudo-ruminant because it has a three-chambered
and not a four-chambered stomach. The camel’s cecum digests foraged material.
4 Which statement about animal digestion is true?
A Roughage is digested very quickly.
B Birds eat large quantities at one time.

Advanced Placement Biology Instructor’s Solution Manual


626 25 | Animal Nutrition and the Digestive System

C Birds have a four-chambered stomach.


D In pseudo-ruminants, roughage is digested in the cecum.
Solution The solution is (D). The elongated cecum of pseudo-ruminants provides more
surface area for the digestion of cellulose-containing roughage.
5 Chemical and mechanical digestion begins in the mouth, where food is prepared into a
_____, which is then swallowed.
blank

A bolus
B trachea
C peristalsis
D sphincter
Solution The solution is (A). Chemical and mechanical digestion begins in the mouth, where
food is prepared into a bolus, which results from chewing and wetting in the mouth.
The bolus is then swallowed.
6 Which statement about digestion is true?
A Pepsin is converted to pepsinogen with the help of hydrochloric acid.
B The digestion of starches begins in the mouth.
C Wave-like muscle movements called peristalsis move food from the stomach to
the mouth.
D Amino acids are absorbed through the intestinal lining of the ileum.
Solution The solution is (B). Starch digestion begins in the mouth. Amylase, which is secreted
by the saliva, aids in the breakdown of starches to simpler sugars such as maltose.
7 Chyme is highly acidic. What is secreted by the pancreas to neutralize chyme?
A Hydrochloric acid
B Bicarbonate
C Bile
D Amylase
Solution The solution is (B). Bicarbonate, which is alkaline, is secreted by the pancreas to
neutralize the acidic chyme.
8 How does the liver assist in fat digestion?
A By producing bicarbonate
B By concentrating bile salts
C By producing bile
D By producing pepsin

Advanced Placement Biology Instructor’s Solution Manual


25 | Animal Nutrition and the Digestive System 627

Solution The solution is (C). The liver assists in fat digestion by producing bile, which breaks
down fat.
9 When you eat food, it is vital that the nutrients be absorbed. How does absorption occur?
A Food is mechanically and chemically broken down into smaller molecules.
B Alternating waves of muscular contraction facilitate the movement of food.
C Partially digested food flows into the small intestine to prevent food regurgitation.
D Nutrients diffuse across the intestines.
Solution The solution is (D). When you eat food, nutrients are absorbed by diffusion across
the intestines, which allows nutrients, water, salts, and some vitamins to be utilized
by the body.
10 Certain organs control the release of hormones that have vital roles in digestion. What
controls hunger and satiety signals?
A Thymus
B Adrenal cortex
C Thyroid
D Hypothalamus
Solution The solution is (D). Among its other functions, the hypothalamus region of the brain
controls hunger and satiety signals.
11 One cup of which food has the most calories?
A Spaghetti with tomato sauce
B Deep-fried zucchini
C Mixed fruit
D Scrambled eggs
Solution The solution is (B). One cup of deep-fried zucchini has the most calories. Although
zucchini is a vegetable, it is high in fat when it is deep fried. Fat has approximately
9 kcal/g.
12 Plant materials, such as fruits and vegetables, are difficult to digest because they are
difficult to break down. How are humans able to obtain nutrients from fruits and
vegetables?
A Humans produce cellulase, which breaks down cellulose.
B Intestinal flora have enzymes that break down some of the fiber.
C Bile is released from the gallbladder to break down fiber.
D In the stomach, pepsin is produced to break down plant material.

Advanced Placement Biology Instructor’s Solution Manual


628 25 | Animal Nutrition and the Digestive System

Solution The solution is (B). Humans are able to obtain nutrients from fruits and
vegetables because intestinal flora have enzymes that break down some of
the fiber in these foods.
13 Which statement is NOT an example of how fat is beneficial?
A Fat helps absorb lipid-soluble vitamins.
B Fat helps produce lipid-soluble hormones.
C Fat has low energy density.
D Fat makes you feel full more quickly.
Solution The solution is (C). Fat does not have low energy density. Fat has high energy
density, with approximately 9 kcal/g.
14 Certain molecules are required by but not produced by the body. Fat- and water-soluble
_____ are organic molecules that cannot be produced by the body but are required for
blank

many enzymatic functions.


A minerals
B vitamins
C amino acids
D sugars
Solution The solution is (B). Fat- and water-soluble vitamins are organic molecules that
cannot be produced by the body but are required for many enzymatic functions.
Vitamins act as coenzymes that are required for many enzymatic functions.
15 What is the result of insufficient amounts of the mineral iodine in the body?
A Muscle weakness
B Poor immune function
C Mood disturbances
D Goiters
Solution The solution is (D). The result of iodine insufficiency is goiters. Iodine is required for
thyroid hormone synthesis, and goiters can form when an individual is iodine
deficient.
16 Adenosine triphosphate, or ATP, is the source of energy for cells. In which bonds does ATP
store energy?
A Carbohydrate
B Glycolysis
C Glycogen
D Phosphodiester

Advanced Placement Biology Instructor’s Solution Manual


25 | Animal Nutrition and the Digestive System 629

Solution The solution is (D). Adenosine triphosphate, or ATP, is the source of energy for cells.
ATP stores energy in phosphodiester bonds, which release energy when broken.
17 Which statement about glycogen is true?
A When an individual is sedentary, glycogen is converted to glucose.
B The liver releases glycogen when blood sugar drops.
C ATP is produced by excess glycogen and glucose.
D During glycolysis, glycogen is converted to pyruvic acid.
Solution The solution is (B). The liver releases glycogen when blood sugar drops. Some
glycogen is stored in the liver and then released and broken down into glucose and
ATP when blood sugar drops; this process increases blood sugar.
18 What is produced from excess ATP and glucose?
A Glycogen
B Pyruvate
C Peptides
D Essential nutrients
Solution The solution is (A). Glycogen is formed when excess ATP and glucose stimulate
glycogen synthesis. Once excess ATP and glucose form glycogen, it is stored in the
liver and in skeletal muscles to be used at a later time.
19 Which statement is NOT a reason animals require ATP?
A ATP is needed to build organic molecules.
B ATP provides energy for muscle contraction.
C ATP assists in electrical signal transmission.
D ATP is the body’s fuel source.
Solution The solution is (D). ATP is not the body’s fuel source. Glucose is the body’s fuel
source, and glycolysis produces ATP, which is considered energy currency for cells.
20 Different macromolecules have varying amounts of energy density. What is the least
energy dense?
A Protein
B Fat
C Fiber
D Carbohydrates
Solution The solution is (C). Fiber is the least energy-dense option; it is approximately 1.5 to
2.5 kcal/g.

Advanced Placement Biology Instructor’s Solution Manual


630 25 | Animal Nutrition and the Digestive System

21 What does NOT play a role in masticating food?


A Teeth
B Pharynx
C Saliva
D Tongue
Solution The solution is (B). The pharynx is not involved in food mastication. Once the food
bolus is swallowed, it goes into the pharynx.
22 Which statement about the process of digestion is true?
A Organisms absorb large molecules through digestive cells.
B The last step of digestion is absorption.
C Food is only mechanically broken down in the mouth.
D Food is prepared into a bolus before it is swallowed.
Solution The solution is (D). Food is prepared into a bolus before it is swallowed. A bolus
results from the chewing and wetting of food in the mouth.
23 Which enzyme is involved in carbohydrate digestion?
A Pancreatic amylase
B Elastase
C Trypsin
D Pepsin
Solution The solution is (A). Pancreatic amylase is involved in carbohydrate digestion.
Pancreatic amylase breaks polysaccharides into disaccharides and monosaccharides
in the small intestines.
24 In protein digestion, what happens in the stomach?
A Aminopeptidase and dipeptidase break peptides into amino acids.
B Pepsin breaks proteins into peptides.
C Trypsin, elastase, and chymotrypsin break proteins into peptides.
D Carboxypeptidase breaks peptides into amino acids and peptides.
Solution The solution is (B). In protein digestion, pepsinogen is secreted by stomach cells and
then converted to pepsin, which breaks proteins into peptides.
25 Water reabsorption is an essential component of processing food. Where is the majority
of water reabsorbed?
A Small intestines
B Rectum

Advanced Placement Biology Instructor’s Solution Manual


25 | Animal Nutrition and the Digestive System 631

C Colon
D Anus
Solution The solution is (C). The majority of water is reabsorbed in the colon. The colon is the
site of the majority of intestinal flora, which aid in both digestion and water
reabsorption.
26 If you come down with the flu, you might experience emesis. What causes emesis?
A Stomach muscle contractions
B Neural signals that urge elimination
C Inadequate water reabsorption
D Excess water reabsorption
Solution The solution is (A). Emesis refers to stomach muscle contractions that stimulate
vomiting.
27 Not all organs involved in processing food are involved in digestion. Which organ is NOT
involved in digestion?
A Mouth
B Anus
C Stomach
D Small intestine
Solution The solution is (B). The anus is not involved in digestion, but it is involved in food
elimination.
28 Which statement about digestion of food in the large intestine is true?
A Mechanical digestion occurs by bacteria.
B Semisolid waste is moved by wave-like muscle contractions.
C Most nutrients are absorbed in this organ.
D Peristaltic mixing occurs in this organ.
Solution The solution is (B). Semisolid waste is moved through the colon by wave-like muscle
contractions called peristalsis.
29 Taking in food, or _____, is the first step of gaining nutrients from food.
blank

A digestion
B ingestion
C elimination
D absorption

Advanced Placement Biology Instructor’s Solution Manual


632 25 | Animal Nutrition and the Digestive System

Solution The solution is (B). Ingestion, swallowing, or drinking is the first step of gaining
nutrients from food. Food must be ingested before it can be broken down and the
nutrients absorbed.
30 What is the correct order of processes by which nutrients and energy are obtained
from food?
A Digestion → ingestion → absorption → elimination
B Ingestion → absorption → digestion → elimination
C Ingestion → digestion → absorption → elimination
D Ingestion → digestion → elimination → absorption
Solution The solution is (C). The correct order of processes by which nutrients and energy are
obtained from food is ingestion, digestion, absorption, and then elimination. Once
food is ingested, it is broken down by chemical and mechanical digestion, the
nutrients and other molecules are absorbed, and waste is then eliminated.
31 Gastric control has three phases that assist in digesting food. Which phase is initiated
by chyme?
A Intestinal
B Gastric
C Cephalic
D Digestive
Solution The solution is (A). The intestinal phase of gastric control is initiated by chyme, which
triggers digestive secretions.
32 What occurs during the cephalic phase of gastric control?
A Salivation is triggered.
B Food is processed by gastric acids and enzymes.
C Gastrin is produced.
D Digestive secretions are released.
Solution The solution is (A). Seeing, smelling, and ingesting food can trigger salivation during
the cephalic phase of gastric control.
33 Hormones are essential for digesting ingested food items. Which hormone controls the
release of bile from the gallbladder?
A Pepsin
B Gastrin
C Amylase
D Cholecystokinin

Advanced Placement Biology Instructor’s Solution Manual


25 | Animal Nutrition and the Digestive System 633

Solution The solution is (D). Cholecystokinin, or CCK, is the hormone that stimulates
production of pancreatic juices as well as controls the release of bile into the
duodenum from the gallbladder.
34 What is the role of gastrin in food digestion and absorption?
A Gastrin stimulates the release of stomach acid.
B Gastrin stimulates production of bicarbonate.
C Gastrin stimulates production of pancreatic juices.
D Gastrin stops the release of stomach acid.
Solution The solution is (A). The role of gastrin in food digestion and absorption is the release
of stomach acid. The presence of protein stimulates the release of stomach acid for
chemical digestion.
35 The gastric phase assists in processing ingested materials. When does the gastric
phase begin?
A When food is smelled
B When food reaches the stomach
C When chyme enters the small intestines
D When food is ingested
Solution The solution is (B). The gastric phase begins when food reaches the stomach and
enzymes as well as gastric acids process the food.

CRITICAL THINKING QUESTIONS


36 How do villi and microvilli aid in absorption?
A Villi and microvilli increase the surface area of the small intestine, which aids in the
absorption of bile salts and vitamin B12.
B Villi and microvilli increase the surface area of the small intestine, which increases the
absorption of nutrients by diffusion.
C Villi and microvilli form the inner layer of epithelial tissue in the small intestine and
increase the absorption of nutrients from chyme.
D Villi and microvilli absorb food through the small intestine via smooth muscle
contractions called peristalsis.
Solution The solution is (B). Villi, folds on the inner surface of the small intestine, and
microvilli, projections on the epithelial cell surface, increase the surface area of the
small intestine, which increases the absorption of nutrients by diffusion.

Advanced Placement Biology Instructor’s Solution Manual


634 25 | Animal Nutrition and the Digestive System

37 Ruminants, such as this goat, are able to digest large amounts of plant material. How is
plant material passed through, digested, and absorbed in the ruminant digestive system?

A Food is chewed in the mouth, and then passes through the esophagus into the rumen
and then the reticulum, which contains microbes that break down cellulose and
ferment the ingested plant material. The ruminant regurgitates cud from the rumen,
and the food is passed into the omasum for water removal and then into the small
and large intestines for nutrient and further water absorption. Waste is excreted
through the anus.
B Food is chewed in the mouth, and then passes through the esophagus into the rumen
and then the reticulum, which contains microbes that break down cellulose and
ferment the ingested plant material. The ruminant regurgitates cud from the rumen,
and the food is passed into the abomasum for water removal and then into the small
and large intestines for nutrient and further water absorption. Waste is excreted
through the anus.
C Food is chewed in the mouth, and then passes through the esophagus into the rumen
and then the reticulum, which contains microbes that break down proteins and
ferment the ingested plant material. The ruminant regurgitates cud from the rumen,
and the food is passed into the omasum for water removal and then into the small
and large intestines for nutrient and further water absorption. Waste is excreted
through the anus.
D Food is chewed in the mouth, and then passes through the esophagus into the
reticulum and then the rumen, which contains microbes that break down cellulose
and ferment the ingested plant material. The ruminant regurgitates cud from the
rumen, and the food is passed into the omasum for water removal and then into the
small and large intestines for nutrient and further water absorption. Waste is excreted
through the anus.
Solution The solution is (A). Ruminants have a four-chambered stomach. Food is chewed in
the mouth, and then passes through the esophagus into the rumen and then the
reticulum, which contains microbes that break down cellulose and ferment the
ingested plant material. The ruminant regurgitates cud from the rumen, and the
food is then passed into the omasum for water removal. The cud then passes into

Advanced Placement Biology Instructor’s Solution Manual


25 | Animal Nutrition and the Digestive System 635

the abomasum, where enzymes digest the cud. The food then passes into the small
and large intestines for nutrient and further water absorption, and the waste is
excreted through the anus.
38 How does a stomach ulcer form? How could you prevent a stomach ulcer from forming in
your stomach?
A When the serosa layer of the stomach ruptures and does not reform, an open wound
is formed. It may be caused by bacteria. Ulcers can be prevented by eliminating the
ingestion of items that cause degradation of the mucous lining, like foods that irritate
the stomach.
B When the mucous lining of the stomach ruptures and does not reform, an open
wound is formed. It may be caused by a virus. Ulcers can be prevented by eliminating
the ingestion of items that cause degradation of the mucous lining, like foods that
irritate the stomach.
C When the mucous lining of the stomach ruptures and does not reform, an open
wound is formed. It may be caused by bacteria. Ulcers can be prevented by ingesting
items that will increase the acid content of the stomach.
D When the mucous lining of the stomach ruptures and does not reform, an open
wound forms. It may be caused by bacteria. Ulcers can be prevented by eliminating
the ingestion of items that cause degradation of the mucous lining, such as foods that
irritate the stomach.
Solution The solution is (D). A stomach ulcer forms when the mucous lining of the stomach
ruptures and does not reform, which may be caused by bacteria; an open wound is
formed. Ulcers can be prevented by eliminating the ingestion of items that cause
degradation of the mucous lining, such as foods that irritate the stomach.
39 How is the gallbladder involved in digestion, even though it is considered an
accessory organ?
A The gallbladder secretes bile to the duodenum, which uses bile to break down
proteins. The gallbladder is considered an accessory organ because food does not
directly pass through it.
B The gallbladder secretes bile to the duodenum, which uses bile to break down fats.
The gallbladder is considered an accessory organ because food does not directly pass
through it.
C The gallbladder secretes bile to the ileum, which uses bile to break down fats. The
gallbladder is considered an accessory organ because food does not directly pass
through it.
D The gallbladder secretes bile to the ileum, which uses bile to break down proteins. The
gallbladder is considered an accessory organ because only a very small amount of
digestion takes place in the gallbladder.

Advanced Placement Biology Instructor’s Solution Manual


636 25 | Animal Nutrition and the Digestive System

Solution The solution is (B). This is the manner in which the gallbladder, though an accessory
organ, is involved in digestion.
40 What is the role of saliva in the digestive system?
A Saliva contains an enzyme called amylase, which starts the chemical digestion in the
mouth by breaking down proteins.
B Saliva contains an enzyme called lipase, which starts chemical digestion in the mouth
by breaking down proteins.
C Saliva contains an enzyme called maltase, which starts chemical digestion in the
mouth by breaking down carbohydrates.
D Saliva contains an enzyme called amylase, which starts chemical digestion in the
mouth by breaking down carbohydrates.
Solution The solution is (D). Saliva contains an enzyme called amylase, which starts chemical
digestion in the mouth by breaking down carbohydrates.
41 What are the biological benefits of a balanced diet?
A A balanced diet provides excess energy to be stored in the body and nutrients to
maintain good health and increase reproductive capability.
B A balanced diet allows excess energy to be stored in the body, thereby increasing the
rate of metabolic reactions.
C A balanced diet provides nutrients needed to maintain proper bodily functions and
vitamins and minerals needed to maintain good health and reproductive capability.
D A balanced diet provides nutrients needed to maintain proper bodily functions and
vitamins and minerals needed to increase reproductive capability.
Solution The solution is (C). A balanced diet provides nutrients that are needed to maintain
proper bodily functions and vitamins and minerals that are needed to maintain good
health and reproductive capability.
42 Why is it important to eat carbohydrates, which provide organic carbons?
A They are needed to provide insulation to mammals.
B They help fight infections.
C They are needed to produce antibodies.
D They are needed to build cells and tissues.
Solution The solution is (D). Organic molecules including carbon are needed to build cells and
tissues, and organic carbons must be consumed.

Advanced Placement Biology Instructor’s Solution Manual


25 | Animal Nutrition and the Digestive System 637

43 Why is it necessary to consume essential nutrients? What are two examples of fat-soluble
essential vitamins, and what are their functions in the human body?
A Essential nutrients are synthesized by the body but not in sufficiently large numbers to
maintain body health. Vitamins B and C are two fat-soluble essential vitamins.
Vitamin B helps maintain eyesight, and vitamin C is essential for blood clotting.
B Essential nutrients are not synthesized by the body, but they are necessary for proper
body function. Vitamins A and K are two fat-soluble essential vitamins. Vitamin A helps
maintain connective tissue, and vitamin K is essential for blood clotting.
C Essential nutrients are synthesized by the body but not in sufficiently large numbers to
maintain body health. Vitamins D and K are two fat-soluble essential vitamins.
Vitamin D helps maintain a stable nervous system, and vitamin K is essential for blood
clotting.
D Essential nutrients are not synthesized by the body, but they are necessary for proper
body function. Vitamins A and K are two fat-soluble essential vitamins. Vitamin A helps
maintain eyesight, and vitamin K is essential for blood clotting.
Solution The solution is (B). Essential nutrients are nutrients—including vitamins, minerals,
and some amino acids—that are not synthesized by the body but that are necessary
for proper body function. Therefore, essential nutrients must be consumed. Vitamin
A is critical to the development of bones, teeth, and skin; it helps maintain eyesight
and enhances the immune system, fetal development, and gene expression. Vitamin
D is critical for calcium absorption, which is necessary for bone development and
strength; it also maintains a stable nervous system and a normal and strong
heartbeat and helps in blood clotting. Vitamin E lessens oxidative damage of cells
and prevents lung damage from pollutants; it is also vital to the immune system.
Vitamin K is essential for blood clotting.
44 What happens to glycogen when blood sugar drops?
A It stimulates the release of insulin, which can regulate blood sugar levels.
B It is released from the liver and converted to glucose to increase blood sugar levels.
C It is converted to starch, which breaks down to form glucose and increase blood sugar
levels.
D It is released from the liver and converted to pyruvate, which can then form glucose to
increase blood sugar levels.
Solution The solution is (B). When blood sugar drops, the liver—one place where glycogen is
stored—releases glycogen, which is then converted to glucose to increase blood
sugar levels.

Advanced Placement Biology Instructor’s Solution Manual


638 25 | Animal Nutrition and the Digestive System

45 What is the evolutionary significance of glycogen production?


A Excess ATP and glucose produce glycogen, which can be used at later to act as
cofactor if, for example, a good source is scarce.
B Excess proteins and glucose produce glycogen, which can be used at later to produce
energy if, for example, food is scarce.
C Excess ATP and glucose produce glycogen, which can be used at later to produce
energy if, for example, food is scarce.
D Excess proteins and fats produce glycogen, which can be used at later to act as source
of nitrogen if, for example, a good source is scarce.
Solution The solution is (C). Excess ATP and glucose stimulate the production of glycogen,
which can be used to produce energy at a later time—for example, if food is scarce.
46 How can eating too much bread and pasta physiologically promote obesity?
A Excess blood glucose increases the amount of urea, which is converted into fatty acids.
Fatty acids are stored in areolar cells, which increase the amount of body fat.
B Excess blood glucose increases the amount of pyruvate, which is converted into fatty
acids. Fatty acids are stored in adipose cells, which increase the amount of body fat.
C Bread and pasta are rich in fats. Their digestion produces fatty acids and glycerol. Fatty
acids are stored in adipose cells, which increase the amount of body fat.
D Bread and pasta are rich in fats. Their digestion produces fatty acids and glycerol. Fatty
acids are stored in areolar cells, which increase the amount of body fat.
Solution The solution is (B). Glycolysis produces pyruvate. Excess blood glucose from
digesting carbohydrates increases the amount of pyruvate, which is converted into
fatty acids. Fatty acids are stored in adipose cells, which increase the amount of
body fat.
47 How do ingestion and digestion differ?
A Ingestion is the process of taking food in through mouth, where mechanical digestion
begins. Chemical digestion begins in the stomach, where food is further broken down
into smaller molecules that can be absorbed and used by the body.
B Ingestion is the process of taking food in through the mouth, where mechanical and
chemical digestion begin to break down the food into smaller molecules that can be
absorbed and used by the body.
C Ingestion is the process of taking food in through the mouth, where mechanical and
chemical digestion begins. Digestion in the stomach breaks down proteins and fats
present in food into smaller molecules that can be absorbed and used by the body.
D Ingestion is the transfer of food from the mouth to the esophagus, where mechanical
and chemical digestion begin to break down the food into smaller molecules that can
be absorbed and used by the body.

Advanced Placement Biology Instructor’s Solution Manual


25 | Animal Nutrition and the Digestive System 639

Solution The solution is (B). Ingestion is the process of taking food in through the mouth,
where mechanical and chemical digestion begin to break down the food into smaller
molecules that can be absorbed and used by the body.
48 Why are some dietary lipids a necessary part of a balanced diet?
A Dietary lipids aid in the absorption of water-soluble vitamins, including B and C, which
are needed for various bodily functions.
B Dietary lipids aid in the absorption of some minerals, including folic acid, iron, and
magnesium, which are needed for various bodily functions.
C Dietary lipids aid in the absorption of vitamins, including A, B, C, D, E, and K, which are
needed for various bodily functions.
D Dietary lipids aid in the absorption of fat-soluble vitamins, including A, D, E, and K,
which are needed for various bodily functions.
Solution The solution is (D). Dietary lipids aid in the absorption of fat-soluble vitamins
because these vitamins dissolve in the fats and are transported with the fats into the
digestive system. The fat-soluble vitamins are vitamins A, D, E, and K, which are
needed for various bodily functions.
49 What happens to undigested food after the water is reabsorbed?
A Undigested food is moved through the colon, where intestinal flora aid in digestion by
peristalsis; the food is then stored in the rectum until it is eliminated through the
anus.
B Undigested food is moved through the colon, where intestinal flora aid in digestion by
peristalsis; further absorption takes place in the rectum, where the food is stored until
it is eliminated through the anus.
C Undigested food is moved through the colon, where intestinal flora aid in digestion by
segmentation; the food is then stored in the rectum until it is eliminated through the
anus.
D Undigested food is moved through the ileum, where intestinal flora aid in digestion by
peristalsis; the food is then stored in the rectum until it is eliminated through the
anus.
Solution The solution is (A). The undigested food is moved through the colon, where
intestinal flora aid in digestion by peristalsis, which are wave-like muscle
contractions; the food is then stored in the rectum until it is eliminated through
the anus.

Advanced Placement Biology Instructor’s Solution Manual


640 25 | Animal Nutrition and the Digestive System

50 What are micelles? Why are micelles integral to lipid absorption?


A Micelles are lipoproteins designed for the transport of lipids that enter lacteals.
Micelles facilitate absorption by microvilli, where the fatty acids and proteins diffuse
out to form lipoproteins.
B Micelles are lipoproteins designed for the transport of lipids that enter lacteals.
Micelles facilitate absorption by microvilli, where the fatty acids and monoglycerides
diffuse out to form triglycerides.
C Micelles are fatty acids and phospholipids surrounded by bile salt. Micelles facilitate
absorption by microvilli, where the fatty acids and monoglycerides diffuse out to form
triglycerides.
D Micelles are fatty acids and monoglycerides surrounded by bile salt. Micelles facilitate
absorption by microvilli, where the fatty acids and monoglycerides diffuse out to form
triglycerides.
Solution The solution is (C). Micelles are fatty acids and monoglycerides surrounded by bile
salt. In the digestive system, they do not contain protein components. Micelles
facilitate absorption by microvilli, where the fatty acids and monoglycerides diffuse
out to form triglycerides. However, they do not directly interact with the lacteals of
the microvilli.
51 On a cellular level, why must food be broken down?
A Large molecules present in intact food pass through the digestive epithelium and
enter the cell through the nuclear membrane, thereby damaging the membrane.
Hence the molecules must be broken down.
B Fats present in intact food contain very large molecules, which cannot pass through
cell membranes. Fats need to be passed through the digestive epithelium to be
utilized.
C Large molecules present in intact food cannot pass through cell membranes. Nutrients
need to be passed through the digestive epithelium to be utilized.
D Large molecules, if not broken down, produce toxic substances that pass through
the epithelium of the digestive tract and are absorbed by cells. This can be lethal
to the cell.
Solution The solution is (C). Large molecules of proteins, carbohydrates, and lipids in intact
food cannot pass through cell membranes. Nutrients need to be passed through the
digestive epithelium to be utilized.
52 What is the importance of neural responses to food stimuli?
A Neural responses facilitate secretion of fumarase needed for chemical digestion of
food as well as other involuntary responses like peristalsis.
B Neural responses facilitate secretion of enzymes needed to digest or break down food
as well as to enable other involuntary responses like segmentation in the stomach.

Advanced Placement Biology Instructor’s Solution Manual


25 | Animal Nutrition and the Digestive System 641

C Neural responses facilitate secretion of enzymes needed to digest or break down food
as well as to enable other involuntary responses like peristalsis.
D Neural responses facilitate secretion of salivary amylase needed to digest or break
down food as well as to enable secretion of hormones like secretin and gastrin.
Solution The solution is (C). Neural responses facilitate secretion of enzymes that are needed
to digest, or break down, the food as well as to enable other involuntary responses
such as peristalsis, which are wave-like muscle contractions.
53 How do hormones regulate digestion?
A Hormones regulate aspects of digestion such as increasing the peristaltic movements
in the esophagus when food is sensed.
B Hormones regulate digestion by signaling when the stomach is full or empty so that an
individual will consume food or stop eating.
C Hormones like gastrin, secretin, and adrenocorticotropic are released from the
pituitary to regulate which digestive secretions are released.
D Hormones regulate aspects of digestion such as which digestive secretions are
released, as well as when they are released.
Solution The solution is (D). Hormones are signal molecules that regulate aspects of
digestion, such as which digestive secretions are released, as well as when they are
released.
54 When you are eating a meal, how do you know when you are full?
A The pituitary gland release hormones when the stomach is full, which therefore
reduces hunger.
B The brain signals that you are satiated when the stomach is full, which therefore
reduces hunger.
C The stomach signals when it is full, which therefore reduces hunger.
D Low blood-sugar levels stimulate a neurotransmitter, which sends a signal to the brain
when the stomach is full and therefore reduces hunger.
Solution The solution is (B). When your stomach is full, the brain signals that you are satiated
and therefore reduces hunger.

TEST PREP FOR AP® COURSES


55 Simple cuboidal epithelial cells line the ducts of certain human exocrine glands. Various
materials are transported into or out of the cells by diffusion. The formula for the surface
area of a cube is 6 × S2, and the formula for the volume of a cube is S3, where S is the
length of a side of a cube. Which cube-shaped cell in the figure would be most efficient in
removing waste by diffusion?

Advanced Placement Biology Instructor’s Solution Manual


642 25 | Animal Nutrition and the Digestive System

A 10 μm
B 20 μm
C 30 μm
D 40 μm
Solution The solution is (A). The 10-µm cell would be the most efficient at removing waste by
diffusion, because it is the smallest option. Smaller cubes have a greater surface
area-to-volume ratio, which allows more nutrient molecules to pass through.
56 Celiac disease is dangerous in affected individuals because ingesting gluten damages the
villi of the small intestine. Why is this potentially life threatening?
A Villi aid in mechanical digestion of food particles. When they are damaged, nutrients
cannot be digested properly in the body.
B Villi increase the surface area of the small intestine, which aids in the absorption of
bile salts. This nutrient cannot be absorbed when villi are damaged.
C Villi decrease the small intestine’s surface area that is available for absorption.
Nutrients cannot properly enter the bloodstream when they are damaged.
D Villi increase the surface area available for nutrient absorption. When villi are
damaged, nutrients cannot properly enter the bloodstream.
Solution The solution is (D). Villi and microvilli increase the amount of surface area available
for nutrient molecules to pass through. When villi and microvilli are damaged,
nutrients cannot properly enter the bloodstream.
57 One of the key features of villi and microvilli in the digestive system is their finger-like
projection shape. What is an example of how the shape of microvilli can enhance nutrient
absorption?
A Nutrients can enter the bloodstream through the blood vessels that are located in the
middle of the microvilli.
B Larger microvilli have more surface area over which more nutrients are absorbed.

Advanced Placement Biology Instructor’s Solution Manual


25 | Animal Nutrition and the Digestive System 643

C The microvilli projections aid in the mechanical digestion of food particles.


D The finger-like projections prevent large particles of food from passing through the
digestive system.
Solution The solution is (A). The finger-like projection shape can enhance nutrient absorption
by facilitating how nutrients enter the bloodstream through the blood vessels in the
middle of the microvilli and villi. Lymphatic and blood vessels run through the
microvilli and villi, and these structures enable nutrients to enter the bloodstream
after absorption.
58 Microvilli greatly increase the efficiency of nutrient uptake in the small intestines. How do
the size and shape of microvilli promote this efficiency?
A They have a greater surface area-to-volume ratio than larger cells. The finger-like
projection shape provides more surface area over the small intestines from which they
absorb nutrients and contains blood vessels so nutrients passing through them can
enter the bloodstream readily.
B They have a greater surface area-to-volume ratio than larger cells. The finger-like
projection shape is present in the middle of microvilli, which have more surface area
over the small intestines from which they absorb nutrients and also contains blood
vessels so nutrients can enter the blood easily.
C They have a greater surface area-to-volume ratio than larger cells. The finger-like
projections prevent large particles of food from passing through the digestive system
and also contain blood vessels so nutrients passed through them can readily enter the
bloodstream.
D They have a greater surface area-to-volume ratio than larger cells. The finger-like
projections aid in mechanical digestion of food particles and contain blood vessels so
nutrients passing through them can enter the bloodstream readily.
Solution The solution is (A). Microvilli are small cells, which means that, compared to larger
cells, they have a greater ratio of surface area to volume. The finger-like projection
shape allows microvilli to provide more surface area over the small intestines from
which they absorb nutrients. The projections also contain blood vessels, so nutrients
passed through the microvilli can readily enter the bloodstream.
59 Birds have several unique physical differences from other vertebrates, and several pertain
to how birds process food. Some differences are obvious, such as the presence of a beak
and no teeth, whereas other differences can be observed only in their internal features.
For example, like most other vertebrates, birds have a monogastric digestive system;
however, the structure of a bird’s digestive system differs from that of most other
monogastric vertebrates. Which statement is true about how birds process food?

Advanced Placement Biology Instructor’s Solution Manual


644 25 | Animal Nutrition and the Digestive System

A Beak emergence coincided with insect inclusion in the bird diet.


B The gizzard is the primary site of mechanical digestion.
C Birds excrete nitrogenous waste and feces through separate openings.
D Birds digest plant material more slowly than ruminants do.
Solution The solution is (B). Birds use the gizzard as the primary site of mechanical digestion.
Birds swallow stones and grit, which grind food in the gizzard.
60 As shown in this figure, the oral cavity has several components that contribute to
ingestion and the initial stages of digestion. How do the components of the oral cavity
work together to complete the first step of food processing?

A The teeth and jaw mechanically chew the food, and saliva from the salivary glands
moistens the food and begins chemical digestion. The tongue then physically moves
the food to the pharynx, where peristalsis moves the food into the stomach.
B The teeth and jaw mechanically chew the food, and saliva from the salivary glands
moistens the food and initiates mechanical and chemical digestion. The tongue then
physically moves the food to the pharynx, where peristalsis moves the food into the
stomach.
C The teeth and jaw mechanically chew the food, and saliva from the salivary glands
moistens the food and begins chemical digestion. The tongue then physically moves
the food to the larynx, where peristalsis moves the food into the stomach.
D The teeth and jaw mechanically chew the food, and saliva from the salivary glands
moistens the food and initiates mechanical and chemical digestion. The tongue then
physically moves the food to the pharynx, where segmentation moves the food into
the stomach.
Solution The solution is (A). The teeth and jaw mechanically chew the food, which is made
easier by the release of enzyme-rich saliva from salivary glands to moisten the food
and begin chemical digestion. The tongue then physically moves the food to the
pharynx, where wave-like muscle movements (peristalsis) move the food into the
stomach.

Advanced Placement Biology Instructor’s Solution Manual


25 | Animal Nutrition and the Digestive System 645

61 Most mammals have a monogastric digestive system, which means they have one
stomach chamber. Ruminants and pseudo-ruminants consume a large amount of plant
material and have polygastric digestive systems, which means they have more than one
stomach chamber. Why is an increased number of stomach chambers beneficial for
ruminants and pseudo-ruminants?
A Microbes in the chambers break down and ferment plant material.
B Extended exposure to stomach acid breaks down more cellulose.
C Increased amounts of peristalsis crush more of the plant fibers.
D Having more stomach chambers increases exposure for nutrients to be absorbed.
Solution The solution is (A). The different chambers contain microbes that have cellulase,
which breaks down cellulose, and the nutrients can subsequently be absorbed.
62 This figure shows the majority of the digestive tracts of two organisms that consume
different food sources.

Advanced Placement Biology Instructor’s Solution Manual


646 25 | Animal Nutrition and the Digestive System

Which digestive tract belongs to the herbivore? How did you determine this?
A The top digestive tract belongs to the herbivore. Herbivores have a shorter intestinal
tract, which allows stronger smooth muscle contractions called peristalsis in a shorter
area, providing more opportunity for nutrients to be obtained and absorbed.
B The bottom digestive tract belongs to the herbivore. Herbivores have a longer
intestinal tract, which provides more opportunity for nutrients to be obtained and
absorbed, since plant material is difficult for animals to break down.

Advanced Placement Biology Instructor’s Solution Manual


25 | Animal Nutrition and the Digestive System 647

C The bottom digestive tract belongs to the herbivore. Herbivores have a longer
intestinal tract, which provides more opportunity for the nutrients to react with the
intestinal enzymes for better absorption, since plant material is difficult for animals to
break down.
D The top digestive tract belongs to the herbivore. Herbivores have a shorter intestinal
tract, which provides more opportunity for nutrients to be obtained and absorbed,
since plant material is difficult for animals to break down.
Solution The solution is (B). The herbivorous digestive tract is shown at the bottom. The
digestive tract shown at the bottom has a longer intestinal tract. Plant material is
difficult for animals to break down, so a longer intestinal tract provides more
opportunity for nutrients to be obtained and absorbed.
63 The ruminant digestive system has evolved several differences from the traditional
mammalian monogastric digestive system because they consume large amounts of plant
material. Which of the following is NOT a component of the ruminant digestive system
that has evolved to more efficiently digest plant fibers?
A Omasum
B Abomasum
C Reticulum
D Gizzard
Solution The solution is (D). The gizzard is found in birds and some invertebrates and is the
primary site of mechanical digestion in these organisms.
64 This figure shows the three main components of the large intestine.

How do these three parts contribute to processing as food material passes through the
large intestine?
A The cecum receives semisolid waste from the small intestine and absorbs water,
vitamins, and minerals. Then, the colon further digests some material. The rectum
stores the fecal matter until it is excreted.

Advanced Placement Biology Instructor’s Solution Manual


648 25 | Animal Nutrition and the Digestive System

B The cecum receives semisolid waste from the small intestine. Then the colon digests
some materials. The rectum absorbs water and some vitamins and minerals and then
stores the fecal matter until it is excreted.
C The cecum receives semisolid waste from the small intestine. Then, the colon absorbs
water and some vitamins and minerals and further digests some material. The rectum
stores the fecal matter until it is excreted.
D The cecum receives semisolid waste from the small intestine. The colon is the only
region where absorption of vitamins takes place in the digestive system. The rectum
stores the fecal matter until it is excreted.

Advanced Placement Biology Instructor’s Solution Manual


25 | Animal Nutrition and the Digestive System 649

Solution The solution is (C). The cecum receives semisolid waste from the small intestine.
Then, the colon absorbs water and some vitamins and minerals, and the intestinal
flora of the colon further digest some material. The rectum stores the fecal matter
until it is excreted.
65 This figure shows involuntary muscle movement in part of the digestive system. What
stimulates this involuntary response?

A Smelling food
B Seeing food
C Chewing food
D Swallowing food
Solution The solution is (D). Swallowing food promotes an involuntary reflex called peristalsis
that moves food from the oral cavity to the stomach.
66 This figure shows the digestive system of a ruminant animal. How does this polygastric
digestive system enhance digestion efficiency in ruminants?

A Multiple stomach chambers in ruminant animals contain microbes that have cellulase,
which breaks down plant material. Plant material is difficult to digest because animals
lack cellulase to break down cellulose.

Advanced Placement Biology Instructor’s Solution Manual


650 25 | Animal Nutrition and the Digestive System

B Multiple stomach chambers in ruminant animals allow stronger smooth muscle


contractions, which break down plant material. Plant material is difficult to digest
because animals lack cellulase to break down cellulose.
C Multiple stomach chambers in ruminant animals contain cellulase, which breaks down
plant material. Plant material is difficult to digest because individual stomach
chambers lack sufficient cellulase to break down cellulose.
D Multiple stomach chambers in ruminant animals allow the food to stay in the stomach
for a longer time so that peristaltic movements and the action of enzymes on food
particles occurs for a longer time.
Solution The solution is (A). Ruminants consume large amounts of plant material, which is
difficult to digest because animals lack cellulase to break down cellulose. The
multiple stomach chambers contain microbes that have cellulase and thus can break
down the plant material. Consequently, the plant nutrients and nutrients from
ruminant microbes can be absorbed.
67 An experiment to measure the rate of respiration in mice at 10 °C and 25 °C was
performed using a respirometer, an apparatus that measure changes in gas volume.
Respiration was measure in milliliters of O2 consumed per gram of organism over several
5-min trials, and the following data were obtained.

According to these data, the mouse at 10 °C demonstrated greater oxygen consumption


per gram of tissue than the mouse at 25 °C demonstrated. Which statement best explains
this observation?
A The mouse at 10 °C had a higher rate of ATP production than the mouse at 25 °C had.
B The mouse at 10 °C had a lower metabolic rate than the mouse at 25 °C had.
C The mouse at 25 °C weighed less than the mouse at 10 °C weighed.
D The mouse at 25 °C was more active than the mouse at 10 °C was.
Solution The solution is (A). Cold environments require greater amounts of energy to
maintain homeostasis because organisms have to carry out metabolic or body
activities that produce heat.
68 ATP is essential for organisms because it provides energy to cells. How does ATP provide
this energy on a physiological level?
A When energy is needed, ATP is converted to ADP and a phosphate group. Energy is
released from the breaking of the phosphodiester bonds.

Advanced Placement Biology Instructor’s Solution Manual


25 | Animal Nutrition and the Digestive System 651

B When energy is needed, ATP is converted to ADP and a phosphate group. Energy is
released from the breaking of the glycosidic bonds.
C When energy is needed, ATP is formed from ADP and a phosphate group. Energy is
released from the breaking of the phosphodiester bonds.
D When energy is needed, ATP is formed from ADP and a phosphate group. Energy is
released from the breaking of the phosphoanhydride bonds.
Solution The solution is (A). The release of phosphate from ATP releases energy. When
energy is needed, ATP is converted to ADP and a phosphate group. Consequently,
energy is released from the breaking of the phosphodiester bonds.
69 An omnivore comes across potatoes, avocados, kale, and eggs and craves only the eggs.
The animal is likely deficient in which nutrient?
A Carbohydrates
B Protein
C Fiber
D Fatty acids
Solution The solution is (B). Eggs are very high in protein, so consuming eggs would help
remedy a protein deficiency.
70 Carbohydrates often get a bad reputation for their role in promoting weight gain when
consumed in excess. However, carbohydrates are necessary for biological functions. Why
is it important to consume carbohydrates?
A Carbohydrates are broken down into glucose, which provides energy as ATP through
metabolic pathways. ATP helps to maintain connective tissue.
B Carbohydrates are broken down into glucose, which is essential for blood clotting.
C Carbohydrates, along with proteins, help maintain connective tissue and are essential
to blood clotting.
D Carbohydrates are broken down into glucose, which provides energy as ATP through
metabolic pathways. ATP is required for proper cellular function.
Solution The solution is (D). Carbohydrates are broken down into glucose, which provides
energy as ATP through metabolic pathways. ATP is the energy source of cells and is
required for proper cellular function.
71 Excess ATP is combined with excess glucose and stored as glycogen in the liver and in
skeletal muscle. Under which circumstance would glycogen storage in skeletal muscle
prove beneficial for a rabbit?
A A rabbit has not eaten recently, and its blood sugar drops.
B There is an overabundance of food available to a rabbit.
C A rabbit spots a coyote and flees in response.

Advanced Placement Biology Instructor’s Solution Manual


652 25 | Animal Nutrition and the Digestive System

D A young rabbit with an adequate food source is developing into an adult rabbit.
Solution The solution is (C). Glycogen is released from skeletal muscle for use during intense
exercise.

SCIENCE PRACTICE CHALLENGE QUESTIONS


25.1 Digestive Systems
72 E. coli colonize the human gastrointestinal tract. The temperature of that environment is
tightly regulated. However, the pH ranges from the highly acidic stomach (pH 4.5) to the
relatively basic lower intestine (pH 9). Over the entire pH range of the environment, the
pH of the E. coli cytoplasm is maintained in a narrow range between 7.2 and 7.8. Wilks
and Slonczewski (Journal of Bacteriology, 189, 2007) used a fluorescent dye to follow the
recovery of cytoplasmic pH after an acid shock comparable to what occurs in the human
stomach. They found that the pH within the cell recovered in approximately 2 min.
Rapid restoration of cytoplasmic pH does not occur in the presence of ATPase inhibitors.
Explain the mechanisms that maintain homeostasis with a model of exchange of
hydrogen ions (H+) between the cell and its extracellular environment.
Solution Sample answer: In response to a sudden increase in the extracellular concentration
of hydrogen ions, transport of these ions from within the cell to maintain
homeostasis must occur against a gradient. This is accomplished as an outcome of
ATP synthesis in respiration. But the fact that inhibition of ATPase alters the
response shows that active transport is occurring. The glutamate coupled pump that
is largely responsible is described by Krulwich et al. (Nature Reviews, 9, 2011).
25.2 Nutrition and Energy Production
73 We need an explanation of the common experience that an "upset stomach" (functional
dyspepsia) or constipation can result from stress. Irritable bowel syndrome is a chronic
gastrointestinal disease; it is treated with the neurotransmitter serotonin. Serotonin
receptors are located on the cell membranes of neurons and activate second messenger
cascades that regulate gene expression.
A. In humans, most serotonin is synthesized in neurons that enervate smooth muscle
cells lining the gastrointestinal tract. There is an association of serotonin with a sense
of well-being.
Based on these data, justify the claim that serotonin regulates the timing of the passage
of food in the gastrointestinal tract.
B. The effect of serotonin on smooth muscle is a clue, but it doesn't provide a mechanism
connecting stress to the symptoms of functional dyspepsia. Serotonin is synthesized by all
Bilateria (animals with bilateral symmetry, including humans) and is released as a
response to stress (Puglisi-Allegra and Andolini, Behavioral Brain Research, 277, 2015).
Serotonin is also synthesized by plants to regulate root growth. Describe the role for
serotonin that is indicated across domains.

Advanced Placement Biology Instructor’s Solution Manual


25 | Animal Nutrition and the Digestive System 653

C. Evaluate the effect that stress produces in serotonin production, the association of
stress with functional dyspepsia, and the role of serotonin in the regulation of expression
in smooth muscle cells in terms of evidence of a negative feedback produced by serotonin
as a medication.
D. Justify your evaluation of the stress and the role of serotonin as a response to the
stress in the form of a feedback loop diagrammatically.
E. Quorum sensing coordinates bacterial expression, stimulating virulence factors; it also
coordinates behavior, inducing the formation of biofilms. Knecht et al. (EBioMedicine, 9,
2016) have demonstrated that serotonin functions as a quorum-sensing messenger
among bacteria in the gastrointestinal tract of mice.
Construct an explanation of the effect of serotonin as a treatment of functional
dyspepsia.
Solution Sample answer:
A. Passage through the gastrointestinal tract is actively controlled. This means that
the smooth muscles lining the tract must be responsible; the fact that these muscles
are activated by serotonin establishes the role of the neurotransmitter. Where else
should the chemical message relaying the effect of a good meal originate?
B. The role of serotonin indicated is cell-cell communication.
C. If stress increases serotonin levels, then use of serotonin in the treatment of the
effect of stress is not displaying a regulation based on negative feedback.
D. The diagram should show the disturbance of homeostasis with an appropriate
label and indicate that, according to the data provided, serotonin would amplify
rather than suppress the stress, and so would be a positive rather than a negative
feedback.
E. The effect indicated is that serotonin treatment is disrupting the quorum sensing
of bacteria in the gastrointestinal tract. The explanation then is that quorum-sensing
messengers used by the bacteria cannot function where multiple or conflicting
messengers are present.

Advanced Placement Biology Instructor’s Solution Manual


ne System

28 | The Endocr658 System PAGE \* MERGEFOR

26 | THE NERVOUS SYSTEM


REVIEW QUESTIONS
1 Where are parasympathetic preganglionic cell bodies located?
A Cerebellum
B Brainstem
C Dorsal root ganglia
D Spinal cord
Solution The solution is (B). Parasympathetic preganglionic cell bodies are located in the
brainstem.
2 Which statement about the parasympathetic nervous system is true?
A Controls fight-or-flight response
B Can reset organ function to the normal range
C Transmits information from the skin to the central nervous system
D Stimulates glycogen breakdown
Solution The solution is (B). The parasympathetic nervous system can reset organ function to
the normal range.
3 Proper nervous system function involves various types of organic molecules. In particular,
what is released by motor nerve endings onto muscle cells or tissue?
A Acetylcholine
B Norepinephrine
C Dopamine
D Serotonin
Solution The solution is (A). Acetylcholine, the main neurotransmitter released by motor
neurons, is released onto target organs such as muscle.
4 If the sensory-somatic nervous system of an animal is damaged, what might happen?
A Enhanced processing of environmental information
B Decreased digestion ability
C Perpetually low heart rate
D Impaired control of motor movements
Solution The solution is (D). If the sensory-somatic nervous system of an animal is damaged,
motor movement control might be impaired because both sensory and motor
movements could be impaired by damage to the sensory-somatic nervous system.

Advanced Placement Biology Instructor’s Solution Manual


AT 733658

655 29 | The Musc659skel


5 The nervous system regulates proper processing of information and behavior control.
Which nervous system includes the parasympathetic and sympathetic nervous systems?
A Autonomic
B Sensory-somatic
C Central
D Fight or flight
Solution The solution is (A). The parasympathetic and sympathetic nervous systems are part
of the autonomic nervous system, which is part of the peripheral nervous system.
6 Medications can be used to treat certain neurodevelopmental disorders. For example,
which medications are often used to treat patients with ADHD?
A Tranquilizers
B Blood pressure medications
C Stimulants
D Anticonvulsant medications
Solution The solution is (C). Stimulants have the opposite effect on individuals with ADHD and
thus can be used to treat ADHD symptoms.
7 If a child appears to have impaired social skills, such as difficulty reading social cues or
making eye contact, what might they be tested for?
A Major depression
B Attention deficit hyperactivity disorder (ADHD)
C Schizophrenia
D Autism spectrum disorder
Solution The solution is (D). A child with difficulty reading social cues or making eye contact
might be tested for autism spectrum disorder, which is characterized by symptoms
that include impaired social skills.
8 Parkinson’s disease is a neurodegenerative disease that can produce symptoms such as
tremors, slowed movement, speech changes, balance and posture problems, and rigid
muscles. Parkinson’s disease is caused by the degeneration of neurons that release —
A serotonin
B dopamine
C glutamate
D norepinephrine
Solution The solution is (B). Parkinson’s disease is caused by the degeneration of neurons
that release dopamine.

Advanced Placement Biology Instructor’s Solution Manual


ne System

28 | The Endocr658 System PAGE \* MERGEFOR

CRITICAL THINKING QUESTIONS


9 When you stick your hand in a bucket of ice, it grows numb after a while. Based on what
you know about neuronal signaling, how is the sensation of touch blocked from signaling
to the brain?
Solution Sample answer: Numbness occurs because the signal between neurons is disrupted.
In cold conditions, blood flow to the nerve cells will be limited and the signal will be
disrupted.
10 Lidocaine is a local anesthetic that works by blocking voltage-gated sodium channels.
Explain how blocking voltage-gated sodium channels would cause numbness and pain.
Solution Voltage-gated sodium channels are responsible for depolarizing sensory neurons.
When blocked, signals from sensory neurons, such as pain receptors, cannot be
transmitted to the brain. This numbs the sensation of pain.
11 What are the main differences between the sympathetic and parasympathetic nervous
systems?
A The sympathetic nervous system is activated by stressful situations, whereas the
parasympathetic nervous system resets organ function of sympathetic reactions and
allows animals to “rest and digest.”
B The parasympathetic nervous system is activated by stressful situations, whereas the
sympathetic nervous system resets organ function of sympathetic reactions and allows
animals to “rest and digest.”
C The sympathetic nervous system is involved in unconscious body function control,
whereas the parasympathetic nervous system is involved in conscious body function
control.
D The parasympathetic nervous system is involved in unconscious body function control,
whereas the sympathetic nervous system is involved in conscious body function
control.
Solution The solution is (A). Although both are involved in unconscious body function control,
the sympathetic nervous system is activated by stressful situations, whereas the
parasympathetic nervous system resets organ function of sympathetic reactions and
allows animals to “rest and digest.”
12 How is the sensory-somatic nervous system involved in sensing information and motor
function?
A The sensory-somatic nervous system transmits information from the skin, muscles,
and sensory organs to the peripheral nervous system. Motor information is sent to
and from the central nervous system and the muscles.
B The sensory-somatic nervous system transmits information from the skin, muscles,
and sensory organs to the central nervous system. Motor information is sent to and
from the central nervous system and the muscles.

Advanced Placement Biology Instructor’s Solution Manual


AT 733658

657 29 | The Musc659skel

Advanced Placement Biology Instructor’s Solution Manual


ne System

28 | The Endocr658 System PAGE \* MERGEFOR


C The sensory-somatic nervous system transmits information from the skin, muscles,
and sensory organs to the central nervous system. Motor information is sent to and
from the peripheral nervous system and the muscles.
D The sensory-somatic nervous system transmits information from the skin, muscles,
and sensory organs to the peripheral nervous system. Motor information is sent to
and from the peripheral nervous system and the muscles.
Solution The solution is (B). The sensory-somatic nervous system transmits information from
the skin, muscles, and sensory organs to the central nervous system. Motor
information is sent to and from the central nervous system and the muscles.
13 Public speaking can be very stressful. How can anticipating giving a public speech
stimulate the sympathetic nervous system?
A During stress, multiple preganglionic neurons can synapse on one postganglionic
neuron, and the adrenal gland releases adrenaline.
B During stress, one preganglionic neuron can synapse on multiple postganglionic
neurons, and the thymus gland releases norepinephrine.
C During stress, one postganglionic neuron can synapse on multiple preganglionic
neurons, and the adrenal gland releases norepinephrine.
D During stress, one preganglionic neuron can synapse on multiple postganglionic
neurons, and the adrenal gland releases norepinephrine.
Solution The solution is (D). When an individual experiences stress, one preganglionic neuron
can synapse on multiple postganglionic neurons, and the adrenal gland releases
norepinephrine. This is a sympathetic nervous system response.
14 What might make you suspect that an individual has Alzheimer’s disease?
A Disruptive memory loss, confusion about time or place, difficulty with planning and
executing tasks, poor judgment, and/or personality changes
B Slowed movements, balance and posture problems, rigid muscles, speech changes,
and/or psychological symptoms such as dementia
C Impaired social skills, repetitive motor behaviors, strict adherence to certain rituals,
and preoccupation with specific subjects
D Balance and posture problems, repetitive motor behaviors, difficulty with planning
and executing tasks, poor judgment, and/or personality changes
Solution The solution is (A). Disruptive memory loss, confusion about time or place, difficulty
with planning and executing tasks, poor judgment, and/or personality changes are
common symptoms of Alzheimer’s disease.

Advanced Placement Biology Instructor’s Solution Manual


AT 733658

659 29 | The Musc659skel


15 What treatment options are available for an individual diagnosed with major depression?
A Blood pressure medication, deep-brain stimulation, taking monoamine oxidase
inhibitors, psychotherapy, and physical therapy
B Psychotherapy, electroconvulsive therapy, deep-brain stimulation, taking monoamine
oxidase inhibitors, and/or taking selective melatonin reuptake inhibitors
C Psychotherapy, electroconvulsive therapy, deep-brain stimulation, taking monoamine
oxidase inhibitors and/or taking selective serotonin reuptake inhibitors
D Blood pressure medication, classes of antipsychotics, psychotherapy,
electroconvulsive therapy, deep-brain stimulation, and/or taking selective serotonin
reuptake inhibitors
Solution The solution is (C). Some common treatments for major depression include
psychotherapy, electroconvulsive therapy, deep-brain stimulation, taking
monoamine oxidase inhibitors, and/or taking selective serotonin reuptake inhibitors.

TEST PREP FOR AP® COURSES


16 If a neuron has damaged synapses, what would be impaired?
A Integration of signals from several synapses
B Speed of signal transduction
C Receiving signals from other neurons
D Ability to recharge electrical signals
Solution The solution is (C). If a neuron has damaged synapses, signal reception from other
neurons would be impaired. Synapses are essential for sending and receiving signals
between cells in the nervous system.
17 Signal transmission from one neuron to another requires a series of processes pertaining
to different components of each neuron. What happens at the axon terminals to facilitate
signal transmission to another neuron?
A Chemicals released at the axon terminals transmit signals through synapses into other
neurons via the second neuron’s dendrites.
B Chemicals released at the axon terminals transmit signals through synapses into other
neurons via the second neuron’s axons.
C Chemicals released at the dendrites transmit signals through synapses into other
neurons via the second neuron’s axon terminal.
D Chemicals released at the axon terminals transmit signals directly into other neurons
via the second neuron’s axons.
Solution The solution is (A). Chemicals are released at the axon terminals, which transmit
signals through synapses into other neurons via the second neuron’s dendrites.

Advanced Placement Biology Instructor’s Solution Manual


ne System

28 | The Endocr658 System PAGE \* MERGEFOR


18 The figure shows a malformed neuron. Why would this neuron be nonfunctional?

A This neuron would not be able to receive signals.


B This neuron would not be able to recharge the signal.
C This neuron would not be able to integrate information from numerous synapses.
D This neuron would not be able to send signals.
Solution The solution is (D). This neuron would be nonfunctional because the neuron would
not be able to send signals. This neuron lacks axon terminals, and both axon and
axon terminals are responsible for sending signals to other cells.
19 This figure shows the transmission of a signal among a network of neurons.

How is a signal transferred from one neuron to another?

Advanced Placement Biology Instructor’s Solution Manual


AT 733658

661 29 | The Musc659skel


A A signal is released from an axon, passes through the axon terminal, and synapses
with dendrites. Dendrites receive the signal, which passes through the soma. Multiple
signals from a single synapse are integrated at the axon hillock, which then passes the
signal into the axon, where the signal is transferred to another cell.
B A signal is released from axon terminal, passes through the axon, and synapses with
dendrites. Dendrites receive the signal, which passes through the soma. Multiple
signals from multiple synapses are integrated at the axon hillock, which then passes
the signal into the axon, where the signal is transferred to another cell.
C A signal is released from an axon and passes through the axon terminal, which
synapses with dendrites. Dendrites receive the signal as it passes through the soma.
Multiple signals from multiple synapses are integrated at the axon hillock, which then
passes the signal into the axon, where the signal is transferred to another cell.
D A signal is released from the axon terminal, passes through the axon, and synapses
with dendrites. Dendrites receive the signal as it passes through the soma. Multiple
signals from a single synapse are integrated at the axon hillock, which then passes the
signal into the axon, where the signal is transferred to another cell.
Solution The solution is (B). A signal passes through the axon and is released from axon
terminals, which synapse with dendrites. Dendrites receive the signal, which passes
through the soma. Multiple signals from multiple synapses are integrated at the
axon hillock, and the signal is then passed into the axon, where the signal is
transferred to another cell.
20 Transmission of signals between two neurons requires proper communication between
neurons. Dendrites are a component of many neurons that facilitate signal reception.
What is true of dendrites?
A All neurons have several dendrites for signal reception.
B Dendritic spines decrease possible synaptic connections.
C Dendrites carry the signal to the soma.
D Chemical release at dendrites allows signal communication to other cells.
Solution The solution is (C). Dendrites receive signals from axons and carry them to the soma.

Advanced Placement Biology Instructor’s Solution Manual


ne System

28 | The Endocr658 System PAGE \* MERGEFOR


21 Resting membrane potential has a negative charge. Which ions correspond to each row of
data in the chart?

A Ion 1: , Ion 2: Na+, Ion 3: K+

B Ion 1: Na+, Ion 2: K+, Ion 3:

C Ion 1: K+, Ion 2: Na+, Ion 3:

D Ion 1: , Ion 2: K+, Ion 3: Na+


Solution The solution is (B). Because resting membrane potential has a negative charge, there
would be more K+ in the cell and more outside of the cell relative to Na+.
22 Voltage-gated ion channels are essential for producing an action potential and returning a
neuron to its resting state. Why would it be impossible to trigger an action potential
without voltage-gated ion channels?
A The cell would not undergo depolarization, which is necessary to fire an action
potential and then return the cell to the resting state.
B The cell would not undergo repolarization, which is necessary to fire an action
potential and then return the cell to the resting state.
C The cell would not undergo depolarization, repolarization, and hyperpolarization,
which are necessary to fire an action potential and then return the cell to the
resting state.
D The cell would not undergo depolarization and hyperpolarization, which are necessary
to fire an action potential and then return the cell to the resting state.
Solution The solution is (C). These channels open and close in response to stimuli. Without
these channels, the cell would not undergo depolarization, repolarization, and
hyperpolarization, which are necessary to fire an action potential and then return to
the resting state.

Advanced Placement Biology Instructor’s Solution Manual


AT 733658

663 29 | The Musc659skel

Advanced Placement Biology Instructor’s Solution Manual


ne System

28 | The Endocr658 System PAGE \* MERGEFOR


23 When an action potential is fired, what happens immediately after the peak action
potential occurs? Refer to the figure.

A Na+ channels open.


B K+ channels open.
C K+ channels close.
D Na+/K+ transporter restores resting potential.
Solution The solution is (B). When an action potential is fired, K+ channels open immediately
after the peak action potential occurs. This allows K+ ions to leave the cell.
24 Potassium channel blockers, such as amiodarone and procainamide, which are used to
treat abnormal electrical activity in the heart, impede the movement of K + through
voltage-gated K+ channels. Which part of the action potential would potassium channels
affect, and why?
A Depolarization after peak action potential would be affected because that is the point
when K+ begins to leave the cell.
B Repolarization after peak action potential would be affected because that is the point
when K+ begins to leave the cell.
C Repolarization after peak action potential would be affected because that is the point
when K+ begins to enter the cell.
D Polarization after peak action potential would be affected because that is the point
when K+ begins to enter the cell.
Solution The solution is (B). Repolarization after peak action potential would be affected
because that is the point when K+ begins to leave the cell.

Advanced Placement Biology Instructor’s Solution Manual


AT 733658

665 29 | The Musc659skel


25 This figure shows the transfer of an action potential through a neuron.

What is occurring in panel 3?


A Depolarization occurs closest to the cell body.
B The first part of the neuron cannot fire another action potential.
C The first part of the neuron can fire another action potential.
D Sodium channels have closed.
Solution The solution is (C). In panel 3, the first part of the neuron has reached resting state
again and can fire another action potential.
26 This figure depicts an essential component of signal formation and transmission in
neurons.

What is happening in this figure?

Advanced Placement Biology Instructor’s Solution Manual


ne System

28 | The Endocr658 System PAGE \* MERGEFOR


+ +
A A nerve impulse opens the Na channel, which makes Na enter the cell and
depolarizes the membrane.
B A nerve impulse opens the Ca2+ channel, which makes Ca2+ enter the cell and
depolarizes the membrane.
C A nerve impulse opens the Na+ channel, which makes Na+ enter the cell and
repolarizes the membrane.
D A nerve impulse opens the K+ channel, which makes K+ enter the cell and polarizes the
membrane.
Solution The solution is (A). A nerve impulse opens the Na+ channel, which makes Na+ enter
the cell and depolarizes the membrane.
27 Chemical and electrical synapses are two mechanisms by which signals can be transferred
between neurons. What occurs during chemical synapse?
A Repolarization occurs at the presynaptic membrane.
B Calcium influx causes synaptic vesicles to fuse to the membrane.
C Neurotransmitters diffuse out of gap junctions.
D Neurotransmitters bind to synaptic vesicles.
Solution The solution is (B). During chemical synapse, calcium influx causes synaptic vesicles
to fuse to the membrane. When this happens, neurotransmitters are released into
the synaptic cleft.
28 Chemical synapse is a multiple-step process in which neurotransmitters undergo transfer
and binding to different parts of the cell. What happens when a neurotransmitter binds to
ligand-gated ion channels?
A The ligand-gated ion channels open.
B The presynaptic neuron reuptakes the neurotransmitter.
C The neurotransmitter diffuses away from the synapse.
D The neurotransmitter is enzymatically degraded.
Solution The solution is (A). When a neurotransmitter binds to ligand-gated ion channels, the
ligand-gated ion channels open. After the ligand-gated ion channels open, the
neurotransmitter can be disposed of.
29 Different components of the brain control different parts of the body. One important part
of the brain is the occipital lobe. What might happen if an individual’s occipital lobe was
damaged?
A The individual would not feel hot or cold.
B The individual would be unable to form new memories.

Advanced Placement Biology Instructor’s Solution Manual


AT 733658

667 29 | The Musc659skel


C The individual would be unable to recognize certain objects.
D The individual would have no sense of smell.
Solution The solution is (C). If an individual’s occipital lobe was damaged, the individual
would be unable to recognize certain objects, because the occipital lobe contains
the visual cortex.
30 Both cerebral hemispheres are essential for proper body function. However, the left
cerebral hemisphere controls the right side of the body, whereas the right cerebral
hemisphere controls the left side of the body. Why is this the case?
A The descending neural connections are not switched in the brainstem, which means
that the neural connections of the left hemisphere are transmitted to the right side of
the body and vice versa.
B The ascending neural connections are not switched in the brainstem, which means
that the neural connections of the left hemisphere are transmitted to the right side of
the body and vice versa.
C The descending neural connections are switched in the brainstem, which means that
the neural connections of the left hemisphere are transmitted to the right side of the
body and vice versa.
D The ascending neural connections are switched in the brainstem, which means that
the neural connections of the left hemisphere are transmitted to the right side of the
body and vice versa.
Solution The solution is (D). The ascending neural connections are switched in the brainstem,
which means that the neural connections of the left hemisphere are transmitted to
the right side of the body and vice versa.

Advanced Placement Biology Instructor’s Solution Manual


ne System

28 | The Endocr658 System PAGE \* MERGEFOR


31 If an increased number of folds in the cortical sheets of the brain is associated with
increased social complexity, which animal has the greatest social complexity? Refer to
the figure.

A Rat
B Dolphin
C Chimpanzee
D Cat
Solution The solution is (B). The dolphin would have the greatest social complexity because
the dolphin brain has the most folds of the organisms shown.

Advanced Placement Biology Instructor’s Solution Manual


AT 733658

669 29 | The Musc659skel


32 This image shows a cross section of the spinal column.

How does gray matter facilitate communication along the spinal column?
A All myelin sheaths are located in the gray matter and transmit signals along the brain
and spinal cord through the gray matter.
B All synapses are located in the gray matter and transmit signals along the brain and
spinal cord through the gray matter.
C All synapses are located in the gray matter and transmit signals along the spinal cord
through the gray matter.
D All dendrites are located in the gray matter and transmit signals along the spinal cord
through the gray matter.
Solution The solution is (C). All synapses are located in the gray matter and transmit signals
along the spinal cord through the gray matter. This allows signals to be sent to and
from the brain to the rest of the body.
33 This figure depicts the parts of the body that are controlled by different parts of the motor
cortex.

What can be inferred about the organization of the motor cortex relative to the
organization of muscles in the body?

Advanced Placement Biology Instructor’s Solution Manual


ne System

28 | The Endocr658 System PAGE \* MERGEFOR


A The motor cortex is found throughout the body.
B Motor cortex neurons are generally located near neurons that control nearby
body parts.
C Motor cortex neurons control speaking and processing what an individual reads.
D The motor cortex controls involuntary muscle movements.
Solution The solution is (B). This figure shows that motor cortex neurons are generally
oriented by muscle group and located near neurons that control nearby body parts.
34 This figure represents a split-brain individual processing information.

What has happened to the brain of this individual? Why does the processing of
information occur as depicted?
A The parietal lobe has been cut, which severs the ability of the left hemisphere to
communicate but increases the ability of the right hemisphere.
B The corpus callosum has been cut, which severs the ability of the left hemisphere to
communicate but increases the ability of the right hemisphere.
C The frontal lobe has been cut, which severs the ability of the left and right
hemispheres to communicate.
D The corpus callosum has been cut, which severs the ability of the left and right
hemispheres to communicate.
Solution The solution is (D). The corpus callosum has been cut, which severs the ability of the
left and right hemispheres to communicate. In this image, visual input in the left
visual field enters the right hemisphere but cannot signal to the speech center.
35 The thalamus is part of the brain that is involved in various functions in the human body.
What might result from damage to an individual’s thalamus?
A Insomnia
B Lack of interest in everything

Advanced Placement Biology Instructor’s Solution Manual


AT 733658

671 29 | The Musc659skel


C Lack of fear
D Inability to learn new motor tasks
Solution The solution is (A). Damage to an individual’s thalamus could produce insomnia
because the thalamus regulates sleep state.

SCIENCE PRACTICE CHALLENGE QUESTIONS


26.1 Neurons and Challenge Questions
36 A neurotransmitter provides a chemical signal between neurons to inhibit or excite an
action potential.
A. Describe a model of this signaling, and in this description, include the roles played by
synapse, receptors, post- and pre-synaptic neurons, exocytosis, endocytosis, ligand-gated
ion channel, and the electric potential of the membrane.
B. Explain the stimulatory or inhibitory effect of key ionic elements, Na+ and Cl-, on the
electric potential of the post-synaptic membrane.
C. Modify the diagram to create a representation of the effect explained above. Select
from the list to fill in the blanks.

 Na+
 Cl–
 stimulatory
 inhibitory
D. In the 1960s, Burnstock and coworkers provided evidence that ATP is a
neurotransmitter. This was received skeptically and largely rejected until 1984 when a
modified form of ATP that was known to block the intracellular function of ATP was
shown to affect extracellular signal transmission. Based on the central role played by ATP
in biological systems, justify the resistance within the scientific community to accept a
role for ATP as a neurotransmitter. Based on the fact that ATP has been conserved
throughout evolution of life on Earth, justify such a role for ATP. Based on these two
perspectives, analyze the role of cooperative interactions in the positive selection of ATP
as a neurotransmitter.

Advanced Placement Biology Instructor’s Solution Manual


ne System

28 | The Endocr658 System PAGE \* MERGEFOR


Solution Sample answer:
A. A change in the electric potential across the membrane of the presynaptic neuron
initiates the formation of a vesicle containing ATP. The vesicle fuses with the neuron
cell membrane at the synapse, and ATP is released into the synapse. This is called
exocytosis. ATP docks at a receptor site on the postsynaptic neuron membrane, and
endocytosis is initiated with activation of an ion-ligand channel. If sodium ions move
through the channel, the postsynaptic cell membrane is depolarized with a
reduction in the electric potential difference. The effect is excitatory, and if the cell
membrane potential reaches a threshold value, then an action potential is created. If
chlorine ions move through the channel, then the membrane is hyperpolarized and
the effect is inhibitory.
B. Transport of positive charge over the membrane reduces the magnitude of the
electric potential difference, causing the interior of the cell to have a less negative
voltage. Less negative is positive. Transport of chlorine ions has the opposite effect
and the magnitude of the potential difference increases.
C.

D. The production of ATP is a target of metabolism, and evolution has selected for
increased production—for the cell. However, at the organism level, at which
selection also acts, we see that cooperation between cells, at the expense of the
individual, is selected for. The role of cooperation in biological systems has received
increased attention over the last few decades. For a memoir concerning this debate
see Burnstock (Trends in Pharmacological Sciences, 20, 2006).
26.2 How Neurons Communicate
37 Neurons and muscle cells maintain a high concentration gradient of potassium ions across
the plasma membrane. The extracellular space has a high concentration of sodium ions.
At the rest electric potential, the cell membrane is polarized.
A. Construct a representation of the cell membrane with annotation of the diagram that
includes the following:

Advanced Placement Biology Instructor’s Solution Manual


AT 733658

673 29 | The Musc659skel


 A labeled arrow that indicates the direction in which the motion of potassium
ions is driven by the concentration gradient
 A labeled arrow that indicates the direction in which the motion of sodium ions
is driven by the concentration gradient
 A brief statement of the roles of potassium and sodium ion pumps in maintaining
the rest electric potential
 A labeled arrow that indicates the relative sign of the electric potential
difference (voltage) between intracellular and extracellular spaces at the rest
electric potential

When an excitatory neurotransmitter receptor is activated, the electric potential


difference of membrane of a neuron is lowered, inducing a change in the configuration of
sodium pump proteins.
B. Justify the effect on the flux of sodium ions across the membrane as a positive
feedback in a situation in which the electric potential difference falls below a threshold
voltage and an action potential is created.
C. The action potential is transmitted along the neuron as a voltage wave. One cycle of the
wave is shown at the right of the diagram at the instant at which the maximum of the
electric potential of the membrane has been reached.

Construct a representation of the key elements of the signal propagation with annotation
of the diagram that includes the following:

Advanced Placement Biology Instructor’s Solution Manual


ne System

28 | The Endocr658 System PAGE \* MERGEFOR


 A labeled arrow that indicates the direction in which the motion of potassium
ions is driven by the concentration gradient
 A labeled arrow that indicates the direction in which the motion of potassium
ions is driven by the electric potential difference across the membrane
 A brief statement of the roles of potassium and sodium ion pumps in terminating
the action potential
D. Most neurons must transmit a signal quickly. The sarcolemmas (muscle cell
membranes) of the cardiac muscles receive signals that integrate information from both
the sympathetic (quick response with shorter time scale) and parasympathetic (steady
response with longer time scale) divisions of the autonomic nervous system. The action
potential that induces periodic contractions of the cardiac muscle (see figure below) is
broadened at the maximum by the release of from the smooth endoplasmic
reticulum, referred to as calcium-induced calcium release (CICR).

In terms of the function of the heart in the supply of oxygen and nutrients during fight-or-
flight or restful conditions, justify the claim that this broadening demonstrates that the
coordination of events must be regulated.
E. To stop a beating heart during open-heart surgery, a solution of KCl is injected into the
cardiac muscle. Explain the effect of a large dose of extracellular K+ on the transmission of
the action potential in the sarcolemma.
Solution Sample answer:
A. At the rest state, sodium ions are not allowed transport over the membrane. The
membrane is open to the transport of potassium ions from the interior to the
exterior of the cell. The result is that the interior is maintained at a more negatively
(less positively) charged state.

Advanced Placement Biology Instructor’s Solution Manual


AT 733658

675 29 | The Musc659skel

B. When the electric potential difference reaches a critical value that is smaller in
value than the initial value of the polarized state, the sodium ion pump begins to
further increase the positive charge of the interior of the neuron. This is positive
feedback because the action induced by the effect amplifies the effect as the interior
of the cell quickly becomes more positive.
C. When the electric potential difference reaches a second critical value, the sodium
ion pump closes. The potassium ion pump rapidly releases positive charge to the
extracellular space, and the membrane returns to the original polarity.

D. A single time period (heartbeat rate) is not sufficient to maintain homeostasis or


to cope with environmental stresses. The time period of the heartbeat rate must be
regulated, so the CICR system is selected by the advantage provided.
E. An injection of potassium ions into the extracellular space breaks the sequence
that propagates the action potential by reducing the concentration-driven transport
of potassium from the interior to the exterior of the cell.
26.3 The Central Nervous System
38 The brain integrates new information through the formation of memories and by learning.
Alternative explanations of the ability of the brain to remodel in response to experience,
called plasticity, are given. This item explores those explanations.
A. Consider the interaction of these three cell types that integrate information to produce
a response to external cues:

Use the figure to construct a representation of the direction of information flow.

Advanced Placement Biology Instructor’s Solution Manual


ne System

28 | The Endocr658 System PAGE \* MERGEFOR


B. The central body of a neuron is elaborated by tree-like structures called dendrites.
These allow the neuron to integrate information from multiple sensory receptors.
Describe what refinement of the basic stimulus-response system in the diagram is needed
to achieve even the simplest response: “move away.” Awareness of orientation and
motion within a body is called proprioception.
Describe how multiple neurons are required to acquire proprioception.
C. The generation of neurons occurs during development. However, adults continue to
form memories and learn. Rearrangement of connections between neurons is a possible
explanation, and in several studies steroid hormones have been shown to produce
dendritic plasticity. The hippocampus is active during memory formation and learning and
significant variations in the number of dendrites were observed in the hippocampus
(Wooley et al., Journal of Neuroscience, 10, 1990) were correlated with variations in
estrogen during the estrus cycle. More recently, variation in these structures is implicated
in a collection of behaviors known as chronic unpredictable mild stress (CUMS). In rats,
CUMS can be induced by environmental factors such as electric shock, immobilization, or
isolation (Qiao et al., Neural Plasticity, 2016).
Pose two scientific questions that can be investigated to connect the dynamic
homeostasis and survival advantage of the individual to dendritic plasticity.
D. An alternative explanation of the manner in which the brain integrates new
information is through synaptic plasticity. This has been demonstrated by Nabavi and
coworkers (Nature, 511, 2014). An associated memory was created in a rat by pairing two
stimuli: an audio tone and a foot shock. The animal had previously been trained to avoid
pressing a lever that delivers a reward by associating the lever press with a shock. After
conditioning, the animal responded to the tone as if it was a shock and avoided reward.
The ratio of stimulatory to inhibitory receptors at the synaptic membrane was shown to
increase with the experience.
A miniaturized device, optically activated and controlled by flashing light, was inserted in
the nuclei of neurons transmitting the tone to the rat’s brain. When the experimenters
used light with 1 flash per second (1 Hz), the device caused the expression by the cell of
one type of protein, and when a light with 100 flashes per second (100 Hz) was used, the
device caused expression of another type of protein. Each of the graphs describes the
response of the rat to environmental cues. One day elapsed between each data collection
represented by one graph.

Analyze these data in terms of the evidence provided for synaptic plasticity.

Advanced Placement Biology Instructor’s Solution Manual


AT 733658

677 29 | The Musc659skel


E. A third explanation for the formation of memory and learning is found in the lab of
David Glanzman (Elife, 2014). The sea slug (Aplysia) can be trained to withdraw its siphon
tube. Sensory and motor neurons can be grown in tissue culture. The addition of
serotonin to the tissue culture increases the number of synaptic connections and the
training can be induced in vitro. Cells that had acquired the stimulus-response behavior
were treated with an agent that destroys the synaptic receptors. Yet the trained response
was retained, and there were indications that the information was retrieved from the
neuron nucleus.
Suppose that this work concerning the location of memory is confirmed. Create a
representation of information flow in which a fourth box labeled “neuron nucleus” is
added to the diagram in part A between the stimulus and the neuron. Annotate the
representation to indicate the flow of information.
Explain
 How this form of plasticity is more dynamic than theories in which memory
resides in synaptic or dendritic structures, and
 How it might lead to treatments for disorders such as post-traumatic stress
syndrome, in which recollection creates a disability.
Pose questions regarding the ethical or social consequences of this technology.
Solution Sample answer:
A.

Information flows only from the sensory cells through the sensory nervous system.
Information flows in both directions in the somatic nervous system. The information
originates in the environment.
B. If a sensory cell receives a signal—for example, a pressure-induced deformation—
other signals must also be received in order to locate the signal spatially. A direction
away cannot be determined without additional data. Body awareness requires
information about the relative motion of body parts. At a minimum, this requires
two neurons.
C. Since the estrus cycle is periodic, a change in the number of dendrites must be
reversible. Are the stress-induced formations of dendrites reversible? Are they lost
over time (is the memory forgettable)? Both of these are scientific questions in that
they can be addressed through experience. A third question would be an essential
first step in an inquiry: Are there only these two reported cases of dendritic
plasticity? In fact, dendritic plasticity seems to be uncommon. For a review see
Tavosanis (Developmental Neurobiology, 72, 2012).

Advanced Placement Biology Instructor’s Solution Manual


ne System

28 | The Endocr658 System PAGE \* MERGEFOR


D. Synaptic receptors that receive stimulatory (100 Hz) and inhibitory (1 Hz)
responses are synthesized by the cell. The data reveal that the learning is reversible.
It can be erased by the 1 Hz signal and then restored by the 100 Hz signal.
E. If the nucleus is involved in memory retrieval, then the likelihood is that memory
is somehow encoded in DNA. Glanzman’s papers have raised the question, but it
seems that such a claim has not been made. Since the learning is restorable, it is
more recoverable than memories encoded in static structures such as dendrites or
synaptic receptors. PTSD was immediately discussed in the popular press
descriptions of this astonishing result. PTSD is thought to be enforced (the memory
is strengthened) with each recollection—much as a learned response can be made
more robust by repetition. It seems that the technology might be developed to erase
memories.

In fact, a diagram such as the one above reminds us that memories could be created
with such a technology through a process of directed selection—or even just gene
modification. The optogenetics technology used in this experiment was anticipated
by Francis Crick. Wikipedia has a page devoted to it. This is very exciting technology,
but can we use it? The questions that need to be asked are not scientific, in that they
have no technical answer. Who will have access to this technology? What effect
would the possibility of memory manipulation have on our willingness to trust?
What effect would there be on the value of life experience?
26.4 The Peripheral Nervous System
39 Autism is a collection of communication and socialization behaviors. Evidence of
inheritance of genes predisposing the individual during early development is indicated by
pedigrees such as the figure (after Allen-Brady, Molecular Psychiatry, 14, 2009). Males
(squares) and females (circles) are affected when the symbol is filled, are struck through
when deceased and the genome cannot be determined, and are dashed when living and
the genome was not determined.

Advanced Placement Biology Instructor’s Solution Manual


AT 733658

679 29 | The Musc659skel


A. Other evidence indicates that autism is not x-linked. Give an alternative explanation
that can account for these data.
B. Stem cells taken from fathers who do not present characteristics of autism and from
their sons were induced to form tissue cultures of neurons. Compared to the father, those
taken from the son showed accelerated growth with a higher number of synapses.
Describe possible consequences for the integration of information if this in vitro growth
also occurs in vivo.
C. The variety of phenotypes and large number of genes that have been implicated in this
disorder have led researchers to refer to the characteristics as autisms, described by a
spectrum of disorders, ASD. One of the genes implicated is bola2. While humans and
other primates have genomes that are reported to have only a 2 percent deviation, the
particular form of bola2 that occurs in 99 percent of human genomes that have been
mapped does not occur in other primates. And bola2 is not present in the Neanderthal
genome. Even more interesting is that single nucleotide variations in human bola2 are
significantly less frequent than genes associated with other brain disorders such as
schizophrenia.
Evaluate the selection pressure and direction (positive or negative) indicated by this
observation.
D. Several hundred genes have been implicated in ASD, and many others probably will
eventually be discovered. Expression in a gene network can depend on factors that are
both genetic and environmental. Given the complexity of ASD, what questions should be
researched by the physician of children or their parents when genetic screening is
considered?
Solution Sample answer:
A. Autism could be a multifactorial disorder in which certain traits present in males
but not females could influence progression of the disease.
B. This result indicates that autism is associated with an accelerated growth of
synapses. Perhaps the synapses grow so fast that they are not able to make normal
connections.
C. None of the autistic individuals in this tree has offspring. This indicates that autism
could have a negative selection pressure, in which the autistic condition lowers the
chance of an individual reproducing.
D. The physician should ask about the history of autism or other developmental
disorders, such as schizophrenia, in the family. The physician should also ask about
environmental factors, such as drugs taken by the mother during pregnancy, which
could have led to development of the disease.
40 Describe how neurons transmit information.

Advanced Placement Biology Instructor’s Solution Manual


ne System

28 | The Endocr658 System PAGE \* MERGEFOR


Solution Sample Answer: The answer to this question has been the subject of thousands
of research papers. The Biology Curriculum Framework is very specific about the expected
scope of content knowledge that can be assessed:
 A typical neuron has a cell body, axon, and dendrites. Many axons have a myelin
sheath that acts as an electrical insulator.
 The structure of the neuron allows for the detection, generation, transmission, and
integration of signal information.
 Schwann cells, which form the myelin sheath, are separated by gaps of unsheathed
axon over which the impulse travels as the signal propagates along the neuron.
 Membranes of neurons are polarized by the establishment of electrical potentials
across the membranes.
 In response to a stimulus, Na+ and K+ gated channels sequentially open and cause the
membrane to become locally depolarized.
 Na+/K+ powered by ATP, work to maintain membrane potential.
41 You are probably acquainted with the effects of local anesthetics. While the injection of
lidocaine at the dentist is unpleasant, no injection would be more so. Lidocaine is a
sodium channel blocker.
A. Explain the absence of pain in terms of the effect of lidocaine on signal reception and
transduction.
The pain of the dentist’s drill is caused by trauma at the cellular level. Chemical
messengers such as cytokines, serotonin, and prostaglandins are released by broken cells.
The receptors for these messages of trauma are called nociceptors, whose activation is
transmitted to the central nervous system by specialized cells called the A and C fibers.
B. The nervous system is a network of cells and tissues that is activated by these chemical
messengers. Identify another system that should be activated by these messengers and
support your claim by applying the idea that dynamic homeostasis is maintained by
timing and coordination of regulated events.
C. Chronic pain often persists after damaged tissue has healed. This pain is often
accompanied by sterile inflammation with components of the innate immune system such
as macrophages. Refine the model of coordinated response identified in part B to
describe how chemical messengers associated with the immune response can cause
chronic pain.
Unlike local anesthetics, general anesthetics block signal transduction of the entire central
nervous system and the brain. However, while the patient is unconscious, the peripheral
nervous system continues to support signaling to other systems such as heart and lungs.
An explanation might be that the signals in the central and peripheral nervous systems are
segregated and that the latter functions without cognitive integration (thought), as the
name “autonomic” implies. The respiratory center that provides autonomic control of
breathing is part of the medulla oblongata.

Advanced Placement Biology Instructor’s Solution Manual


AT 733658

681 29 | The Musc659skel


D. Create a visual representation of a system comprising only the cortex, the medulla
oblongata, the heart, and the lungs. Using arrows, describe the flow of information.
Consider “holding your breath” in creating your representation. Consider why you always
stop holding your breath eventually. Consider “holding your heart.” Experimental data on
the voluntary control of heart rate by people who practice yoga have been reported
(Raghavendra et al., International Journal of Yoga, 6, 2013; Telles et al., Integrative
Physiological and Behavioral Science, 39, 2004).
E. Analyze the data provided in the following sketch of blood flow, a process controlled by
the autonomic nervous system, in the two ears of a rabbit (after Blessing, Trends in
Neuroscience, 20, 1997) in terms of cognitive integration of the response to the stimulus
provided by touching the rabbit.

Solution Sample answer:


A. Sodium channels open in response to a nerve impulse. In the presence of
lidocaine, the sodium channels of nociceptors cannot open, so the pain signal is not
sent to the brain.

Advanced Placement Biology Instructor’s Solution Manual


ne System

28 | The Endocr658 System PAGE \* MERGEFOR


B. Besides triggering a pain response, cytokines stimulate cells in the immune
system. Pain makes us aware that there is tissue damage so that we can take
appropriate measures to mend the damage. The immune system acts to heal
damaged tissue and fight potential infections. Both of these reactions maintain
homeostasis by helping us return our bodies to a healthy condition when they
become damaged.
C. The immune response is supposed to be temporary, but in some cases
inflammation or other immune responses can continuously trigger the release of
cytokines and other chemical messengers that induce the immune response. When
this happens, a person can feel pain long after the tissue has healed.
D. Sensory information flows from the lungs and heart to the medulla. Limited
information flows to the cortex because these are autonomic activities. Voluntary
control by the cortex over the lungs allows people to hold their breaths. Voluntary
control over the heart is generally not possible and is achieved through conditioning
of more primitive systems.

E. Touching the rabbit’s ears sends a sensory signal to the sensory cortex of the
brain. This circuit passes information to the brain stem, including the medulla
oblongata, which controls blood flow. Constriction of these blood vessels reduces
blood flow.

Advanced Placement Biology Instructor’s Solution Manual


27 | Sensory Systems 683

27 | SENSORY SYSTEMS
REVIEW QUESTIONS
1 What is a type of general sense in humans?
A Gustation
B Olfaction
C Proprioception
D Equilibrium
Solution The solution is (C). Proprioception is a general sense for controlling the position of
bones, joints, and muscles.
2 Suppose you burned your tongue and could not taste food for a day. Which sense is
affected?
A Olfaction
B Gustation
C Proprioception
D Kinesthesia
Solution The solution is (B). If you burned your tongue and could not taste food for a day, you
affected gustation. Gustation is the ability to sense tastes.
3 Where does perception occur?
A Spinal cord
B Cerebral cortex
C Receptors
D Thalamus
Solution The solution is (B). Perception occurs in the cerebral cortex. Different parts of the
cerebral cortex interpret different types of stimuli.
4 If a person’s cold receptors no longer convert cold sensory signals into electrical signals,
that person has a problem with —
A Reception
B Receptive field
C Perception
D Transduction
Solution The solution is (D). If a person’s cold receptors no longer convert cold sensory signals
into electrical signals, that person has a problem with transduction. Transduction is

Advanced Placement Biology Instructor’s Solution Manual


684 27 | Sensory Systems

the process by which a sensory signal is converted to an electrical signal in the


nervous system.
5 What is the smallest difference in stimuli that can be detected?
A Receptor potential
B Sensory transduction
C Just-noticeable difference
D Perception
Solution The solution is (C). The smallest difference in stimuli that can be detected is the just-
noticeable difference.
6 In peppers, spicy heat is rated in SHUs, where zero is the least amount of spicy heat. If the
just-noticeable difference (JND) of the ability to perceive difference in heat detection for
an individual is 30 percent, which option represents the JND of two different peppers?
A 8,500 vs. 11,000 SHU
B 4,050 vs. 15,000 SHU
C 15,000 vs. 18,000 SHU
D 10,500 vs. 15,000 SHU
Solution The solution is (D). The JND of the two different peppers is 10,500 vs 15,000 SHU,
because this represents a difference of 30 percent.
7 What is the role of sensory receptors in sensory perception?
A Detection of specific stimuli
B Sensation interpretation
C Sending electrical signals to the cortex
D Transmitting signals from the brain to the rest of the body
Solution The solution is (A). Sensory receptors are specialized cells that detect specific stimuli,
such as light or pressure.
8 Which mechanoreceptors in the skin are unencapsulated?
A Merkel’s disks
B Meissner’s corpuscles
C Ruffini endings
D Pacinian corpuscles
Solution The solution is (A). Merkel’s disks are slow-adapting, unencapsulated
mechanoreceptors that respond to light touch.

Advanced Placement Biology Instructor’s Solution Manual


27 | Sensory Systems 685

9 If an individual is born without the ability to sense high-frequency vibrations, he may have
been born with a mutation in a gene that codes for —
A Merkel’s disks
B Meissner’s corpuscles
C Ruffini endings
D Pacinian corpuscles
Solution The solution is (D). If an individual is born without the ability to sense high-frequency
vibrations, he may have been born with a mutation in a gene that codes for Pacinian
corpuscles. Pacinian corpuscles are rapidly adapting, encapsulated
mechanoreceptors that are involved in detecting pressure and high-frequency
vibrations.
10 If you were to burn your epidermis, what receptor type would most likely burn?
A Free nerve endings
B Ruffini endings
C Pacinian corpuscles
D Krause end bulbs
Solution The solution is (A). If you were to burn your epidermis, you would most likely burn
free nerve endings, which extend into the middle of the epidermis.
11 _____ are found only in _____ skin, and detect skin deflection.
blank blank

A Meissner’s corpuscles; hairy


B Ruffini endings; glabrous
C Pacinian corpuscles; glabrous
D Hair receptors; hairy
Solution The solution is (D). Hair receptors wrap around the base of hair follicles and can
detect skin deflection.
12 To what does nociception respond?
A Injured stimuli
B Deep, fleeting pressure
C Fine touch
D Cold
Solution The solution is (A). Nociception, or pain, responds to injured stimuli from damaged
tissues.

Advanced Placement Biology Instructor’s Solution Manual


686 27 | Sensory Systems

13 Why do peppers that contain capsaicin taste hot?


A Capsaicin is corrosive and damages tissue.
B Capsaicin contains large quantities of heat that are released upon ingestion.
C Capsaicin and warm receptors open the same calcium channels.
D Capsaicin stimulates Krause end bulbs.
Solution The solution is (C). Peppers taste hot because both capsaicin and warm receptors
open the same calcium channels, which gives the feeling of heat when consuming
peppers.
14 What are mechanoreceptors that facilitate proper gripping of objects?
A Merkel’s disks
B Meissner’s corpuscles
C Ruffini endings
D Pacinian corpuscles
Solution The solution is (C). Ruffini endings are mechanoreceptors that facilitate proper
gripping of objects. Ruffini endings are slow-adapting, encapsulated
mechanoreceptors that have roles such as feedback for gripping objects.
15 How many different taste molecules can an individual taste cell detect?
A One
B Five
C Depends on the type of taste receptor
D Depends on the part of the tongue
Solution The solution is (A). An individual taste cell can only detect one taste molecule.
16 How are gustation and olfaction similar?
A Both sense different stimuli in the environment.
B Both can have hundreds of millions of types of receptors.
C Both obtain stimuli from within the body.
D Signals from both are transmitted through the medulla.
Solution The solution is (A). Gustation and olfaction are similar in that both obtain stimuli
from their environments and are able to discern the different stimuli.
17 What is the term for savoriness in food?
A Gustation
B Tastants

Advanced Placement Biology Instructor’s Solution Manual


27 | Sensory Systems 687

C Umami
D Pheromone
Solution The solution is (C). Umami, or savoriness, was only recently discovered to be
physiologically different from the other types of taste.
18 If an individual becomes repeatedly poisoned from eating spoiled food, what kind of
receptors might they lack?
A Salty
B Sweet
C Umami
D Sour
Solution The solution is (D). If an individual becomes poisoned from eating excessive amounts
of spoiled food, they might lack receptors for sour. Sour can signal that food is
spoiled.
19 A typical dog has approximately how many times more olfactory receptors than a typical
human?
A 2
B 8
C 83
D 333
Solution The solution is (C). Most dogs, except for those that were specially bred to have
improved olfaction, have approximately one billion olfactory receptors, whereas
typical humans only have 12 million olfactory receptors. This is an approximately
83-fold difference in the number of olfactory receptors.
20 Which statement is true of dog olfaction?
A Most dogs have 4 billion olfactory receptors.
B Bloodhounds have more receptors than humans, but humans have more receptors
than most dogs.
C Rabbits and most dogs have the same number of olfactory receptors.
D Dogs bred for sense of smell can have four times more olfactory cells than most
other dogs.
Solution The solution is (D). It is true that dogs bred for sense of smell can have four times
more olfactory cells than most other dogs. For example, bloodhounds have
approximately 4 billion olfactory receptors, whereas most dogs have approximately
1 billion.

Advanced Placement Biology Instructor’s Solution Manual


688 27 | Sensory Systems

21 What has the most taste receptors?


A Fungiform papillae
B Circumvallate papillae
C Foliate papillae
D Filiform papillae
Solution The solution is (C). Foliate papillae have 1,300 taste receptors per taste bud, which is
more than fungiform, circumvallate, and filiform papillae.
22 In sound, pitch is measured in _____, and volume is measured in _____.
blank blank

A decibels (dB); hertz (Hz)


B decibels (dB); nanometers (nm)
C nanometers (nm); decibels (dB)
D hertz (Hz); decibels (dB)
Solution The solution is (D). In sound, pitch, or frequency, is measured in hertz, whereas
volume, or amplitude, is measured in decibels.
23 Which animal would be affected by a 120,000-Hz sound?
A Cat
B Bat
C Dolphin
D Dog
Solution The solution is (C). A dolphin would be affected by a 120,000-Hz sound, because
dolphins can hear up to 150,000 Hz.
24 Louder sounds have a _____ compared with softer sounds.
blank

A greater amplitude
B lower amplitude
C greater frequency
D lower frequency
Solution The solution is (A). Louder sounds have a greater amplitude compared with softer
sounds. Amplitude describes volume of sounds, and greater amplitudes are
associated with increased volumes.
25 What is true of frequency?
A Frequency is heard as volume.
B Louder sounds have a higher frequency.

Advanced Placement Biology Instructor’s Solution Manual


27 | Sensory Systems 689

C Men hear higher frequencies than women.


D Frequency is measured in number of sound waves per unit time.
Solution The solution is (D). It is true that frequency is measured in hertz, which represents
the number of sound waves per unit time. Frequency is detected as pitch.
26 What contains receptors for transduction of mechanical waves to produce electrical
signals?
A Tympanum
B Cochlea
C Pinna
D Stape
Solution The solution is (B). The cochlea contains receptors for transduction of mechanical
waves to produce electrical signals. The cochlea is a fluid-filled, snail-shaped organ
that contains receptors that receive stimuli to produce electrical signals.
27 The _____ contains _____, which produces action potentials along the auditory nerve.
blank blank

A incus; stapes
B ear canal; tympanum
C tympanum; oval window
D organ of Corti; stereocilia
Solution The solution is (C). The organ of Corti contains stereocilia, which produce action
potential along the auditory nerve. In the cochlea, the organ of Corti is the site of
sound transduction, wherein stereocilia produce action potential that travels along
the auditory nerve.
28 Which structure is found in both the auditory system and the vestibular system?
A Basilar membrane
B Hair cells
C Semicircular canals
D Ossicles
Solution The solution is (B). Hair cells are part of both the auditory system and vestibular
system, but they are excited in different ways in the two systems.
29 You are in a car that suddenly decelerates. What happens inside the ear as the car comes
to a stop?
A Fluid in the semicircular canals moves.
B Stereocilia are bent.

Advanced Placement Biology Instructor’s Solution Manual


690 27 | Sensory Systems

C Deceleration signals are sent to the brain.


D Fluid in the semicircular canals stops moving.
Solution The solution is (D). Fluid in the semicircular canals stops moving when a car comes to
a stop. Once the deceleration ceases, so does the movement of the fluid in the
semicircular canals.
30 Of the following waves, which waves have the highest frequency?
A Microwaves
B Ultraviolet rays
C X-rays
D Gamma rays
Solution The solution is (D). Gamma rays have a higher frequency than microwaves,
ultraviolet rays, and X-rays.
31 Of the following colors, which color is associated with the shortest wavelength?
A Red
B Yellow
C Green
D Blue
Solution The solution is (D). Blue has a shorter wavelength than red, yellow, or green.
32 Which statement is true of light detection?
A Humans see most of the light spectrum.
B Light signals can pass through a vacuum into the eye.
C Decibels are used as the unit of wavelength.
D Violet light has a longer wavelength than red light.
Solution The solution is (B). It is true that light signals can pass through a vacuum into the eye
because light does not require a medium to pass through as sound does.
33 The fovea is responsible for _____, because it has a high density of cones.
blank

A night vision
B nearsightedness
C farsightedness
D acute vision
Solution The solution is (D). The fovea is responsible for acute vision because it has a high
density of cones. When your eyes orient on an object, the image in focus falls on
the fovea.

Advanced Placement Biology Instructor’s Solution Manual


27 | Sensory Systems 691

34 Why do people over 55 years old often need reading glasses?


A Their cornea no longer focuses correctly.
B Their lens no longer focuses correctly.
C Their eyeball has elongated with age, causing images to focus in front of their retina.
D Their retina has thinned with age, making vision more difficult.
Solution The solution is (B). People over 55 often need reading glasses because their lens no
longer focuses correctly. The flexibility of the lens decreases with age and can cause
a form of farsightedness, which means they are unable to see items that are close to
the eyes, such as books and other reading material.
35 Where does some visual processing occur before information reaches the brain?
A Cornea
B Lens
C Iris
D Retina
Solution The solution is (D). Some visual information is processed in the retina before it
reaches the brain. The retina is the location of photoreceptive cells.
36 A person catching a ball must coordinate her head and eyes. Which part of the brain helps
the person do this?
A Hypothalamus
B Superior colliculus
C Thalamus
D Pineal gland
Solution The solution is (B). The superior colliculus controls eye movements that are
coordinated with auditory information, and therefore helps a person catch a ball.

CRITICAL THINKING QUESTIONS


37 Which statement explains how the two types of sensory transduction differ?
A Receptors can respond to multiple stimuli, whereas free nerve endings are specialized
cells that detect a specific stimulus.
B Receptors are specialized cells that detect a specific stimulus, whereas free nerve
endings can respond to multiple stimuli.
C Receptors are similar for different stimuli, whereas free nerve endings are different for
different stimuli.
D Receptors are specialized cells that detect a specific stimulus, whereas free nerve
endings can respond to pressure.

Advanced Placement Biology Instructor’s Solution Manual


692 27 | Sensory Systems

Solution The solution is (B). Receptors are specialized cells that detect a specific stimulus,
such as light alone, whereas free nerve endings can respond to multiple stimuli,
including touch or temperature.
38 How would the steps of sensory perception be affected if a person sustains damage to
axons that lead from sensory receptors to the central nervous system?
A Reception would not be affected. However, signal transduction and perception will be
incomplete.
B Perception would not be affected. However, signal transduction and reception will be
incomplete.
C Signal transduction would not be affected. However, reception and perception will be
incomplete.
D Reception and signal transduction would not be affected. However, perception will be
incomplete.
Solution The solution is (A). Reception would not be affected. However, signal transduction
and perception will be incomplete.
39 What is an example of how Weber’s law is applicable to a just-noticeable difference?
A A difference between 20 and 21 units of weight is more likely detectable than a
difference between 1 and 2 units.
B A difference between 1 and 2 units of weight is more likely detectable than a
difference between 20 and 21 units.
C A difference between 1 and 2 units of weight is more likely detectable than a
difference between 2 and 4 units.
D A difference between 20 and 21 units of weight is more likely detectable than a
difference between 1 and 4 units.
Solution The solution is (B). Greater proportional differences are more likely to be
detected. For example, 1 versus 2 units, such as weight, is a 50 percent increase
and therefore more likely detectable than a difference between 20 and 21 units,
which is a 4.5 percent difference, even though both values only differ by 1 unit.
40 Humans have both special and general senses. Which statement explains what both types
of senses have in common?
A All types of senses undergo sensory transduction by converting a stimulus into a
chemical signal via the central nervous system.
B All types of senses undergo sensory transduction by converting a stimulus into an
electrical signal via the peripheral nervous system.

Advanced Placement Biology Instructor’s Solution Manual


27 | Sensory Systems 693

C All types of senses undergo sensory transduction by converting a stimulus into a


chemical signal via the nervous system.
D All types of senses undergo sensory transduction by converting a stimulus into an
electrical signal via the nervous system.
Solution The solution is (D). Although different receptors can detect specific stimuli, all types
of senses undergo sensory transduction by converting a stimulus into an electrical
signal via the nervous system.
41 Why are there are more Merkel’s disks and Meissner’s corpuscles in your fingertips than
in your palms?
A These two types of thermoreceptors are used to detect warmth and cold, which is
necessary to maintain body temperature.
B These two types of mechanoreceptors are used to detect fine details necessary for
many roles, such as typing, which are important for fingertips but not palms.
C These two types of proprioceptors are used to detect fine details necessary for many
roles, such as typing, which are important for fingertips but not palms.
D These two types of mechanoreceptors are used to detect fine details, which are
necessary for the many roles both fingertips and palms play.
Solution The solution is (B). Humans use their hands for very fine manipulations; therefore,
these two types of mechanoreceptors are used to detect fine details, which are
necessary for many roles of fingertips, such as typing.
42 What can be inferred about the relative sizes of the areas of cortex that process signals
from skin that is NOT densely innervated with sensory receptors versus skin that is
densely innervated with sensory receptors?
A Areas of the cortex that process signals from skin with fewer sensory receptors are
likely to be larger than those having large numbers of sensory receptors.
B Areas of the cortex that process signals from skin with fewer sensory receptors are
likely to be smaller than those having large numbers of sensory receptors.
C Areas of the cortex that process signals from skin with fewer sensory receptors and
large numbers of sensory receptors will likely be the same.
D There is no relationship between the relative sizes of areas of cortex that process
signals from skin and the sensory receptor numbers.
Solution The solution is (B). The areas of the cortex that process signals from skin with fewer
sensory receptors is likely to be smaller than the areas of the cortex that process
signals from skin with large numbers of sensory receptors.

Advanced Placement Biology Instructor’s Solution Manual


694 27 | Sensory Systems

43 Why do some people think that peppers are painful or hot, while other people do NOT
find peppers painful or hot?
A Peppers contain capsaicin, which opens the same sodium channels as warm receptors.
Excess stimulation gives the perception of pain. Thus, people who can tolerate more
heat find peppers to be less painful.
B Peppers contain capsaicin, which opens the same calcium channels as warm
receptors. Excess stimulation gives the perception of pain. Thus, people who can
tolerate more heat find peppers to be less painful.
C Peppers contain quinine, which opens the same calcium channels as warm receptors.
Excess stimulation gives the perception of pain. Thus, people who can tolerate more
heat find peppers to be less painful.
D Peppers contain quinine, which opens the same sodium channels as warm receptors.
Excess stimulation gives the perception of pain. Thus, people who can tolerate more
heat find peppers to be less painful.
Solution The solution is (B). Peppers contain capsaicin, which opens the same calcium
channels as warm receptors. Excessive amounts of stimulation give the perception
of pain. Therefore, people who can tolerate greater amounts of heat find peppers to
be less painful.
44 How does the location of mechanoreceptors affect their ability to sense different stimuli?
A Merkel’s disks and Meissner’s corpuscles are found in specialized regions and detect
the amount of stretch. Pacinian corpuscles and Ruffini endings are able to sense
deeper touch, such as deeper pressure.
B Merkel’s disks and Meissner’s corpuscles are found deeper in the skin and are able to
sense deeper touch, such as deeper pressure. Pacinian corpuscles and Ruffini endings
are able to better detect fine touch.
C Merkel’s disks and Meissner’s corpuscles are found deeper in the skin and detect fine
touch. Pacinian corpuscles and Ruffini endings are able to sense deeper touch, such as
deeper pressure.
D Merkel’s disks and Meissner’s corpuscles are found in more upper parts of the skin
and detect fine touch. Pacinian corpuscles and Ruffini endings are able to sense
deeper touch, such as deeper pressure.
Solution The solution is (D). Merkel’s disks and Meissner’s corpuscles are found in more
upper parts of the skin compared with Pacinian corpuscles and Ruffini endings, and
are therefore able to better detect fine touch. Pacinian corpuscles and Ruffini
endings are able to sense deeper touch, such as deeper pressure.

Advanced Placement Biology Instructor’s Solution Manual


27 | Sensory Systems 695

45 What happens to the ability to perceive taste and smell as people age?
A All senses decline with age, most dramatically by age 50 and then continue to decline
thereafter.
B All senses increase with age, most dramatically by age 50 and then continue to
increase thereafter.
C All senses decline with age, most dramatically by age 50 and then increase thereafter.
D All senses increase with age, most dramatically by age 50 and then decline thereafter.
Solution The solution is (A). All senses, including taste and smell, decline with age; most
dramatically by age 50 and then continue to decline thereafter.
46 What is a possible effect on an animal of NOT being able to perceive taste?
A The animal might not be able to eat food.
B The animal might not be able to eat sweet and unspoiled food.
C The animal might not be able to distinguish food that is bitter and sour.
D The animal might not be able to distinguish food that is dangerous, bitter, spoiled,
sour, or sweet.
Solution The solution is (D). The animal might not be able to distinguish food that is
dangerous, such as toxic substances, which are often bitter, or spoiled, which are
often sour. Moreover, the animal might not know which foods are high in calories,
which are often sweet, when food availability is limited.
47 If a young child goes missing, why would a bloodhound, NOT a poodle, be used to find
the child?
A Bloodhounds were bred to have a better sense of smell, and thus have fewer olfactory
receptors and larger olfactory epithelia.
B Bloodhounds were bred to have a better sense of smell, and thus have more olfactory
receptors and larger olfactory epithelia.
C Bloodhounds were bred to have a better sense of smell, and thus have more olfactory
receptors and smaller olfactory epithelia.
D Bloodhounds were bred to have a better sense of smell, and thus have more olfactory
bulbs and larger olfactory receptors.
Solution The solution is (B). Bloodhounds were bred to have a better sense of smell, and thus
have more olfactory receptors and larger olfactory epithelia than poodles. Thus
bloodhounds can detect lower levels of smells, which in turn increases their
sensitivity to smell.

Advanced Placement Biology Instructor’s Solution Manual


696 27 | Sensory Systems

48 How do pheromones differ from other odorants, from the perspective of the recipient of
the signal?
A Pheromones are sent to the main olfactory bulb instead of the amygdala and are not
consciously perceived.
B Pheromones are sent to the amygdala instead of the main olfactory bulb and are
consciously perceived.
C Pheromones are sent to the amygdala instead of the main olfactory bulb and are not
consciously perceived.
D Pheromones are sent to the main olfactory bulb instead of the amygdala and are
consciously perceived.
Solution The solution is (C). Pheromones are sent to the amygdala instead of the main
olfactory bulb and are not consciously perceived, unlike other odorants.
49 You are sitting with a dog and a cat and decide to test a 50,000-Hz ringtone. Which one of
you is likely to respond to the sound? Explain why.
A The human and dog will respond, because both can hear up to 50,000 Hz.
B The cat and dog will respond, because both can hear up to 50,000 Hz.
C Only the dog will respond, because dogs can hear up to 50,000 Hz.
D Only the cat will respond, because cats can hear up to 50,000 Hz.
Solution The solution is (D). Only the cat will respond, because cats can hear up to
50,000 Hz, whereas dogs can only hear up to 40,000 Hz and humans can only
hear up to 20,000 Hz.
50 You are having a debate with someone in a library. A librarian asks you to “speak softer.”
What characteristic of sound does the librarian want you to change and how can you
change it?
A Wavelength, by lowering the amplitude at which you are speaking
B Amplitude, by lowering the frequency at which you are speaking
C Frequency, by lowering the volume at which you are speaking
D Amplitude, by lowering the volume at which you are speaking
Solution The solution is (D). The librarian wants you to reduce the amplitude at which you are
speaking, which is heard as volume. Therefore, you are being told to lower the
volume at which you are speaking.

Advanced Placement Biology Instructor’s Solution Manual


27 | Sensory Systems 697

51 If an individual was born without the malleus and incus in either ear, why might he or she
have problems with hearing?
A Without the malleus and incus, the vibrations of the tympanum would not be able to
reach the stapes and then be sent to the cochlea.
B Without the malleus and incus, the vibrations of the pinna would not be able to reach
the stapes and then be sent to the cochlea.
C Without the malleus and incus, sound waves would not be collected by the
tympanum.
D Without the malleus and incus, sound waves would not be collected by the pinna.
Solution The solution is (A). The malleus and incus are two of the three bones in the middle
ear that transmit vibrations from the tympanum to the cochlea. Without the malleus
and incus, the vibrations of the tympanum would not be able to reach the stapes
and then be sent to the cochlea.
52 How might being on the moon, which has less gravity than Earth, affect vestibular
sensation? Explain why.
A Vestibular sensation relies on gravity’s effects on tiny crystals in the inner nostril;
therefore, reduced gravity on the moon would likely impair vestibular sensation.
B Vestibular sensation relies on gravity’s effects on huge crystals in the inner ear;
therefore, reduced gravity on the moon would likely impair vestibular sensation.
C Vestibular sensation relies on gravity’s effects on tiny crystals in the inner ear;
therefore, reduced gravity on the moon would likely impair vestibular sensation.
D Vestibular sensation relies on gravity’s effects on tiny crystals in the outer ear;
therefore, reduced gravity on the moon would likely impair vestibular sensation.
Solution The solution is (C). Vestibular sensation relies on gravity’s effects on tiny crystals in
the inner ear. Therefore, reduced gravity, as on the moon, would likely impair
vestibular sensation. However, the sense of motion can still be detected by the eyes.
53 Why are you unable to see the heat emitted by a cricket?
A Ultraviolet light includes heat emitted by prey organisms of reptiles, which is outside
the visual spectrum for humans because the wavelength is less than 380 nm.
B Infrared light includes heat emitted by prey organisms of reptiles, which is outside the
visual spectrum for humans because the wavelength is less than 380 nm.
C Infrared light includes heat emitted by prey organisms of reptiles, which is outside the
visual spectrum for humans because the wavelength is more than 400 nm.
D Ultraviolet light includes heat emitted by prey organisms of reptiles, which is outside
the visual spectrum for humans because the wavelength is more than 400 nm.

Advanced Placement Biology Instructor’s Solution Manual


698 27 | Sensory Systems

Solution The solution is (B). Infrared light, which includes heat emitted by prey organisms of
reptiles, is outside the visual spectrum for humans because the wavelength is less
than 380 nm. However, infra light produces heat, which stimulates other receptors.
54 What are the color receptors in your eyes perceiving if you see a white building?
A All of the color receptors in your eyes are equally stimulated when you see the
color white.
B Both L and M cones are equally stimulated in your eyes when you see the color white.
C Only the S cones are stimulated in your eyes when you see the color white.
D L cones are stimulated strongly and S cones are weakly stimulated when you see the
color white.
Solution The solution is (A). All of the color receptors in your eyes are equally stimulated
when you see the color white. Thus, the red, green, and blue sensitive receptors in
the three different types are being stimulated.
55 How is the relationship between photoreceptors and bipolar cells different from other
sensory receptors and adjacent cells?
A Photoreceptors and bipolar cells are depolarized, whereas other sensory receptors
typically remain polarized.
B Photoreceptors and bipolar cells are hyperpolarized, whereas other sensory receptors
typically remain polarized.
C Photoreceptors and bipolar cells are depolarized, whereas other sensory receptors
typically become hyperpolarized.
D Photoreceptors and bipolar cells are hyperpolarized, whereas other sensory receptors
typically become depolarized.
Solution The solution is (B). Activation of rhodopsin begins a cascade of events, and at the
end, the Na+ channels are closed in the photoreceptor membrane. This is different
from other sensory receptors and adjacent cells because these cells are
hyperpolarized, whereas other sensory receptors typically become depolarized.
56 What happens once visual signals reach the visual cortex?
A Some signals go to the temporal lobe, which detects where information, and other
signals go to the parietal lobe, which detects where and what signals.
B Some signals go to the parietal lobe, which detects where information, and other
signals go to the temporal lobe, which detects what signals.
C Some signals go to the parietal lobe, which detects where and what information and
other signals go to the temporal lobe, which also detects where and what signals
D Some signals go to the parietal lobe, which detects where information, and other
signals go to the temporal lobe, which detects where and what signals.

Advanced Placement Biology Instructor’s Solution Manual


27 | Sensory Systems 699

Solution The solution is (D). The visual signals travel in two different directions. Some signals
go to the parietal lobe, which detects where information, and other signals go to the
temporal lobe, which detects where and what signals.

SCIENCE PRACTICE CHALLENGE QUESTIONS


27.1 Sensory Processes
57 Odorants are mixtures of many different molecules. The complexity of the human sense
of smell can be represented visually as a grid of 100 cells (10 × 10 grid) with each cell
associated with a unique molecule-receptor pair. An odorant is detected when the brain
integrates the signals generated by each molecule in the mixture.
Four olfactory sensors, each innervated by a nerve that transmits information to the brain
as an action potential, are shown in the diagram. Three of the sensors each respond to
one of the three odor molecules in the geometric representation of the odorant mixture.

A. In the diagram, the odorant-receptor pairing is imagined geometrically; a round peg fits
in a round hole and a square peg does not. The receptors are located in the epidermal cell
surface, shown in the drawing as a light gray line. Create a geometric representation by
drawing receptors on the surfaces of the sensors that are activated by one of the
molecules in the mixture. Draw a geometric representation of a fourth receptor surface
that is not activated by a molecule in the odorant mixture.
B. Construct an explanation of the mechanism for transmission of information when the
odorant molecule is detected at receptor A using this signaling cascade. In your
explanation, include the role of positive feedback and the mechanism of the generation of
an action potential.

Advanced Placement Biology Instructor’s Solution Manual


700 27 | Sensory Systems

C. Signal integration allows the brain to discriminate this particular odorant mixture from
others using the time dependence in each signal. The sensitivity of an olfactory system
increases as the number of unique receptors increases.
Complete the following table to construct a mathematical representation of sensitivity to
the chemical landscape assuming that there are 100 unique odorant molecules. Use the
following mathematic routine to determine the number of odors caused by groups of
molecules selected from the 100 odorant molecules:
Number, r, of Odor-
Producing Molecules in Number of Different
the Odorant Odorants
3

4 Blank

5 Blank

6 Blank

7 Blank

number of groups = ,

where and .
Bushdid et al. (Science 343, 2014) extended this model and then used human subjects to
experimentally determine the number of unique odorant molecules that they could
discriminate to obtain an estimate of 1.72 trillion different detectible smells.
D. Olfactory receptor proteins that recognize chemicals as odors are expressed in humans
by approximately 400 different genes. This is the largest number of genes coding for a
single function in the human genome (Nimura, Human Genetics 4, 2009). Other mammals
have an even greater diversity of olfactory receptors: roughly 800 and 1,200 genes in dogs
and rats, respectively. Some olfactory receptors are adapted for odorants in an aqueous
environment and some are adapted for an air environment.

Advanced Placement Biology Instructor’s Solution Manual


27 | Sensory Systems 701

Use the representation, showing classes of genes within groups of organisms, to construct
a representation of the phylogenetic relationships among these groups. Annotate your
representation to show gene additions and deletions. To your representation, also add
annotation that connects the phenotype to the environment.
Solution Sample answer:
A.

B. The signal cascade is initiated when the molecule docks at the receptor. A
membrane-bound protein (G-protein) releases a secondary messenger (AMP) that
opens the calcium transport channel. Calcium is moved across the membrane,
increasing the positive charge within the cell. The electric potential difference of the
membrane is made more positive, relative to the exterior of the cell. Positive
feedback amplifies the increase in the electric potential by activating other ligand
gated channels until the membrane potential reaches a threshold value and an
action potential is established.

Advanced Placement Biology Instructor’s Solution Manual


702 27 | Sensory Systems

C.
Number, r, of Odor-
Producing Molecules in Number of Different
the Odorant Odorants
3

4 3,921,225
5 75,287,520
6 1,192,052,400
7 96,045,364,800
D.

27.2 Somatosensation
58 A. When you get cold, you may shiver. The shivering response is a reflex in which the
hypothalamus in the brain integrates sensory input, comparing input against a
temperature set point. When a threshold signal is detected, nerves of the somatic
nervous system that innervate skeletal muscle are stimulated. The extension and
contraction of muscle cells releases heat energy that elevates the body temperature.
Construct a representation of the information flow for the shivering response. Annotate
your representation to include the following:
 The sensory input
 Signal transmission to the hypothalamus
 Signal transmission from the hypothalamus to the somatic nervous system
 Transmission to muscle tissue
 Output response
Include in your annotation the negative feedback loop that is established.

Advanced Placement Biology Instructor’s Solution Manual


27 | Sensory Systems 703

B. Shivering occurs when you have a fever, even though your body temperature has not
fallen. Identify the component of the information flow represented above that can cause
this effect and describe how the brain has integrated the immune system and nervous
system to maintain homeostasis. Thermogenesis from accelerated metabolism in adipose
(fat) tissue is a non-shivering response to a cold stimulus. Using a neurotropic virus,
investigators are able to trace the paths of nerves in animals.
C. Neurons infected by virus can then be visualized by exposing the tissue to antibodies
that can be stained or using dyes that fluoresce. Ryu and coworkers (Journal of
Neuroscience, 35, 2015) used this technique to demonstrate communication between
nerves of the sympathetic nervous system and nerves of the sensory nervous system
innervating thermoreceptors that are sensitive to hot and cold.
Construct a representation of information flow and annotate the representation with
labels for the following:
 Signal input caused by low temperature
 Signal transmission through nerves of the sensory system
 Transmission of signal from the sensory system to the sympathetic system
 Transmission to adipose cells
 Output response of cells
Include in your annotation the negative feedback loop that is established.
Solution Sample answer:
A.

B. The thermostat is at the hypothalamus. If you shiver when you are not cold, then
the set point has been raised. The immune system and nervous system have
coordinated the response to change the set point at the hypothalamus.
C.

Advanced Placement Biology Instructor’s Solution Manual


704 28 | The Endocrine System

28 | THE ENDOCRINE SYSTEM


REVIEW QUESTIONS
1 Although most types of hormones are lipid insoluble, there are some that are lipid soluble
and can therefore diffuse through plasma membranes. What class of hormones can
diffuse through plasma membranes?
A Lipid-derived hormones
B Amino acid-derived hormones
C Peptide hormones
D Glycoprotein hormones
Solution The solution is (A). Lipid-derived hormones can pass through plasma membranes
because they are lipid soluble.
2 Mary produces relatively low amounts of estradiol and cortisol, but is able to produce
proper amounts of other hormones such as thyroxine and insulin. Which chemical class of
hormones does Mary have difficulty producing?
A Lipid-derived hormones
B Amino acid-derived hormones
C Peptide hormones
D Glycoprotein hormones
Solution The solution is (A). Mary has difficulty producing lipid-derived hormones. Lipid-
derived hormones end in –ol or –one.
3 An endocrinologist is a physician whose specialty is treating endocrine-related disorders.
Which disease would be monitored and treated by an endocrinologist?
A Schizophrenia
B Alzheimer’s disease
C Parkinson’s disease
D Graves’ disease
Solution The solution is (D). Graves’ disease is an autoimmune disease that affects the thyroid
gland and would thus be monitored by an endocrinologist.
4 Robert underwent a test to determine whether he was diabetic. Within 1.5 h of eating, his
blood glucose was normal. What can be determined from these results?
A The pancreas is overproducing insulin.
B The pancreas is underproducing insulin.

Advanced Placement Biology Instructor’s Solution Manual


28 | The Endocrine System 705

C The pancreas is producing an adequate amount of insulin.


D The pancreas is not producing any insulin.
Solution The solution is (C). The pancreas would have to release insulin in response to the
intake in food to lower blood sugar level. It would then have to stop secreting insulin
when a normal blood glucose level is reached. It seems to have done both of those
and is likely functioning normally.
5 The hormone that triggers the flight-or-fight response, epinephrine, is derived from
tyrosine, and the hormone that regulates circadian rhythms, melatonin, is derived from
tryptophan. What class of hormone are these?
A Lipid-derived hormones
B Amino acid-derived hormones
C Peptide hormones
D Glycoprotein hormones
Solution The solution is (B). Melatonin and epinephrine are amino-acid derived hormones,
because they are small and derived from the amino acids tyrosine and tryptophan.
6 A newly discovered hormone contains four amino acids linked together. Under which
chemical class would this hormone be classified?
A Lipid-derived hormones
B Amino acid-derived hormones
C Peptide hormones
D Glycoprotein hormones
Solution The solution is (C). This hormone would be classified as a peptide hormone. Peptide
hormones are made up of polypeptide chains.
7 Changes in the body can be mediated by direct or indirect mechanisms. What facilitate(s)
direct mediation of change by hormones?
A Intracellular hormone receptors
B Cell surface receptors
C Up-regulation
D Down-regulation
Solution The solution is (A). Intracellular hormone receptors are the most direct way in which
hormones can stimulate change when they reach a target cell.
8 How do thyroid hormones stimulate metabolic activity throughout the body?
A Specialized thyroid-receiving tissues are located throughout the body.
B Blood circulation stimulates metabolic activity.

Advanced Placement Biology Instructor’s Solution Manual


706 28 | The Endocrine System

C Different tissues have thyroid hormone receptors.


D Thyroid tissue is located throughout the body.
Solution The solution is (C). Hormones from the thyroid activate metabolism in tissues
throughout the body that have matching receptors.
9 Different hormones have different roles in cell signaling. What is the term for a hormone
that initiates cell-signaling pathways?
A Intracellular hormone receptor
B Cell surface receptor
C First messenger
D Second messenger
Solution The solution is (C). The first messenger initiates a cell signaling pathway.
10 A new antagonist molecule has been discovered that binds to and blocks plasma
membrane receptors. Which statement describes the effect this antagonist will have on
testosterone, which is a steroid hormone?
A This molecule will block testosterone from binding to its receptor.
B This molecule will block testosterone from activating cAMP signaling.
C This molecule will increase testosterone-mediated signaling.
D This molecule will not affect testosterone-mediated signaling.
Solution The solution is (D). This molecule will not affect testosterone-mediated signaling,
because lipid-derived hormones pass through plasma membranes and bind to
intracellular receptors.
11 Identify what triggers a reduction in the number of receptors if there are elevated
hormone levels.
A intracellular hormone receptors
B cell surface receptors
C up-regulation
D down-regulation
Solution The solution is (D). Down-regulation refers to a decrease in sensitivity by decreased
numbers of receptors on the target cells.
12 Consuming alcoholic beverages causes a change in urine output. This likely occurs because
these products —
A inhibit ADH release
B stimulate ADH release
C inhibit TSH release

Advanced Placement Biology Instructor’s Solution Manual


28 | The Endocrine System 707

D stimulate TSH release


Solution The solution is (A). Drinking alcoholic beverages causes an increase in urine output
because alcohol inhibits antidiuretic hormone release. Lack of antidiuretic hormone
results in less water being retained and thus promotes dehydration.
13 What stimulates FSH and LH release from the anterior pituitary?
A TSH
B GnRH

Advanced Placement Biology Instructor’s Solution Manual


708 28 | The Endocrine System

C T3
D PTH
Solution The solution is (B). GnRH stimulates release of FSH and LH.
14 Which hormone is produced by beta cells of the pancreas?
A T3
B Glucagon
C Insulin
D T4
Solution The solution is (C). Beta cells of the pancreas produce insulin in response to elevated
blood glucose levels.
15 What does PTH stimulate when blood calcium levels are low?
A Excretion of calcium from the kidneys
B Excretion of calcium from the intestines
C Osteoblasts
D Osteoclasts
Solution The solution is (D). When blood calcium levels are low, parathyroid hormone (PTH)
stimulates osteoclasts, which release calcium from the bones into the blood.
16 What enzyme is released when blood pressure drops and stimulates a cascade of events
for hormones that promote water reabsorption?
A Aldosterone
B Renin
C Antidiuretic hormone
D Osmoreceptors
Solution The solution is (B). Antidiuretic hormone is released when blood pressure drops and
stimulates a cascade of events for hormones that promote water reabsorption.
17 Drinking alcoholic beverages causes an increase in urine output. This most likely occurs
because alcohol ____.
A inhibits antidiuretic hormone release
B stimulates antidiuretic hormone release
C inhibits parathyroid hormone release
D stimulates parathyroid hormone release

Advanced Placement Biology Instructor’s Solution Manual


28 | The Endocrine System 709

Solution The solution is (A). Drinking alcoholic beverages causes an increase in urine output
because alcohol inhibits antidiuretic hormone release. Lack of antidiuretic hormone
results in less water being retained and thus promotes dehydration.

18 The hypothalamus is responsible for a diverse array of metabolic and autonomic nervous
system functions. What does the hypothalamus produce during puberty?
A Follicle-stimulating hormone
B Luteinizing hormone
C Inhibin
D Gonadotropin-releasing hormone
Solution The solution is (D). The hypothalamus produces gonadotropin-releasing hormone
during puberty. Gonadotropin-releasing hormone stimulates production of gonad
hormones.
19 Sandra wants to get pregnant but learns that she does not produce a sufficient amount of
prolactin. How will her reproductive abilities be affected?
A Her uterus will not contract during childbirth.
B She will not ovulate.
C Her body will not be prepared for pregnancy.
D She will be unable to produce milk.
Solution The solution is (D). Sandra will be unable to produce milk, because prolactin is
involved in milk production.
20 Different hormones are released as short-term and long-term stress responses. What is
released as a long-term stress response?
A Epinephrine
B Parathyroid hormone
C Corticosteroids
D T3 and T4
Solution The solution is (C). Corticosteroids are released by the adrenal cortex as a response
to long-term stress.
21 When blood calcium levels are low, parathyroid hormone (PTH) stimulates ___.
A excretion of calcium from the kidneys
B excretion of calcium from the intestines
C osteoblasts
D osteoclasts

Advanced Placement Biology Instructor’s Solution Manual


710 28 | The Endocrine System

Solution The solution is (D). When blood calcium levels are low, parathyroid hormone (PTH)
stimulates osteoclasts, which release calcium from the bones into the blood.
22 If you overproduce insulin after eating a meal, you may experience —
A diabetes mellitus
B diabetes insipidus
C hyperglycemia
D hypoglycemia
Solution The solution is (D). If you overproduce insulin after eating a meal, you may
experience hypoglycemia. Hypoglycemia occurs when blood glucose levels drop and
can be caused by overproduction of insulin.
23 Charlie has been diagnosed with acromegaly. What are the symptom(s) he may have?
A Symmetric body formation
B Excessive body growth
C Enlarged hand, feet, and face bones
D Weak bones and nervous system impairment
Solution The solution is (C). Charlie may have enlarged hand, feet, and face bones, as
enlargement of these bones can indicate acromegaly.
24 What hormone is produced by beta cells of the pancreas in response to elevated blood
glucose levels?
A T3
B Glucagon
C Insulin
D T4
Solution The solution is (C). Beta cells of the pancreas produce insulin in response to elevated
blood glucose levels.
25 There are substantially more hormones that undergo a negative-feedback loop than a
positive-feedback loop. Which hormone is regulated by a positive-feedback loop?
A Thyroxine
B Oxytocin
C Triiodothyronine
D Insulin
Solution The solution is (B). Oxytocin is regulated by a positive-feedback loop. In childbirth,
oxytocin is released when the uterus stretches and stimulates contraction, which in
turn stimulates the release of more oxytocin.

Advanced Placement Biology Instructor’s Solution Manual


28 | The Endocrine System 711

26 Which description explains what occurs if a hormone is released by a hormonal stimulus?


A Hormone release is stimulated by the nervous system.
B Hormone release is stimulated by change in the blood.
C Hormone release is stimulated by the external environment.
D Hormone release is stimulated by another hormone.
Solution The solution is (D). If a hormone is released by a hormonal stimulus, the hormone is
released by another hormone. Hormonal release means that one hormone
stimulates another.
27 A rise in blood glucose levels triggers a release of insulin from the pancreas. What
stimulus initiates insulin release?
A Humoral
B Hormonal
C Neural
D Negative
Solution The solution is (A). A humoral stimulus is a stimulus from the extracellular fluid, and
therefore initiates the mechanism of insulin release.
28 Why is the release of thyroid-stimulating hormone (TSH) considered a hormonal stimulus?
A TSH production is triggered by the nervous system.
B TSH production is triggered by blood ion concentration change.
C TSH triggers epinephrine production.
D TSH triggers the production of T3 and T4.
Solution The solution is (D). TSH release is considered a hormonal stimulus because TSH
triggers the production of T3 and T4. Hormonal stimuli occur when one hormone
stimulates another hormone.
29 What kind of stimulus promotes hormone release in response to ion concentrations in
the blood?
A Humoral stimulus
B Hormonal stimulus
C Neural stimulus
D Negative stimulus
Solution The solution is (A). A humoral stimulus, which is a stimulus from the extracellular
fluid, promotes hormone release in response to ion concentrations in the blood.
30 Which endocrine gland controls kidney function?

Advanced Placement Biology Instructor’s Solution Manual


712 28 | The Endocrine System

A Thyroid glands
B Adrenal glands
C Gonads
D Pancreas
Solution The solution is (B). Adrenal glands, which are a type of endocrine gland, are located
on top of the kidneys.
31 The islets of Langerhans are clusters of endocrine cells in the pancreas. What is the
function of the islets of Langerhans? The function of the islets of Langerhans is to
regulate —
A circadian rhythms
B secondary sex characteristics
C blood calcium levels
D blood glucose
Solution The solution is (D). The islets of Langerhans contain alpha cells, which produce
glucagon, and beta cells, which produce insulin. Glucagon and insulin regulate blood
glucose levels.
32 What do adipose tissues release after a meal?
A Erythropoietin
B Leptin
C Atrial natriuretic peptide
D Gastrin
Solution The solution is (B). Adipose tissue releases leptin and promotes satiety after you eat
a meal.

Advanced Placement Biology Instructor’s Solution Manual


28 | The Endocrine System 713

33 How do the kidneys and adrenal cortex work together to increase blood volume?
A The adrenal cortex produces renin, which affects aldosterone secretion by the kidneys.
B The kidneys produce renin, which affects aldosterone secretion by the adrenal cortex.
C The kidneys produce calcitrol, which affects renin secretion by the adrenal cortex.
D The kidneys produce calcitrol, which affects aldosterone secretion by the adrenal
cortex.
Solution The solution is (B). The kidneys produce renin, which affects aldosterone secretion
by the adrenal cortex. Renin and aldosterone work together to retain sodium and
water, which increases blood volume.
34 The anterior pituitary produces several hormones. Which hormone is NOT produced by
the anterior pituitary?
A Oxytocin
B Growth hormone
C Prolactin
D Thyroid-stimulating hormone
Solution The solution is (A). Oxytocin is not produced by the anterior pituitary; it is produced
by neurons in the hypothalamus.

CRITICAL THINKING QUESTIONS


35 Although there are many different hormones in the human body, they can be divided into
three classes based on their chemical structure. What are these classes? Give one factor
that distinguishes each.
A The classes are peptide hormones, which are water insoluble, and amino acid-derived
and lipid-derived hormones, which are water soluble.
B The classes are lipid-derived hormones, which are water insoluble, and amino acid-
derived and peptide hormones, which are water soluble.
C The classes are lipid-derived hormones, which are water soluble, and amino acid-
derived and peptide hormones, which are water insoluble.
D The classes are amino acid-derived hormones, which are water insoluble, and lipid-
derived and peptide hormones, which are water soluble.
Solution The solution is (B). The classes include lipid-derived, amino acid-derived, and peptide
hormones. Lipid-derived hormones are water insoluble, whereas amino acid-derived
and peptide hormones are water soluble.

Advanced Placement Biology Instructor’s Solution Manual


714 28 | The Endocrine System

36 How do hormones promote homeostasis in the body?


A Hormones increase cell activity by binding to the receptors present in the cell.
B Hormones facilitate communication between cells present at very distant locations in
the body.
C Hormones facilitate communication between cells at any distance in the body.
D Hormones mediate changes by directly binding to the intracellular hormone receptors.
Solution The solution is (C). Hormones facilitate communication between cells at any distance
in the body, including neighboring and very distant cells.
37 A new hormone is discovered that binds to receptors on the target cell surface. Which
chemical class of hormone could this hormone belong to, and how you could discern the
exact class?
A It belongs to the class lipid-derived or peptide hormone as it is water insoluble and
thus requires a cell surface receptor. The exact class can be determined on the basis of
the hormone’s size.
B It belongs to the class amino acid derived hormones as it is lipid insoluble and thus
requires a cell surface receptor. This can be discerned by determining the exact
structure of the hormone.
C It belongs to the class lipid-derived or peptide hormone as it is water insoluble and
thus requires a cell surface receptor. This can be discerned by determining the exact
structure of the hormone.
D It belongs to the class amino acid derived or peptide hormone as it is lipid insoluble
and thus requires a cell surface receptor. The exact class can be determined on the
basis of the hormone’s size.
Solution The solution is (D). This hormone is either an amino acid-derived or peptide
hormone because it is lipid insoluble and therefore requires a cell surface receptor.
The size of the hormone can be used to determine if it is an amino acid-derived
(small) or peptide (large) hormone.
38 Which statement explains why hormones are able to travel through the body but only
affect certain cells?
A Hormones only affect cells that have plasma membrane receptors.
B Hormones only affect cells that have corresponding hormone receptors.
C Hormones only affect cells that have intracellular receptors.
D Hormones only affect cells that are infected.
Solution The solution is (B). Hormones only affect cells that have corresponding hormone
receptors. The signaling hormone has chemical and physical properties that allow it
to selectively attach to the receptor.

Advanced Placement Biology Instructor’s Solution Manual


28 | The Endocrine System 715

39 Which statement describes the important functions of hormone receptors?


A Hormone receptors can induce cell-signaling pathways and mediate changes in target
cells in the presence of hormones.
B Hormone receptors can mediate changes in target cells and act as transcription
regulators in the presence of hormones.
C Hormone receptors can induce cell-signaling pathways and act as transcription
regulators in the presence of hormones.
D Hormone receptors can mediate changes in target cells and can stimulate signaling
pathways in the presence of hormones.
Solution The solution is (C). Hormone receptors can induce cell-signaling pathways and act as
transcription regulators in the presence of hormones. Hormones do this by
selectively attaching to the receptor.
40 How are cell surface receptors and intracellular receptors similar, and how are they
different?
A Both are types of hormone receptors at target cells. Cell surface receptors facilitate
indirect mediation, whereas intracellular receptors facilitate direct mediation.
B Both are types of hormone receptors at target cells. Cell surface receptors facilitate
direct mediation, whereas intracellular receptors facilitate indirect mediation.
C Both reside in the cytoplasm or in the nucleus. Cell surface receptors facilitate indirect
mediation, whereas intracellular receptors facilitate direct mediation.
D Both reside in the cytoplasm or in the nucleus. Cell surface receptors facilitate direct
mediation, whereas intracellular receptors facilitate indirect mediation.
Solution The solution is (A). Both are types of hormone receptors at target cells. Cell
surface receptors facilitate indirect mediation because peptide and amino acid-
derived hormones cannot pass through cell membranes, whereas intracellular
receptors facilitate direct mediation because lipid-derived hormones can pass through
cell membranes.
41 Why does drinking alcohol often result in dehydration?
A Drinking alcohol reduces antidiuretic hormone production, which is a hormone that
helps retain water.
B Drinking alcohol increases antidiuretic hormone production, which is a hormone that
helps with water loss.
C Drinking alcohol reduces thyroid-stimulating hormone production, which is a hormone
that helps retain water.
D Drinking alcohol increases thyroid-stimulating hormone production, which is a
hormone that helps with water loss.

Advanced Placement Biology Instructor’s Solution Manual


716 28 | The Endocrine System

Solution The solution is (A). Drinking alcohol reduces antidiuretic hormone production, which
is a hormone that helps retain water. Without antidiuretic hormone, water cannot
be as efficiently retained.
42 How is gonadotropin-releasing hormone production controlled by a negative-
feedback loop?
A Gonadotropin-releasing hormone decreases production of follicle-stimulating
hormone, which decreases hormone production in the gonads. An increase in gonad
hormone production then inhibits gonadotropin-releasing hormone production.
B Gonadotropin-releasing hormone stimulates production of follicle-stimulating
hormone, which regulates hormone production in the gonads. An increase in gonad
hormone production then inhibits gonadotropin-releasing hormone production.
C Gonadotropin-releasing hormone decreases production of luteinizing hormone, which
decreases hormone production in the gonads. An increase in gonad hormone
production then inhibits gonadotropin-releasing hormone production.
D Gonadotropin-releasing hormone stimulates production of luteinizing hormone, which
regulates hormone production in the gonads. An increase in gonad hormone
production then inhibits gonadotropin-releasing hormone production.
Solution The solution is (D). The hypothalamus produces gonadotropin-releasing hormone.
Gonadotropin-releasing hormone stimulates production of luteinizing hormone,
which regulates hormone production in the gonads. An increase in gonad hormone
production then inhibits gonadotropin-releasing hormone production.
43 Where is insulin stored, and under what circumstances is it released?
A It is stored in the pancreas and is released as glucose increases in the blood to
enhance the rate of glucose uptake.
B It is stored in the liver and is released as glucose increases in the blood to enhance the
rate of glucose uptake.
C It is stored in the pancreas and is released as glucose levels decrease in blood to
decrease the rate of glucose uptake.
D It is stored in the liver and is released as glucose levels decrease in the blood to
decrease the rate of glucose uptake.
Solution The solution is (A). Insulin is stored in the pancreas and is released as glucose
increases in the blood to enhance the rate of glucose uptake.
44 Ralph is always thirsty and recently learned that he synthesizes mutated antidiuretic
hormone (ADH). Why would Ralph be at higher risk for diabetes insipidus?
A ADH helps in the loss of water. Overproduction of ADH would cause improper
functioning of kidneys.
B Underproduction of ADH inhibits the release of aldosterone that would cause
improper functioning of kidneys.

Advanced Placement Biology Instructor’s Solution Manual


28 | The Endocrine System 717

C ADH helps retain water. Underproduction of ADH would cause improper functioning
of kidneys.
D ADH helps in retaining electrolytes. Underproduction of ADH causes improper
functioning of kidneys.
Solution The solution is (C). ADH helps retain water. Without ADH, an individual may drink
water constantly but still be dehydrated and kidney function cannot be properly
maintained.
45 How does stress promote water retention, which can lead to weight gain?
A Stress decreases osmolality, which increases antidiuretic hormone secretion.
Antidiuretic hormone helps retain water.
B Stress increases osmolality, which increases antidiuretic hormone secretion.
Antidiuretic hormone helps retain water.
C Stress increases osmolality, which decreases antidiuretic hormone secretion.
Antidiuretic hormone helps with water loss.
D Stress decreases osmolality, which decreases antidiuretic hormone secretion.
Antidiuretic hormone helps with water loss.
Solution The solution is (B). Stress releases several hormones that affect water balance in the
body. The overall outcome is stress increases osmolality, which increases
antidiuretic hormone secretion. Antidiuretic hormone helps retain water.
46 Which type of feedback mechanism primarily controls hormone production and release,
and why does this occur?
A The positive-feedback loop helps maintain the water concentration in the body.
B The negative-feedback loop helps maintain the water concentration in the body.
C The negative-feedback loop helps maintain proper bodily functions within a
specific range.
D The positive-feedback loop helps maintain proper bodily functions within a
specific range.
Solution The solution is (C). Hormone production and release is primarily controlled by
a negative-feedback loop to help maintain proper bodily functions within a
specific range.
47 Compare and contrast humoral and hormonal stimuli.
A Both humoral and hormonal stimuli release proteins. Humoral stimuli are hormones
that stimulate other hormones, whereas hormonal stimuli are extracellular fluid-
related stimuli.
B Both humoral and hormonal stimuli release proteins. Humoral stimuli are extracellular
fluid-related stimuli, whereas hormonal stimuli are hormones that stimulate other
hormones.

Advanced Placement Biology Instructor’s Solution Manual


718 28 | The Endocrine System

C Both humoral and hormonal stimuli release hormones. Humoral stimuli are
extracellular fluid-related stimuli, whereas hormonal stimuli are hormones that
stimulate other hormones.
D Both humoral and hormonal stimuli release hormones. Humoral stimuli are hormones
that stimulate other hormones, whereas hormonal stimuli are extracellular fluid-
related stimuli.
Solution The solution is (C). Both humoral and hormonal stimuli release hormones. Humoral
stimuli are extracellular fluid-related stimuli, whereas hormonal stimuli are
hormones that stimulate other hormones.
48 Why would it be problematic if most hormones were regulated by a positive-
feedback loop?
A Excessive production of actions would be stimulated by hormones such as growth and
blood glucose levels.
B Production of actions would be reduced by hormones such as growth and blood
glucose levels.
C Inhibition of GnRH production by increase in gonad hormone levels.
D Inhibition of release of insulin by decrease in blood glucose concentrations.
Solution The solution is (A). Specific ranges of parameters (such as ion concentration and
body temperature) need to be maintained in the body, which negative feedback
promotes. Positive feedback would result in excessive production of actions
stimulated by hormones such as growth and blood glucose levels.
49 What does aldosterone regulate, and how is it stimulated?
A Aldosterone regulates the amount of water excreted by the kidneys and causes direct
water reabsorption from the kidney tubules. It is stimulated by decreased water
concentration in blood, or increased amounts of blood potassium.
B Aldosterone regulates sodium concentrations in urine, sweat, the pancreas, and saliva.
It is stimulated by decreased blood sodium ion concentrations, blood volume, or blood
pressure, or increased amounts of blood potassium.
C Aldosterone regulates calcium concentrations in urine, saliva and the pancreas. It is
stimulated by decreased blood calcium ion concentrations, blood pressure, blood
volume, or increased amounts of blood potassium.
D Aldosterone regulates blood glucose levels by stimulating the breakdown of glycogen
to glucose. It is stimulated by decreased concentrations of glucose levels in blood,
blood volume, or blood pressure.
Solution The solution is (B). Aldosterone regulates sodium concentrations in the urine, sweat,
pancreas and saliva when stimulated decreased blood sodium levels or increased
amounts of blood potassium, which aldosterone stimulates the release of into the
urine for removal from the body.

Advanced Placement Biology Instructor’s Solution Manual


28 | The Endocrine System 719

50 Which aspect of the endocrine system often renders extremely thin females less fertile,
and why does this occur?
A Adipose tissue releases leptin, which is needed to produce gonadotropin-releasing
hormone and gonadotropin. Leptin cannot be released without sufficient body fat.
B Adipose tissue releases thymosins needed to produce gonadotropin-releasing
hormone and gonadotropin. Thymosins cannot be produced without sufficient
body fat.
C Adipose tissue releases leptin needed to produce gonadotropin-releasing hormone
and gonadotropin. Leptins cannot be produced in the absence of body fat.
D Adipose tissue releases leptin needed to produce estrogens and progesterone. Leptin
cannot be released without sufficient body fat.
Solution The solution is (A). Adipose tissues release leptin, which is needed to produce
gonadotropin-releasing hormone and gonadotropin. Without sufficient body fat,
leptin cannot be released.
51 The adrenal medulla secretes two types of hormones. What is secreted, and what are
their functions?
A They secrete cortisol and aldosterone, which increase heart rate, breathing rate,
muscle contractions, blood pressure, and blood glucose as short-term stress
responses.
B They secrete epinephrine and norepinephrine, which increase heart rate, breathing
rate, muscle contractions, blood pressure, and blood glucose as long-term stress
response.
C They secrete cortisol and aldosterone, which increase heart rate, breathing rate,
muscle contractions, blood pressure, and blood glucose as long-term stress responses.
D They secrete epinephrine and norepinephrine, which increase heart rate, breathing
rate, muscle contractions, blood pressure, and blood glucose as short-term stress
responses.
Solution The solution is (D). The adrenal medulla contains secretory cells that produce
epinephrine and norepinephrine, which increase heart rate, breathing rate, muscle
contractions, blood pressure, and blood glucose as short-term stress response (fight-
or-flight response).

Advanced Placement Biology Instructor’s Solution Manual


720 28 | The Endocrine System

TEST PREP FOR AP® COURSES


52 There are three types of hormones based on chemical structure: lipid-derived, amino
acid-derived, and peptide hormones. Which structure represents a hormone that is lipid-
derived?

A C

B
D
Solution The solution is (A). Structure a represents estradiol, which is compositionally similar
to cholesterol.
53 Hormones are essential for facilitating communication between cells, which can help
maintain homeostasis within the body. How do hormones facilitate communication on a
molecular level?
A A hormone is released in response to a stimulus, travels through the body, and then
binds to receptors of the target cell to elicit a response.
B A hormone is released from the pituitary gland in response to a stimulus, travels
through the body, and then binds to a receptor of a target cell to elicit a response.
C A hormone is released in response to a stimulus, travels through the body, and then
binds to receptors to stimulate the signaling pathway.
D A hormone is released in response to a stimulus, travels through the body and then
binds to the intracellular receptors of target cells to elicit a response.
Solution The solution is (A). A hormone is released in response to a particular stimulus,
travels through the body, and then binds to receptors of the target cell to elicit a
response. The signaling hormone has chemical and physical properties that allow it
to selectively attach to the receptor the target cell.
54 Steroid hormones are known to circulate in the blood longer than peptide hormones.
Why does this occur?
A Peptide hormones cannot pass through cell membranes.
B Steroid hormones are water insoluble.
C Peptide hormones are water insoluble.
D Steroid hormones cannot pass through cell membranes.

Advanced Placement Biology Instructor’s Solution Manual


28 | The Endocrine System 721

Solution The solution is (B). Because steroid hormones are water insoluble, they are more
difficult to degrade in the blood than peptide hormones, which are water soluble.
55 Lipid-derived hormones, such as steroid hormones, utilize intracellular receptors, whereas
peptide and amino acid-derived hormones utilize cell surface receptors. Why do these
hormones utilize different types of receptors?
A Lipid-derived hormones have receptors located in the nucleus, and thus utilize
intracellular receptors, whereas peptide and amino acid-derived hormones have
receptors only on the surface of the cell.
B Lipid-derived hormones can permeate the plasma membrane and thus utilize
intracellular receptors. Peptide and amino acid-derived hormones are lipid insoluble
and thus require cell surface receptors.
C Lipid-derived hormones can permeate plasma membranes as they need to remain in
circulation for a longer duration. Peptide and amino acid-derived hormones are lipid
insoluble and need surface receptors.
D Lipid-derived hormones can permeate plasma membranes and thus utilize
intracellular receptors. Some peptide and amino acid-derived hormones can cross the
membrane but most are lipid insoluble and thus require cell surface receptors.
Solution The solution is (B). Lipid-derived hormones can permeate plasma membranes of
cells and therefore utilize intracellular receptors, whereas peptide and amino acid-
derived hormones are lipid insoluble and therefore require cell surface receptors for
signals to be passed to the inside of the cell.
56 There are three types of hormones based on chemical structure: lipid-derived, amino
acid-derived, and peptide hormones. Which structure represents the peptide hormone?

A C

B
D
Solution The solution is (A). Structure (A) represents estradiol, which is compositionally
similar to cholesterol.

Advanced Placement Biology Instructor’s Solution Manual


722 28 | The Endocrine System

57 Cellular activity can vary based on sensitivity to hormones, and cellular activity can
therefore either be up-regulated or down-regulated by those hormones. What would
likely cause a greater response from cells that are controlled by a hormone?
A Hormone levels increase and the number of target cell receptors increase.
B Hormone levels decrease and the number of target cell receptors increase.
C Hormone levels increase and the number of target cell receptors decrease.
D Hormone levels decrease and the number of target cell receptors decrease.
Solution The solution is (A). Increasing hormone concentration, as well as the number of
receptors in the target cells, would likely cause a greater response from cells that are
controlled by that hormone.
58 What kind of hormone is undergoing binding in this figure? Explain how you know.

A A lipid-derived hormone because it is fat insoluble and therefore able to bind to


receptors on the outer surface of the plasma membrane
B A lipid-derived hormone because it is fat soluble and therefore able to pass through
the cell membrane to reach intracellular receptors
C A polypeptide-derived hormone because it is fat soluble and therefore able to pass
through the cell membrane to reach intracellular receptors
D A polypeptide-derived hormone because it is fat insoluble and therefore binds to
receptors on the outer surface of the plasma membrane

Advanced Placement Biology Instructor’s Solution Manual


28 | The Endocrine System 723

Solution The solution is (A). This figure shows the hormone passing through the cell
membrane and being transported to the nucleus. A lipid-derived hormone is
depicted because it is fat soluble and therefore able to pass through the cell
membrane to reach intracellular receptors.
59 In this figure, what kind of hormone is bound to the target cell receptor?

A Steroid
B Lipid-derived
C Estradiol
D Amino acid-derived
Solution The solution is (D). In this figure, an amino acid-derived hormone is bound to the
target cell receptor, because amino acid-derived hormones are lipid insoluble and
therefore require cell surface receptors to stimulate signals.
60 Blood pressure and blood volume are increased by the production of the hormones
antidiuretic hormone (ADH) and aldosterone. How does renin promote release of ADH
and aldosterone?
A Renin cleaves angiotensinogen.
B Renin directly simulates ADH and aldosterone production.
C Renin produces angiotensin II.
D Angiotensin I is converted into angiotensin II.
Solution The solution is (A). Renin promotes release of ADH and aldosterone by cleaving
angiotensinogen. Cleaving of angiotensinogen produces angiotensin I; angiotensin I
forms angiotensin II, which directly stimulates ADH and aldosterone production.

Advanced Placement Biology Instructor’s Solution Manual


724 28 | The Endocrine System

61 Antidiuretic hormone (ADH) is essential for water regulation in the kidneys. Once released
from the pituitary, ADH travels through to the kidneys. How does ADH promote water
reabsorption?
A ADH initiates a series of events leading to release of more vasopressin hormone in the
kidneys, leading to the movement of water out of kidneys.
B ADH initiates a series of events leading to temporary insertion of aquaporins in the
kidneys, through which water moves in the kidneys.
C ADH initiates a series of events leading to temporary insertion of aquaporins in the
kidneys, through which water moves out of the kidneys.
D ADH initiates a series of events leading to the movement of water out of the kidneys
through simple diffusion.
Solution The solution is (C). Antidiuretic hormone has the overall effect of conserving or
retaining body water. ADH initiates a series of events that lead to the temporary
insertion of aquaporins, or water channels, in the kidneys, through which water
moves out of the kidneys.
62 David, an athlete, wants to enhance his baseball performance by taking erythropoietin.
What David is trying to do?
A Build more muscle
B Improve endurance
C Reduce fertility
D Decrease need to urinate
Solution The solution is (B). David is trying to improve his endurance. Erythropoietin
stimulates red blood cell production, which increases oxygen delivery to muscles.
63 Some athletes may want to take synthetic hormones to improve their performance in
their given sport. However, the use of certain synthetic hormones is banned in many
professional sports. Why are synthetic hormones often banned?
A There can be severe side effects such as insomnia, depression, and prostate
enlargement. These side effects are often severe and irreversible.
B There can be severe side effects such as impaired heart function, testicular atrophy,
and prostate enlargement. These side effects can be cured through surgeries.
C There can be severe side effects such as impaired heart function, testicular atrophy,
and prostate enlargement. These are often severe and irreversible.
D There can be severe side effects such as insomnia, depression, and prostate
enlargement. These side effects can be cured through surgeries.
Solution The solution is (C). Synthetic hormones bind similarly to the receptor of the
analogous naturally produced hormone. Binding of the synthetic hormone to
analogous receptors is not regulated by the body, and there can be severe side

Advanced Placement Biology Instructor’s Solution Manual


28 | The Endocrine System 725

effects to synthetic hormone use, especially when used non-medically, such as


impaired heart function, testicular atrophy, and prostate enlargement. These side
effects are often severe and irreversible.
64 This figure shows how growth hormone communicates with cells to promote muscle and
bone growth.

What is growth hormone’s exact role?


A To inhibit growth hormone-inhibiting hormone release
B To stimulate growth hormone-releasing hormone release
C To break down glycogen
D To activate insulin-like growth factors
Solution The solution is (D). Growth hormone’s role is to activate insulin-like growth factors.
Insulin-like growth factors promote protein synthesis by stimulating target cells to
take up amino acids.
65 Growth hormone helps promote growth by accelerating the rate of protein synthesis.
How does growth hormone switch the energy source of most cells?
A Growth hormone is able to break down triglycerides through lipogenesis. When
triglycerides are released, most tissues switch to fatty acids as energy sources.
B Growth hormone is able to break down triglycerides through lipolysis. When
triglycerides are released, most tissues switch to fatty acids as an energy source.
C Growth hormone is able to break down triglycerides through lipolysis. When
triglycerides are released, most tissues switch to glucose as an energy source.
D Growth hormone is able to break down triglycerides through lipogenesis. When
triglycerides are released, most tissues switch to glucose as an energy source.

Advanced Placement Biology Instructor’s Solution Manual


726 28 | The Endocrine System

Solution The solution is (B). Growth hormone is able to break down triglycerides through
lipolysis. When triglycerides are broken down and released, most tissues switch
from using glucose to fatty acids as an energy source.
66 Positive-feedback loops are rare in the endocrine system but some do exist. What is an
example of a positive-feedback loop?
A Insulin facilitates decrease of blood sugar levels.
B Oxytocin release stimulates milk release.
C Increased blood calcium levels halt PTH production.
D Increased amounts of T3 and T4 inhibit further production.
Solution The solution is (B). Oxytocin release stimulating milk release is an example of a
positive-feedback loop. Increased suckling by a baby promotes oxytocin, which
stimulates myoepithelial cell contraction around milk-producing mammary glands.
Increased suckling continues to release oxytocin and promote milk secretion.
67 Although positive-feedback loops are rare in the endocrine system, they are present in
childbirth. How is childbirth controlled by a positive-feedback loop?
A When a child pushes on the cervix, a signal is sent to stimulate oxytocin release, which
stimulates more contractions. This promotes more oxytocin release that allows the
child to be pushed through the birth canal.
B When a child pushes on the cervix, a signal is sent to stimulate oxytocin release, which
stimulates contractions. This promotes the release of progesterone that allows the
child to be pushed through the birth canal.
C When a child pushes on the cervix, a signal is sent to stimulate prolactin release, which
stimulates more contractions. This promotes the release of more prolactin that allows
the child to be pushed through the birth canal.
D When a child pushes on the cervix, a signal is sent to stimulate progesterone release,
which stimulates contractions. This promotes the release of oxytocin that allows the
child to be pushed through the birth canal.
Solution The solution is (A). The role of oxytocin when a mammal gives birth is to stimulate
uterine contractions. Oxytocin stimulates uterine contractions by a positive-
feedback loop.
68 Osmoreceptors are essential for monitoring water concentrations within the body. How
do osmoreceptors complete this task?
A Osmoreceptors insert aquaporins in the kidneys.
B Osmoreceptors signal increase sodium reabsorption.
C Osmoreceptors detect when blood electrolyte levels change.
D Osmoreceptors signal increased sodium reabsorption.

Advanced Placement Biology Instructor’s Solution Manual


28 | The Endocrine System 727

Solution The solution is (C). Osmoreceptors monitor water concentrations in the body by
detecting when blood electrolyte levels change. When osmoreceptors detect change
in blood electrolyte levels, the amount of antidiuretic hormone secreted is
stimulated to change.
69 Laura has type 1 diabetes and her body cannot properly produce insulin in response to
elevated blood glucose levels. To which type of stimulus is Laura unable to respond?
A Humoral
B Hormonal
C Neural
D Negative
Solution The solution is (A). Laura is unable to respond to humoral stimuli if her body cannot
produce insulin in response to elevated blood glucose levels, because a humoral
stimulus is a stimulus from the extracellular fluid.
70 Terry recently gained weight and has been more tired than usual. Terry’s doctor
suggested that he might not produce enough thyroid-stimulating hormone (TSH). Why
would a TSH deficiency cause Terry’s weight gain and fatigue?
A Without TSH, there would be excessive production of T3 and T4 leading to a high
metabolic rate, causing weight gain and fatigue.
B Without TSH, there would be excessive production of T3 and T4 leading to a low
metabolic rate, causing weight gain and fatigue.
C Without TSH, T3 and T4 cannot be properly produced, leading to a high metabolic rate,
causing weight gain and fatigue.
D Without TSH, T3 and T4 cannot be properly produced, leading to a low metabolic rate,
causing weight gain and fatigue.
Solution The solution is (D). TSH stimulates the production of T3 and T4, which increase
metabolism. Without TSH, T3 and T4 cannot be properly produced, and Terry would
have a low metabolic rate, causing weight gain and fatigue.
71 Marcus experienced nervous system damage in a car accident. Which endocrine-related
body function will most likely be impaired as a result?
A Ability to lower blood glucose levels
B Fight-or-flight response
C Urine production
D Body heat regulation
Solution The solution is (B). The fight-or-flight response might be impaired if Marcus
experienced nervous system damage, because epinephrine and norepinephrine,
which promote the fight-or-flight response, are triggered by neural stimuli.

Advanced Placement Biology Instructor’s Solution Manual


728 28 | The Endocrine System

SCIENCE PRACTICE CHALLENGE QUESTIONS


28.1 Types of Hormones
72 Many hormones repress or enhance protein expression by binding to transcription factors
called nuclear receptors. Other hormones regulate processes by binding to cell surface
receptors that induce cell signaling cascades.
A. Represent these two different processes for cell-cell communication by creating a flow
chart that shows the direction of information in a process regulating homeostasis, such as
the control of blood sugar or water loss. Annotate your diagram to describe the key
elements of each step in the particular example that you choose.
B. Justify the claim that a developmental process is more likely to involve nuclear
receptors, while regulation of metabolism is more likely to involve cell surface receptors.
The lancelet is a chordate, fish-like filter feeder that buries itself in marine sands. Their
ancestors and vertebrates diverged approximately 500 million years ago. Lancelets and
humans are members of a superphylum of animals with bilateral symmetry,
Deuterostomia. In one model of animal evolution, the other superphylum, Protostomia,
includes the fruit fly and nematode. In an alternative model, insects are included with
lancelets and humans in a superphylum called the Coelomata, and the nematode is
separate. Molecular studies of the fruit fly, lancelet, human, and nematode show that
homologous genes, when present, are strongly conserved. For example, the genes for the
receptors of germ cell nuclear factor (GCNF) and retinoids, hormones that regulate
cellular differentiation during development, are present in each species. Homologous
genes for nuclear receptor estrogen and thyroid hormone are present in lancelets and
humans, but are missing in nematodes and fruit flies. Genes for the vitamin D receptor are
absent in fruit flies, but are present in the other species. Many neuropeptide hormones
found in humans, fruit flies, and nematodes such as GnRH (gonadotropin-releasing
hormone) and the gonadotropins luteinizing and follicle stimulating hormones (LH and
FSH) have not been found in lancelets. However, genes for the cell surface receptors
activated by these hormones are expressed. The same is true for many different
hormones and their corresponding receptors. A large number of such “orphan” nuclear
receptors have been reported in animals where there is no evidence of the presence of
the hormone to which they bind.
C. Analyze these data (a table to organize the data is suggested) for supporting evidence
of the alternative superphylum, Coelomata. Consider:
 Separate selection of signaling molecules and the proteins that detect them
 Constitutive (always transcribed) expression
 The possible effect of gene deletion
D. Evaluate the legitimacy of a claim that a hormone is not present although the receptor
for that hormone has been found. Genes for receptors and the hormones to which they
bind are often found on different chromosomes and have co-evolved. Other ligands may

Advanced Placement Biology Instructor’s Solution Manual


28 | The Endocrine System 729

have an affinity for orphaned receptors. By understanding the three-dimensional shape of


the ligand binding site of a nuclear receptor, new drugs can be developed to match the
site and regulate expression when the endocrine system fails. Even greater flexibility in
drug design is allowed when small molecules, called positive (PAM) and negative (NAM)
allosteric modulators, change the shape of the ligand binding site. The receptors for
follicle stimulating hormone (FSH) and luteinizing hormone (LH) were subjects in a drug
discovery program using PAMs and NAMs (Nataraja et al., Frontiers in Endocrinology,
6, 2015).
E. Explain how the refinement of nuclear receptor by attachment of a molecule other
than the ligand of the receptor can increase the level of expression of genes targeted by
the transcription factor.
Solution Sample answer:
A. The AP Biology Curriculum Framework was designed to shift the emphasis on the
AP Exam from memorization to reasoning. At multiple points in the Framework, such
as the description of regulation of homeostasis (Enduring Understanding 2D), many
example systems are suggested. You should expect to be asked to use one of these
to illustrate this concept. Here are three of the many possible answers to this
question:
B. One form of evidence is the time involved in a feedback loop if transcription is
required. More rapid response to changes in hormone concentrations can be
achieved with signaling cascades. Important examples are the andrenergic receptors
for epinephrine and norepinephrine and the insulin receptor. A second form of
evidence is that nuclear receptors acting as transcription factors control the
expression of DNA directly. Developmental processes involve carefully controlled
timing in sequences of expression. Examples here are the steroids estradiol and
testosterone. But the distinction between metabolic and growth regulation is not
sharp—growth involves changes in metabolism. For example, growth hormone has a
surface receptor.
Thyroid/ GCNF/
Species Peptide-Based Estrogen Retinoid Vitamin D
Human Hormone and Yes Yes Yes
receptor
Lancelet Just receptor Yes Yes Yes
Fruit fly Hormone and No Yes No
receptor
Nematode Hormone and No Yes Yes
receptor
C. There is no evidence to suggest any more complex phylogeny than a last common
ancestor of both Deuterostomia and Protostomia, with insects and nematodes
grouped together. That superphylum is referred to as Ecdysozoa. The absence of a

Advanced Placement Biology Instructor’s Solution Manual


730 28 | The Endocrine System

vitamin D receptor in the fruit fly can be a gene deletion. Similarly, the absence of
hormones in the lancelet could be gene deletion. However, it might also be that the
regulatory function has been lost and the expression is constitutive. And it also could
be that the hormones just haven’t been seen; see (D). Gene deletions in the nuclear
receptors are widespread among animals (see Bertrand et al., Molecular Biology and
Evolution 21, 2004; King-Jones and Thummel, Nature Reviews 6, 2005).
D. The rule is that absence of evidence is not evidence of absence.
E. The three-dimension structure of a polymer can be modified through the
interaction with another molecule. And when the molecule is not interacting with
the active site, it can change the dynamics of interaction of the polymer with a
substrate. This is called allosteric modulation and this is the enduring understanding
described in 4A of the Framework.
28.2 How Hormones Work
73 Gonadotrophin-releasing hormone (GnRH) stimulates the production of follicle-
stimulating hormone (FSH) and luteinizing hormone (LH). The concentration of these
hormones over time in females regulates the menstrual cycle. GnRH expressing neurons
(GEN) are scattered throughout the forebrain and hypothalamus and secrete into ducts
leading to the pituitary gland, causing the secretion of FSH and LH into the bloodstream.
concentrations oscillate in time, changing the electric membrane potential of the
GnRH neurons, GEN. The period of the oscillation is only a few seconds in duration. The
mechanism controlling the oscillating membrane potential is unknown, but during
maturation the frequency of the oscillations of the concentration in the GEN
increases.
A. Explain how the absence of direct contact among the GnRH neurons and the
coordinated release of FSH and LH suggest another timing mechanism.An association of
high concentrations of anti-Müllerian hormone (AMH) and elevated LH levels among
women with a common reproductive disorder called polycystic ovary syndrome (PCOS)
caused investigators to study the effects of AMH on membrane potentials of GnRH
expressing neurons. The researchers (Cimino et al., Nature Communications 7, 2016)
found that very small concentrations of AMH initiated oscillations.
B. The graph illustrates the time dependence of LH and estrogen in the blood of a female.
A goal of research in human reproductive physiology is to construct a model that accounts
for the spike in luteinizing hormone at approximately 14 days. Predict how a positive-
feedback loop involving the release of a hormone such as AMH could be used to account
for this response. Explain why estrogen is probably not a good candidate for use in
this model.

Advanced Placement Biology Instructor’s Solution Manual


28 | The Endocrine System 731

FSH and LH inhibit production of GnRH. Exposure of ovarian follicles to high levels of FSH
and LH causes the follicle to rupture, releasing an oocyte to the oviduct. Birth control
drugs suppress the LH surge by maintaining a constant blood concentration of
progesterone (PG). After 40 years of use, the mechanism of suppression is still unknown.
However, we do know that during pregnancy progesterone levels remain elevated and
that cells in both the pituitary gland and GnRH expressing neurons (GEN) have
progesterone nuclear receptors (PGR) that act as transcription factors to either (i) inhibit
production of GnRH possibly by blocking transcription of GNRH1, the gene coding for
GnRH, or (ii) inhibit production of LH by blocking transcription in the GnRH neurons of the
gene, LH1, coding for LH.
C. Construct two alternative explanations of the role of progesterone as a birth control
drug. Your explanation can be in the form of descriptions or in terms of diagrams involving
the key molecular components PG, PGR, LH, LH1, GEN, GnRH, and GnRH1. An endocrine
disruptor is a molecule with a structure that mimics a hormone and can interfere with
regulation. The chemical bis-phenol A (BPA) has been used for many years in the
production of plastics and is now widespread. BPA mimics estrogen. BPA is present in
most humans at the concentration of 5 micrograms per kilogram of body weight. Whether
or not that concentration is harmful is uncertain.

In a study of oscillations in GnRH expressing neurons, Klenke and coworkers


(Endocrinology 157, 2016) presented evidence that BPA switches off oscillations even at
these very low concentrations. This suggests a mechanism for the results from an earlier
investigation in which female rats were fed low and high levels of estrogen (EE) and BPA
during pregnancy. Measurements were made of the levels of expression of an estrogen
nuclear receptor in the hypothalamus of the pups just after birth (Cao et al., Toxicological
Sciences 133, 2013). Their data are shown in the graph.

Advanced Placement Biology Instructor’s Solution Manual


732 28 | The Endocrine System

Using the data, the graph at the left compares expression in a “do-nothing” control
(Naïve) and a control (Vehicle) in which a feeding tube is used to deliver nutrients during
pregnancy. The graph at the right shows expression levels in pups born to female rats with
four different treatments of estrogen (EE) and bis-phenol A (BPA).
D. Compared to the naïve control where homeostasis is not disrupted, qualitatively
analyze these data for the disruption of the endocrine system in offspring of a female rat
exposed to both physical and chemical stresses.
E. In female sheep, the estrus cycle is coordinated by the length of the day and does not
begin until day length shortens. The gestation period of sheep is roughly 150 days.
 Predict the effect on fitness of this control of gene expression by an external
stimulus.
 The “ram effect” is a livestock management practice in Australia to induce
ovulation by exposing an anestrus (when the estrus cycle is inactive) ewe to a
ram. Compare this phenomenon with the results of delivering nutrients and
hormones using a feeding tube to construct a possible explanation for the ram
effect.
Solution Sample answer:
A. If the time-dependent response of multiple cells is coordinated, then there must
be information sharing among the cells.
B. If a small messenger molecule present at very low levels, like AMH, whose role
may not have been thus far detected, functions in a positive feedback with
production of the messenger as well as the response, the positive feedback and
integrated response could result in a surge. Estrogen has been considered as the
initiator of the surge. However, this has led to a paradox that has not been resolved.
Estrogen must then both suppress the surge (pregnant women suppress the LH
surge with high constant levels of estrogen and a high constant level of estrogen is a
component of contraceptive drugs) and enhance the surge. The idea of a critical
level with positive and negative feedback that depends on position relative to the
critical level is often used as a model.
C. Progesterone may be docking at the PGR of the nuclear receptor or it may be
docking at the PGR of the pituitary gland to block the effect of the GnRH in the

Advanced Placement Biology Instructor’s Solution Manual


28 | The Endocrine System 733

secretion of LH.
D. These data are very provocative. A ban on BPA would involve retooling our world
of packaging and would (if the past predicts the future) lead to significant costs both
to industry and to individuals, litigation, and remediation. Most vexing is the fact
that low BPA shows a stronger effect on ER production than does high BPA. This is
opposite to observed, and expected, proportional response to EE. However, these
conclusions depend on comparison with the vehicle control which is not significantly
different from the “do-nothing” control. This paper was very careful to point this
out. Alarming reports regarding BPA have led to controversies. In one case a
researcher was accused of misrepresenting his results and lost his position and
future funding.
E. By bearing young during warmer periods survival is enhanced. The “ram effect”
has been documented carefully (Fabre-Nys et al., PLoS One 11, 2016). The authors
suggest pheromone stimulation but one could also consider the effect of stress, as
was demonstrated in the very significant response to tube feeding as shown in part.

Advanced Placement Biology Instructor’s Solution Manual


734 29 | The Musculoskeletal System

29 | THE MUSCULOSKELETAL SYSTEM


REVIEW QUESTIONS
1 Which type of animal skeletal system is formed by the pressure exerted by the aqueous
contents of the coelom on surrounding organs?
A Hydrostatic skeleton
B Exoskeleton
C Axial skeleton
D Appendicular skeleton
Solution The solution is (A). The hydrostatic skeleton is formed by the pressure exerted by
the aqueous contents of the coelom on surrounding organs. This type of skeleton is
well-suited to soft-bodied animals like invertebrates.
2 The skeletal system is necessary to support the body, protect internal organs, and
allow movement. Which organism must shed and then regenerate its skeletal system
as it grows?
A Earthworm
B Lobster
C Snake
D Sea anemone
Solution The solution is (B). A lobster must shed and then regenerate its skeletal system as it
grows. Lobsters have exoskeletons that, because they are acellular, need to be shed
and regenerated to allow for growth.
3 What is a role of the human vertebral column?
A To protect the heart and lungs
B To support the head
C To attach the upper limbs to the axial skeleton
D To permit locomotion
Solution The solution is (B). The role of the human vertebral column is to support the head.
4 Abnormalities of the thoracic cage are relatively rare, but they can cause disorders when
they exist. What would most be at risk if someone had a defective thoracic cage?
A Middle ear
B Brain

Advanced Placement Biology Instructor’s Solution Manual


29 | The Musculoskeletal System 735

C Spinal cord
D Heart
Solution The solution is (D). If someone had a defective thoracic cage, the heart would be
most at risk because the thoracic cage protects the heart.
5 Which type of animal skeletal system comprises a hard skeletal layer found on the
exterior of the organism?
A Hydrostatic skeleton
B Exoskeleton
C Axial skeleton
D Appendicular skeleton
Solution The solution is (B). The exoskeleton comprises a hard exterior skeletal layer that
provides protection against predators while supporting the body.
6 Which type of organism moves by peristaltic muscle contraction, extending its body in
one direction, then shortening and pulling itself along a surface?
A Ants
B Lizards
C Earthworms
D Sponges
Solution The solution is (C). Earthworms move by extending their body, gripping the surface,
and pulling the back of the body forward.
7 The cells responsible for bone resorption are —
A osteoclasts
B osteoblasts
C fibroblasts
D osteocytes
Solution The solution is (A). The cells responsible for bone resorption are osteoclasts.
Osteoclasts are large bone cells with up to 50 nuclei that remove bone structure by
releasing lysosomal enzymes and acids that dissolve bony matrix, helping with the
regulation of the blood’s calcium concentration. The dissolved bone material may
also be resorbed, a process important during bone remodeling.

Advanced Placement Biology Instructor’s Solution Manual


736 29 | The Musculoskeletal System

8 The bones of human skeletons are classified by their shape. How does the structure of
sesamoid bones differ from that of short bones?
A Sesamoid bones are small and flat, whereas short bones are approximately equal in
height and width.
B Sesamoid bones are thin and broad, whereas short bones have a central shaft and
two ends.
C Sesamoid bones are small, flat, and irregularly shaped, whereas short bones are
cuboidal in shape.
D Sesamoid bones are thin and broad, whereas short bones are small and flat.
Solution The solution is (A). Sesamoid bones are small and flat, whereas short bones are
approximately equal in height and width.
9 The cells responsible for bone resorption are_____.
A Osteoclasts
B Osteoblasts
C Fibroblasts
D Osteocytes
Solution The solution is (A). The cells responsible for bone resorption are osteoclasts.
Osteoclasts are large bone cells with up to 50 nuclei that remove bone structure by
releasing lysosomal enzymes and acids that dissolve bony matrix, helping with the
regulation of the blood’s calcium concentration. The dissolved bone material may
also be resorbed, a process important during bone remodeling.
10 A patient was diagnosed with low calcium levels in the blood, which affect nervous system
and muscle functions. You decide to examine the functioning of various cells composing
the patient’s bone tissue.
Which type of bone cell is most likely dysfunctional?
A Osteoclasts
B Osteoblasts
C Osteoprogenitor cells
D Osteocytes
Solution The solution is (A). Osteoclasts may be dysfunctional. Osteoclasts are large bone
cells with up to 50 nuclei that remove bone structure by releasing lysosomal
enzymes and acids that dissolve the bony matrix. Dissolving the bony matrix releases
calcium into the blood. Therefore, a low calcium concentration in the blood could
result from osteoclast malfunction.
11 Which bones or bone structures form primarily due to endochondral ossification?
A Most of the cranial bones

Advanced Placement Biology Instructor’s Solution Manual


29 | The Musculoskeletal System 737

B Flat bones of the face


C Cartilage
D Femur
Solution The solution is (D). The femur bone forms primarily due to endochondral
ossification. Endochondral ossification is the process of bone development from
hyaline cartilage. All of the bones of the body (including the femur), except the flat
bones of the skull, mandible, and portions of the clavicles, are formed this way.
Cartilage is formed by chondrification, not ossification.

Advanced Placement Biology Instructor’s Solution Manual


738 29 | The Musculoskeletal System

12 What outcome might you expect in an individual with premature epiphyseal plate
closure?
A Overly thick bones
B Short stature
C Tall stature
D Overly thin bones
Solution The solution is (B). Should epiphyseal plate closure occur prematurely, you might
expect the individual to be of short stature. Epiphyseal plate closure is associated
with the stoppage of long bone lengthening as an individual enters adulthood.
13 Which type of bone has a central shaft and two rounded ends?
A Flat bones
B Irregular bones
C Sutural bones
D Long bones
Solution The solution is (D). Long bones have a central shaft (called a diaphysis) and two
rounded ends (called epiphyses). Examples include the long limb bones like the
femur, tibia, radius, and ulna.
14 Synchondroses and symphyses are connected by —
A fibers
B synovial fluid
C cartilage
D condyloid
Solution The solution is (C). Synchondroses and symphyses are connected by cartilage.
Cartilaginous joints are joints in which the bones are connected by cartilage. The two
types of cartilaginous joints are synchondroses and symphyses.
15 What is a characteristic of the synovial fluid?
A Stability
B Lubrication
C Minor movement
D Increase of friction
Solution The solution is (B). Lubrication is a characteristic of the synovial fluid. The cavity in a
synovial joint is filled with synovial fluid. Synovial fluid lubricates the joint, reducing
friction between the bones and allowing for greater movement.
16 Which of the following is an example of a pivot joint?

Advanced Placement Biology Instructor’s Solution Manual


29 | The Musculoskeletal System 739

A Elbow
B Thumb
C Hip
D The joint of the wrist
Solution The solution is (D). The joint of the wrist that allows the palm of the hand to be
turned up and down is a pivot joint.
17 Synovial joints allow different types of movement. Turning your head to look over your
shoulder is an example of —
A abduction
B medial rotation
C lateral rotation
D adduction
Solution The solution is (C). Turning your head to look over your shoulder is an example of
lateral rotation. Lateral rotation is the movement of a bone as it rotates away from
the midline of the body.

Advanced Placement Biology Instructor’s Solution Manual


740 29 | The Musculoskeletal System

18 Which of the following is a fibrous joint?


A Suture
B Planar joint
C Hinge joint
D Pivot joint
Solution The solution is (A). A suture is a fibrous joint. There are three types of fibrous joints:
sutures, syndesmoses, and gomphoses.
19 Joints are classified based on the material composing the joint and the presence or
absence of a cavity in the joint. Which of the following are the weakest joints?
A Synchondrosis
B Fibrous
C Synovial
D Symphyses
Solution The solution is (C). Synovial joints are the weakest joints. Synovial joints are capable
of the greatest movement of the three structural joint types. The more mobile a
joint, the weaker the joint.
20 Which muscles are also called voluntary muscles?
A Cardiac
B Smooth
C Connective
D Skeletal
Solution The solution is (D). Skeleton muscles are also called voluntary muscles. Because it
can be controlled by thought, skeletal muscle is also called voluntary muscle.
21 Which statement best describes a function of smooth muscle tissue?
A It affects the flow of blood and blood pressure.
B It stimulates contraction of the heart.
C It changes your facial expression.
D It maintains your posture.
Solution The solution is (A). Smooth muscle tissue determines the flow of blood in the
arteries. Smooth muscle tissue is an involuntary muscle found around passages such
as blood vessels.
22 In relaxed muscle, the myosin-binding site on actin is blocked by —
A ATP

Advanced Placement Biology Instructor’s Solution Manual


29 | The Musculoskeletal System 741

B tropomyosin
C the cross-bridge
D troponin
Solution The solution is (B). When the muscle relaxes, actin and myosin are separated. To
keep actin from binding to the active site on myosin, tropomyosin blocks myosin-
binding sites on actin molecules.
23 Which statement does NOT accurately describe muscle contraction?
A During muscle contraction, the sarcomere shortens.
B During muscle contraction, the thick and thin filaments shorten.
C During muscle contraction, the H zone shortens.
D During muscle contraction, the A band does not shorten.
Solution The solution is (B). For a muscle cell to contract, the sarcomere must shorten. The
thick and thin filaments do not shorten. Instead, they slide by one another, causing
the sarcomere to shorten while the filaments remain the same length.
24 The small structures that make up myofibrils are called —
A sarcolemma
B myosins
C myofilaments
D sarcomeres
Solution The solution is (C). The small structures that make up myofibrils are called
myofilaments. Myofibrils are composed of smaller structures called myofilaments.

CRITICAL THINKING QUESTIONS


25 You discover an unusual new organism in the deep sea and are studying its skeletal
system in a laboratory aquarium. The organism appears to have a central, fluid-filled
cavity. The fluid within the cavity is incompressible and allows the organism to move by
peristalsis.
What kind of skeletal system does this organism most likely have?
A Exoskeleton
B Hydrostatic skeleton or exoskeleton
C Hydrostatic skeleton
D Endoskeleton
Solution The solution is (C). Based upon the description of the organism and its habitat, these
characteristics are consistent with it being a tube-shaped invertebrate, which
commonly has a hydrostatic skeleton. The skeleton involves a cavity containing

Advanced Placement Biology Instructor’s Solution Manual


742 29 | The Musculoskeletal System

incompressible fluid that is propelled forward when body muscles are contracted.
An alternating pattern of contraction and relaxation against a hydrostatic skeleton
creates movement through peristalsis.
26 You discover the carcass of an unusual new organism on the beach while on vacation. As a
scientist, you decide to chemically analyze the carcass in order to characterize it. The
chemical analysis reveals significant amounts of chitin. What other molecule might you
also find associated with the skeletal system of this organism?
A Calcium carbonate
B Calcium phosphate
C Hydroxyapatite
D Magnesium carbonate
Solution The solution is (A). The presence of chitin suggests that this organism has an
exoskeleton. One would expect to additionally find calcium carbonate, a molecule
commonly found in combo with chitin to strengthen the exoskeleton.
27 What types of issues related to bone function would you predict for an individual suffering
from a genetic lysosomal storage disease?
A Individuals may have deformities in the appendicular skeleton and possible phosphate
homeostasis issues.
B Individual may have skeletal deformities and possible calcium homeostasis issues.
C Individuals may have axial skeleton deformities and possible calcium homeostasis
issues.
D Individual’s brain would be more prone to injury and a possible calcium homeostasis
issues.
Solution The solution is (B). Osteoclasts are large multi-nucleated cells that are responsible
for releasing lysosomal contents to dissolve the bony matrix. In doing so, properly
functioning osteoclasts contribute to both maintaining proper calcium homeostasis
and bone remodeling. Thus, one could predict that individuals suffering from a
genetic lysosomal storage disease may have skeletal deformities and possible
calcium homeostasis issues.
28 A world-class pianist gets into a serious car accident. Injuries to which group of bones
would most significantly interfere with her career as a pianist?
A Injuries to carpals, humerus, radius, ulna, clavicle, metacarpals, metatarsals, tarsals,
and various flat bones like scapulae would cause issues.
B Injuries to carpals, humerus, radius, ulna, clavicle, metacarpals, phalanges, and various
flat bones like scapulae would cause issues.
C Injuries to carpals, humerus, radius, ulna, clavicle, metacarpals, femur, tibia, and
various flat bones like scapulae would cause issues.

Advanced Placement Biology Instructor’s Solution Manual


29 | The Musculoskeletal System 743

D Injuries to carpals, humerus, femur, tibia, metatarsals, tarsals, phalanges, and various
flat bones like scapulae would cause issues.
Solution The solution is (B). Injuries to short bones (carpals) of the wrist will be problematic,
as will injuries to various long bones like those of the arm (humerus, radius, or ulna),
or the clavicle. Injuries to the long bones in the hand (metacarpals and phalanges)
would also be problematic. Additionally, injuries to various flat bones, including the
scapulae (shoulder blades), would also cause issues.
29 What issue related to bone development and repair would you expect to be associated
with someone suffering from osteogenesis imperfecta, a genetic disorder most commonly
resulting from defects in collagen production and processing?
A Bones that are misshapen due to excessive levels of calcification
B Abnormally weak bones prone to breakage and fracturing upon mild trauma
C Very long bones that confer above-average height but are unusually flexible
D Bones that do not have adequate innervation or vascularization
Solution The solution is (B). Collagen is required for bone formation during both development
and repair. Calcification during ossification requires the presence of collagen and it is
calcification that gives bones more strength. Thus, one would expect that individuals
suffering from this condition would have abnormally weak bones prone to breakage
and fracturing upon mild trauma.
30 Osteoporosis is a clinical manifestation of an imbalance between bone resorption and
bone formation, resulting in weakened bones that easily fracture. How might one design a
therapeutic drug targeting specific bone cell type(s) for the treatment of osteoporosis?
A A therapeutic drug that inhibits osteoblast activity would be useful. A drug that
enhances the bone formation activity of osteoclasts may also be a good strategy.
B A therapeutic drug that inhibits osteoclast activity would be useful. One that reduces
the activity of osteoblasts may also be a good strategy.

Advanced Placement Biology Instructor’s Solution Manual


744 29 | The Musculoskeletal System

C A therapeutic drug that inhibits osteoblast activity would be useful. One that enhances
the bone formation activity of osteoblasts may also be a good strategy.
D A therapeutic drug that inhibits osteoclast activity would be useful. One that enhances
the bone formation activity of osteoblasts may also be a good strategy.
Solution The solution is (D). Resorption is carried out by osteoclasts while bone formation is
carried out by osteoblasts. In osteoporosis, osteoclast activity exceeds the bone
forming activity of osteoblasts. Thus, a therapeutic drug that inhibits osteoclast
activity would be useful for the treatment of osteoporosis. Alternatively, a drug that
enhances the bone formation activity of osteoblasts may also be a good strategy for
the treatment of osteoporosis.
31 Why are shoulder dislocations more common than dislocations of the carpal bones in the
hand?
A The less mobile a joint, the weaker the joint. The shoulder joint allows the greatest
range of motion, while carpal bones are more stable.
B The less mobile a joint, the weaker the joint. Carpal bones allow the greatest range of
motion, while shoulder joints are more stable.
C The more mobile a joint, the weaker the joint. Carpal bones allow the greatest range
of motion, while shoulder joints are more stable.
D The more mobile a joint, the weaker the joint. The shoulder joint allows the greatest
range of motion, while carpal bones are more stable.
Solution The solution is (D). Dislocation happens when the bones in a joint are forced
apart and out of their usual positions. The shoulder joint is a ball-and-socket joint, which
allows the greatest range of motion. The more mobile a joint, the weaker the joint. The
carpal bones in the hand are more stable. They have planar joints which have a limited
range of motion and no rotation.
32 What angular movements occur at the hip joint and knees as you bend down to touch
your toes?
A Both the hip joint and knees are flexed.
B Both the hip joint and knees are extended.
C The hip joint is extended and the knees are flexed.
D The hip joint is flexed and the knees are extended.
Solution The solution is (D). As you bend down to touch your toes, the hip joint is flexed and
the knees are extended.
33 What is the main structural difference between synovial joints and cartilaginous and
fibrous joints?
A Synovial joints allow movement, while fibrous and cartilaginous joints do not move
at all.

Advanced Placement Biology Instructor’s Solution Manual


29 | The Musculoskeletal System 745

B Synovial joints do not move at all, while fibrous and cartilaginous joints allow
movement.
C Synovial joints do not have a space between the adjoining bones, while fibrous and
cartilaginous joints have a joint cavity.
D Synovial joints have a space between the adjoining bones, while fibrous and
cartilaginous joints do not have a joint cavity.
Solution The solution is (A). The main structural difference is the presence of a cavity in the
joint. Synovial joints are the only joints that have a space between the adjoining
bones. Fibrous joints and cartilaginous joints do not have a joint cavity.
34 What do all types of muscle tissue have in common?
A They all contain actin and myosin.
B They are all under voluntary control.
C They are all involuntary.
D They all require calcium binding to tropomyosin to contract.
Solution The solution is (A). All three types of muscle tissue have actin and myosin.

Advanced Placement Biology Instructor’s Solution Manual


746 29 | The Musculoskeletal System

35 How would muscle contractions be affected if ATP were completely depleted in a


muscle fiber?
A Myosin heads will detach rapidly from the actin-binding sites, resulting in muscle
relaxation. In a live person this causes rigor mortis, while in a recently dead person, it
results in writer’s cramp.
B Myosin heads will not detach from the actin-binding sites, resulting in muscle stiffness.
In a live person this causes rigor mortis, while in a recently dead person, it results in
writer’s cramp.
C Myosin heads will detach rapidly from the actin-binding sites, resulting in muscle
relaxation. In a live person this causes writer’s cramp, while in a recently dead person,
it results in rigor mortis.
D Myosin heads will not detach from the actin-binding sites, resulting in muscle stiffness.
In a live person this causes writer’s cramp, while in a recently dead person, it results in
rigor mortis.
Solution The solution is (D). Without ATP, the myosin heads cannot detach from the actin-
binding sites. All of the stuck cross-bridges result in muscle stiffness. In a live person,
this can cause a condition like writer’s cramp. In a recently dead person, it results in
rigor mortis.
36 What is the difference between voluntary and involuntary muscles? Give an example
of each.
A Voluntary muscles cannot be consciously controlled, as with cardiac muscles.
Involuntary muscles can becontrolled by an individual’s will, as with skeletal muscles.
B Voluntary muscles can be controlled by an individual’s will, as with cardiac muscles.
Involuntary muscles cannot be consciously controlled, as with skeletal muscles.
C Voluntary muscles cannot be consciously controlled, as with the movement of legs
while walking. Involuntary muscles can be controlled by an individual’s will, as with
muscles in the digestive system.
D Voluntary muscles can be controlled by an individual’s will, as with skeletal muscles.
Involuntary muscles cannot be consciously controlled, as with cardiac muscles.
Solution The solution is (D). Voluntary muscles are the ones that you can control by thought,
such as when you move your legs to walk. Involuntary muscles cannot be
consciously controlled, such as the muscles in your digestive system.

SCIENCE PRACTICE CHALLENGE QUESTIONS


29.1 Types of Skeletal Systems
37 Biomimetics is the design of engineering solutions guided by biological solutions to similar
problems. Support materials in biological systems must be tough, flexible, and light.
Engineers define a tough material as one that can absorb energy without fracturing.

Advanced Placement Biology Instructor’s Solution Manual


29 | The Musculoskeletal System 747

Flexibility is the ability to both deform elastically and return to the original shape. Size in
both biological and designed systems is often determined by many factors. Density of a
material becomes important when size cannot be adjusted but lightness is needed. An
analysis of several biological materials in terms of these parameters (Wegst and Ashby,
Philosophical Magazine, 84, 2004) shows that selection has conserved a bounding value in
the product of these parameters; the square root of all of the ratios of toughness and
elasticity lie above the dashed line. Patterns like this lead to new science but, in
applications to biomimetics, they can also lead to improved technologies.

Biopolymers like collagen have higher toughness and higher elasticity. Biominerals like
enamel have lower toughness but provide rigidity. Some organisms support soft tissue by
combining biopolymers and biomaterials. Some rely more on biopolymers. Note that this
is a logarithm scale, not a linear scale, such that the difference in toughness of keratin and
bone is roughly 100, and the difference in rigidity of these to materials is nearly that large.
A. Use this graph to pose questions about the fitness of various biological solutions to the
problem of capturing, storing, and using free energy. (Remember that engineers define
“toughness” as the ability to absorb energy without fracturing.)
B. Refine this representation by identifying an important property of a biomaterial that is
missing and explain its importance in terms of free energy acquisition and use.
C. Explain how these data indicate that the properties of biopolymers will lead to a better
biomimetic design of the humanoid robot than is imagined in C3PO of Star Wars fame.
Solution Sample answer:

Advanced Placement Biology Instructor’s Solution Manual


748 29 | The Musculoskeletal System

A. We notice that combinations of biopolymers and biominerals occur. In


invertebrates, keratin and cuticle are combined. In vertebrates, bone and collagen
are combined. Crustacea are immersed in water, while insects often fly. Some
vertebrates fly, while others do not. Scientific questions that could lead to a search
for patterns might include the following:
 Can a phylogeny of connective tissue be constructed?
 What exoskeleton characteristics are typical of marine and terrestrial
invertebrates and how do these differ?
 What endoskeleton characteristics are typical of groups of terrestrial
vertebrates and how are they different?
 What are the developmental and molecular features of the attachment of
connective tissue to the skeleton?
 What are the physical laws that constrain biological systems to lie above the
diagonal line in this diagram?
 Is there a pattern in the distances of materials along a line perpendicular to
line of constrain?
 Are there patterns in the hydration of these materials?
B. The three characteristics mentioned in the narrative were flexibility, toughness,
and lightness. This diagram needs another dimension on which the density of
materials is compared.
C. C3PO, as well as the Terminator, has serious design flaws in their analogs of
connective tissue. But the goal here is either human or fright. Humanoid robots of
the future will be able to dance.
29.4 Muscle Contraction and Locomotion
38 An investigation of the evolution of muscle was made by Steinmetz et al. (Nature, 487,
2012), and a sample of data is presented in the diagram. Below the accepted phylogeny of
major groups are rows with families of genes that code for different muscle proteins,
including actin and myosin. Shared genes are shown where the cell is darkened. Samples
of genes associated with striated and smooth muscle and the Z-disc that terminates the
actin-myosin pair are clustered. These are shown in the diagram, where groups in which
muscle cells occur lie within the box whose edges are dashed lines.

Advanced Placement Biology Instructor’s Solution Manual


29 | The Musculoskeletal System 749

A. Analyze these data in terms of evidence of common ancestry and evidence of the
convergent evolution of the muscle cell.
Molecular phylogenetics has provided many insights into the evolution of genes. Because
this work is framed by the central organizing principle of evolution, it is possible to forget
that there are people living today who need evidence that speciation and extinction have
occurred throughout Earth’s history.
B. Describe in broad outline a plan for using molecular databases for either protein or
DNA sequences that you could help those who are in need of evidence construct it.
Solution Sample answer:
A. Muscle cell shape is regulated by the relative motion of actin fibers and myosin
polymers. But cell shape is regulated in eukaryotic cells more generally by
remodeling of actin fibers that form a cellular cytoskeleton. The presence of actin in
all eukaryotic cells is evidence of common ancestry. The actin-myosin interactions in
the anemones, hydras, and jellyfish do not share many of the gene families found in
the Bilaterians. Often, a gene deletion can occur as seen in the empty cells of each of
the Bilaterians. However, the Cnidarians and Ctenophore groups display a pattern
that cannot reasonably be attributed to gene deletion. This implies that the actin-
myosin interactions in these three groups are not due to shared ancestry. Therefore,
these interactions are innovations that appeared independently in these lineages.

Advanced Placement Biology Instructor’s Solution Manual


750 29 | The Musculoskeletal System

B. Everyone has access to BLAST. With it, a comparison can be made of sequences in
either of these biopolymers. A reasonable question that forms the basis for an
inquiry is whether it is possible for this enormous degree of similarity to occur in
species that have no ancestral relationship. And where two species in which
significant differences appear in certain genes but not in others, it is possible that
there is no shared ancestor who is not still in existence.

Advanced Placement Biology Instructor’s Solution Manual


30 | The Respiratory System 751

30 | THE RESPIRATORY SYSTEM


REVIEW QUESTIONS
1 What is a primary function of the respiratory system?
A Provides body tissues with oxygen
B Provides body tissues with oxygen and carbon dioxide
C Establishes how many breaths are taken per minute
D Provides the body with carbon dioxide
Solution The solution is (A). The respiratory system functions to bring air into and out of the
body. This allows oxygen to enter the blood and carbon dioxide to be removed from
the blood and from the body through exhalation.
2 What is the order of airflow during inhalation?
A Nasal cavity, larynx, trachea, bronchi, bronchioles, alveoli
B Nasal cavity, larynx, trachea, bronchioles, bronchi, alveoli
C Nasal cavity, trachea, larynx, bronchi, bronchioles, alveoli
D Nasal cavity, trachea, larynx, bronchioles, bronchi, alveoli
Solution The solution is (A). During inhalation, air travels from the oral or nasal cavities into
the pharynx, then into the larynx, trachea, bronchi, bronchioles, and alveoli.
3 What advantages does warming air within the nasal passages provide?
A Reduces rates of lung infection
B Prevents foreign objects from entering the lungs
C Prevents cell damage caused by cold air
D Reduces fluid accumulation from cold, wet air
Solution The solution is (C). Cold air can reduce the efficiency of, and even kill, respiratory
system cells. Therefore, warming the air is vital to maintaining the efficiency of
respiratory system cells.
4 Emphysema is a disease characterized by a great reduction in the number of functioning
alveoli in the lungs. What effect would emphysema likely have on the respiratory system?
A Less gas exchange would occur with the blood.
B Less air could travel through the trachea.
C Air could no longer pass through the nasal cavity.
D Air could no longer vibrate the vocal cords of the larynx.

Advanced Placement Biology Instructor’s Solution Manual


752 30 | The Respiratory System

Solution The solution is (A). If fewer functioning alveoli were present in the lungs, this would
decrease the surface area available for gas exchange with the blood, as the primary
function of the alveoli is to allow blood close enough to air to permit oxygen and
carbon dioxide to diffuse.
5 The inspiratory reserve volume measures the amount of air —
A remaining in the lung after a maximal exhalation
B that the lung holds
C that can be further exhaled after a normal breath
D that can be further inhaled after a normal breath
Solution The solution is (D). The inspiratory reserve volume measures the amount of air that
can be further inhaled after a normal breath.
6 The total lung capacity is calculated using which formula?
A Residual volume + tidal volume + inspiratory reserve volume
B Residual volume + expiratory reserve volume + inspiratory reserve volume
C Expiratory reserve volume + tidal volume + inspiratory reserve volume
D Residual volume + expiratory reserve volume + tidal volume + inspiratory reserve
volume
Solution The solution is (D). total lung capacity = residual volume + expiratory reserve volume
+ tidal volume + inspiratory reserve volume
7 Why is the partial pressure of oxygen lower in the lungs than in the external air?
A Oxygen is constantly being removed by air in the lungs through the blood.
B Carbon dioxide does not mix with oxygen in the lungs.
C The lungs exert pressure on the air to reduce the oxygen pressure.
D Changes in water vapor pressure cause less oxygen to enter the air in the lungs.
Solution The solution is (A). Air in the lungs is humidified; therefore, water vapor pressure
alters the pressure. Also, carbon dioxide mixes with oxygen in the lungs and oxygen
is constantly being removed by air in the lungs through the blood.
8 How can a decrease in the percent of oxygen in the air affect the movement of oxygen in
the body?
A It would not affect the rate of diffusion into the blood because there would be no
difference between the partial pressures of the air and blood.
B It would decrease the rate of diffusion into the blood because it increases the
difference between the partial pressures of the air and blood.

Advanced Placement Biology Instructor’s Solution Manual


30 | The Respiratory System 753

C It would increase the rate of diffusion into the blood because it reduces the difference
between the partial pressures of the air and blood.
D It would decrease the rate of diffusion into the blood because it reduces the difference
between the partial pressures of the air and blood.
Solution The solution is (D). Oxygen moves from the lung to the bloodstream to the tissues
according to the pressure gradient. This is measured as the partial pressure of
oxygen. If the amount of oxygen drops in the inspired air, there would be reduced
partial pressure. This would decrease the driving force moving the oxygen into the
blood. Partial pressure of oxygen is also reduced at high elevations: Partial pressure
of oxygen at high elevations is lower than at sea level because the total atmospheric
pressure is less than atmospheric pressure at sea level.
9 What role do the intercostal muscles play in inspiration and expiration?
A They move down, causing inspiration, and move up, causing expiration.
B They move up, causing inspiration, and move down, causing expiration.
C They relax, causing inspiration, and contract, causing expiration.
D They contract, causing inspiration, and relax, causing expiration.
Solution The solution is (D). During inspiration, the intercostal muscles contract, increasing
the volume of the thoracic cavity and allowing inspiration to occur. During
expiration, the intercostal muscles relax, decreasing the volume of the thoracic
cavity and allowing expiration to occur.
10 How would paralysis of the diaphragm alter inspiration?
A It would prevent contraction of the intercostal muscles.
B It would prevent effective inhalation because the intrapleural pressure would not
change as much.
C It would decrease the intrapleural pressure and allow more air to enter the lungs.
D It would slow expiration because the lungs would not relax.
Solution The solution is (B). Paralysis of the diaphragm would greatly reduce how much the
thoracic cavity could change in volume. If the diaphragm could not move down, the
thoracic cavity could not expand as much and the intrapleural pressure could not
drop as much, reducing inspiration.
11 Restrictive airway diseases —
A increase the compliance of the lung
B decrease the compliance of the lung
C increase the lung volume
D decrease the work of breathing

Advanced Placement Biology Instructor’s Solution Manual


754 30 | The Respiratory System

Solution The solution is (B). Restrictive airway diseases reduce the elasticity of the lung, also
known as the compliance of the lung.
12 A patient is suffering from an allergic reaction that causes his airways to swell up. How
would this most likely affect the resistance of the respiratory system, and why?
A Resistance decreases because it will be harder for air to flow through the airways.
B Resistance decreases because the airways are more compliant.
C Resistance increases because it will be harder for air to flow through the airways.
D Resistance increases because the airways are more compliant.
Solution The solution is (C). Swelling of the airways will obstruct the airways by making them
smaller. This increases the resistance of the respiratory system.
13 What is a physiological dead space and what could be its cause?
A A physiological dead space is a region of lung tissue that is leaking air. It could be
caused by a puncture wound.
B A physiological dead space is a region of lung tissue that is not receiving electrical
stimulation from the nervous system. It could be caused by a reduction in
neurotransmitters.
C A physiological dead space is a region of lung tissue that is not involved in gas
exchange because the tissue is not functioning properly. It could be caused by the
alveoli capillaries are not receiving blood flow.
D A physiological dead space is a region of lung tissue that is not involved in gas
exchange because the tissue has a defect in its structure. It could be caused by a
genetic condition.
Solution The solution is (C). Physiological dead space is a region of lung tissue that is not
involved in gas exchange because the tissue is not functioning properly, such as if
alveoli capillaries are not receiving blood flow.
14 How will the respiratory rate and volume of air per breath change if alveolar ventilation is
too high or too low?
A If ventilation is low, the respiratory rate will decrease, while the volume of air per
breath is decreased. If ventilation is high, the respiratory rate will increase while the
volume of air per breath decreases.
B If ventilation is low, the respiratory rate will decrease, while the volume of air per
breath is increased. If ventilation is high, the respiratory rate will increase while the
volume of air per breath decreases.

Advanced Placement Biology Instructor’s Solution Manual


30 | The Respiratory System 755

C If ventilation is low, the respiratory rate will increase, while the volume of air per
breath is decreased. If ventilation is high, the respiratory rate will increase while the
volume of air per breath increases.
D If ventilation is low, the respiratory rate will increase, while the volume of air per
breath is decreased. If ventilation is high, the respiratory rate will increase while the
volume of air per breath decreases.
Solution The solution is (D). If ventilation is low, the respiratory rate will increase, while the
volume of air per breath is decreased (hyperventilation). If ventilation is high, the
respiratory rate is decreased while increasing the volume per breath.
15 What does elastic recoil in the lungs refer to and does it aid inspiration or expiration?
A The ability of the diaphragm to contract and relax to change the volume of the lung;
inspiration
B The reserve air in the lungs that is present even after maximum exhalation; expiration
C The ability of the bronchioles to expand and retract as air enters and exits; inspiration
D The ability of lung tissue to exert pressure back towards the lung’s interior; expiration
Solution The solution is (D). Elastic recoil refers to how lung tissue exerts pressure back
toward the interior of lung. This pressure aids exhalation because it causes the lung
to shrink in volume.
16 Low pH in the blood will _____ hemoglobin’s affinity for _____.
blank blank

A increase; carbon dioxide


B increase; oxygen
C decrease; carbon dioxide
D decrease; oxygen
Solution The solution is (D). Low pH in the blood will decrease hemoglobin’s affinity
for oxygen.
17 What will NOT facilitate the transfer of oxygen to tissues?
A Decreased body temperature
B Decreased pH of the blood
C Increased carbon dioxide
D Increased exercise
Solution The solution is (D). Increased exercise increases the temperature of the body,
reducing hemoglobin’s affinity for oxygen.

Advanced Placement Biology Instructor’s Solution Manual


756 30 | The Respiratory System

18 How is the majority of carbon dioxide in the blood transported?


A By binding to hemoglobin
B Via dissolution in the blood
C As bicarbonate
D By binding to plasma proteins
Solution The solution is (C). The majority of carbon dioxide in the blood is transported as
bicarbonate following its conversion from carbon dioxide and is carried by the blood
plasma.
19 How is the chloride shift related to the transport of carbon dioxide in the blood?
A It allows carbon dioxide, in the form of bicarbonate ions, to enter the blood plasma.
B It creates carbaminohemoglobin within the red blood cells.
C It allows the conversion of carbon dioxide into carbonic acid within red blood cells.
D It prevents the formation of bicarbonate ions in the blood.
Solution The solution is (D). The chloride shift refers to the process by which a bicarbonate
ion is transported out of the red blood cell into the liquid component of the blood in
exchange for a chloride ion. When the blood reaches the lungs, the bicarbonate ion
is transported back into the red blood cell in exchange for the chloride ion.

CRITICAL THINKING QUESTIONS


20 How does the structure of alveoli maximize gas exchange?
A Their sac-like structure increases their surface area.
B Their direct connection to the bronchi maximizes their access to air.
C They actively transport the gases between the air and blood.
D They are spheres that fully fill with blood, which will come in contact with air.
Solution The solution is (A). The sac-like structure of the alveoli increases their surface area.
In addition, the alveoli are made of thin-walled parenchymal cells. These features
allow gases to easily diffuse across the specialized cell membranes.
21 What structures of the respiratory system warm and remove impurities from inhaled air?
A The epiglottis deflects impurities out of the trachea and into the esophagus, while the
nasal cavity warms the air.
B Hair and mucus in the nose and trachea catch impurities, while the nasal cavity warms
the air.
C Saliva in the mouth and hair in the trachea catch impurities, while the pharynx warms
the air.
D The closed-off compartments of the larynx trap impurities in the air and warm the air.

Advanced Placement Biology Instructor’s Solution Manual


30 | The Respiratory System 757

Solution The solution is (B). The nose contains hairs and mucus-releasing cells, which remove
impurities while the nasal passage warms the air. The trachea contains mucus-
producing cells as well as ciliated epithelium, which remove impurities from the
trachea. The lungs, themselves, also produce mucus, further trapping impurities.
22 If you were travelling in a miniaturized ship through the respiratory system, from the
pharynx to an alveolus, which structures would you pass along the way, and in what
order?
A trachea, larynx, bronchi, and bronchioles
B larynx, trachea, bronchi, and bronchioles
C bronchioles, bronchi, trachea, and larynx
D bronchioles, trachea, bronchi, and larynx
Solution The solution is (B). If travelling from the pharynx to an alveolus, one would pass
through the larynx, trachea, bronchi, and bronchioles before reaching an alveolus.
23 What does FEV1/FVC measure and why will the ratio increase with lung fibrosis?
A The forced expiratory volume in 1 s in relation to the total forced vital capacity; lung
fibrosis causes the lungs to decrease in size.
B The functional expiratory volume in 1 s in relation to the total functional vital capacity;
lung fibrosis causes the lungs to decrease in size.
C The functional expiratory volume in 1 s in relation to the total functional vital capacity;
lung fibrosis causes the lungs to become less pliable.
D The forced expiratory volume in 1 s in relation to the total forced vital capacity; lung
fibrosis causes the lungs to become less pliable.
Solution The solution is (D). FEV1/FVC measures the forced expiratory volume in 1 s in
relation to the total forced vital capacity (the total amount of air exhaled from the
lung during a maximal inhalation). This ratio increases as the lungs become less
pliable due to fibrosis, asthma, and COPD.
24 Compare the partial pressure of carbon dioxide between venous blood in an alveolus and
air, and between arterial blood and body tissues.
A Higher in the blood than in the air and higher in the blood than in the body tissues
B Lower in the blood than in the air and higher in the blood than in the body tissues
C Higher in the blood than in the air and lower in the blood than in the body tissues
D Lower in the blood than in the air and lower in the blood than in the body tissues
Solution The solution is (C). In an alveolus, the partial pressure of carbon dioxide is higher in
the blood than in the air because the blood contains more carbon dioxide than the
air. In the body tissues, the partial pressure of carbon dioxide is higher in the body

Advanced Placement Biology Instructor’s Solution Manual


758 30 | The Respiratory System

tissues than in the blood because the body tissues have more carbon dioxide than
the blood.
25 What conditions of the lungs would cause an increase in FEV1/FVC? What about a
decrease in FEV1/FVC?
A This ratio increases as the lungs become stiff and less pliable, increasing further when
there is increased resistance in the lungs.
B This ratio decreases as the lungs become stiff and less pliable, increasing when there is
increased resistance in the lungs.

Advanced Placement Biology Instructor’s Solution Manual


30 | The Respiratory System 759

C This ratio increases as the lungs become stiff and less pliable, decreasing when there is
increased resistance in the lungs.
D This ratio decreases as the lungs become stiff and less pliable, decreasing further when
there is increased resistance in the lungs.
Solution The solution is (C). FEV1/FVC measures the forced expiratory volume in one second
relative to the total forced vital capacity (the total amount of air that is exhaled from
the lungs from a maximal inhalation). This ratio increases as the lungs become stiff
and less pliable due to fibrosis, asthma, and COPD. Conversely, when the FEV1/FVC
ratio is low, there is resistance in the lungs, which is characteristic of asthma. In this
instance, it is hard for the patient to get the air out of his or her lungs, and it takes a
long time to reach the maximal exhalation volume.
26 Amphibians, such as frogs, breathe by collecting air in a pouch below their throat. Muscles
then contract the pouch and force air into their lungs. How does this differ from
inhalation in humans and other mammals?
A Inhalation in humans and other mammals involves the openings called spiracles, which
connect to the tubular network to allow the oxygen to pass into the body.
B Inhalation in humans and other mammals involves direct diffusion across the outer
membrane to meet oxygen requirements. Gases can diffuse quickly through direct
diffusion.
C Inhalation in humans and other mammals involves contracting the thoracic cavity by
creating negative pressure in the lungs, which causes air to diffuse into the lungs.
D Inhalation in humans and other mammals involves expanding the thoracic cavity by
creating negative pressure in the lungs, which causes air to diffuse into the lungs.
Solution The solution is (D). Inhalation in humans and other mammals does not involve
forcing air into the lungs. Instead, negative pressure is created in the lungs by
expanding the thoracic cavity. This causes air to diffuse into the lungs during
inhalation.
27 If a patient has increased resistance in his or her lungs, how can this be detected by a
doctor? What does this finding mean?
A This can be detected using a nebulizer. By detecting the rate at which air can be taken
into the lung, a diagnosis of a restrictive disease can be made.
B This can be detected using spirometry. By detecting the rate at which air can be taken
into the lung, a diagnosis of a restrictive disease can be made.
C This can be detected using a nebulizer. By detecting the rate at which air can be
expelled from the lung, a diagnosis of a restrictive disease can be made.
D This can be detected using spirometry. By detecting the rate at which air can be
expelled from the lung, a diagnosis of a restrictive disease can be made.

Advanced Placement Biology Instructor’s Solution Manual


760 30 | The Respiratory System

Solution The solution is (D). A doctor can detect a restrictive disease using spirometry. By
detecting the rate at which air can be expelled from the lung, a diagnosis of fibrosis
or another restrictive disease can be made.
28 When someone is standing, gravity stretches the bottom of the lung down toward the
floor to a greater extent than the top of the lung. What implication could this have on
ventilation in the lungs?
A Concentration gradient leads to increased ventilation farther down in the lung.
B Pleural pressure gradient leads to increased ventilation farther down in the lung.
C Pleural pressure gradient leads to decreased ventilation farther down in the lung.
D Concentration gradient leads to decreased ventilation farther down in the lung.
Solution The solution is (B). The lung is particularly susceptible to changes in the magnitude
and direction of gravitational forces. When someone is standing or sitting upright,
the pleural pressure gradient leads to increased ventilation further down in the lung.
29 How does the administration of 100 percent oxygen save a patient from carbon monoxide
poisoning? Why wouldn’t giving carbon dioxide work?
A At that concentration, oxygen will be transported in the body at a high rate by
dissolving in blood. Oxygen has more affinity for hemoglobin than carbon dioxide.
B At that concentration, oxygen will displace the carbon monoxide from the
hemoglobin. Oxygen has more affinity for hemoglobin than carbon dioxide.
C At that concentration, oxygen will displace the carbon monoxide from the
hemoglobin. Carbon dioxide has more affinity for hemoglobin than oxygen.
D At that concentration, oxygen will be transported in the body at a high rate by
dissolving in blood. Carbon dioxide has more affinity for hemoglobin than oxygen.
Solution The solution is (B). Carbon monoxide has a higher affinity for hemoglobin than
oxygen. Therefore, carbon monoxide will bind to hemoglobin more readily than
oxygen in inhaled air. Administration of 100 percent oxygen is an effective therapy
because at that concentration, oxygen will displace the carbon monoxide from the
hemoglobin.
30 What would happen if no carbonic anhydrase was present in red blood cells?
A Carbon dioxide would be hydrolyzed into carbonic acid or bicarbonate. The maximum
amount of carbon dioxide would be transported in the blood away from the tissues.
B Carbon dioxide would not be hydrolyzed into carbonic acid or bicarbonate. The
maximum amount of carbon dioxide would be transported in the blood away from the
tissues.

Advanced Placement Biology Instructor’s Solution Manual


30 | The Respiratory System 761

C Oxygen would not be hydrolyzed into carbonic acid or bicarbonate. Only 15 percent of
carbon dioxide would be transported in the blood away from the tissues.
D Carbon dioxide would not be hydrolyzed into carbonic acid or bicarbonate. Only a
relatively small percentage of carbon dioxide would be transported in the blood away
from the tissues.
Solution The solution is (B). Without carbonic anhydrase, carbon dioxide would not be
hydrolyzed into carbonic acid or bicarbonate. Because most carbon dioxide is
transported in the blood in these forms, less carbon dioxide than usual would be
transported in the blood away from the tissues.
31 What is sickle cell anemia and how does it affect the perfusion of oxygen in the blood?
A It is a genetic disease in which red blood cells are sickle-shaped, reducing oxygen
perfusion into the blood.
B It is a genetic disease in which red blood cells are sickle-shaped, increasing oxygen
perfusion into the blood.
C It is an enzyme deficiency disease in which red blood cells are sickle-shaped, reducing
oxygen perfusion into the blood.
D It is an enzyme deficiency disease in which red blood cells are sickle-shaped,
increasing oxygen perfusion into the blood.
Solution The solution is (A). Sickle cell anemia is a genetic disease in which red blood cells are
misshapen, reducing oxygen perfusion into the blood.

TEST PREP FOR AP® COURSES


32 The cell of the unicellular algae Ventricaria ventricosa, shown in the photo, is one
of the largest known, reaching 1 to 5 cm in diameter. Like all single-celled organisms,
V. ventricosa exchanges gases across the cell membrane.

What adaptations would V. ventricosa likely have evolved related to its large size and
ability to exchange materials with the outside environment?
A Adaptations that would decrease cell metabolism to meet the needs of the large cell
B Adaptations that would make the cell thicker, to reduce the loss of nutrients

Advanced Placement Biology Instructor’s Solution Manual


762 30 | The Respiratory System

C Adaptations that make diffusion or nutrient passage across their cell membrane more
efficient due to the large size of the cell
D Adaptations that allow the cell to take in larger food objects using the components of
its cell membrane
Solution The solution is (C). The large size of V. ventricosa would cause it to be difficult for the
cell to perform an adequate amount of nutrient exchange with the outside
environment. Therefore, V. ventricosa have likely evolved adaptations that make
diffusion or nutrient passage across their cell membrane more efficient.
33 In the past, Earth has experienced environmental changes that have changed the amount
of available oxygen and carbon dioxide in the water and air. For example, there is
evidence of less oxygen being available in the air during the time of the dinosaurs, a result
of high volcanic activity creating a large amount of carbon dioxide.
How might red blood cells in the dinosaurs have evolved, in terms of size and shape, to
adapt to the lower-oxygen atmosphere?
A Smaller size and flatter shape
B Larger size and a pointy shape
C Smaller size and a thicker shape
D Larger size and a shorter shape
Solution The solution is (A). Red blood cells may have evolved smaller size and/or a flatter
shape in order to maximize the amount of oxygen they can absorb as they pass
through the lungs.
34 The diagram shows a human alveolus, which is part of the respiratory system.

What do arrows A and B represent in the diagram?


A A: inhaled air; B: blood traveling from the heart
B A: exhaled air; B: blood traveling from the heart
C A: inhaled air; B: blood traveling to the heart
D A: exhaled air; B: blood traveling to the heart

Advanced Placement Biology Instructor’s Solution Manual


30 | The Respiratory System 763

Solution The solution is (B). Arrow A represents air traveling away from an alveolus through a
bronchiole. Therefore, this is exhaled air. Arrow B represents blood moving toward
the air sacs of the alveolus, where it will be oxygenated. This blood arrived at the
alveolus via the pulmonary vein, which takes blood away from the heart.
35 Ambulance crews use intubation, which is a procedure that allows a person to breathe if
part of the respiratory system is blocked by a foreign object (or otherwise injured). During
intubation, a long, plastic tube is placed in the respiratory system so that air can bypass
the obstructed area and reach the lungs. Typically, air is supplied artificially using a
squeezable bag that connects to the top of the tube. The illustration shows the human
respiratory system. The nasal cavity is a wide cavity above and behind the nostrils, and
the pharynx is the passageway behind the mouth. The nasal cavity and pharynx join and
enter the trachea through the larynx. The larynx is somewhat wider than the trachea and
flat. The trachea has concentric, ring-like grooves, giving it a bumpy appearance. The
trachea bifurcates into two primary bronchi, which are also grooved. The primary bronchi
enter the lungs, and branch into secondary bronchi. The secondary bronchi in turn branch
into many tertiary bronchi. The tertiary bronchi branch into bronchioles, which branch
into terminal bronchioles. The diaphragm pushes up against the lungs. There is an
intubation site indicated at the beginning of the pharynx. A patient has been surgically
intubated in the location shown in the diagram

Based on this information, where did the injury likely occur in the patient’s respiratory
system? Justify your answer.
A In the oral cavity, because it is above the injury
B In the oral cavity, because it is below the injury
C In the larynx, because it is above the injury
D In the larynx, because it is below the injury
Solution The solution is (D). The patient should be intubated through the larynx because the
larynx is located below the pharynx, where there obstruction is. Intubation has to
occur below the obstruction so that the tube bypasses the obstruction.

Advanced Placement Biology Instructor’s Solution Manual


764 30 | The Respiratory System

36 An organism’s body systems work to maintain homeostasis by adjusting when body cells
need more oxygen or are experiencing a buildup of carbon dioxide. How would the body
most likely react to the difference between the blood vessel and body cell, as shown in
the diagram?

A Generating neural signals that stimulate the heart to beat at a slower rate
B Releasing hormones that stimulate body cells to undergo more active transport
C Releasing red blood cells that can accept oxygen using diffusion as opposed to
facilitated passive transport
D Adjusting blood pH to decrease the partial pressure of carbon dioxide in the body cells
Solution The solution is (A). Oxygen will no longer diffuse from the blood vessel into the cell
seeing how the partial pressure of oxygen difference between them is equal.
Therefore, the body could slow down the heart rate at this point, because the cell
likely has sufficient oxygen.
37 The diagram shows a red blood cell in a capillary and a cell in a body tissue.

Advanced Placement Biology Instructor’s Solution Manual


30 | The Respiratory System 765

In which direction should the arrows point for the diffusion of oxygen and carbon dioxide?
How should each partial pressure (body cell and red blood cell or RBC) be labeled as high
or low to accomplish this diffusion?
A O2 → CO2 ←; Body cell PO2 = low; RBC PO2 = high; Body cell PCO2 = high,
RBC PCO2 = low
B O2 ← CO2 →; Body cell PO2 = high; RBC PO2 = low; Body cell PCO2 = low,
RBC PCO2 = high
C O2 ← CO2 →; Body cell PO2 = low; RBC PO2 = high; Body cell PCO2 = high,
RBC PCO2 = low
D O2 → CO2 ←; Body cell PO2 = high; RBC PO2 = low; Body cell PCO2 = low,
RBC PCO2 = high
Solution The solution is (C). Within body tissues, oxygen will diffuse from an RBC to the body
cells, while carbon dioxide will diffuse from the body cell to an RBC. To accomplish
this, the body cell partial pressure of oxygen must be lower than the RBC partial
pressure of oxygen, while the body cell partial pressure of oxygen must be higher
than the RBC partial pressure of oxygen.
38 The graph plots percent oxygen saturation of hemoglobin as a function of oxygen
partial pressure in the alveoli. Oxygen saturation increases in an S-shaped curve, from
0–100 percent as the partial pressure of oxygen increases from 0 to 100.

What happens as the curve levels off around a partial pressure of 60 mm Hg?
A As the percent saturation of hemoglobin increases to its maximum, hemoglobin’s
affinity for oxygen increases as the availability of oxygen increases.
B As the percent saturation of hemoglobin decreases (without all of the oxygen
dissociating), hemoglobin’s affinity for oxygen decreases as the availability of oxygen
decreases.

Advanced Placement Biology Instructor’s Solution Manual


766 30 | The Respiratory System

C As the percent saturation of hemoglobin increases to very high levels, hemoglobin’s


affinity for oxygen decreases due to its decreasing ability to bind oxygen.
D As the percent saturation of hemoglobin decreases, hemoglobin’s affinity for oxygen
increases as the availability of oxygen decreases.
Solution The solution is (C). As the percent saturation of hemoglobin increases, hemoglobin’s
affinity for oxygen decreases, even if the availability of oxygen increases, as
indicated by higher partial pressures of oxygen.
39 The graph shows an oxygen dissociation curve for hemoglobin.

Based on the graph, what would likely cause the curve to shift to the left, as shown by the
dotted plot line?
A Decreasing carbon dioxide, increasing pH, or decreasing temperature
B Increasing carbon dioxide, decreasing pH, or decreasing temperature
C Decreasing carbon dioxide, decreasing pH, or decreasing temperature
D Increasing carbon dioxide, increasing pH, or increasing temperature
Solution The solution is (A). A decrease in carbon dioxide, increase in pH, or a decrease in
temperature will lower the efficiency of which oxygen is carried by the blood cells.
40 The graph shows an oxygen dissociation curve for hemoglobin.

Advanced Placement Biology Instructor’s Solution Manual


30 | The Respiratory System 767

Based on the graph, what would likely cause the curve to shift to the right, as shown by
the dotted plot line?
A Decreasing carbon dioxide, increasing pH, or decreasing temperature
B Decreasing carbon dioxide, decreasing pH, or decreasing temperature
C Increasing carbon dioxide, increasing pH, or increasing temperature
D Increasing carbon dioxide, decreasing pH, or increasing temperature
Solution The solution is (D). An increase in carbon dioxide, decrease in pH, or an increase
in temperature will all increase the amount of oxygen that is entering the red
blood cells.

SCIENCE PRACTICE CHALLENGE QUESTIONS


30.1 Systems of GAS exchange
41 Oxygen deprivation produces symptoms such as fatigue, headaches, and confusion that
are collectively referred to as hypoxia. A vacation in the Rocky Mountains in the United
States, where the partial pressure of oxygen is just 15 percent lower than you are
accustomed to on the coast, can induce these symptoms. The body has a very narrow
range of environment oxygen tolerance. It essentially has no capacity for storage of
oxygen that is continuously consumed to maintain energy homeostasis.
The response to oxygen deprivation is to increase breathing rate and increase the volume
of each breath. The input for a negative-feedback loop that maintains homeostasis by
detecting oxygen concentrations is a sensor (the carotid body) located on the interior of
the carotid artery. The output is at the diaphragm. The signal is processed in the
respiratory centers (RCs) of the medulla in the brainstem.
A. Describe how the nervous system integrates information about oxygen concentration
in the blood to maintain homeostasis. In your description, include the concepts of
negative feedback and a set point. An alternative model (Evens et al., Biochemical
Journal, 473, 2016) of the response to oxygen deprivation is suggested by the observation
that high-altitude Andean populations have a gene for a protein kinase (AMPK encoded by
PRKAA1) that has a fixed single nucleotide variation. This AMP-activated kinase is coupled
with mitochondrial oxidative phosphorylation to detect reduced oxygen and signal the
RCs of the medulla directly—a distributed network for detection and response. The
authors of this work note that a homologous gene in yeast allows colony-wide signaling to
switch individuals in the colony from glycolysis to oxidative phosphorylation in response
to changes in glucose resources.
B. Describe the connection of a changing environment to changing genomes in both
species in terms of the adaptive advantage provided and the likelihood that an AMPK
signaling process has been conserved across domains. Studies of a genetic adaptation in
Tibetan population have shown that other mutations have been selected. One mutation
EPAS1 was shown to be correlated with increased lactic acid concentrations in the blood.
Another mutation, PPARA, was found to be correlated with fatty acid production, which is

Advanced Placement Biology Instructor’s Solution Manual


768 30 | The Respiratory System

typically seen during hibernation (Ge et al., Molecular Genetics and Metabolism, 106,
2012; Lorenzo et al., Nature Genetics, 46, 2014).
C. Analyze these observations in terms of the regulation of metabolism due to changes in
genetic makeup and construct an explanation for the divergence of the homeostatic
mechanisms as an adaptation to the environment. The Tibetan population is not isolated.
However, these investigations show the near dominance of the mutated form of these
two genes has arisen in just 8,000 years. Neanderthal, Denisovans, and ancestors of
modern humans were contemporaries.
D. Describe conditions that lead to speciation in terms of the accumulation of many small
genetic changes where very sharp differences in oxygen availability were geographically
imposed.
Solution Sample answer:
A. This sympathetic response is initiated in the carotid body, a mat of chemical
receptors on the interior of the carotid artery. These sensors detect oxygen
concentration in the blood through activation of a potassium channel in neurons of
the carotid bodies and transmit a signal to the respiratory centers, cells in the
medulla that compare the signal to a set point and transmit a signal to the
diaphragm controlling the rate and volume of breathing.
B. The suggestion is that an AMPK-based response is conserved and that positive
selection by low oxygen levels has led to a modification of the gene.
C. The Tibetan population shows a shift toward metabolic pathways that demand
less oxygen. The lactic acid concentration indicates increased reliance on glycolysis
and the fatty acid concentration indicates an increase in fatty acid metabolism—the
combustion of fat, just as is demonstrated by hibernating animals.
D. Variation in one critical gene family could produce geographic isolation in a
situation in which a sharp, rather than a gradual, variation in a particular
environmental factor was imposed. Through a long period of isolation, the molecular
clock of random mutation and selection can lead to speciation.

Advanced Placement Biology Instructor’s Solution Manual


31 | The Circulatory System 769

31 | THE CIRCULATORY SYSTEM


REVIEW QUESTIONS
1 Which organism has an open circulatory system?
A A cat
B A bee
C A human
D An earthworm
Solution The solution is (B). Bees are arthropods, and arthropods have an open circulatory
system.
2 What is an advantage of an open circulatory system?
A It uses less metabolic energy.
B It enables an organism to move faster.
C It is a more efficient way to move gases, nutrients, and waste around an
organism’s body.
D It allows organisms to grow larger.
Solution The solution is (A). The advantage of an open circulatory system is that it uses less
metabolic energy than a closed circulatory system, which makes it a good fit for less
complex organisms.
3 Which statement about circulatory systems is false?
A In closed circulatory systems, blood flows through vessels that are separate from the
interstitial fluid of the body.
B The earthworm has a closed circulatory system.
C In an open circulatory system, hemolymph empties into the body cavity.
D Lobsters are organisms that have a closed circulatory system.
Solution The solution is (D). It is not true that lobsters are organisms that have a closed
circulatory system. Lobsters are crustaceans; like other arthropods, they have an
open circulatory system.
4 Which statement best defines the open circulatory system?
A In an open circulatory system, blood mixes with interstitial fluid in the hemocoel.
B In an open circulatory system, the blood is separated from the bodily interstitial fluid
and contained blood vessels.

Advanced Placement Biology Instructor’s Solution Manual


770 31 | The Circulatory System

C Blood circulates unidirectionally from the heart around the systemic circulatory route.
D An open circulatory system uses more energy than a closed circulatory system uses.
Solution The solution is (A). In an open circulatory system, blood mixes with interstitial fluid
in the hemocoel. It is called hemolymph.
5 What does a simple organism such as a jellyfish use to exchange gases and nutrients with
the surrounding environment?
A Blood
B Diffusion
C Atria
D Blood vessels
Solution The solution is (B). Jellyfish use diffusion for gas exchange. Jellyfish are simple
organisms that consist of a few cell layers; they are able to exchange gases and
nutrients with the external environment using diffusion aided by the pulsing of
their bodies.
6 Fish and birds have a closed circulation. They both have a systemic circulatory system,
which delivers blood from the heart and out to the organs of the body. Birds differ from
fish in that they have a circuit that leads through the lungs and back to the heart.
What is this called?
A Pulmonary circulatory circuit
B Gill circulatory circuit
C Pulmocutaneous circulatory circuit
D Lymph circulatory circuit
Solution The solution is (A). The pulmonary circulatory circuit carries blood away from the
heart and through the lungs, where oxygenation occurs; the blood is then returned
to the heart again.
7 A student in a biology lab is attempting to reorganize some preserved animals in jars that
have lost their labels. The student notices that each jar contains notes that provide
information on each animal’s classification. The student labels the unknown animals A
through F and enters each animal’s circulatory system characteristics in the table.

Advanced Placement Biology Instructor’s Solution Manual


31 | The Circulatory System 771

If the student wished to create a phylogeny of organisms A through F, which organism


would likely be closest to the root of the phylogenetic tree?
A Organism A
B Organism B
C Organism E
D Organism F
Solution The solution is (D). The root of the phylogenetic tree represents the most primitive
method of circulation in the phylogeny. Diffusion would likely be at the root of the
tree, as this method evolved before any other given method of circulation.
8 A student in a biology lab is attempting to reorganize some preserved animals in jars
that have lost their labels. The student notices that each jar contains notes that
provide information on each animal’s classification. The student labels the unknown
animals A through F and enters each animal’s circulatory system characteristics in the
following table.

Advanced Placement Biology Instructor’s Solution Manual


772 31 | The Circulatory System

The student wants to place organisms A through F along a single branch of a phylogenetic
tree. In what order would the organisms most likely appear on the tree, beginning at the
root and moving farther away, and why?
A The order would be F, E, C, A, B, D. The general trend in circulatory system evolution is
toward increasingly closed systems.
B The order would be B, D, A, C, E, F. The general trend in circulatory system evolution is
toward increasingly open systems.
C The order would be F, E, C, A, D, B. The general trend in circulatory system evolution is
toward increasingly closed systems.
D The order would be F, E, C, D, B, A. The general trend in circulatory system evolution is
toward increasingly open systems.
Solution The solution is (C). The organisms would appear in the order F, E, C, A, D, B. The
general trend in circulatory system evolution is toward increasingly closed systems.
9 Differences in human blood types show how genetic differences have evolved over time,
affecting red blood cell structure. What is the basis for blood type classification?
A It is based on antigens made of carbohydrates, specifically glycoside and N-
acetylglucosamine, found on the surface of red blood cells.
B It is based on antigens made of glycolipids and glycoproteins, found on the surface of
red blood cells.
C It is based on antigens made of proteins, specifically peripheral and integral proteins,
found on the surface of red blood cells.
D It is based on antigens made of lipids, specifically glycerophospholipids, found on the
surface of red blood cells.
Solution The solution is (B). The classification of different blood types are based on antigens
made of glycolipids and glycoproteins found on the surface of red blood cells.

Advanced Placement Biology Instructor’s Solution Manual


31 | The Circulatory System 773

10 If a person has blood type , what antibodies relating to blood type will be found in
the person’s blood?
A A antibodies
B A antibodies and B antibodies
C Rh antibodies
D B antibodies

Solution The solution is (C). If a person has blood type , Rh antibodies will be found in
the blood. A person with blood has Rh antibodies in their blood.
11 Components found in human blood include white blood cells, red blood cells, and —
A platelets
B ostia
C hemolymph
D cardiomyocytes
Solution The solution is (A). Components found in human blood include white blood cells, red
blood cells, and platelets.
12 Up to four components can be derived from donated blood. One of those components is
plasma. Which item is NOT a basic component of plasma?
A Water
B Proteins
C Salts
D Red blood cells
Solution The solution is (D). Red blood cells are not a basic component of plasma. Red blood
cells, white blood cells, and plasma make up blood. Plasma is the liquid part of
blood; it does not include red blood cells.
13 Many platelets converge and stick together at a wound site, eventually forming a platelet
plug, also called a fibrin clot. Platelets continue to arrive at the wound site until the plug is
completely formed.
Which feedback mechanisms take place, and what would likely happen if part of the
platelet plug broke away before the wound was healed?
A This is a positive-feedback loop. The loop would restart if part of the platelet plug
broke away, calling more platelets to the site to repair the broken plug.
B This is a negative-feedback loop. The loop would restart if part of the platelet plug
broke away, calling more platelets to the site to repair the broken plug.

Advanced Placement Biology Instructor’s Solution Manual


774 31 | The Circulatory System

C This is a positive-feedback loop. The loop would not restart if part of the platelet plug
broke away.
D This is a negative-feedback loop. The loop would not restart if part of the platelet plug
broke away.
Solution The solution is (A). This is an example of a positive feedback loop, which would
restart if a part of the platelet plug broke away; more platelets would be called to
the site to repair the broken plug.
14 The diagram shows a fibrin clot forming within a blood vessel.

What constituents of the blood interact to form the clot?


A Red blood cells, serum, and vitamin K
B Fibrin, megakaryocytes, and blood proteins
C Granulocytes, platelets, and red blood cells
D Platelets, fibrinogen, and clotting factors
Solution The solution is (D). Platelets, fibrinogen, and clotting factors are the constituents
that interact to form the clot.
15 The diagram models the four different types of red blood cells in humans.

Advanced Placement Biology Instructor’s Solution Manual


31 | The Circulatory System 775

What is represented by the colored shapes on the surface of the cells, and what is the
function of the shapes?
A The shapes represent antibodies, which identify red blood cells as either part of the
body or foreign. Foreign red blood cells may be attacked by antibodies within the
blood.
B The shapes represent glycolipids and glycoproteins, which identify red blood cells as
either part of the body or foreign. Foreign red blood cells may be attacked by antigens
within the blood.
C The shapes represent glycolipids and glycoproteins, which identify red blood cells as
either part of the body or foreign. Foreign red blood cells may be attacked by
antibodies within the blood.
D The shapes represent antibodies, which identify the red blood cells as either part of
the body or foreign. This allows the body to neutralize foreign cells.
Solution The solution is (C). The different shapes on the surface of the three cells represent
glycoproteins. These glycoproteins identify the red blood cells as self and non-self.
The non-self cells are attacked by antibodies within the blood.
16 Your heart is a pump that circulates blood and oxygen around your body. Which
statement about the circulatory system is false?
A Blood in the pulmonary veins is deoxygenated.
B Blood in the inferior vena cava is deoxygenated.
C Blood in the pulmonary artery is deoxygenated.
D Blood in the aorta is oxygenated.
Solution The solution is (A). It is false that blood in the pulmonary vein is deoxygenated.
Although veins in the human body typically carry deoxygenated blood, pulmonary
veins carry oxygen-rich blood from the lungs to the left side of the heart.
17 Which statement about the heart is false?
A The mitral valve separates the left ventricle from the left atrium.
B Blood travels through the bicuspid valve to the left atrium.
C Both the aortic and the pulmonary valves are semilunar valves.
D The mitral valve is an atrioventricular valve.
Solution The solution is (B). It is false that blood travels through the bicuspid valve to the left
atrium. Blood passes through the bicuspid valve to the left ventricle.
18 In a healthy heart, a heartbeat begins within an electrical signal from which part of
the heart?
A Bundle of His
B Atrioventricular (AV)

Advanced Placement Biology Instructor’s Solution Manual


776 31 | The Circulatory System

C Sinoatrial (SA) node


D Atrial diastole
Solution The solution is (C). In a healthy heart, a heartbeat begins within an electrical signal
from the sinoatrial (SA) node, which is located near the wall of the right atrium. This
causes the two atria to contract in unison.
19 What is the cardiac cycle, and what drives it?
A The heart contracts to pump blood through the body during systole; the heart is filled
with blood during diastole. An electrical charge spontaneously pulses from the SA
node, causing the two atria to contract. The pulse reaches the AV node, where it
pauses before spreading to the walls of the ventricles. It enters the bundle of His, then
moves to the left and right bundle branches extending through the interventricular
septum. Purkinje fibers conduct the impulse from the apex up the ventricular
myocardium, causing the ventricles to contract. This pause allows the atria to empty
their contents into the ventricles before the ventricles pump out the blood.
B The heart contracts to pump blood through the body during diastole; the heart is filled
with blood during systole. An electrical charge spontaneously pulses from the SA
node, causing the two atria to contract. The pulse reaches the AV node, where it
pauses before spreading to the walls of the ventricles. It enters the bundle of His, then
moves to the left and right bundle branches extending through the interventricular
septum. Purkinje fibers conduct the impulse from the apex up the ventricular
myocardium, causing the ventricles to contract. This pause allows the atria to empty
their contents into the ventricles before the ventricles pump out the blood.
C The heart contracts to pump blood through the body during systole; the heart is filled
with blood during diastole. An electrical charge spontaneously pulses from the AV
node, causing the two atria to contract. The pulse reaches the SA node, where it
pauses before spreading to the walls of the ventricles. It enters the bundle of His, then
moves to the left and right bundle branches extending through the interventricular
septum. Purkinje fibers conduct the impulse from the apex up the ventricular
myocardium, causing the ventricles to contract. This pause allows the atria to empty
their contents into the ventricles before the ventricles pump out the blood.
D The heart contracts to pump blood through the body during systole and is filled with
blood during diastole. An electrical charge spontaneously pulses from the SA node,
causing the two atria to contract. The pulse reaches the AV node, where it pauses
before spreading to the walls of the ventricles. It enters the Purkinje fibers, then
moves to the left and right bundle branches extending through the interventricular
septum. The bundle of His conducts the impulse from the apex up the ventricular
myocardium, causing the ventricles to contract. This pause allows the atria to empty
their contents into the ventricles before the ventricles pump out the blood.

Advanced Placement Biology Instructor’s Solution Manual


31 | The Circulatory System 777

Solution The solution is (A). The cardiac cycle is the filling and emptying of the heart of blood.
It is timed by an electrical signal in the muscle of the heart, which causes the heart
to contract. In the part of the cardiac cycle called systole, the heart contracts,
pushing out the blood and pumping it through the body. Diastole is the relaxation
phase, when the heart fills with blood. The internal pacemaker of the heart starts at
the sinoatrial (SA) node, located near the wall of the right atrium. An electrical
charge spontaneously pulses from the SA node, which causes the two atria to
contract together. The pulse reaches the second node, called the atrioventricular
(AV) node, located between the right atrium and right ventricle, where it pauses for
0.1 s before spreading to the walls of the ventricles. From the AV node, the electrical
impulse enters the bundle of His, and then moves to the left and right bundle
branches extending through the interventricular septum. Finally, the Purkinje fibers
conduct the impulse from the apex of the heart up the ventricular myocardium,
causing the ventricles to contract. This pause allows the atria to empty their
contents into the ventricles before the ventricles pump out the blood.
20 Compare and contrast veins and arteries.
A Both veins and arteries have three distinct layers. Veins take blood away from the
heart and arteries bring blood back to the heart.
B Both veins and arteries have three distinct layers. Arteries take blood away from the
heart and veins bring blood back to the heart.
C Both veins and arteries have valves to prevent the backflow of blood. Arteries take
blood away from the heart and veins bring blood back to the heart.
D Both veins and arteries have valves to prevent the backflow of blood. Veins take blood
away from the heart and arteries bring blood back to the heart.
Solution The solution is (B). Both veins and arteries have three distinct layers (or tunics). The
first tunic is a smooth, inner lining of endothelial cells that are in contact with red
blood cells. This endothelial tunic is continuous with the endocardium of the heart.
Veins and arteries have two further tunics that surround the endothelium; the
middle tunic is composed of smooth muscle and the outermost layer is connective
tissue. The elastic connective tissue stretches and supports the blood vessels, while
the smooth muscle layer helps regulate blood flow by altering vascular resistance.
Arteries are designed to take blood away from the heart, and veins are designed to
bring blood back to the heart. Compared to veins, arteries have thicker smooth
muscle and connective tissue to accommodate the higher speed and pressure of
freshly pumped blood. Veins are structurally differently; they have valves to prevent
the backflow of blood. Because veins have to work against gravity to get blood
back to the heart, contraction of skeletal muscle assists with the flow of blood back
to the heart.

Advanced Placement Biology Instructor’s Solution Manual


778 31 | The Circulatory System

21 The diagram shows the neural structures that control and coordinate the beating of
the heart.

How would the cardiac cycle be affected if neural signals were blocked within the Purkinje
fiber?
A The atria and ventricles would contract at the same time.
B The ventricles would not contract.
C The atria would contract first, followed by the ventricles.
D Only the left atrium would contract.
Solution The solution is (B). If neural signals were blocked within the Purkinje fiber, ventricles
would not contract.
22 The diagram shows the neural structures that control and coordinate the beating of
the heart.

Advanced Placement Biology Instructor’s Solution Manual


31 | The Circulatory System 779

How would the cardiac cycle be affected if the signal were blocked at the atrioventricular
node, and why?
A The atria would contract, but the ventricles would not, because the atrioventricular
node passes signals to the Purkinje fibers, which allows the ventricles to contract.
B The ventricles would contract, but the atria would not, because the atrioventricular
node passes signals to the Purkinje fibers, which allows the ventricles to contract.
C The atria would contract, but the ventricles would not, because the atrioventricular
node passes signals to the Purkinje fibers, which allows the atria to contract.
D The ventricles would contract, but the atria would not, because the atrioventricular
node passes signals to the Purkinje fibers, which allows the atria to contract.
Solution The solution is (A). If signals bypassed the atrioventricular node, the atria would
contract but the ventricles would not.

CRITICAL THINKING QUESTIONS


23 What is a closed circulatory system? Compare the differences in heart structure for
animals with closed circulatory systems.
A A closed circulatory system is a system in which the blood mixes with the interstitial
fluid. Fish have a two-chambered heart. Amphibians and reptiles have a three-
chambered heart. Mammals and birds have a four-chambered heart and double
circulation.
B A closed circulatory system is a system in which blood is separated from the interstitial
fluid. Fish have a two-chambered heart. Amphibians and reptiles have a three-
chambered heart. Mammals and birds have a four-chambered heart and double
circulation.
C A closed circulatory system is a system in which blood is separated from the interstitial
fluid. Amphibians have a two-chambered heart. Fishes and reptiles have a three-
chambered heart. Mammals and birds have a four-chambered heart and double
circulation.
D A closed circulatory system is a system in which blood mixes with the interstitial fluid.
Amphibians have a two-chambered heart. Fishes and reptiles have a three-chambered
heart. Mammals and birds have a four-chambered heart and double circulation.
Solution The solution is (B). A closed circulatory system is one in which the blood is separated
from the interstitial fluid. Vertebrates have closed circulatory systems; between
vertebrate groups there are differences in heart structure and circulatory systems
due to evolution. Fish have a two-chambered heart with unidirectional circulation.
Amphibians have a three-chambered heart; most reptiles also have a three-
chambered heart, which allows little mixing of the blood. Mammals and birds have a
four-chambered heart, in which no mixing of the blood occurs, and double
circulation.

Advanced Placement Biology Instructor’s Solution Manual


780 31 | The Circulatory System

24 A circulatory system is the main method for transporting gases and nutrients throughout
the body. What happens in a closed circulatory system, and how does a closed circulatory
system compare to an open circulatory system?
A Blood in a closed circulatory system is present inside blood vessels; it follows a
unidirectional path from the heart and around the systemic circulatory route, and
then returns to the heart. It is less controlled and structured than an open circulatory
system, but it transfers nutrients and waste products more efficiently.
B Blood in a closed circulatory system is not enclosed in blood vessels; it is pumped into
a hemocoel, which circulates around the organs, and then reenters the heart through
ostia. It is more structured and controlled than an open circulatory system, and it
transports nutrients and waste products more efficiently.
C Blood in a closed circulatory system is not enclosed in blood vessels; it is pumped into
a hemocoel, which circulates around the organs, and then reenters the heart through
ostia. It is less controlled and structured than an open circulatory system, but it
transports nutrients and waste products more efficiently.
D Blood in a closed circulatory system is present inside blood vessels; it follows a
unidirectional path from the heart around the systemic circulatory route, and then
returns to the heart. It is more structured and controlled, and transports nutrients and
waste products more efficiently than an open circulatory system.
Solution The solution is (D). In a closed circulatory system, blood is contained inside blood
vessels and follows a unidirectional path from the heart and around the systemic
circulatory route, then returns to the heart. Compared to an open circulatory
system, a closed circulatory system is more structured and controlled, and nutrients
and waste products are transported more efficiently.
25 What is one advantage of a four-chambered heart over a three-chambered heart?
A In a four-chambered heart, oxygenated blood carried by the left side of the heart is
more effectively separated from deoxygenated blood carried by the right side, which
assists in more efficient movement of oxygen around the body.
B In a four-chambered heart, oxygenated blood carried by the right side of the heart is
more effectively separated from deoxygenated blood carried by the left side, which
assists in more efficient movement of oxygen around the body.
C In a four-chambered heart, oxygenated blood carried by the left side of the heart is
less effectively separated from deoxygenated blood carried by the right side, which
assists in more efficient movement of oxygen around the body.
D In a four-chambered heart, oxygenated blood carried by the right side of the heart is
less effectively separated from deoxygenated blood carried by the left side, which
assists in more efficient movement of oxygen around the body.

Advanced Placement Biology Instructor’s Solution Manual


31 | The Circulatory System 781

Solution The solution is (B). In a four-chambered heart, oxygenated blood carried by the right
side of the heart is more effectively separated from deoxygenated blood carried by
the left side, which assists in more efficient movement of oxygen around the body.
26 What are red blood cells also known as?
A Lymphocytes
B Monocytes
C Erythrocytes
D Basophils
Solution The solution is (C). Red blood cells are also known as erythrocytes.
27 How does the structure of mammalian red blood cells allow them to deliver oxygen to the
cells of the body?
A Their size and shape allow them to carry and transfer oxygen.
B Their disc shape contains many small vesicles that allow them to carry and transfer
oxygen.
C They have nuclei and do not contain hemoglobin.
D They contain coagulation factors and antibodies.
Solution The solution is (A). Their size and shape allow red blood cells to carry and transfer
oxygen. The small size and larger surface area of red blood cells allow for rapid
diffusion of oxygen and carbon dioxide across the plasma membrane.
28 Which statement best describes plasma?
A It is a protein synthesized in the liver.
B It is a liquid that contains only lipids and antibodies.
C It is a blood component that is separated by spinning blood.
D It is an antibody produced in the mucosal lining.
Solution The solution is (C). Plasma is the liquid component of blood; it is separated from
blood cells and platelets by spinning the blood in a centrifuge.
29 What is the heart’s internal pacemaker?
A It is an internal implant that sends an electrical impulse through the heart.
B It is the part of the heart that initiates an electrical impulse, called the sinoatrial node.
C It is the excitation of cardiac muscle cells at the atrioventricular and sinoatrial nodes.
D It is the contracting of muscles that starts in the aorta.
Solution The solution is (B). The heart’s internal pacemaker is the part of the heart that
initiates an electrical impulse, called the sinoatrial node.

Advanced Placement Biology Instructor’s Solution Manual


782 31 | The Circulatory System

30 Cardiomyocytes are similar to skeletal muscle because they —


A beat involuntarily
B are attached to bones
C pulse rhythmically
D are striated
Solution The solution is (D). Cardiomyocytes are striated like skeletal muscle cells.
31 The diagram shows the internal anatomy of the heart.

How would blood circulation beyond the heart be most directly affected if the pulmonary
valve could NOT open?
A Blood could not reach the rest of the body.
B Blood could not reach the lungs.
C Blood could not return from the lungs.
D Blood could not return from the rest of the body.
Solution The solution is (B). If the pulmonary valve could not open, blood could not reach
the lungs.
32 The diagram shows the internal anatomy of the heart.

Advanced Placement Biology Instructor’s Solution Manual


31 | The Circulatory System 783

How would blood circulation beyond the heart be affected if the tricuspid valve could
NOT open?
A Blood could not enter the pulmonary veins; therefore, it could not reach the lungs
B Blood could not enter the pulmonary artery; therefore, it could not reach the heart.
C Blood could not enter the pulmonary artery; therefore, it could not reach the lungs.
D Blood could not enter the pulmonary veins; therefore, it could not reach the heart.
Solution The solution is (C). If the tricuspid valve could not open, blood could not enter the
pulmonary artery; therefore, it could not reach the lungs.
33 Why is it useful for blood to travel slowly through capillary beds?
A To allow antibodies to enter infected cells and to promote the diffusion of fluid into
the interstitial space
B To assist with gas and nutrient exchange and to prevent the diffusion of fluid into the
interstitial space
C To assist with gas and nutrient exchange and to promote the diffusion of fluid into the
interstitial space
D To allow antibodies to enter infected cells and to prevent the diffusion of fluid into the
interstitial space
Solution The solution is (C). Gas exchange (oxygenation) happens in the capillary beds. The
slow rate of travel through the capillary beds, which reach almost every cell in the
body, assists with gas and nutrient exchange and promotes the diffusion of fluid into
the interstitial space. After the blood has passed through the capillary beds to the
venules, veins, and main venae cavae, the rate of flow increases again; however, the
rate is still much slower than the initial rate in the aorta.

TEST PREP FOR AP® COURSES


34 Based only on their differences in circulatory systems, would insects or birds be favored
during a food shortage? Which is the most likely explanation why?
A Insects because their open circulatory system often requires less energy to move
blood throughout the body than a closed circulatory system.
B Birds because their closed circulatory system stores energy better than a open
circulatory system.
C Insects because their closed circulatory system often requires less energy to move
blood throughout the body than a closed circulatory system.
D Birds because their open circulatory system is more efficient at delivering nutrients
than a closed circulatory system.

Advanced Placement Biology Instructor’s Solution Manual


784 31 | The Circulatory System

Solution The solution is (A). Insects have an open circulatory system while birds have a closed
circulatory system. Because an open circulatory system travels through a large cavity
rather than relativity small blood vessels, less energy is required to maintain blood
flow in an open circulatory system then in a closed circulatory system.
35 This diagram shows how insects have a heart that is elongated throughout much of their
body. This is opposed to the heart of vertebrates, such as humans, which does not spread
throughout the length of the body.

Why was it likely advantageous for insects to evolve an elongated heart as opposed to the
centrally located heart of vertebrates?
A Because an elongated heart allows blood to easily reach all the parts of the body from
the heart
B Because the elongated heart allows more blood to reach all the parts of the body, it
allows for more nutrient exchange
C Because an elongated heart allows blood to easily return to the heart from a few
select points within the body cavity
D Because an elongated heart allows blood to easily return to the heart from most
points within the body cavity
Solution The solution is (D). The elongated heart is advantageous for insects because insects
have an open circulatory system in which hemolymph is circulated through the body
cavity rather than through blood vessels. The elongated heart allows blood to easily
return to the heart from most points within the body cavity.
36 Insects have open circulatory systems in which hemolymph circulates through the body
cavity rather than through closed blood vessels. Birds, like other vertebrates, have closed
circulatory systems in which blood remains within arteries, veins, and capillaries as it
circulates.
How has this difference most likely influenced the evolution of birds and insects?
A The open circulatory system of birds warms their entire body quicker, allowing them
to live in colder climates more successfully than insects.
B The closed circulatory system allows blood flow to increase to the flight muscles
during flight, allowing insects to be better fliers than birds.

Advanced Placement Biology Instructor’s Solution Manual


31 | The Circulatory System 785

C The greater efficiency of a closed circulatory system allowed birds to become larger
than insects.
D The simplicity of an open circulatory system allows insects to have a higher
metabolism than birds.
Solution The solution is (C). The greater efficiency of a closed circulatory system allowed birds
to become larger than insects.
37 Serum is used to diagnose and monitor diseases, and it is also used to classify blood types.
Which statement best describes serum?
A It is a liquid that consists of 90 percent water and has coagulation factors.
B It is plasma without the coagulation factors.
C Serum is made only of albumin.
D Serum is the same substance as interstitial fluid.
Solution The solution is (B). Serum is plasma but without the coagulation factors.
38 How are blood types distinguished from one another?
A Blood types are distinguished on the basis of the presence or absence of different
metals found on the surface of red blood cells.
B Blood types are distinguished on the basis of the presence or absence of different
proteins found inside the red blood cells.
C Blood types are distinguished on the basis of the presence or absence of different
lipids found inside the red blood cells.
D Blood types are distinguished based on the presence or absence of different antigens
found on the surface of the red blood cells.
Solution The solution is (D). A blood type is often referred to as a blood group. The blood
group is classified by the presence or absence of different antigens found on the
surface of the red blood cells.
39 The inflammatory response involves increasing blood flow to areas of the body that
contain immune system cells fighting a pathogen or foreign object. The inflammatory
response is an example of a positive feedback loop.
Based on this information, what would cause the inflammatory response to stop in a
certain area of the body?
A The white blood cells destroy all of the pathogens and foreign objects.
B The white blood cells disperse away from the site.
C More white blood cells arrive to fight the pathogens and foreign objects.
D Fewer white blood cells are needed to finish fighting the infection.

Advanced Placement Biology Instructor’s Solution Manual


786 31 | The Circulatory System

Solution The solution is (B). The inflammatory response would stop in a certain area of the
body if the white blood cells dispersed away from the site.
40 What is the phase of the cardiac cycle when the heart contracts, which pushes out the
blood and pumps it around the body?
A Diastole
B Cardiac output
C Myocardial infarction
D Systole
Solution The solution is (D). Systole is the phase of the cardiac cycle when the heart
contracts, pushing the blood out of the heart and pumping it around the body.
41 In routine physicals, doctors often test for problems with the electrical activity of a
person’s heart. What can measure the electrical impulses in the heart?
A An endocardium
B An electrocardiogram (ECG)
C A peripheral resistance
D A cardiac output
Solution The solution is (B). An electrocardiogram (ECG) can measure the electrical impulses
in the heart.
42 The diagram shows a cross section of a blood vessel.

Which part of the labeled structure would allow vasodilation in the blood vessel and
under what conditions would vasodilation occur?
A Part B would allow vasodilation in the blood vessel if the blood vessel brings blood to
relatively inactive cells, such as fat cells.
B Part C would allow vasodilation in the blood vessel if the heart and body’s metabolism
slow.

Advanced Placement Biology Instructor’s Solution Manual


31 | The Circulatory System 787

C Part B would allow vasodilation in the blood vessel if the blood vessel takes blood
away from an active organ system.
D Part C would allow vasodilation in the blood vessel if the blood vessel takes blood to a
muscle cell that is contracting quickly.
Solution The solution is (D). Part C would allow vasodilation in the blood vessel if the blood
vessel took blood to a muscle cell that was contracting quickly.
43 The diagram shows a cross-section of an artery and a vein.

What is the function of the tunica externa and why is it thicker in the artery than in
the vein?
A The tunica externa prevents blood cell adhesion to the wall of the vessel. It is thicker
in the artery to protect against the higher blood pressure found in arteries.
B The tunica externa protects the vessel from wear and tear and provides support for
the vessel. It is thicker in the artery to protect against the higher blood pressure found
in arteries.

Advanced Placement Biology Instructor’s Solution Manual


788 31 | The Circulatory System

C The tunica externa protects the vessel from wear and tear and provides support for
the vessel. It is thicker in the artery to protect against the lower blood pressure found
in arteries.
D The tunica externa prevents blood cell adhesion to the wall of the vessel. It is thicker
in the artery to protect against the lower blood pressure found in arteries.
Solution The solution is (B). The tunica externa protects the vessel from wear and tear and
provides support for the vessel. It is thicker in the artery to protect against the
higher blood pressure found in arteries.

SCIENCE PRACTICE CHALLENGE QUESTIONS


31.1 Overview of the Circulatory System
44 Like Gram-positive bacteria, red blood cells have carbohydrates on the cell surface. The A,
B, A/B, and O blood types are designations of the phenotypes expressed by the alleles
that code for these cell surface carbohydrates.
A. Describe the relationship between regulation of expression and the differences among
the A, B, A/B, and O types of blood cells.
B. Explain why the blood group phenotype does not display non-Mendelian inheritance,
and describe the simplest alternative model that explains this deviation.
Immune system T-cells recognize cell surface carbohydrates of bacterial and red blood
cells. That these similarities have consequences for survival is indicated by the observation
that individuals with blood type O are more susceptible to infection by Vibrio, the Gram-
negative bacterium that causes cholera; in contrast, individuals with type A/B are more
susceptible to infections from a broad range of E. coli variants, all of which are also Gram-
negative.
C. Describe the likely reason for the increased susceptibility of individuals with type A/B
blood in terms of the antigen-antibody model of specific immune response. Justify the
selection of data that would allow a test of your reasoning.
D. The distribution of blood types is determined in a population. The results are displayed
in the table. Fill in the frequency of each genotype. Don’t forget that the total should add
up to 1.00 (100 percent).
Type Observed Frequency
A 501
B 794
AB 236
O 601
Total, N 2,132

Advanced Placement Biology Instructor’s Solution Manual


31 | The Circulatory System 789

Recall that the frequency of an allele can be determined if the genotype is known. If we
take an example of a gene with two possible alleles, of which one is dominant and one is
recessive, we need to calculate the number of each allele and divide by the total number
of alleles (not genotypes). For example, let’s say that you have 50 homozygous dominant
individuals and 50 homozygous recessive individuals. That means that you have
100 dominant alleles (two per homozygous dominant individual) and 100 recessive
alleles (two per homozygous recessive individuals) and a total of 200 alleles. The
frequency of the dominant allele is 100/200=0.5.
 Using the observed numbers of each genotype in the table, calculate the
frequency of each allele as described above. Don’t forget that the total should
add up to 1.00 (100 percent of the alleles).
Genotype Observed Frequency
I AI A 320
I A io 181
IBIB 349
Bo
Ii 445
A B
II 236
ii 601
Total, N 2,132

Allele Probability
A
B
O
 Using these probabilities, calculate the expected frequencies (E) of each blood
type using the Hardy-Weinberg equation for three alleles given below.
p = frequency of IA
q = frequency of IB
r = frequency of i
p2 + 2pq + q2 + 2pr + 2qr + r2
Genotype Symbol Frequency/Total individuals
I AI A p2 0.061/131
I AI B 2pq 0.160/342
IBIB q2 0.104/222
A
Ii 2pr 0.213/454

Advanced Placement Biology Instructor’s Solution Manual


790 31 | The Circulatory System

Genotype Symbol Frequency/Total individuals


IBi 2qr 0.277/591
2
ii r 0.184/392
 Use the information in the table above to calculate expected numbers to add to
this table.
Type Observed (O) Expected Frequency (E)
A 501
B 794
A/B 236
O 601
 Apply your understanding of the conceptual foundation of these equations by
restating in words the relationship represented by E(A).
 Apply a χ2 test at the 95 percent confidence level and 3 degrees of freedom
(number of traits minus one) to evaluate the claim that these data indicate
Hardy-Weinberg equilibrium of the ABO system for this population. The
definition of this statistic is provided on the AP Biology Exam.

This table is also provided on the AP Biology Exam.


p 1 2 3 4 5 6 7 8
0.05 3.84 5.99 7.82 9.49 11.07 12.59 14.07 15.51
0.01 6.64 9.32 11.34 13.28 15.09 16.81 18.48 20.09
Degrees of Freedom
 Hardy-Weinberg equilibrium is consistent with the assumption of no change in
the distribution of alleles over time. Justify the selection of data that should be
obtained to further test this assumption. Homologous genes coding for the
carbohydrates that are present on the surfaces of red blood cells are found in
amphibians and mammals but not in fish, implying a last common ancestor for
the ABO gene system at least 20 million years ago.
E. The difference between genes coding for A and B is a single nucleotide replacement.
Evaluate the likelihood that negative selection pressures have been active in the evolution
of this system.
Solution Sample answer:
A. These molecules on the surface of the red blood cell are synthesized by proteins
that are expressions of the individual’s genome. The A and B alleles are codominant,
with both alleles being dominant to the allele that forms type O blood. Therefore,

Advanced Placement Biology Instructor’s Solution Manual


31 | The Circulatory System 791

type O blood is only possible if an individual has neither an A nor a B allele. The
codominance also means that an individual with one A allele and one B allele with
have both alleles expressed, resulting in the AP blood type.
B. Both strands of DNA are expressed so that AB heterozygosity leads to the
production of both carbohydrates (AB), homozygosity (AA or AO, BB or BO) leads to
only a single form, and alleles that do not encode functional proteins (OO) lead to no
surface carbohydrates.
C. Individuals with type AB can have no antibodies that recognize type A or type B
surface molecules as antigens. Because E. coli are Gram-negative, they have surface
molecules that are very similar. Individuals with either A or B blood types should
therefore be susceptible to a narrower range of E. coli variants. To test this we
need data describing frequency of infection and its dependence on infection-
causing agents.
D. The table shows the frequencies of the different blood types in a population of
2,132 individuals.
Type Observed Frequency
A 501 0.235
B 794 0.372
AB 236 0.111
O 601 0.282
Total, N 2,132 1.000
The frequencies of the genotypes are shown in the table.
Genotype Observed Frequency
I AI A 320 0.150
Ao
Ii 181 0.085
B B
II 349 0.164
IBio 445 0.209
I AI B 236 0.111
ii 601 0.281
Total, N 2,132 1.00
Frequencies of the A, B, and O alleles can be calculated as follows:

Advanced Placement Biology Instructor’s Solution Manual


792 31 | The Circulatory System

Allele Probability
A 0.2479
B 0.265
O 0.282
Expected frequencies of each genotype:
Genotype Symbol Frequency/Total Individuals
A A 2
II p 0.061/131
I AI B 2pq 0.160/342
IBIB q2 0.104/222
A
Ii 2pr 0.213/454
B
Ii 2qr 0.277/591
ii r2 0.184/392
Expected frequencies:
Type Observed (O) Expected Frequency (E)
A 501 585
B 794 813
AB 236 342
O 601 392
The total number of instances of a particular genome is equal to the products of the
probabilities of the individual alleles multiplied by the number of copies. So for each
homozygous individual, the probability of the genome is p2. But the phenotype of
the homozygous trait might also be caused by expression of a functional protein on
only one strand: for example, AO. Since this can occur in two ways, the number of
instances is 2pr, where r is the probability of the nonexpressed allele, O.
The value of χ 2 is calculated as 3.87, using the formula provided. This value is less
than 7.82 so that assumptions made by the Hardy-Weinberg model are not falsified.
A careful investigation requires a follow-up collection of phenotypes in subsequent
generations. Because this is unlikely, one may take these data as a tentative
indication that the system is not evolving.
E. The stability of the system of multiple alleles for which a mechanism for selection
is suggested by the susceptibility to infection indicates some other influences. The
search for these influences is ongoing.

Advanced Placement Biology Instructor’s Solution Manual


31 | The Circulatory System 793

31.2 Components of the Blood


45 The immune system rejection of transplanted organs and the availability of organ donors
are key factors in determining survival. Blood ABO compatibility is always a criterion in
matching donors and recipients in adult patients; it was once a consideration for infant
patients. Period of time on the waitlist for a suitable donor is critical because the health of
the patient degrades while waiting. Dipchand et al. (American Journal of Transplantation,
10, 2010) made a comparison of survival rates for infants when the donor heart was ABO
compatible and incompatible, as shown in the graph.

Based on these data, justify the claim that expression of blood group immune response
develops over time and that this provides a window of opportunity for transplantation.
Solution Sample answer: Blood group compatibility is always a criterion for adult patients;
there is a lower risk of immune system rejection in infants. The evidence is that,
following the loss of lives associated with waiting for ABO compatible donors, the
parallel slopes of the two curves in the diagram indicate no risk of rejection in the
ABO incompatible transplantation.

Advanced Placement Biology Instructor’s Solution Manual


794 32 | Osmotic Regulation and Excretion

32 | OSMOTIC REGULATION AND


EXCRETION
REVIEW QUESTIONS
1 Why is the sodium ion at the highest concentration in extracellular fluid?
A Sodium diffuses freely through the cell membrane.
B The sodium/potassium pump removes sodium ions from the cell.
C The blood contains a high concentration of sodium.
D Sodium is actively taken up by the cells for use in the cytoplasm.
Solution The solution is (B). Due to the removal of sodium ions from the cell, sodium is
present in higher concentration in the extracellular, not intracellular, fluid.
2 What is given to a dehydrated human patient when he or she needs to be given fluids
intravenously?
A Water, which is hypotonic with respect to body fluids
B Saline at a concentration that is isotonic with respect to body fluids
C Glucose because it is a nonelectrolyte
D Blood
Solution The solution is (B). The osmole concentration of saline is the same as solute
concentration of cells, so no lysis will be caused.
3 What is the approximate osmolarity of body fluids?
A 100 mOsm
B 300 mOsm
C 1,000 mOsm
D It is not constantly maintained.
Solution The solution is (B). At this osmolarity, the body fluids are isotonic—that is, the
osmotic pressure is equal across two compartments.
4 One milliequivalent of a ferric (III) ion (Fe+3) is equal to how many millimoles?
A 0.13
B 0.75
C 0.5
D 0.25

Advanced Placement Biology Instructor’s Solution Manual


32 | Osmotic Regulation and Excretion 795

Solution The solution is (D). Electrolyte concentrations are expressed in terms of


milliequivalent per liter (meq/L), which is equal to the ion concentration (in
millimoles) multiplied by the number of electrical charges on the ion. Therefore, for
a charge of 3, 1 milliequivalent is equal to 0.25 millimoles.
5 What is the difference between an osmoregulator and an osmoconformer?
A Osmoregulators can change the osmotic pressure of their body fluids, while
osmoconformers cannot.
B Osmoconformers can change the osmotic pressure of their body fluids, while
osmoregulators cannot.
C Osmoregulators match the osmotic pressure of their cells with that of the
environment, while osmoconformers do not.
D Osmoconformers match the osmotic pressure of their cells with that of the
environment, while osmoregulators do not.
Solution The solution is (D). An osmoregulator regulates its body chemistry to maintain an
osmotic pressure in its cells that is different from that of the surrounding
environment. An osmoconformer, on the other hand, regulates its body chemistry to
maintain an osmotic pressure that is similar to that of the surrounding environment.
6 Why does a salmon have to be an osmoregulator?
A They need to remove excess salt from ocean water.
B They move between areas of fresh and salt water.
C They travel for short stretches across the land between streams.
D Freshwater is osmotically neutral compared with their body fluids.
Solution The solution is (B). Osmoregulation allows salmons to be tolerant of a wide range of
aquatic environments, which is needed for such euryhaline organisms.
7 What tends to happen to cells in a hypotonic environment?
A They remain the same size.
B They shrink due to water loss.
C They allow water to flow in and out at the same rate.
D They swell due to water gain.
Solution The solution is (D). Hypotonic solutions have a low concentration of salt. In such an
environment the cell will swell and possibly burst due to the movement of water
into the cell.
8 What is a function of the kidney?
A Osmoregulation
B Excretion of solid waste

Advanced Placement Biology Instructor’s Solution Manual


796 32 | Osmotic Regulation and Excretion

C Production of oxygen
D Reabsorption of all nutrients
Solution The solution is (A). The kidney is important in osmoregulation. It can aid in the
increase of oxyhemoglobin indirectly but cannot produce oxygen.
9 The kidney is made up of over a million nephrons that give it a granular appearance. What
is a nephron?
A A network of capillaries
B The cup-shaped chamber surrounding the glomerulus
C The functional unit of the kidney
D The bow-shaped artery at the base of medullary pyramids
Solution The solution is (C). This is the definition of a nephron.
10 How do juxtamedullary nephrons differ from cortical nephrons?
A Juxtamedullary nephrons have a longer loop of Henle, allowing them to regulate urine
concentration better than cortical nephrons.
B Juxtamedullary nephrons have a shorter loop of Henle, allowing them to regulate
urine concentration better than cortical nephrons.
C Juxtamedullary nephrons have a larger glomerulus, allowing them to filter blood at a
greater rate than cortical nephrons.
D Juxtamedullary nephrons have a smaller glomerulus, allowing them to filter blood at a
greater rate than cortical nephrons.
Solution The solution is (A). Juxtamedullary nephrons have a longer loop of Henle than
cortical nephrons.
11 What is the casing around the glomerulus called?
A Distal tubule
B Loop of Henle
C Bowman’s capsule
D Renal pyramid
Solution The solution is (C). The Bowman’s capsule or glomerular capsule is the cup-shaped
structure that encloses the glomerulus.

Advanced Placement Biology Instructor’s Solution Manual


32 | Osmotic Regulation and Excretion 797

12 In the loop of Henle, the permeability of the membrane changes. The descending limb
and ascending limb are permeable to what, respectively?
A Solutes and water
B Proteins and solutes
C Water and proteins
D Water and solutes
Solution The solution is (D). The descending limb is permeable to water, not solutes. The
ascending limb is permeable to solutes, not water.
13 What do humans excrete, and how soluble is it compared to uric acid?
A Urea, which is less soluble than uric acid
B Urea, which is more soluble than uric acid
C Ammonia, which is more soluble than uric acid
D Ammonia, which is less soluble than uric acid
Solution The solution is (B). Mammals excrete urea due to the formation of the toxic
ammonia.
14 How is the network of blood vessels of the kidney described?
A Blood enters and exits the kidney at the renal hilum, and the renal blood supply starts
with the branching of the aorta into the renal arteries. They end with the exiting of the
renal veins to join the superior vena cava. Each segmental renal artery formed by the
splitting of the renal arteries splits again into interlobar arteries and enters the renal
columns. These again split to form arcuate arteries, from which cortical radiate
arteries radiate out and branch into many afferent arterioles that enter the capillaries
supplying the nephrons. Veins trace the path of arteries and have similar names,
except there are no segmental veins.
B Blood enters and exits the kidney at the renal hilum, and the renal blood supply starts
with the branching of the aorta into the renal arteries. They end with the exiting of the
renal veins to join the inferior vena cava. Each segmental renal artery formed by the
splitting of the renal arteries splits again into interlobar arteries and enters the renal
columns. These again split to form arcuate arteries, from which cortical radiate
arteries radiate out and branch into many afferent arterioles that enter the capillaries
supplying the nephrons. Veins trace the path of arteries and have similar names,
except there are no segmental veins.
C Blood enters and exits the kidney at the renal hilum, and the renal blood supply starts
with the branching of the aorta into the renal arteries. They end with the exiting of the
renal veins to join the inferior vena cava. Each segmental renal artery formed by the
splitting of the renal arteries splits again into interlobar arteries and enters the renal
columns. These again split to form cortical radiate arteries that radiate out and
branch into many afferent arterioles that enter the capillaries supplying the nephrons.

Advanced Placement Biology Instructor’s Solution Manual


798 32 | Osmotic Regulation and Excretion

Veins trace the path of arteries and have similar names, except there are no
segmental veins.
D Blood enters and exits the kidney at the renal hilum, and the renal blood supply starts
with the branching of the aorta into the renal arteries. They end with the exiting of the
renal veins to join the inferior vena cava. Each segmental renal artery formed by the
splitting of the renal arteries splits again into interlobar arteries and enters the renal
columns. These again split to form arcuate arteries, from which cortical radiate
arteries radiate out and branch into many afferent arterioles that enter the capillaries
supplying the nephrons. Veins trace the path of arteries and have all the same names
as the arteries following the same path.
Solution The solution is (B). The arteries, veins, and nerves that supply the kidney enter and
exit at the renal hilum. The renal blood supply starts with the branching of the aorta
into the renal arteries. Each of these is named based on the region of the kidney
they pass through. They end with the exiting of the renal veins to join the inferior
vena cava. The renal arteries split into several segmental arteries upon entering the
kidneys. Each segmental artery splits further into several interlobar arteries and
enters the renal columns, which supply the renal lobes. The interlobar arteries split
at the junction of the renal cortex and medulla to form the arcuate arteries. The
arcuate arteries form arcs along the base of the medullary pyramids. Cortical radiate
arteries radiate out from the arcuate arteries. The cortical radiate arteries branch
into numerous afferent arterioles and then enter the capillaries supplying the
nephrons. Veins trace the path of the arteries and have similar names, except there
are no segmental veins.
15 Which statement about contractile vacuoles in microorganisms is true?
A They exclusively perform an excretory function.
B They contain digestive enzymes.
C They originate from the cell membrane.
D They are only used for fluid storage.
Solution The solution is (C). Contractile vacuoles originate from the cell membrane.
16 Some unicellular organisms, such as the amoeba, ingest food by endocytosis. The food
vesicle fuses with a lysosome, which digests the food. Waste is excreted by which
process?
A Exocytosis
B Filtration
C Osmoregulation
D Dilatation
Solution The solution is (A). The process by which waste is excreted by single cell organisms,
such as amoebas, is called exocytosis.

Advanced Placement Biology Instructor’s Solution Manual


32 | Osmotic Regulation and Excretion 799

17 What does active transport of K+ in the Malpighian tubules ensure?


A Water leaves the Malpighian tubules.
B Water moves into the Malpighian tubules.
C K+ moves into the Malpighian tubules.
D K+ leaves the Malpighian tubules.
Solution The solution is (D). Exchange pumps lining the Malpighian tubules actively transport
H+ ions K+ ions out of the tubules where they can be reabsorbed by the body.
18 Flame cells are primitive excretory organs found in which organisms?
A Arthropods
B Annelids
C Mammals
D Flatworms
Solution The solution is (D). Free-living flatworms possess flame cells, along with rotifers and
nemertines.
19 A cell has a malfunction in its contractile vacuoles. How would this affect the cell?
A The cell’s nephrons would no longer be able to reabsorb water.
B The cell’s cytoplasm would become very concentrated.
C The cell could no longer excrete waste products.
D The cell would uncontrollably take on water until it burst.
Solution The solution is (D). Contractile vacuoles protect a cell from absorbing excessive
amounts of water, preventing it from bursting by excreting excess water.
20 Which toxic substance is formed by nitrogenous waste?
A Chlorine
B Potassium
C Ammonia
D Sodium
Solution The solution is (C). Nitrogenous waste contains toxic nitrogen, which is eliminated as
ammonia, urea, and uric acid.
21 What substance is secreted by most aquatic animals and why?
A Ammonia, because it can be easily diluted in an aquatic environment
B Uric acid, because aquatic animals must preserve water
C Urea, because ammonia needs to be converted before it can safely be stored in
the body.

Advanced Placement Biology Instructor’s Solution Manual


800 32 | Osmotic Regulation and Excretion

D Chlorine, because it can easily be converted to a gas


Solution The solution is (A). Aquatic animals and many invertebrates tend to excrete
ammonia. Mammals excrete urea or guano and birds, insects, and many reptiles
excrete uric acid. Chlorine is not a nitrogenous waste.
22 Which water-insoluble compound is similar to purines found in nucleic acids and tends to
form a white paste or powder?
A Urea
B Guanine
C Ammonia
D Uric acid
Solution The solution is (D). Uric acid is water insoluble and forms a white paste or powder
when it is excreted.
23 The urea cycle is also referred to as the —
A Krebs cycle
B ornithine cycle
C citric acid cycle
D uric acid cycle
Solution The solution is (B). The urea cycle is also known as the ornithine cycle because of the
conversion of the amino acid L-ornithine into different intermediates.
24 Renin is produced in the kidney nephrons and performs what function?
A Increases vasoconstriction
B Prevents loss of sodium
C Increases blood pressure
D Prevents water loss
Solution The solution is (C). Renin is secreted by a part of the juxtaglomerular complex and
increases blood pressure by acting on angiotensinogen.
25 Which hormone elicits the fight-or-flight response, and under which circumstance is it
released?
A Antidiuretic hormone, presence of alcohol in the blood
B Atrial natriuretic peptide, high blood pressure
C Aldosterone, low water content in the blood
D Epinephrine, extreme stress

Advanced Placement Biology Instructor’s Solution Manual


32 | Osmotic Regulation and Excretion 801

Solution The solution is (D). Epinephrine and norepinephrine are released by the adrenal
medulla and nervous system, respectively. They are the fight-or-flight hormones that
are released when the body is under extreme stress.
26 What are the cause and treatment of Addison’s disease?
A An excess of uric acid in the blood, uric acid-dissolving drugs
B An inability to produce ADH, drink excess water
C An excess of K+ secretion into the urine, take in excess potassium
D An inability to produce aldosterone, take in excess sodium
Solution The solution is (D). Patients who have Addison's disease have a failing adrenal cortex
and are incapable of producing aldosterone. This causes them to lose sodium in their
urine constantly. If the sodium supply is not replenished, this condition could lead
to death.
27 How does atrial natriuretic peptide hormone act to lower blood pressure?
A As a vasoconstrictor
B As a vasodilator
C As an inhibitor
D Like vasopressin
Solution The solution is (B). Atrial natriuretic peptide is produced in the heart atrium and acts
as a vasodilator to decrease blood pressure.
28 Which cells or organs secrete renin?
A Granular cells of the juxtaglomerular apparatus
B The liver
C The nephrons
D Water-reabsorbing cells of the collecting duct
Solution The solution is (A). Renin is secreted by a part of the juxtaglomerular complex and
increases blood pressure by acting on angiotensinogen.

CRITICAL THINKING QUESTIONS


29 An organism is excreting ions in its urine. What is likely the current tonicity of the
organism’s cells, and why?
A Hypotonicity due to too much solute in its body fluids
B Hypertonicity due to less solute in its body fluids
C Hypertonicity due to too much solute in its body fluids
D Hypotonicity due to less solute in its body fluids

Advanced Placement Biology Instructor’s Solution Manual


802 32 | Osmotic Regulation and Excretion

Solution The solution is (C). If an organism is excreting ions in its urine, then its cells are likely
in a hypertonic environment created by too much solute (ions) in its body fluids.
30 A student measures the osmolality of two aqueous solutions, A and B. The student finds
that the osmolality of solution B is much higher than solution A. Based on this result,
how do solutions A and B likely differ in the concentration of solutes in their solution,
and why?
A Solution A likely is the more concentrated solution because osmolality measures the
moles of solute per kilogram of solute.
B Solution B likely is the more concentrated solution because osmolality measures the
moles of solute per kilogram of solvent.
C Solution A likely is the more concentrated solution because osmolality measures the
moles of solute per kilogram of solvent.
D Solution B likely is the more concentrated solution because osmolality measures the
moles of solute per kilogram of solute.
Solution The solution is (B). Solution B likely is the more concentrated solution, containing a
greater proportion of dissolved solute relative to water (the solvent) than solution A.
This is because osmolality measures the moles of solute per kilogram of solvent.
31 Would an organism that is constantly in a hypertonic environment likely be an
osmoregulator or an osmoconformer? Why?
A Osmoconformer, because it would need to prevent water from leaving its body to
remain alive
B Osmoregulator, because it would need to prevent solutes from leaving its body to
remain alive
C Osmoconformer, because it would need to prevent solutes from leaving its body to
remain alive
D Osmoregulator, because it would need to prevent water from leaving its body to
remain alive
Solution The solution is (D). An organism that is constantly in a hypertonic environment
would likely be an osmoregulator because it would need to prevent water from
leaving its body in order to remain alive.
32 Why is excretion important in order to achieve osmotic balance?
A The body accumulates water within itself when excretion does not occur, which can
have dire consequences.
B Excretion regulates the movement of water within the membranes, which ultimately
maintains osmotic balance.
C In the absence of excretion, there is a shift in the concentrations, which disrupts
osmotic balance.

Advanced Placement Biology Instructor’s Solution Manual


32 | Osmotic Regulation and Excretion 803

D The body builds up many chemical compounds that need to be excreted to maintain
homeostasis and osmotic balance.
Solution The solution is (D). The body builds up a variety of chemical compounds that disrupt
homeostasis. The body rids itself of waste through excretion, reabsorbing and
recycling what it can. Excreting waste causes a shift in concentrations, allowing the
body to return to balance.
33 Which statement best describes the structure of a nephron?
A The nephron has three major components: the glomerulus, the renal tubule, and the
associated capillary network originating from the cortical radiate arteries.
B The nephron has three major components: the renal corpuscle, Bowman’s capsule,
and the associated capillary network originating from the cortical radiate arteries.
C The nephron has three major components: the renal corpuscle, the renal tubule, and
the associated capillary network originating from the segmental renal artery
D The nephron has three major components: the renal corpuscle, the renal tubule, and
the associated capillary network originating from the cortical radiate arteries.
Solution The solution is (D). The nephron has three major components: the renal corpuscle,
the renal tubule, and the associated capillary network originating from the cortical
radiate arteries.
34 How does the loop of Henle act as a countercurrent multiplier?
A The descending limb of the loop of Henle is water permeable, so the water flows from
the filtrate to the interstitial fluid. Osmolality in the limb decreases, and it is lower
inside the loop than in the interstitial fluid. As the filtrate enters the ascending limb,
Na+ and Cl− ions exit through ion channels present in the cell membrane. Farther up,
only sodium is passively transported out of the filtrate.
B The descending limb of the loop of Henle is water impermeable, so the water flows
from the filtrate to the interstitial fluid. Osmolality in the limb increases, and it is
higher inside the loop than in the interstitial fluid. As the filtrate enters the ascending
limb, Na+ and Cl− ions exit through ion channels present in the cell membrane. Farther
up, only sodium is passively transported out of the filtrate.
C The descending limb of the loop of Henle is water impermeable, so the water flows
from the filtrate to the interstitial fluid. Osmolality in the limb increases, and it is
higher inside the loop than in the interstitial fluid. As the filtrate enters the ascending
limb, Na+ and Cl− ions exit through ion channels present in the cell membrane. Farther
up, sodium is actively transported out of the filtrate, and chlorine ions follow.
D The descending limb of the loop of Henle is water permeable, so the water flows from
the filtrate to the interstitial fluid. Osmolality in the limb increases, and it is higher
inside the loop than in the interstitial fluid. As the filtrate enters the ascending limb,
Na+ and Cl− ions exit through ion channels present in the cell membrane. Farther up,
sodium is actively transported out of the filtrate, and chlorine ions follow.

Advanced Placement Biology Instructor’s Solution Manual


804 32 | Osmotic Regulation and Excretion

Solution The solution is (D). A countercurrent multiplier is an osmotic gradient in the renal
medulla that is responsible for concentration of urine. The loop of Henle acts as a
countercurrent multiplier that uses energy to create concentration gradients. The
descending limb of the loop of Henle is water permeable. Water flows from the
filtrate to the interstitial fluid, so osmolality inside the limb increases as it descends
into the renal medulla. The osmolality is higher inside the loop than in the
interstitial fluid at the bottom. So, as filtrate enters the ascending limb of the loop
of Henle, sodium and chlorine ions exit through ion channels present in the cell
membrane. Further up, sodium is actively transported out of the filtrate, and
chlorine ions follow.
35 Why might specialized organs have evolved for excretion of wastes?
A Specialized organs have evolved to provide a measure of safety for organisms.
B Specialized organs have evolved to distinguish different types of organisms.
C Specialized organs have evolved for excretion of wastes to conserve metabolic energy.
D Specialized organs have evolved for excretion of wastes so that organisms can survive
in adverse conditions.
Solution The solution is (A). The removal of wastes, which could otherwise be toxic to an
organism, is extremely important for survival. Having organs that specialize in this
process and that operate separately from other organs provides a measure of safety
for the organism.
36 What are two different excretory systems other than the kidneys?
A (1) An excretory mechanism occurs in annelids through the Malpighian tubules.
Metabolic wastes like uric acid freely diffuse into the tubules. Uric acid is excreted as a
thick paste or powder. (2) An excretory mechanism occurs in the flatworm, which
contains two tubules with cells called flame cells. They have cilia that propel waste
matter down the tubules and out of the body.
B (1) An excretory mechanism occurs in arthropods through a pore called the
nephridiopore. These organisms have a system for tubular reabsorption. (2) An
excretory mechanism occurs in annelids through the Malpighian tubules. Metabolic
wastes like uric acid freely diffuse into the tubules. Uric acid is excreted as a thick
paste or powder.
C (1) An excretory mechanism is endocytosis, which occurs when vacuoles merge with
the cell membrane and excrete cellular wastes in the environment. (2) An excretory
mechanism occurs in annelids through a pore called the nephridiopore. These
organisms have a system for tubular reabsorption.
D (1) An excretory mechanism is exocytosis, which occurs when vacuoles merge with the
cell membrane and excrete cellular wastes in the environment. (2) An excretory
mechanism occurs in flatworms, which consists of two tubules containing cells called

Advanced Placement Biology Instructor’s Solution Manual


32 | Osmotic Regulation and Excretion 805

flame cells. They have a cluster of cilia that propel waste matter down the tubules and
out of the body.
Solution The solution is (D). (1) Microorganisms engulf food by endocytosis—the formation of
vacuoles by involution of the cell membrane within the cells. The same vacuoles
interact and exchange metabolites with the intracellular environment. Cellular
wastes are excreted by exocytosis when the vacuoles merge with the cell membrane
and excrete wastes into the environment. (2) Flatworms have an excretory system
that consists of two tubules. The cells in the tubules are called flame cells; they have
a cluster of cilia that propel waste matter down the tubules and out of the body. (3)
Annelids have nephridia, which have a tubule with cilia. Excretion occurs through a
pore called the nephridiopore. Annelids have a system for tubular reabsorption by a
capillary network before excretion. (4) Malpighian tubules are found in some species
of arthropods. They are usually found in pairs, and the number of tubules varies with
the species of insect. Malpighian tubules are convoluted, which increases their
surface area, and they are lined with microvilli for reabsorption and maintenance of
osmotic balance. Metabolic wastes like uric acid freely diffuse into the tubules.
Potassium ion pumps line the tubules, which actively transport out K + ions, and
water follows to form urine. Water and electrolytes are reabsorbed when these
organisms are faced with low-water environments, and uric acid is excreted as
a thick paste or powder. By not dissolving wastes in water, these organisms
conserve water.
37 How do contractile vacuoles work as excretory systems in microorganisms?
A Contractile vacuoles excrete excess water and waste by the process of endocytosis, in
which these vacuoles merge with the cell membrane and expel wastes into the
environment.
B Contractile vacuoles excrete uric acid by the process of exocytosis, in which water as
well as uric acid is excreted by contraction of a cell when the vacuole merges with the
cell membrane.
C Contractile vacuoles excrete excess water and uric acid by the process of endocytosis
when the vacuole merges with the cell membrane.
D Contractile vacuoles excrete excess water and waste by the process of exocytosis, in
which the vacuoles merge with the cell membrane and expel wastes into the
environment.
Solution The solution is (D). Organisms such as the unicellular eukaryotic amoeba use
contractile vacuoles to expel waste products from their systems. Through a process
known as endocytosis, the microorganism engulfs its food, forming a vesicle. The
vesicle interacts with a lysosome that digests the food. Excess water and waste are
then excreted by exocytosis, in which the contractile vacuoles merge with the cell
membrane and expel wastes into the environment.
38 What is the urea cycle?

Advanced Placement Biology Instructor’s Solution Manual


806 32 | Osmotic Regulation and Excretion

A The urea cycle is the mechanism of conversion of urea to ammonia involving five
intermediate steps catalyzed by five different enzymes. Of the five steps, the first two
occur in the mitochondria and the last three in the cytosol.
B The urea cycle is the mechanism of conversion of ammonia to urea involving five
intermediate steps catalyzed by five different enzymes. Of the five steps, the first two
occur in the mitochondria and the last three in the cytosol.
C The urea cycle is the mechanism of conversion of ammonia to urea involving five
intermediate steps catalyzed by five different enzymes. Of the five steps, the first two
occur in the cytosol and the last three in the mitochondria.
D The urea cycle is the mechanism of conversion of ammonia to urea involving five
intermediate steps all catalyzed by one enzyme. Of the five steps, the first two occur
in the mitochondria and the last three in the cytosol.
Solution The solution is (B). The urea cycle has five intermediate steps that are catalyzed by
five different enzymes in order to convert ammonia to urea. The overall chemical
reaction by which ammonia is converted to urea is

The first two reactions occur in the mitochondria and the last three reactions occur
in the cytosol.
39 How are the formation of urea and uric acid similar and different?
A In birds, reptiles, and insects, the urea cycle converts ammonia to urea. In mammals,
the uric acid cycle converts ammonia to uric acid. Formation of urea from ammonia
requires less energy and is less complex than uric acid formation.
B In mammals, the urea cycle converts ammonia to urea. In birds, reptiles, and insects,
the uric acid cycle converts ammonia to uric acid. Formation of urea from ammonia
requires more energy and is less complex than uric acid formation.
C In mammals, the urea cycle converts ammonia to urea. In birds, reptiles, and insects,
the uric acid cycle converts ammonia to uric acid. Formation of urea from ammonia
requires less energy and is more complex than uric acid formation.
D In mammals, the urea cycle converts ammonia to urea. In birds, reptiles, and insects,
the uric acid cycle converts ammonia to uric acid. Formation of urea from ammonia
requires less energy and is less complex than uric acid formation.
Solution The solution is (D). The urea cycle is the primary mechanism by which mammals
convert ammonia to urea. Urea is made in the liver and excreted in urine. The urea
cycle utilizes five intermediate steps, catalyzed by five different enzymes, to convert
ammonia to urea. Birds, reptiles, and insects, on the other hand, convert toxic
ammonia to uric acid instead of urea. Conversion of ammonia to uric acid requires
more energy and is much more complex than conversion of ammonia to urea.
40 In terms of evolution, why might the urea cycle have evolved in organisms?

Advanced Placement Biology Instructor’s Solution Manual


32 | Osmotic Regulation and Excretion 807

A So organisms could adapt to the changing environment when terrestrial life forms
evolved
B So organisms could evolve the ability to switch between direct ammonia excretion
and urea
C So organisms could reduce their excretion of ammonia in the form of urea
D So organisms could adapt to the changing environment and excrete higher
concentrations of uric acid
Solution The solution is (A). It is believed that the urea cycle evolved to adapt to a changing
environment when terrestrial life forms evolved. Arid conditions probably led to the
evolution of the uric acid pathway as a means of conserving water.

Advanced Placement Biology Instructor’s Solution Manual


808 32 | Osmotic Regulation and Excretion

41 How do hormones regulate blood pressure, blood volume, and kidney function?
A Different regions of the liver have specialized cells that respond to chemical
messengers and hormones like epinephrine, renin, aldosterone, ADH, and ANP. These
regulate the needs of the body and communication between different organ systems.
B Different regions of the nephrons have specialized cells that respond to chemical
messengers and hormones like epinephrine, renin, aldosterone, ADH, and ANP. These
regulate the rate of respiration and communication between the different organ
systems.
C Different regions of the kidneys have specialized cells that respond to chemical
messengers and hormones like epinephrine, renin, aldosterone, ADH, and ANP. These
regulate the rate of respiration and communication between the different organ
systems.
D Different regions of the nephrons have specialized cells that respond to chemical
messengers and hormones like epinephrine, renin, aldosterone, ADH, and ANP. These
regulate the needs of the body and communication between the different organ
systems.
Solution The solution is (D). Hormones are small molecules that act as messengers within the
body. Different regions of the nephron bear specialized cells, which have receptors to
respond to chemical messengers and hormones. The hormones carry messages to the
kidney. These hormonal cues help the kidneys synchronize the osmotic needs of the
body. Hormones like epinephrine, norepinephrine, renin-angiotensin, aldosterone,
anti-diuretic hormone, and atrial natriuretic peptide help regulate the needs of the
body as well as the communication between the different organ systems.
42 How does the renin-angiotensin-aldosterone mechanism function?
A Renin, which is secreted by part of the juxtaglomerular complex, acts on angiotensin
to form angiotensin I, which is then converted to angiotensin II by ACE. Angiotensin II
then stimulates the release of aldosterone and ADH. Angiotensin II acts to destabilize
blood pressure and volume.
B Renin, which is secreted by part of the juxtaglomerular complex, acts on angiotensin
to form angiotensin II, which is then converted to angiotensin I by ACE. Angiotensin II
then stimulates the release of aldosterone and ADH. Angiotensin II acts to stabilize
blood pressure and volume.
C Renin, which is secreted by part of the juxtaglomerular complex, acts on angiotensin
to form angiotensin I, which is then converted to angiotensin II and ADH by ACE. ADH
then stimulates the release of aldosterone. Angiotensin II acts to stabilize blood
pressure and volume.
D Renin, which is secreted by part of the juxtaglomerular complex, acts on angiotensin
to form angiotensin I, which is then converted to angiotensin II by ACE. Angiotensin II
then stimulates the release of aldosterone and ADH. Angiotensin II acts to stabilize
blood pressure and volume.

Advanced Placement Biology Instructor’s Solution Manual


32 | Osmotic Regulation and Excretion 809

Solution The solution is (D). The renin-angiotensin-aldosterone system goes through several
steps to produce angiotensin II, which acts to stabilize blood pressure and volume.
Thus, the kidneys control blood pressure and volume directly. Renin acts on
angiotensinogen, which is made in the liver and converts it to angiotensin I. ACE
(angiotensin converting enzyme) converts angiotensin I to angiotensin II.
Angiotensin II raises blood pressure by constricting blood vessels. It triggers the
release of aldosterone from the adrenal cortex, which in turn stimulates the renal
tubules to reabsorb more sodium. Angiotensin II also triggers the release of anti-
diuretic hormone from the hypothalamus, which leads to water retention. It acts
directly on the nephrons and decreases GFR.
43 What is the fight-or-flight response, and what is its effect on the excretory system?
A Aldosterone is the fight or flight that is released by the adrenal medulla under extreme
stress. This hormone constricts the smooth muscles of the blood vessels. It constricts
the afferent arterioles, causing the flow of blood into the nephrons to stop.
B Epinephrine and norepinephrine are the fight-or-flight hormones that are released by
the adrenal medulla and the nervous system, respectively, under extreme stress.
These hormones constrict the smooth muscles of the blood vessels. They constrict the
afferent arterioles, causing the flow of blood into the nephrons to stop.
C ADH is the fight-or-flight hormone that is released by the adrenal medulla under
extreme stress. This hormone constricts the smooth muscles of the blood vessels. It
constricts the efferent arterioles, causing the flow of blood into the nephrons to stop.
D Epinephrine and norepinephrine are the fight-or-flight hormones that are released by
the adrenal medulla and the nervous system, respectively, under extreme stress.
These hormones constrict the smooth muscles of the blood vessels. They constrict the
efferent arterioles, causing the flow of blood into the nephrons to stop.
Solution The solution is (B). Epinephrine and norepinephrine are released by the adrenal
medulla and nervous system, respectively. These hormones are the fight-or-flight
hormones that are released when the body experiences extreme stress. During this
extreme stress, much of the body’s energy is used to combat imminent danger.
Kidney function is halted temporarily by epinephrine and norepinephrine. These
hormones function by acting directly on the smooth muscles of the blood vessels to
constrict them. Once the afferent arterioles are constricted, blood flow into the
nephrons stops.

Advanced Placement Biology Instructor’s Solution Manual


810 32 | Osmotic Regulation and Excretion

TEST PREP FOR AP® COURSES


44 Patients with kidney illnesses use dialysis machines to remove harmful urea from their
blood. The blood is separated from a solution, called the dialysate, which is designed to
remove wastes by diffusion through a semipermeable membrane, as shown in the
diagram.

How does the concentration of solutes likely differ between the upper component of the
dialyzer and the lower compartment, containing the fresh dialysate, for the dialysis to
successfully remove wastes from the blood?
A In the upper component, the dialysate has a higher solute concentration than the
blood, which allows the urea to diffuse to the lower dialysate down its concentration
gradient.
B In the upper component, the dialysate has a lower solute concentration than the
blood, which allows the urea to be separated via active transport down the
concentration gradient.
C In the upper component, the dialysate has a higher solute concentration than the
blood, which allows the urea to utilize facilitated diffusion in order to diffuse to the
lower dialysate.
D In the upper component, the dialysate has a lower solute concentration than the
blood, which allows the urea to diffuse to the lower dialysate down its concentration
gradient.

Advanced Placement Biology Instructor’s Solution Manual


32 | Osmotic Regulation and Excretion 811

Solution The solution is (D). To remove urea from the blood in the upper component, the
dialysate likely has a lower solute concentration than the blood. This will allow the
urea, a solute of the blood, to diffuse into the dialysate down its concentration
gradient.
45 The diagram shows red blood cells in two different NaCl solutions.

What is likely causing the cells to differ in shape in the two solutions?
A Solution A has high osmolarity. Solution B has low osmolarity.
B Solution A has low osmolarity. Solution B has high osmolarity.
C The cells in solution A are osmoregulators. The cells in solution B are osmoconformers.
D The cells in solution A are osmoconformers. The cells in solution B are osmoregulators.
Solution The solution is (A). The shrinking of cells in solution A is because they are kept in a
hypertonic solution, which causes water to flow out of them. The swelling of cells in
solution B is because they are kept in a hypotonic solution, which causes water to
rush into them.

Advanced Placement Biology Instructor’s Solution Manual


812 32 | Osmotic Regulation and Excretion

46 This diagram models the osmotic pressures experienced by a fish.

Based on the direction of water and solute movements shown in the diagram, is this fish
likely a saltwater or freshwater fish? How do you know?
A Freshwater, because the fish is osmoregulating in response to a hypertonic solution
B Freshwater, because the fish is osmoregulating in response to a hypotonic solution
C Saltwater, because the fish is osmoregulating in response to a hypertonic solution
D Saltwater, because the fish is osmoregulating in response to a hypotonic solution
Solution The solution is (B). In order to maintain needed amounts of ions in the body, a
freshwater fish constantly removes water from its body in the form of dilute urine.
The water outside has less ions than the body of the fish, making the solution
hypotonic.
47 The diagram models the osmotic pressures experienced by a fish.

Why would this fish most likely drink little water but excrete dilute urine? Explain your
answer in terms of the osmolarity of the water it lives in.
A The high osmolarity of the water would cause accumulation of too many salts in the
body of the fish.
B The water it lives in has very low osmolarity, which causes water to constantly diffuse
into the fish’s body.

Advanced Placement Biology Instructor’s Solution Manual


32 | Osmotic Regulation and Excretion 813

C The water it lives in has very high osmolarity, which causes water to constantly diffuse
out of the fish’s body.
D The water it lives in has very high osmolarity, which causes water to constantly diffuse
into the fish’s body.
Solution The solution is (B). This fish drinks little water and excretes dilute urine because the
water it lives in has a very low osmolarity. This causes water to constantly diffuse
into the fish’s body, which the fish regulates by drinking little and constantly diluting
its urine in excess water.
48 Patients with kidney illnesses use dialysis machines to remove harmful urea from their
blood. The blood is separated from a solution, called the dialysate, which is designed to
remove wastes by diffusion through a semipermeable membrane, as shown in the
diagram.

The semipermeable membrane is likely permeable to _____ and impermeable to _____.


blank blank

A red blood cells; urea


B dialysate; blood plasma
C blood plasma; urea
D urea; red blood cells
Solution The solution is (C). The membrane has to be permeable to blood plasma, and the
urea waste it contains, to remove these wastes from the blood.

Advanced Placement Biology Instructor’s Solution Manual


814 32 | Osmotic Regulation and Excretion

49 The diagram models the countercurrent exchange mechanism within the loop of Henle.
The numbers within the loop show the osmolarity of the filtrate, while the numbers
between the two loops indicate the osmolarity of the interstitial fluid within the kidney
tissue.

What would likely occur to the osmolarity of the filtrate in the lower ascending limb in the
short term if the active transport of NaCl stopped?
A Filtrate osmolarity would increase and then decrease.
B Filtrate osmolarity would stay the same.
C Filtrate osmolarity would decrease.
D Filtrate osmolarity would increase.
Solution The solution is (B). The ascending limb is permeable to NaCl, so the filtrate
osmolarity would stay the same.
50 The diagram models the countercurrent exchange mechanism within the loop of Henle.
The numbers within the loop show the osmolarity of the filtrate, while the numbers
between the two loops indicate the osmolarity of the interstitial fluid within the kidney
tissue.

Advanced Placement Biology Instructor’s Solution Manual


32 | Osmotic Regulation and Excretion 815

What would happen to the osmolarity of the interstitial fluid if water could NOT exit the
descending limb?
A Osmolarity of the interstitial fluid would increase.
B Osmolarity of the interstitial fluid would decrease.
C There would be no change in the osmolarity.
D Osmolarity would increase or decrease depending upon the amount of water.
Solution The solution is (B). If water could not exit the descending limb, the osmolarity of the
interstitial fluid would increase. This would reverse the decreasing trend in
osmolarity, preventing the descending limb from functioning.
51 The diagram shows a cross-section of a kidney.

Advanced Placement Biology Instructor’s Solution Manual


816 32 | Osmotic Regulation and Excretion

What would likely occur if there was a blood clot in the renal artery?
A Filtration in the glomerulus would decrease.
B Fluid levels in the renal pelvis would increase.
C Blood would not drain into the convoluted tubule.
D Urea production would increase.
Solution The solution is (A). The renal artery enters the kidney to supply blood. A clot in this
artery would decrease the filtration in the glomerulus.
52 The diagram shows the left kidney.

Why do the capillaries carrying blood from the renal artery run over the top of the renal
pyramids?
A The capillaries deliver blood to the glomerulus and run parallel to the proximal
convoluted tubule. Both are located in the medulla.
B The capillaries deliver blood to the glomerulus and run perpendicular to the proximal
convoluted tubule. Both are located in the cortex.
C The capillaries deliver blood to the glomerulus and run perpendicular to the distal
convoluted tubule. Both are located in the cortex.
D The capillaries deliver blood to the glomerulus and run parallel to the distal
convoluted tubule. Both are located in the cortex.
Solution The solution is (C). The capillaries carrying blood from the renal artery run over the
top of the renal pyramid because they deliver blood to the glomerulus, which is
located in the cortex above each pyramid, as well as run parallel to the distal
convoluted tubule, which is also located in the cortex.

Advanced Placement Biology Instructor’s Solution Manual


32 | Osmotic Regulation and Excretion 817

53 The figure shows the components of a nephron located within the kidneys.

What would likely occur in the collecting duct if there was increased blood flow to the
glomerulus?
A More water would enter the collecting duct.
B More urea would enter the collecting duct.
C Less NaCl would leave the collecting duct.
D Less urea would leave the collecting duct.
Solution The solution is (A). If blood flow to the glomerulus is increased, it will force water
out of the glomerular capillaries, and hence more water will ultimately enter the
collecting duct.

Advanced Placement Biology Instructor’s Solution Manual


818 32 | Osmotic Regulation and Excretion

54 The figure shows the components of a nephron located within the kidneys.

Alcohol impairs the pituitary gland, which controls how much water is reabsorbed by the
nephrons. The hormone produced by the pituitary gland, anti-diuretic hormone, increases
water reabsorption by the kidney. How would impairment of this hormone likely affect
the various components of the nephron pictured?
A Absorption of water from the filtrate would decrease, indicated by decreased loss of
water in the descending loop of Henle, increased solute secretion into the distal
tubule, and decreased water absorption in the collecting duct.
B Absorption of water from the filtrate would decrease, indicated by decreased loss of
water in the ascending loop of Henle, increased solute secretion into the distal tubule,
and increased water absorption in the collecting duct.
C Absorption of water from the filtrate would decrease, indicated by decreased loss of
water in the ascending loop of Henle, increased solute secretion into the distal tubule,
and decreased water absorption in the collecting duct.
D Absorption of water from the filtrate would decrease, indicated by decreased loss of
water in the descending loop of Henle, increased solute secretion into the distal
tubule, and increased water absorption in the collecting duct.
Solution The solution is (C). Impairment of this hormone would decrease the absorption of
water from the filtrate. This would be indicated by decreased loss of water in the
ascending loop of Henle, increased solute secretion into the distal tubule, and
decreased water absorption in the collecting duct.

Advanced Placement Biology Instructor’s Solution Manual


32 | Osmotic Regulation and Excretion 819

55 The diagram models the countercurrent exchange mechanism within the loop of Henle.
The numbers within the loop show the osmolarity of the filtrate, while the numbers
between the two loops indicate the osmolarity of the interstitial fluid within the kidney
tissue.

What would likely happen to the osmolarity of the filtrate in the ascending limb if the
body released urea into the interstitial fluid?
A The osmolarity would decrease, allowing the interstitial fluid to reabsorb solutes.
B The osmolarity would decrease, allowing the interstitial fluid to reabsorb water.
C The osmolarity would increase, allowing the interstitial fluid to reabsorb solutes.
D The osmolarity would increase, allowing the interstitial fluid to reabsorb water.
Solution The solution is (D). Water follows in order to maintain osmotic pressure behind
sodium, which is actively transported out from the ascending limb. No such
following will occur if urea is released into the interstitial fluid, thereby increasing
the osmolarity.
56 Planaria are flatworms that live in freshwater. Their excretory system, or protonephridia,
consists of two tubules connected to a highly branched tube system. The intake end of the
tubes contains cilia that propel waste matter down the tubules and out of the body
through excretory pores that open on the body surface. Cilia also draw water from the
interstitial fluid, allowing for filtration. Any valuable metabolites are recovered by
reabsorption.
What structure in the human kidneys most closely resembles the cilia of the
protonephridia, and why?

Advanced Placement Biology Instructor’s Solution Manual


820 32 | Osmotic Regulation and Excretion

A The renal artery, because it facilitates the exchange of nutrients with the blood
B The convoluted tubule, because it facilitates the exchange of nutrients with the blood
C The glomerulus, because it facilitates filtering of the blood
D The ureter, because it facilitates filtering of the blood
Solution The solution is (C). The glomerulus in the human kidneys and the cilia of the
protonephridia in flatworms facilitate filtration of blood.
57 Planaria are flatworms that live in freshwater. Their excretory system, or protonephridia,
consists of two tubules connected to a highly branched tube system. The intake end of the
tubes contains cilia that propel waste matter down the tubules and out of the body
through excretory pores that open on the body surface. Cilia also draw water from the
interstitial fluid, allowing for filtration. Any valuable metabolites are recovered by
reabsorption.
What structure in the human kidneys most closely resembles the highly branched tube
system of the protonephridia, and why?

Advanced Placement Biology Instructor’s Solution Manual


32 | Osmotic Regulation and Excretion 821

E The renal artery, because it facilitates the exchange of nutrients with the blood
F The convoluted tubule, because it facilitates the exchange of nutrients with the blood
G The glomerulus, because it facilitates filtering of the blood
H The ureter, because it facilitates filtering of the blood
Solution The solution is (B). Nutrients are exchanged with blood by the highly branched tube
system of the protonephridia in flatworms just as by the convoluted tubule in
human kidneys.
58 Planaria are flatworms that live in freshwater. Their excretory system, or protonephridia,
consists of two tubules connected to a highly branched tube system. The intake end of the
tubes contains cilia that propel waste matter down the tubules and out of the body
through excretory pores that open on the body surface. Cilia also draw water from the
interstitial fluid, allowing for filtration. Any valuable metabolites are recovered by
reabsorption.
What structure in the human kidneys most closely resembles the excretory pores of the
protonephridia, and why?
A The urethral opening, because this is where wastes leave the body
B The convoluted tubule, because this is where reabsorption and secretion occur
C The glomerulus, because this is where reabsorption and secretion occur
D The ureter, because this is where wastes leave the body
Solution The solution is (A). The excretory pores resemble the urethral opening through
which excretory wastes are thrown out of the body.
59 The Malpighian tubules filter waste materials out of the hemolymph, of insects. There are
cells lining the tubules that pump solutes (mainly ions) into the space surrounding the
Malpighian tubules. If you observed a gradual increase in the solute concentration outside
of the Malpighian tubules, what would you expect to happen?
A Water would be drawn out of the hemolymph within the tubule.
B Water would be drawn into the tubule.
C Ions would be drawn out of the hemolymph within the tubule.
D Ions would be drawn into the tubule.
Solution The solution is (A). With the increase in the solute concentration outside of the
Malpighian tubules, water would move out of the hemolymph present in the tubule.

Advanced Placement Biology Instructor’s Solution Manual


822 32 | Osmotic Regulation and Excretion

60 The flame cells of a protonephridia filter waste materials out of the blood, or hemolymph,
of invertebrates. What would this be most similar to, in function, in the human excretory
system?
A The ascending loop of Henle
B The descending loop of Henle
C The distal convoluted tubule
D The Bowman's capsule
Solution The solution is (D). Just like the flame cells of flatworms, the Bowman’s capsule
functions to filter solutes from blood to form urine.
61 Terrestrial arthropods, birds, and reptiles convert toxic ammonia to uric acid or the closely
related compound guanine (guano). However, the conversion of ammonia to uric acid
requires more energy and is much more complex than the conversion of ammonia to
urea, or the excretion of ammonia as performed by fish.
Based on these findings, how may the excretory system of one of the terrestrial organisms
listed above change if it evolved to spend most of its time in water?
A They may evolve the ability to switch between uric acid and direct ammonia excretion.
B They would further reduce their excretion of ammonia.
C They may evolve the ability to excrete uric acid without having to dissolve it in any
water.
D They would excrete higher concentrations of uric acid.
Solution The solution is (A). Terrestrial arthropods, birds, and reptiles convert ammonia to
uric acid or guano, but they can directly excrete ammonia if they are evolved to
spend most of their time in water.
62 Birds and reptiles convert toxic ammonia to uric acid or the closely related compound
guanine (guano), reflecting the close evolutionary ancestry of these groups. However,
terrestrial arthropods also convert ammonia to uric acid. This is as opposed to fish, which
excrete ammonia directly, without converting it to another substance. However, the
conversion of ammonia to uric acid requires more energy and is much more complex than
the conversion of ammonia to urea.
What do these findings suggest about why these organisms evolved the conversion of
ammonia to uric acid?
A To evolve the ability to switch between uric acid and ammonia excretion
B To conserve water to allow them to persist on land
C For reduction in excretion of ammonia
D For excretion of higher concentrations of ammonia

Advanced Placement Biology Instructor’s Solution Manual


32 | Osmotic Regulation and Excretion 823

Solution The solution is (B). These organisms are all terrestrial and likely evolved the
conversion of ammonia to uric acid to conserve water, allowing them to persist on
land. The conversion of ammonia is simpler, but it also consumes more water.
63 The kidneys are controlled by hormones from the brain, liver, and other locations.
However, the kidneys also produce the hormone renin in their juxtaglomerular complex.
How would damage to the juxtaglomerular complex affect the renin-angiotensin-
aldosterone system?
A Aldosterone will not be produced, decreasing blood volume.
B Angiotensin I will not be produced, decreasing blood pressure.
C Angiotensin-converting enzyme will not be produced, increasing sodium reabsorption.
D Angiotensin II will not be produced, increasing the glomerular filtration rate.
Solution The solution is (D). Damage to the juxtaglomerular complex affects the production of
angiotensin II, thereby resulting in the increase of glomerular filtration rate.
64 The atrial natriuretic peptide (ANP) lowers blood pressure by acting as a vasodilator. It is
released by cells in the atrium of the heart in response to high blood pressure and in
patients with sleep apnea. ANP prevents sodium reabsorption by the renal tubules.
Therefore, what excretory system symptom might someone with sleep apnea also
experience and why?
A Reduction in urination due to reduction of water reabsorption in the kidneys
B Excessive sodium reabsorption by renal tubes due to increase in water reabsorption in
the kidneys
C Excessive sodium reabsorption by renal tubes due to reduction of water reabsorption
in the kidneys
D Excessive urination due to reduction of water reabsorption in the kidneys
Solution The solution is (D). A person with sleep apnea may also experience excessive
urination because, by preventing sodium reabsorption by the renal tubules, ANP
reduces water reabsorption in the kidney and acts as a diuretic.

Advanced Placement Biology Instructor’s Solution Manual


824 32 | Osmotic Regulation and Excretion

65 This diagram was made by a student to illustrate the angiotensin-aldosterone system.

What part of this diagram contains an error?


A ADH is not produced in this system.
B The diagram is missing ANP.
C ACE and renin should be switched.
D ACE and angiotensin should be switched.
Solution The solution is (D). ACE converts angiotensin I to angiotensin II.
66 This diagram was made by a student to illustrate the angiotensin-aldosterone system.

How would you complete this diagram to make it an accurate model of the renin-
angiotensin system?
A Renin acts on angiotensin to directly stimulate the release of aldosterone and ADH.
B Renin acts on angiotensin to form ACE and angiotensin II, which then stimulates the
release of aldosterone and ADH.

Advanced Placement Biology Instructor’s Solution Manual


32 | Osmotic Regulation and Excretion 825

C Angiotensin II is formed from angiotensin, which is then converted to angiotensin I by


ACE. Aldosterone and ADH are then stimulated to be released from angiotensin I.
D Angiotensin I is formed from angiotensin, which is then converted to angiotensin II by
ACE. Aldosterone and ADH are then stimulated to be released from angiotensin II.
Solution The solution is (D). To complete the diagram, one would continue with angiotensin I,
which is then converted to angiotensin II by ACE. Angiotensin II then stimulates the
release of aldosterone and ADH.

Advanced Placement Biology Instructor’s Solution Manual


826 33 | The Immune System

33 | THE IMMUNE SYSTEM


REVIEW QUESTIONS
1 What is the primary mechanism by which the skin provides protection against diseases
caused by organisms?
A High pH
B Mucus
C Tears
D Desiccation
Solution The solution is (D). The skin environment is low moisture and dries out many
potential pathogens.
2 How does the human body use a chemical barrier as part of the innate immune response?
A Mucous secretions trap and rinse pathogens out of the body.
B Urination carries pathogens out of the urinary tract.
C Low pH conditions in the stomach kill some pathogens and prevent other pathogens
from growing.
D Cilia in the nasal passages and respiratory tract push mucus containing trapped
pathogens out of the body.
Solution The solution is (C). Low pH conditions in the stomach kill some pathogens and
prevent other pathogens from growing.
3 What produces an immediate innate immune response by recognizing an invading
pathogen and engulfing it?
A Macrophage
B Cytokine
C Inflammation
D Antibody
Solution The solution is (A). Macrophages are cells capable of engulfing other cells through
phagocytosis. When a pathogen is recognized, macrophages can then engulf it.
4 How does inflammation develop?
A Inflammation is induced by molecules such as cytokines and histamine that are
produced by various host cells in response to pathogens at the site of injury or
infection.
B During inflammation, all blood cells retreat from the site of infection to protect the
circulatory system from pathogen infection.

Advanced Placement Biology Instructor’s Solution Manual


33 | The Immune System 827

C Inflammation is an ongoing condition in the human body and is part of the way that
the innate immune system can respond immediately to an infection.
D When an infection occurs, an immediate inflammatory response occurs as soon as
pathogens enter the body.
Solution The solution is (A). Inflammation is induced by molecules such as cytokines and
histamine that are produced by various host cells in response to pathogens at the
site of injury or infection.
5 Which innate immune system component uses major histocompatibility class (MHC) I
molecules directly in its defense strategy?
A Macrophages
B Neutrophils
C Natural killer
D Cellular interferon
Solution The solution is (C). Natural killer cells recognize and attack infected cells through the
MHC presented on the cell surface.
6 What is the difference between natural killer cells and macrophages?
A Natural killer cells are not always present in the body and must be induced, whereas
macrophages are constantly present.
B Natural killer cells actually kill foreign cells, whereas macrophages serve only a
signaling function.
C Only macrophages can invade host tissues to fight foreign cells that make their way
into those tissues.
D Natural killer cells kill foreign cells through the processes of lysis and proteolysis,
whereas macrophages kill foreign cells by phagocytosis.
Solution The solution is (D). Natural killer cells kill foreign cells through the processes of lysis
and proteolysis, whereas macrophages kill foreign cells by phagocytosis.
7 What is the composition of major histocompatibility class (MHC) I molecules?
A Lipids
B Nucleic acids
C Carbohydrates
D Proteins
Solution The solution is (D). MHC I molecules are proteins.

Advanced Placement Biology Instructor’s Solution Manual


828 33 | The Immune System

8 What is the function of major histocompatibility class (MHC) I molecules?


A MHC I molecules assist with strengthening the cell membrane.
B MHC I molecules present antigens on the surface of a cell.
C MHC I molecules allow movement of materials across the cell membrane.
D MHC I molecules provide signals for processes involved in cell division.
Solution The solution is (B). MHC I molecules present antigens on the surface of a cell.
9 What is the complement system?
A The complement system contains macrophages that phagocytize foreign pathogens.
B The complement system monitors MHC I molecules on cells and destroys any cell that
displays an antigen belonging to a pathogen.
C The complement system contains a group of about 20 proteins in the blood that attack
pathogens in a cascading fashion to mark and destroy them.
D The complement system is made up of antibodies specific to each pathogen that are
synthesized when a pathogen enters the body.
Solution The solution is (C). The complement system contains a group of about 20 proteins in
the blood that attack pathogens in a cascading fashion to mark and destroy them.
10 What occurs more rapidly as the result of activation of the complement system?
A Pathogen invasion
B Pathogen detection
C Pathogen reproduction
D Pathogen engulfment
Solution The solution is (D). Pathogen engulfment by phagocytes is accelerated through the
complement system.
11 What is another term for adaptive immunity?
A Acquired immunity
B Innate immunity
C Passive immunity
D Humoral immunity
Solution The solution is (A). Immunity developed in response to recognition of a specific
pathogen is acquired or adaptive immunity.

Advanced Placement Biology Instructor’s Solution Manual


33 | The Immune System 829

12 How does adaptive immunity work?


A A person is born having specific immune responses against any pathogen that the
body is exposed to.
B A person is born with the ability to develop a specific immune response against any
pathogen within minutes of being exposed to that pathogen.
C A person does not have any immune response against a pathogen, but develops a
temporary specific response to that pathogen that is then forgotten.
D A person does not have specific immune response against a pathogen, but develops a
specific response to that pathogen that is rapidly recalled later if infection by the same
pathogen occurs again.
Solution The solution is (D). A person does not have a specific immune response against a
pathogen, but develops a specific response to that pathogen that is rapidly recalled
later if infection by the same pathogen occurs again.
13 What component of the innate immune system is NOT part of the adaptive immune
system?
A T cells
B Antibodies
C Mast cells
D B cells
Solution The solution is (C). Mast cells are a type of white blood cell and play a role in the
innate immune system.
14 What is the difference between innate and adaptive immune responses?
A The adaptive immune system is faster-acting than the innate immune system.
B The adaptive immune system produces a longer-lasting defense than the innate
immune system.
C The innate immune system produces a more specific defense than the adaptive
immune system.
D The innate immune system has a more sophisticated memory than the adaptive
immune system.
Solution The solution is (B). The adaptive immune system produces a longer-lasting defense
than the innate immune system.
15 Which cells are unique to the humoral immune response?
A Cytotoxic T cells
B Antigen-presenting cells

Advanced Placement Biology Instructor’s Solution Manual


830 33 | The Immune System

C Helper T cells
D B cells
Solution The solution is (D). B cells are a type of white blood cell unique to the humoral
immune response, where they secrete antibodies.
16 How does the humoral immune response operate?
A The humoral immune response primarily targets infected host cells to destroy them
before the infecting pathogen can reproduce.
B The humoral immune response produces antibodies that are specifically targeted
against each pathogen.
C The humoral immune response produces cytotoxic T cells that induce apoptosis in
pathogen-infected cells.
D The humoral immune response is the only system that involves memory cells that are
able to respond later to a pathogen that has invaded the body at an earlier time.
Solution The solution is (B). The humoral immune response produces antibodies that are
specifically targeted against each pathogen.
17 An antibody sometimes binds to an antigen other than the antigen that elicited its
synthesis. What term refers to this behavior?
A Avidity
B Cross reactivity
C Hypersensitivity
D Affinity
Solution The solution is (B). Cross reactivity is the binding of an antibody to an antigen
different from the one that elicited its synthesis.
18 A biologist runs some tests and observes that the same antibody binds to several different
proteins. Why might this occur?
A The antibody is showing that it has high affinity for antigens.
B The antibody is showing that it has high avidity for antigens.
C The antibody is showing cross-reactivity to the antigens.
D The antibody is showing hypersensitivity to the antigens.
Solution The solution is (C). The antibody is showing cross-reactivity to the antigens.
19 How many and what types of polypeptides make up an antibody molecule?
A One heavy polypeptide chain and one light polypeptide chain
B Two heavy polypeptide chains and one light polypeptide chain
C Two heavy polypeptide chains and two light polypeptide chains

Advanced Placement Biology Instructor’s Solution Manual


33 | The Immune System 831

D One heavy polypeptide chain and two light polypeptide chains


Solution The solution is (C). Two heavy polypeptide chains and two light polypeptide chains
make up an antibody molecule.
20 Why do antibodies isolated from the same individual show a variety of constant domains?
A The antibodies were synthesized in response to infections by different pathogens.
B The antibodies were transmitted as the result of passive immunity.

Advanced Placement Biology Instructor’s Solution Manual


832 33 | The Immune System

C The antibodies have different specificities of antigen binding.


D The antibodies belong to different classes of immunoglobulin molecules.
Solution The solution is (D). The antibodies belong to different classes of immunoglobulin
molecules.
21 What enzyme is responsible for the random excision of variable gene segments making up
an antibody light chain in DNA?
A DNA polymerase
B DNA recombinase
C DNA ligase
D DNA helicase
Solution The solution is (B). DNA recombinase creates variability in the DNA sequence by
randomly excising and splicing segments.
22 How are antibodies produced in a B cell?
A RNA processing → DNA rearrangement → transcription → translation → mature
antibody
B Transcription → RNA processing → DNA rearrangement → translation → mature
antibody
C DNA rearrangement → RNA processing → transcription → translation → mature
antibody
D DNA rearrangement → transcription → RNA processing → translation → mature
antibody
Solution The solution is (B). Antibodies are produced through DNA rearrangement followed
by the standard transcription and translation process.
23 What is a definition of hypersensitivity?
A Maladaptive immune responses to otherwise harmless proteins
B Measure of the attraction between an antigen and an antibody
C Total binding strength of a multivalent antibody with an antigen
D Binding of an antibody to an antigen different from the one that elicited its synthesis
Solution The solution is (A). Hypersensitivity occurs due to immune response to harmless
proteins. The response leads to undesired side effects.

Advanced Placement Biology Instructor’s Solution Manual


33 | The Immune System 833

24 An allergy has been described as a nonprotective immune response. Which statement


provides information supporting this description?
A An allergic response occurs when B cells produce IgE molecules in response to a
foreign protein, which causes histamine to be released.
B An allergy results from an immune response to an antigen that does not cause harm
and that the body needs no defense against.
C People with allergies develop unpleasant symptoms such as watery, itchy eyes,
swollen tissues, and sneezing.
D When their blood is tested, people with allergies show that they have antibodies that
bind to one or more foreign proteins.
Solution The solution is (B). An allergy results from an immune response to a protein that
does not cause harm and that the body needs no defense against. Because the body
is not in danger from the protein, the immune response is not needed and provides
no protection.
25 What is the term for antibodies that inappropriately mark self components as foreign?
A Cross-reactive
B Epitopes
C Allergens
D Autoantibodies
Solution The solution is (D). Autoantibodies mark components generated by the self as
foreign, leading to a nonprotective immune response.
26 Why do some people with type 1 diabetes develop the disease as the result of an immune
response?
A Their B cells produced IgE molecules in response to a harmless protein, which caused
histamine to be released.
B Their B cells responded to a self protein to produce autoantibodies, which elicited
inflammation that caused organ damage.
C They had a delayed response to a harmful protein that entered the body and activated
both the innate and adaptive immune systems.
D They acquired antibodies to insulin through passive immunity, and these antibodies
bind to insulin whenever it is produced in the body.
Solution The solution is (B). Their B cells responded to a self protein to produce
autoantibodies, which elicited inflammation that caused organ damage. Damage to
the pancreas can lead to diabetes.

Advanced Placement Biology Instructor’s Solution Manual


834 33 | The Immune System

27 What are the three types of hypersensitivity?


A Innate, acquired, and immunodeficiency
B Variable, constant, and recombinant
C Immediate, delayed, and autoimmunity
D Active, passive, and adaptive
Solution The solution is (C). Immediate (such as an allergic reaction), delayed (such as
development of rash after contact with jewelry), and autoimmunity (from
recognition of self antigens) are the types of hypersensitivity.
28 What is the definition of autoimmunity?
A Binding of an antibody to a viral antigen
B Immune response to self antigens
C Maladaptive immune response to harmless foreign proteins
D Failure to mount an immune response
Solution The solution is (B). Autoimmunity is the immune response to self antigens, or when
the immune system is triggered by the self.

CRITICAL THINKING QUESTIONS


29 Why does the human body need more than its skin to function as a barrier to infecting
pathogens?
A Skin works only against some types of bacteria. To prevent the entry of other
pathogens, other physical or chemical barriers are needed.
B Skin does not provide a broad coverage against invasion of the body by any foreign
particle, so it is not a very effective barrier.
C Pathogens could enter the body through several places that are not covered by skin
that need to have a barrier to prevent infection.
D Skin acts only as a chemical barrier against pathogens. The body also needs physical
barriers to prevent various types of infection.
Solution The solution is (C). Pathogens could enter the body through several places not
covered by skin because of different functions. Examples are the digestive system
(mouth), respiratory system (nose), urinary system, eyes, and ears. Each of these
places needs to have some type of physical or chemical barrier to prevent infection.
30 Cell surface recognition occurs during many types of immune responses, including
immediate and induced immune responses. How do natural killer cells and interferons
represent one of each type of immune response (immediate and induced), and how does
cell surface recognition play a role?

Advanced Placement Biology Instructor’s Solution Manual


33 | The Immune System 835

A Natural killer cells are an example of induced immune response as they attack host
cells that have lost normal cell surface markers. Interferons are an example of
immediate immune response as they are induced after cell surface markers on
invading pathogens are recognized by host cells.
B Natural killer cells are an example of immediate immune response as they attack host
cells that have lost normal cell surface markers. Interferons are an example of induced
immune response as they are induced after cell surface markers on invading
pathogens are recognized by host cells.
C Natural killer cells are an example of immediate immune response as they are induced
after cell surface markers on invading pathogens are recognized by host cells.
Interferons are an example of induced immune response as they attack host cells that
have lost normal cell surface markers.
D Natural killer cells are an example of induced immune response as they are induced
after cell surface markers on invading pathogens are recognized by host cells.
Interferons are an example of immediate immune response as they attack host cells
that have lost normal cell surface markers.
Solution The solution is (B). Natural killer cells are always active in the body, making them
available for an immediate immune response. They recognize normal cell surface
markers on host cells and attack host cells that have lost these normal markers due
to infection or to changes induced by cancer. Interferons are a type of cytokine, a
chemical signal produced by host cells that have been infected by a pathogen.
Because they are produced only after a cell becomes infected, they are an example
of an induced immune response. They are induced after cell surface markers on
invading pathogens are recognized by host cells such as macrophages, which then
release interferons in response to the invasion.
31 Why might different MHC I molecules between donor and recipient cells lead to rejection
of a transplanted organ or tissue?
A The natural killer cells in the recipient will identify the MHC I molecules on
transplanted organ as non-self proteins, causing lysis of transplanted cells. Other host
cells will join to phagocytize the foreign cells.
B The neutrophils in the recipient will identify the MHC I molecules on transplanted
organ as non-self proteins, causing lysis of transplanted cells. Other host cells will join
to phagocytize the foreign cells.

Advanced Placement Biology Instructor’s Solution Manual


836 33 | The Immune System

C B lymphocytes in the recipient will identify the MHC I molecules on transplanted organ
as non-self proteins. The foreign cells will then be engulfed and destroyed by B
lymphocytes.
D The macrophages in the recipient will identify the MHC I molecules on transplanted
organ as non-self proteins, causing lysis of transplanted cells. Other host cells will join
to phagocytize the foreign cells.
Solution The solution is (A).The natural killer cells in the recipient will identify the MHC I
proteins on the cell surfaces of the transplanted organ as non-self proteins. This will
induce the natural killer cells to attack the transplanted cells, perforate them, and
break them down using proteolytic enzymes. Other host cells will join to phagocytize
the foreign cells.
32 Suppose a person was born without the ability to produce MHC I molecules. What
problem would that create?
A A person without the ability to produce MHC I molecules would die immediately.
B A person without the ability to produce MHC I molecules would recognize self as non-
self, resulting in autoimmune disease.
C The person’s immune system would not be able to distinguish self and non-self. This
would make the person very vulnerable to infection.
D The person’s immune system would not be able to destroy foreign pathogen due to
lack of hydrolytic enzymes. This would make the person very vulnerable to infection.
Solution The solution is (C). The person’s immune system would not have a way to distinguish
self from non-self. Because of this, the immune system might not attack any cells
since it has no signals telling it that an invader is present. This would make the
person very vulnerable to infection since they would be missing a vital part of the
normal immune response.
33 Suppose a series of genetic mutations prevented some, but not all, of the complement
proteins from binding antibodies or pathogens. Would the entire complement system be
compromised? Why or why not?
A No, because the complement system functions as a cascade, with each protein
triggering the activity of the next protein in the cascade
B Yes, because the complement system functions as a cascade, with each protein
triggering the activity of the next protein in the cascade
C Yes, because all the proteins of the complement system function independently
D No, because all the proteins of the complement system function independently
Solution The solution is (B). Yes, because the complement system functions as a cascade, or
chain reaction, with each protein triggering the activity of the next protein in the
cascade. Once you have one protein that malfunctions, the cascade will come to a
halt at that point.

Advanced Placement Biology Instructor’s Solution Manual


33 | The Immune System 837

34 What is a likely reason to explain why vertebrate animals evolved an adaptive immune
system rather than an innate system involving specific responses to specific pathogens?
A An adaptive immune system requires an immense amount of information to be
stored, which allows vertebrate cells to be able to mount specific responses to every
pathogen.
B As new pathogens evolve all the time, it is more conservative of energy and
information storage to have an adaptive immune system that can respond to same
pathogens in different ways.
C As new pathogens evolve all the time, it is more conservative of energy and
information storage to have an adaptive immune system that can respond to different
pathogens in a specific way.
D As new pathogens evolve all the time, it is more conservative of energy and
information storage to have an adaptive immune system that can respond to different
pathogens in a nonspecific way.
Solution The solution is (C). An innate system would require an immense amount of
information stored in vertebrate cells to be able to mount specific responses to
every pathogen. Even then, new pathogens evolve all the time, so the information
would have to have some way of being updated. In the end, it is more conservative
of energy and information storage to have an adaptive immune system that can
respond to different pathogens in a specific way.
35 Invertebrate animals have innate immune systems but lack adaptive immune systems.
Vertebrates, including fish, amphibians, reptiles, birds, and mammals, have both systems.
What does this suggest about the evolution of these two immune systems?
A This suggests that the innate immune system evolved first. Invertebrates and
vertebrates had a common ancestor, which had an innate immune system. After the
two lineages diverged, the vertebrate line developed adaptive immunity, which
continued to evolve in all vertebrates.
B This suggests that the adaptive immune system evolved first. Invertebrates and
vertebrates had a common ancestor, which had an innate immune system. After the
two lineages diverged, the vertebrate line developed adaptive immunity, which
continued to evolve in all vertebrates.
C This suggests that the innate immune system evolved first. Invertebrates and
vertebrates had a common ancestor, which had an adaptive immune system. After the
two lineages diverged, the vertebrate line developed innate immunity, which
continued to evolve in all vertebrates.
D This suggests that the adaptive immune system evolved first. Invertebrates and
vertebrates had a common ancestor, which had an adaptive immune system. After the
two lineages diverged, the vertebrate line developed innate immunity, which
continued to evolve in all vertebrates.

Advanced Placement Biology Instructor’s Solution Manual


838 33 | The Immune System

Solution The solution is (A). This suggests that the innate immune system evolved first and
the adaptive immune system evolved later. Invertebrates and vertebrates had a
common ancestor, which likely had an innate immune system, but not an adaptive
immune system. After the two lineages diverged, the vertebrate line developed
adaptive immunity, which was successful and so continued to evolve in all
vertebrates. The invertebrate lineage did not develop adaptive immunity but
continued to be successful with its innate immune systems.
36 What are naïve B or T cells, and how do they function in cell-mediated and humoral
immune responses?
A Naïve B and T cells are lymphocytes of the B and T types that have come into contact
with pathogenic antigens. Naïve T cells produce antibodies in the humoral immune
response, while naïve B cells stimulate the cell-mediated immune response.
B Naïve B and T cells are lymphocytes of the B and T types that normally circulate in the
body at all times and have not come into contact with any pathogenic antigens.
Activated T cells produce antibodies in the humoral immune response, while activated
B cells stimulate the cell-mediated immune response.
C Naïve B and T cells are lymphocytes of the B and T types that normally circulate in the
body at all times and have not come into contact with any pathogenic antigens.
Activated B cells produce antibodies in the humoral immune response, while activated
T cells stimulate the cell-mediated immune response.
D Naïve B and T cells are lymphocytes of the B and T types that have come into contact
with pathogenic antigens. Naïve B cells produce antibodies in the humoral immune
response, while naïve T cells stimulate the cell- mediated immune response.
Solution The solution is (C). Naïve B and T cells are lymphocytes of the B and T types that
normally circulate in the body at all times, even in the absence of a pathogen
infection. They are considered naïve because they have not come into contact with
any pathogenic antigens that are presented on any antigen-presenting cells (APCs).
Both of these cells become active once they do come into contact with pathogenic
antigens on APCs. Activated B cells go on to produce antibodies in the humoral
immune response, while activated T cells go on to stimulate the cell-mediated
immune response.
37 A person given a flu vaccine in November comes down with a severe case of influenza in
January. What can you conclude about the flu vaccine and cross-reactivity?
A The flu vaccine elicited antibodies in the person’s body that were specific to a
particular flu virus. Unfortunately, the flu virus that infected the person later in
January was different enough for cross-reactivity to occur between the virus antigens
causing the infection.
B The flu vaccine suppressed antibodies in the person’s body that were specific to a
particular flu virus. Unfortunately, the flu virus that infected the person later in

Advanced Placement Biology Instructor’s Solution Manual


33 | The Immune System 839

January was different enough for cross-reactivity to occur between the virus antigens
causing the infection.
C The flu vaccine elicited antibodies in the person’s body that were specific to a
particular flu virus. Unfortunately, the flu virus that infected the person later in
January was similar enough for cross-reactivity to occur between the virus antigens
causing the infection.
D The flu vaccine suppressed antibodies in the person’s body that were specific to a
particular flu virus. Unfortunately, the flu virus that infected the person later in
January was similar enough for cross-reactivity to occur between the virus antigens
causing the infection.
Solution The solution is (A). The flu vaccine elicited antibodies in the person’s body that were
specific to a particular flu virus. Unfortunately, the flu virus that infected the person
later in January was not the same virus that was used to develop the vaccine. In fact,
it was different enough that there was little cross-reactivity between the virus
antigens causing the infection and the antibodies raised in response to the vaccine,
so the person was not protected by the vaccine.
38 What function does the diversity of the variable region of an antibody help it perform?
A It helps in communication of antibodies with other components of immune system.
B It helps the antibodies to function with very low affinity and specificity.
C It enables many different antibodies to be made that all have different specificities of
binding.
D It enables many different antibodies to be made that all have same specificities of
binding.
Solution The solution is (C). The large diversity of structure of the variable region enables
many different antibodies to be made that all have different specificities of binding.
Each antibody binds to a specific antigen, which allows the antibody to function with
very high affinity and specificity to bind and prevent pathogens from continuing their
infection process.
39 How can you explain that the same antibodies found in an infant’s body are also present
in the infant’s mother?
A Antibodies produced in the mother’s body are passed to the infant via passive
immunity through breast milk.
B Antibodies produced in the mother’s body are passed to the infant via active immunity
through breast milk.
C Antibodies produced in the mother’s body are passed to the infant via passive
immunity through experiencing the same environment.
D Antibodies produced in the infant’s body are passed to the mother via active immunity
through the placenta.

Advanced Placement Biology Instructor’s Solution Manual


840 33 | The Immune System

Solution The solution is (A). The antibodies are produced in the mother’s body, with the
mother’s immune system, and passed to the infant via passive immunity through
breast milk.
40 Researchers have been working on developing methods for stimulating the human
immune system to recognize foreign proteins as self proteins. Why would this research be
applicable to treating allergies?
A In an allergy, a person’s immune system has been compromised, which causes
unpleasant symptoms.
B In an allergy, a person’s immune system reacts to its own proteins, which causes
unpleasant symptoms.
C In an allergy, a person’s immune system reacts to a harmless protein from the
environment, which causes unpleasant symptoms.
D In an allergy, a person’s immune system reacts to a harmless protein from the
environment, which causes pleasant symptoms.
Solution The solution is (C). In an allergy, a person’s immune system reacts to a harmless
protein from the environment, which causes unpleasant symptoms. Since the
protein causing the allergy is harmless, there is no need for the person’s immune
system to react this way and cause the person to become uncomfortable. So, if
researchers could find a method for getting a person’s immune system to recognize
a foreign protein as a self protein, then this could be a way to treat an allergy so that
the person’s immune system would no longer react to the harmless protein and stop
the unpleasant side effects.
41 A patient has just been informed that they have an autoimmune disease that attacks
the salivary glands. How would you explain to the patient what is happening inside
their body?
A The immune system is producing antibodies against their own proteins present in their
salivary glands, causing the salivary glands to break down and become nonfunctional.
B The ability of the immune system to fight the antigen present in the salivary glands
might have been compromised, causing the salivary glands to break down and
become nonfunctional.
C The immune system might have reacted in an abnormal way to an antigen that may
have entered the salivary glands, causing the salivary glands to break down and
become nonfunctional.
D Some pathogen might have entered the salivary glands, causing the salivary glands to
break down and become nonfunctional.

Advanced Placement Biology Instructor’s Solution Manual


33 | The Immune System 841

Solution The solution is (A). The person’s immune system is producing antibodies that are
targeting their own proteins present in their own salivary glands. Because of this, the
antibodies are destroying these proteins and causing the salivary glands to break
down and become nonfunctional.
42 An allergic response sometimes leads to a person’s death. How can you explain this?
A The allergen may bind to the hemoglobin, decreasing the affinity of hemoglobin for
oxygen, leading to death.
B If a person has a very strong and fast response to an allergen, the tissues in the throat
can swell so much in a very short time that the person cannot breathe and blood
pressure may increase very quickly. Oxygen will not be carried to the brain and the
person may die.
C If a person has a very strong and fast response to an allergen, the tissues in the throat
can swell so much in a very short time that the person cannot breathe and blood
pressure may drop very quickly. Oxygen will reach the cells at a faster rate and the
person may die.
D If a person has a very strong and fast response to an allergen, the tissues in the throat
can swell so much in a very short time that the person cannot breathe and blood
pressure may drop very quickly. Oxygen will not be carried to the brain and the person
may die.
Solution The solution is (D). If the person with the allergy has a very strong and very fast
response to an allergen, the tissues in the throat can swell so much in a very short
time that the person cannot breathe. Also, the person’s blood pressure may drop
very quickly. With both of these physiological changes, oxygen will not be carried to
the brain, which creates a fatal situation and the person may die.

TEST PREP FOR AP® COURSES


43 A pathogenic bacterium has been engulfed by a phagocytic cell as part of the innate
immune response. Which illustration best represents the response? (source: AP Biology
Course and Exam Description Fall 2012, p. 132)

Advanced Placement Biology Instructor’s Solution Manual


842 33 | The Immune System

A C

B D

Solution The solution is (C). Lysosomes are used to digest the bacterium.
44 How do natural killer cells react to healthy cells compared to cells infected with a
pathogen?
A Natural killer cells recognize MHC I on a healthy cell and do not kill it, while the
infected cells that do not present MHC I are killed.
B Natural killer cells recognize MHC I on an infected cell and kill it, while the healthy cells
that do not present MHC I are not killed.
C Natural killer cells recognize MHC II on a healthy cell and do not kill it, while the
infected cells that do not present MHC II are killed.
D Natural killer cells recognize MHC II on an infected cell and kill it, while the healthy
cells that do not present MHC II are not killed.
Solution The solution is (A). The MHC I presents on the surface of healthy cells and signals to
the natural killer cells.
45 The incomplete diagram represents a series of events during an innate immune response.
The labels A, B, and C need to be replaced with the names of the cells involved.

Which set of cells correctly completes this diagram?

Advanced Placement Biology Instructor’s Solution Manual


33 | The Immune System 843

Advanced Placement Biology Instructor’s Solution Manual


844 33 | The Immune System

A A = infected host cell, B = pathogen, C = healthy host cell


B A = healthy host cell, B = pathogen, C = dendritic cell
C A = dendritic cell, B = infected host cell, C = pathogen
D A = pathogen, B = dendritic cell, C = healthy host cell
Solution The solution is (D). Pathogens produce PAMPS, which stimulate recognition of
the pathogen, including dendritic cell activation, and triggering responses in healthy
host cells.
46 A healthy person produces antibodies to pathogens that invade the body. However, if this
person becomes infected with HIV, the body loses the ability to produce antibodies.
Research has shown that the virus attacks and destroys CD4+T cells.
Why does destruction of CD4+T cells lead to a loss of antibody synthesis in HIV-infected
patients?
A CD4+T cells are a required intermediate in a series of cell-to-cell signaling events that
must be completed before B cells can mature.
B CD4+T cells have CD4 molecules covalently bound to their cell surfaces and do not
induce apoptosis in other cells during an immune response.
C CD4+T cell counts are about 1,000 per microliter in a healthy person, but drop below
400 per microliter in a person who cannot mount an immune response.
D CD4+T cell precursors are formed in the bone marrow and then migrate to the
thymus, where they develop their T cell receptors.
Solution The solution is (A). CD4+ T cells are a required intermediate in a series of cell-to-cell
signaling events that must be completed before B cells can mature. Destruction of
these cells interrupts the immune response.
47 Which diagram best illustrates how a macrophage activates a helper T cell (TH cell)?

Advanced Placement Biology Instructor’s Solution Manual


33 | The Immune System 845

D
Solution The solution is (D). TH cells enhance the activities of macrophages through cytokine
secretion.

Advanced Placement Biology Instructor’s Solution Manual


846 33 | The Immune System

48 The graph shows changes in a person’s blood after they receive a vaccination.

How is cell communication involved in bringing about the changes depicted in the graph?
A The vaccine introduces antigens specific to a pathogen into the person’s blood. These
antigens are moved to the cell surface of antigen-presenting cells present in the blood.
Receptors on helper T cells bind to the antigens present on the antigen-presenting
cell. This direct cell-to-cell contact initiates a series of events that leads to production
of antibodies by B lymphocytes.
B The vaccine introduces antigens specific to a pathogen into the person’s blood. These
antigens bind to the receptors on the surface of T cells. This direct cell-to-cell contact
initiates a series of events that leads to production of antibodies by B lymphocytes.
C The vaccine introduces antigens specific to a pathogen into the person’s blood. These
antigens are moved to the cell surface of antigen-presenting cells present in the blood.
This direct cell-to-cell contact initiates a series of events that leads to production of
antibodies by B lymphocytes.
D The vaccine introduces antigens specific to a pathogen into the person’s blood. These
antigens are moved to the cell surface of antigen-presenting cells present in the blood.
Receptors on helper T cells bind to the antigens present on the antigen-presenting
cell. This direct cell-to-cell contact initiates a series of events that activates the
complement system.
Solution The solution is (A). The vaccine injection introduces antigens specific to a pathogen
into the person’s blood. These antigens are taken up and moved to the cell surface
of antigen-presenting cells present in the blood. Receptors on helper T cells bind to
the antigens present on the antigen-presenting cell. This direct cell-to-cell contact
initiates a series of events that leads to production of antibodies by B lymphocytes.

Advanced Placement Biology Instructor’s Solution Manual


33 | The Immune System 847

49 B cells are important immune cells that fight infections. How is a naïve B cell stimulated to
mature into a plasma cell that secretes antibodies?
A T cells secrete cytokines, which help the B cell to multiply and mature into an
antibody-producing plasma cell.
B Natural killer cells secrete cytokines, which help the B cell to multiply and mature into
an antibody-producing plasma cell.
C T cells secrete interferons, which help the B cell to multiply and mature into an
antibody-producing plasma cell.
D Natural killer cells secrete interferons, which help the B cell to multiply and mature
into an antibody-producing plasma cell.
Solution The solution is (A). B cells are stimulated by T cells through secretion of cytokines.
50 The diagram illustrates a process taking place during an immune response.

What process is represented by this diagram?


A Opsonization
B Apoptosis
C Neutralization
D Complement activation
Solution The solution is (C). Bacterial toxins are recognized by host cell receptors and then
neutralized by antibodies.
51 Scientists performed an experiment using a cell from a mouse embryo and a B cell from
an adult mouse. The mouse embryo cell does not make antibodies, yet its DNA contains
nucleotide sequences encoding antibody polypeptides. The adult mouse B cell makes and
secretes a single type of antibody. In the experiment, radiolabeled DNA probes were
synthesized to be complementary to the DNA encoding the light chains of antibody
produced by the adult mouse B cells. Then, DNA from the mouse embryo cell and from
the adult B cell were isolated and tested to see if either hybridized with the synthesized
radiolabeled DNA probes. The results are shown in the diagram.

Advanced Placement Biology Instructor’s Solution Manual


848 33 | The Immune System

Which claim is best supported by the data?


A The mouse genome contains an enormous number of antibody genes, which accounts
for the huge diversity of antibody molecules that can be made.
B Rearrangement of gene segments encoding antibody polypeptides occurs at the level
of DNA to produce an enormous diversity of antibody molecules.
C The tremendous diversity of antibody molecules that can be made results from post-
translational modifications of antibody polypeptide chains.
D Each antibody is encoded by its own unique gene in the DNA, which explains how
antibodies can have different antigen binding properties.
Solution The solution is (B). Random excision and splicing of DNA segments leads to diversity
of antibody molecules.
52 How does an antibody molecule bind specifically to one antigen but not to others?
A Due to the presence of a specific antigen binding site
B Due to the consistency of the constant region

Advanced Placement Biology Instructor’s Solution Manual


33 | The Immune System 849

C Due to diversity of the constant region


D Due to the complete antibody structure
Solution The solution is (A). The antigen binding site is specific to the antigen, making the
antibody specific to that antigen.
53 The human genome contains less than 50,000 genes, yet a human has the capability of
producing more than 1,012 different antibody molecules. How can this evidence be used
to support the claim that the human body has an immune system that is both effective
and efficient?
A There are many different antibody molecules that can be made, each of which can
specifically target a particular pathogen to destroy it. This specificity makes the
immune system more effective. The immune system is also efficient because each
antibody need to have its own gene.
B There are many different antibody molecules that can be made, each of which can
non-specifically target a particular pathogen to destroy it. This non-specificity makes
the immune system more effective. The immune system is also efficient because each
antibody does not need to have its own gene.
C There are many different antibody molecules that can be made, each of which can
specifically target a particular pathogen to destroy it. This specificity makes the
immune system more efficient. The immune system is also effective because each
antibody does not need to have its own gene.
D There are many different antibody molecules that can be made, each of which can
specifically target a particular pathogen to destroy it. This specificity makes the
immune system more effective. The immune system is also efficient because each
antibody does not need to have its own gene.
Solution The solution is (D). The information supports the claim that the human immune
system is effective because there are so many different antibody molecules that can
be made, each of which can specifically target a particular pathogen to destroy it.
This specificity makes the immune system more effective than if it used the same
molecules in a non-specific way to try to defeat the enormous variety of pathogens
that can infect the body. The immune system is also efficient because each antibody
does not need to have its own gene. This is shown by the fact that the number of
possible antibodies that can be made far exceeds the amount of DNA available in the
human genome. This means that human cells can use recombination of DNA to
create a large variety of peptide segments using a much smaller number of gene
segments.
54 An allergy is caused by the immune system reacting to a foreign protein to produce IgE
molecules that recognize the protein. These IgE molecules become associated with mast
cells that respond to future exposures to the protein by releasing histamines into the
body. The diagram shows this release and also how a drug called an antihistamine can
help an allergy sufferer reduce their allergy symptoms.

Advanced Placement Biology Instructor’s Solution Manual


850 33 | The Immune System

Which statement explains how an antihistamine helps restore homeostasis during an


allergic reaction?
A Antihistamines prevent mast cells from becoming associated with IgE molecules that
recognize the foreign protein allergen.
B Antihistamines prevent mast cells from releasing histamines and causing unpleasant
allergy symptoms.
C Antihistamines prevent histamines that have been released by mast cells from
stimulating the itching and swelling of body tissues.
D Antihistamines prevent mast cells from producing histamines, which halts their effect
on the body.
Solution The solution is (C). Antihistamines prevent histamines that have been released by
mast cells from stimulating the itching and swelling of body tissues. This blocks the
development of the immune response and the development of allergy symptoms.
55 The diagram shows the normal feedback loop that controls the production of thyroid
hormones in the human body.

Advanced Placement Biology Instructor’s Solution Manual


33 | The Immune System 851

Graves’ disease is an autoimmune disease in which the body produces autoantibodies to


the TSH receptor. When bound to the receptor, these autoantibodies mimic the action of
the TSH hormone. How would the feedback loop and the regulated production of thyroid
hormones shown in the diagram be affected in a person with Graves’ disease?
A The feedback loop would be disrupted. Autoantibodies would bind to the TSH
receptors, allowing them to continue to produce thyroid hormones. As a result, there
would be an overproduction of thyroid hormones because the negative feedback
system would be unable to function.
B The feedback loop would be not be disrupted. Autoantibodies would bind to the TSH
receptors, allowing them to continue to produce thyroid hormones. As a result, there
would be an overproduction of thyroid hormones because the negative feedback
system would be unable to function.
C The feedback loop would be disrupted. Autoantibodies would not bind to the TSH
receptors, allowing them to continue to produce thyroid hormones. As a result, there
would be an overproduction of thyroid hormones because the negative feedback
system would be unable to function.
D The feedback loop would be disrupted. Autoantibodies would bind to the TSH
receptors, allowing them to continue to produce thyroid hormones. As a result, there
would be an overproduction of thyroid hormones because the negative feedback
system was functional.
Solution The solution is (A). The feedback loop would be disrupted. Even though the pituitary
would stop producing TSH in response to the negative feedback control of elevated
thyroid hormones, the autoantibodies would still bind to the TSH receptors, giving
them the signal to continue to produce thyroid hormones. As a result, there would
be an overproduction of thyroid hormones because the negative feedback system
would be unable to function.
56 Myasthenia gravis is an autoimmune disease that initially presents with muscle weakness
and can progress to complete impairment of muscle movement. The diagram compares a
healthy individual with an individual suffering from this disease.

Which statement best explains what happens to bring about this disease?

Advanced Placement Biology Instructor’s Solution Manual


852 33 | The Immune System

A The body produces antibodies against nerve cells, which prevent the nerve cells from
releasing acetylcholine during signal transmissions to muscle.
B The body produces antibodies against acetylcholine, which prevent acetylcholine from
transmitting signals from nerves to muscle.
C The body produces antibodies against receptors in muscle, which prevent
acetylcholine from binding and completing nerve signal transmission.
D The body produces antibodies against acetylcholine, which prevent acetylcholine from
breaking down after signal transmission is complete.
Solution The solution is (C). The body produces antibodies against receptors in muscle, which
prevent acetylcholine from binding and completing nerve signal transmission. The
muscle cell is no longer able to receive the acetylcholine signal and this leads to the
symptoms of muscle weakness.

SCIENCE PRACTICE CHALLENGE QUESTIONS


33.2 Adaptive Immune Response
57 Describe the difference between the mammalian cell-mediated and humoral responses to
a pathogen. Include in your description the roles of antigens and antibodies, T-cells, B-
cells, inoculation with vaccines, and the relative response times.
Solution Sample answer: The AP Biology Curriculum Framework identifies the required
content knowledge:
 The mammalian immune system includes two types of specific responses:
cell mediated and humoral.
 In the cell-mediated response, cytotoxic T cells, a type of lymphocytic white
blood cell, “target” intracellular pathogens when antigens are displayed on
the outside of the cells.
 In the humoral response, B cells, a type of lymphocytic white blood cell,
produce antibodies against specific antigens.
 Antigens are recognized by antibodies to the antigen.
 Antibodies are proteins produced by B cells, and each antibody is specific to a
particular antigen.
 A second exposure to an antigen results in a more rapid and enhanced
immune response.
The question also inserts “inoculation with vaccines” as an illustration of how the
narrative in the question can attempt to “prime the pump” with reference to
assumed prior knowledge.

Advanced Placement Biology Instructor’s Solution Manual


33 | The Immune System 853

33.4 Disruptions in the Immune System


58 Immune response must recognize threat by distinguishing wellness from un-wellness.
Recognition of pathogen-generated antigens is an adaptive response. Generic pathogen
identifiers such as bacterial surface polysaccharides or viral capsid proteins are recognized
by cells of the innate immune system. However, cellular debris and dysfunctional whole
cells such as tumor cells or cells marked for apoptosis must also be recognized for
autophagy (“self-devouring”) by the innate immune system.
A. Describe the characteristics of a model of innate immunity that distinguishes wellness
from un-wellness without the specificity of antigen receptors.
Autoimmunity is the breakdown in the immune system’s ability to distinguish self from
non-self. The list of diseases categorized as autoimmune diseases grows, as does the
intensity of research into the causes. One criterion for classification as an autoimmune
disease is the presence of high antibody concentrations in the blood. An auto
inflammatory disease is defined by chronic inflammation, unprovoked by an infection,
when antibody concentrations are not high. Consider the following:
 Macrophages and phagocytes share information with the adaptive immune
system by displaying protein fragments of their targets on their surfaces to
communicate with T cells.
 The onset of autoimmune disease often follows an infection.
 After an infection has passed, tissue must be repaired, and macrophages and
phagocytes remain active in the affected tissue, often leading to chronic
inflammation.
 Type 2 diabetes, traditionally categorized as a metabolic disease, begins as
chronic inflammation and is now classified as an autoimmune disease by many
scientists.
 Autophagy is induced by stresses such as starvation.
B. Evaluate the strength of the question, do cell fragments produced by the innate
immune response during inflammation communicate self as non-self to T cells leading to
autoimmunity? In the “fight or flight” response, neural signals are sent to the adrenal
gland activating the release of epinephrine. Epinephrine increases metabolic activity,
heart rate and alertness. The activation of the endocrine system by the nervous system
has been positively selected by survival in response to threat.
C. Describe a model for the evolution of the activation of immune system by the nervous
system in an environment in which wounds are frequent and potentially lethal. Include
the selection process in your description. In the famous experiment of Pavlov, a reward
(unconditioned stimulus) received synchronously with a bell (conditioned stimulus)
became associated in the mind of the dog. After a few repetitions, the response of the
dog to the bell was to drool. The digestive system had been activated by the nervous
system through a conditioned stimulus. An emerging area of research is

Advanced Placement Biology Instructor’s Solution Manual


854 33 | The Immune System

psychoneuroimmunology, the study of the relationship between the immune system and
the nervous system. Exton and coworkers (Brain Behavioral Research, 110, 2000) have
shown that a reward (an injection of epinephrine) received synchronously with candy
(conditioned stimulus) become associated through repetition. The response of the patient
to a shot of epinephrine is the proliferation of natural killer, NK, cells (a cell type of the
innate immune system). After conditioning, the patient displayed an increase in NK cells in
response to the candy. The drug cyclosporine is used to suppress immune rejection of
organ transplants by inhibiting production of interleukin, a cytokine messenger that
increases expression in T cells.
D. Design an experiment to test the use of conditioning using the administration of
cyclosporine as the unconditioned stimulus and sugar water as the conditioned stimulus
by measuring the blood concentration of interleukin. Assume that interleukin is flushed
from the blood in a single day. In your design include a vehicle control where conditions
are identical to the treatment and a “do-nothing” or naïve control.
Solution Sample answer:
A. The innate immune must be able to discriminate wellness from un-wellness
without the production of antigens that activate receptors on T cells and B cells. So it
must be a memory-less system that has receptors that allow docking of molecular
signatures of pathogens. But it must also somehow recognize cellular and molecular
materials that do not have these signatures. It must be continuously surveilling the
intercellular space. There is much that is not known about how this is achieved, but
the complement system does have these characteristics. The scope required for the
AP Biology Exam is the need for non-specific identification of threat.
B. The response should identify the question as one that can lead to insight in the
cause and effect relationship between events (pathogenic and inflammatory tissue
damage) and the effect (autoimmune disease). The connections should be made
that starvation might over time be equivalent to chronic depression of blood sugar
and that the presentation by T cells of cellular molecular fragments could program
T cell attack on self. Research in this area is very active; rheumatoid arthritis is now
designated as an autoimmune disease and it has (van Loosdregt et al., European
Journal of Immunology, 47, 2016) traditionally been regarded as an
autoinflammatory disease.
C. A rapid immune response following conflict is positively selected by survival.
Systems for cross-talk between the nervous system and the endocrine system
increase the speed of response and so are positively selected.
D. Subjects (let’s suppose that the rat is being used) need to be dosed in known
amounts and at known times. It is necessary to know the interval of time needed for
the drug to suppress interleukin concentrations. It is also necessary to know the
interval of time needed for interleukin to be flushed from the system. Preparation
for a treatment regime could involve food and water twice daily for several days.
Some subjects are then given the drug in their sugar water. A vehicle control is given

Advanced Placement Biology Instructor’s Solution Manual


33 | The Immune System 855

only sugar water. A naïve control continues to be given food and water with no
amendments to the earlier schedule. Treatment continues for several days while
blood is drawn daily after a period of time needed for interleukin suppression to
occur. After a number of days thought to be sufficient to establish the conditioned
and unconditioned association (which would need confirmation through a series of
experiments in which that time period is extended), the treated subject is given only
the sugar water. If interleukin levels do not fall following the administration of the
conditioned stimulus (and after several attempts to reinforce the training through
extended repetitions), the experiment will have demonstrated that conditioning was
unsuccessful.

Advanced Placement Biology Instructor’s Solution Manual


856 34 | Animal Reproduction and Development

34 | ANIMAL REPRODUCTION AND


DEVELOPMENT
REVIEW QUESTIONS
1 Which method of reproduction produces identical offspring and is most successful in a
stable environment?
A Asexual
B Sexual
C Conjugation
D Inbreeding
Solution The solution is (A). Asexually produced offspring are genetically identical and can
adapt equally well to a stable environment.
2 Which method produces genetically unique offspring?
A Parthenogenesis
B Budding
C Fragmentation
D Sexual reproduction
Solution The solution is (D). Sexual reproduction produces individuals who have a
combination of maternal and paternal genetic material.
3 Which statement is false?
A Budding is a method of asexual reproduction.
B Fragmentation is a method of asexual reproduction.
C Parthenogenesis is a type of sexual reproduction that produces diverse offspring.
D Binary fission is a method of asexual reproduction.
Solution The solution is (C). Parthenogenesis is not a method of sexual reproduction.
4 Sea stars are broken apart by workers to save the clams they feed on, and then thrown
back into the ocean. Often, the numbers of sea stars are seen to double after this. Why
does this happen?
A Regeneration
B Fragmentation
C Budding
D Presence of suitable conditions

Advanced Placement Biology Instructor’s Solution Manual


34 | Animal Reproduction and Development 857

Solution The solution is (A). Regeneration of broken sea stars leads to doubling their
numbers.
5 Which form of reproduction might be used by a sexually reproducing animal that has
limited mobility?
A Fragmentation
B Budding
C Hermaphroditism
D Parthenogenesis
Solution The solution is (C). Hermaphrodites, which have both male and female reproductive
parts in the same individual, reproduce sexually, as seen in earthworms and snails.
6 In sexual reproduction in mammals, gametes with either an X or Y chromosome are
formed in males, whereas gametes in females have one of two possible X chromosomes.
What is the nature of both male and female gametes?
A Diploid
B Haploid
C Homozygous
D Heterozygous
Solution The solution is (B). Both male and female gametes are haploid in nature.
7 External fertilization most commonly occurs in which type of environment?
A Aquatic
B Forests
C Savanna
D Steppe
Solution The solution is (A). External fertilization is most likely to occur in aquatic
environments.
8 Why is broadcast spawning considered advantageous?
A Production of many eggs increases the chance of fertilization.
B Production of many sperm increases the chance of fertilization.
C It results in mixing of genes and greater genetic diversity.
D Animals that are motile carry out this process.
Solution The solution is (C). Broadcast spawning results in mixing of genes and greater
genetic diversity.

Advanced Placement Biology Instructor’s Solution Manual


858 34 | Animal Reproduction and Development

9 Which term applies to egg development outside the female with nourishment derived
from a yolk?
A Oviparity
B Viviparity
C Ovoviparity
D Ovoparity
Solution The solution is (A). In oviparous animals, embryos derive nourishment from the yolk
outside the female.
10 How are the offspring of viviparous animals different from the offspring of oviparous
animals?
A They are protected from the external environment.
B They are produced in higher numbers.
C They can live longer.
D They can use nutrients better.
Solution The solution is (A). The growing offspring of viviparous animals are protected from
the external environment.
11 Which structure is involved in mating in birds?
A Cloaca
B Spermatheca
C Uterus
D Coelom
Solution The solution is (A). The cloaca is the opening used by birds for mating.
12 The spermatheca, which is found in many insects, worms, and mollusks, is useful for —
A spermatogenesis
B sperm motility
C the growth of embryos
D storing sperm and as a site for fertilization
Solution The solution is (D). Sperm can be stored in the spermatheca, and it can be a site for
fertilization.
13 Most of the semen is made by the _____ in the male reproductive system?
A scrotum
B seminal vesicles

Advanced Placement Biology Instructor’s Solution Manual


34 | Animal Reproduction and Development 859

C seminiferous tubules
D prostate gland
Solution The solution is (B). Seminal vesicles produce 60 percent of the total semen value.
14 How is an oocyte released from a human ovary so it can enter the oviduct?
A The beating action of the flagellum on the oocyte
B The force of the follicular ejection directing the oocyte into the oviduct
C The wavelike beating of cilia lining the oviduct
D Muscular contraction of the ovaries
Solution The solution is (B). The oocyte moves into the oviduct with the help of fimbriae.
15 Which female organ has the same embryonic origin as the penis?
A Clitoris
B Labia majora
C Greater vestibular glands
D Vagina
Solution The solution is (A). The clitoris and the penis have similar embryonic origins.
16 Which structure is only related to the male urinary system, as opposed to the male
reproductive system?
A Urinary bladder
B Cowper’s gland
C Bulbourethral gland
D Urethra
Solution The solution is (A). The urinary bladder is not involved in the sexual response.
17 How many eggs are produced as a result of one meiotic series of cell divisions?
A One
B Two
C Three
D Four
Solution The solution is (A). One egg is viable, and the polar bodies degenerate.
18 Which event activates the germ cells at puberty to produce spermatozoa?
A Activation by gonadotropin-releasing hormone
B Activation by increased hair growth

Advanced Placement Biology Instructor’s Solution Manual


860 34 | Animal Reproduction and Development

C Increase in blood volume


D Increase of secretion of testosterone
Solution The solution is (A). Gonadotropin-releasing hormone generated at the onset of
puberty stimulates spermatozoa production.
19 What stimulates Leydig cells in the testes to produce testosterone?
A FSH
B LH
C Inhibin
D Estrogen
Solution The solution is (B). LH stimulates Leydig cells to make and release testosterone.
20 In both males and females, the hormones FSH and LH play a critical role in the
reproductive phase. What regulates the secretion of these hormones?
A GnRH
B Estrogen
C Progesterone
D Inhibin
Solution The solution is (A). Gonadotropin-releasing hormone (GnRH) is released by the
hypothalamus, causing the anterior pituitary to release FSH and LH.
21 Which hormone prepares the endometrial lining of the uterus for potential implantation?
A Testosterone
B Estrogen
C GnRH
D Progesterone
Solution The solution is (D). Progesterone, released by the corpus luteum, causes the
endometrium to thicken.
22 With the onset of menopause, the follicles stop responding to which hormone?
A Estradiol and progesterone
B Progesterone and estrogen
C FSH and LH
D FSH and GnRH
Solution The solution is (C). The follicles stop responding to FSH and LH.

Advanced Placement Biology Instructor’s Solution Manual


34 | Animal Reproduction and Development 861

23 After ovulation, increasing amounts of _____ cause the endometrium to start to thicken.
blank

A fluid
B LH
C progesterone
D GnRH
Solution The solution is (C). Progesterone levels continue to increase after ovulation, causing
the endometrium to start to thicken.
24 What characterizes the acrosomal reactions?
A The sperm’s degradation of the outermost layer of the egg to penetrate it
B Motility of the sperm
C Fusion of the egg and sperm
D Breakdown of the envelope covering the sperm head
Solution The solution is (A). In the acrosomal reactions, the sperm degrades the outermost
layer of the egg to penetrate it.
25 What occurs as a result of fertilization of an egg and sperm?
A Fertilization restores diploidy.
B Fertilization always results in a viable embryo.
C Fertilization merges two diploid cells into a haploid cell.
D Fertilization precedes ovulation.
Solution The solution is (A). The fusion of an egg and sperm will restore diploidy.
26 Which statement about early embryonic stages is false?
A The endoderm, mesoderm, and ectoderm are germ layers.
B The trophoblast is one of the germ layers.
C The inner cell mass is a source of embryonic stem cells.
D The blastula is often a hollow ball of cells.
Solution The solution is (B). The trophoblast is the outermost layer of the blastocyst.
27 Gastrulation is the process in which the cells in the blastula rearrange themselves and
form three layers of cells. Each layer will differentiate into a different organ system. At
what point of development do the major organs begin to develop in humans?
A Fertilization
B First trimester

Advanced Placement Biology Instructor’s Solution Manual


862 34 | Animal Reproduction and Development

C Second trimester
D Third trimester
Solution The solution is (B). The heart, liver, and lungs form during the first trimester from
the three layers of the gastrula.
28 What happens to the number of cells during cleavage?
A It increases.
B It decreases.
C It doubles with every cell division.
D It does not change significantly.
Solution The solution is (A). Mitotic divisions lead to an increase in the number of cells.
29 The blastula stage is a mass of specialized cells. What forms the liver cells?
A Inner cell mass
B Trophoblast
C The entire blastula
D All of the blastomeres
Solution The solution is (A). The inner cell mass, also known as the embryoblast, contains
embryonic stem cells that give rise to future liver cells.
30 Which germ layer forms the skin cells?
A Endoderm
B Ectoderm
C Mesoderm
D Trophoblast
Solution The solution is (B). The ectoderm forms the skin cells.
31 What are the three phases of embryonic development in chronological order?
A Blastula → gastrula → cleavage
B Blastula → cleavage → gastrula
C Cleavage → gastrula → blastula
D Cleavage → blastula → gastrula
Solution The solution is (D). The correct sequence for the three phases of embryonic
development is cleavage → blastula → gastrula.

Advanced Placement Biology Instructor’s Solution Manual


34 | Animal Reproduction and Development 863

32 Which option does NOT describe axes of symmetry in animals?


A Anterior-posterior
B Dorsal-ventral
C Upper-lower
D Lateral-median
Solution The solution is (C). Symmetry is defined by terms such as anterior-posterior, dorsal-
ventral, and lateral-median, but not upper-lower, which are very general terms.
33 Which statement best summarizes the factors controlling symmetry?
A Genetics are important in axis formation.
B Body symmetry is independent of genes.
C Body symmetry is determined entirely at the blastula stage.
D Body symmetry is determined as the embryo grows in length.
Solution The solution is (A). The formation of the axis is determined by genes.
34 The neural plate undergoes folding and movement of cells to form which structure?
A Neural tube
B Epidermis
C Mesoderm
D Neural cord
Solution The solution is (A). The neural plate undergoes formation, bending, convergence,
and closure to form the neural tube.
35 What is necessary for normal human fertilization to occur?
A Many eggs must be released.
B The uterus must be enlarged.
C One sperm needs to penetrate one egg.
D Secretion of pituitary FSH and LH must decrease.
Solution The solution is (C). Fusion of sperm and egg is necessary for normal fertilization.
36 Before pregnancy, progesterone is produced by the ovaries to thicken the endometrial
lining and ensure pregnancy. During the third trimester of pregnancy, which organ
produces progesterone?
A Placenta
B Endometrial lining

Advanced Placement Biology Instructor’s Solution Manual


864 34 | Animal Reproduction and Development

C Chorion
D Corpus luteum
Solution The solution is (A). The placenta produces progesterone.
37 Which hormone is primarily responsible for the contractions during labor?
A Oxytocin
B Estrogen

C HCG
D Progesterone
Solution The solution is (A). Oxytocin helps with labor contractions.
38 What happens before the baby and placenta are expelled?
A Thinning and dilation of the cervix take place.
B There is increased blood flow to the baby.
C Enlargement of the uterus takes place.
D Increased production of estrogen occurs.
Solution The solution is (A). Thinning and dilation of the cervix take place before labor.
39 Which type of short-term contraceptive method is generally more effective than others?
A Barrier
B Hormonal implants
C Natural family planning
D Withdrawal
Solution The solution is (B). Hormonal implants under the skin have a failure rate of less than
1 percent.
40 Which option best indicates that a female is ovulating?
A Slight decrease in body temperature
B Decrease in cervical volume
C More pliable cervical secretions
D Change in breast size
Solution The solution is (C). More pliable cervical secretions are a true indicator of when a
woman is ovulating.
41 Fertility is generally unaffected by —
A sexually transmitted diseases
B obstruction of reproductive tubes

Advanced Placement Biology Instructor’s Solution Manual


34 | Animal Reproduction and Development 865

C drug use
D genetic factors
Solution The solution is (D). In studies so far, infertility has not been found to be due to
genetic factors.
42 What occurs in the procedure known as in vitro fertilization (IVF)?
A A sperm is injected into the egg externally.
B An egg is fertilized by the sperm internally in the oviduct.
C Eggs and sperm are combined externally and then implanted immediately.
D Egg and sperm are combined externally and the resulting embryo is implanted.
Solution The solution is (D). Eight-celled embryos are typically implanted in the uterus.

CRITICAL THINKING QUESTIONS


43 Due to global climate change and pollution, a lake experiences changes in temperature
and pH. The lake has many species, including sexually reproducing frogs, water fleas that
multiply by parthenogenesis, hydra that multiply by budding, and sponges that multiply
by fragmentation.
Which species will most likely survive the changing conditions of the lake?
A Hydra
B Sponges
C Water fleas
D Sexually reproducing frogs
Solution The solution is (D). Sexually reproducing frogs will be the most likely to survive
because they have a mix of maternal and paternal genetic material, which may help
them survive these environmental changes. Hydra, water fleas, and sponges are all
asexually reproducing animals and are genetically identical to the parent, so they do
not have the advantage of a mix of genes to survive the changes in the environment.
44 Why is sexual reproduction useful when only half the individuals reproduce and two cells
must combine to form a new cell?
A It completes in a very short period of time.
B It results in the rapid production of many offspring.
C It increases genetic diversity, allowing organisms to survive in an unpredictable
environment.
D It requires less energy and leads to genetic variation in the offspring.

Advanced Placement Biology Instructor’s Solution Manual


866 34 | Animal Reproduction and Development

Solution The solution is (C). Sexually reproducing animals contain genetic material from both
parents, providing genetic diversity. As conditions change, this will help them survive
in a fluctuating or unpredictable environment.
45 Sex determination in humans and other mammals is dictated by the presence of sex
chromosomes. Are there different factors that determine the sex of other types of
animals?
A No, the sex of an individual is only determined by the presence of sex chromosomes.
B Yes, temperature also determines the sex of an individual.
C Yes, humidity and temperature determine the sex of an individual.
D Yes, pH and humidity determine the sex of an individual.
Solution The solution is (B). In some animals, temperature determines sex. Warmer
temperatures favor female development during egg development, whereas cooler
temperatures favor development of males, as seen in crocodiles. Turtles also exhibit
environmental sex determination.
46 What are some advantages of internal compared with external fertilization?
A Internal fertilization leads to more genetic variations and increases the survival rates
of offspring.
B Internal fertilization increases the survival rates of offspring, and large numbers of
offspring are produced.
C Internal fertilization increases the survival rates of offspring, and the chance of
fertilization with a specific partner also increases.
D Internal fertilization increases the survival rates of offspring and decreases the chance
of fertilization with a specific partner.
Solution The solution is (C). Internal fertilization increases the survival rates of offspring
because they are protected from external predators and unfavorable environmental
conditions. Also, the chance of fertilization with a specific partner is more likely.
In many cases, the fewer number of offspring produced also receive better
parental care.
47 What are the mechanisms that protect and nurture the embryo in oviparous animals?
A The hard, leathery exterior of bird eggs and the hard calcium covering of reptile
eggs provides protection to the growing embryo. Nourishment is provided by yolk in
the eggs.
B The hard, leathery exterior of reptile eggs and the hard calcium covering of bird eggs
provide protection to the growing embryo. Nourishment is provided by endosperm in
the eggs.

Advanced Placement Biology Instructor’s Solution Manual


34 | Animal Reproduction and Development 867

C The hard, leathery exterior of reptile eggs and the hard calcium covering of bird
eggs provide protection to the growing embryo. Nourishment is provided by placenta
in the eggs.
D The hard, leathery exterior of reptile eggs and the hard calcium covering of bird
eggs provide protection to the growing embryo. Nourishment is provided by yolk in
the eggs.
Solution The solution is (D). In oviparous animals like reptiles, most amphibians, birds, and
some mammals, the fertilized egg is laid outside the female body until it hatches.
The eggs are rich in yolk and provide nourishment. The hard, leathery exterior of
reptile eggs and the hard calcium covering of bird eggs provide protection to the
growing embryo.
48 Compare the specialization of reproductive structures found in different types of land
animals.
A In birds, an opening called the cloaca is used to transfer sperm, whereas in mammals,
the presence of the penis and vagina allows direct delivery. Complete reproductive
systems are formed in insects, with eggs maturing in the testes and sperm maturing in
the ovaries.
B In birds, an opening called the cloaca is used to transfer sperm, whereas in mammals,
the presence of the penis and vagina allows direct delivery. Complete reproductive
systems are formed in insects, with eggs maturing in the ovaries and sperm maturing
in the testes.
C In birds, sperm is transferred via the spermatheca, whereas in mammals, the presence
of the penis and vagina allows direct delivery. Complete reproductive systems are
formed in insects, with eggs maturing in the ovaries and sperm maturing in the testes.
D In birds, an opening called the cloaca is used to transfer sperm, whereas in mammals,
the presence of the penis and vagina allows direct delivery. Insects always use
parthenogenesis.
Solution The solution is (B). Annelids, such as earthworms, were among the first land animals
to produce sperm and eggs from undifferentiated cells and store them in the
coelom. Gonads that produced and stored gametes were formed later. Complete
reproductive systems were formed in insects, with eggs maturing in the ovaries and
sperm maturing in the testes. In birds, an opening called the cloaca is used to
transfer sperm, whereas in mammals, the presence of the penis and vagina allows
direct delivery.
49 What is the fate of the newly released secondary oocyte after ovulation?
A If it fuses with a sperm, the resulting zygote enters the cervix for implantation. If it is
not fertilized, it will return to the oviduct.
B If it fuses with a sperm, the resulting zygote enters the uterus for implantation. If it is
not fertilized, it will return to the oviduct.

Advanced Placement Biology Instructor’s Solution Manual


868 34 | Animal Reproduction and Development

C If it fuses with a sperm, the resulting zygote enters the uterus for implantation. If it is
not fertilized, it will degrade and exit the body.
D If it fuses with a sperm, the resulting zygote enters the cervix for implantation. If it is
not fertilized, it will degrade and exit the body.
Solution The solution is (C). The secondary oocyte is swept into the oviduct by the action of
fimbriae projecting from the oviduct. The beating action of cilia moves the egg
toward the uterus. If it fuses with a sperm, the secondary oocyte forms an egg and a
polar body. The resulting zygote will make its way into the uterus for implantation. If
it is not fertilized, it will degrade and exit the body.
50 What are the similarities and differences in sexual response in males and females?
A Both males and females show specific arousal, but the sexual response differs in
intensity and duration.
B Both males and females show specific arousal. In males, breathing rate and heart rate
are increased. In females, there is a decrease in breathing rate and heart rate.
C Vasodilation occurs in both males and females, allowing blood to engorge erectile
tissue in the nipples, clitoris, labia, vagina, and penis. In males, breathing rate
and heart rate are increased. In females, there is a decrease in breathing rate and
heart rate.
D Both males and females show an increase in heart rate, breathing rate, and blood
pressure during phase one and phase two. However, sexual response differs in
intensity and duration in males and females. Also, males show specific arousal, while
females show nonspecific arousal.
Solution The solution is (D). There are four phases in the human sexual response. During
phase one (excitement), vasodilation leads to vasocongestion and enlargement of
erectile tissues. Vaginal secretions lubricate the vagina during intercourse. In phase
two (plateau), stimulation continues, the outer vaginal walls enlarge with increased
blood supply, and heart rate and breathing increase. In phase three (orgasm),
accessory glands and tubules constrict, depositing semen in the urethra, which
contracts, expelling the semen through the penis. In women, the uterus and vaginal
muscles contract. In phase four (resolution), the events of the three phases reverse
themselves and return to their normal state.
51 Compare and contrast spermatogenesis and oogenesis.
A Both are the form of gametogenesis that takes place through mitosis.
Spermatogenesis is the process of formation of four sperm in the testes in males. The
process of formation of one ovum in the ovaries in females is called oogenesis.
B Both are the form of gametogenesis that takes place through meiosis.
Spermatogenesis is the process of formation of four sperm in the testes in males. The
process of formation of four ova in the ovaries in females is called oogenesis.

Advanced Placement Biology Instructor’s Solution Manual


34 | Animal Reproduction and Development 869

C Both are the form of gametogenesis that takes place through meiosis.
Spermatogenesis is the process of formation of four sperm in the testes in males. The
process of formation of one ovum in the ovaries in females is called oogenesis.
D Both are the form of gametogenesis that takes place through meiosis.
Spermatogenesis is the process of formation of one sperm in the testes in males, while
the process of formation of one ovum in the ovaries in females is called oogenesis.
Solution The solution is (C). During spermatogenesis, the stem cells (spermatogonia) start the
process of meiosis and produce sperm until old age. Four sperm are produced per
round of meiosis. Egg production begins before birth and is arrested during meiosis
until puberty, and then individual cells continue through each menstrual cycle. One
egg is produced from each meiotic process, with the extra chromosomes and
chromatids going into polar bodies, which degenerate and are reabsorbed by the
body. The production of viable eggs is halted at menopause.
52 How does the hypothalamus regulate the secretion of reproductive hormones in males?
A The hypothalamus releases FSH and LH at puberty by secreting GnRH. FSH stimulates
the Leydig cells in the testes, and LH stimulates the Sertoli cells to synthesize and
secrete testosterone.
B The hypothalamus releases FSH and LH at puberty by the secretion of GnRH. FSH
stimulates the Sertoli cells in the testes, and LH stimulates the Leydig cells to
synthesize and secrete testosterone.
C The hypothalamus stimulates the release of FSH and LH at puberty by secreting GnRH.
FSH stimulates the Sertoli cells in the testes, and LH stimulates the Leydig cells to
synthesize and secrete testosterone.
D The hypothalamus releases TSH and LH at puberty by the secretion of GnRH. TSH
stimulates the Sertoli cells in the testes and LH stimulates the Leydig cells to
synthesize and secrete testosterone.
Solution The solution is (C). The hypothalamus causes the release of FSH and LH at puberty by
the secretion of gonadotrophin-releasing hormone (GnRH). FSH stimulates the
Sertoli cells in the testes and LH stimulates the Leydig cells to synthesize and secrete
testosterone. High testosterone levels result in negative feedback to stop the
release of GnRH, LH, and FSH.
53 What are the events that take place in a nonpregnant woman after ovulation?
A Because a fertilized egg is not implanted into the uterus in a nonpregnant woman, the
corpus luteum degenerates, and the levels of estrogen and progesterone decrease.
The endometrium begins to degenerate as the progesterone level drops, initiating the
next menstrual cycle.
B Because a fertilized egg is not implanted into the uterus in a nonpregnant woman, the
corpus luteum degenerates, and the levels of estrogen and progesterone increase. The

Advanced Placement Biology Instructor’s Solution Manual


870 34 | Animal Reproduction and Development

endometrium begins to degenerate as the estrogen level increases, initiating the next
menstrual cycle.
C Because a fertilized egg is not implanted into the uterus in a nonpregnant woman, the
corpus luteum degenerates and the levels of estrogen and progesterone increase. The
endometrium begins to degenerate as the progesterone level rises, initiating the next
menstrual cycle.
D Because a fertilized egg is not implanted into the uterus in a nonpregnant woman, the
corpus luteum degenerates and the levels of estrogen and progesterone decrease.
The myometrium begins to degenerate as the progesterone level drops, initiating the
next menstrual cycle.
Solution The solution is (A). In the ovaries, the remaining cells of the follicle become a
structure called the corpus luteum, which starts producing progesterone. The
progesterone facilitates the regrowth of the uterine lining and inhibits the release of
further FSH and LH. The uterus is being prepared to accept a fertilized egg, should it
occur during this cycle. In a nonpregnant woman, the fertilized egg is not implanted
into the uterus, the corpus luteum degenerates, and the levels of estrogen and
progesterone decrease. The endometrium begins to degenerate as the progesterone
level drops, initiating the next menstrual cycle.
54 The side effects of menopause can be diminished by hormone replacement therapy
(HRT). However, many doctors are hesitant to recommend it. What are the possible
reasons for this?
A Its negative side effects include increased risk of colon cancer, osteoporosis, heart
disease, macular degeneration, and possibly depression.
B Its negative side effects include increased risk of stroke or heart attack, blood clots,
breast cancer, ovarian cancer, endometrial cancer, gall bladder disease, and possibly
depression.
C Its negative side effects include increased risk of stroke or heart attack, blood clots,
breast cancer, colon cancer, endometrial cancer, gall bladder disease, and possibly
dementia.
D Its negative side effects include increased risk of stroke or heart attack, blood clots,
breast cancer, ovarian cancer, endometrial cancer, gall bladder disease, and possibly
dementia.
Solution The solution is (B). Hormone replacement therapy causes an increased risk of stroke
or heart attack, blood clots, breast cancer, ovarian cancer, endometrial cancer, gall
bladder disease, and possibly depression.

Advanced Placement Biology Instructor’s Solution Manual


34 | Animal Reproduction and Development 871

55 If multiple sperm were to combine with an egg in an animal, what would be the outcome?
A A cortical reaction would occur if multiple sperm combine with an egg, resulting in a
genetically inviable embryo.
B The embryo would be genetically inviable and would die in a few days. The zygote
might have multiple sets of chromosomes.
C The zygote might have multiple sets of chromosomes, which will result in a neural
tube defect in the developing fetus.
D The zygote would be viable, but the resulting embryo would be genetically inviable.
Solution The solution is (B). When a sperm combines with an egg, a layer is formed around
the egg, preventing other sperm from penetrating. In the case of many sperm fusing
with the egg (polyspermy), the resulting embryo is not genetically viable and will die
in a few days. The zygote in that situation may have multiple sets of chromosomes,
which is generally incompatible with life.
56 After the blastula is formed, where do the embryonic stem cells and germ layers
originate?
A The inner cell mass has embryonic stem cells, which arrange themselves into the three
germ layers.
B The trophoblast in the blastula has embryonic stem cells, which arrange themselves
into three germ layers.
C The inner cell mass has embryonic stem cells, whereas the germ layer cells originate
from the trophoblast.
D The embryonic stem cells and germ layers originate from the blastocoel present inside
the blastula.
Solution The solution is (A). In the blastula, the embryonic stem cells arrange to form an inner
cell mass. The cells then arrange themselves into three germ layers—the endoderm,
ectoderm, and mesoderm—forming the gastrula stage.
57 What determines whether a zygote will undergo total or partial cleavage?
A Total cleavage takes place in eggs having a large amount of yolk, whereas partial
cleavage occurs in eggs having very little or no yolk.
B Total cleavage occurs when eggs have equal concentration of yolk at both poles,
whereas partial cleavage occurs when the yolk is not equally distributed.
C Total cleavage takes place in eggs having little or no yolk, whereas partial cleavage
occurs in eggs having a large amount of yolk.
D Total cleavage occurs when divisions of the blastomeres are separate, whereas partial
cleavage occurs when blastomeres stay partially connected.

Advanced Placement Biology Instructor’s Solution Manual


872 34 | Animal Reproduction and Development

Solution The solution is (C). In eggs that have very little or no yolk, total (holoblastic) cleavage
takes place. In eggs that have a large amount of yolk, partial (meroblastic) cleavage
takes place.
58 During organogenesis, the ectoderm forms the neural cells and the epidermal cells. How
do the ectoderm cells determine which type of cells to form?
A Growth factors signal some of the ectodermal cells to form epidermal cells, and the
remaining cells form the neural plate.
B The notochord cells of the mesoderm signal the ectodermal cells to form epidermal
cells as well as the neural plate.
C Growth factors signal some of the ectodermal cells to form epidermal cells, and the
remaining cells form neural crest cells.
D Proteins involved in the Wnt signaling pathway signal the ectodermal cells to form the
epidermal cells and the neural plate.
Solution The solution is (A). Cell-signaling molecules called growth factors signal some of the
cells at the edge of the ectoderm to form epidermal cells. The remaining cells in the
center form the neural plate.
59 What will be the outcome if the axis is not formed during the developmental stages?
A The animal will have two notochords and may not have a dorsal-ventral or anterior-
posterior side.
B The animal will lack an anterior-posterior or dorsal-ventral side and may not have
complete differentiation of cell layers.
C The animal will lack an anterior-posterior or lateral-medial side and may not have
complete differentiation of cell layers.
D The animal will have incorrect positioning of the dorsal-ventral and lateral-medial
sides, and differentiation of cell layers will be incomplete.
Solution The solution is (B). Several developmental genes determine the body plan of an
animal. For example, dorsal cells are programmed to form the notochord. If the axis
is not formed, the animal will not have an anterior-posterior or dorsal-ventral side,
and may not have complete differentiation of cell layers.
60 What is the mesoderm, and what does it eventually differentiates into?
A The mesoderm develops into various connective tissues. It is reorganized into groups
of cells called somites, which develop into facial cartilage, ribs, and lungs.
B The mesoderm develops into various connective and muscle tissues such as the ribs,
lungs, segmental muscle, and the notochord, which forms the central axis of body of
most animals.

Advanced Placement Biology Instructor’s Solution Manual


34 | Animal Reproduction and Development 873

C The mesoderm develops into various connective and muscle tissues. It is reorganized
into groups of cells called somites, which develop into ribs, lungs, segmental muscle,
and the notochord.
D The mesoderm develops into various connective tissues such as the facial cartilage,
ribs, and lungs.
Solution The solution is (C). The mesoderm develops into the various connective and muscle
tissues of the animal body. A spatial pattern of gene expression reorganizes the
mesoderm into groups of cells called somites with spaces between them. The
somites will further develop into the ribs, lungs, and segmental muscle. The
mesoderm also forms a rod-shaped structure called the notochord, which forms the
central axis of the body.
61 Which option best describes the three stages of labor?
A During stage one, the cervix thins. During stage two, the cervix dilates to about 10 cm
and the baby is expelled from the uterus. The last stage is the passage of the placenta
after the baby has been born.
B During stage one, the cervix thins and dilates to about 10 cm. During stage two, the
baby is expelled from the uterus. The last stage is the passage of the placenta after the
baby has been born.
C During stage one, the cervix thins. During stage two, the cervix dilates to about 10 cm.
During the last stage, the baby is expelled from the uterus, followed by the placenta.
D During stage one, the cervix thins and may or may not be dilated. During stage two,
the baby is expelled from the uterus. The last stage is the passage of the placenta after
the baby has been born.
Solution The solution is (B). During stage one, the cervix thins and dilates. Dilation is
necessary for the baby and placenta to be expelled during birth. The cervix will
eventually dilate to about 10 cm. During stage two, the baby is expelled from the
uterus. The uterus contracts and the mother pushes as she compresses her
abdominal muscles to aid the delivery. The last stage is the passage of the placenta
after the baby has been born and the organ has completely disengaged from the
uterine wall.

TEST PREP FOR AP ® COURSES


62 If a plant species with a diploid number of 32 is crossed with another plant species with a
diploid number of 24, what will be the diploid number of the resulting first generation of
offspring? After the zygote multiplies to form a mass of cells, what is the chromosome
number of each cell?
A 32, 28
B 24, 24
C 28, 28

Advanced Placement Biology Instructor’s Solution Manual


874 34 | Animal Reproduction and Development

D 28, 32
Solution The solution is (C). Meiosis forms gametes with 16 and 12 chromosomes, which
combine to form 28 chromosomes; the zygote divides by mitosis, resulting in cells
with the same number of chromosomes, 28.
63 Compare and contrast sex determination in birds and mammals.
A In mammals, sex is determined by the presence of XX (homozygous) in males and XY
(heterozygous) in females, while in birds, sex is determined by the presence of ZZ
(homozygous) in females and ZW (heterozygous) in males.
B In mammals, sex is determined by the presence of XX (homozygous) in males and XY
(heterozygous) in females, while in birds, sex is determined by the presence of ZW
(heterozygous) in females and ZZ (homozygous) in males.
C In mammals, sex is determined by the presence of XX (homozygous) in females and XY
(heterozygous) in males, while in birds, sex is determined by the presence of ZZ
(homozygous) in females and ZW (heterozygous) in males.
D In mammals, sex is determined by the presence of XX (homozygous) in females and XY
(heterozygous) in males, while in birds, sex is determined by the presence of ZW
(heterozygous) in females and ZZ (homozygous) in males.
Solution The solution is (D). In mammals, a Y chromosome determines sex in males, so the
presence of two X chromosomes, the homozygous condition, results in a female,
and the presence of sex chromosomes in a heterozygous condition, XY, results in a
male. In birds, the presence of ZZ results in a male bird, whereas ZW results in a
female bird.
64 What is the advantage of sexual reproduction?
A Sexual reproduction allows animals to conserve resources and reproduce only during
optimal conditions.
B Sexual reproduction results in offspring with diverse phenotypes, which may enhance
survival of a population in a changing environment.
C Sexual reproduction guarantees that both parents will provide parental care.
D Sexual reproduction yields more numerous offspring more rapidly than is possible
with asexual reproduction.
Solution The solution is (B). Sexual reproduction can produce offspring with diverse
phenotypes, which may enhance survival of a population in a changing environment.
65 In sexual reproduction, two cells of a species combine to form another cell. Why is this
useful?
A It produces a new combination of genes, which enables offspring to survive during
environmental changes.
B It results in the production of many offspring in a short period of time.

Advanced Placement Biology Instructor’s Solution Manual


34 | Animal Reproduction and Development 875

Advanced Placement Biology Instructor’s Solution Manual


876 34 | Animal Reproduction and Development

C It does not involve gamete formation and needs less energy.


D It allows immobile organisms to reproduce in the absence of a mate.
Solution The solution is (A). Sexual reproduction produces a new combination of genes in the
offspring from those of either parent. This new combination may better enable the
offspring’s survival during changes in the environment, thereby assisting in the
survival of species.
66 Spawning is often triggered by a signal such as water temperature or day length. What is
an advantage of using this type of signal versus using individual courtship behaviors?
A It allows many individuals to spawn simultaneously without males and females having
to choose individual mates.
B It is the only way that males and females can time gamete release simultaneously.
C These are the only types of signals that can be used.
D It increases the distance that gametes can travel.
Solution The solution is (A). These types of signals can trigger many individuals to spawn at
once. This allows for coordination of reproduction and can be especially important
for species that release gametes into the water during external fertilization to
ensure that the eggs and sperm meet. When males and females are in closer
contact, individual behaviors become even more important for external fertilization.
67 Which statement about hormone regulation of the female reproductive cycle is true?
A LH and FSH are produced in the ovaries, and estradiol and progesterone are produced
in the pituitary.
B Estradiol and progesterone secreted from the corpus luteum cause the myometrium
to thicken.
C Progesterone is produced by the corpus luteum.
D Secretion of GnRH by the hypothalamus is inhibited by high levels of estradiol, but
stimulated by low levels of estradiol.
Solution The solution is (C). Progesterone is produced by the corpus luteum. LH and FSH are
produced by the pituitary. Estradiol and progesterone cause the endometrium to
thicken. GnRH levels are stimulated by high levels of estradiol.
68 Which statement about the menstrual cycle is true?
A Estrogen levels rise during the luteal phase of the ovarian cycle and the secretory
phase of the uterine cycle.
B Menstruation occurs much before LH and FSH levels peak.
C Menstruation occurs after progesterone levels rise.
D Progesterone levels rise before ovulation, while estrogen levels rise after.

Advanced Placement Biology Instructor’s Solution Manual


34 | Animal Reproduction and Development 877

Solution The solution is (B). Menstruation occurs much before LH and FSH levels peak. LH and
FSH levels peak after menstrual flow and the proliferative phase.
69 What stimulates Leydig cells in the testes to produce testosterone?
A FSH
B LH
C Inhibin
D Estrogen
Solution The solution is (B). LH stimulates Leydig cells to make and release testosterone.
70 What is the nature of the oogonium and the secondary oocyte? Which process results in
the formation of the secondary oocyte?
A A diploid oogonium forms a haploid oocyte by the process of mitosis.
B A haploid oogonium forms a diploid oocyte by the process of meiosis.
C A diploid oogonium forms a haploid oocyte by the process of meiosis.
D A haploid oogonium forms a haploid oocyte by the process of meiosis.
Solution The solution is (C). A diploid (2n) oogonium forms a haploid (n) oocyte by the
process of meiosis.
71 Why are the diploid zygotes produced after fertilization of sperm cells produced by
spermatogenesis not similar?
A Their chromosome numbers are not the same.
B The size of the sperm produced is different.
C Some sperm may have a tail, whereas others may not.
D Crossing over occurs during spermatogenesis.
Solution The solution is (D). Crossing over in prophase I in spermatogenesis and oogenesis
will result in exchange of genetic material. When they combine with other gametes,
the resulting zygotes will not be similar.
72 The endocrine system incorporates feedback mechanisms that maintain homeostasis.
Which mechanism shows negative feedback by the reproductive system in mammals?
A Increasing levels of testosterone inhibit the production of GnRH, LH, and FSH by the
hypothalamus and pituitary.
B LH and FSH stimulate the interstitial cells of Leydig to release testosterone.
C The growing follicle starts releasing estrogen in increasing amounts.
D The corpus luteum releases progesterone after ovulation.

Advanced Placement Biology Instructor’s Solution Manual


878 34 | Animal Reproduction and Development

Solution The solution is (A). Increasing levels of testosterone inhibit the production of GnRH,
LH, and FSH by the hypothalamus and pituitary. Negative feedback occurs because
increasing levels of testosterone inhibit production.
73 What are the positive feedback mechanisms by LH and FSH during the follicular and
ovulation phase of the ovarian cycle?
A The stimulation of the anterior pituitary by GnRH causes it to secrete the hormones LH
and FSH. The increasing amounts of these hormones stimulate several follicles in the
ovary to start growing to be released.
B The stimulation of the anterior pituitary by GnRH causes it to secrete the hormone LH
only. The increasing amounts of these hormones stimulate several follicles in the
ovary to start growing, but only one of these matures to release the egg.
C The stimulation of the anterior pituitary by GnRH triggers the secretion of the
hormones LH and FSH. The increasing amounts of these hormones stimulate
several follicles in the ovary to start growing, but only one of these matures to
release the egg.
D The stimulation of the anterior pituitary by GnRH causes it to secrete the hormones LH
and TSH. The increasing amounts of these hormones stimulate a single follicle in the
ovary to start growing.
Solution The solution is (C). When the anterior pituitary is stimulated by GnRH, it secretes
the hormones LH and FSH. The increasing amounts of these hormones stimulate
several follicles in the ovary to start growing. Only one of these will mature to
release the egg.
74 Which possible event would occur if the corpus luteum did NOT produce increasing
amounts of progesterone?
A The implanted zygote will not be able to derive enough nutrition from the
endometrium.
B Even if fertilization is successful, the zygote may not be able to implant successfully. If
it does manage to implant, it will not be able to derive enough nutrition from the
myometrium.
C Even if fertilization is successful, the zygote may not be able to implant successfully. If
it does manage to implant, it will not be able to derive enough nutrition from the
endometrium.
D The contraction of the uterus during childbirth and lactation will not take place,
causing problems and complications in the mother.
Solution The solution is (C). Progesterone helps facilitate the thickening of the endometrium
of the uterus to prepare for the implantation of a fertilized egg. If the corpus luteum
does not produce enough progesterone, even if fertilization has been successful, the
zygote may not be able to implant successfully. If it does manage to implant, it will
not be able to derive sufficient nutrition from the endometrium.

Advanced Placement Biology Instructor’s Solution Manual


34 | Animal Reproduction and Development 879

75 What does a female contraceptive pill that inhibits the release of GnRH from the
hypothalamus do?
A Reduce the secretion of FSH and LH from the anterior pituitary gland.
B Initiate ovulation.
C Increase the flow phase of the menstrual cycle.
D Increase the production of estrogen and progesterone by the ovaries.
Solution The solution is (A). Inhibiting GnRH will also lead to reduced secretions of FSH and LH
from the anterior pituitary gland.
76 What do the rising levels of FSH and LH in the follicular phase cause?
A The follicles on the surface of the ovary start growing in preparation for ovulation.
B The endometrium starts to thicken.
C The corpus luteum starts secreting progesterone.
D One of the mature follicles bursts, releasing the egg.
Solution The solution is (A). The follicles on the surface of the ovary start growing in
preparation for ovulation as FSH and LH levels rise.
77 A couple has been trying to conceive for some time and goes to an endocrinologist for
advice. If the endocrinologist diagnoses an obstruction in the oviducts of the female, what
type of treatment can she recommend?
A Opting for in vivo fertilization or ligation of the fallopian tubes surgically so that the
tubes are sealed
B Opting for in vivo fertilization or surgical removal of the fallopian tubes
C Opting for in vitro fertilization or ligation of the fallopian tubes surgically so that the
tubes are opened
D Opting for in vitro fertilization or removal of the obstruction surgically and then rejoin
of the ends of the fallopian tubes
Solution The solution is (D). After the egg is released from the follicle, it makes its way to the
oviducts, where a possible fertilization event will take place. Since there is an
obstruction in the oviducts, the endocrinologist can advise the couple on opting for
in vitro fertilization, where the eggs and sperms will be fused outside the body, and
then the embryos can be implanted into the uterus. The other option is to remove
the obstruction surgically and rejoin the ends of the fallopian tubes.
78 How does a reproductive hormone that is secreted directly from the anterior pituitary
stimulate the reproductive organs to produce hormones?
A By traveling through the blood to reach the target organs
B By binding with proteins to reach the reproductive organs

Advanced Placement Biology Instructor’s Solution Manual


880 34 | Animal Reproduction and Development

C By sending a chemical messenger to activate the reproductive organs


D By converting into an active form before targeting the reproductive organs
Solution The solution is (A). Hormones travel through blood to reach the target organs.
79 By which mechanisms do hormones from the brain and ovaries interact, eventually
leading to menstruation?
A After the release of the egg from the follicle, the corpus luteum is formed, which
inhibits FSH and LH production, which then inhibits GnRH production, causing no other
follicle to develop. When no fertilization takes place, the corpus luteum degenerates
and the progesterone level declines, initiating the breakdown of the myometrium and
the start of the menstrual cycle.
B After the release of the egg from the follicle, the corpus luteum is formed, which
inhibits FSH and LH production, which then inhibits GnRH production, causing no other
follicle to develop. When no fertilization takes place, the corpus luteum degenerates
and the progesterone level declines, initiating the breakdown of the endometrium and
the start of the menstrual cycle.
C After the release of the egg from the follicle, the corpus luteum is formed, which
inhibits FSH and LH production, which then facilitates GnRH production, causing no
other follicle to develop. When no fertilization takes place, the corpus luteum
degenerates and the progesterone level declines, initiating the breakdown of the
endometrium and the start of the menstrual cycle.
D After the release of the egg from the follicle, the corpus luteum is formed, which
inhibits FSH and LH production, which then inhibits GnRH production, causing no other
follicle to develop. When no fertilization takes place, the corpus luteum degenerates
and the progesterone level rises, initiating the breakdown of the endometrium and
the start of the menstrual cycle.
Solution The solution is (C). After the egg is released from the follicle, it forms a structure
known as the corpus luteum, which produces progesterone and estrogen. This
inhibits FSH and LH production, which inhibits GnRH production via negative
feedback, causing no other follicle to develop. Progesterone facilitates regrowth of
the endometrium and gets it ready for any future implantation. When no
fertilization takes place, the corpus luteum degenerates and the progesterone level
declines, initiating the breakdown of the endometrium and the start of the
menstrual cycle.
80 Multiple hormones work together to coordinate the female reproductive cycle. Which
hormone is also responsible for the development of female secondary sexual
characteristics, including breast development and hip widening?
A Estrogen
B Progesterone

Advanced Placement Biology Instructor’s Solution Manual


34 | Animal Reproduction and Development 881

C Follicle-stimulating hormone
D Luteinizing hormone
Solution The solution is (A). Estrogen is the hormone that is responsible for the development
of major female secondary sexual characteristics, including breast development and
hip widening.
81 Which hormone is released by the anterior pituitary as a part of the positive feedback
loop between it and the ovary?
A Progesterone
B GnRH
C LH
D Estradiol
Solution The solution is (C). LH is released from the anterior pituitary, causing the follicle to
burst and release the egg.
82 How does the feedback mechanism of hormones lead to muscular contractions
during labor?
A A feedback relay occurs between the uterus, hypothalamus, and posterior pituitary to
assure an adequate supply of oxytocin, which causes the contraction of smooth
muscles of the uterus, leading to the birth of the baby.
B The posterior pituitary continuously produces oxytocin, which is sufficient for
muscular contraction in the uterus, thereby causing the birth of the baby.
C A feedback relay occurs between the uterus, hypothalamus, and posterior pituitary to
assure an adequate supply of prolactin, which causes the contraction of the smooth
muscles of the uterus, leading to the birth of the baby.
D A feedback relay occurs between the uterus, hypothalamus, and posterior pituitary to
assure an adequate supply of progesterone, which causes the contraction of the
smooth muscles of the uterus, leading to the birth of the baby.
Solution The solution is (A). As the third trimester nears its end, the uterine walls bind the
hormone oxytocin, which is produced by the posterior pituitary. The oxytocin causes
the smooth muscles in the uterus to contract. At the same time, the placenta
releases prostaglandins in the uterus, increasing contractions. A positive feedback
relay occurs between the uterus, hypothalamus, and posterior pituitary to assure an
adequate supply of oxytocin, so the contraction can continue, leading to the birth of
the baby.

Advanced Placement Biology Instructor’s Solution Manual


882 34 | Animal Reproduction and Development

83 During childbirth, the hormone oxytocin causes contraction of the uterine wall muscles.
As muscular contractions increase, more oxytocin is released from the pituitary, leading to
more contractions. This is an example of which mechanism?
A End product inhibition
B Negative-feedback mechanism
C Positive-feedback mechanism
D Feedback inhibition
Solution The solution is (C). Oxytocin causes an increase in muscular contractions, leading to
further release of oxytocin. This is an example of a positive-feedback mechanism.

SCIENCE PRACTICE CHALLENGE QUESTIONS


34.1 Reproduction Methods
84 Humans are bilateral and have biaxial symmetry. However, small variations in body
symmetry occur. One foot may be slightly larger than another, and we might never know.
Because the eyes are so important in communication, the condition called heterochromia
iridium is very noticeable. Body plans are very noticeable aspects of the phenotype.
In guppy courtship, the male displays himself to the female. Some males are more orange
than others and some males have asymmetry, with one side being more brightly colored
orange than the other side. A pattern in the courtship display is shown in the graphs
(Gross et al., Proceedings of the Royal Society B: Biological Sciences, 274, 2007).

A. Analyze the data to support the claim that the female guppy is responding to
information, that the male guppy is aware of this response, and that this communication
affects natural selection. If there were no process that maintained variation in the gene
controlling the orange coloration, then perhaps all male guppies would be symmetrically
bright orange. Yet variation remains.
B. A guppy is a small fish, and small fish are eaten by bigger fish. Make and justify a claim
regarding the effect of allelic variation on the stability of the guppy population. Phenotype
is an expression of the genotype and, within a population, genotypic variation occurs. For
a population to maintain stability in a changing environment, genetic variation within the
population provides selective advantage at the population level.

Advanced Placement Biology Instructor’s Solution Manual


34 | Animal Reproduction and Development 883

C. Make and justify a claim regarding the effect of genetic variation in guppies on the
stability of an ecosystem of which the guppy is a member. The fluctuation of symmetry in
the body plan of an organism, such as the asymmetry of guppy coloration, is unusual.
Large-scale fluctuations where the fully developed organism is viable, such as the
emergence of an appendage from the eye of the Drosophila, are very rare. The overall
body plan is controlled in Drosophila by a cluster of genes called the Hox genes. In the
fruit fly, there is a single cluster, and the arrangement of genes in the genome is a map of
the anterior to posterior body plan. In all vertebrates, multiple Hox clusters control the
development of the body plan, and they too show the sequential anterior to posterior
arrangement in the genome.
D. During development, the transcription factors expressed by the Hox genes initiate and
terminate the expression of specialized cells and tissues. Describe the evidence for shared
ancestry in this conserved strategy for communication between cells through regulation
of transcription factors.
E. Describe one other example of conserved core shared by all domains or within one
domain.
Solution Sample answer:
A. The graph at the left shows that females prefer orange males: they respond to the
information contained in the color signal. The graph at the right shows that male
guppies “show her his best side” and so also respond to a communication. Mating
affects natural selection.
B. When the effect of color in predation is considered and it is supposed that the
predator also might respond to the bright color, then not just the individual but all
guppies in the vicinity are disadvantaged by a bright display. Maintaining allelic
variation increases the fitness of the population.
C. If overpredation of guppies occurs, then a resource is lost to the next trophic
level, and this reduces the stability of the ecosystem as a whole.
D. The idea that development is regional with the genome sequence corresponding
to the symmetry axis of the body is shared by vertebrates and invertebrates. As the
body develops, new tissues are initiated by existing tissues that then terminate their
development while initiating the next. This cell-cell communication is the basis of
self-assembly.
The Curriculum Framework specifically identifies the Hox genes and this
development sequence as content for which the student is responsible: 2.E.1.a.
Observable cell differentiation results from the expression of genes for tissue-
specific proteins. 2.E.1.b. Induction of transcription factors during development
results in sequential gene expression. Evidence of student learning is a
demonstrated understanding of each of the following: 1. Homeotic genes are
involved in developmental patterns and sequences. 2. Embryonic induction in

Advanced Placement Biology Instructor’s Solution Manual


884 34 | Animal Reproduction and Development

development results in the correct timing of events. This item illustrates the
appropriate scope for this content.
E. Unlike the specific content knowledge described in (D), the Curriculum Framework
usually allows teacher and student interest to determine which examples to use in
demonstrating a principle. Examples are often suggested in the Framework. In this
case, they include: cytoskeleton (a network of structural proteins that facilitate cell
movement, morphological integrity, and organelle transport); membrane-bound
organelles (mitochondria and/or chloroplasts); linear chromosomes; and
endomembrane systems, including the nuclear envelop.

Advanced Placement Biology Instructor’s Solution Manual

You might also like